You are on page 1of 304

Demystified

Series

Physics Demystified

for UTME, PUTME, SSCE, GCE and


other O’level exams

Dr Aifuwa Timothy

!!! Notice – If you don’t want to score high in physics, don’t read this
book………
Demystified Series Physics Demystified by Dr Timothy

© Giegie Nigeria Innovations

All rights reserved. No part of this publication may be reproduced or transmitted in any form or by any means
without prior permission from the author.

Printed in Nigeria By
GIEGIE NIGERIA INNOVATIONS
Suite 77, S&T Barrack Road,
Opp. Mountain of Fire and Miracles Ministries
Uselu Lagos Road,
Benin City, Edo State.
07034623544, 08034602197, 08076342217
- UTME Fundamental Secrets of Post UTME

2
Demystified Series Physics Demystified by Dr Timothy
Preface
Physics Demystified is a panacea to all pathologies in physics and a masterpiece of its kind. It is
written for students enrolling physics in WAEC, UTME, PUTME and other related O’level exams. Each
chapter of this book contains explicit and concise notes, comprises over 3,000 O’level exams questions
with detailed/explicit explanation and JAMB past questions for self-assessment.
This book is highly recommended for all students and should be well dichotomized before the exam
and also recommended for tutors in this field of study, as it will be of crucial help in teaching and
inculcating the rightful knowledge and information to the students.
The comment session is open to all kind of suggestions or corrections of any erroneous statement
in this book, as they will be highly accepted and appreciated.
I will strongly advice all candidates sitting for any O’level exam not to enter the exam hall, if
he/she has not read Physics Demystified.
!!! The questions in this book are all exam based questions from SSCE, JAMB, PUTME and other
O’level exams.

➢ Happy Reading!!!

Dr Aifuwa Timothy
MBBS in view
Edo state,Nigeria.
08024672601/08151907988

Acknowledgement
In the course of my writing of this book, I have consulted several textbooks and tutors of esteemed
worth. I sincerely express appreciation to ; M.W Anyakowa[author of New School physics], O. Ajaja and H.
B. Olaniyi [authors of lamlard’s physics], Afolabi J. R [author of the science physics], L. Okolosi [author of
hidden facts in SSCE physics], P. N. Okeke [author of senior secondary school physics], M. K. Nelkon
[author principles of physics] and Dr Aniefuna Onyebuchi .C. [author of revision physics] and to all O’level
exam bodies, for using their materials in the course of writing this book, they were indeed helpful.
I also want express my profound gratitude to my parents : Mr and Mrs Peter AIfuwa, my sisters :
Nosakhare Aifuwa and Osasere Aifuwa, Dr Ige Emmanuel, Sir O. J. Olaoye [C.E.O pedagogue house], all my
friends and mentors, special thank you to you all for your support.

To God be all the glory………..

Dedication
This book is dedicated to God, my parents, friends and to all students.

3
Demystified Series Physics Demystified by Dr Timothy
Table of Contents
Chapter Title Pages
1 Measurement, Scalars and Vectors 5
2 Motion : Linear motion, Projectile motion, Linear momentum and 17
Newton’s laws of motion, Circular motion, Simple harmonic motion
3 Friction in solids, Fluids friction or viscosity, Surface Tension 38
4 Work, Energy and Power 51
5 Force field, Gravitational field 57
6 Equilibrium of forces 63
7 Elastic properties, Elasticity 72
8 Simple machines 79
9 Equilibirum in liquids : Density, Relative density and Upthrust 88
10 Pressure in solids and liquids, Kinetic theory of matter and Gas laws 97
11 Temperature and its measurement 116
12 Thermal expansivity 123
13 Thermal energy, Heat capacity, Latent heat and Vapours 131
14 Heat transfer 148
15 Wave: Wave types, Propagation and Properties 156
16 Sound waves 167
17 Light waves : Rectilinear propagation of light , Reflection of light 182
in plane and curved or spherical surface.
18 Refraction of light in plane and spherical surfaces/lenses. 195
19 Optical instruments, Application of light waves 209
20 Dispersion of light, Electromagnetic spectrum 217
21 Electrostatics, Electric field, Capacitors 222
22 Electric cells, Conduction of electricity through liquids and gases 234
23 Current electricity, Electrical power and energy 240
24 Magnetic field, Magnetism and Electromagnetism 256
25 Simple A.C circuits 275
26 Elementary Atomic Physics : Atomic theories and discovery of atoms, 280
Energy quantization and line spectra, Thermionic emission, X-ray
production, Photoelectric emission, Radioactivity and nuclear
reactions, Wave-particle duality.
27 Basic Electronics : Semi-conductors, Diodes and Transistors 299

4
Demystified Series Physics Demystified by Dr Timothy

CHAPTER 1 – MEASUREMENT, SCALARS AND VECTORS.


● Measurement – This is the estimation of the spring balance. It works on Hooke’s law.
magnitude of a physical quantity. A measured - The weight of a body varies with change in
quantity consists of its measured value or gravity.
magnitude and its unit e.g the length of a rule is - Measurement of length –
1m. 1. Meter rule – Measures length to 2d.p in cm or
● Fundamental and derived quantities 1d.p in mm , (M±0.5)mm. The reading accuracy
and their units – of a meter rule is 0.5mm or 0.05cm. It is used to
- Fundamental quantities – These are physical measure short distance like the length of a book
quantities from which others are derived. Mass, or a piece of paper, length of a wire e.t.c.
length and time are the three main fundamental 2. Vernier caliper – Measures small lengths to
quantities. The international agreed system of 2d.p in cm , (M±0.01)cm or 1d.p in mm. The
units for measurements are kilogram (kg) for reading accuracy of a Vernier caliper is 0.1mm or
mass, meter (m) for length and second (s) for 0.01cm. It is used to measure both the internal
time. and external diameter of round objects e.g a
Fundamental quantity Unit cylindrical test tube, thickness of a meter rule
1.Mass Kilogram (kg) or a plate, depth of a cavity and diameter of a
2.Length Meter (m) rod.
3.Time Seconds (s) - How to read a Vernier caliper –
4.Temperature Kelvin (K) i.Read the main scale till the mark just before the
5.Electric current Ampere (A) Vernier scale and take note of the reading in 1d.p
6.Luminous intensity or Candela (cd) in cm. ii.Read the Vernier scale until the point
brightness Newton- where it meets or coincides with the Vernier
7.Amount of a substance meter(Nm) scale and take note of the reading in 2d.p in cm.
iii.The reading is the sum of the main scale
- Derived quantities – They are derived from reading and vernier scale reading.
fundamental quantites e.g area,volume,speed 3.Micrometer screw guage – Measures very
density, force e.t.c. Examples of derived small length to 3d.p in cm or 2d.p in mm i.e
quantities and their units; (M±0.01)mm. The reading accuracy of a
Derived quantity Unit micrometer screw guage is 0.01mm or 0.001cm.
1.Volume cube meter(m3) It is used to measure external or outside
2.Speed or velocity meter per second diameter only of a round objects e.g
(ms-1) cylindrical test tubes, thickness of a thin
3.Force newton(N=kgms-2) converging lens or pendulum bulb, thickness
4.Impulse or newton-second(Ns= of a piece of paper or rod.
momentum Kgms-1) - How to read a micrometer screw guage –
5.Energy or joule(𝐽=Nm= kgm2s-2 i.Read the sleeve scale just before the thimble
workdone scale and take note of the reading in 1d.p in mm.
6.Moment or couple Newton-meter(Nm) ii.Read the thimble scale until the point where it
or torque meets or coincides with the sleeve scale and take
7.Energy density 𝐽m-3= Kgm-1s-2 note of the reding in 2d.p in mm. iii.The reading
is the sum of the sleeve reading and thimble
- Measurement of time – Time is measured by
reading.
stop watches, clocks, ticker tape timer.
- Measurement of volume – The volume of an
- The most accurate clocks in the world are
irregular solid is measured by a measuring
atomic clock and quartz crystal clocks.
cylinder i.e the volume of water displaced by the
- The time scale used for astronomical purposes
solid from the cylinder is equal to its volume.
is called sidereal time.
● Error in measurement – The deviation of a
- Ticker tape timer measures both distance and
measured value in measurement from the actual
time simultaneously.
or original value. A measurement is always in the
- Measurement of mass – Mass is the quantity of
form of : (𝑴 ± 𝒆), 𝑀 is the measured value, e is
matter contained in a body. It is measured by
the error i.e the measurement within the range of
beam balance or chemical balance. It works on
𝑀 + 𝑒 to 𝑀 − 𝑒.
the principle of moments.
- Absolute error=Measured value−Actual
- The mass of a body is constant and does not
value, 𝒆 = 𝑴. 𝑽 − 𝑨. 𝑽.
change with change in gravity. 𝑨𝒃𝒔𝒐𝒍𝒖𝒕𝒆 𝒆𝒓𝒓𝒐𝒓
- Measurement of weight – Weight is the earth’s - Percentage error= × 100
𝑨𝒄𝒕𝒖𝒂𝒍 𝒗𝒂𝒍𝒖𝒆
gravitational pull on a body. It is measured by a - Maximum or greatest possible error or least

5
Demystified Series Physics Demystified by Dr Timothy
possible error or estimated uncertainty is 8.
equal to half the smallest scale division. mm 35
16 30
Examples : 25
1. Which of the following statements are true
about the spring balance and the chemical What is the reading of the micrometer screw
balance? I.Both are used to measure the mass of guage? A.17.76mm B.16.26mm C.17.56mm
an object II.Either of them may be used to D.16.25mm.
measure the weight of an object III.the springs Sleeve reading=17.5mm, Thimble reading=
balance works on the principle of Hooke’s law 0.26mm. Reading=17.76mm.
while the chemical balance works on principle of 9. Which of the following instruments is most
moment IV.a change in gravity changes the suitable for measuring the outside diameter of a
reading of a spring balance but not that a narrow pipe a few millimeters in diameter?
chemical balance A.I and IV B.II and III C.III and A.Pair of calipers B.Metre rule C.Micrometer
IV D.I and III E.I,II and III. screw guage D.Tape rule.
Spring balance measures the weight of an object A mircometer meter screw guage measures the
while chemical balance measures the mass of an external (outer) diameter of round or cylindrical
object. Spring balance works on the principle of objects to 2 decimal places in millimeter.
Hooke’s law while chemical balance works on the 10. The thickness of the central portion of a thin
principle of moment. Gravity affects weight (the converging lens can be determined very
reading of spring balance) but does not affect accurately by using? A.vernier caliper B.a
mass (the reading of a chemical balance). micrometer screw guage C.a telescope D.a
2. The basic principle of operation of a beam microscope.
balance is A.Archimedes principle B.law of 11. I.Diameter of a small ball bearing II.Thickness
floatation C.Hooke’s law D.principle of of a piece of paper III.Diameter of a measuring
moments. cylinder IV.Length of a piece of wire. Which of the
3. Which of the following instruments has a above can be measured using a micrometer
reading accuracy of 0.5mm? A.vernier calipers screw guage? A.I,II and IV B.I and II C.II and III
B.micrometer screw guage C.meter rule D.III and IV.
D.spring balance. The diameter of a measuring cylinder can be
4. Which of the following represents the correct internal or external. Since it’s not specified it
precision if the length of a piece of wire is means both are being considered in this case.
measured with a meter rule? A.35mm 12. The volume of a stone having an irregular
B.35.0mm C.35.00mm D.35.01mm. shape can be determined using? A.meter rule
A meter rule measures the length of a material to B.measuring cylinder C.vernier calliper
1 decimal place in millimeter. D.micrometer screw guage.
5. 6 cm 7 Main scale 13. The volume of an irregular solid can be
measured using A.burette B.pipette C.conical
flask D.displacement can.
Vernier scale 14. The least possible error in using a scale
0 5 graduated in millimeters is A.0.1mm B.0.5mm
The diagram above represents a portion of C.1.0mm D.2.0mm.
vernier caliper. What is its reading? A.7.10cm The smallest scale division of a scale graduated in
B.7.09cm C.6.90cm D.6.34cm E.6.27cm. millimeter=1mm,
Main scale reading=6.2cm Vernier scale Least possible error=half the smallest scale
reading=0.07cm. Reading=6.27cm. 1
division= × 1= 0.5mm.
2
6. Which of the following readings gives the
15. What is the percentage reading error in
correct precision of the length of a rod using a
measuring a distance of 5.00cm using a meter
vernier caliper? A.4.1cm B.4.13cm C.4.120cm
rule marked in mm? A.10% B.5.0% C.1.0%
D.4.125cm.
D.0.5%.
A vernier calliper measures length to the nearest Smallest scale division of a meter rule=1mm,
2 decimal place in centimeter. 1
7.The inner diameter of a small test tube can be reading error= × smallest scale division
2
1
measured accurately using? A.pair of vernier reading error= ×1=0.5mm, actual reading=
2
calliper B.micrometer screw guage C.meter rule 5.00cm=50mm,
D.pair of dividers. 𝑟𝑒𝑎𝑑𝑖𝑛𝑔 𝑒𝑟𝑟𝑜𝑟
%reading error = × 100
A vernier caliper measures both the internal 𝐴𝑐𝑡𝑢𝑎𝑙 𝑣𝑎𝑙𝑢𝑒
0.5
(inner) and external (outer) diamater of round or %reading error = × 100 = 1%.
50
cylindrical objects. 16. A student measures the internal and external

6
Demystified Series Physics Demystified by Dr Timothy
diameter of a cylindrical vessel as (100±1)mm to its fundamental makeup.
and (104±1)mm repectively. Determine the 2.Substitue the fundamental quantities with
percentage error in the thickness of the tube. their individual dimension and assigning any
A.50% B.100% C.25% D.75%. power if present. Incase any coefficient is present
Percentage error =
𝐴𝑏𝑠𝑜𝑙𝑢𝑡𝑒 𝑒𝑟𝑟𝑜𝑟
× 100 e.g 2M, ignore the number “2” and remove it from
Actual value
the dimension.
Actual value of thickness = Ext.dia−Int.dia =
3.Simplify to get the overall dimension.
104−100=4mm,
Examples :
Absolute error 𝑒 = 𝑒1 + 𝑒2 =1+1 = 2mm
2 1. Speed, velocity and acceleration.
Percentage error = ×100 = 50%. a. Speed=distance/time , distance is a measure of
4
17. Which of the following are derived 𝐿
length hence, Speed = = 𝐿𝑇 −1 .
quantities? I.thrust II.temperature III.area 𝑇
b. Velocity=displacement/time, displacement is
IV.pressure A.I and IV B.II,III and IV C.I,III and IV
also a measure of length hence,
D.I,II,III and IV. 𝐿
Temperature is a fundamental quantity. Velocity = = 𝐿𝑇 −1 .
𝑇
18. Which of the following is a derived unit? Speed and velocity has the same dimension.
I.meter II.coulomb III.kilogram IV.ampere V.joule c. Acceleration = velocity/time,
A.I and III B.II and V C.II,IV and V D.I,II III,IV 𝑣 𝐿𝑇 −1
𝑎= = = 𝐿𝑇 −2 .
and V. 𝑡 𝑇
𝑚 𝑀
The units : meter for length, kilogram for mass 2.Density=mass/volume, 𝜌 = = 3 = 𝑀𝐿−3 .
𝑉 𝐿
and ampere for electric current are all 3. Force, Workdone, energy, power, momentum,
fundamental units. Coulomb for quantity of impulse, energy density.
electricity or charge and joule for energy or a. Force=mass×acceleration= 𝑚 × 𝑎,
workdone are derived units. 𝐹 = 𝑀 × 𝐿𝑇 −2 = 𝑀𝐿𝑇 −2 .
19. The watt is equivalent to A.Nms-1B.Js b. Workdone=Force×distance= 𝐹 × 𝑠,
C.Kgm2s-2 D.Ns. 𝑊 = 𝑀𝐿𝑇 −2 × 𝐿 = 𝑀𝐿2 𝑇 −2 .
The unit for power =watt(W)= Js-1 = Nms-1. c. Energy and workdone have the same
20. In which of the following physical quantities dimension.
are the units correctly indicated? I.Weight(N) d. Power=workdone/time= ,
𝑊
𝑡
II.Energy(Nm) III.Momentum (kgms-1) 𝑀𝐿2 𝑇 −2
IV.Acceleration(Nkg-1). A.I and II B.III and IV C.I,II 𝑃= = 𝑀𝐿 𝑇 . 2 −3
𝑇
and III D.I,II,III and IV. e. Momentum=mass×velocity=𝑚 × 𝑣,
Weight=Force=N, Energy=𝐽=Nm, Momentum= 𝑃 = 𝑀 × 𝐿𝑇 −1 = 𝑀𝐿𝑇 −1 .
Ns=Kgms-1, Acceleration=ms-2=Nkg-1. f. Impulse=Force×time= 𝐹 × 𝑡,
21.Which of the following quantities has the 𝐼 = 𝑀𝐿𝑇 −2 × 𝑇 = 𝑀𝐿𝑇 −1 .
same unit as the kilowatt-hour? A.Force×time Impulse and momentum have the same
B.Force×acceleration C.Force×distance dimension.
D.Force×velocity. g. Energy density = energy/volume =
Kilowatt-hour=Power×time=Energy or work workdone/volume = 𝑊/𝑉
done= Force×distance. Impules = Force×time. Energy density=
𝑀𝐿2 𝑇 −2
= 𝑀𝐿−1 𝑇 −2 .
Power = Force×velcoity. 𝐿3
22.Which of the following units is equivalent to 4. Pressure, tensile stress, tensile strain, young
the ratio of power÷acceleration? A.Js-1B.Ns modulus, surface tension and viscosity or
C.Nms D.Ns-1. coefficient of viscosity.
𝐹
Power/acceleration=momentum=kgms-1=Ns. a. Pressure=Force/area= ,
𝐴
𝑀𝐿𝑇 −2
𝑃= = 𝑀𝐿 𝑇 −1 −2
.
● Dimensional analysis – Dimensional 𝐿2
𝐹
analysis shows the relations of derived quantities b. Tensile stress=Force/area= ,
𝐴
to the fundamental quantities from which they Tensile stress=
𝑀𝐿𝑇 −2
= 𝑀𝐿−1 𝑇 −2 .
are obtained. In dimensional analysis, the 𝐿2
𝑒
fundamental quantities are represented by a c.Tensile strain=extension/original length =
𝐿
𝐿
single letter and while derived quantities are Tensile strain= = 1. Hence, Tensile strain is
𝐿
represented by a combination of these dimensionless.
fundamental quantities. In dimension, 𝐿 is used d.Young modulus=Tensile stress/Tensile strain
to specify length, 𝑀 is used to specify mass, 𝑇 is 𝑀𝐿−1 𝑇 −2
used to specify time, 𝐼 is used to specify time, θ is 𝐸= = 𝑀𝐿−1 𝑇 −2 .
1
used to specify temperature. Energy density, pressure, tensile stress and
Steps in dimension of a physical quantity Young modulus all have the same dimension.
𝐹
1.Write out the formula of the quantity as related e. Surface tension=Force/Length= ,
𝐿

7
Demystified Series Physics Demystified by Dr Timothy
S.T=
𝑀𝐿𝑇 −2
= 𝑀𝑇 −2 . 𝑀𝑜 𝐿𝑜 𝑇 = 𝐾 . 𝑀X𝐿−3𝑋 . 𝐿Y. 𝑀Z𝑇 −2𝑧 ,
𝐿 𝑀𝑜 𝐿𝑜 𝑇 = 𝐾𝑀X+Z. 𝐿−3𝑋+𝑌 . 𝑇 −2𝑧 ,
f. Viscosity=Force×distance/velocity×area
𝐹×𝑑 𝑀𝐿𝑇 −2 ×𝐿
Equate like terms on both sides of the equation;
𝜂= = −1 2 = 𝑀𝐿−1 𝑇 −1 . 𝑀𝑜 = 𝑀X+Z , x+z=0, x= −z,
𝑣×𝐴 𝐿𝑇 ×𝐿
5.Universal gravitation constant, 𝐺 𝐿𝑜 = 𝐿−3𝑋+𝑌 , −3x+y=0, y=3x,
𝐺𝑚1 𝑚2 𝐹𝑟 2 𝐹𝑟 2 𝑇 = 𝑇 −2𝑧 , 1= −2z, z= −1/2 ,
𝐹= , 𝐺= = ,
𝑟2 𝑚1 𝑚2 𝑚2 x= −𝑧 = −(−1/2) =1/2 , y=3x= 3 ×1/2=3/2 ,
𝑀𝐿𝑇 −2 ×𝐿2
𝐺= = 𝑀−1 𝐿3 𝑇 −2 . Substitute “x=1/2, y=3/2 and z=−1/2” back into
𝑀2
the original equation – 𝑇 = 𝐾𝜌X𝑟Y𝑌Z ,
6.Specific heat capacity, 𝑐
𝑐 =Heat energy/mass×temperature change 𝐾𝜌1/2 𝑟 3/2 𝜌𝑟 3
𝑇 = 𝐾 (𝜌)1/2. (𝑟)3/2. (𝑌)-1/2 = = 𝐾√ .
𝐻 𝑀𝐿2 𝑇 −2 𝑌 1/2 𝑌
𝑐= = =𝐿 𝑇 𝜃 . 2 −2 −1
2. If the frequency (𝑓) of a simple pendulum is
𝑚∆𝜃 𝑀×𝜃
7. Quantity of electricity or Charge, assumed to be affected by the following
Electromotive force. quantities – mass of the bulb(𝑚), length of the
a. Quantity of electricity or charge, 𝑄𝑄 = electric bulb(𝐿) and acceleration due to gravity (g).
current×time= 𝐼𝑡 = 𝐼𝑇. Derive a formula by method of dimension
b. Electromotive force = Workdone/Charge showing the relationship between 𝑓 and the
𝑊 𝑀𝐿2 𝑇 −2 other variables?
𝐸= = = 𝑀𝐿2 𝑇 −3 𝐼 −1 .
𝑄 𝐼𝑇 Since frequency (𝑓) depends on mass (𝑚),
- Dimensionless quantities: They are physical length (𝐿) and acceleration due to gravity (g)
quantities without unit and dimension e.g then, 𝑓 ∝ 𝑚X𝐿YgZ, where x, y and z are the
mechanical advantage or force ratio, velocity powers to which each of the quantities are
ratio, coefficient of friction, tensile strain, relative raised in the formula.
density e.t.c. 𝑓 ∝ 𝑚X𝐿YgZ , 𝑓 = 𝐾𝑚X𝐿YgZ , 𝐾=constant of
- Application of dimensional analysis : proportionality. Dimension of 𝑓 = 𝑇 −1 =
1.It is used to derive formulas from a given sets 𝑀𝑜 𝐿𝑜 𝑇 −1 . Dimension of 𝑚=𝑀, Dimension of 𝐿=𝐿,
of variables. Dimension of g= 𝐿𝑇 −2 (acceleration).
2.Dimension is used to test the uniformity or Input the dimensions into the equation,
homogeneity of an equation. An equation is said 𝑓 = 𝐾𝑚X𝐿YgZ , to replace the variables.
to be homogenous if it is dimensionally correct i.e 𝑀𝑜 𝐿𝑜 𝑇 −1 = 𝐾(𝑀)X (𝐿)Y (𝐿𝑇 −2 )Z
the dimension of each quantity of the equation 𝑀𝑜 𝐿𝑜 𝑇 −1 = 𝐾. 𝑀X. 𝐿Y . (𝐿Z 𝑇 −2𝑧 )
are be the same. 𝑀𝑜 𝐿𝑜 𝑇 −1 = 𝐾. 𝑀X. 𝐿Y+Z. 𝑇 −2𝑍
Rules for uniformity or homogeneity of an Equate like terms on both sides of the equation;
equation 𝑀𝑜 = 𝑀X , x=0. 𝐿𝑜 = 𝐿Y+Z , y+z=0, y= −z,
1.If an equation is given as 𝑃 = 𝑄, then 𝑃 and 𝑄 𝑇 −1 = 𝑇 −2𝑧 , −2z = −1, z =-1/-2=1/2 ,
have the same dimensions. y= −z= −1/2 .
2.If an equation is given as 𝑃 = 𝑄 ± 𝑅, then 𝑃,𝑄 1 1
and 𝑅 have the same dimension. Substitute “x=0, y=− and z= ” back into the
2 2
3.If an equation is given as 𝑃 = 𝑄𝑅, then the original equation – 𝑓 = 𝐾𝑚X𝐿YgZ,
dimension of 𝑃 is equal to the product of the 𝑓 = 𝐾(m)0. (𝐿)-1/2. (g)1/2 =
𝐾𝑔1/2
=
𝐾√𝑔
= 𝐾√ .
𝑔
dimensions of 𝑄 and 𝑅. 𝐿 1/2 √𝐿 𝐿
3. The velocity of a moving particle is given by :
Examples : 𝑣 2= 𝑢2+2𝑎𝑠. Show that the formula is
1. The period of vibration 𝑇, of a liquid surface of dimensionally correct?.
a drop depends on the density (𝜌), radius (𝑟) and 𝑣=final velocity, 𝑢=initial velocity,
surface tension of the liquid (𝑌). Derive a formula 𝑎=acceleration, 𝑠=distance
using dimension, showing the relationship 𝑇 and Applying rule 2, the dimension of 𝑣 2 is same as
the other variables?. that of 𝑢2 and 𝑎𝑠.
Since period of vibration (𝑇) depends on density Dimension of 𝑣 2=(𝐿𝑇 −1 )2= 𝐿2𝑇 −2
(𝜌), radius (𝑟) and surface tension (𝑇) then, Dimension of 𝑢2=(𝐿𝑇 −1 )2= 𝐿2𝑇 −2
𝑻 ∝ 𝝆X𝒓Y𝒀Z , where x,y and z are the powers to Dimension of 𝑎𝑠 =(𝐿𝑇 −2 × 𝐿)= 𝐿2𝑇 −2 .
wich each of the quantities are raised in the 2.The distance travelled by a particle is given by
formula. 𝑇 ∝ 𝜌X𝑟Y𝑌Z , 𝑇 = 𝐾𝜌X𝑟Y𝑌Z , 𝐾=constant the equation, 𝑠=(𝑎𝑡2 + 𝑏𝑡 + 𝑐)m. What are the
of proportionality, Dimension of 𝑇 = 𝑇 = 𝑀𝑜 𝐿𝑜 𝑇, dimension of 𝑎, 𝑏 and 𝑐.
Dimension of density, 𝜌 = 𝑀𝐿−3 , Dimension of Applying rule 2 and rule 3, dimension of 𝑠 is the
𝑟 = 𝐿 Dimension of surface tension, 𝑌 = 𝑀𝑇 −2, same as that of 𝑎𝑡2, 𝑏𝑡 and 𝑐.
Input the dimensions into the equation, Dimension of 𝑠 = 𝐿,
𝐿
𝑇 = 𝐾𝜌X𝑟Y𝑌Z, to replace the variables i. 𝑠 = 𝑎𝑡 2, 𝐿 = 𝑎𝑇 2, 𝑎 = 2 = 𝐿𝑇 −2 .
𝑇
𝑀𝑜 𝐿𝑜 𝑇 = 𝐾 (𝑀𝐿−3 )X (𝐿)Y (𝑀𝑇 −2 )Z , 𝐿
ii. 𝑠 = 𝑏𝑡, 𝐿 = 𝑏𝑇, 𝑏= = 𝐿𝑇 −1 .
𝑇

8
Demystified Series Physics Demystified by Dr Timothy
iii. 𝑠 = 𝑐, 𝐿 = 𝑐, 𝑐 = 𝐿. the vectors or angular difference is 0°. III.Their
Question Drill – resultant is given by the arithmetic sum of both
1. The resonant frequency 𝑓o of a closed air vectors and acts in their common direction ,
column is known to depend on pressure 𝑃, the R=𝑨 ⃗⃗ + 𝑩
⃗⃗ . IV.The resultant of two or more
density 𝜌 of the air and the length 𝐿 of the air vectors is maximum or highest when they are
column. Derive a formula by method of parallel (θ=0°).
dimension showing the relationship between 𝑓o 2. Anti-parallel vectors – I.They act in opposite
and the other variables? Ans – 𝒇o= √ .
𝑲 𝑷 directions in a straight line. II.The angle between
𝑳 𝝆 the vectors or angular difference is 180°. III.Their
2. If the period of vibration 𝑇 of a simple resultant is given by the arithmetic substraction
pendulum depends on the mass of the bulb 𝑚, of both vectors and acts in the direction of the
length of the bulb 𝐿 and acceleration due to larger vector , R = 𝑨 ⃗⃗ − 𝑩⃗⃗ . IV.The resultant of
gravity g. Derive a formula by method of two or more vectors is minimum or least
dimension showing the relationship between 𝑇 when they are anti-parallel (θ=180°).
and the other variables? Ans – 𝑻 = 𝑲√ .
𝒈 - Two anti-parallel vectors of equal
𝑳 magnitudes will produce a zero resultant.
3.A wave is set up in a stretched string by 3. Perpendicular vectors – I.They act at right
plucking it. The velocity 𝑉, of the wave depends angles to each other. II.The angle between the
on the tension 𝑇, in the string its length 𝐿 and its vectors or angular difference is 90°. III.The
linear density 𝜇. Derive a formula by method of resultant is given by pythagora’s theorem i.e
dimension showing the relationship between 𝑇 𝑹𝟐 = 𝑨𝟐 + 𝑩𝟐 . The direction of the resultant
and the other variables? NB: Tension is a form of vector is represented as 𝜽.
force, linear density is the same as mass per unit 4. Diagonal vectors – I.They acts in diagonal
length. Ans – 𝑽 = √ .
𝑻 direction and are represented by the adjacent
𝝁 sides of a parallelogram. II.The angle between
4. The distance travelled by a particle is given by the vectors is between 0° and 180° (0° < 𝜃 <
the equation; 180°). III.The diagonal of the parallelogram
𝑠 = 𝑎𝑡3 + 𝑏𝑡2 + 𝑐𝑡 + 𝑑 in meters, the unit of 𝑎 is draw from the intersection of both vectors
A.ms-2 B.ms-3 C.ms-1 D.m. represents the resultant and it gotten by cosine
5. Which of the following properties of steel bar rule. There are two forms of orientation of
can be measured in terms of dimension of length diagonal vectors :
only? A.weight B.density C.pressure D.volume. a. The two vectors which meet at a common
point 𝑃 and their resultant originate the point 𝑃
● Scalars and Vectors – and θ is the angle between them;
- Scalar quantities are quantities which have
magnitude only but no specified direction. They 𝑨 𝑹
are described by their magnitudes only. 𝜽 𝜶 The resultant 𝑅 is given by ;
- Vector quantities are quantities which have 𝑷 𝑩 R2 = 𝑨2 + 𝑩2 + 2𝑨𝑩 𝒄𝒐𝒔 𝜽
magnitude and specified direction. They are b. The two vectors arranged in a triangle such
described by their magnitude and direction. that the resultant from the tail of one vector to
Vector quantities Scalar quantities the tip of the other and θ is the angle between
Displacement,velocity Distance,speed, them.
acceleration,force, mass,time,length,
weight,momentum, volume,viscosity, R A
impulse,moment, temperature,work, 𝜽
couple(torque), energy,power,heat P 𝜶 The resultant is given by :
friction, all field inertia,pressure, all B R2 = 𝑨2+𝑩2−2𝑨𝑩 𝒄𝒐𝒔𝜽 .
intensities, stress, e.t.c. potentials,velocity The direction of the resultant 𝛼 is calculated
𝒔𝒊𝒏 𝜶 𝐬𝐢𝐧 𝜽 𝑨𝒔𝒊𝒏𝜽
ratio,mechanical using ; = OR 𝐭𝐚𝐧 𝜶 = .
𝑨 𝐑 𝑩+𝑨𝒄𝒐𝒔𝜽
advantage,density, - Triangle of vectors – When three concurrent
current,potential vectors i.e pass through a common point, acts on
difference e.tc body, hence they can be represented in
- Resultant vector R – It is the single vector that magnitude and direction by the three sides of a
gives the same effect in magnitude and direction triangle.
as a set of vectors acting together at a point. 1.Two vectors and their resultant – The tip to
- Orientation and arrangement of vectors : tail rule is used in describing the resultant i.e the
1. Parallel vectors – I.They act in the same resultant starts from the tail of one vector and
direction in a straight line. II.The angle between ends in the tip of the other vector.

9
Demystified Series Physics Demystified by Dr Timothy
the other is zero.
⃗𝑷
⃗ ⃗⃗
𝑸 5. Resolution of more than two vectors or
multiple vectors – Each of the vectors are
⃗𝑹
⃗ resolved into their perpendicular components.
By the tip to tail rule, the resultant of the vectors The resultant is then calculated from the two
⃗𝑸⃗ and ⃗𝑹 ⃗ in the diagram above, is ⃗𝑷 ⃗ as it starts perpendicular components using pythagor’s
theorem.
from the tail of 𝑹 ⃗⃗ and ends at the tip of 𝑸 ⃗⃗ . The
- Position vectors ⃗𝑽 – Position vector is a vector
vector equation relating the vector is :
whose effect is on one point in space. Position
⃗𝑷⃗ = ⃗𝑸 ⃗ + ⃗𝑹⃗ or ⃗𝑹 ⃗ = ⃗𝑷 ⃗ − ⃗𝑸⃗ or ⃗𝑸
⃗ = ⃗𝑷
⃗ − ⃗𝑹
⃗. 𝑥
2. Three equilibrant vectors – When three vectors in the (x, y) coordinates is given as (𝑦).
vector ⃗𝑷 ⃗ , ⃗𝑸
⃗ and ⃗𝑹 ⃗ acts simultaneously on a body y
to keep it stationary i.e in equilibrium, then each ⃗1 3
𝑉
vector is the equilibrant of the other two . 𝑹 ⃗⃗ is the ⃗2
𝑉 2 𝑉⃗3

⃗ ⃗⃗ ⃗⃗
equlibrant of 𝑷 and 𝑸, or 𝑸 is the equilibrant of ⃗𝑷 ⃗ 1 ⃗4
𝑉
and 𝑹. ⃗

⃗⃗
𝑸 −4 −3 −2 −1 1 2 3 4 x
⃗𝑷
⃗ ⃗𝑷
⃗ ⃗𝑸
⃗ −1 ⃗5
𝑉
⃗⃗
𝑹 𝑉⃗7 −2 𝑉⃗6
⃗⃗
𝑹 ⃗𝑉8 −3
In the triangle diagram above, notice that all the ⃗𝑉1 = (−1) , 𝑉 ⃗ 2 = (−3) , 𝑉⃗ 3 = (1), 𝑉⃗ = (3),
vectors moves in similar direction round the 3 2 2 4 1
triangle with the tip of one touching the tail of the ⃗𝑉5 = ( 2 ), 𝑉⃗ 6 = ( 3 ), 𝑉
⃗ 7 = (−2), 𝑉 ⃗ 8 = (−2).
−1 −2 −2 −2
other. The vector equation of equilibrant - Relative velocity – It is the velocity of a body in
vectors is : ⃗𝑷 ⃗ + ⃗𝑸 ⃗ + ⃗𝑹⃗ = 𝟎. Thus, the vector respect to another body.
sum of equilibrant vectors is zero. - One dimensional relative velocity – Relative
- Equilibrant vector of two or more vectors is velocity between bodies along parallel lines.
the single vector which will balance all other - If two bodies A and B are moving with speed 𝑣 A
vectors keeping them in equilibrium. and 𝑣 B respectively then, the relative velocity of
- Equilibrant vector is equal and directly body A respect to body B given by :
opposite to the resultant vector (equal in I. 𝒗AB = 𝒗A−𝒗B , if they are moving in the same
magnitude but opposite in direction). direction.
- Resolution of vectors into components – II. 𝒗AB = 𝒗A+𝒗B , if they are moving in the
1. Resolution of oblique vectors – A vector P opposite direction.
inclined at angle θ to the horizontal or vertical, - Two bodies are moving in opposite direction
has two perpendicular componets; PY (vertical when approaching and moving in the same
component) and PX (horizontal component). direction when receeding(chasing).
PY PY - Two dimensional relative velocity – Relative
velocity between bodies along non-parallel lines.
P P
𝜽 e.g assume a bus and truck took of from the same
𝜽 point with the bus travelling northwards with
1. 𝑷X 2. PX velocity 𝑣 B while the truck is travelling eastwards
In diagram 1, P is inclined at angle θ to the with velocity 𝑣 T.The relative velocity of the bus
horizontal, PY = 𝐏𝒔𝒊𝒏𝜽 while PX = 𝐏𝒄𝒐𝒔𝜽. In with respect to the truck 𝑣 BT is represented as –
diagram 2, P is inclined at angle θ to the 𝒗B N
vertical, 𝐏Y= 𝐏𝒄𝒐𝒔θ while PX= 𝐏𝒔𝒊𝒏θ. 𝒗B 𝒗BT W E
- The component (axis) that makes an angle θ
S
with the vector takes 𝒄𝒐𝒔 𝜽 while the 𝒗T 𝒗T
perpendicular component takes 𝒔𝒊𝒏 𝜽. The direction of 𝑣 T was reversed inorder for the
2. Resolution of parallel vectors – The resultant 𝑣 BT to obey the tip-to tial rule, i.e runs
component of each parallel vector along their from the tail of 𝑣 T to the tip of 𝑣 B. 𝒗BT2= 𝒗B2+𝒗T2.
𝒗
common direction is its full positive magnitude. The direction of 𝑣 BT is given by – 𝒕𝒂𝒏θ= 𝑩 .
𝒗𝑻
3. Resolution of anti-parallel vectors – The
component of each anti-parallel along the
Examples :
direction of the other is its full negative
1. Which of the following are vector quantities?
magnitude.
I.Work II.Displacement III.Acceleration
4. Resolution of perpendicular vectors – The
IV.Electric field intensity V.Magnetic induction
component of each vector along the direction of

10
Demystified Series Physics Demystified by Dr Timothy
A.I,II and III B.II,III and IV C.III,IV and V D.II,III,IV to arrive at a point T. What is the total
and V. displacement A.6km south B.4km north C.6km
Work and energy are scalar quantity. All field north D.4km east.
intensity(electrical,magnetic & gravitational) are 4km AB=10km, BC=4km,
vectors. Magnetic induction is same as magnetic B C CD=6km, DT=4km
flux density and it is a vector. 10km 6km
2. Which of the following pairs of physical T D
quantities comprises vectors? A.Capacitance and 4km
inductance B.Force ratio and velocity ratio A
C.Friction and momentum D.Energy and power The lorry starts its travel from A and ends in T.
E.Electric field potential and electric field Horizontal component= BC−DT = 4−4 = 0km.
intensity. Vertical component = AB−CD = 10−6 = 4km.
Capacitance,inductance,energy,power,force ratio Thus, the total displacement = AT = 4km due
(mechanical advantage), velocity ratio, all North. Total distance travelled by the
potential (electric and gravitational) are all lorry=10+4+6+4=24km.
scalars. Friction, momentum and electric field 7. If the angle between two vectors 𝑃 and 𝑄 is 0°,
intensity are vectors. the vectors are said to be A.parallel
3. Which of these quantities are vectors? B.perpendicular C.intersect at angle of 45°
I.Moment II.Temperature gradient III.Density D.intersect at angle of 60°.
IV.Temperature A.I and II B.II and III C.I and IV Two vectors are said to be parallel when their
D.II,III and IV. angular difference is 0°.
Moment and temperature gradient are vectors 8. The resultant of two forces acting on an object
while temperature and density are scalars. minimum when the angle between them is
4. A man walks 5km south and then 3km in the A.180° B.90° C.45° D.0° .
direction 60° west of south.His distance from the 9. The resultant of two forces acting on an object
starting point is A.7.00km B.7.50km C.8.00km maximum when the angle between them is
D.10.72km. A.180° B.90° C.45° D.0° .
Distance is a scalar quantity and is independ- 10. The resultant of two forces acting on an
ent of the direction. Distance=5+3=8km. object is maximum when the angle between them
NB – If the question had stated that we should is A.180° B.90° C.45° D.0°.
calculate for the displacement, then we will have 11. Two forces 𝐴 and 𝐵 act at a point. If their
to take the direction into consideration. resultant is given by (𝐵 − 𝐴) in the direction of 𝐵,
N then A.𝐴 and 𝐵 are equal B.𝐴 is greater than 𝐵
W E C.The angle between 𝐴 and 𝐵 is 0° D.The angle
5km S between 𝐴 and 𝐵 is 90° E.The angle between 𝑨
R θ θ=180−60=120°, and 𝑩 is 180°.
Displacement=R, The two vectors are anti-parallel i.e θ=180°,as
60°
. 3km R2= 52+32−2×5×3 𝑐𝑜𝑠120°, their resultant is given by their arithemetic
R2=25+9−30(−0.5)=34+15=49, difference. Also 𝐵 is greater than 𝐴, as the
R = √49 = 7km. option A. resultant moves in the direction of 𝐵.
5. A boy travels 12km eastwards to a point B and 12. A vector of magnitude 5 units in the north
then 5km southward to another point C. direction is combined with another vector to give
Calculate the difference between the magnitude a zero resultant. The other vector is A.5units in
of the displacement of the boy and the distance the north direction B.5units in the south
travelled by him A.4.0km B.7.0km C.13.0km direction C.5units in the east direction D.5units
D.17.0km E.30.0km in the west direction E.−5units in the south
A 12km B direction.
Anti-parallel vectors(vectors acting in opposite
5km directions), of equal magnitude produce zero
resusltant.
C 13. What is the magnitude of the resultant two
Distance=AB+BC(neglecting the direction, as it forces 20𝑁 and 15𝑁 inclined at 90° to each other?
is a scalar quantity)=12+5=17km. A.35𝑁 B.5𝑁 C.25𝑵 D.10𝑁.
Displacement=AC , AC2 = AB2+BC2 = 122+52 , 20𝑁 20𝑁
AC2 = 144+25=169 , AC = √169 =13km. R2 = 202+152 ,
Difference between the magnitude of the R2 = 400+225
displacement and distance =17−13 =4km. R2 = 625 ,
6. A lorry travels 10km northwards, 4km 15𝑁 R = √625 =25N.
eastwards, 6km southwards and 4km westwards 14. An object moves 4m eastward(E) and then

11
Demystified Series Physics Demystified by Dr Timothy
3m southward(𝑆). Its displacement from its magnitude 5N acts due east, the other of
original position is? A.1m SE B.5m SE C.7m SE magnitude 3N acts in a direction of 030°.
D.12m SE. Calculate the resultant vector and the direction?
4m 3N The direction of the 3N
θ force is 030°.
𝑅 3m R2 = 32+42 = 9+16 = 25, 30° Hence, the angle between
R2= √25 = 5m. 60° the forces=90−30=60°.
The direction of the resultant is in the south- 5𝑁
eastern (SE) direction. Displacement = 5m SE. 3N 3N
15. Two forces each of magnitude 10N acts on a R R2=32+52+2×3×5 𝑐𝑜𝑠60° ,
body, one towards the North and the other R2=9+25+30(0.5)
towards the East.What is the magnitude and 60° 𝛼 R2=34+15=49, R = √49,
direction of the resultant force? A.10√𝟐N, 5N R =7N.
𝑠𝑖𝑛 𝛼 sin 60° 3 sin 60° 2.6
N45°E B.10√2N, N45°W C.20N, N45°W D.20N, = , sin 𝛼 = = = 0.371 ,
3 7 7 7
N45°E . 𝛼 = tan 0.371 =21.77° ≈ 21.8°. Alternatively,
−1
10N 10N , 3𝑠𝑖𝑛60° 2.6 2.6
tan 𝛼 = = = = 0.4 ,
𝜃 5+3𝑐𝑜𝑠60° 5+1.5 6.5
𝜃 R R2 = 102+102 , 𝛼 = tan 0.4 = 21.8°.
−1

R2 = 100+100 = 200 , 20. An aircraft travelled from Calabar to Kano as


follows. It flew first to Ilorin covering a distance
10N R = √200 =10√2𝑁.
10 of 300Km, 30°West of North and then flew
tan 𝜃 = = 1 , 𝜃 = tan 1 = 45°,
−1
400km, 60°East of North to Kano. What is the
10
The resultant force is 10√2𝑁, 𝑁45°𝐸. total displacement? A.567km B.594km C.500km
16. Two forces P and Q acting perpendicularly to D.594km E.600km.
each other have resultant of 50N. If P makes an 60° +30°=90° K I=Ilorion, K=Kano,
angle of 30° with the resultant, calculate the 400km C=Calabar.
magnitude of Q A.55.0N B.43.3N C.40.0N I 60°
D.25.0N E.15.8N. 90° Displacement=KC,
P P 30° KC2 = 4002+3002 ,
30° sin 30° =
𝑄
, 300km 30° KC2=160000+90000,
50
C KC2 = 250000 ,
30° 50N Q = 50sin 30° ,
Q = 50×0.5 = 25N. KC = √250000 = 500km. Total distance of the
Q aircraft = 300+400= 700km (neglecting the
17. A block of mass 2.0kg resting on a smooth direction as distance is a scalar quantity).
horizontal plane is acted upon simultaneously by 21. A man walks 8km North and then 5km in a
two forces, 10N due North and 10𝑁 due East. The direction 60°East of North.Calculate the total
magnitude of the acceleration produced by the displacement of the man. A.11.36km B.12.36km
forces on the block is A.0.10ms-2 B.7.07ms-2 C.13.00km D.14.36km.
C.10.00ms-2 D.14.10ms-2 E.20.00ms-2 . R=Displacement,
10N 10N N 5km θ=90° + 30° =120°,
60°
𝑅 R2 = 102+102 ,
R2 = 100+100 = 200 , 30° R2=82+52−2×8×5 𝑐𝑜𝑠120° ,
90° R R2=64+25−80(−0.5),
R = √200 =10√2N.
8km R2=89+40=129,
10N R = 𝐹 = 𝑚𝑎,
R = √129 = 11.36km.
𝐹=resultant force=10√2N, m=mass=2.0kg,
𝐹 10√2
22. Which of the diagrams in the diagram vectors
𝑎=acceleration, 𝑎 = = = 5√2 = 7.07ms-2. P and Q?
𝑚 2
18. Two forces of magnitude 7N and 3N act at I. P II. P III.P
right angles to each other. The angle θ between R R R
the resultant and the 7N force is given by
3
A.𝑐𝑜𝑠 𝜃 = B.𝑠𝑖𝑛 𝜃 = .
3 Q Q Q
7
𝟑 3
7 IV. P V. P
C.𝒕𝒂𝒏 𝜽 = D.𝑐𝑜𝑡θ= R R
𝟕 7
7N 7N
𝜃 3 Q Q
R tan 𝜃 = ,
𝜃
7
1 7 A.I B.II C.III D.IV E.V .
cot θ = = . Using the tip to tail rule, in diagram V, the
tan 𝜃 3
3𝑁 resultant R starts from the tail of Q and ends at
19. Two forces act on a body, the one of the tip of P. The vector equation is : R = P + Q .

12
Demystified Series Physics Demystified by Dr Timothy
23. ⃗⃗⃗⃗
𝑣1 ⃗⃗⃗⃗
𝑣2 I. Resoving into the total vectors in either
The figure above shows two velocities ⃗⃗⃗⃗ 𝑣1 and ⃗⃗⃗⃗
𝑣2 . direction, with the vertical component RY taking
Which of the following diagrams correctly sine and horizontal component RX taking cosine.
represents the vector difference: 𝑤 ⃗⃗ = ⃗⃗⃗⃗
𝑣1 − ⃗⃗⃗⃗
𝑣2 Also take note of the positive sign used for +Y
A. ⃗⃗⃗⃗
𝒗𝟏 B. ⃗⃗⃗⃗
𝑣1 and +X axes and negative sign used for −Y and
−X axis.
⃗𝒘
⃗⃗ ⃗⃗⃗⃗
𝒗𝟐 𝑤
⃗⃗ ⃗⃗⃗⃗
𝑣2 RY= 3√3 + 10𝑠𝑖𝑛30° − 6𝑠𝑖𝑛60°
1 √3
RY=3√3 + 10( )−6 =3√3 +5−3√3 =5N
2 2
√3 1
C. D. RX=3+10𝑐𝑜𝑠30° −6cos60° =3+10( )−6( )
2 2
𝑤
⃗⃗ 𝑤
⃗⃗
RX=3+5√3 −3=5√3N.
⃗⃗⃗⃗
𝑣2 𝑣2
⃗⃗⃗⃗
II. First resolve the oblique (inclined) forces into
ther vertical and horizontal components.
𝑣1
⃗⃗⃗⃗ 𝑣1
⃗⃗⃗⃗ .
Vertical component of 10N – 10𝑠𝑖𝑛30° = 5N,
The vector difference equation given above is :
Horizontal component of 10N – 10𝑐𝑜𝑠30° =
𝑤
⃗⃗ = ⃗⃗⃗⃗
𝑣1 − ⃗⃗⃗⃗ 𝑣2 , rearranging the vector equation we
5√3N.
will have, ⃗⃗⃗⃗ 𝑣1 = 𝑤 𝑣2 . Hence, ⃗⃗⃗⃗
⃗⃗ + ⃗⃗⃗⃗ 𝑣1 is the resultant
vector of 𝑤 ⃗⃗ and ⃗⃗⃗⃗
𝑣2 . Using the tip to tail rule, the Vertical component of 6N – 6𝑠𝑖𝑛60° = 3√3N,
resultant vector ⃗⃗⃗⃗ 𝑣1 should start from the tail of 𝑤 ⃗⃗ Horizontal component of 6𝑁 – 6𝑐𝑜𝑠60° = 3N,
and end at the tip of ⃗⃗⃗⃗ 𝑣2 (option A). (3√3 +5)N The two 3√3N in the
- Option B – ⃗⃗⃗⃗ 𝑣2 = 𝑤 𝑣1 , ⃗⃗⃗⃗
⃗⃗ + ⃗⃗⃗⃗ 𝑣2 is the resultant of vertical axis cancel out
⃗⃗ and ⃗⃗⃗⃗
𝑤 𝑣1 . 3N (3+5√3)N
- Option C – 𝑤 ⃗⃗ + ⃗⃗⃗⃗ 𝑣2 = 0 , they are
𝑣1 + ⃗⃗⃗⃗ The two 3𝑁 in the horiz-
equilibrant vectors. 3√3N ontal axis cancel out.
- Option D – 𝑤 ⃗⃗ = ⃗⃗⃗⃗
𝑣1 + ⃗⃗⃗⃗𝑣2 , 𝑤
⃗⃗ is the resultant of ⃗⃗⃗⃗
𝑣1 Net vertical force, RY = 5N. Net horizontal force,
and ⃗⃗⃗⃗
𝑣2 . RX= 5√3N.
24. Two forces 3N and 4N act on a body in III. This is the most quick method but involves a
directions due to north and due east respectively. calculator or knowledge of quadrant. The angles
Calculate their equilibrant A.5N, 53°east of north which meets the forces starting from the positive
B.5N, 53°west of south C.5N, 37°north of east X-axis in an anticlockwise direction :
D.7N, 37°west of north E.7N, 37°south of north Vertical forces, 𝑅 Y =10𝑠𝑖𝑛30° + 3√3𝑠𝑖𝑛90° +
3N 3N R2 = 32+42 = 9+16 , 6𝑠𝑖𝑛(180+60)° + 3𝑠𝑖𝑛360° ,
𝑅 𝜃 R2 = 25 , 𝑅 = √25, √3
𝑅 Y = 5+3√3 +6(− )+0 = 5+3√3 −3√3 = 5𝑁.
𝜃 R = 5𝑁 2
tan θ = 4/3 = 1.33, Horizontal forces, 𝑅 X=10𝑐𝑜𝑠30° + 3√3𝑐𝑜𝑠90° +
4𝑁 θ = tan−1 1.33 = 53° 6𝑐𝑜𝑠(180+60)° + 3𝑐𝑜𝑠360° ,
1
The resultant force is 5N, N53°E. 𝑅 Y = 5√3 +0+6(− )+ 3 = 5√3 −3+3 = 5√3N.
2
The equilibrant vector is equal in magnitude but
𝑅Y 𝑅 2 = 𝑅 Y2+𝑅 X2 =52+(5√3)2 ,
opposite in direction to the resultant. Thus, the
𝑅 𝑅 2 =25+25(3) = 25+75 ,
equilibrant force = 5N, S53°W or 53° West of
𝑅 2=100, 𝑅 = √100 =10N.
South.
𝑅X
25. The component of vector X along the
27. A force of 25N inclined at N38°E and another
direction of vector Y is zero when A.X and Y are
force 15N inclined at N22°W act on an object.
in opposite direction to each other B.X is at an
Calculate the magnitude of their resultant. A.20N
angle of 45° to Y C.X is perpendicular to Y D.X
B.25N C.30N D.35N.
and Y are parallel to each other .
N 25N
The component each perpendicular vector
15N
(𝜃 =90°) along the direction of the other is zero.
26. 3√3𝑁
22° 38°
10𝑁
W E
30° θ=22°+38°=60° . The angle θ between the 15N
3𝑁
and 25N force is 60°.
60° P
15N 25N
60°
6𝑁
The figure above shows 4 forces 3N, 10N, 3√3N R2 = 152+252+2×15×25cos 60° ,
and 6N acting on a particle P, the resultant of the R2 = 225+625+750(0.5) = 850+375 ,
four forces is A.10√3N B.10N C.5√3N D.5N. R2 = 1225 , R = √1225 = 35N.

13
Demystified Series Physics Demystified by Dr Timothy
Alternatively : Resolution of multiple vectors : 31. A boy pulls a nail from the wall with a string
25N tied to the nail. The string is inclined at angle of
15N 60° to the wall.If the tension in the string is 4N,
what is the effective force used in pulling the
22° 38° nail? A.2N B.2√𝟑N C.4N D.4N.
68° 52° The effective force used in
Resolve each forces into its vertical and 60° pulling or removing the nail,
horizontal component : is the horizontal component
15N force ; FY = 15sin 68° = 13.91N, wall 4N of the applied force or tension
FX = 15cos 68° = 5.62N. in the string. 𝐹 X= 𝑇𝑠𝑖𝑛θ (as the string is inclined
25N force ; FY = 25sin 52° = 19.70N, to the wall i.e vertical). 𝐹 X = 4sin 60° = 2√3𝑁.
FX = 25cos 52° = 15.39N. 32. North 10𝑁 A body on the ground is
(13.91+19.70)N acted on by a force of 10N
at point 𝑃 as shown in the
30° diagram. What force is
5.62N 15.39N 𝑃 East needed to stop the
ΣFY=13.91+11.82=33.61N, body from moving northward? A.5N in the
ΣFX=15.39−5.62=9.77N, direction of north B.5N in the direction south
33.61N C.5√𝟑N in the direction of south D.10N is the
south-west direction.
R The vertical(northward) component of the 10N
force=10𝑐𝑜𝑠30° = 10(√3/2)=5√3N. The force
9.77N
required to stop the body from moving
R2 = 33.612+9.772 = 1129.63+95.45 ,
northward must be acting in an opposite
R2 = 1225.08 , R = √1225.08 = 35.00N. direction i.e southward and of equal magnitude
28. 10N
5√3N.
33. Y 10N

17N 20N X1 30° X


O
6N 3𝑁
Four co-planar forces of magnitudes 10N, 17N,
In the diagram above, the resultant force in Y-
6N and 20N cats at a point O as shown in the
direction is A.2N B.5.7𝑁 C.8.0𝑁 D.11.7N .
diagram above. Determine the magnitude of the
The vertical component of the
resultant force A.53.0N B.21.0N C.7.0N D.5.0N.
10𝑁=10𝑠𝑖𝑛30° =5𝑁. The resultant force in the
FY = 10 − 6 = 4N. FX = 20 – 17 = 3N.
Y-direction, 𝐹 =5−3=2𝑁. (The 5𝑁 and 3𝑁 acts
4N 4N R2 = 42 + 32 = 16 + 9 ,
upward and downward respectively).
R R2 = 25 , R = √25 34. X Find the resultant of the forces
R = 5.0N. along OY in the diagram
60N
3N 120° 80N
29. When a force R is resolved into two O Y
perpendicular components P and Q such that the A.20N B.28N C.30N D.50N E.110N.
angle between P and R is θ. Which of the In finding the resultant along the OY direction,
following pairs represents the magnitude of 𝑃 we will have to figure out the angle which the
and 𝑄 respectively A.R𝑠𝑖𝑛θ and R𝑐𝑜𝑠θ B.R𝒄𝒐𝒔θ 60N force makes with the horizontal.
and R𝒔𝒊𝒏θ C.R𝑐𝑜𝑠θ and R𝑡𝑎𝑛θ D.Rtanθ and 60𝑁 The horizontal component of the
R𝑠𝑖𝑛θ E.Rtanθ and Rcosθ. 60𝑁 force = 60𝑐𝑜𝑠60° = 30N
The vector R is inclined at θ to the vertical. The
60° 120°
vertical component P = Rcos θ while the 60cos 60° 80𝑁
horizontal component Q = Rsin θ. 30N 80N
30. A ball is moving at 18ms-1 in a direction The resultant force along OY = 80−30 = 50N.
inclined at 60° to the horizontal. The horizontal 35. 100𝑁
component of its velocity is A.9√3ms-1 B.6√3ms-
1 C.6ms-1 D.9ms-1
10kg 45° 60𝑁
𝑣Y 𝑣Y 𝑣 =18ms-1,
𝑣 𝑣 X = 𝑣𝑐𝑜𝑠𝜃 = 18𝑐𝑜𝑠60° , Two forces 100𝑁 and 60𝑁 act on an object of
60°
𝑣 X = 9ms-1. mass 10kg as illustrated in the diagram above.
𝑣X 𝑣 Y = 𝑣 sin θ .

14
Demystified Series Physics Demystified by Dr Timothy
Neglecting friction, calculate the acceleration of vectors.
the object A.4.0ms-2 B.13.1ms-2 C.16.0ms-2 39. The component vectors Q and R are (-3.0,5.5)
D.22.6ms-2. and (9.2,4.4) respectively. Determine the
The motion of the box is along the horizontal components of Q+R A.(3.7,7.4) B.(6.2,9.9)
direction hence, the force causing the motion is C.(2.5,-4.8) D.(14.7,1.4).
the 60N and the horizontal component of the Q+R = (−3.0+9.2, 5.5+4.4) = (6.2, 9.9).
100N force. Horizontal component of 100N= 40. Two cars moving in the same direction have
100𝑐𝑜𝑠45° = 70.7N. speeds of 100km/h and 130km/h. What is the
Total force F causing motion=60+70.7=130.7𝑁. velocity of the faster car as measured by an
F = ma, a= =
𝐹 130.7
= 13.07 ≈ 13.1ms-2. observer in a slower car? A.230km/h B.130km/h
𝑚 10
C.200km/h D.30km/h.
36. 16N
𝑣 F=speed of the fast car=130km/h, 𝑣 S=speed of
4N
the slow car=100km/h, 𝑣 FS=speed of the faster
30° car relative to the slower car=?, 𝑣 FS= 𝑣 F−𝑣s
30° (same direction). 𝑣 FS=130−100=30km/h.
2N
41. The driver of a car moving with uniform
In the diagram, the resultant force along the x-
speed of 40m/s observes a truck approaching in
axis is A. 11.46N B.13.46N C.19.29N D.14.02N.
the opposite direction with a speed of 20m/s.
Along the x-axis, the 16N force is 60° i.e
Calculate the speed of the car relative to that of
(30° +30°) to the axis, the 4N force is 30° to the
the truck. A.60m/s B.40m/s C.20m/s D.10m/s
aixs. Resultant force along the x-axis is the total
𝑣 C=speed of car=40m/s, 𝑣 T=speed of truck=
force along the horizontal.
√3
20m/s, 𝑣 CT=velocity of car relative to the truck=?,
FX=16𝑐𝑜𝑠60° + 4𝑐𝑜𝑠30° + 2 = 16(0.5)+4( )+2, 𝑣 CT= 𝑣C+𝑣 T(approaching, opposite direction),
2
FX=8+2√3 +2=10+2√3 = 10+3.46=13.46N. 𝑣 CT= 40+20=60m/s.
37. 8𝑁 42. A boat man facing north wants to cross a
flowing river to a point directly opposite his
4N 60° 10N position at the other bank. If the river is flowing
60° eastwards, in what direction should he row his
6N boat? A.west B.east C.north-west D.north-east
The diagram above shows forces 4N, 6N,10N and E.south-east.
N
8N which act at a point O in the direction 𝑣B
indicated. The net horizontal force is A.7√3N 𝑣B 𝑣 BR W E
B.17N C.√3N D.13N. θ S
Resolving into the horizontal direction – 𝑣R 𝑣R
𝐹 X=8𝑐𝑜𝑠60° + 6𝑐𝑜𝑠60° +10−4 = 4+3+6 = 13𝑁 The direction of the river is reversed in order to
due east. obey the tip to tail rule, hence the direction 𝜃 of
- Question Drill – Calculate the horizontal the resultant velocity of the boat 𝑣 BR relative to
component of the forces in the diagram the river is north-west direction.
5N 8N 43. A ship moving notthward with speed 𝑣𝑠 is
acted upon by a wind blowing due west at a
60° speed 𝑣𝑤 . Which of the following diagrams
45° correctly indicates the velocity 𝑣𝑅 of the wind
A.13.00𝑁 B.9.49N C.8.26N D.8.16N E.1.33N. relative to the ship?
38. Which of the vector diagram below A. 𝒗W B. 𝑣W
represents the following position vectors?
𝒗S 𝑣S
⃗ 1 = (1), 𝑉
𝑉 ⃗ = (2), 𝑉⃗ = ( 2 ), 𝑉
⃗ =( 1 )
1 2 2 3 −2 4 −2 𝒗R 𝑣R
A. B. ⃗1
𝑉 𝑉⃗2
⃗𝟏
𝑽 ⃗𝟐
𝑽 C. 𝑣W D. 𝑣W

𝑣S 𝑣S
⃗𝑽𝟒 ⃗𝑽𝟑 ⃗3 ⃗4 𝑣R 𝑣R
𝑉 𝑉
C. ⃗1
𝑉 ⃗2
𝑉 D. The velocity of the ship is 𝑣 S due north while the
⃗1
𝑉 ⃗2
𝑉 velocity of wind is due west.
𝑣W The resultant velocity of the ship
relative to the wind 𝑣 SW or 𝑣 R is
⃗3
𝑉 ⃗4
𝑉 ⃗3 𝑉
𝑉 ⃗4 𝑣 S in the north-west direction.
Option A is the correct match for the position According to the tip-to-tail rule,

15
Demystified Series Physics Demystified by Dr Timothy
𝑣 R runs from the tail of 𝑣 S to the tip of 𝑣 W (option 47. A tugboat is travelling from Asaba to Onitsha
A). across the river Niger with a resultant speed of
44. A bird flying due north with a velocity of 20knots. If the river flows at 12knots, the
15ms-1 shed a feather in an air current blowing at direction of motion of the boat relative to the
20ms-1 due east. The feather will move with a direction of water flow is A.36.87° B.53.13°
velocity of A.25ms-1 east of south B.25ms-1 east C.90° D.136.87°.
of north C.25ms-1 west of south D.20ms-1 east 12knots O 12knots O River flow reversed
E.15ms-1 north. River flow θ
20ms-1
20knots
15ms-1 𝑣R 𝑣 R2= 152+202 ,
θ 𝑣 R2 = 225+400 , A A
12
𝑣 R2 = 625, 𝑣 R= √625 = 25ms-1 cos θ = = 0.6 , θ = cos 0.6 = 53.13°.
−1
20 4 20
tan 𝜃 = = = 1.33, 𝜃 = tan−1 1.33 = 53°. 48. A stream is flowing at 0.75ms-1 and a boat
15 3
The velocity of the feather is 25ms-1 at N53°E or heading perpendicularly to the stream landed at
53° east of north. the opposite bank at an angle 30°. Calculate the
45. A particle A travels North west at 10ms-1 and velocity of the boat? A. 1.50ms-1 B.1.00ms-1
another B travels 15° South of East at 6ms-1. What C.0.86ms D.0.65ms .
-1 -1

is the velocity of A relative to B? A.15.5ms-1 0.75ms-1


B.5.7ms-1 C.8.7ms-1 D.16ms-1. Stream flow
30°
North West bearing or North East bearing or
South West bearing or South East bearing is 30° 𝑣b
𝑵45°𝑾 or 𝑵45°𝑬 or 𝑺45°𝑾 or 𝑺45°𝑬
0.75 0.75 0.75
respectively. Hence particle A travels 45° North sin 30° = , 0.5 = , 𝑣b = = 1.5ms-1.
𝑣𝑏 𝑣𝑏 0.5
of west.
A 𝑣 AB= 𝑣 A2+𝑣 B2−2𝑣 A𝑣 B𝑐𝑜𝑠θ,
Jamb past questions on measurements,
θ=75° + 90° + 45° = 210°,
scalars and vectors :
10ms-1 45° 𝑣 AB [1978/1,9,10,1980/11,27,1981/15,1982/3,28,
1983/2,39,1984/3,21,1985/1,7,1986/4,1987/1,
75° 2,1988/2,4,5,1989/1,2,1990/1,2,1992/1,1993/
15°
6ms-1 1,2,4,1994/1,3,1995/1,3,4,1996/2,1997/1,1998
B /1,2,3,12,2000/2,7,2001/6,9,2002/7,2003/4,12,
𝑣 AB2=102+62−2×10×6𝑐𝑜𝑠210° , 2004/18,2005/33,2006/20,32,2007/42,2008/1
𝑣 AB2 = 136−(−103.92) = 239.92 , ,2,4,7,2009/1,3,2011/5,2012/2,2013/4,2014/2,
3,4,2015/3,2016/16,23,2017/28].
𝑣 AB= √239.92 = 15.48ms-1 = 15.5ms-1.
46. An aircraft attempts to fly due north at
100kmh-1. If the wind blows against it from east
to west at 60kmh-1, its resultant velocity is
A.117kmh-1, N31°E B.127kmh-1, N31°E
C.117kmh-1, N31°W D.127kmh-1, N31°W
60kmh-1

100kmh-1
𝑣R θ
𝑣 R2=1002+602=10000+3600,
𝑣 R2 = 13600 , 𝑣 R= √13600 = 116.6 = 117kmh-1.
60
tan θ = = 0.6 , θ = tan−1 0.6 = 30.9 =31°.
100
The resultant velocity is 117kmh-1, N31°W.

16
Demystified Series Physics Demystified by Dr Timothy

CHAPTER 2 – MOTION
● Motion – This is the change in position of a - Uniform speed 𝒗 – speed of a body when it
body with time. It is divided into two : kinematics covers equal distances in equal time interval.
and dynamics. Kinematics deals with the motion - Eqaution of linear motion :
of a body neglecting the force which causes 1. 𝒗 = 𝒖 ± 𝒂𝒕 2. 𝒗2= 𝒖2±2𝒂𝒔
motion. Dynamics deals with the motion of a 𝒗+𝒖
3. 𝒔 = ( ) 𝒕
𝟏
4. 𝒔 = 𝒖𝒕 ± 𝒂𝒕2
body and the force which causes the motion. 𝟐 𝟐
𝑣=final velocity, 𝑢=initial velocity, 𝑠=distance,
- Types of motion : 𝒗+𝒖
I. Translational motion – Motion of objects in a 𝑎=acceleration. Average velocity = ( ) .
𝟐
straight line. All points are moved in a parallel I. When a body moves from or at rest or
direction and with equal distance. e.g motion of a stationary, 𝒖 =0. II. When a body is brought to
vehicle in a road. rest or stop, 𝒗 =0. III. 𝒂 = −ve, when retarded
II. Rotational motion – Motion of a body about or decelerated. IV. If a body is moving with a
the center of its axis e.g blade of a fan, wheels of constant or uniform velocity in a straight line,
moving cars. the acceleration is zero (𝒂 =0).
- The centre of a coin as it rolls down an inclined - To convert speed or velocity from kilometer per
plane undergoes translational motion only while hour(Km/hr) to meter per second(m/s) –
other parts of the coin undergo rotational 𝒗(m/s) = 𝒗(km/hr) × .
𝟓
𝟏𝟖
motion. - Velocity - time graph :
- A moving car is undergoing both rotational (the velocity(𝒗)
wheel of the tyre) and translational motion (the b
whole vehicle).
III. Oscillatory or vibrational motion – To and a c
fro motion or a periodic motion of a body about a time(𝒕)
fixed point e.g simple harmonic motion, wave a.ascending line(moving upwards) represents
motion. constant acceleration, b.horizontal line
IV. Random motion – Haphazard or disordered represents constant or uniform velocity
movement of a body e.g motion of gass (acceleration is zero), c.descending line(moving
molecules, Brownian motion. downwards) represents constant deceleration.
- Position x, y – It is the location of a body in a - Acceleration of a velocity-time is given by the
line, surface or space. slope of the ascending line.
- Distance 𝒔 – The separation between two 𝒄𝒉𝒂𝒏𝒈𝒆 𝒊𝒏 𝒗𝒆𝒍𝒐𝒄𝒊𝒕𝒚 𝒗 −𝒗
points or separation between two different 𝒂= = 𝟐 𝟏.
𝒕𝒊𝒎𝒆 𝒊𝒏𝒕𝒆𝒓𝒗𝒂𝒍 𝒕𝟐 −𝒕𝟏
positions of a body in motion. - The slope of the horizontal line is zero i.e
- Displacement 𝒔 – The distance of a body acceleration is zero and it shows uniform
measured in a specified direction or distance motion(constant velocity).
between the initial and final position of a body in - The slope of the descending line is equal to the
motion. deceleration or retardation (negative
- Speed 𝒗 – The distance covered per unit time. acceleration).
- Velocity 𝒗 – The displacement of the body per - The total distance travelled in a velocity-time
unit time or speed measured in a specified graph is the area of the figure under the graph
direction. - Displacement(distance) - time graph :
- Acceleration 𝒂 – The rate of change of velocity Displacement/distance(𝒔)
with time. The decrease in velocity with time is b
called retardation or deceration (negative
acceleration). a c
time(𝒕)
● Rectilinear or Linear motion and The ascending or descending straight line a and
Motion under gravity c represents uniform motion i.e constant
- Linear motion – Motion of a body in a straight velocity, horizontal line b represents a
path in a constant direction. stationary body (body at rest).
- Uniform acceleration – Increase in velocity in - The slope of a displacement - time graph is
equal time interval (constant acceleration) or velocity while the slope of a distance-time graph
constant rate of change change with time. is speed.
- Uniform velocity – velcotiy of a body when - When a body is moving with a uniform
equal displacement is covered in equal time acceleration in a straight line, the distance
interval. It is referred to as uniform motion. covered by a body in the 𝑛th second or last one

17
Demystified Series Physics Demystified by Dr Timothy
𝒂
second of motion is given by – 𝒔 = 𝒖 + (2𝒕 −1), (displacement) - square time graph 2, graph of
𝟐
𝑠=distance, 𝑢=initial velocity, 𝑎=acceleration, a body falling under gravity i.e free fall is given
𝑡=nth time. below :
- Instantaneous velocity 𝒗 is the velocity of a 𝒉 or 𝒔 1. 𝒉 or 𝒔 2.
body at a particular instant of time or the
differential function of distance(x) and
substituting the values of time(t). 𝒕 𝒕2
- Intantaneous velocity of a moving vehicle is 1. The graph 1, is a parabola shaped graph.
measured by a speedometer. 2. The slope of graph 2, equal to half the
𝟏
- Instantaneous acceleration 𝒂 is the acceleration , 𝒂.
𝟐
acceleration of a body at a particular instant of - Graph 1 and 2 is also valid for a body that
time or differential function of velocity (v) and accelerates uniformly from rest.
substituting the value of time(t). - Vertical projection of bodies from a height :
- Motion of a body under gravity – If a body is I. When a body is projected vertically upward
thrown upward it rises to a particular height and from the top of a tower/cliff of height 𝐻 with an
then falls back to the earth. All bodies moving initial velocity 𝑢, to the ground or foot of the
under gravity in air, have the same acceleration tower (from a higher to a lower plane), at a time
i.e acceleration due to gravity, g=9.8ms-2 or 𝟏
𝑇, then – 𝑯 = 𝒈𝑻𝟐 − 𝒖𝑻, 𝐻=height of the
≈10ms-2. 𝟐
tower/cliff, 𝑢=initial velocity, 𝑇=total time of
- Free fall – It is the vertical motion of a body
flight. The landing or impact velocity 𝑣 of the
downwards under the influence of gravity only.
body at the base of the tower is given as –
- Two objects of different masses will fall to the
𝒗2= 𝒖2+2𝒈𝑯.
ground at the same time from same height
II. When a body is projected vertically upward
neglecting air resistance or viscosity.
from the ground to the top of a tower or clif of
- In the presence of air resistance or viscosity
height 𝐻 with an initial velocity 𝑢, i.e lower to a
the heavier mass experiences the greatest 𝟏
gravitational acceleration, hence the least air higher plane, at a time 𝑇, then – 𝑯 = 𝒖𝒕 − 𝒈𝑻𝟐 .
𝟐
resistance and lands on the ground first The landing or impact velocity 𝑣 of the body at
before the lower mass. the top of the tower or cliff is given as –
- Two bodies of diferrent masses will have the 𝒗2= 𝒖2−2𝒈𝑯.
same acceleration (g=10m/s2) at a height Alternatively, time taken to reach maximum
𝒖
above the ground. height 𝐻 is given by – 𝒕𝒎𝒂𝒙 = . Time taken to
𝒈
- Equation of motion under gravity :
return from maximum height to the ground is
1. 𝒗 = 𝒖 ± 𝒈𝒕 2. 𝒗2= 𝒖2±𝟐𝒈𝒉
𝟐(𝑯𝒐 +𝐇)
𝟏
3. 𝒉 = 𝒖𝒕 ± 𝒈𝒕2 . given by – 𝒕L = √ . The total time of flight
𝟐 𝒈
I. When a body falls from a height, 𝒖=0. II. At 𝒖 𝟐(𝑯𝒐 +𝐇)
maximum height, 𝒗=0. III. 𝒈=+ve when a body 𝑻 is given by – 𝑻 = 𝒕𝒎𝒂𝒙 + 𝒕L = +√ .
𝒈 𝒈
is falling or moving downwards and 𝒈=−ve 𝟏 𝒖𝟐
when a body is moving or thrown upwards. Maximum height, 𝑯𝒐 = 𝒈𝒕𝒎𝒂𝒙 𝟐 = .
𝟐 𝟐𝒈
IV. Time taken to reach the maximum height =
Time taken to fall from maximum height to the Examples :
ground = 𝒕 . V. Total time of flight=twice the 1. Which of the following types of motion is
time taken to reach maximum height=2𝒕. oscillatory? I.a diving board when used by a
- Velocity-time graph of a body under gravity diver II.the motion of the balance wheel of a wrist
: The velcocity of a body decreases when thrown watch III.the motion of turn-table of a record
upward and increases when returning to the player IV.the motion of the centre of ten kobo
ground. piece as it rolls down on an inclined plane V.the
𝒗 𝒗 motion of a needle of a d.c ammeter into which a
low a.c current is passed A.I,II and III B.I and II
C.I,II and IV D.II,III and IV E.III,IV and V.
𝒕 2𝒕 2. The motion of the prongs of a sounding tuning
−𝒗 𝒕 fork is A.Random B.Translational C.Rotational
a. b. D.Vibrational E.Vibrational and rotational.
Graph a and b are both correct but graph a is The prongs a sounding tuning fork vibrates when
most appropriate as the direction of motion is set in rotational motion.
considered(upward and downward motion) A diving board, balance wheel of a wrist watch or
but ignored in graph b. clock, moving skin of a talking drum, swinging
- Height (displacement) - time 1, height pendulum bob are examples of oscillatory or

18
Demystified Series Physics Demystified by Dr Timothy
vibratory motion i.e to and fro. Determine the magnitude of its acceleration
3. The type of motion which a cylindrical drum A.0ms-2 B.1ms-2 C.3ms-2 D.4ms-2.
rolling down an inclined plane undergoes are When a body is moving with a uniform/constant
A.circular and translational B.rotational and speed or velocity, the acceleration is zero as the
translational C.rotational and circular velocity or speed is not changing with time.
D.rectilinear and circular. 13. A body accelerates from rest at the rate of
4. A body can undergo the following tyoes of 3ms-2 for 8s. Calculate the distance covered by
motion except A.random B.rotational the body during acceleration A.38m B.46m
C.translational D.relative. C.96m D.82m .
Relative motion exist between two bodies and Initial velocity, 𝑢=0(as the body moves from
not a body only. Relative motion is not a type of rest), acceleration, 𝑎=3ms-2, 𝑡=8s,
motion. 1 1
𝑠 = 𝑢𝑡 + 𝑎𝑡 2 = 0×8+ ×3×82 = 1.5×64 = 96m.
2 2
5. A boy cycles continuously through a distance
14. A bus travelling at 15m/s accelerates
of 1.0km in 5minutes. Calculate his average
uniformly at 4m/s2. What is the distance covered
speed. A.6.67ms-1 B.3.33ms-1 C.2.00ms-1
in 10s? A.350m B.240m C.380m D.470m
D.4.00ms . -1
𝑢=15m/s, 𝑎=4m/s2, 𝑡=10s, 𝑠=?,
Distance, 𝑠=1.0km=1000m, time, 𝑡=5minutes= 1 1
5×60=300s, Average speed 𝑣=?, 𝑠 = 𝑢𝑡 + 𝑎𝑡 2=15×10+ ×4×102=150+2×100
2 2
𝑣= =
𝑠 1000
= 3.33ms-1. 𝑠 = 150+200 = 350m.
𝑡 300 15. A particle starts from rest and moves with a
6. The average speed of an object is determined constant acceleration of 0.5ms-2. Calculate the
by dividing the sum of the initial and final speeds
time taken by the particle to cover a distance of
by two, only when the acceleration is A.uniform 25m. A.2.5s B.7.1s C.10.0s D.50.0s D.100.0s
B.changing C.increasing D.decreasing. 𝑢=0(from rest), 𝑎=0.5ms-2, 𝑠=25m, 𝑡=?,
𝑣+𝑢
Average speed 𝑣 = and this holds only when 1 1 1
2 𝑠 = 𝑢𝑡 + 𝑎𝑡 2 = 0 × 𝑡 + 𝑎𝑡2 , 𝑠 = 𝑎𝑡 2 ,
2 2 2
the acceleration is constant or uniform. 2𝑠 2×25
7. A bird flies at 10ms-1 for 3s, 15ms-1 for 3s and 𝑡2 = = = 100 , 𝑡 = √100 = 10.0s.
𝑎 0.5
20ms-1 for 4s. Calculate the bird’s average speed 15. A body accelerates uniformly from rest at
A.4.5ms-1 B.15.0ms-1 C.15.5ms-1 D.51.7ms-1. 2m/s2. Calculate its velocity after travelling 9m.
Distance 𝑠 = velocity 𝑣 × time 𝑡 , A.3ms-1 B.6ms-1 C.9ms-1 D.18ms-1.
Total distance = 10×3+15×3+20×4 , 𝑢=0(from rest), 𝑎=2ms-2, 𝑠=9m, final velocity,
Total distance = 30+45+80 = 155m , 𝑣=? 𝑣 2= 𝑢2+2𝑎𝑠 = 02+2×2×9=36,
Average speed 𝑣 =
𝑇𝑜𝑡𝑎𝑙 𝑑𝑖𝑠𝑡𝑎𝑛𝑐𝑒
=
155
=
155
, 𝑣 = √36 = 6ms-1.
𝑇𝑜𝑡𝑎𝑙 𝑡𝑖𝑚𝑒 𝑡𝑎𝑘𝑒𝑛 3+3+4 10
16. A boy moving with a velocity of 3m/s is
Average speed 𝑣 = 15.5ms . -1
brought to rest by a constant force, after
8. A car starts travels with a constant velocity of
travelling 15m. Calculate the retardation.
45km/hr for 20s. What distance does it cover in
A.0.15ms-2 B.0.45ms-2 C.0.30ms-2 D.0.75ms-2.
this time? A.250m B.500m C.750m D.650m
5 𝑢=3ms-1, 𝑣=0(as the body is brought to rest),
constant velocity=45km/hr=45× =12.5ms-1, 𝑠=15m, 𝑎 = −ve(retardation), 𝑣 2 = 𝑢2−2𝑎𝑠,
18
𝑠
𝑡=20s, 𝑣 = , 𝑠 = 𝑣𝑡 = 12.5×20 = 250m. 9
02 = 32−2𝑎 ×15, 30𝑎 = 9, 𝑎 = = 0.3ms-2.
𝑡 30
9. A car accelerates uniformly from rest at 5ms-2. 17. A body which is uniformly retarded comes to
Determine its speed after 10s A.50.0ms-1 rest in 10s after travelling a distance of 20m.
B.25.0ms-1 C.2.0ms-1 D.0.5ms-1. Calculate its initial velocity A.0.5ms-1 B.2.0ms-1
𝑢=0(from rest), 𝑎=5ms-2, 𝑡=10s, 𝑣=?, C.4.0ms-1 D.20.0ms-1
𝑣 = 𝑢 + 𝑎𝑡 = 0 + 5×10 = 50ms-1. 𝑠=20m, 𝑡=10s, 𝑣=0(comes to rest), 𝑢=?,
10. A body starts from rest and accelerates 𝑣+𝑢 0+𝑢
𝑠 = ( ) 𝑡 , 20 = ( )10 , 2×20 = 10𝑢,
uniformly at 5ms-2 until it attains a velocity of 2
40
2
25ms-1. Calculate the time taken to attain this 10𝑢 = 40, 𝑢 = = 4.0ms-1.
10
velocity A.2.5s B.5.0s C.10.0s D.125.0s. 18. Two particles X and Y starting from rest cover
𝑢=0(from rest), 𝑎=5ms-2, 𝑣=25ms-1, 𝑡=?, the same distance. The acceleration of X is twice
25
𝑣 = 𝑢 + 𝑎𝑡 , 25 = 0 + 5𝑡 = 5𝑡 , 𝑡 = = 5.0s. that of Y. The ratio of the time taken by X to that
5
11. A train has an initial velocity of 44ms-1 and an taken by Y is A.1/2 B.2 C.1/√2 D.√2 E.4
acceleration of −4ms-2. Its velocity after 10sec is 𝑢X=𝑢Y=0(starts from rest), 𝑠X=𝑠Y(same distance),
1
A.2ms-1 B.4ms-1 C.8ms-1 D.12ms-1 E.16ms-1. 𝑎X=2𝑎Y, 𝑡X:𝑡Y=?, 𝑠 = 𝑢𝑡 + 𝑎𝑡 2 ,
2
𝑢=44ms-1, 𝑎=−4ms-1, 𝑡=10s, 𝑣=?, 1 1
For X : 𝑠X = 0× 𝑡X + 𝑎X𝑡X2 = 𝑎 X𝑡X2 ,
𝑣 = 𝑢 + 𝑎𝑡 = 44 + (−4 ×10) = 44 – 40 = 4ms-1. 2
1
2
1
12. An object of mass 2kg moves with a uniform For Y : 𝑠Y = 0× 𝑡Y + 𝑎Y𝑡Y2 = 𝑎 Y𝑡Y2 ,
2 2
speed of 10ms-1 for 5s along a straight path. 𝑠X = 𝑠 Y ,
1 1
𝑎 X𝑡X2 = 𝑎 Y𝑡Y2 , 𝑎X𝑡X2 = 𝑎Y𝑡Y2 ,
2 2

19
Demystified Series Physics Demystified by Dr Timothy
𝑡𝑋 2 𝑎𝑌 𝑎𝑌 1 𝑡𝑋 1 1 A. 𝑣 B. 𝑣
= = = , 𝑡X:𝑡Y = =√ = or 1: √2.
𝑡𝑌 2 𝑎𝑋 2𝑎𝑌 2 𝑡𝑌 2 √2
19. A body starts from rest and moves with
uniform acceleration of 6ms-2. What distance 𝑡 𝑡
does it cover in the third second? A.15m B.18m C. 𝒗 D. 𝑣
C.27m D.30m
𝑢=0(from rest), 𝑎=6ms, 𝑡=3s(third second), 𝑠=?,
𝑎 6 𝒕 𝑡
𝑠 = 𝑢 + (2𝑡 −1) = 0+ (2×3−1) = 3(6−1) ,
2 2 The graph in option C represents the velocity-
𝑠 = 3(5)=15m. time graph of a body decelerating or retarding
20. If a car starts from rest and moves with a uniformly in a straight line. The slope of the
uniform acceleration of 10ms-1 for ten seconds, graph represents the deceleration or retardation.
the distance it covers in the last one second of its 25. Which of the velocity-time graph below
motion is A.95m B.100m C.500m D.905m correctly describes the motion of a body pulled
E.1000m. by a force that is equal to the frictional force
𝑢=0(from rest), 𝑎-10ms-2, 𝑡=10s, distance acting on it?
covered in the last one second , 𝑠=?, A. 𝑣/ms-1 B. 𝑣/ms-1
𝑎 10
𝑠 = 𝑢 + (2𝑡 −1) = 0+ (2×10 − 1) = 5(20−1)
2 2
𝑠 = 5(19) = 95m.
21. The area under a velocity-time graph 0 𝑡/s 0 𝑡/s
represents A. speed B. acceleration C. moment C. 𝑣/ms-1 D. 𝒗/ms-1
D. distance.
The area under a velocity-time graph represents
the total distance covered by the body during 0 𝑡/s 0 𝒕/s.
motion. When the force pulling the body is equal to the
22. vel.(m/s) frictional force(opposing force) acting on it, the
the resultant force acting on the body will be
60
equal to zero and the acceleration of the body will
also be zero. Thus, the velocity becomes uniform
0 20 25 time(s) or constant as acceleration is zero.
From the velocity-time graph shown above, 26. The slope of a straight line displacement-time
which of the following quantities cannot be graph indicates A. distance travelled B. uniform
determined? A.Deceleration B.Initial velocity velocity C. uniform acceleration D. acceleration
C.Total distance travelled D.Initial acceleration. at an instant E. uniform speed.
The figure in the first phase of motion as seen in The slope of a horizontal line displacement-time
the graph is uniform(constant) velocity, the graph is zero(velocity is zero and the body is
second phase of motion shows deceleration, the stationary).
initial velocity is 60m/s i.e the body does not 27. Velocity
starts from rest. No acceleration is seen in the
graph above, hence it cannot be determined.
23. 𝑣(m/s)
F G Time
The diagram above shows the velocity-time
E graph of a truck which accelerates uniformly
from rest at 2ms-2 for 10s,maintains a steady
0 H 𝑡(s) velocity for 30s and is then brought to rest in 5s
The diagram above shows a 𝑉 − 𝑇 graph. The under a uniform retardation.The total distance
statement that is true about this motion is that travelled is A.1500m B.1200m C.750m D.900m
the car A.has a constant speed between points E 𝑣(m/s)
and F B.has no acceleration between points F
and G C.decelerates between points F and G 20
D.decelerates between points F and G.
The car undergoes constant acceleration 10s 30s 5s
between E and F, uniform or constant velocity i.e 0 10 40 45 𝑡(s)
no acceleration, between points F and G and The truck accelerated from rest at 2ms-2 for
constant deceleration between points G and H. 10s.The final maximum velocity is calculated by:
24. The velocity-time graph of a body moving in 𝑣 = 𝑢 + 𝑎𝑡 =0+2×10=20ms-1
Total distance𝑠 =Area of the trampezium =
a straight line and decelerating uniformly to rest 1 1
is represented by (sum of parallel sides)height= (𝑎 + 𝑏)ℎ.
2 2

20
Demystified Series Physics Demystified by Dr Timothy
ℎ in the graph represents maximum velocity. 31. 𝑣(m/s)
1
𝑠 = (30+45)20= 75×10=750m. 15
2
28. 𝑣(m/s) 10
P Q
5
10 20 30 40 50 𝑡(s)
The diagram above shows a velocity-time graph
O 6 11 21 𝑡(s)
representing the motion of a car. Find the total
The diagram above represents the velocity-time
distance covered during the acceleration and
graph of a body in motion. The total distance
retardation periods of the motion. A.75m
travelled by the body is 195m. Calculate the
B.150m C.300m D.375m.
acceleration of the body OP of the graph.
Distance covered during the acceleration period
A.2.5ms-2 B.5.0ms-2 C.3.5ms-2 D.7.5ms-2. 1 1
1 = ×(10−0)×(10−0)= ×10×10=5×10=50m
𝑠 = (𝑎 + 𝑏)ℎ, 𝑠=195m, 𝑎=21s, 𝑏=11−6=5s, 2 2
2
1 Distance covered during the retardation period
ℎ=𝑣=final maximum velocity, 195 = (21+5)𝑣 , 1 1
390
2 = ×(45−40)×(10−0)= ×5×10=2.5×10=25m.
2 2
26𝑣 = 2×195 = 390 , 𝑣 = = 15ms-1. Distance covered in both acceleration and
26
Acceleration of the body represented by the retardation period = 50+25 = 75m.
15−0
section OP = slope of OP = = 2.5ms-2. 32. 𝑣/ms-1
6−0
29. A particle accelerates uniformly from rest at
10
6.0ms-2 for 8s and decelerates uniformly to rest
in the next 5s. Determine the magnitude of the 4
deceleration A.9.6ms-2 B.24.0ms-2 C.30.0ms-2
D.48.0ms-2. 0 4 12 𝑡/s
During the acceleration phase : 𝑢=0(from rest), The diagram above illustrates the velocity-time
𝑎=6ms-2, 𝑡=8s, 𝑣=?, 𝑣 = 𝑢 + 𝑎𝑡 = 0 + 6 × 8 , graph of a body. Calculate the distance covered
𝑣 = 48ms-1. by the body during the motion. A.15m B.84m
During the deceleration phase : final velocity of C.72m D.48m.
the acceleration phase=initial velocity of the The distance covered by the body between 0 – 4s
decelration phase. 𝑢=48ms-1, 𝑡=5s, 𝑣=0(comes to = 4×4 =16m. The distance covered by the body
1 1
rest), 𝑎=−ve(deceleration), 𝑣 = 𝑢 − 𝑎𝑡 , between 4 – 12s= ×(4+10)×(12−4)= ×14×8
2 2
48
0 = 48 − 5𝑎 , 5𝑎 = 48 , 𝑎 = = 9.6ms-2. = 7×8 = 56m. Total distance = 16+56 = 72m.
5
Alternatively, using graphical method : 33. Two points on a velocity time graph have co-
𝑣/ms-1 𝑣 = 𝑢 + 𝑎𝑡 = 0 + 6×8, ordinates (5s,10ms-1) and (20s,20ms-1).Calculate
𝑣 = 48ms-1. the acceleration between the two points
48
A.0.67ms-2 B.0.83ms-2 C.1.50ms-2 D.2.00ms-2
𝑣2 =20ms-1, 𝑣1 =10ms-1, 𝑡2 =20s, 𝑡1 =5s,
8 5 𝑎=
𝑐ℎ𝑎𝑛𝑔𝑒 𝑖𝑛 𝑣𝑒𝑙𝑜𝑐𝑖𝑡𝑦 𝑣2 −𝑣1
= =
20−10 10
= = 0.67ms-2.
0 8 13 𝑡/s 𝑡𝑖𝑚𝑒 𝑖𝑛𝑡𝑒𝑟𝑣𝑎𝑙 𝑡2 −𝑡1 20−5 15
The deceleration is equal to the slope of the 34. A body moving with uniform acceleration has
∆𝑣 𝑣 −𝑣 0−48 −48
descending line. 𝑎 = = 2 1 = = , two points (5,15) and (20,60) on the velocity-
∆𝑡 𝑡2 −𝑡1 13−8 5
𝑎 = − 9.6ms-2(negative acceleration). time graph of its motion. Calculate the
30. 𝑣(m/s) acceleration between the two points A. 0.25ms-2
B. 3.00ms-2 C. 4.00ms-2 D. 9.00ms-2 E. 16.00ms-2.
20 𝑣2 =60ms-1, 𝑣1 =15ms-1, 𝑡2 =20s, 𝑡1 =5s,
𝑐ℎ𝑎𝑛𝑔𝑒 𝑖𝑛 𝑣𝑒𝑙𝑜𝑐𝑖𝑡𝑦 𝑣2 −𝑣1 60−15 45
15 𝑎= = = = = 3.00ms-2.
𝑡𝑖𝑚𝑒 𝑖𝑛𝑡𝑒𝑟𝑣𝑎𝑙 𝑡2 −𝑡1 20−5 15
10 A B C
35. The distance 𝑥m travelled by a particle in
5
time, t seconds is described by the equation
5 10 15 𝑡(s)
𝑥 =10+12𝑡2. Find the average speed of the
The diagram below illustrates the velocity-time
particle between time interval, 𝑡=2s and t=5s.
graph of the motion of a graph of the motion of a
A.60ms-1 B.72ms-1 C.84ms-1 D.108ms-1.
body. Calculate the total distance covered by the
body. A.125m B.250m C.225m D.500m. Differentiate the equation to get the derivative of
𝑑𝑥
Total distance covered by the body 𝑠 = Area of the speed. Speed, 𝑣 = = 0+2×12𝑡 2−1 =24𝑡,
𝑑𝑡
the shape A + Area of the shape B + Area of the Speed at 𝑡=2s, 𝑣 =24𝑡 = 24(2)=48ms-1,
shape C . Speed at 𝑡=5s, 𝑣 =24𝑡 = 24(5)=120ms-1
1 1 48+120 168
𝑠 = (20+10)5 + 20×(10−5) + ×(15−10)×20 Average speed = = = 84ms-1.
2 2 2 2
𝑠 = [30×2.5]+[20×5]+[5×10] = 75+100+50 Alternatively, first substitute the value of time to
𝑠 =225m. get the distance at that instant of time,

21
Demystified Series Physics Demystified by Dr Timothy
Distance at 𝑡=2s, 𝑥 =10+12(22)=10+48 decreases as it moves upwards and increases
𝑥 =58m, Distance at 𝑡=5s, 𝑥 =10+12(52) when it is returning to the ground. Graph B and C
𝑥 =10+300=310m. are correct but C is most correct as it indicates
𝑥 −𝑥 310−58 252
Average speed 𝑣 = 2 1 = = , the direction. Graphj A shows the velocity-time
𝑡2 −𝑡1 5−2 3
graph of a body in a lift or elevator.
Average speed=84ms-1.
42. Which of the following sketches represents
36. The velocity 𝑣 of a particle in time 𝑡 is given
the velocity-time graph of the motion of a stone
by the equation 𝑣 =10+2𝑡2. Find the
projected vertically upwards and allowed to
instantaneous acceleration after 5seconds.
return to the point of projection?
A.60ms-2 B.20ms-2C.15ms-2 D.10ms-2
A. 𝑣 B. 𝑣
Differentiathe the equation to get the derivative
of the acceleration,
𝑑𝑣
Acceleration, 𝑎 = = 0+2×2𝑡 2−1 = 4𝑡, 𝑡 𝑡
𝑑𝑡
Acceleration at 𝑡=5s, 𝑎 = 4𝑡 =4×5=20ms-2. C. 𝒗 D. 𝑣
37. A body starts from rest and moves with
constant acceleration. Which of the following
quantities varies/vary linearly with the square of 𝒕 𝑡 .
the time. I.velocity II.displacement Graph C is the correct representation of a body
III.momentum A.I B.II C.III D.I and II E.II and III. vertically and returning to the ground .
1 1 43. Two balls X and Y weighing 5g and 50kg
𝑠=displacement , 𝑠 = 𝑢𝑡 + 𝑎𝑡2 = 0× 𝑡 + 𝑎𝑡2 ,
1 𝑠 1
2
1
2 respectively were thrown up vertically at the
𝑠 = 𝑎𝑡 2, 2 = 𝑎, 𝑠 ∝ 𝑡2, slope= 𝑎 (half the same time with a velocity of 100ms-1. How will
2 𝑡 2 2
acceleration). their positions be one second later A.X will be
38. The distance travelled by a particle starting 500m ahead of Y B.X and Y will both be 500m
from rest is plotted against the square of the time from the point of throw C.X and Y will be 500m
elapsed from the commencement of motion. The from each other D.Y will be 500m ahead of X .
resulting graph is linear. The slope of this graph Initial velocity of both bodies=100ms-1,
is a measure of A.initial displacement B.initial 𝑢X=𝑢Y=100ms-1, 𝑡X=𝑡Y=1s, ℎX=ℎY=ℎ=?
1
velocity C.half the initial velocity D.acceleration ℎ = 𝑢𝑡 − 𝑔𝑡2(upward motion), 𝑔=10ms-2
2
E.half the acceleration. 1
ℎ = 100×1− ×10×12 = 100−5 = 95m , both X
39. A body dropped from a certain height above 2
the ground level, falls with a uniform A.speed and Y will be 95m from the point of throw after
B.velocity C.acceleration D.retardation. 1s. None of the options are correct.
All bodies under free fall, falls with a uniform 44. A war-plane on a level flight releases a bomb
acceleration of 9.8ms-2 ≈ 10ms-2. X.Twenty seconds later, it releases a second bom
40. Two objects , one having three times the mass Y. The bomb Y will reach the ground A.at the
of the other, are dropped at the same time from a same time with X B.10s after X C.20s after X D.40s
tall building.When they are above the ground, the after X
two objects will have the same A.kinetic energy The height(altitude) of the war-plane is the same
B.potential energy C.momentum D.acceleration. for both bomb release, hence both bombs will
Both bodies under free fall, will have the same spend the same time in air before reaching the
acceleration(acceleration due to gravity). Their target on the ground.Since, Y is released 20s after
momentum, potential energy and kinetic energy X, it will also reach the ground 20s after X reaches
are different as they all depend on the mass of the the ground.
bodies. 45. A body falls from rest to the ground in
41. A stone thrown vertically upwards returns to 0.5s.Calculate the height from which it
the ground. Which of the following figures falls?(g=10ms-2) A.0.125m B.0.5m C.1.0m
represents the velocity-time graph? D.1.25m E.5.0m
A. vel B. vel 𝑡=0.5s, 𝑢=0(falling from rest), 𝑔=10ms-2, ℎ=?,
1
ℎ = 𝑢𝑡 + 𝑔𝑡2(downward motion),
2
1 1 1
ℎ =0× 𝑡 + 𝑔𝑡2= 𝑔𝑡2 = ×10×0.52 = 5×0.25 ,
2 2 2
0 𝑡 2𝑡 0 𝑡 2𝑡 ℎ =1.25m.
C. vel D. vel 46. A pal fruit dropped to the ground from the
top of a tree 45m tall. How long does it take to
reach the ground (g=10ms-2) A.9s B.4.5s C.6s
D.7.5s E.3s.
0 𝒕 𝟐𝒕 0 𝑡 2𝑡 ℎ=45m, 𝑢=0(falling from a height), 𝑔=10ms-2, 𝑡=?
1
ℎ = 𝑢𝑡 + 𝑔𝑡2(downward motion),
The velocity of a body thrown vertically upwards 2

22
Demystified Series Physics Demystified by Dr Timothy
1 45
45 = 0× 𝑡 + ×10𝑡2 , 45 = 5𝑡2 , 𝑡2 = =9, Alternatively : 𝑇 = 𝑡𝑚𝑎𝑥 + 𝑡L =
𝑢
+√
2(𝐻𝑜 +H)
.
2 5
𝑔 𝑔
𝑡 = √9 = 3s.
𝑢2 102 100 10 2(𝐻𝑜 +5)
47. A ball is thrown vertically upwards from the 𝐻= = = = 5m. 4 = +√ ,
2𝑔 2×10 20 10 10
ground with an initial velocity of 20ms-1. The
maximum height reached by the ball is A.7.5m 4 = 1 +√
(𝐻𝑜 +5)
, 4−1 = √
(𝐻𝑜 +5)
, 3=√
(𝐻𝑜+5)
,
5 5 5
B.10.0m C.20.0m D.22.5m. (𝐻𝑜 +5)
𝑢=20ms-2, 𝑣=0(at maximum height), 𝑔=10ms-2, 32 = , 9×5 = 𝐻𝑜 + 5 , 𝐻𝑜 = 45−5 = 40m.
5
ℎ=max.height, 𝑣 2= 𝑢2−2𝑔ℎ (upward motion), 53. A ball was projected vertically from sea level
𝑢2 202
0= 𝑢2−2𝑔ℎ, ℎ = = =
400
= 20m. into the air.It took a total of 5s to reach a 40m
2𝑔 2×10 20
height cliff on its way back. Find the velocity at
48. A ball is dropped from a height of 45m above
the top of the cliff A.13ms-1 B.15ms-1C.17ms-1
the ground. Calculate the velocity of the ball just
D.19ms-1.
before it strikes the ground (Neglect air
𝐻=40m, 𝑇=5s, g=10ms-2, 𝑢=?, 𝑣=?,
resistance and take g as 10ms-2). A.21.2ms-1
The ball is thrown from the ground i.e sea level to
B.30.0ms-1 C.300.0ms-1 D.450.0ms-1 E.900.0ms-1.
the top of a cliff i.e lower to a higher plane, the
ℎ=45m, 𝑢=0(falling from a height), g=10ms-2, 𝑣=? 1
𝑣 2 = 𝑢2+2𝑔ℎ (downward motion), height of the cliff is given by – 𝐻𝑜 = 𝑢𝑡 − 𝑔𝑇 2 ,
2
1
𝑣 2 = 02+2 ×10×45 = 900 , 𝑣 = √900 = 30ms-2. 40= 𝑢 ×5− ×10×52 , 40=5𝑢 −125,
2
49. A body falls freely under gravity (g=9.8ms-2) 5𝑢 = 40+125 = 165, 𝑢 = = 33ms-1.
165
from a height of 40m ontop of a platform 0.8m 5
above the ground.Its velocity on reaching the The velocity at the top of the cliff 𝑣 is the landing
platform is A.784ms-1 B.80ms-1 C.78.4ms-1 or impact velocity – 𝑣 2 = 𝑢2−2𝑔𝐻𝑜
D.39.2ms-1 E.27.7ms-1. 𝑣 2 = 332−2×10×40 = 1089−800,
𝑢=0(falling from a height), g=9.8ms-2, height of 𝑣 2 = 289, 𝑣 = √289 = 17ms-1.
fall ℎ=40−0.8=39.2m, 𝑣=?, 𝑣 2 = 𝑢2+2𝑔ℎ , 54. A body is projected vertically upwards with
𝑣 2 = 02+2𝑔ℎ = 2𝑔ℎ = 2×9.8×39.2, a speed of 10ms-1 from a point 2m above the
𝑣 2 = 768.32, 𝑣 = √768.32 = 27.7ms-1. ground. Calculate the total time taken for the
50. An object falls freely from a height of 25m body to reach the ground (g=10ms-2) A.1.00s
onto the roof of a building 5m high. Calculate the B.2.00s C.2.18s D.3.00s.
velocity with which the object strikes the roof 𝑢=10ms-1, 𝐻𝑜 =2m, g=10ms-2, 𝑇=?,
𝑢2 102 100
(g=10ms-2) A.17.3ms-2 B.20.0ms-1 C.24,5ms-1 𝐻= = = = 5m.
2𝑔 2×10 20
D.125.0ms-1 E.200.0ms-1.
𝑢 2(𝐻𝑜 +H) 10 2(2+5)
𝑢=0(falling from a height), g=10ms-2, height of 𝑇 = 𝑡𝑚𝑎𝑥 + 𝑡L = +√ = +√ ,
𝑔 𝑔 10 10
fall ℎ=25−5=20m, 𝑣=?, 𝑣 2 = 𝑢2+2𝑔ℎ ,
7
𝑣 2 = 02+2𝑔ℎ = 2𝑔ℎ = 2×10×20, 𝑇 = 1 + √ = 1 +√1.4 = 1+1.18 = 2.18s.
5
𝑣 2 = 400, 𝑣 = √400 = 20.0ms-1. 1
51. A body is projected upwards with a velocity Alternatively : 𝐻𝑜 = 𝑔𝑇 2 − 𝑢𝑇 .
2
of 200ms-1. How long does it take to return to the
point of projection? (g=10ms-2) A.2s B.4s C.10s ● Projectile motion
D.40s. - A projectile is a body which when released
𝑢=200ms-2, g=10ms-2, time taken to return to the travels along a parabolic path called trajectory
point of projection is the total time of flight=𝑇. e.g a bullet fired from a gun, a ball rebouncing
Considering the upward motion: 𝑣 = 𝑢 − 𝑔𝑡, at from a wall, an athlete doing a high jump e.t.c
𝑢
maximum height, 𝑣=0, hence 𝑢 = 𝑔𝑡, 𝑡 = , - A projectile undergoes vertical and
𝑔
𝑡=time taken to reach maximum height. horizontal motion simultaneously.
2𝑢 2×200 1. Motion of an object projected horizontally
Total time of flight 𝑇 =2𝑡 = = = 40s. from a height – Two independent motions i.e
𝑔 10
52. A ball is thrown vertically upwards from the vertical motion(motion of free fall) and
top of a building with velocity of 10ms-1. If it takes horizontal motion are combined here. The time
4s for the ball to reach the ground level, the of fight is same for both motion.
height of the building is A.5m B.40m C.45m 𝒖
D.50m. 𝒗𝒙
𝑢=10ms-1, 𝑇=4s, g=10ms-2, 𝐻𝑜 =? 𝑯𝒐 𝒗
The ball is thrown from the top of a building to 𝒗𝒚
the ground i.e higher to a lower plane, the height 𝒗𝒙
1
of the tower is given by – 𝐻𝑜 = 𝑔𝑇 2 − 𝑢𝑇 𝑹 𝒗
2
1 𝒗𝒚
𝐻𝑜 = × 10×42 − 10×4 = 80−40 = 40m.
2 I.The horizontal component of the velocity is

23
Demystified Series Physics Demystified by Dr Timothy
constant throughout the motion and its equal to - Maximum height is attained when the angle
the initial velocity of projection 𝒗𝒙 = 𝒖. Hence, of projection 𝜽 = 90°.
the horizontal motion is a uniform motion. - The maximum height attained by a body
The velocity just before reaching the ground depends on : i.Angle of projection θ. ii.Initial
is equal to its initial velocity 𝒖. velocity/speed projection 𝑢. iii.Aire resistance.
II.The vertical component of the velocity iv.Acceleration due to gravity.
increases from 𝒖𝒚 =𝟎 at height 𝐻, to 𝒗𝒚 =𝒈𝒕 on VII.Range 𝑹 is the horizontal distance from the
hitting the ground. point of projection to the point where the
III.Horizontal acceleration is zero, 𝑎𝑥 =0. Vertical projectile hits the projection plane.
acceleration is acceleration due to gravity, 𝑎𝑦 =𝑔. 𝒖𝟐 𝒔𝒊𝒏𝟐𝜽
𝑹 = 𝒖𝒄𝒐𝒔𝜽 𝑻 = .
𝟏 𝟐𝒈
IV.Height of projection , 𝑯𝒐 = 𝒈𝑻2. Range or - Maximum range is attained when the angle
𝟐
distance from the foot of platform/tower to the of projection 𝜽 = 45° or 2𝜽 = 90°..
point of landing , 𝑹 = 𝒖𝑻. 𝑇=time of flight. - When two bodies A and B are projected with the
- The range and height of progection are related same initial velocity, if the angles are
𝑹𝟐 𝒈 complementary i.e sum of their angle is equal
by – 𝑯𝒐 = 𝟐 .
𝟐𝒖 to 90° [25°& 65°,30°& 60°,75°& 15°] then, their
V.Instantaneous velocity 𝒗 after time 𝑡is given
range is equal (𝑹A = 𝑹B).
by – 𝒗 = √𝒗𝒚 𝟐 + 𝒗𝒙 𝟐 = √𝒖𝟐 + (𝒈𝑻)𝟐 . - When the sum of their angles is greater or less
than 90°, the body with the smaller angle would
have the greatest range.
2. Motion of an object projected from the - The relationship between maximum height and
ground at an angle to the horizontal/vertical 𝑯 𝒕𝒂𝒏𝜽
– The body is projected from the ground level at range is – = .
𝑹 𝟒
an angle θ, takes a parabolic path and returns
back to its plane of projection. 3. Motion of an object projected vertically
𝒚 𝑻 upward from a tower or cliff of height 𝑯𝒐 at an
angle θ – The body is projected from a tower or
𝒗𝒚 𝒗 𝑯 cliff of height 𝐻𝑜 at angle θ to the horizontal , the
body initial moves upward to a maximum height
𝑶 θ 𝒙 𝐻 from the projection plane and then returns or
𝒗𝒙 𝑷 falls to the ground (not the projection) plane
𝑹 after a period of time 𝑇 i.e the total time of flight.
I.Vertical acceleration is acceleration due to 𝒖𝒄𝒐𝒔θ
gravity, 𝑎𝑦 = ±𝑔. i.e positive for downward 𝒖 𝑯
motion(𝑇𝑃) and negative for upward motion θ
(𝑂𝑇). Horizontal acceleration is zero, 𝑎𝑥 =0. 𝒖𝒔𝒊𝒏θ
II.The horizontal component of the velocity is 𝑯𝒕 𝑯𝒐
constant throughout motion. 𝒗𝒙 = 𝒖𝒄𝒐𝒔θ. The
vertical component of the velocity changes with
time. 𝒗𝒚 = 𝒖𝒔𝒊𝒏θ−𝒈𝒕.
𝑹
III.Instantaneous velocity at time 𝑡 is given by –
The body is projected from the top of a tower or
𝒗 = √𝒗𝒚 𝟐 + 𝒗𝒙 𝟐 . The direction of the cliff of height 𝑯𝒐 , the body first moved upward
to a height 𝑯 before returning to the ground.
instantaneous velocity at that point is – 𝐭𝐚𝐧 𝜷 =
𝒗𝒚 I.Time taken to reach maximum height 𝐻 is given
. 𝒖 𝒔𝒊𝒏𝜽
𝒗𝒙 by – 𝒕𝒎𝒂𝒙 = . Time taken to return from
𝒈
At maximum height, 𝒗𝒚 =0 and the velocity is
maximum height to the ground is given by –
equal horizontal component of the velocity,
𝟐(𝑯𝒐 +𝐇)
𝒗𝒙 = 𝒗𝒄𝒐𝒔θ . 𝒕L = √
𝒈
.
IV. Time of flight 𝑻 is the total time required for
II.The maximum height 𝐻 above the projection
a projectile to return to its plane of projection. 𝒖𝟐 𝒔𝒊𝒏𝟐 𝜽
𝑻=
𝟐𝒖 𝒔𝒊𝒏𝜽
. plane is given by – 𝑯 = .
𝟐𝒈
𝒈
V.Time taken to reach maximum height 𝒕𝒎𝒂𝒙 . The maximum height above the ground or total
𝒖 𝒔𝒊𝒏𝜽 height of fall of the body 𝑯𝒕 = 𝑯𝒐 + 𝑯.
𝒕𝒎𝒂𝒙 = . 𝑻 = 2𝒕𝒎𝒂𝒙 . III. The total time of flight 𝑻 is given by –
𝒈
VI.Maximum height 𝑯 is the highest vertical 𝟏
𝑻 = 𝒕𝒎𝒂𝒙 + 𝒕L or 𝑯𝒐 = 𝒈𝑻𝟐 − 𝒖𝒔𝒊𝒏θ𝑻.
distance attained from the highest vertical point 𝟐

𝒖𝟐 𝒔𝒊𝒏𝟐 𝜽
IV.The range 𝑹 is given by – 𝑹 = 𝒖𝒄𝒐𝒔𝜽 𝑻.
to the plane of projection. 𝑯 = .
𝟐𝒈

24
Demystified Series Physics Demystified by Dr Timothy
Examples : 1
II. Height of projection, 𝐻𝑜 = 𝑔𝑡 2= ×10×5.712,
1
2 2
1. A is projected horizontally from the top of a
𝐻𝑜 =163.0m
tower. Neglecting air resistance, which of the
III. Instantaneous velocity after 3s, 𝑣=√𝑣𝑦 + 𝑣𝑥 ,
following statements about its motion is/are
correct? I.Vertical acceleration on the ball 𝑣 = √𝑢2 + (𝑔𝑡)2 = √802 + (10 × 3)2 ,
decreases as it falls II.The vertical force on the 𝑣 = √6400 + 900 = √7300 =85.4ms-1.
ball remains constant III.The vertical speed of 7. An object is projected with a velocity of 100ms-
ball increaseas as it falls A.I only B.II only C.I and 1 at an angle of 60° to the vertical. Calculate the

II only D.II and III only. taken by the object to reach the highest
In horizontal projection from a height : the point(g=10ms-2) A.5.0s B.8.7s C.10.0s D.17.3s
horizontal speed is constany throughout the E.20.0s
motion while the vertical speed increases as the 𝑢=100ms-1, 𝛼=angle of inclination to the
body falls. The vertical acceleration(acceleration vertical=60°, 𝜃=angle of inclination to the
due to gravity) and the vertical force(weight of horizontal=90−𝛼 =90−60=30°,𝑡𝑚𝑎𝑥 =?,
the body) remains constant. 𝑡𝑚𝑎𝑥 =
𝑢 𝑠𝑖𝑛𝜃
=
100𝑠𝑖𝑛30°
= 10 × 0.5 = 5s.
2. R 𝑔 10

X Y 8. An object is projected with a velocity of 50ms-


1 from the ground level at an angle 𝛼 to the

vertical. If the total time of flight of the projectile


P Q is 5s, what is the value of 𝛼? A.0° B.30° C.45°
An object R leaves a platform XY with a D.60° D.90° (g=10ms-2).
horizontal velocity 7ms-1 and lands at Q. If it takes 𝑢=50ms-1, g=10ms-2, 𝛼=angle of inclination to the
the same object 0.3s to fal freely from 𝑌 to 𝑃, vertical, θ=angle of inclination to the
calculate the distance 𝑃𝑄. (g=10ms-2) A.7.30m horizontal=90−𝛼, 𝑇=5s,
2𝑢 𝑠𝑖𝑛𝜃 2𝑢 sin (90−𝛼) 2×50𝑠𝑖𝑛(90−𝛼)
B.2.55m C.2.1m D.1.65m E.0.45m 𝑇= , 𝑇= , 5= ,
𝑔 𝑔 10
𝑢=7ms-1, 𝑇=0.3s, 𝑃𝑄=𝑅=range, 𝑅 = 𝑢𝑇, 50=100𝑠𝑖𝑛(90 − 𝛼), 𝑠𝑖𝑛(90 − 𝛼)= = 0.5,
50
𝑅 = 7×0.3= 2.1m. 100
3. A ball is projected horizontally from the top of (90 − 𝛼)= 𝑠𝑖𝑛 0.5=30°, 𝛼 =90−30=60°.
−1

a hill with a velocity of 20ms-1. If it reaches the OR 𝑠𝑖𝑛(90 − 𝛼) = 𝑐𝑜𝑠𝛼, 50 = 100𝑐𝑜𝑠𝛼 ,


50
ground 4 seconds later, what is the height of the 𝑐𝑜𝑠𝛼 = = 0.5, 𝛼 = 𝑐𝑜𝑠 −1 0.5 =60°.
100
hill? (g=10ms-2) A.20m B.80m C.40m D.60m 9. A ball is thrown with an initial velocity 𝑢 at
1
𝑢=20ms-1, 𝑇=4s, 𝐻𝑜 =?, 𝐻𝑜 = 𝑔𝑇2, angle 60° to the horizontal, if the time taken to
2
1 reach the plane of projection is 8.66s and the
𝐻𝑜 = × 10×42=5×16=80m.
2 horizontal range is 𝑆, determine the values of 𝑢
4. A body is projected horizontally from the top and 𝑆 (g=10ms-2) A.50.0ms-1 and 216.5m
of a cliff 45m above the ground. If the body lands B.25.0ms-1 and 125.0m C.20.0ms-1 and 108.3m
at a distance 30m from the foot of the cliff, D.10.0ms-1 and 64.3m
calculate the speed of the projection (g=10ms-2) θ=60°, 𝑇=8.66s, 𝑢=?, 𝑅=𝑆=?,
A.10ms-1 B.15ms-1 C.20ms-1 D.30ms-1 2𝑢 𝑠𝑖𝑛𝜃 2𝑢 𝑠𝑖𝑛60°
𝑅2𝑔
𝑇= , 8.66= , 86.6 = 2𝑢 𝑠𝑖𝑛60°,
𝑔 10
𝐻𝑜 =45m, 𝑅=30m, g=10ms-2, 𝑢=? 𝐻 = , 86.6
2𝑢2 86.6= 2𝑢 × 0.866, 𝑢 = = 50ms-1.
𝑅2𝑔 302 ×10 2×0.866
𝑢2 = = = 100, 𝑢 = √100 =10ms-1 𝑢2 𝑠𝑖𝑛2𝜃 502 𝑠𝑖𝑛2(60°)
2𝐻𝑜 2×45 𝑅= , 𝑅=
5. A stone is projected horizontally from the top 2𝑔 10
2500𝑠𝑖𝑛120°
of a tower with a speed of 10ms-1.It lands at a 𝑅=𝑆= = 250×0.866=216.5m.
10
horizontal distance of 20m from the foot of the OR, 𝑅 = 𝑢𝑐𝑜𝑠θ 𝑇 = 50cos 60° × 8.66= 216.5m.
tower.Calculate the height of the tower(g=10ms- 10. An object is projected with a velocity of
2) A.4.9m B.9.8m C.10.0m D.19.6m E.20.0m
80m/s at angle of 30° to the horizontal. The
𝑅=20m, 𝑢=10ms-1, g=10ms-2,𝐻=?, maximum height reached is A.20m B.80m
𝑅2 𝑔 202 ×10
𝐻𝑜 = 2 = = 20m. C.160m D.320m.
2𝑢 2×102
6. A body is projected is projected horizontally 𝑢=80ms-1, θ=30°, g=10ms-2, 𝐻=?,
𝑢2 𝑠𝑖𝑛2 𝜃 802 ×𝑠𝑖𝑛2 30° 6400×(0.5)2
with a velocity of 80ms-1 from the top of a tower. 𝐻= = = ,
2𝑔 2×10 20
If it strikes the ground at a horizontal distance of 𝐻 = 320×0.23.5 = 80m.
456.8m from the foot of the tower, find the I.total 11. A projectile attains its maximum height when
time of flight II.height of the tower and the angle of projection is A.30° B.45° C.60° D.75°
III.velocity 3seconds after projection. E.90°.
𝑢=80ms-1, 𝑅=456.8m, 𝑡=total time of flight, 12. A missle is projected so as to attain its
𝑅 456.8
I.𝑅 = 𝑢𝑇, 𝑇 = = = 5.71ms-1. maximum range. Calculate the maximum
𝑢 80
attained if the initial velocity of projection is

25
Demystified Series Physics Demystified by Dr Timothy
200ms-1 (g=10ms-2) A.1500m B.1000m C.500m 1
𝐻𝑜 = 𝑔𝑇 2 − 𝑢𝑠𝑖𝑛θ𝑇 ,
2
D.250m. 1
𝑢=200ms-1, g=10ms-2, range is maximum at 80 = ×10× 𝑇 2 −60𝑠𝑖𝑛30°𝑇,
2
𝑢2 𝑠𝑖𝑛2 𝜃 2002 ×𝑠𝑖𝑛2 45° 80=5𝑇 2 −30𝑇, 5𝑇 2 −30𝑇 −80=0,
θ=45°, 𝐻=?, 𝐻 = = ,
2𝑔 2×10 𝑇 2 −6𝑇 −16=0, (𝑇 −8)(𝑇 −2)=0,
40000×(1/√2)2
𝐻= = 2000 × = 1000m.
1
𝑇 = 8 or −2. Time of flight is 8s as time cannot be
20 2
negative.
13. A shooter wants to fire a bullet in such a way
𝑢 𝑠𝑖𝑛𝜃 2(𝐻𝑜+H)
that its horizontal range is three times its Alternatively , 𝑇 = 𝑡𝑚𝑎𝑥 + 𝑡L = +√
𝑔 𝑔
maximum height. At what angle should he fire the
𝑢2 𝑠𝑖𝑛2 𝜃 602 𝑠𝑖𝑛2 30° 3600×(0.5)2
bullet to achieve this? A.64° B.53° C.45° D.30°. 𝐻=
2𝑔
=
2×10
=
20
=45m.
𝐻 𝑡𝑎𝑛𝜃 𝐻 𝑡𝑎𝑛𝜃 1 tan θ
𝑅=3𝐻, θ=?, = , = , = , 60 𝑠𝑖𝑛30° 2(80+45)
4
𝑅 4 3𝐻 4 3 4 𝑇= +√ = 3+5 = 8s.
tan θ = = 1.33 , θ = tan 1.33 = 53°.−1 10 10
3
14. At a birthday party the celebrant pops a
corked fruit wine. If the cork shoots out of the ● Momentum and Newton Law’s of
bottle at an angle of 40° to the horizontal and motion
travels a horizontal distance of 4.50m in 1.25s, - Linear momentum – Linear momentum is the
calculate the initial speed of the cork A.4.2ms -1 product of mass and velocity of a body. It is the
B.4.7ms-1 C.5.6ms-1 D.7.1ms-1. quantity of motion in a body. 𝑷 = 𝒎𝒗.
θ=40°, R=4.50m, T=1.25s, 𝑢=?, R = 𝑢𝑐𝑜𝑠𝜃 T , - A body with a higher mass will move with a
4.50 𝟏
4.50 = 𝑢 cos 40° × 1.25 , = 𝑢 cos 40° , smaller speed or velocity as 𝒎 ∝ .
1.25 𝒗
3.6 3.6
3.6 = 𝑢 cos 40° , 𝑢 = = = 4.7ms-1. - A body at rest has a linear momentum=0.
cos 40° 0.766
15. At what angle to the horizontal must a noozle - Newton’s Laws of motion :
of a machine gun be kept when firing to obtain a 1.Newton’s First law or Law of inertia – It
maximum horizontal range for the bullets? states that a body will continue in its state of rest
A.0.0° B.22.5° C.30.0° D.45.0° D.90.0°. or uniform motion in a straight line unless acted
16. Two bodies X and Y are projected on the same upon by a force. Inertia is the reluctance of a
horizontal planes with the same initial speed but body to start moving while at rest or to stop
at angles 30° and 60° respectively to the moving while in uniform motion. Inertia
horizontal. Neglecting air resistance, the ratio of increases with mass.
2.Newton’s Second law – It states that the rate
the range of X to that of Y is A.1:1 B.1:2 C.√3:1
of change of momentum is directly proportional
C.1:√3 . to the applied force in the direction of the force.
The sum of the angles gives 90°, hence they have OR It states that a body which is subjected to a
the same range and the ratio of their range, net external force acquires an acceleration which
RX:RY=1:1. is directly proportional to the force and inversely
17. A stone Q is thrown with a velocity 𝑢 at an proportional to the mass of the body. i.e
angle of 75° to the horizontal. Another stone R is ∆𝑷 ∆𝑷 ∆𝑷 𝒎(𝒗−𝒖)
thrown with the same velocity 𝑢 but at an angle 𝑭 ∝ , 𝑭 = 𝒌 , 𝑘=1, 𝑭 = = .
𝒕 𝒕 𝒕 𝒕
(𝒗−𝒖)
of 15° to the horizontal. The ranges covered by 𝒂= , 𝑭 = 𝒎𝒂. OR 𝒂 ∝ , 𝑭 = 𝒎𝒂.
𝑭
𝒕 𝒎
the stones will be A.greater for Q B.greater for R
𝒎𝒖=intial momentum, 𝒎𝒗=final momentum,
C.the same for Q and R D.greater for the heavier
Change in momentum ∆𝑷=𝒎𝒗 − 𝒎𝒖=𝒎(𝒗 − 𝒖).
stones. On rebounds, change in momentum ∆𝑷 = 𝒎𝒗 +
18. A projectile is fired at an angle of 30° to the
𝒎𝒖 = 𝒎(𝒗 + 𝒖). Change in velocity ∆𝒗 = 𝒗 − 𝒖.
horiozontal with a velocity of 40ms-1. Calculate
- On rebouncing, the direction of the velocity
the velocity attained after 1s. A.18.01ms-1 changes to opposite its initial direction
B.36.02ms-1 C.54.06ms-1 D.72.04ms-1(g=10ms-2)
- Acceleration is inversely proportional to the
θ=30°, 𝑢=40ms-1, g=10ms-2, 𝑡=1s. The velocity 𝟏
attained after 1s is the instantaneous speed 𝑣. mass of body at a constant force. 𝒂 ∝ ,
𝒎
𝑣 = √𝑣𝑦 + 𝑣𝑥 . 𝑣𝑥 = 𝑢𝑐𝑜𝑠θ = 40cos 30°= 34.6ms-1. (𝐹=constant) and the graphical relationship is
𝑣𝑦 =𝑢𝑠𝑖𝑛θ−𝑔t=40sin 30° −10×1=20−10=10ms-1 given below :
𝒂 𝒂
𝑣 = √𝑣𝑦 2 + 𝑣𝑥 2 = √102 + 34.62 ,
𝑣 = √100 + 1197.16 = √1297.16 = 36.02ms-2.
19. An object is projected from a height of 80m 𝟏/𝒎 𝒎
above the ground with a velocity of 60ms-1 at an 3.Newton’s Third law – To every action there is
angle of 30° to the horizontal. The time of flight is an equal but opposite reaction i.e action and
A.16s B.10s C.8s D.4s . reaction are equal and opposite.
𝐻𝑜 =80m, 𝑢=60ms-1, g=10ms-2, θ=30°, 𝑇=?, - Impulse is the product of force and its time of

26
Demystified Series Physics Demystified by Dr Timothy
action. 𝑰 = 𝑭𝒕. 𝑭 =
𝒎(𝒗−𝒖)
, 𝑰 = 𝑭𝒕 = 𝒎(𝒗 − 𝒖). velocities. It is the comparison between the final
𝒕 and initial velocities of colliding bodies to the
- Impulse is change in momentum (and not just
determine the elastic nature of the collision. 𝒆 =
momentum) while force is rate of change of 𝒗𝟐 −𝒗𝟏
momentum. . 𝑢1 and 𝑢2are intial velocities before
𝒖𝟐 −𝒖𝟏
- The area of the the figure under a force-time collision, 𝑣 1 and 𝑣 2 are the final velocities after
graph is equal to impulse or change in collision.
momentum. 𝒆 =0 , 𝒗𝟐 = 𝒗𝟏 or 𝒗𝟐 − 𝒗𝟏 =0, for a perfectly
- At constant or uniform velocity, change in inelastic collision.
momentum and force is equal to zero while 𝒆 =1 , 𝒗𝟐 − 𝒗𝟏 = 𝒖𝟐 − 𝒖𝟏 , for a perfectly elastic
momentum is constant i.e initial momentum collision.
equals the final momentum. - Application of Newton’s third law of motion:
- Coservation of linear momentum – It states I. Explosion Recoil of a gun – When a gun is
that the total momentum before collision is equal fired, the bullets moves with a high velocity hits
to the total momentum after collision. the target in minimum time possible due to its
- Collision of objects – Elastic and inelastic; small mass and the gun move in opposite
1. Elastic collision – Momentum and kinetic direction to that of the bullet with a small velocity
energy is conserved due to conservative forces due to its high mass.
e.g motion of gas molecules and the - The recoil velocity of a gun is less than that of
container,two moving bodies, photon and the bullet fired from it because of the difference
electron,collision of billiard balls. in mass . Momentum of the gun=Momentum of
I. Total momentum before collision is equal to the bullet, 𝑴g𝑽g= 𝒎b𝒗b . 𝒎 ∝1/𝒗 (a heavy body or
total momentum after collision. body with higher mass a moves slowly or has a
𝒎1𝒖1+𝒎2𝒖2= 𝒎1𝒗1+𝒎2𝒗2. low velocity).
II. Total kinetic energy before collision is equal to - The total momentum of the gun-bullet
the total kinetic energy after collision. system is zero, 𝑴g𝑽g−𝒎b𝒗b = 𝟎.
𝟏 𝟏
Before collision : 𝑲. 𝑬𝒊 = 𝒎1𝒖12+ 𝒎2𝒖22, II. Jet and rocket propulsion – A jet engine or
𝟐
𝟏
𝟐
𝟏 rocket burns and expels stream of hot gas at very
After collision : 𝑲. 𝑬𝒇 = 𝒎1𝒗12+ 𝒎2𝒗22 high temperature.The rate of change in
𝟐 𝟐
𝟏 𝟏 𝟏
𝒎1𝒖12+ 𝒎2𝒖22 = 𝒎1𝒗12+ 𝒎2𝒗22.
𝟏
momentum of the exhausted or expelled gas is
𝟐 𝟐 𝟐 𝟐
III. Sum of the initial and final velocities of both equal and opposite to the thrust(force)
colliding bodies are equal for bodies of equal experienced by the rocket or air craft.
∆𝒎 ∆𝑚
masses or identical bodies. Thrust, 𝑭 = × 𝒗Exhaust . = mass of gas
𝒕 𝑡
𝒖1+𝒖2= 𝒗1+𝒗2. ejected through the exhaust per second,Kgs-1,
2. Inelastic collision – Momentum is conserved 𝑣 Exhaust= velocity of the exhaust gas relative to the
but kinetic energy is not conserved i.e decreases engine.
due to dissipative forces e.g photoelectric
effect(photon and atom), bullet and target, ball Examples :
dropped onto a mud and bodies sticking together 1. Newton’s first law of motion states that a body
after collision. continues in its state of rest or uniform motion in
I.Total momentum before collision=Total a straight line unless A.The composition of the
momentum after collision. body changes B.The force of gravity changes
𝒎1𝒖1+𝒎2𝒖2 =(𝒎1+𝒎2)𝑽, 𝑉=common or C.There is action and reaction D.Forces are
composite velocity of both bodies when they impressed on the body E.There is no changes in
stick sticked together. the direction of motion of the body.
II. Total kinetic energy before collision 𝑲. 𝑬𝒊 = Newton’s first law is also called law of
𝟏 𝟏
𝒎1𝒖12+ 𝒎2𝒖22. Total kinetic energy after inertia.Inertia is the reluctance of a body to start
𝟐 𝟐
𝟏 moving while at rest or to stop moving while in
collision 𝑲. 𝑬𝒇 = (𝒎1+𝒎2)𝑽2. uniform motion.Inertia increases with mass.
𝟐
- Kinetic energy lost = 𝑲. 𝑬𝒊 − 𝑲. 𝑬𝒇 . Option C denotes “Newton’s third law of motion”.
III. Ratio of the final 𝐾. 𝐸 2 to initial 𝐾. 𝐸 1 kinetic 2. A boy sits in a train moving with uniform speed
energy is –
𝑲.𝑬𝒇
=
(𝒎𝟏 +𝒎𝟐 )𝑽𝟐
. in a straight track.If from his outstretched
𝑲.𝑬𝒊 𝒎𝟏 𝒖𝟏 𝟐 +𝒎𝟐 𝒖𝟐 𝟐 palm,he tossed a coin upwards,the coin will fall
- If both bodies are moving in the same direction A.in front of his palm B.behind his palm C.beside
the velocity carry same sign but carry opposite his plam D.into his palm
signs when they are moving in opposite A body moving with uniform or constant speed is
directions. like a body at rest(stationary body.Hence,the
- Coefficient of restitution 𝒆 – It is the ratio of displaced coin returns to the position from which
the final relative velocities to initial relative it was displaced.

27
Demystified Series Physics Demystified by Dr Timothy
3. The quantity of motion of a body is its hitting the vertical wall, with the same speed),
A.acceleration B.momentum C.displacement impulse, 𝐼=?, 𝐼 = 𝐹𝑡 = 𝑚(𝑣 + 𝑢) i.e rebounds,
D.velocity. 𝐼 = 5(2+2) = 20kgms-1 or 20Ns.
4. An object of mass 5.0kg moves with a velocity 12. The area under a force-time graph represents
of 10ms-1. Calculate its momentum A.50.0kgms- A.change in kinetic energy B.change in
1 B.15.0kgms-1 C.2.0kgms-1 D.0.5kgms-1. momentum C.workdone D.change in internal
𝑚=5.0kg, 𝑣-10ms-1, 𝑃=?, 𝑃 = 𝑚𝑣 , energy.
𝑃 = 5×10 = 50kgms-1. - The area under a force-distance graph
5. A constant force of 5𝑁 acts for 5s on a mass of represents workdone which is equal to change
5kg initially at rest. Calculate the final in kinetic energy.
momentum A.125kgms-1 B.25kgms-1 C.15kgms- 13. 𝐹(𝑁)
1 D.5kgms-1 E.0kgms-1 20
𝐹=5𝑁, 𝑡=5s, 𝑚=5kg, 𝑢=0(initially at rest),
𝐹𝑡 = 𝑚(𝑣 − 𝑢) = 𝑚𝑣 − 𝑚𝑢, 𝑚𝑢=0,
Final momentum, 𝑚𝑣 = 𝐹𝑡 = 5×5 = 25kgms-1. 2 4 𝑡(𝑠)
6. A resultant force of 15.00𝑁 acts for 6s on a The graph above illustrates a force 𝐹 acting on a
body of mass 4kg. Calculate the change in ball initially at rest on a smooth surface for a time
momentum of the body within this period. 𝑡. Calculate the change in momentum of the ball
A.3.75kgms-1 B.10.00kgms-1 C.22.50kgms-1 in 𝑁𝑠 after 4s. A.10Ns B.20Ns C.30Ns D.40Ns.
D.90.00kgms . -1 Area of a force-time(𝐹-𝑡) graph is equal to
𝑚=4kg , 𝐹=15𝑁, 𝑡=6s, ∆𝑃=?, impulse or change in momentum. 𝑢=0(at rest),
Change in momentum ∆𝑃 = 𝑚(𝑣 − 𝑢) = 𝐹𝑡 Initial momentum =𝑚𝑢=0.
1
Change in momentum ∆𝑃 =15×6=90kgms-1. Change in momentum = ×4×20 = 40Ns.
2
7. A body of mass 2kg moving vertically upwards 14. A rope is being used to pull a mass of 10kg
has its initial velocity increased uniformly from vertically upward. Determine the tension in the
10ms-1 to 40ms-1 in 4s. Neglecting air resistance, rope if, starting from rest, the mass acquires a
calculate the upward vertical force A.15N B.20N velocity of 4ms-1 in 8s. (g=10ms-2) A.105N B.95N
C.35N D.45N. C.50N D.5N.
𝑚=2kg, 𝑢=10ms-1, 𝑣=40ms-1, 𝑡=4s, 𝐹=?, 𝑢=0(from rest), 𝑣=4ms-1, 𝑚=10kg, g=10ms-2,
𝑚(𝑣−𝑢) 2(40−10)
𝐹= = = 15𝑁. 𝑡=8s, 𝐹 =
𝑚(𝑣−𝑢)
=
10(4−0) 40
= = 5N,
𝑡 4
𝑡 8 8
8. A force acts on a body for 0.5s changing its Tension 𝑇 = 𝑊 − 𝐹, weight 𝑊 = 𝑚𝑔,
momentum from 16.0kgms-1 to 21.0kgms-1, 𝑇 = 10×10 – 5 = 100−5 = 95N.
calculate the magnitude of the force A.42.0N 15. A body of mass 4kg is accelerated from rest
B.37.0N C.32.0N D.10.0N. by a steady force of 9𝑁. What is its speed when it
𝑡=0.5s, 𝑃1 =16kgms-1, 𝑃2 =21kgms-1, 𝐹=?, has travelled a distance of 8m? A.2ms-1 B.4ms-1
∆𝑃 𝑃 −𝑃 21−16 5
𝐹= = 2 1= = = 10N. C.6ms-1 D.8ms-1
𝑡 𝑡 0.5 0.5
9. What change in velocity would be produced on 𝑢=0, 𝐹=9N, 𝑠=8m, 𝑚=4kg, 𝑣 2= 𝑢2+2𝑎𝑠,
a body of masses 4kg if a constant force of 6N acts 𝑣2 𝑚𝑣 2
𝑣 2=2𝑎𝑠 (𝑢2=0), 𝑎 = , 𝐹 = 𝑚𝑎 =
2𝑠 2𝑠
on it for 2s? A.0.5ms-1 B.3.0ms-1 C.8.0ms-1 4𝑣 2 9×16
D.32.0ms E.128.0ms-1.
-1 9= , 𝑣2 = =36, 𝑣 = √36 = 6ms-1.
2×8 4
𝑚=4kg, 𝐹=6N, 𝑡=2s, ∆𝑣=?, 𝐹𝑡 = 𝑚(𝑣 − 𝑢) , 16. A car of mass 800kg moves from rest on a
𝐹𝑡 6×2
∆𝑣 = (𝑣 − 𝑢) = = = 3ms-1. horizontal track and travels 60m in 20s with
𝑚 4
10. A ball of mass 0.1kg approaching a tennis uniform acceleration. Assuming there were no
player with a velocity of 10ms-1 is hit in the frictional forces, calculate the accelerating force
opposite direction with a velocity of 15ms-1. If the A.120N B.240N C.480N D.960N.
1
time of impact between the racket and the ball is 𝑢=0, 𝑚=800kg, 𝑠=60m, 𝑡=20s, 𝑠 = 𝑢𝑡 + 𝑎𝑡2
2
0.01s, calculate the magnitude of the force with 1
𝑢𝑡=0, 𝑠 = 𝑎𝑡2 , 𝑎 =
2𝑠
, 𝐹 = 𝑚𝑎 =
2𝑚𝑠
,
𝑡2 𝑡2
which the ball is hit. A.5000𝑁 B.500𝑁C.250𝑵 2
2×800×60 96000
D.50𝑁. 𝐹= = = 240N.
202 400
𝑚=0.1kg, 𝑢=10ms-1, 𝑣=15ms-1, 𝑡=0.01s, 𝐹=?, 17. A force of 16N applied to a 4kg block that is
𝐹=
𝑚(𝑣−𝑢)
, 𝐹=
𝑚(𝑣+𝑢)
(moving in opposite at rest on a smooth horizontal surface. What is
𝑡 𝑡
0.1(15+10)
the velocity of the block at 𝑡=5s? A.4ms-1 B.10ms-
direction). 𝐹 = =10(25)=250𝑁. 1 C.20ms-1 D.50ms-1 E.80ms-1.
0.01
11. A ball of mass 5.0kg hits a smooth vertical 𝐹=16N, 𝑚=4kg, 𝑢=0, 𝑡=5s, 𝑣=?, 𝑣 = 𝑢 + 𝑎𝑡, 𝑣 =
𝑣 𝑚𝑣 𝐹𝑡
wall normally with a speed of 2ms-1. Determine 𝑎𝑡 (𝑢=0), 𝑎 = , 𝐹 = 𝑚𝑎 = , 𝑣= ,
𝑡 𝑡 𝑚
the impulse experienced by the wall A.20kgms- 16×5
1 B.10.0kgms-1 C.5.0kgms-1 D.2.5kgms-1. 𝑣= = 20ms-1.
4
𝑚=5kg, 𝑢=2ms-1, 𝑣=2ms-1(the ball rebounds on 18. The driver in a motor car of which the total

28
Demystified Series Physics Demystified by Dr Timothy
mass is 800kg and which is travelling at 20ms-1, moves off with a speed of 2ms-1 in the original
suddenly observes a stationary dog in his path direction of 𝑃. Calculate the initial velocity of 𝑃
50m ahead. If the car brakes exert a force of A.2ms-1 B.4ms-1 C.6ms-1 D.8ms-1.
2000N, what will most likely happen? A.The car Elastic collision: 𝑚1=𝑚2=𝑚(as bot bodies are
will be able to stop immediately the driver 1 1 2
identical) 𝑢1=𝑢, loses 𝑢, 𝑣 1=𝑢 − 𝑢= 𝑢, 𝑢2=0(at
3 3 3
notices the dog B.The car will stop 30m after
rest), 𝑣 2=2ms-1, Sum of the initial and final
hitting the dog C.The car will stop 20m in front
velocities of both colliding bodies are equal for
of the dog C.The driver will quickly reverse the
bodies of equal masses or identical bodies.
car D.The car will stop 5m beyond the dog 2 2
𝐹=2000N, 𝑢=20ms-1, 𝑚=800kg, 𝑠=80m, The car 𝑢1+𝑢2= 𝑣 1+𝑣2 , 𝑢 + 0 = 𝑢 + 2, 𝑢 − 𝑢 = 2 ,
3 3
1
comes to rest by the application of brakes, hence 𝑢 = 2, 𝑢 = 2×3 = 6ms-1.
3
𝑣=0 and 𝑎 is deceleration or negative 23. A trolley of mass 4kg moving on a smooth
acceleration or retardation. 𝑣 2= 𝑢2−2𝑎𝑠, horizontal platform with a speed of 1.0ms-1
𝑢2
0= 𝑢2−2𝑎𝑠, 𝑢2=2𝑎𝑠 , 𝑎 = , 𝐹 = 𝑚𝑎, collides perfectly with a stationary trolley of the
2𝑠
𝑚𝑢2 𝑚𝑢2 202 ×800 same mass on the same platform. Calculate the
𝐹= , 𝑠= = =80m. total momentum of the two trolleys immediately
2𝑠 2𝐹 2×2000
The distance moved by the motor car when after the collision. A.4Ns B.6Ns C.8Ns D.10Ns.
brakes is apllied is 80m, which is greaterthan the 𝑚1=4kg, 𝑢1=1.0ms-1, 𝑚2=4kg(same mass),
distance of the dog from the car i.e 50m. Hence, 𝑢2=0(stationary trolley),
the car will stop 30m after hitting the dog. Momentum before collision=Momentum after
19. Four identical trolleys are loaded with collision.
different masses and move along a straight road Momentum before collsion=𝑚1𝑢1+𝑚2𝑢2
at the same speed. Which of the trolleys has the Momentum after collision=𝑚1𝑣 1+𝑚2𝑣 2,
greatest inertia Total momentum after collision=Total
A. 2kg B. 4kg C. 6kg D. 8kg momentum before collision.
Total momentum after collision = 𝑚1𝑢1+𝑚2𝑢2
= 4×1+4×0 = 4Ns or 4kgms-1.
Inertia increases with mass thus, the trolley with 24. A ball of mass 0.5kg moving at 10ms-1 collides
a load of 8kg will have the greatest inertia. with another ball of equal mass at rest. If the two
20. The impact of two colliding objects is said to balls move off together after impact, calculate
be perfectly elastic if A.the total kinetic energy their common velocity. A.2.5ms-1B.5.0ms-1
of the object after collision is equal to that C.7.5ms D.10.0ms .
-1 -1
before collision B.the body coalesce and move 𝑚1=0.5kg, 𝑢1=10ms-1, 𝑚2=0.5kg, 𝑢2=0,
together with a common speed after collision 𝑉=common velocity. Inelastic collision –
C.after collision,the kinetic energy of one object is 𝑚1𝑢1+𝑚2𝑢2=(𝑚1+𝑚2)𝑉
equal to that of the other object D.it obeys law of 0.5×10+0.5×0=(0.5+0.5)𝑉, 5=(1)𝑉,
conservation of linear momentum 𝑉 = 5ms-1.
Both elastic and inelaatic collisions obeys law of 25. A ball of mass 6.00kg moving with a velocity
conservation of linear momentum i.e total of 10ms-1 collides with a 2.00kg ball moving in
momentum before and after collision are the opposite direction with a velocity of 5.0ms-1.
equal.Kinetic energy is conserved in elastic After the collision, the two balls coalesce and
collision but not conserved in inelastic collision. move in the same direction. Calculate the velocity
21. Two bodies each of mass m, moving with the of the composite body A.30.00ms-1 B.12.00ms-1
same constant speed, 𝑣 are approaching each C.8.75ms-1 D.6.25ms-1 E.5.00ms-1.
other on a straight line. Their total kinetic energy 𝑚1=6kg, 𝑢1=10ms-1, 𝑚2=2kg, 𝑢2=−5ms-1
𝐸 K and momentum 𝑃 are given by A.𝐸 K=0, (opposite direction), 𝑉=?,
1
𝑃 =2𝑚𝑣 B.𝐸 K= 𝑚𝑣 2, 𝑃 = 𝑚𝑣 C.𝐸 K=𝑚𝑣 2, 𝑃 = 𝑚1𝑢1+𝑚2𝑢2=(𝑚1+𝑚2)𝑉, 6×10+2(−5)=(6+2)𝑉
2 50
𝑚𝑣 D.𝑬K= 𝒎𝒗2, 𝑷 =0. 60−10 = 8𝑉, 50 = 8𝑉, 𝑉 = = 6.25ms-1.
8
𝑚1=𝑚2=𝑚, 𝑣 1=𝑣 2=𝑣 , Kinetic energy is scalar 26. A particle of mass 𝑚 initially at rest splits into
quantity, hence independent of their direction. two. If one of the particle of mass 𝑚1 moves with
Total kinetic energy of both bodies, velocity 𝑣 1, the second particle moves with
1 𝑚 𝑣 𝒎 𝒗 𝑚 𝑣 𝑚𝑣
𝐸 K=2× 𝑚𝑣 2= 𝑚𝑣2. Momentum is a vector velocity A. 1 1 B. 𝟏 𝟏 C. 1 1 D. .
2 𝑚−𝑚1 𝒎𝟏 −𝒎 𝑚 𝑚−𝑚1
quantity hence, is dependent on their direction. Initial momentum before the particle splits=0 as
Approaching bodies are moving in opposite the body is initially at rest. mass of the first
direction. Total momentum 𝑃 = 𝑚𝑣 +(−𝑚𝑣)=0. particle=𝑚1, velocity of first particle=𝑣 1
22. A ball 𝑃 of mass 0.25kg, loses one-third of its mass of the second particle=𝑚2= 𝑚 − 𝑚1,
velocity when it makes a head on collision with velocity of second particle,𝑣 2=?
an identical ball 𝑄 at rest. After the collision, 𝑄 Final momentum after the particle splits =

29
Demystified Series Physics Demystified by Dr Timothy
𝑚1𝑣 1+𝑚2𝑣 2= 𝑚1𝑣 1+(𝑚 − 𝑚 1)𝑣 2 varying direction, though their magnitude is
Initial momentum=Final momentum, constant. 3.Acceleration and force are directed
0= 𝑚1𝑣 1+(𝑚 − 𝑚1)𝑣 2,−(𝑚 − 𝑚1)𝑣 2= 𝑚1𝑣 1 towards the centre while velocity is tangent to
(−𝑚 + 𝑚1)𝑣 2= 𝑚1𝑣 1, (𝑚1−𝑚)𝑣 2= 𝑚1𝑣 1, the path. 4.Acceleration and force are
𝑚 𝑣
𝑣2 = 1 1 . perpendicular to the velocity. 5.No work is done
𝑚1 −𝑚
as its movement is about a fixed point(no
27. An arrow of mass 0.3kg is fired with a velocity
displacement). 6.Kinetic energy is constant.
of 100ms-1 into a wodden block of mass 0.7kg.
7.Uniform circular motion is a conservative
Calculate the final 𝐾. 𝐸 after impact, given that
motion.
the wooden block can freely move. A.450J B.500J
- Centripetal or radial acceleration 𝒂Cis the
C.600J D.700J.
acceleration of a body moving a circular path and
𝑚1=0.3kg, 𝑢1=10ms-1, 𝑚2=0.7kg, 𝑢2=0, 𝑉=?
its directed towards the centre of the circular
Momentum of arrow=Momentum of arrow and 𝒗𝟐
block. 𝑚1𝑢1 =(𝑚1+𝑚2)𝑉, path. 𝒂𝒄 = = 𝝎𝟐 𝒓 = 𝒗𝝎. 𝑣=uniform speed,
𝒓
0.3×100=(0.3+0.7)𝑉, 30=(1)𝑉, 𝑉 =30ms-1 𝑟=radius of the circular path, 𝜔=angular velocity
1
𝐾. 𝐸 after impact, 𝐾. 𝐸 2= (𝑚1+𝑚2)𝑉 2 or angular frequency
2
1 1 - Centripetal or radial force 𝑭C is the inward
𝐾. 𝐸 2= (0.3+0.7)×302= ×1×900=450𝐽.
2 2 force on a body moving in a circular path and is
28. Which of the following explains why the directed towards the centre of the path.
velocity of a recoiling gun is lower than that of the 𝒎𝒗𝟐
ejected bullet? I.The bullet must hit a target in the 𝑭𝒄 = 𝒎𝒂𝒄 = = 𝒎𝝎𝟐 𝒓 = 𝒎𝒗𝝎.
𝒓
minimum time possible II.The mass of the bullet - Centrifugal force is the outwardly directed
is much smaller than that of the gun III.The force. It is an equal and opposite force which
momentum of the gun-bullet system must be balances the centripetal force to keep a body
zero A.I B.II C.I and II D.II and III E.I,II and III. moving in a circular path. The effect of
29. A gun of mass 2.0kg fires a bullet of mass centrifugal force is felt a driver in car as it
1.6×10-2kg due East. If the bullet leaves the turns a corner.
nozzle of the gun with a velocity of 150ms-1, what - The force holding the moon in its orbit
is the recoil velocity of the gun? A.150ms-1 due around the earth is centripetal force.
West B.1.2×10-4ms-1 due West C.1.2ms-1 due - Angular velocity 𝝎 is the angular distance
𝜽
West D.1.2ms-1 due East E.150ms-1 due East covered with time. 𝝎 = . Angular velocity and
𝒕
𝑀g=2kg, 𝑉 g=? 𝑚b=1.6×10-2kg, 𝑣 b=150ms-1, linear or tangential velocity are related by –
𝑀g𝑉 g= 𝑚b𝑣 b, 2× 𝑉 g=1.6×10-2×150, 𝒗
1.6×10−2 ×150
𝝎 = , 𝒗 = 𝝎𝒓. In one complete circle the
𝒓
𝑉g = = 1.2ms-1, The recoil velocity of angular distance 𝜃= 360° or 2𝜋 radian.
2
the gun is due West as it moves in opposite - Angular acceleration 𝜶is the change in angular
direction to the propelled bullet. velocity with time. 𝜶=
𝝎𝒇 −𝝎𝒊
. Angular
30. 50g fuel is burnt each second in a rocket 𝒕
acceleration and linear acceleration are related
engine and ejected as a gas with a speed of
by – 𝒂 = 𝜶𝒓.
4,000ms-1. The thrust on the rocket is A.20N 𝟐𝝅𝒓
B.80N C.200N D.500N. - 𝒗= , 𝑇=period or time taken for one
𝑻
∆𝑚=50g=0.05kg, 𝑡=1s,
∆𝑚
=
0.05
=0.05kgs-1, complete circle.
𝑡 1
∆𝑚 - 𝝎𝒇 = 𝝎𝒊 + 𝜶𝒕 , 𝝎𝒇 2 = 𝝎𝒊 2+2𝜶θ,
𝑣 Exhaust=4000ms-1. Thrust, 𝐹 = × 𝑣 Exhaust , 𝟏
𝑡 θ = 𝝎𝒊 𝒕 + 𝜶𝒕2. 𝝎𝒇 =final angular velocity
𝐹 = 0.05×4000 = 200N. 𝟐
31. The exhaust gases ejected at a speed of (𝜔𝑓 =0,when a body is brought to rest), 𝝎𝒊 =initial
800ms-1 from a jet engine produces a thrust of angular velocity (𝜔𝑖 =0,when a body moves from
560N. The mass of gases ejected every second is rest).
A.0.70kg B.1.43kg C.7.00kg D.14.29kg. - Tension in a string 𝑇 on a body whirled round a
∆𝑚 horizontal circle is provided by the centripetal
𝑣 Exhaust=800ms-1, Thrust, 𝐹=560N, =?
∆𝑚 ∆𝑚
𝑡 force and independent of the weight of the body.
𝐹= × 𝑣Exhaust, 560= × 800, 𝒎𝒗𝟐
𝑡 𝑡 Tension = Centripetal force. 𝑻 = 𝑭𝒄 = .
∆𝑚 560 𝒓
= = 0.7kg. - Tension in a string 𝑇 on a body whirled round a
𝑡 800
vertical circle depends on the weight of the body
● Circular motion and the centripetal force.
- Motion of a body about a circular path with a I. Tension maximum at the base or bottom of the
constant or uniform speed. circular path(𝜃 =0°). 𝑻𝒎𝒂𝒙 = 𝑭𝒄 + 𝑾 ,
𝒎𝒗𝟐
- A body undergoing circular motion : 1.Has a 𝑻𝒎𝒂𝒙 = + 𝒎𝒈.
𝒓
constant speed but varying direction. 2.Have II. Tension is minimum at the top or apex of the
varying velocity, momentum, acceleration due to

30
Demystified Series Physics Demystified by Dr Timothy
circular path(θ=90°). 𝑻𝒎𝒊𝒏 = 𝑭𝒄 − 𝑾 , For a body undergoing circular motion : its
𝑻𝒎𝒊𝒏 =
𝒎𝒗𝟐
− 𝒎𝒈. kinetic energy and speed (do not depend on its
𝒓 direction) are constant while the linear
- A body moving undergoing circular motion momentum (depends on its direction) varies.
will move along tangent to its path on 7. I.Its velocity is constant II.No work is done on
removal of the force causing it to move. the body III.It has constant acceleration directed
away from the centre IV.The centripetal force is
Examples : directed towards the centre. Which combination
1. If a body moves with a constant speed and at of the above is ture of a body moving with
the same time undergoes acceleration, its motion constant speed in a circular track? A.II and IV
is said to be A.circular B.oscillatory C.rectilinear B.I and IV C.II and III D.II and III.
D.rotational A body undergoing circular motion with constant
A body undergoing circular motion has a uniform speed has velocity and acceleration of constant
or constant speed, but it direction changes i.e magnitude but varying direction. The centripetal
velocity changes. Since velocity changes it also force and acceleration is directed towards the
has acceleration. center. No work is done as the body moves about
2. An object moves with uniform speed round a a fixed point i.e no distance or displacement.
circle. Its acceleration has A.constant 8. A body of mass 𝑚 moving round a circle of
magnitude and varying direction B.constant radius 𝑟 with a uniform speed 𝑣 experiences a
magnitude and constant direction C.varying centripetal force 𝐹. The workdone by the
magnitude and constant direction D.varying centripetal force on it is A.𝐹𝑣B.zero C.𝐹𝑚𝑟
magnitude and varying direction. D.𝐹𝑟/𝑚 .
3. A body is whirled round a circular path at 9. A man was moving round a circular path
constant speed. Which of the following diagrams continuously for 4 miutes and covered the
correctly indicates the direction of te body’s following distances in the times stated below :
velocity (𝑣) and acceleration (𝑎). Distance(in meters) : 200 400 600 800
A. 𝑎 B. 𝒗 C. D. Tim(in miutes) : 1 2 3 4
𝑣 𝒂 𝑣 𝑎 𝑣 𝑎 Which of the following statements is correct
about the motion of the man? A.The man was
moving with uniform speed but non-uniform
The velocity and acceleration of a body velocity B.The speed was changing constantly
undergoing circular motion are always C.The velocity was uniform D.It was an instance
perpendicular, with the acceleration towards the of motion in which both speed and velocity are
centre and the velocity tanget to the circular path the same.
as shown in option B. In circular motion, the speed is constant or
4. A ball is swung in a horizontal circle of centre uniform while the velocity is constantly changing
O with a constant speed. What is the direction of (non-uniform) due to change in the direction of
its acceleration? A.Away from O B.Toward O motion.
C.Along a tangent to the circle D.Along the 10. A ball attatched to one end of a string moves
circumference of the circle. anti-clockwisely around a circle whose centre is
5. Which of the following statements is/are O. If the string suddenly breaks when the ball is
correct about a car moving round a circular track at point P, along which of the following paths will
at a uniform speed I.its velocity is uniform II.it it move?
undergoes an acceleration III.the friction A. B. C. D.
between its tyres and the ground provides the
necessary centripetal force A.I B.II C.III D.II and O P O P O P O P
III E.I,II and III. A body moving undergoing circular motion will
A car moving in circle has a varying velocity and move along tangent to its path on removal of the
acceleration.The frictional force 𝐹𝑅 between the force causing it to move.
road and tyres provide the necessary centripetal 11. A body moves along a circular path with
𝑚𝑣 2
or radial force 𝐹𝑐 , 𝐹𝑅 = 𝐹𝑐 , 𝜇𝑅 = . uniform angular speed of 0.6rads-1 and at a
𝑟
6. Which of the following is true of a particle constant speed of 3.0ms-1.Calculate the
moving in a horizontal circle with constant acceleration of the body A.0.2ms-2 B.1.8ms-2
angular velocity? A.The energy is constant but C.5.0ms-2 D.5.4ms-2 E.25.0ms-2
the linear momentum varies B.The linear 𝜔=0.6rads-1, 𝑣=3.0ms-1, 𝑎𝑐 =?,
momentum is constant but the energy varies 𝑎𝑐 = 𝑣𝜔 = 3×0.6 = 1.8ms-2.
C.Both energy and linear momentum are 12. A body is rotating in a horizontal circle of
constant D.The speed and the linear momentum radius 2.5m with an angular speed of 5rads-1.
are both constant. Calculate the magnitude of the radial

31
Demystified Series Physics Demystified by Dr Timothy
acceleration of the body. A.62.5ms-2 B.31.2ms-2 Tension in a horizontal circle is provided by the
C.12.50ms-2 D.10.0ms-2 centripetal force. 𝑇 = 𝐹𝑐 = ,𝑇=
𝑚𝑣 2
= 162N.
6×92
𝜔=5rads-1, 𝑟=2.5m, 𝑎𝑐 =?, 𝑎𝑐 = 𝜔2 𝑟, 𝑟 3

𝑎𝑐 = 52×2.5 = 25×2.5 = 62.5ms-2. 20. An object of mass 15kg is whirled round a


13. An object of mass 2kg moves in a circular vertical circle of radius 3m with a constant speed
path of radius 0.5m with a speed of 1ms-1. What of 4.5ms-1. Find the maximum tension in the
is its angular velocity A.8rads-1 B.4rads-1 C.2rads- string (g=10ms-2) A.101.3N B.179.5N C.202.5N
1 D.1rads-1 . D.251.3N.
𝑚=2kg, 𝑟=0.5, 𝑣=1ms-1, 𝜔=0.5rads-1, Tension in a vertical circle is given by a combined
1 effect of the centripetal force and weight.
1 = 𝜔 ×0.5, 𝜔 = = 2rads-1. 𝑚𝑣 2
0.5
Maximum tension, 𝑇𝑚𝑎𝑥 = + 𝑚𝑔
14. A stone is whirled round a circular path of 𝑟
15×4.52
radius 15cm, if the stone makes 30 0scillations in 𝑇𝑚𝑎𝑥 = + 15×10 = 101.25+150 ,
3
10s, calculate the angular speed of the stone 𝑇𝑚𝑎𝑥 = 251.25𝑁 = 251.3𝑁.
(𝜋=3.14) A.9.42rads-1 B.12.5rads-1C.18.84rads-1 21. A mass attatched to a string is moving in a
D.62.80rads-1 E.141.30rads-1. circular path. If the speed is doubled, the tension
1 oscillation = 1 cycle = 360°, in the string will be A.doubles B.halved C.four
30 oscillations = 30 cycles = 30×360° times as great D.one-fourth as much.
360° = 2𝜋 rad, hence 30 oscillations 𝑚𝑣 2
=30×2𝜋 =60𝜋 rad. Angular distance 𝜃 =60𝜋 𝑇 = 𝐹 C= , 𝑇 ∝ 𝑣2, 𝑇1=𝑇, 𝑣 1=𝑣, 𝑇2=?, 𝑣 2=2𝑣,
𝑟
𝜃 60𝜋 𝑇1 𝑇2 𝑇 𝑇 𝑇×4𝑣 2
rad, 𝑡=10s, 𝜔=?, 𝜔 = = = 6𝜋 = 6×3.14, = , 2
= (2𝑣) 2 , 𝑇 2= = 4𝑇 .
𝑡 10 𝑣1 2 𝑣2 2 𝑣2 𝑣2
𝑛 30
𝜔= 18.84rads-1. Alternatively : 𝑓 = = , Alternatively, 𝑇 ∝ 𝑣 2,
if speed is doubled then,
𝑡 10
𝑓 = 3Hz. 𝜔 = 2𝜋𝑓 = 2×3.14×3 = 18.84rads-1. 𝑇 ∝ 22 = 4 , four times as great as the original
15. If a wheel 1.2m in diameter rotates at one value.
revolution per second, calculate the velocity of 22. A stone is tied to one end of a light string and
the wheel A.7.5ms-1 B.4.0ms-1 C.3.6ms-1 whirled in a vertical circle. The tension in the
D.3.8ms-1. string is maximum when the A.stone is at the top
𝑑 1.2 of the circular path B.stone is at the botton of
Diameter, 𝑑=1.2m, 𝑟 = = =0.6m, 𝑓=1revs-1,
2 2 the circular path C.string lies in a horizontal
1 revolution=1 complete cycle=360°=2𝜋 rad, direction D.stone is at any other point along the
𝜔 = 2𝜋𝑓 = 2𝜋 rads-1, 𝑣 = 𝜔𝑟 = 2𝜋 × 0.6 =1.2 𝜋, path .
𝜋=3.14, 𝑣 = 1.2×3.14 = 3.8ms-1. Tension of a body moving in a vertical circle is
16. A body of mass 12kg moves with a speed of maximum at the bottom or base of the circular
10ms-1 in a circular path of radius 5m. Calculate path and minimum at the top or apex of the
the magnitude of the force acting on the body circular path.
A.24N B.30N C.125N D.240N.
𝑚=12kg, 𝑣=10ms-1, 𝑟=5m, 𝐹𝑐 =?, ● Simple Harmonic motion
𝑚𝑣 2 12×102
𝐹𝑐 = = = 240N. - Simple Harmonic motion(SHM) is the
𝑟 5
17. A particle of mass 10-2kg is fixed to the tip of periodic motion of a body or particle along a
a fan blade which rotates with angular velocity of straight line such that the acceleration of the
100rads-1. If the radius of the blade is 0.2m, the body is always directed towards a fixed point i.e
centripetal force is A.2N B.20N C.200N D.400N. centre of motion and is proportional to the
𝑚=10-2kg, 𝜔=100rads-1, 𝑟=0.2m, displacement from that point e.g motion of a
𝐹𝑐 = 𝑚𝜔2 𝑟 = 10-2×1002×0.2 = 20N. simple pendulum,vertical motion of a body
18. An object of mass 5kg moves round a circle of suspended from a spring, vertical motion of a
radius 6m. If the period of motion is 𝜋s, calculate loaded test-tube in a liquid.
the force towards the centre A.60N B.120N - The restoring force and acceleration is directly
C.240N D.360N. proportional to its displacement, 𝑭 ∝ 𝒙 , 𝒂 ∝ 𝒙 ,
𝑚=5kg, 𝑟=6m, Period, 𝑇= 𝜋s, 𝐹𝑐 =?, 𝑣 =
2𝜋𝑟
, 𝑭 = −𝒌𝒙 , 𝒂 = 𝝎𝟐 𝒙.
2𝜋×6 𝑚𝑣 2 5×122
𝑇 - Amplitude 𝑨 is the maximum displacement of
𝑣= = 12ms-1, 𝐹 C= = = 120𝑁. a body from equilibrium position or rest.
𝜋 𝑟 6
19. An object of mass 6kg is whirled round a - The amplitude and angular velocity/speed
horizontal circle of radius 3m by a revolving in SHM naturally remains constant.
string inclined to the vertical. Calculate the - The distance between two extreme points in
tension in the string if the uniform speed of the SHM equals twice the amplitude(2𝑨).
object is 9ms-1 (g=10ms-2) A.27N B.54N C.162N - Period 𝑻 is the time taken for one complete
𝒕
D.173N. oscillation. 𝑻 = , 𝑡=time, 𝑛=number of
𝒏
𝑚=6kg, 𝑟=3m, 𝑣=9ms-1, Tension, 𝑇=?, oscillation.

32
Demystified Series Physics Demystified by Dr Timothy
- Frequency or rate of oscillation 𝒇 is the 𝑳 𝟐𝝅
𝒏 2. Period 𝑇 – 𝑻 = 𝟐𝝅√ = .
number of oscillations in one seconds. 𝒇 = . 𝒈 𝝎
𝒕
- Angular velocity or angular frequency 𝝎 is 3. Frequency or rate of oscillation 𝑓 –
𝜽 𝟏 𝑳 𝝎
the angular distance convered in time. 𝝎 = , 𝒇= √𝒈 = 𝟐𝝅 .
𝒕 𝟐𝝅
𝟐𝝅
𝝎 = = 𝟐𝝅𝒇. 𝑇=period, 𝑓=frequency.
𝑻
- Period of oscillation and frequency(rate of
- The displacement of a body in SHM is oscillation) of a simple pendulum depends on
represented as – 𝒙 = 𝑨𝒄𝒐𝒔𝝎𝒕 = 𝑨𝒄𝒐𝒔𝟐𝝅𝒇𝒕. length and acceleration due to gravity only.
𝑥=displacement, 𝐴-amplitude, 𝜔=angular 𝑳
1. 𝑻 ∝ √ thus, the period increases when the
velocity, 𝑓=frequency, 𝑡=time. 𝒈
- Relationship between SHM quantities : length increases and acceleration due to gravity
1.Force and displacement are in phase also decreases. The period of a simple pendulum in
acceleration and displacement are in phase, moon is greater than on earth (gmoon<gearth).
[force and acceleration are maximum when 𝑻𝟏 𝑻𝟐 𝑻𝟏 √𝑳𝟏 𝑳𝟏
- 𝑻 ∝ √𝑳 , = , = =√ .
displacement is maximum]. 2.Amplitude and √𝑳𝟏 √𝑳𝟐 𝑻𝟐 √𝑳𝟐 𝑳𝟐
energy are in phase, 𝑬 ∝ 𝑨2 [energy is maximum 𝒈
when amplitude is maximum]. 2. 𝒇 ∝ √ thus, the frequency increases when the
𝑳
3.Velocity/energy and displacement are not in length decreases and acceleration due to gravity
phase or antiphase i.e energy or velocity is increase.
maximum when displacement is zero or - 𝒈 is greater at poles and less at the equator,
minimum. 4.Amplitude and time are antiphase, hence a pendulum will have a lower period
𝟏
𝑨 ∝ , amplitude of an oscillating body decreases and higher frequency (swings faster) at the
𝒕
with time, due energy losses by friction. poles than the equator.
- The velolcity of a body in SHM at a distance x - The time taken for a pendulum to pass the
lowest point twice equal to the period.
from the equlibrium position : 𝒗 = 𝝎√𝑨𝟐 − 𝒙𝟐
- Simple pendula with the same length in the
- Velocity and Kinetic energy is maximum at
same location will have the same period.
equilibrium position (minimum displacement),
- If two pendula makes different oscillations in
displacement equals zero, 𝒙= 0.
the same time interval, their period and number
𝒗𝒎𝒂𝒙 = 𝝎𝑨 = 𝝎𝒓. Velocity and kinetic energy is
of oscillation is given by: T1n1=T2n2
minimum or zero at the extreme positions i.e
maximum displacement. - A graph of 𝑻2 vs 𝑳 or 𝑻 vs √𝑳 is a straight line
𝟒𝝅𝟐
- Acceleration and potential energy is maximum graph with slope = .
𝒈
at the extreme positions or end of path,
- A graph of 𝑳 vs 𝑻2 or√𝑳 vs 𝑻 is a straight line
displacement 𝒙 = Amplitude 𝑨 . 𝒂𝒎𝒂𝒙 = 𝝎2𝑨 = 𝒈
𝝎2𝒓. Acceleration and potential energy is graph with slope= 𝟐 .
𝟒𝝅
minimum or zero at equilibrium position. - A graph of 𝒍𝒐𝒈 𝑻 vs 𝒍𝒐𝒈 𝑳is a straight line graph.
- Total mechanical energy is constant. 𝑻 or 𝑻2
𝟏
- Kinetic energy 𝑲. 𝑬 = 𝒎𝒗𝟐 .
𝟐
𝟏
- Potential energy 𝑷. 𝑬 = 𝒌𝒙𝟐 . √𝑳 or 𝑳
𝟐
- The total workdone or energy stored or NB : All these graphs originate from the origin
maximum 𝐾. 𝐸 or 𝑃. 𝐸 in SHM is : (intercept, 𝑐 =0).
𝟏 𝟏
𝑬 = 𝒌𝑨2 = 𝒎𝝎2𝑨2. 𝑘=force constant. - A simple pendulum as maximum 𝐾. 𝐸 and
𝟐 𝟐
- Graphical representation of the variation of velocity at equilibrium position (lowest point)
potential energy and kinetic energy with time and maximum 𝑃. 𝐸 and height at extreme end
in SHM : (highest point).
Energy (J) - The maximum velocity of a simple pendulum is
𝑲. 𝑬 𝑷. 𝑬 given by – 𝒗𝒎𝒂𝒙 = √𝟐𝒈𝒉 . ℎ=vertical distance or
height covered by a pendulum during oscillation.
- Potential or kinetic energy or workdone 𝐸 in a
simple pendulum is – 𝑬 = 𝒎𝒈𝑳(𝟏 − 𝒄𝒐𝒔𝜽).
𝐿=length of the pendulum, 𝜃=angular
𝑻 𝑻 𝟑𝑻 𝑻 time (s) displacement.
𝟒 𝟐 𝟒 - Energy loses in a simple pendulum is due to
- SHM in simple pendulums – viscosity of the air.
1. Angular speed 𝜔 – 𝝎 = √ =
𝒈 𝟐𝝅
= 𝟐𝝅𝒇. - Assumptions made in simple pendulum
𝑳 𝑻
experiment : 1.The suspending string must be
inextensible. 2.The diameter(not mass) of the

33
Demystified Series Physics Demystified by Dr Timothy
bulb must be kept small. 3.The initial angle of A.acceleration is always directed towards a fixed
oscillation should be small. point B.path of motion is a straight line
- SHM in mass-spring system – C.acceleration is directed towards a fixed
𝒌 𝟐𝝅 point and proportional to its distance from
1. Angular speed 𝜔 – 𝝎 = √ = = 𝟐𝝅𝒇. the point D.acceleration is constant and directed
𝒎 𝑻
𝒎 𝒆 𝟐𝝅 towards a fixed point.
2. Period 𝑇 – 𝑻 = 𝟐𝝅√ = 𝟐𝝅√ = . 𝑘=force A motion is said to be simple harmonic when the
𝒌 𝒈 𝝎
constant or stiffness constant of spring, acceleration is directed towards a fixed point and
𝑒=extension of the spring. is proportional to its distance from the point.
𝟏 𝒌 𝝎
3. If a freely suspended object is pulled to one
3. Frequency 𝑓 – 𝒇 = √ = . side and released, it oscillates about the point of
𝟐𝝅 𝒎 𝟐𝝅
4.The tension in the spring 𝑇 is given by the suspension because the A.aceleration is
combined effect of the restoring force and weight directly proportional to the displacement
of the body attached to the spring. B.motion is directed away from the equilibrium
𝑻 = 𝑭 + 𝑾 = 𝒌𝒆 + 𝒎𝒈. point C.acceleration is directly proportional to
- SHM in a loaded test tube in a liquid – the square of the displacement D.velocity is
1. Depth of immersion of the tube ℎ – minimum at equilibrium point
𝒉=
𝒎
. 𝑚=mass of the test tube+mass of the A body exhibiting SHM as its restoring force and
𝝆𝑨 displacement directly proportional to its
liquid in the test tube, 𝜌=density of the liquid in displacement. Velocity is minimum at the
which the test tube is placed, 𝐴=area of the test extreme point and maximum and minimum at
tube=𝜋𝑟2. equilibrium point.
2. Period 𝑇 – 𝑻 = 𝟐𝝅√
𝒎
. 3. Which of the following statements about an
𝝆𝑨𝒈 object performing simple harmonic motion is
- Damped oscillation – This is the gradual correct. Its acceleration A.is minimum at the
decrease in the oscillation of an isolated body due extreme ends B.is constant and directed towards
decrease in amplitude of the body caused by a fixed point C.is zero when its is displaced from
energy loses due to friction. its equilibrium position D.varies linearly with
- Forced vibration – It is the vibration of a body the displacement from the fixed point and is
with a frequency of an external periodic force directed towards the fixed point.
which is applied to keep the body oscillating A body performing SHM has an acceleration
rather than its own natural frequency. The which is proportional to its displacement and is
amplitude and energy of forced oscillation directed towards the fixed point.The acceleration
depends on the difference between the frequency is maximum at the extreme ends and minimum at
of the external periodic force and the natural the equilibrium position. The acceleration is not
frequency of the body i.e the lower the difference constant.
in their frequencies the larger the amplitude and 4. The frequency of vibration of an oscillator is
𝟏
energy , 𝑨 or 𝑬 ∝ . 4𝐻𝑧. If it makes 12 vibrations in 𝑡 seconds,
∆𝒇
calculate the value of 𝑡 A.0.33s B.3.00s C.3.33s
- Resonance – It occurs when the frequency of
C.24.00s E.48.00s
the external periodic force coincides or equals 𝑛
the natural frequency of the oscillating body. The 𝑓=4𝐻𝑧, 𝑛=12 vibrations, 𝑡=?, 𝑓 = ,
𝑡
𝑛 12
frequency at which resonance occurs is called 𝑡= = = 3s.
𝑓 4
natural frequency. The amplitude and energy
5. A simple harmonic oscillator has a period of
is maximum at resonance.
0.02s and amplitude of 0.25m. Calculate the
- Application of forced vibration and
speed in ms-1 at the centre of the oscillation
resonance –
A.1.25𝜋B.25.00𝝅 C.100.00𝜋 D.400.00𝜋
I.Simple pendula with equal lengths vibrate with 2𝜋 2𝜋
large amplitude while ones with different lengths 𝑇=0.02s, 𝐴=0.25m, 𝜔 = = =100𝜋rads-1
𝑇 0.02
vibrates with small swings. maximum velocity, 𝑣 max= 𝜔𝐴 =100𝜋 ×0.25,
II.Swings in play grounds for children with 𝑣 max=25ms-1.
natural, greater and equal frequency. 6. A particle executing a simple harmonic motion
III.Walking along a bridge. Walking in steps or has a frequency of 50Hz and an amplitude of
uniformly sets the bridge to vibrate at the same 8.0×10-3m.Calculate the maximum velocity of its
frequency as that of the people walking or motion A.2.51ms-1 B.2.20ms-1 C.2.00ms-1
matching i.e resonance, hence causes the bridge D.1.50ms-1
to collapse. 𝑓=50𝐻𝑧𝐴=8.0×10-3m, 𝜔 = 2𝜋𝑓,
𝑣 max= 𝜔𝐴 = (2𝜋𝑓)𝐴 ,
Examples : 𝑣 max = (2𝜋 ×50)×8.0×10-3 = 0.8𝜋 = 2.51ms-1.
1. The motion of a body is simple harmonic if the 7. A body moves in SHM between two points 20m

34
Demystified Series Physics Demystified by Dr Timothy
on the straight line joining the points. If the B.Displacement and force on the body
angular speed of the body is 5rads-1, calculate its C.Velocity and acceleration D.Force acting on the
speed when it is 6m from the centre of motion body and acceleration.
A.20ms-1 B.30ms-1 C.40ms-1 D.50ms-1. Force and displacement are in phase. Velocity
Distance between two extreme points is twice and acceleration are not in phase i.e antiphase.
the amplitude, 20m= 2𝐴, 𝐴=10m, 𝜔=5rads-1, Velocity and displacement are not in phase i.e
𝑥=dislplacement=6m, 𝑣=?, 𝑣 = 𝜔√𝐴2 − 𝑥 2 , antiphase.
𝑣 = 5√102 − 62 = 5√100 − 36 = 5√64 , 14. When the energy of the vibrating skin of a
𝑣 = 5×8=40ms-1. talking drum is decreased, the sound waves
8. A body is esecuting simple harmonic motion as emanating from the drum would have a
an angular speed of 2𝜋rads-1. Its period of corresponding decrease in their A.wavelength
oscillation is (𝜋=3.14) A.0.05s B.1.00s C.2.00s B.amplitude C.quality D.speed.
D.3.14s D.6.28s Amplitude and energy are in phase i.e energy
2𝜋 2𝜋 2𝜋 increases when amplitude increases.
𝜔=2𝜋rads-1, 𝑇=?, 𝜔 = , 𝑇 = , 𝑇 = = 1s.
𝑇 𝜔 2𝜋 15. Which of the following statements about the
9. The angular velocity of a body undergoing motion of a simple pendulum is true? A.It is a
simple harmonic motion with acceleration 𝑎 and simple harmonic motion when the angle of
𝑎 𝒂 𝑥 𝑥 𝑎2 displacement is large B.It passes the equilibrium
displacement 𝑥 is A. B.√ C. D.√ E.
𝑥 𝒙 𝑎 𝑎 𝑥 position with minimum speed C.It possesses
𝑎
𝑎 = 𝜔2 𝑥, 𝜔2 = , 𝜔 = √ .
𝑎 maximum kinetic energy at the extreme
𝑥 𝑥 positions D.It swings faster at the poles than at
10. A block of mass 4.0kg causes a spiral spring the equator
to extend by 0.16m from its unstretched position. Motion of a simple pendulum is simple harmonic
The block is removed and another body of mass motion when the angle of displacement is small.It
0.50kg is hung from the same spiral spring. If the passes the equilibrium position with maximum
spring is then stretched and released, what is the speed. Kinetic energy is minimum at the extreme
angular frequency of the subsequent motion? positions but maximum at the equilibrium
(g=10ms-2) A.10√𝟓rads-1 B.5√2rads-1 C.5rads-1 position. 𝑓 ∝ √𝑔 , 𝑓 increases and swings faster
D.√5rads-1. as g increases. g is greater at poles and less at the
First calculate for the force constant 𝑘 of the equator, hence a pendulum will have a lower
spiral spring using the data given in the first period and higher frequency (swings faster) at
sentence. 𝑚1=4kg, 𝑒=0.16m, g=10ms-2, the poles than at the equator.
𝑚 𝑔
𝐹 = 𝑘𝑒 , 𝐹 = 𝑊 = 𝑚1𝑔 , ∴ 𝑚1𝑔 = 𝑘𝑒 , 𝑘 = 1 , 16. Which of the following processes will
𝑒
4×10 increase the rate of oscillation of a pendulum
𝑘= = 250Nm-1. On placing the second body:
0.16 A.increasing the length of the pendulum
𝑚2=0.50kg, 𝑘=250Nm-1, 𝜔 = √
𝑘
=√
250 B.decreasing the length of the pendulum
𝑚2 0.5 C.increasing the mass of the pendulum
𝜔 = √500 = 10√5rads-1. D.decreasing the mass of the pendulum
11. The bob of a simple pendulum of mass E.decreasing the amplitude of oscillation of the
0.025kg is displaced 0.1m from its equilibrium pendulum.
position.If the angular frequency is 4rads-1 and Rate of oscillation or frequency is inversely
assuming SHM,calculate the energy of the system proportional to square root of the length of a
A.5×10-4𝐽 B.2×10-3𝑱 C.5×10-3𝐽 D.7.9×10-3𝐽 pendulum i.e 𝑓 ∝
1
, the rate of oscillation or
𝑚=0.025kg, 𝐴=0.1m, 𝜔=4rads-1, 𝐸=?, √𝐿
1 1 1 frequency of a pendulum increases when the
𝐸 = 𝐾𝐴2= 𝑚𝜔2𝐴2 = ×0.025×42×0.12, length decreases.
2 2 2
𝐸 = 2.0×10-3𝐽. 17. Which of the following affects the period of a
12. The amplitude of the motion of a body simple pendulum? I.mass of the pendulum
performing simple harmonic motion decreases II.length of the pendulum III.acceleration due to
with time because A.frictional forces dissipate gravity A.I and II B.I and III C.II and III D.I,II and
the energy of motion B.the frequency of III.
oscillation varies with time C.the period of The period 𝑇 of a simple pendulum is affected by
oscillation varies with time D.energy is supplied the length of pendulum and acceleration due to
by some external agencies 𝐿
Amplitude decreases due energy loses caused by gravity only, 𝑇 = 2𝜋√ .
𝑔
frictional forces i.e damping. 18. Two simple pendula A and B of equal lengths
13. Which of the following with respect to a body and masses 5g and 20g respectively are located
performing simple harmonic motion are in in the same located in the same environment. The
phase? A.Displacement and velocity of the body periods, 𝑇A and 𝑇B of their respective oscillations

35
Demystified Series Physics Demystified by Dr Timothy
1 𝑡1 +𝑡2 +𝑡3 80+72+67 219
are related by the equations A.𝑇A= 𝑇B Mean time 𝑡 = = = = 69.6s,
4 3 3 3
B.𝑇A=4𝑇B C.𝑇A=5𝑇B D.𝑻A= 𝑻B. Mean period 𝑇 = = = 2.32s.
𝑡 69.6
𝑛 30
The period of simple pendulum is depends on the
Alternatively, first calculate their respective
length of the pendulum and acceleration due to
period, then you find the mean period.
gravity(which depends on the location) Hence, 𝑡 80 𝑡 72
two pendula of the same length and in the same 𝑇1= 1 = =2.33s, 𝑇2= 2 = =2.4s,
𝑛 30 𝑛 30
𝑡3 67 𝑇1 +𝑇2 +𝑇3
location i.e g is constant, will have the their 𝑇 3= = =2.23s, Mean period 𝑇 = ,
𝑛 30 3
period will be the same, 𝑇A= 𝑇 B. 2.33+2.4+2.23 6.96
𝑇= = = 2.32s
19. The diagram below represents a system of 3 3
simple pendula of different lengths. If 𝑃 is set into 24. A simple pendula 0.64m long has a period of
oscillation while others are forced by it to 1.2s. Calculate the period of a similar pendulum
oscillate, which of the pendula will resonate 0.36m long in same location A.0.7s B.0.9s C.1.1s
with P? D.1.8s E.3.2s.
𝑇1=1.2s, 𝐿1=0.64m, 𝑇2=?, 𝐿2=0.36m,
P A B 𝑇1 𝐿1 𝑇1 2 𝐿1
𝑇 ∝ √𝐿 , =√ or = ,
C D . 𝑇2 𝐿2 𝑇2 2 𝐿2

Simple pendula with equal lengths vibrate in 𝑇1 𝐿 1.2 0.64 1.2 0.8
=√ 1, =√ , = ,
resonance with large amplitude while ones with 𝑇2 𝐿2 𝑇2 0.36 𝑇2 0.6
different lengths vibrates with small swings. 𝑇2 =
0.6×1.2
= 0.9s.
0.8
Pendulum B has the same length 𝑃 and will have 25. If in a simple pendulum experiment the
the same natural frequency, hence vibrate with length of the inextensible string is increased by a
large swing due to resonance. factor of 4,its period is increased by a factor of
20. The length if a displaced pendulum bob A.4 B.𝜋/2 C.1/4 D.2𝜋 E.2.
which passes its lowest point twice every second
𝑇1 𝐿 𝑇 𝐿
is A.0.25m B.0.45m C.0.58m D.1.00m (g=10ms-2 𝑇1=𝑇, 𝐿1=𝐿, 𝑇2=?, 𝐿2=4𝐿, =√ 1, =√
𝑇2 𝐿2 𝑇2 4𝐿
, 𝜋2≈10)
The time taken for a pendulum to pass the lowest 𝑇 1 1
= √ = , 𝑇2 =2𝑇 , increased by a factor of 2.
𝑇2 4 2
point twice equal to the Period. 𝑇=1s,
𝐿 𝐿 𝐿 10 Alternatively, 𝑇 ∝ √𝐿 , 𝐿 is increased by a factor
𝑇 = 2𝜋√ , 𝑇2 = 4𝜋2 , 1= 4𝜋2 , 𝐿= , of 4, 𝑇 = √4 , 𝑇 =2 i.e increased by a factor of 2.
𝑔 𝑔 10 4𝜋2

𝐿=
10
= 0.25m. 26. A simple pendulum 0.6m long, has a period of
4×10 1.5s. What is the period of a similar pendulum
21. The bob of a simple pendulum takes 0.25s to
0.4m long in the same location? A.1.5√(𝟐/𝟑)s
swing from its equilibrium position to one
extreme end. Calculate its period. A.2.5s B.2s B.1.5√(3/2)s C.2.25s D.1.00s E.2.00s.
C.1.5s D.1s . 𝑇1=1.5s, 𝐿1=0.6m, 𝑇2=?, 𝐿2=0.4m,
𝑇1
=√
𝐿1
,
Period of a pendulum is the time taken for one 𝑇2 𝐿2

complete oscillation. If it takes 0.25s to move 1.5 0.6 0.4 2


=√ , 𝑇2 = 1.5√ = 1.5√ s.
from the equilibrium position to the extreme end, 𝑇2 0.4 0.6 3
the period= 4×0.25 = 1s. 27. The period of a simple pendulum of length
22. The period of a simple pendulum X is 5s. 80.0cm was found to have doubled when the
What is the period of a simple pendulum Y which length of the pendulum was increased by 𝑥.
makes 50 vibrations in the same time it takes X to Calculate 𝑥 A.26.7cm B.40cm C.160.0cm
make 20 vibrations? A.12.5s B.2.5s C.2.0s D.1.2s. D.240.0cm.
𝑇1=5s, 𝑛1=20 vibrations, 𝑇2=?, 𝑛2=50 vibrations, 𝑇1=𝑇, 𝐿1=80cm, 𝑇2=2𝑇, 𝐿2=(80+𝑥)cm,
𝑡 𝑇1 2 𝑇2
Period, 𝑇 = . if two pendula vibrate at the same =
𝐿1
, =
80
,
1
=
80
,
𝑛 𝑇2 2 𝐿2 (2𝑇)2 80+𝑥 4 80+𝑥
1
time hence, 𝑇 ∝ , 𝑡=constant, 𝑇1𝑛1= 𝑇 2𝑛2, 80+𝑥 = 4×80 = 320, 𝑥 = 320−80 = 240cm.
𝑛
5×20 = 𝑇2×50, 𝑇2= = 2s.
100 28. Which of the following assumptions is made
50 in a simple pendulum experiment? The
23. A boy times 30 oscillations of a certain
A.suspending string is inextensible B.bob has
pendulum thrown and obtained 1min 10s, 1min
finite size C.bob has a definite mass D.initial angle
12s and 1min 7s respectively. The mean peiod of
of oscillation must be large.
oscillation of the pendulum is A.0.14s B.0.43s
29. The diagram below illustrates the
C.2.32s D.6.97s
equilibrium position Q and the
𝑛=30 oscillations, 𝑡1=1min 10s = 60+10 = 80s,
extreme positions P and R of a
𝑡2=1min 12s=60+12=72s, 𝑡3=1min 7s = 60+7 =
swinging pendulum bob. Which
67s. Find the mean time and divide by the
P Q R of the following statements about
number of oscillations to get the mean period.
the bob is not correct? A.the kinetic energy of

36
Demystified Series Physics Demystified by Dr Timothy
the bob at P is equal to that at Q B.the bob 34. The length of a simple pendulum is plotted
possesses no kinetic energy at R C.the bob at P against the square of the corresponding period.
has its maximum potential energy at Q D.the The slope of the graph is equal to A.
𝑔2
B.
𝜋 4𝜋2
C.
mass of the bub at 𝑃 is equal to its maximum at Q 4𝜋 4𝑔2 𝑔
𝒈
E.the mechanical energies at P, Q and R are equal. D. 𝟐 .
𝟒𝝅
Potential energy or kinetic energy at P, Q and R is 𝐿 𝐿 𝑔
constant. The total mechanical energy at P, Q and 𝑇 = 2𝜋√ , 𝑇2 = 4𝜋2 , 𝐿=( ) 𝑇 2 , 𝐿 ∝ 𝑇 2.
𝑔 𝑔 4𝜋2
R is also constant. The bob has maximum The slope of the graph is 2 .
𝑔

potential energy and minimum (or zero) kinetic 4𝜋


35. An object of weight 120N vibrates with a
energy at the extreme positions(P and R). The
period of 4.0s when hung from a spring .
bob has maximum kinetic energy and minimum
Calculate the force per unit length of the spring
(or zero) potential energy at the equilibrium
[g=10ms-2, 𝜋=3.142] A.7.4Nm-1 B.9.9Nm-1
position (Q). The mass of the bob is constant i.e
C.14.8Nm-1 D.29.6Nm-1.
does not change. The bod has maximum speed at
𝐿
Q and minimum speed at P and R. 𝐹=W=120N, 𝑇=4.0s, g=10ms-2, 𝐹/𝐿=?, 𝑇 = 2𝜋√
𝑔
30. The mass of a loaded spiral spring oscillates 𝐿 𝑔 10 10×16
vertically between two extreme positions P and 𝑇2 = 4𝜋2 , 𝐿 = ( ) 𝑇2 = × 42 = ,
𝑔 4𝜋2 4×3.1422 39.49
R equidistant from the equilibrium position Q. 𝐿 = 4.05m. Force per unit length, = ,
𝐹 120
Which of the following statements about the 𝐿 4.05
system is not correct? A.The momentum of the 𝐹 = 29.6Nm-1.
mass is maximum at Q B.The elastic potential 36. Marching soldiers crossing a suspension
energy of the spring is maximum at P C.The bridge are usually advised to break their step to
kinetic energy is maximum at P D.The total avoid damaging the bridge owing to
energy of the system is always constant. A. resonance B. swinging C. vibration D.
Velocity and kinetic energy is maximum at the oscillation.
equilibrium position Q and minimum at the Walking in steps or uniformly sets the bridge to
equilibrium positions P and R. Momentum is vibrate at the same frequency as that of the
proportional to the velocity (𝑃 ∝ 𝑣, m=constant) people walking or matching (resonance), hence
thus, the momemtum is maximum at Q and causes the bridge to collapse.
minimum at P and R. Potential energy is 37. Which of the following graphs correctly
maximum at the extreme positions P and R. The represents the relationship between kinetic
total mechanical energy of the system is always energy (𝐾. 𝐸) and the displacement (𝑑) from the
constant. point of release of a pendulum bob?
31. A swinging pendulum between the rest A. 𝐾. 𝐸 B. 𝑲. 𝑬
position and its maximum displacement
possesses A.kinetic energy only B.potential
energy only C.gravitational energy only D.both
kinetic and potential energy. 𝑑 𝒅
32. A bob of weigt 0.1𝑁 hangs on a massless C. 𝐾. 𝐸 D. 𝐾. 𝐸
string of length 50cm. A variable horizontal force
which increases from zero is applied to pull the
bob until the string makes an angle of 60° with
the vertical. The workdone is A.0.025J B.0.050J 𝑑 𝑑.
C.0.500J D.0.250J. The kinetic energy in SHM e,g swinging simple
𝑊=𝑚𝑔=0.1𝑁, 𝐿=50cm=0.5m, θ=60°, 𝐸=?, pendulum , is maximum at the equilibrium
The workdone or energy stored in a simple position and minimum at the two extreme
pendulum , 𝐸 = 𝑚𝑔𝐿(1 − 𝑐𝑜𝑠θ) positions as seen in the graph in option B.
𝐸 = 0.1×0.5(1−𝑐𝑜𝑠60°) = 0.05(1−0.5),
𝐸 = 0.05×0.5 = 0.025𝐽. Jamb past questions on Motion :
33. A body of mass 40g is suspended from the [1987/3,4,7,11,1979/49,1980/12,23,1981/25,
end of spiral spring whose force constant is 46,1982/11,12,13,34,1983/30,1984/1,26,36,38,
0.8Nm-1. If it executes simple harmonic motion 1985/2,4,5,9,12,1986/7,13,16,1987/3,4,1988/
and has an amplitude of 0.3m, calculate the total 1,6,7,1989/6,1990/4,5,6,1991/3,4,5,1992/2,4,5,
energy of the system A.0.012J B.0.036J C.0.072J 1993/3,5,6,1994/5,6,7,1995/5,6,1997/2,4,6,5,7,
D.0.192J E.0.240J. 1998/5,7,9,1999/2,6,8,2000/7,8,10,11,12,2001
𝑚=40g, 𝑘=0.8Nm-1, 𝐴=0.3m, 𝐸=?, /9,2002/1,2003/7,10,2004/14,2005/3,28,2006
1 1 /13,19,2007/44,48,50,2008/5,6,10,2009/4,5,7,
𝐸 = 𝑘𝐴2 = ×0.8×0.32 = 0.4×0.09 = 0.036J.
2 2 8,9,2011/7,2012/6,7,12,2013/5,6,2014/5,7].

37
Demystified Series Physics Demystified by Dr Timothy

CHAPTER 3 – FRICTION IN SOLIDS, FLUID


FRICTION(VISCOSITY) AND SURFACE TENSION
● Friction – Friction is the force which acts at and kinetic friction is greater than rolling
the surface of separation between two bodies in friction. Hence, it is easier to roll a body than to
contact and tends to oppose the relative slide it on the ground and it is easier to keep a
motion(not the force) between both bodies i.e. it body in motion that to get a body at rest or
acts in a direction opposite motion. stationary in motion.
- Friction acts tangential to the surface of - Coefficient of limiting or static friction is
separation between two bodies in contact. greater coefficient of kinetic or sliding
- Solid friction – Limiting or static friction, friction.
kinetic or dynamic or sliding friction and rolling - Properties of solid friction :
friction. 1.It depends on the nature i.e roughness or
- Limiting or static friction 𝑭𝒔 is the frictional smoothness,of the surface in contact.
force that must be overcomed for a body at rest 2.It always opposes relative motion of solids.
or stationary to move over another body. 3.It depends on weight of the object/normal
𝑭𝒔 = 𝝁𝒔 𝑹. 𝐹𝑠 =limiting or static frictional force, reaction.
𝑅=normal reaction=weight of the body=𝒎𝒈, 4.It is independent of surface area of the surfaces
𝜇𝑠 =coefficient of limiting or static friction. in contact,relative velocity of the bodies and
- Normal reaction must be perpendicular to the pressure.
surface of contact while frictional force must be 5.Frictional force increases to the same extent as
perpendicular to the normal reaction. the force tending to cause motion i.e preventing
- The frictional force of a body attached to a the body from moving.
spring balance is equal to the spring balance 6.Static friction is always equal to the applied
reading force when the object is stationary or moves with
- The force required to just move/slide a body is a uniform or constant speed.
the static frictional force. 7.Frictional force is proportional the normal
- If the applied force 𝑭 on the rough surface reaction of between the surfaces(or weight of the
exceeds the frictional force 𝑭𝒔 , it causes the body body in contact). The slope of the graph of
to accelerate 𝒂. 𝑭 > 𝑭𝒔 , 𝑭 − 𝑭𝒔 = 𝒎𝒂, frictional force 𝐹𝑅 vs normal reaction 𝑅 is equal
𝑭𝒔 = 𝝁𝒔 𝑹 = 𝝁𝒔 𝒎𝒈, 𝑭 − 𝝁𝒔 𝒎𝒈 = 𝒎𝒂. to coefficient of friction, 𝝁 (it has no unit).
- Kinetic or dynamic or sliding friction 𝑭𝒌 is - The coefficient of friction and friction in a
the frictional force that must be overcomed to perfectly smooth surface is 0 while that of
keep a body moving in uniform or constant perfectly rough surface is 1.
speed. 𝑭𝒌 = 𝝁𝒌 𝑹, 𝐹𝑘 =kinetic or dynamic or - A moving vehicle comes to rest because of the
sliding frictional force, 𝑅=normal reaction, frictional force between the brakes and the
𝜇𝑘 =coefficient of kinetic friction. wheels,frictional force between the tyres and
- If frictional force is applied on a body inorder to road,external force applied opposite to the
bring it to rest or stop after moving a distance 𝒔 direction of motion of the vehicle and the
with an initial speed of 𝒖, then coefficient of resistance of the strong wind blowing against the
dynamic or kinetic friction 𝜇𝑘 is – 𝝁𝒌 =
𝒖𝟐
. vehicle.
𝟐𝒈𝒔 - Advantages of friction :
- Roads are banked in order to reduce the effect I.Friction enables a nail or screw to remain in
of centrifugal force and prevent skidding. In the place when screwed into position.
absence of banking the frictional force between II.Friction is utilised fan and conveyor belts used
the tyres and the roads helps to prevent skidding. driver wheels or pulleys in machinery.
The coefficientof kinetic friction between the tyre III.Friction enables the brakes to stop the car and
of a vehicle moving round a circular track and the the locomotive to pull the train.
𝒗𝟐
road is given by – 𝝁K= 𝒕𝒂𝒏𝜽 = . 𝜃= banking IV.Friction enables motion e.g we walk only with
𝒈𝒓
the help of friction between our feet and the
angle, 𝑣=speed of the vehicle round the circular
ground, automobile tyres are threaded(rough) to
track , 𝑟=radius of the circular track. 𝒗 = √𝝁𝒌 𝒈𝒓 . give it a good grip on road
𝒗𝟐
- Skidding occurs when 𝒕𝒂𝒏𝜽 > . - Disadvantages of friction :
𝒈𝒓
I.Friction lead to wear and tear on moving parts
- Rolling friction is the frictional force that acts
of machines.
on rolling bodies. It has an extremely small value
II.Friction reduces the efficiency of machines as
and might have a negligible effect.
much work is done in overcoming friction
- Static friction is greater than kinetic friction
III.Friction causes the heating of engines.

38
Demystified Series Physics Demystified by Dr Timothy
- Reducing friction : 𝑭𝒖 = 𝒎𝒈(𝒔𝒊𝒏𝜽 + 𝝁𝒌 𝒄𝒐𝒔𝜽) = 𝟐W𝒔𝒊𝒏𝜽.
I.Lubrication : It involves the use of lubricants e.g - This upward force 𝐹𝑢 prevents the body from
grease,oil,air or graphite, in movingparts. slipping or sliding downwards.
II.Balls or roller-bearings : It is based on the fact - The acceleration 𝒂 produced by this upward
that rolling friction is much less than sliding force is – 𝑭𝒖 − 𝒎𝒈(𝒔𝒊𝒏𝜽 + 𝝁𝒌 𝒄𝒐𝒔𝜽) = 𝒎𝒂,
friction i.e prevents surface from sliding. 𝑭𝒖 − 𝟐W𝒔𝒊𝒏𝜽 = 𝒎𝒂.
III.Streamlining : The shape of cars, ships and ● Tension forces 𝑻 on a string connecting
aeroplanes are streamlined to lessess friction bodies in equilibrium – The tension on a string
between them and their medium of motion. connecting a bodiesis equal to the weight of the
IV.Polishing the surfaces. bodies the string suspends.
● Friction and Inclined planes – An object of Tension 𝑻1 in the upper string
weight 𝑊(𝑚𝑔) on an inclined plane is acted a set 𝑻1 suspends body 𝑚1, 𝑚2 and 𝑚3.
of forces, given below ; 𝒎1 𝑻1 =W1+W2+W3,
𝑅 𝑻1= 𝒈(𝒎1+𝒎2+𝒎3).
𝐹S 𝑻2 Tension 𝑻2 in the middle string
𝒎2 suspends body 𝑚2 and 𝑚3.
𝑻2 =W2+W3 = 𝒈(𝒎2+𝒎3).
θ θ 𝑻3 Tension 𝑻3 in the lowest string
𝒎3 suspends body 𝑚3 only.
W W𝑐𝑜𝑠θ 𝑻3=W3= 𝒎3𝒈.
1.Weight W(𝒎𝒈) acting vertically downward. ● Systems with common acceleration : When
2.Normal reaction 𝑹 acting upwards, two bodies are connected by an inextensible
perpendicular or normal to the plane. string of tension 𝑻, both bodies will move with a
3.Component of weight along or parallel to common acceleration 𝒂.
the plane W𝒔𝒊𝒏θ or 𝒎𝒈𝒔𝒊𝒏θ which naturally - In general, common acceleration 𝒂 is given
tends to make the object slide down the plane. by – Larger w.t − Smaller w.t = 𝒎𝒂,
4.Component of weight perpendicular or 𝒎 = 𝒎1+𝒎2. If the mass 𝑚1 is on a rough
normal to the plane W𝒄𝒐𝒔θ or 𝒎𝒈𝒄𝒐𝒔θ which surface, multiply its weight by the coefficient of
counterbalances the normal reaction. friction 𝜇 of the surface. If the mass 𝑚1 is on an
𝑹 = W𝒄𝒐𝒔θ = 𝒎𝒈𝒄𝒐𝒔θ. inclined plane, multiply its weight by sinθ.
- When the body is at rest or stationary in an - Common acceleration is equal to 0,at the
inclined plane, the limiting or static frictional verge or point of slidding.
force 𝐹𝑠 equals the componeent of weight along I. Air-track glider system – It has a mass 𝑚1
the plane. 𝑭𝒔 = W𝒔𝒊𝒏𝜽 = 𝒎𝒈𝒔𝒊𝒏𝜽. gliding on a horizontal track via an inextensible
- Gradual increase in the angle of inclination 𝜃 string to another mass 𝑚2 hanging in air.
causes the object to be at the verge of sliding or 𝑇
just about to slide downwards. The angle at 𝑚1
which this occurs is called the angle of friction 𝑇
and limiting frictional force is maximum at
this point. The coefficicent of static friction is 𝑚2
given by – 𝝁𝒔 = 𝒕𝒂𝒏𝜽, 𝜃=angle of friction. If the horizontal surface is rough i.e has
- The upward force 𝑭𝑺𝑻 required to keep the body coefficient of friction 𝝁, the weight of 𝑚1
at rest or stationary thereby preventing it from becomes 𝝁𝒎1𝒈. The tension 𝑻 can be given by
acceleration downwards is given by – : 𝒎2𝒈 − 𝑻 = 𝒎2𝒂, or 𝑻 − 𝒎1𝒈 = 𝒎1𝒂 (when the
𝑭𝑺𝑻 = W𝒔𝒊𝒏𝜽 − 𝑭𝒔 , 𝑭𝑺𝑻 = 𝒎𝒈𝒔𝒊𝒏𝜽 − 𝝁𝒔 𝑹 , surface is smooth) or 𝑻 − 𝝁𝒎1𝒈 = 𝒎1𝒂(when
𝑭𝑺𝑻 = 𝒎𝒈𝒔𝒊𝒏𝜽 − 𝝁𝒔 𝒎𝒈𝒄𝒐𝒔𝜽 , the surface is rough).
𝑭𝑺𝑻 = 𝒎𝒈(𝒔𝒊𝒏𝜽 − 𝝁𝒔 𝒄𝒐𝒔𝜽). II. Inclined planes – It has a mass 𝑚1 resting on
- The acceleration 𝒂 of a body down a smooth an inclined plane inclined at an angle to the
inclined plane(frictional force is zero), at an angle horizontal and another mass 𝑚2 hanging in air
to the horizontal is – 𝒂 = 𝒈𝒔𝒊𝒏𝜽. 𝑇
- The acceleration 𝒂 of a body down a rough 𝑇
inclined plane is – 𝒂 = 𝒈(𝒔𝒊𝒏𝜽 − 𝝁𝒌 𝒄𝒐𝒔𝜽).
- When the body is moved up the inclined plane, θ 𝑚2
the applied force 𝑭𝒖 moving the body upward the Weight is 𝒎2𝒈 and 𝒎1𝒈𝒔𝒊𝒏θ (if the plane is
balances the combined effect of component of smooth) or 𝝁𝒎1𝒈𝒔𝒊𝒏θ (if the plane is rough), 𝒎
weight along the plane 𝑊𝑠𝑖𝑛θ and the frictional is the sum of both masses=𝒎1+𝒎2. The tension
force(kinetic or sliding friction) 𝐹𝑘 both acting can be given by : 𝑻 − 𝒎1𝒈𝒔𝒊𝒏θ= 𝒎1𝒂(if the
downward. 𝑭𝒖 = W𝒔𝒊𝒏𝜽 + 𝑭𝒌 . plane is smooth) or 𝑻 − 𝝁𝒎1𝒔𝒊𝒏θ ( if the plane is
𝑭𝒖 = 𝒎𝒈𝒔𝒊𝒏θ+𝝁𝒌 𝒎𝒈𝒄𝒐𝒔θ rough).

39
Demystified Series Physics Demystified by Dr Timothy
III. Pulley system – It has two masses 𝑚1 and 𝑚2 frictional force between two rough surfaces is
hanging in air over a pulley. greater than that between two smooth surfaces
C.the frictional force between two surfaces is
independent of the area in contact D.frictional
𝑇 𝑇 force depends on the type of material which
forms the surfaces which are in contact
𝑚1 𝑚2 E.frictional force is proportional to the normal
Weight is 𝒎1𝒈 and 𝒎2𝒈. If the horizontal surface reaction on one of the surfaces.
is rough i.e has coefficient of friction 𝝁, the The frictional force between two solid surface is
weight of 𝑚1 becomes 𝝁𝒎1𝒈. The tension 𝑻 can lesser when one of the objects is moving(sliding
be given by : 𝒎2𝒈 − 𝑻 = 𝒎2𝒂 or 𝑻 − 𝒎1𝒈 = 𝒎1𝒂 friction) than when both are stationary(static
(when the surface is smooth) or 𝑻 − 𝝁𝒎1𝒈 = friction).
𝒎1𝒂 (when the surface is rough). 2. Which of the following statements about
friction is not correct? A.cars are less likely to
● Lifts or elevators skid on wet roads than on dry roads B.nails
- Weight of a body in a lift or elevator – The hold boards together by friction C.limiting
body in a lift or elevator is acted upon by two friction is directly proportional to the force
forces : acting between two surfaces D.sliding friction is
I.Reaction 𝑹 or Tension 𝑻 in the cord of the lift less than limiting friction E.lubrication reduces
or apparent weight of the body which acts friction
upwards from the floor of the lift unto the body. Cars are more likely to skid on wet roads than on
II.Weight 𝑾(𝒎𝒈)which acts downwards. dry roads due to friction between the tyres and
1.If the lift is at rest or moving upwards or roads. All other options are correct.
downwards with uniform velocity i.e 3. The limiting frictional force between two
acceleration is zero., the apparent weight is equal surfaces depends on I.the normal reaction
to the weight of the body. between the surfaces II.the area of surfaces in
𝑹 = 𝑾 = 𝒎𝒈. contact III.the relative velocity between the
2.If the lift is accelerating upwards(ascending) surfaces IV.the nature of the surfaces A.I B.II C.III
the reaction force(acting upward) is greater than D.I and IV E.I and III.
the weight(acting downward),then the apparent Solid friction (limiting frictional force) depends
weight is more than the weight of the body. on the normal reaction between the surfaces and
𝑹 − 𝒎𝒈 = 𝒎𝒂, 𝑹 = 𝒎𝒂 + 𝒎𝒈, 𝑹 = 𝒎(𝒈 + 𝒂). the nature of the surfaces. It is independent of
It also occurs when the lift decelerates to rest surface area in contact and relative velocity
from a steady speed during downward between the surfaces.
motion. 4. Which of the following statements are true
3.If the lift is accelerating downwards about frictional force? I.It is always a
(decending), the reaction force(acting upward) is disadvantage II.It is sometimes a disadvantage
less than the weight(acting downward),then the III.It always exists where there is a relative
apprarent weight is less than the weight of the motion of bodies in contact IV.It is sometimes
body. 𝒎𝒈 − 𝑹 = 𝒎𝒂, 𝑹 = 𝒎𝒈 − 𝒎𝒂, very useful A.I and II B.I and IV C.II and III D.II
𝑹 = 𝒎(𝒈 − 𝒂). It also occurs when the lift and IV.
decelerates to rest from a steady speed Friction is not always a disadvantage it can also
during upward motion. be an advantage. Friction does not always exists
4.If the lift falls freely at the same rate of g when where there is a relative motion, it also exists
the cord of the lift is cut,then the apparent weight when there is a tendency of motion.
is zero i.e 𝒂 = g and the body experience 5.The cofficient of frictionbetween two perfectly
weightlessness. smooth surface is A.one B.half C.zero D.infinity.
- Upward deceleration or retardation gives 6. Which of the following statements are true of
the same reading as downward acceleration friction? i.frictional force opposes motion
while downward deceleration or retardation ii.rolling friction between two surfaces is greater
gives the same reading as upward than sliding friction between the same two
acceleration. surfaces III.coefficient of friction is proportional
to the limiting frictional force IV.it will be
Examples : impossible to stop a moving car if there were no
1. Which of the following statements is not true friction between the road and the tyres A.I,II,III
of friction between two solids surfaces? A.the and IV B.I,III and IV C.I and IV D.I,II and IV E.II,III
frictional force between two solid surfaces is and IV.
greater when one of the objects is moving Rolling friction is less than sliding friction.
than when they are both stationary B.the Coefficient of friction is the ratio of the frictional

40
Demystified Series Physics Demystified by Dr Timothy
force to the normal reaction. It is limiting The force of friction I.is directed to the right II.is
frictional force that is proportional to the normal directed to the left III.increases as 𝐹 increases
reaction. until the block starts to move IV.increases if a
7. Which of the following could be effectively second block is placed on top of the block A.I and
used to reduce friction? A.water B.petrol III B.II and IV C.II and III D.II,III and IV.
C.grease D.kerosine. Frictional force opposes motion i.e acts in a
8. Which of the following statements about direction opposite that of the force causing
friction is not correct? A.The force of kinetic motion.Hence force of friction is directed to the
friction is less than that of static friction B.The left as 𝐹 is directed right. Frictional force
force of kinetic friction between two surfaces is increases to the same extent as the force 𝐹
independent of the area in contact provided the causing motion until motion starts. If a second
normal reaction is unchanged C.The force of identical block is placed on the block, the weight
rolling friction between two surface is less than W doubles,hence normal reaction 𝑅 also doubles,
the force of sliding friction D.The angle of thereby increasing the frictional force as 𝐹𝑅 ∝ 𝑅.
friction is the angle between the normal 14. A body of mass 6.0kg rests on a rough
reaction and the force of friction E.Friction horizontal surface where the coefficient of static
may be reduced by lubrication friction is 0.25. Determine the magnitude of the
Angle of friction θ is the angle between the plane limiting frictional force (g=10ms-2) A.240.0N
and the horizontal or perpendicular component B.150.0N C.60.0N D.15.0N.
of the weight and the weight at which static 𝑚=6kg, 𝜇𝑠 =0.25, g=10ms-2, 𝑅=𝑊=𝑚𝑔, 𝐹𝑠 =?,
friction is maximum. 𝐹𝑠 = 𝜇𝑠 𝑅 = 𝜇𝑠 𝑚𝑔 = 0.25×6×10 = 15N.
9. The frictional force between two bodies 15. A metal block of mass 7.5kg lies on a rough
A.exists only when there is relative motion or horizontal platform. If a horizontal force of 12𝑁
tendency of motion B.acts so as to oppose the is applied to the block through its centre of mass,
motion C.depeds on the normal reaction between just slides the block, calculate the coefficient of
the two surfaces D.has all of these limiting friction between the block and the
characteristics E.is relatively indendent of the platform (g=10ms-2) A.0.16 B.0.63 C.1.60 D.3.66
area in contact. E.6.33 .
10. The surface of conveyor belts are made rough The horizontal force just required to slide the
so as to A.prevent the load from slipping block is equal to the frictional force. 𝐹 S=12𝑁,
B.make the stronger C.enable them to carry more 𝐹
𝑚=7.5kg, 𝜇𝑠 =?, 𝐹𝑠 = 𝜇𝑠 𝑅, 𝜇𝑠 = 𝑠 = 𝑠 ,
𝐹
𝑅 𝑚𝑔
load D.protect the while carrying loads 12
Rough surfaces enhances friction in the belts, 𝜇𝑠 = = 0.16.
7.5×10
hence ensuring a grip on the load preventing the 16. Spring balance
from slipping. 𝐹
11. When the brakes in a car are apllied, the
frictional force on tyres is A.disadvantage 25N
because it is in the direction of motion of the car In the above diagram the block just moves when
B.a disadvantage because it is opposite in the spring balance reads 15.0N. What is the value
direction of motion of the car C.an advantage of 𝐹? A.0.6N B.10.0N C.15.0N D.40.0N E.375.0N.
because it is in the direction of motion of the car 𝐹 is the frictional force as it acts opposite to the
D.an advantage because it is in opposite direction of motion. The frictional force of a body
direction of motion of the car. attached to a spring balance is equal to the spring
Frictional force acts opposes motion by acting in balance reading hence, 𝐹 =15N.
opposite direction to motion. - If the question had asked us to calculate the
12. Which of the following statements is not coefficient of friction of the surface in which
correct? A moving vehicle comes to rest as a the block moves it given as follows :
result of A.the hot gases coming out of the 𝐹
𝑅 = 𝑊 = 25𝑁, 𝜇𝑠 = 𝑠 = = 0.6.
15
𝑅 25
exhaust pipe B.the frictional force between the
17. A child pulls a toy car weighing 3.00kg along
brakes and the wheels C.the frictional force
a cement floor. If g=10ms-2 and the coefficient of
between the tyres and the surface oon which the
friction is 0.500, the minimum force required to
vehicle is moving D.the resistance of a strong
move the car is A.3.0N B.5.0N C.10.0N D.15.0N
nwind blowing against the vehicle E.the force
E.60.0N.
applied in the opposite direction on the motion of
𝑚=3kg, 𝜇𝑠 =0.5, g=10ms-1, 𝐹𝑠 =?,
the vehicle.
𝐹𝑠 = 𝜇𝑠 𝑅 = 𝜇𝑠 𝑚𝑔 = 0.5×3×10 = 15N.
13. Block 𝐹
18. A wooden block of weight 16𝑁 is placed on a
A block at rest on a horizontal surface is pulled by
rough surface. If the coefficient of friction
a horizontal force 𝐹 as shown in the figure above.
between both surfaces is 0.25, the least
Which of the following statements are correct?

41
Demystified Series Physics Demystified by Dr Timothy
horizontal force required to move the block is A block of wood of mass 5kg is pulled on a
A.0.4N B.4.0N C.6.4N D.64.0N. platform by force of 40N as illustrated in the
𝑊=𝑅=16𝑁, 𝜇𝑠 =0.25, the least horizontal force diagram above. If the frictional force, 𝐹
required to move the block is equal to the experienced by the block is 12N, calculate the
frictional force. 𝐹𝑠 =?, 𝐹𝑠 = 𝜇𝑠 𝑅 = 𝜇𝑠 𝑊, magnitude of the acceleration of the block
𝐹𝑠 = 0.25×16 = 4N. A.2.4ms-2 B.5.6ms-2 C.8.0ms-2 D.10.4ms-2
19. A boy pushes a 500kg box along a floor with 𝐹𝑠 =12𝑁, 𝐹=𝑃=40𝑁, 𝑚=5kg, 𝑎=?,
a force of 2000N. If the velocity of the box is 𝐹 − 𝐹𝑠 = 𝑚𝑎 , 40− 12 = 5𝑎, 28 = 5𝑎,
uniform, the coefficient of friction between the 28
𝑎 = = 5.6ms-2.
5
box and the floor is A.1.0 B.0.8 C.0.65 D.0.5 E.0.4.
26. A motorcycle of mass 100kg moves round in
The frictional force acting on the box is kinetic or
a circle of radius 10m with a velocity of 5ms-1.
dynamic or sliding friction, as it is moving with a
Find the coefficient of friction between the road
uniform velocity, 𝑚=500kg, g=10ms-2,
𝐹𝑘 𝐹 and the tyres (g=10ms-2) A.25.00 B.2.50 C.0.50
𝐹𝑘 =2000𝑁, 𝜇𝑘 =?, 𝐹𝑘 = 𝜇𝑘 𝑅, 𝜇𝑘 = = 𝑘 , D.0.25.
𝑅 𝑚𝑔
𝜇𝑘 =
2000
= 0.4. 𝑚=100kg, 𝑟=10m, 𝑣=5ms-1, g=10ms-2, 𝜇𝑘 =?,
500×10 𝑣2 52 25
20. A body of mass 25kg moving at 3ms-1 on a 𝜇𝑘 = = = =0.25.
𝑔𝑟 10×10 100
rough horizontal floor is brought to rest after 27. A concrete block of mass 35kg is pulled along
sliding through a distance of 2.50m on the floor. a horizontal floor with the aid of a rope inclined
Calculate the coefficient of sliding friction at an angle of 30° to the horizontal. If the
(g=10ms-2) A.0.09 B.0.18 C.0.36 D.0.54 . coefficient of friction is 0.75, calculate the force
𝑚=25kg, 𝑢=3ms-1, 𝑠=2.5m, g=10ms-2, 𝜇𝑘 =?, required to move the block over the floor. A.
𝑢2 32 9 327.77N B.276.86N C.211.52N D.428.65N.
𝜇𝑘 = = = =0.18.
2𝑔𝑠 2×10×2.5 50
The inclined force 𝐹𝑝 at an angle 𝜃 required to
21. A block of mass 50kg is given an initial speed
move a block of weight 𝑊(𝑚𝑔) along a horizontal
of 5ms-1 on a rough horizontal floor. If it slides on
floor of coefficient of friction 𝜇 S is given by –
the floor for 3m before it stops, calculate the 𝝁𝒔 𝑾
coefficient of kinetic friction between the block 𝑭𝒑 = . 𝜇𝑠 =0.75, 𝑊=𝑚𝑔=35×10=350N,
𝝁𝒔 𝒔𝒊𝒏𝜽+𝒄𝒐𝒔𝜽
3 1 𝟓 2 0.75×350 262.5 262.5
and the floor (take g=10ms-1) A. B. C. D. 𝜃=30°, 𝐹𝑝 = = = ,
10 30 𝟏𝟐 3 0.75𝑠𝑖𝑛30°+𝑐𝑜𝑠30° 0.375+0.866 1.241
5
E. . 𝐹𝑝 = 211.52N.
6
𝑚=50kg, 𝑢=5ms-1, 𝑠=3m, g=10ms-2, 𝜇𝑘 =?, The diagram above illustrates a ball of mass 𝑚
𝑢2 32 25 5 sliding down a plank inclined at angle of θ to the
𝜇𝑘 = = = = . horizontal. The kinetic friction between the ball
2𝑔𝑠 2×10×2.5 60 12
22. A moving car of mass 800kg experiences a and the plank 𝐹 and acceleration of free fall is g.
frictional force of 200N. If it accelerates at 2ms-2, 𝐹
calculate the magnitude of the force applied to
the car A.600N B.1000N C.1600N D.1800N . θ
𝑚=800kg, 𝐹 S=200N, 𝑎=2ms-1, 𝐹=?, Use the information to answer questions 28 and
𝐹 − 𝐹𝑠 = 𝑚𝑎, 𝐹 = 𝑚𝑎 + 𝐹𝑠 =800×2+200 , 29.
𝐹 =1600+200 =1800N. 28.The normal force on the ball is A.𝑚𝑔𝑠𝑖𝑛θ
23. A block weighing 15N rests on a flat surface B.𝑚𝑔𝑡𝑎𝑛θ C.𝒎𝒈𝒄𝒐𝒔θ D.𝑚𝑔𝑐𝑜𝑡θ
and a horizontal force of 3N is exerted on it. Normal force or normal reaction 𝑅 is equal to the
Determine the frictional force on the block. component of weight perpendicular to the plane.
A.12N B.15N C.5N D.3N. 𝑅 = 𝑊𝑐𝑜𝑠θ = 𝑚𝑔𝑐𝑜𝑠θ.
Frictional force opposes motion and is equal to 29. The net force on the ball along the plank is
the force tending to cause motion when the body A.𝑚𝑔𝑠𝑖𝑛θ+𝐹 B.𝐹 − 𝑚𝑔𝑐𝑜𝑠θ C.𝒎𝒈𝒔𝒊𝒏θ−𝑭
is stationary. D.𝐹 + 𝑚𝑔𝑐𝑜𝑠θ.
24. A force of 10𝑁 drags a mass 10kg on a The forces acting along the plane are –
horizontal table with an acceleration of 0.2ms-2. component of weight along the plane𝑚𝑔𝑠𝑖𝑛θ
If the acceleration due to gravity is 10ms-2, the acting downwards and the frictional force 𝐹
coefficient of friction between the moving mass acting upwards. Since the ball is sliding down the
and the table is? A.0.02 B.0.08 C.0.20 D.0.80 plane,the component of the weight along the
𝐹=10𝑁, 𝑚=10kg, 𝑎=0.2ms-2, g=10ms-2, 𝜇𝑠 =?, plane must be greater that the frictional force.
𝐹 − 𝐹𝑠 = 𝑚𝑎, 𝐹𝑠 = 𝜇𝑠 𝑅 = 𝜇𝑠 𝑚𝑔., 𝑚𝑔𝑠𝑖𝑛θ > 𝐹, 𝑚𝑔𝑠𝑖𝑛θ−𝐹 = net force ,
𝐹 − 𝜇𝑠 𝑚𝑔 = 𝑚𝑎, 10−𝜇𝑠 ×10×10=10×0.2, 𝑚𝑔𝑠𝑖𝑛θ − 𝐹 = 𝑚𝑎 .
10−100𝜇𝑠 =2, 100𝜇𝑠 = 10−2 = 8 , 30. The limiting frictional force of a mass 4kg
100
𝜇𝑠 = = 0.08. resting on an inclined plane is 20N. The angle of
8
25. 𝐹 5kg 𝑃=40N inclination θ to the plane is A.75° B.60° C.45°
D.30°.

42
Demystified Series Physics Demystified by Dr Timothy
𝑚=4kg, g=10ms-1, 𝐹𝑠 =20N, 𝜃=?, 𝑚=20kg, 𝐹𝑘 =30𝑁, θ=30°, g=9.8ms-2, 𝐹𝑛𝑒𝑡 =?,
When a body is at rest on an inclined plane, the 𝐹𝑛𝑒𝑡 = 𝑚𝑔𝑠𝑖𝑛𝜃 − 𝐹𝑘 = 20×9.8𝑠𝑖𝑛30° − 30 ,
limiting frictional force is completely balanced by 𝐹𝑛𝑒𝑡 =196×0.5−30 = 98−30 = 68N.
the component of the weight of the body along 36. A 50kg crate rests on a platform inclined at
the plane. 𝐹𝑠 = 𝑊𝑠𝑖𝑛𝜃 = 𝑚𝑔𝑠𝑖𝑛𝜃, 30° to the horizontal. The coefficient of froction
20 = 4×10𝑠𝑖𝑛𝜃 , 𝑠𝑖𝑛𝜃 = = 0.5,
20
between the crate and the platform is 0.2. The
40
minimum force needed to prevent the crate from
𝜃 = 𝑠𝑖𝑛−1 0.5 =30°.
sliding down the plane is A.433.0N B.250.0N
31. A body of mass 10kg rests on a rough inclined
C.163.4N D.86.6N.
plane whose angle of tilt 𝜃 is gradually increased
𝑚=50kg , θ=30°, 𝜇𝑠 =0.2. The minimum force
until the body starts to slide down the plane at
needed to prevent the crate from sliding
30°. The coefficient of static friction between the
downwards keeps the crate stationary or at rest.
body and the plane is A.0.30 B.0.50 C.0.58 D.0.87
𝐹𝑆𝑇 = 𝑚𝑔(𝑠𝑖𝑛𝜃 − 𝜇𝑠 𝑐𝑜𝑠𝜃),
𝑚=10kg, 𝜃=30°, 𝜇𝑠 =?, At the verge or point of
𝐹𝑆𝑇 = 50×10(𝑠𝑖𝑛30° −0.2𝑐𝑜𝑠30°),
sliding, 𝜃=angle of friction.
1 𝐹𝑆𝑇 = 500(0.5−0.2×0.866) = 500(0.5−0.1732) ,
𝜇𝑠 = tan 𝜃 = 𝑡𝑎𝑛30° = = 0.58. 𝐹𝑆𝑇 = 500(0.3268) = 163.4N.
√3
32. A load is pulled at uniform speed along a 37. A 5kg block released from rest on a smooth
horizontal floor by a rope at 45° to the floor. If the plane inclined at an angle of 30° to the horizontal.
force in the rope is 1500N, what is the frictional What is its acceleration down the plane A.5.0ms-
force on the load? A.1524N B.1350N C.1260N 2 B.5.8ms-2 C.8.7ms-2 D.25.0ms-2 .

D.1061N . 𝑚=5kg, θ=30°, g=10ms-2, 𝑎=?,


1500𝑁 The acceleration of a body down a smooth
inclined plane is – 𝑎 = 𝑔𝑠𝑖𝑛θ(inclined to the
45° 1500𝑐𝑜𝑠45°
𝐹R horizontal) or 𝑎 = 𝑔𝑐𝑜𝑠θ(inclined to the
When a rope or string is used to pull a body at vertical). 𝑎 = 𝑔𝑠𝑖𝑛θ=10𝑠𝑖𝑛30° = 5ms-2.
angle θ to the floor, the frictional force 𝐹 R is equal 38. A block of mass 2.5kg slides down a plane
to the horizontal component of the tension. inclined at 30° to the horizontal. If the coefficient
𝐹𝑅 = 𝑇𝑐𝑜𝑠𝜃. 𝑇=1500N, 𝜃=45°, 𝐹𝑅 =?, of friction between the surfaces is 0.2, the
𝐹𝑅 =1500𝑐𝑜𝑠45° = 1500(0.7071)= 1060.65N , acceleration of the block is A.3.2ms-2 B.4.2ms-2
𝐹𝑅 =1061N. C.5.2ms-2 D.12.2ms-2 E.21.2ms-2.
33. The diagram below shows a block of wood 𝑚=2.5kg, θ=30°, 𝜇𝑘 =0.2, g=10ms-2, 𝑎=?,
resting on an inclined plane and at the point of The acceleration of a body down a rough inclined
sliding down the plane. plane is – 𝑎 = 𝑔(𝑠𝑖𝑛θ−𝜇𝑘 𝑐𝑜𝑠θ),
𝑎 = 10(𝑠𝑖𝑛30° −0.2𝑐𝑜𝑠30°) =10×0.326,
5cm 𝑎 =3.26ms-2=3.2ms-2.
39. The diagram below shows a body resting on
θ 50N an inclined plane. If the body slides down the
25cm inclined plane, what will be its acceleration?
Calculate the coefficient of friction between the (g=10ms-2)
block and the plane. A.0.1 B.0.2 C.0.3 D.0.4 E.0.5.
At the point of sliding down a plane, θ=angle of
𝑜𝑝𝑝 5 30°
friction. 𝜇𝑠 = 𝑡𝑎𝑛𝜃 = = = 0.2.
𝑎𝑑𝑗 25
15kg
34. A block of mass 50kg sliding down a plane is
A.3.3ms-2 B.6.2ms-2 C.10.5ms-2 D.14.8ms-2
inclined at angle of 30° to the horizontal. If the
E.27.5ms-2.
coefficient of friction is 0.4, calculate the
Since the body is sliding downwards, the 25N
frictional force between te two surfaces
force acting upwards is the frictional force, as it
(g=10ms-2) A.200.00N B.173.21N C.100.00N
opposes the downward motion of the body.
D.62.50N.
𝑚=15kg, g=10ms-2, θ=30°, 𝐹 K=25N, 𝑎=?,
𝑚=50kg, θ=30°, 𝜇𝑘 =0.4(sliding friction), 𝐹𝑘 =?,
𝐹𝑛𝑒𝑡 = 𝑚𝑎 = 𝑚𝑔𝑠𝑖𝑛θ−𝐹𝑘 ,
Net force along the plane pushing the body
15𝑎 =15×10𝑠𝑖𝑛30° −25 =75−25 =50 ,
downwards, 𝐹𝑛𝑒𝑡 = 𝑚𝑎 = 𝑚𝑔𝑠𝑖𝑛𝜃 − 𝐹𝑘 , 50
Perpendicular/normal to the plane, R= 𝑚𝑔𝑐𝑜𝑠θ 𝑎 = = 3.3ms-1.
15
𝐹𝑘 = 𝜇𝑘 𝑅 = 𝜇𝑘 𝑚𝑔𝑐𝑜𝑠𝜃 = 0.4×50×10𝑐𝑜𝑠30° 40. A body of mass 4kg is on the point of slipping
𝐹𝑘 = 200×0.866 = 173.21N. down a plane which is inclined at 30° to the
35. A block of mass 20kg slides down an incline horizontal. What force, parallel to the plane, will
plane of frictional force 30N. Calculate the force just move it up the plane? (g=10ms-2) A.80N
pushing the block down the plane, if the plane is B.60N C.40N D.20N.
inclined at 30° to the horizontal. (take g=9.8ms-2) 𝑚=4kg, θ=30°, g=10ms-2, 𝐹𝑢 =?,
A.196N B.98N C.170N D.68N. The upward force parallel to the plane which
43
Demystified Series Physics Demystified by Dr Timothy
moves the body up the plane (preventing it from = 𝑚1𝑔(smooth surface), smaller weight= 𝑚2𝑔,
slipping donward) : 𝐹𝑢 = 𝑚𝑔(𝑠𝑖𝑛𝜃 + 𝜇𝑘 𝑐𝑜𝑠𝜃) Larger weight−Smaller weight= 𝑚𝑎,
𝜇𝑘 = 𝑡𝑎𝑛𝜃 = 𝑡𝑎𝑛30° =0.58. 6×10−3×10 = (6+3)𝑎, 60−30=9𝑎,
30
𝐹𝑢 = 𝑚𝑔(𝑠𝑖𝑛𝜃 + 𝜇𝑘 𝑐𝑜𝑠𝜃), 30 = 9𝑎 , 𝑎 = = 3.3ms-2.
9
𝐹𝑢 =4×10(𝑠𝑖𝑛30° + 0.58𝑐𝑜𝑠30°) = 40(0.5+0.5)
45. 𝑚1
𝐹𝑢 = 40N. Alternatively, 𝐹𝑢 = 2𝑊𝑠𝑖𝑛𝜃 ,
𝐹𝑢 = 2𝑚𝑔𝑠𝑖𝑛𝜃 = 2×4×10𝑠𝑖𝑛30° = 40N.
41. The figure below shows two
T connected bodies of masses 𝑚2
𝑚1 In the diagram above, the hanging mass 𝑚2 is
𝑚1 and 𝑚2 in equilibrium
adjusted until 𝑚1 is on the verge of sliding. The
under gravity. The tension in
𝑚2 coefficient of static friction between the mass 𝑚1
the upper string is 𝑚 𝑚 𝑔 𝒎 𝑚 𝑔
and the table is A. 1 B. 1 C. 𝟐 D. 2 .
A.(𝑚1−𝑚2)𝑔 B.𝑚1𝑔 C.𝑚2𝑔 D.(𝒎1+𝒎2)𝒈 . 𝑚2 𝑚2 𝒎𝟏 𝑚1
Tension in the string is equal to the weight On the verge of sliding, common acceleration,
suspended below by the string. 𝑇 = 𝑊 1+𝑊 2 , 𝑎=0. 𝑚2 is greater than 𝑚1, since it is adjusted to
𝑇 = 𝑚1𝑔 + 𝑚2𝑔 = (𝑚1+𝑚2)𝑔. make 𝑚1 to be on the verge of sliding.
42. Three bodies A, B and C are Larger weight−Smaller weight= 𝑚𝑎,
connected by light inextensible 𝑚2𝑔 − 𝜇𝑚1𝑔 = (𝑚1+𝑚2)0 = 0,
𝑚 𝑔 𝑚
A 25N strings as shown in the diagram. 𝑚2𝑔 − 𝜇𝑚1𝑔 = 0 , 𝜇𝑚1𝑔 = 𝑚2𝑔, 𝜇 = 2 = 2 .
𝑚1 𝑔 𝑚1
If the weight of the bodies are 46. 𝑇
B 8N 25N, 8N and 5N respectively, 𝑇
what is the tension in the string
C 5N connecting A and B? 30° 30kg
A.13N B.17N C.23N D.33N E.38N. The acceleration of the system above is A.2ms-2
The string connecting A and B suspends weight B B.4ms-2 C.6ms-2 D.8ms-2.
and C. 𝑇 = 8+5 = 13N. 𝑚1=20kg, 𝑚2=30kg, g=10ms-2, θ=30°, 𝑎=?,
43. 2kg Inclined plane system – 𝑚2> 𝑚1, larger weight=
𝑚2𝑔, smaller weight= 𝑚1𝑔𝑠𝑖𝑛θ,
Larger weight−Smaller weight= 𝑚𝑎,
4kg 30×10−20×10𝑠𝑖𝑛30° = (20+30)𝑎,
A mass of 2kg on a surface (𝜇=1/2) is connected 200
to a second mass of 4kg over a a frictionless 300−100 = (50)𝑎 , 200 = 50𝑎, 𝑎 = = 4ms-2.
50
pulley asshown in the diagram. If the acceleration 47.
due to gravity is 9.8ms-2, then the masses will
A.accelerate at 4.9ms-2 B.remain stationary
C.accelerate at 9.8ms-2D.accelerate at 19.6ms-2 30° 3kg
E.accelerate at 39.2ms-2. A 2kg mass on a smooth inclined plane is
𝑚1=2kg, 𝑚2=4kg, g=9.8ms-2, 𝜇=1/2, 𝑎=?, connected over a smoth pulley to a 3kg mass as
Air-track glidder system – 𝑚2> 𝑚1, Larger shown in the diagram above. The acceleration of
weight= 𝑚2𝑔, smaller weight= 𝜇𝑚1𝑔(rough the system is A.g B.3g/4 C.g/2 D.2g/5.
surface). Larger weight−Smaller weight= 𝑚𝑎, 𝑚1=2kg, 𝑚2=3kg, θ=30°, 𝑎=?,
1
4𝑔 − ×2𝑔 =(2+4)𝑎, 4𝑔 − 𝑔 = 6𝑎, 3𝑔 = 6𝑎 , Inclined plane system : 𝑚2> 𝑚1, larger weight=
2
3𝑔 𝑔 9.8
𝑚2g , smaller weight= 𝑚1g𝑠𝑖𝑛θ,
𝑎= = = = 4.9ms-2 , both masses will Larger weight − Smaller weight = 𝑚𝑎,
6 2 2
accelerate at 4.9ms-2. 3g − 2g𝑠𝑖𝑛30° = (2+3)𝑎, 3g − g = 5𝑎,
- The tension 𝑇 in the question : 𝑚2𝑔 − 𝑇 = 𝑚2𝑎, 2g = 5𝑎 , 𝑎 = 2g/5.
𝑇 = 𝑚2(𝑔 − 𝑎) = 4(9.8−4.9) = 4(4.9) = 19.6N. 48.
44. 6kg

3kg
Two blocks of masses 3kg and 6kg are connected 1.5kg 2kg
by a light inextensible string which passes over a Two masses 2kg and 1.5kg are tied to the ends of
smooth pulley as shown in the diagram. light inextensible string which passes over a
Assuming the 6kg mass is sitting on a smooth smooth light pulley which is fixed to the ceiling as
surface, the bodies will accelerate at A.10.0ms -2 shown in the diagram. The acceleration of the
B.6.7ms-2 C.5.0ms-2 D.3.3ms-2. 2kg mass is (g=10ms-2) A.10.00ms-2 B.5.71ms-2
𝑚1=6kg, 𝑚2=3kg, g=10ms-2, 𝑎=?, C.4.29ms-2 D.1.43ms-2.
Air-track glider system : 𝑚1 > 𝑚2, larger weight Larger weight−Smaller weight= 𝑚𝑎,

44
Demystified Series Physics Demystified by Dr Timothy
2×10−1.5×10 = (2+1.5)𝑎, 20−15=3.5𝑎, Apparent loss in weight = 𝑚𝑎 = 70×1.5 = 105N.
5 = 3.5𝑎 , 𝑎 =
5
=1.428=1.43ms-2. 55. An elevator of mass 4800kg is supported by a
3.5
cable which can safely withstand a maximum
49. A man standing on a lift that is descending
tension of 60000𝑁. The maximum upward
does not feel any weight because A.there is no
acceleration the elevator can have is A.2.5ms-2
reaction from the floor of the lift B.there is no
B.5.0ms-2 C.7.5ms-2 D.10.0ms-2 (g=10ms-2)
gravitational pull on the man in the lift C.the
𝑚=4800kg, 𝑅=𝑇=60000N, g=10ms-2, 𝑎=?,
inside of the lift is air tight D.the lift is in vacuum
𝑅 = 𝑇 = 𝑚(𝑔 + 𝑎), a lift ascending.
For a lift descending : 𝑊 > 𝑅 , 𝑊 − 𝑅 = 𝑚𝑎, 60000
𝑊 − 𝑚𝑎 = 𝑅, when there is no reaction on the 60000=4800(10+𝑎), 10+𝑎 = = 12.5,
4800
floor , 𝑅=0, 𝑊 = 𝑚𝑎, the body experiences 𝑎 = 12.5−10 = 2.5ms . -2

weightlessness, as the apparent weight is equal


to zero or weight equals the net force. Jamb past questions on friction in horizontal
50. A man stands on a spring balance placed in a surfaces and inclined planes, lift or elevator :
lift. The lift descends at constant velocity. As a [2012/3,12,13,1990/3,1981/28,1992/8,1991/2
result, the scale reads a weight A.greater than the ,1981/19,1990/11,1995/7,1987/11,1982/48,
weight of a man B.the same as the weight of a 1997/10,14,2005/4,1994/11,1985/8,1995/12,
man C.of zero D.less than the weight of the man 1987/5,1988/12,1989/7,1986/11,2004/16,
E.greater than the weight of the man by 1kg 2005/6,29,2008/132009/11,1978/2,1983/5,
A lift descending or ascending at constant 1995/13,1983/4,1999/18,2000/3,2013/11,
velocity i.e acceleration is zero, will read an 2011/17,2010/9,1982/2,1986/15,1987/6,2002
apparent weight(or tension in the scale) equal to /6,2005/6,2011/3,2013/7,2015/4]
the weight of the body. 𝑊 − 𝑅 = 𝑚𝑎, 𝑎=0, 𝑊 −
𝑅 = 0, 𝑊 = 𝑅. ● Fluid friction and Viscosity
51. A spring balance which is suspended from the - Fluid friction or Viscosity – Viscosity is the
roof of a lift, carries a mass of 1kg at its free end. friction between layers of molecules of a fluid
If the lift accelerates upwards at 2.5ms-2, tending to make it resist free flow.
determine the reading on the spring balance - A more viscous liquid e.g oil,is difficult to flow
(g=10ms-2) A.6.25N B.12.5N C.25.0N D.24.0N. or for a body to fall through it than a less viscous
𝑚=1kg, 𝑎=2.5ms-2, g=10ms-2, reading on the liquid.
spring balance is the tension in the cord or - Properties of viscosity :
apparent weight or reaction on the floor=𝑅, I.It depends on surface area and relative
𝑅 = 𝑚(𝑔 + 𝑎), accelerating upwards. velocity/motion.
𝑅 =1(10+2.5) = 12.5N. II.It is affected by pressure i.e it increases with
52. A girl whise mass is 55kg stands on a spring pressure in the fluid.
weighing machine inside a lift. When the lift III.It reduces with an increase in temperature.
starts to ascend, its acceleration is 2ms-2. What IV.It resists the flow of liquids.
will be the reading on the machine? (take V.It is independent of weight or normal reaction.
g=10ms-2) A.22kg B.33kg C.44kg D.66kg VI.It opposes gravitational force (weight) of a
𝑚=55kg, 𝑎=2ms-2, g=10ms-2, 𝑅=?, falling body, as it acts upward.
𝑅 = 𝑚(𝑔 + 𝑎), ascending in the lift, - Solid friction and viscosity are similar in that
𝑅 = 55(10+2) = 55(12) = 660N. they resist or oppose relative motion between
𝑅 660
𝑅 = 𝑚R𝑔, 𝑚R = = = 66kg. surfaces and depends on the nature of the bodies
𝑔 10
53. A 1000kg elevator is descending vertically in contact. -
with an acceleration of 1ms-2. If the acceleration Stress law – It states that shear stress 𝝈 is
due to gravity is 10.0ms-2, the tension in the directly proportional to the velocity gradient 𝒗X.
𝑭 𝒗 𝑭 𝒗
suspending cable is A.1.0N B.10.0N C.9000.0N 𝝈 ∝ 𝒗X , 𝝈 = 𝜼𝒗X , 𝝈 = , 𝒗X= , = 𝜼
𝑨 𝑳 𝑨 𝑳
D.11000.0N. 𝜼=
𝑭𝑳
, 𝜼 is the constant called viscosity or
𝑚=1000kg, 𝑎=1ms-2, g=10ms-2, 𝑅=𝑇=? 𝑨𝒗
coefficient of viscosity in pascal-second, 𝑷𝒂. 𝒔,
𝑅 = 𝑇 = 𝑚(𝑔 − 𝑎) = 1000(10−1)=1000(9)
𝐹= force, 𝐿=length or distance, 𝐴=surface area,
𝑇 = 9000𝑁.
𝑣=velocity.
54. Calculate the apparent weight loss of a man 𝒕𝑨
weighing 70kg in an elevator moving downwards - Factors affecting viscosity : 𝜼 ∝ .
𝒗𝑻
with an acceleration of 1.5ms-2 (g=10ms-2) I.Time 𝑡 II.Surfacearea 𝐴 III.Temperature 𝑇
A.595N B.581N C.686N D.105N. IV.Velocity 𝑣.
𝑚=70kg, 𝑎=1.5ms-2, g=10ms-2, - The viscosity of lubricants oil is not affected
𝑊 − 𝑅 = 𝑚𝑎, a lift descending. Apparent loss in temperature (viscostatic).
weight for a descending body = 𝑾 − 𝑹 = 𝒎𝒂 , - The luricating effects of lubricants e.g oil and
the resultant force. greases decreases with temperature. Water is

45
Demystified Series Physics Demystified by Dr Timothy
not used as a lubricant because of its low force acting tangentially per unit length at a
viscosity. surface. 𝑻𝒔 or S.T = .
𝑭
𝑳
- The bob of a simple pendulum comes to rest
- It occurs due to net inward cohesion of molecule
more quickly with a cord attached to the string
on the liquid surface. It makes the surface of a
than without a cord because of viscosity of air.
liquid behave like a stretched elastic skin or
- A simple pendulum with a wide area comes to
membrane and contract areas of the liquid
rest quickly due to increased viscous force
surface to the least value
oppositing the pendulums motion.
- Surface tension can also be referred to as the
- A body falling through a viscous liquid is acted
elasticity of a fluid.
upon by three forces: Upthrust (buoyancy
- It can be explained by kinetic/molecular
force) 𝑼 and Viscous drag(fluid friction) 𝑽
theory of matter :
acting upward and the Weight 𝑾 (𝒎𝒈) acting
1.Scatterring and clinging together of hair of
downwards. The two upward force – upthrust
paint brush.
𝑈 and viscous force are together called retarding
2.Water surface in a beaker is not horizontal at
force.
the glass-water surface
- When a stone or ball bearing of mass 𝑚 is falling
3.Floating of denser objects e.g needle floats on
through a viscous liquid, initially the body
water surface
accelerates downwards, as the downward force
4.Dropping of nails into a glass filled with water
(𝑾) is greater than the upward retarding forces
without over flowing .
(𝑼 + 𝑽). The resultant force experienced by the
5.Waterproof materials e.g umbrella or tent, for
body : 𝑾 − 𝑼 − 𝑽 = 𝒎𝒂 or 𝑾 − (𝑼 + 𝑽) = 𝒎𝒂 .
keeping water out .
𝑎 is the acceleration of the stone.
6.Molecules at the surface of a liquid have more
- The viscous force increases with the speed of
potential energy than the molecules in the bulk
the ball bearing, until a certain maximum speed
7.Washing of clothes in water is difficult without
i.e terminal velocity,where the upward forces
soap.
balances the weight of the body.
8.Water maintains a height or rises above its
- At terminal speed(uniform or constant
outside level in a capillary tube.
velocity), the upward or retarding force
- Surface tension depends on the nature of the
(𝑼 + 𝑽) equals or balances the downward
liquid, temperature of the liquid, purity of the
force (𝒎𝒈). The net or resultant force acting
liquid and pressure acting on the liquid
at terminal velocity is zero and the
surface.
acceleration is 0. 𝑾 = 𝒎𝒈 = 𝑼 + 𝑽 or
- Surface tension is affected by temperature,
𝒎𝒈 − (𝑼 + 𝑽) = 𝟎 or (𝑼 + 𝑽) − 𝒎𝒈 = 𝟎. 𝟏
- Raindrops whose motion is opposed by the 𝑻𝒔 ∝ , surface tension decreases with increase
𝑻
viscosity of air attains a terminal velocity before in temperature.
reaching the ground. Hence, it is not harmful - Surface tension is affected by the pressure
when it strikes a person. acting in the liquid, 𝑻𝒔 ∝ 𝑷 , surface tension
- The velocity-time graph of a body falling increases with increase in pressure.
through a viscous liquid, a parachute or a body - Reduction of surface tension – Surface
dropped from an airplane has an initial tension is reduced by the addition of : I.alcohol
ascending line indicating acceleration and a or methylated spirit II.soap or detergent
horizontal line indicating terminal velocity i.e III.kerosene, oil or non-polar liquids.
𝑎=0. - Increasing of surface tension – Surface
Velocity tension is increased by the addition of : I.wax
𝒗o II.Grease.
- Adhesion or Adhesive forces – These are the
Terminal velocity 𝒗o. force of attraction between different molecules
Acceleration 𝒂 e.g water and glass or mercury and glass.
time - Cohesion or Cohesive forces – These are the
- Stoke’s law – It states that the viscous drag force of attraction between same molecules e.g
force 𝐹 retarding a small sphere falling through a between water molecules or mercury molecules.
viscous fluid at terminal velocity is proportional - Currency notes can be counted with moist
to the viscosity of the fluid, its radius and finger due to adhesion of water and the
terminal velocity. 𝑭 = 𝟔𝝅𝜼𝑹𝒗o. 𝜂=viscosity of currency.
the fluid, 𝑅=radius of the sphere, 𝑣 o=terminal - Mercury doesn’t wet glass because the cohesion
velocity or steady speed of the sphere. between mercury molecules is greater than the
adhesion between mercury and glass. Water
● Surface tension wets glass because the adhesion between water
- Surface tension 𝑻𝒔 – Surface tension is the and glasss is greater than the cohesion between

46
Demystified Series Physics Demystified by Dr Timothy
𝒎
water molecules. - Mass rate flow in kg𝑠-1 is the time rate of flow
𝒕
● Capillarity – Capillarity is the rise or fall of of the mass of fluid.
liquid in a narrow tube. 𝑚𝑎𝑠𝑠 𝒎 𝝆𝑽
Mass rate flow= = = = 𝝆𝑨𝒗. 𝑚=mass
- Water or alcohol surface forms a concave 𝑡𝑖𝑚𝑒 𝒕 𝒕
meniscus to air while mercury surface forms of the fluid, 𝜌=density of the fluid.
convex meniscus to air in a capiilary tube. - Force 𝑭 in a pump causing a liquid to flowis
- A wetting liquid i.e water or alcohol, whose given by – 𝑭 = 𝝆𝑨𝒗2.
adhesion is greater that its cohesion, will rise
above the liquid level. Examples :
- A non-wetting liquid i.e mercury, whose 1. The opposing force which one layer of liquid
cohesion is greater than its adhesion, will fall exerts on another is called A.cohesion
below liquid level in a capillary tube. B.viscosity C.limiting force D.dynamic force
- Application of capillarity : Viscosity is the friction between layers of
I.Rise of oil through a lamp wick, rise of candle molecules of a fluid tending to make it resist free
wax through a candle wick, rise or absorption of flow.
ink by a blotting paper 2. The property of fluid that is used to determine
II.Absorption of water by a towel i.e faster rate of its suitability as a lubricant is its A.capillarity
absorption of a drop of water on a damp cloth B.surface tension C.viscosity D.relative density.
than a dry cloth. 3. Which of the following is most viscous at room
III.Rise of water from the soil to the plant and temperature? A.palm oil B.petrol
flow of blood through channels. C.alcohol D.water
IV.Soaking of water by sugar. Palm oil at room temperature is hard to pour out
- Effects of adhesion/cohesion and surface of a container and an object will move slowly in it
tension on capillarity : than any of the other liquids. Hence, palm oil has
1.Rising of water up a capillary tube – the highest viscosity.
I.Adhesive forces pull the water up the side of the 4. Palm oil from a bottle flows out more earlier
tube. after it has been warmed because A.molecules
II.Surface tension holds up the water rising in the are given potential energy during warming
tube. B.friction between the oil layer is reduced
III.Water rises in the tube until surface tension C.oil molecules force one another out D.cohesion
balances or equals the weight of the water between the oil molecules
column. The pulling of water up the side of the Increase in temperature e.g by warming,
tube by adhesive forces gives rise to a concave decreases the viscosity of the palm oil, hence it
meniscus. flows easily.
2.Depresion of mercury in a capillary tube – 5. The similarity between friction and viscosity is
I.Coheisve forces pulls the mercury down the that both A.oppose relative motion between
side of the tube. surfaces B.occur in solids C.depend on relative
II.Surface tension holds down the mercury as it velocities between surfaces D.depend on normal
depresses in the tube. The pulling of the mercury reaction E.depend on areas of surfaces in
down the side of the tube by cohesive forces gives contact.
rise to a convex meniscus. 6. As the pressure of a fluid increases, its
- Angle of contact or contact angle – This is the viscosity A.decreases B.remains constant
angle between the wall of a capillary tuve and the C.increases then decreases D.increases.
tanget to the meniscus of the liquid in the tube. Viscosity depends on pressure, viscosity
Angle of contact depends on the neatness of the increases as pressure increases and vice-versa
wall of the capuillary glass tube. Angle of contact 7. A spherical ball falling through a viscous liquid
is acute i.e <90°, for water and obtuse i.e >90°, acted upon by the I.weight of the ball II.viscous
for mercury. force of the liquid on the ball III.upthrust of the
● Fluids in motion – It involves flow rates and liquid on the ball A.I and II B.I and III C.II and III
viscosity. D.I,II and III.
- Flow rates is the time rate of flow of fluids. The forces acting on a body falling down a
𝑽
- Volume rate flow in m3𝑠-1 is the time rate of viscous lioquid are ; the weight of the ball W,
𝒕 viscous force of the liquid V and the upthrust of
flow of the volume of the fluid. the liquid U.
volume 𝑽
Volume rate flow= = = 𝑨𝒗. 𝑉=volume 8. The effect of a particle in a fluid attaining its
time 𝒕
of the fluid, 𝑡=time of fluid flow, 𝐴=area of the terminal velocity is that the A.acceleration is
vessel through which the fluid flows, 𝑣=velocity maximum B.weight equals the retarding force
at which the fluid flows. C.buoyancy force is equal to the viscous retarding
The volume of the liquid is given by – 𝑽 = 𝑨𝒗𝒕.

47
Demystified Series Physics Demystified by Dr Timothy
force D.buoyancy force is more than the weight 14. Which of the following is obeyed if a force on
of the fluid displaced a sphere moving through a viscous fluid is
At terminal velocity, velocity is uniform or proportional to its speed,radius and viscosity?
constant,hence the acceleration is zero, also the A.Archimede’s principle B.Pascal’s principle
weight equals the retarding force (upthrust or C.Stoke’s law D.Bernolli’s law
buoyancy force and viscous drag or fluid Stoke’s law gives the force acting on sphere
friction). falling through a viscous liquidat terminal
9. A parachute attains a terminal velocity when velocity.𝐹 = 6𝜋𝜂𝑅𝑣 o.
A.its density is equal to the density of air B.the 15. A hose of cross-sectional area 0.5m2 is used
viscous force of the air and the upthrust to discharge water from a water tanker at a
completely counteract its weight C.it expands velocity of 60ms-1 in 20s into a container. If the
as a result of reduced external pressure D.the container is filled completely, the volume of the
viscous force of the air is equal to the sum of the container is A.600m3 B.6000m3 C.240m3
weight and upthrust. D.2400m3.
At terminal velocity, the upthrust and viscous 𝐴=0.5m2, 𝑣=60ms-1, 𝑡=20s, 𝑉=?,
force completely balances the weight of the body. 𝑉
Volume rate flow = = 𝐴𝑣, 𝑉 = 𝐴𝑣𝑡,
𝑡
10. The terminal velocity of a ball-bearing falling
𝑉 = 0.5×60×20 = 600m3.
through a viscous liquid is reached when the
16. The force between molecules of the same
A.upthrust is equal to yhe weight of the ball B.ball
substance is termed A.elastic force B.repulsive
accelerates uniformly C.upthrust is equal to the
force C.cohesive force D.adhesive force.
velocity of the ball D.velocity is uniform.
Cohesive forces are the forces of attraction
Terminal velocity is the maximum uniform
between molecules of the same substance.
velocity attained by a body falling through a
17. The force between the molecules of a liquid
viscous liquid. Initially it accelerates uniformly
in contact with that of a solid is A.adhesive
until terminal velocity is reached. The resultant
B.cohesive C.magnetic D.repulsive.
or net force is zero at terminal velocity.
Adhesive forces are the forces of attraction
11. A metal ball of weight 𝑊 falls through a
between molecules of different substance e.g
column of glycerine of viscosity 𝑉. If the bal
molecules of liquids and solids.
experiences an upthrust 𝑈 and terminal velocity
18. Which of the following can not be explained
is attained, then A.𝑊 > 𝑈 + 𝑉 B.𝑾 = 𝑼 + 𝑽
using the concept surface tension? A.crawling of
C.𝑊 = 𝑈 − 𝑉 D.𝑊 < 𝑈 + 𝑉.
insects on the surface of a pond B.formation of
12. A body of mass 36kg falls through a viscous
spherical droplets when water drips from a tap
liquid which offers a drag force 260N on the
C.floating of balloon is air D.formation of soap
body. The upthrust on the body at terminal
bubbles.
velocity is A.50N B.100N C.310N D.620N
Floating of balloons in air is explained using the
(g=10ms-2)
floatation principle. Other options are
Viscous drag or fluid friction, 𝑉=260𝑁, 𝑚=36kg,
application surface tension
g=10ms-2, upthrust or buoyancy force, 𝑈=?, At
19. The following processes are explained by
terminal velocity, 𝑊 = 𝑈 + 𝑉, 𝑈 = 𝑊 − 𝑉 ,
surface tension only except A.bristles of a paint
𝑈 = 𝑚𝑔 − 𝑉 = 36×10−260 = 360−260 = 100N.
brush cling together when removed from water
13. velocity
B.dirts being dissolved by soap solution C.needle
Z
floating in water D.sphrical droplets of water
X Y
dropping fron a tap E.water rising up a
capillary tube.
O time
Water rising up a capillary tube is due to greater
The motion of a falling sphere through a viscous
adhesion of water molecules to the capillary tube
liquid is represented by the graph. The terminal
than their cohesion and surface tension pulling
velocity is determined as the A.the value of the
the water up the inner walls of the tube until it is
velocity at point X B.value of the velocity at
balanced by the weight of the water column.
point along the portion YZ C.slope of the graph
Hence, it cannot be explained by surface tension
at point Y D.area under the graph OXYZ.
only.
The terminal velocity is reached when the
20. Which of the following substances lowers the
velocity is uniform or constant over a period of
surface tension of water? A.metal B.sand
time i.e portion YZ, the acceleration at this point
C.detergent D.paper.
is zero. The portion OX represent the initial
21. If a beaker is filled with water, it is observed
acceleration phase of the falling body, an the
that the surface of the water is not horizontal at
slope of the graph at the portion OX gives the
the glass-water surface. This behavior is due to
acceleration.
A.friction B.surface tension C.evaporation
D.viscosity.

48
Demystified Series Physics Demystified by Dr Timothy
Surface tension contracts area of the liquid umbrella surface and water droplets is reduced,
surface to the least value, making it to behave like hence the elastic skin is broken and water drips
a streteched elastic membrane, hence the water into the umbrella.
surface is not horizontal. 29. The rising of a liquid in an open ended glass
22. Which of the following additions will not tube of narrow bore is A.osmosis B.adhesion
reduce the surface tension of water? A.Detergent C.capillarity D.surface tension.
B.Grease C.Alcohol D.Camphor. 30. The rise or fall of a liquid in a narrow tube is
Addition of grease to water increases the surface because of the A.the surface tension of the
tension rather than decreasing it. liquid B.the friction between the walls of the
23. Certain insects can move about on the surface tube and the liquid C.the osmotic pressure of the
of water without sinking because A.water is liquid D.the viscosity of the liquid.
denser than such insects B.the insects have tiny Surface tension pulls up the water up the inner
legs C.upthrust is greater than their weight D.the walls of the tube.
water surface acts like an elastic membrane. 31. The action of a blotting paper in use is a
Surface tension is the responsible for the floating demonstration of the principle of A.viscosity
of denser objects i.e objects whose dener than B.evaporation C.capillarity D.surface tension.
water e.g steel needle,razor blade, insects or a 32. The action of a towel in drying the body after
piece of wire guaze. a bath is explained by A.diffusion B.capillarity
24. Which of the following liquids has the highest C.osmosis D.evaporation.
surface tension? A.Soapy water B.Cold water 33. By which of the following processes does a
C.Hot water D.Oily water. piece of blotting paper absorb a drop of ink from
Surface tension decreases with temperature, a sheet of writing paper? A.the action of the
hence hot water has low surface tension. Soap forces of cohesion between the blotting paper
and Oil reduces surface tension.Hence, cold fibres and the ink C.the capillarity rise of the
water has the highest surface tension. ink into the blotting paper fibres D.the
25. Which of the following statememts explains diffusion of the ink into the blotting paper fibres
why hot soapy water is more effective in cleaning E.the action of the force of viscosity between the
oil-stained dishes? A.The oil on the dishes repels molecules of the ink.
the soap B.Soap and heat decreases the 34. The concave meniscus of water in a glass tube
surface tension of water C.Hot water increases is due to the fact that the A.adhesion between
the surface tension of oil D.Soap increases the water and glass molecules is greater than the
surface tension of oil and water. cohesion between water molecules
26. A steel needle floating on the surface of water B.adhesion between glass and water molecules is
sinks when kerosene is added to the water smaller than cohesion between water molecules
because A.kerosene cause an increase in the C.molecules of water near the glass are lighter
surface tension of water B.kerosene causes a than the molecules in the central part of the
decrease in the surface tension of water water column D.weight of the water column pulls
C.kerosene causes a decrease in the density of down on the central part of the water column.
water D.kerosene causes an increases in the The meniscus is convex as in mercury, if the
density of water E.a steel needle is denser than cohesion between the mercury molecules is
kerosene. greater than the adhesion between the mercury
Steel needle floats on water surface due to and glass molecules.
surface tension. Addition of kerosene decreases 35. In a capillary experiment, the rise of mercury
the surface tension of water, hence the steel in a narror capillary tube is A.higher than that of
needle sinks. the container B.lower than that of the
27. Mercury forms little balls when it is spilled on container C.equal to that of the container
a bench because it A.is very viscous B.adheres to D.rising and falling constantly in the container.
the bench C.has a high surface tension D.has a 36. Mercury which is spilled onto a glass surface
high density E.is a liquid metal. forms ball-like shapes because A.mercury is a
Surface tension contracts the area of liquid metal B.the cohesive force between its
surface into small or little balls of reduced shape. molecules is greater than the adhesive forces
28. Water does not drip through an open between mercury and glass molecules C.the
umbrella of silk material unless the indide of the cohesive force between its molecules is less than
umbrella is touched. Which of the following the adhesive forces between mercury and glass
phenomenon is respondible for this? A.surface molecules D.the viscosity of mercury is high.
tension B.hydrostatic upthrust C.viscosity 37. If a tube of small radius opened at both ends
D.diffusion E.osmosis. is placed in a liquid, the liquid will A.fall below
When the inside of an umbrella or raincoat is the liquid level if the liquid does not wet the
touched, the surface tension between the glass B.rise above the liquid level if the liquid

49
Demystified Series Physics Demystified by Dr Timothy
does not wet the glass C.remain at the same level
irrespective of whether the liquid wets the glass
or not D.fall below the liquid level if the liquid
wets the glass.
A non-wetting liquid e.g mercury, falls below the
liquid level in a capillary tube. A wetting liquid e.g
alcohol or water, rises above the liquid level in a
capillary tube.

Jamb past questions on Fluid friction and


surface tension :
[1987/12,1991/14,1997/17,1999/14,2003/8,
2008/12,2009/24,2011/9,2013/23,1994/2,
1998/11,2003/5,2002/4,11,2004/13,2005/49,
2006/18,2009/13]

50
Demystified Series Physics Demystified by Dr Timothy

CHAPTER 4 -WORK , ENERGY AND POWER


● Workdone W – Workdone is the product of 4. Force(𝑁)
force 𝐹 and parallel distance 𝒔 moved in W=Area of a semi-circle = ½𝜋𝑟 2
direction of the force. W= 𝑭 × 𝒔, in Joule(J). 𝑟=𝐷/2.
- Workdone on a body is zero when no
displacement (a body at rest), even if a force Distance(m)
is applied in a specified time e.g a boy standind 5. Force(𝑁)
with a bowl of water on his head, a body pusing a W=Area of a trapezium
fixed or stationary wall. W= ½(𝑎 + 𝑏)ℎ
- The force 𝐹 can be determined by – 𝒎𝒂 (if the
body is accelerated by the force) or W=𝒎g (if the Distance(m)
body is falling freely under gravity) or 𝝁𝑹 (due to
frictional force) or 𝑭𝒄𝒐𝒔θ (inclined to the ● Energy – Eenergy is the ability or capacity to
horizontal) or 𝑭𝒔𝒊𝒏θ (inclined to the vertical). do work.
𝟏
- The distance 𝑠 can be determined by – 𝒔 = 𝒂𝒕2 - Sources of energy – Renewable and non-
𝟐
𝟏 𝒗𝟐 𝒗𝟐 renewable energy source.
or 𝒈𝒕2 or or or 𝒉. 1.Renewable energy source – These are energy
𝟐 𝟐𝒂 𝟐𝒈
𝑎=acceleration, 𝑡=time, 𝑣=velocity, ℎ=height, sources that are naturally and easily
𝑔=acceleration due to gravity,. replenished/replaced as they are used e.g solar
- If a body rolls down a hill, slope or inclined energy i.e energy from the sun, wind energy i.e
plane, the workdone by the body depends on the energy in wind mills, ocean tides and waves,
vertical height of the hill. W= 𝒎𝒈𝒉, 𝒉 = 𝒍𝒔𝒊𝒏θ, hydro (water) energy i.e energy generated in
𝑙=length of the hill or slope or plane, θ=angle of dams.
inclination to the horizontal. 2.Non-renewable energy source – These are
- The workdone by a body climbing a set of steps energy soures that are not easily replenished as
is given by – W = 𝒎𝒈𝒉, 𝑚=mass of the body, they are depleted e.g petroleum (fossil fuels),
ℎ=total in covering a flight of steps =height of coal, nuclear/atomic energy.
one step×number of steps. - Forms of energy – They include ;
- Workdone due to friction 𝑊 involves the 1.Mechanical energy – It is used for mechanical
frictional force 𝐹 R and the distance 𝑠 moved by it. work e.g potential and kinetic energy. 2.Sound
W = 𝑭𝑹 × 𝒔 = 𝝁𝑹 × 𝒔 = 𝝁𝒎𝒈 × 𝒔. energy 3.Solar energy 4.Light energy
- Force-Distance(displacement) graphs – The 5.Chemical energy – Energy stored in fuels,food
area of the figure under a force-distance graph and cells/batteries. 6.Thermal(heat) energy –
represents the workdone by the body. Total internal energy of a body. 7.Atomic
Examples are : energy(Nuclear energy) – Energy stored in the
1. Constant force and varying distance moved nucleus of an atom. 8.Electrical energy.
e.g a push or a pull. - Energy conversion – Energy can be converted
Force(𝑁) Workdone=Area of the rectangle from one for to another. Some energy is always
W = 𝐹𝑠. lost as heat energy during conversion.
𝐹 From To Device
Mechanical Electrical Generator
Electrical Mechanical Electric motor
O 𝑠 Distance/displacement(m) Heat Mechanical Turbine
2.Variable force and varying distance moved Chemical Heat Bunsen burner
e.g workdone in stretching an elastic material Electrical Heat Resistor
i.e spring or catapult. Heat Electrical Thermocouple
Force(𝑁) Workdone=Area of triangle
Solar Electrical Photocell
W = ½𝐹𝑒
Chemical Electrical Cell/battery
Atomic Heat Nuclear reactor
Electrical Sound Televison,
electric bulb
O 𝑠 Distance/displacement(m)
3. Force(𝑁) W =Area of a quadrant= ¼𝜋𝑟 2 Sound Electrical Microphone
- Energy conversion in electric power station –
I.Steam engine : heat →mechanical→electrical
→light and heat e.g coal or gas power station.
Distance(m) II.Turbine : kinetic→mechanical→electrical→

51
Demystified Series Physics Demystified by Dr Timothy
light and heat e.g hydroelectric power station. 𝑭 × 𝒔 = 𝒎𝒗2.
𝟏
𝟐
- Mechanical energy – It is of two types;
● Power 𝑷 is the rate of doing work or rate at
1.Potential energy 𝑷. 𝑬 – Energy of a body due
which energy is espended, in Watt(W).
to is position or state of rest i.e stored up-energy. 𝒘𝒐𝒓𝒌𝒅𝒐𝒏𝒆 𝒆𝒏𝒆𝒓𝒈𝒚 𝑾 𝑬
𝑷. 𝑬 = 𝒎𝒈𝒉. 𝑚=mass of the body, ℎ=height of Power= = .𝑷= = .
𝒕𝒊𝒎𝒆 𝒕𝒊𝒎𝒆 𝒕 𝒕
𝑭×𝒔
the body. Examples 𝑷= = 𝑭𝒗. Energy can be 𝐾. 𝐸 or 𝑃. 𝐸.
𝒕
2.Kinetic energy 𝑲. 𝑬 – Energy possed by a body 𝟏
𝒎𝒗𝟐
𝑷.𝑬 𝒎𝒈𝒉 𝑲.𝑬
𝟏
due to its motion. 𝑲. 𝑬 = 𝒎𝒗2. 𝑣=velocity of the 𝑷= = , 𝑷= =𝟐 .
𝟐 𝒕 𝒕 𝒕 𝒕
body. - Horse power (Hp) is the unit of power in an
- Kinetic energy of a body is varies directly engine. 1Hp=746watts=0.746kilowatts.
with the square of the velocity of the body - The efficiency 𝝐 is given by –
𝑷𝒐𝒘𝒆𝒓 𝒐𝒖𝒕𝒑𝒖𝒕 𝑷
𝑲. 𝑬 ∝ 𝒗2 (mass=constant). Efficiency, 𝝐 = ×100= 𝒐 ×100.
𝑷𝒐𝒘𝒆𝒓 𝒊𝒏𝒑𝒖𝒕 𝑷𝒊
- Law of conservation of energy – It states that Power input is the power developed in the
the total energy of an isolated system is constant machine. Power output is the power expended or
or energy can neither be created nor destroyed 𝒎𝒈𝒉
but can be converted from one form to another. developed in the load = .
𝒕
- Conservation of mechanical energy – It states
that the total mechanical energy of a system is Examples :
constant. Total mechanical energy 𝑀. 𝐸 of a 1. Which of the following statements correctly
system is equal to the sum of its potential energy defines quantitatively, the workdone by a force?
amd kinetic energy. 𝑴. 𝑬 = 𝑷. 𝑬 + 𝑲. 𝑬. The product of the force and the distance moved
- Though the total mechanical energy is constant, A.parallel ot the line of action of the force
energy can be converted from 𝑃. 𝐸 to 𝐾. 𝐸 and from a reference point B.perpendicular to the
from 𝐾. 𝐸 to 𝑃. 𝐸. line of action of the force from a reference point
- 𝑷. 𝑬 – 𝑲. 𝑬 conversion – I.Free fall : For a body C.on a line through the reference point D.by the
falling under gravity, 𝑃. 𝐸 decreases downwards, object.
as heights decreases while 𝐾. 𝐸 increases Workdone involves parallel distance while
downwards, as velocity increases or Moment of a force involves perpendicular
𝐾. 𝐸 decreases and 𝑃. 𝐸 increases when a body is distance.
thrown vertically upward. 2. An object of mass 12kg is held at a height of 5
- The 𝐾. 𝐸 of a body at a height ℎ just before meters above the ground for 30 seconds. The
hitting the ground equals the 𝑃. 𝐸 at that height ℎ work done within this period is A.600J B.200J
or 𝑃. 𝐸 at the top equals K.E during falling. C.60J D.0.
- The kinetic energy of a body at half-way Work done within this period is zero as there is
point (𝒙 = 𝑯/𝟐) is equal to half of its initial no displacement of the body.
potential energy. 3. A boy of weight 300N climbs to the top of a hill
- The initial 𝐾. 𝐸 at the ground level of a body of height 20m.The workdone by the boy against
projected vertically upward from the ground is the force of gravity A.6000J B.600J C.320J D.15J.
equal to total mechanical energy. W=mg=300N, h=20m, W=?
- Percentage of initial potential energy converted W = 𝑚𝑔ℎ = 300×20 = 6000J or 6kJ.
to kinetic energy =
𝑲.𝑬
× 𝟏𝟎𝟎. 4. A body initially at rest is accelerated at the rate
𝑷.𝑬 of 0.2ms-2 for 5s under a constant force of 50𝑁.
- Energy dissipated as heat = 𝑷. 𝑬 − 𝑲. 𝑬. The workdone on the body is A.10J B.50J C.125J
- Energy lost by a falling body falling from a D.250J.
height 𝐻 onto the floor and rebounds to a height 1
ℎ is given by – Energy lost = 𝒎𝒈(𝑯 − 𝒉). 𝑢=0, 𝑎=0.2ms-2, 𝑡=5s, 𝐹=50𝑁, 𝑠 = 𝑎𝑡 2 , W=?,
2
1
- Workdone-energy conversion : 𝑠 = ×0.2×52 = 0.1×25 = 2.5m,
2
I.If the workdone on a body is all utilized in W = 𝐹 × 𝑠 = 50×2.5=125J.
setting it in motion thus, Workdone=change in 5. 𝐹 A force 𝐹 is applied to a body 𝑃
kinetic energy. 𝑾 = ∆𝑲. 𝑬 = 𝑲. 𝑬2−𝑲. 𝑬1. θ as shown in the diagram above.
𝟏
𝑊 = 𝐹 × 𝑠, initial kinetic energy, 𝑲. 𝑬1= 𝒎𝒖2, If the body moves through a
𝟐
𝟏
final kinetic energy, 𝑲. 𝑬2 = 𝒎𝒗2. 𝑢=initial 𝑃 distance 𝑎, which of the follow-
𝟏
𝟐 𝑎 ing represents the workdone
velocity, 𝑣=final velocity, 𝑾 = 𝒎(𝒗2−𝒖2). A.𝐹𝑎 B.𝐹𝑎𝑐𝑜𝑠θ C.𝐹a/𝑐𝑜𝑠θ D.𝑭𝒂𝒔𝒊𝒏θ E.𝐹𝑎/𝑠𝑖𝑛θ.
𝟐
II.Inelastic collision of a body of mass 𝑚 with The resolved component of the force 𝐹 in the
another body by an apllied force after moving a horizontal direction is 𝐹𝑠𝑖𝑛θ, distance 𝑠=𝑎,
distance 𝑠 with a velocity 𝑣, hence the workdone W = 𝐹𝑠𝑖𝑛θ × 𝑎 = 𝐹𝑎𝑠𝑖𝑛θ.
by the force equals the kinetic energy of the body. 6. A load is pulled 5m along a horizontal floor by
a constant force of 20𝑁 which acts at 30° to the

52
Demystified Series Physics Demystified by Dr Timothy
floor. Cacluate the work done by the force. 13. Force(𝑁)
A.10.0J B.17.3J C.50.0J D.86.6J D.100.0J
𝑠=5m, θ=30°, 𝐹 = 𝐹𝑐𝑜𝑠θ, W=?, 50
W = 20𝑐𝑜𝑠30° × 5 = 100𝑐𝑜𝑠30° = 86.6𝐽. 25
7. A 500N box rests on a horizontal floor. A
constant horizontal force is exerted on it so that 0 10 20 30 Displacement(m)
it moves through 8m. If the coefficient of kinetic The diagram above shows the force versus
friction is 0.22,calculate the workdone on the box displacement graph for a body. The work done on
A.880J B.440J C.400J D.110J. the body during the first 20m of motion is
W=𝑚𝑔=500N, 𝑠=8m, 𝜇=0.22, W=?, A.1000J B.875J C.750J D.500J .
W = 𝐹 R × 𝑠 = 𝜇𝑅 × 𝑠 = 𝜇𝑚𝑔 × 𝑠, Workdone=Area of the figure under the force-
W = 0.22×500×8 = 880J. displacement graph.
8. Two forces of 4N and 6N act on an object at an Workdone during the first 10m=50×10=500𝐽
angle of 60° to each other. If the object is moved Workdone during the next 10m i.e between 10m
1 1
through a distance of 5m in the direction of the and 20m= (50+25)(20−10)= ×75×10=375𝐽
2 2
resultant force. Calculate the work done on the Total workdone during the first 20m of
object by the forces. A.51.96J B.43.59J C.36.00J motion=500+375=875𝐽.
D.26.83J E.26.46J. 14. 𝐹(𝑁) A body under the action of a force
4𝑁 The resultant force 𝐹 is such that the force-displacement
𝐹 gotten by cosine rule, 12 graph of the body is semicircular
● 60° 𝐹 2 = 42+62+2×4×6𝑐𝑜𝑠60°, 6 as shown above. The workdone
6𝑁 𝐹 2 = 16+36+48(0.5) , on the body by the force in
𝐹 = 52+24 = 76, 𝐹 = √76 = 8.718N, 𝑠=5m,
2
12 24 𝑥(m) moving through 24m is
𝑊 = 𝐹 × 𝑠 = 8.718×5 = 43.59J. A.36𝜋J B.72𝝅J C.144𝜋J D.288𝜋J.
9. A box of mass 40kg is being 1
Workdone=Area of a semi-circle= 𝜋𝑟 2,
𝑇 dragged along the floor by 𝐷 24
2

Box 60° a rope inclined at 60° to the diameter 𝐷=24m, 𝑟 = = = 12,


2 2
horizontal. The frictional force between the box 1 1 1
𝑊 = 𝜋𝑟2 = 𝜋 ×122 = 𝜋 ×144 = 72𝜋J.
2 2 2
and the floor is 100N and the tension on the rope 15. Force(𝑁)
is 300𝑁. How much work is done in dragging the 20
box through a distance of 4m? A.680𝐽 B.400𝐽
C.200𝐽 D.100𝐽 10
𝑇=300𝑁, 𝐹 R=100𝑁, 𝑠=4m, θ=60°, 𝑚=40kg,
0 10 20
𝑇 The force displacement graph of a body is given
𝐹R Box 60° 𝑇𝑐𝑜𝑠60° by the quadrant of a circle,shown in the diagram
The resultant force moving the block in the abovce.The workdone by the force in moving the
horizontal direction along the floor : body through a distance of 20m is A.400𝜋J
𝐹 = 𝑇𝑐𝑜𝑠60° − 𝐹 R = 300𝑐𝑜𝑠60° −100, B.200𝜋J C.100𝝅J D.50𝜋J.
𝐹 =150−100 = 50N, W = 𝐹 × 𝑠 = 50×4= 200J. 1
Workdone=Area of a quadrant= 𝜋𝑟 2, 𝑟=20m,
10. The area under a force-distance graph 1 1
4

represents A.acceleration B.velocity C.power W = 𝜋 ×202 = 𝜋 ×400 = 100𝜋J.


4 4
D.work E.momentum. 15. Force
11. 𝐹(𝑁) The above graph represents the 80
action of a 60
40
20
0 30 40 60 80 120 distance
𝑠(m)
−20
A.zero force B.variable force C.constant force −40
D.tangential force. From the diagram above, calculate the workdone
12. F/N The diagram above illustrates when the particle moves from 𝑥=0m to 𝑥=80m
30
a force-distance graph for the A.1200J B.2400J C.6000J D.7000J.
20 motion of a wooden block. Workdone from 0m to 70m=Area of the bigger
10 Determine the workdone on 1
traingle= ×70×80=2800J
(0,0 1 2 3 4 5 𝑑/m the block when 2
move
) through a distance of 5m. A.4J B.15J C.25J Workdone from 60m to 80m=Area of smaller
1
D.100J. triangle= ×(80−60)× −40= −400J
2
Workdone on the block = Area of the rectangle, Total workdone=2800+(−400)=2400J.
W = 20×5 = 100J. 16. Which of the following statements is not

53
Demystified Series Physics Demystified by Dr Timothy
correct? A.The P.E of a body is equal to the work D.momentum
the body can do because of its position B.The P.E During fall the total mechanical energy is
is the ability a body possesses to do work because constant, but the potential energy decreases
of stored up energy C.The P.E is the ability a while its kinetic energy increases.
body possesses to do work owing to the 22. A body of mass 1000kg is released from a
movement of its molecules D.Energy cannot height of 10m above the ground. Determine its
normally be destroyed but can be changed into kinetic energy just before it strikes the ground
another form of energy E.A body can have both (g=10ms-2) A.10J B.103J C.104J D.105J.
kinetic and potential energy at the same time 𝑚=1000kg, ℎ=10m, g=10ms-2,
Potential energy is the workdone by a body due The kinetic energy just before striking the
to stored up energy,it involves a change in ground equals the potential energy at the height
position and does not involve motion or from which it was released.
movement of molecules. It is kinetic energy that 𝐾. 𝐸 = 𝑃. 𝐸 = 𝑚𝑔ℎ = 1000×10×10=105J.
is the workdone owing to the movement of 23. A body of mass 0.6kg is thrown vertically
molecules. upwards from the ground with a velocity of
17. A car of mass 800kg initially at rest is 20ms-1. Calculate its potential energy at its
accelerated at the rate of 4ms-2. The kinetic maximum height (g=10ms-2) A.120J B.240J
energy of the car after 5s is A.6.40×103J C.360J D.480J.
B.2.56×104J C.1.60×105J D.6.40×105J. 𝑚=0.6kg, 𝑢=20ms-1, g=10ms-2, 𝑃. 𝐸=?,
𝑚=800kg, 𝑢=0, 𝑎=4ms-2,𝑡=5s, 𝐾. 𝐸=?, 𝑃. 𝐸 at maximum height is equal to the initial 𝐾. 𝐸
𝑣 = 𝑢 + 𝑎𝑡 = 0+𝑎𝑡 = 𝑎𝑡, from the point of through.
1 1 1 1 1
𝐾. 𝐸 = 𝑚𝑣 2 = 𝑚(𝑎𝑡)2 = ×800(4×5)2 𝑃. 𝐸 = Initial 𝐾. 𝐸 = 𝑚𝑢2 = ×0.6×202,
2 2 2 2 2
𝐾. 𝐸 = 400×202 = 1.6×105J. 𝑃. 𝐸 = 0.3×400 = 120J.
18. When the speed of a car is halved, its kinetic 24. A body of mass 4kg is acted by a constant
energy is A.doubled B.quartered C.halved force of 12𝑁 for 3s. The kinetic energy gained by
D.quadrupled. the body at the end of the time is A.162J B.144J
1 1
𝐾. 𝐸 = 𝑚𝑣 2 , 𝐾. 𝐸 ∝ 𝑣 2 ( 𝑚=constant). C.72J D.81J.
2 2 𝑣
1 1 2 1 𝑚=4kg, 𝐹=12𝑁, 𝑡=3s, 𝐾. 𝐸=?, 𝐹 = 𝑚𝑎 , 𝑎 = ,
If the speed 𝑣 is halved ( ), 𝐾. 𝐸 ∝ 𝑣 2 = ( ) = 𝑚𝑣 𝐹𝑡 1 1 𝐹𝑡 2
𝑡
2 2 4
hence, the kinetic energy is quartered. 𝐹= , 𝑣= , 𝐾. 𝐸 = 𝑚𝑣2 = 𝑚( ) ,
𝑡 𝑚 2 2 𝑚
19. An object of mass 0.25kg moves at a height ℎ 1 12×3 2
𝐾. 𝐸 = × 4× ( ) = 2×92 = 162J.
above the ground with a speed of 4ms-1. If its 2 4

mechanical energy at this height is 12J, 25. An object of mass 100g is projected vertically
determine the value ℎ (g=10ms-2) A.0.8m upward from the ground level has a velocity of
B.4.0m C.4.8m D.5.6m. 20ms-1 at a height of 10m. Calculate the initial
𝑚=0.25kg, 𝑣=4ms-1, 𝑀. 𝐸=12J, g=10ms-2, ℎ=?, kinetic energy at the ground level A.10J B.20J
Total mechanical energy at this height is the sum C.30J D.50J.
of the potential and kinetic energy of the body. 𝑚=100g=0.1kg, 𝑣=20ms-1 , g=10ms-1,
1 𝑣 2= 𝑢2−2𝑔ℎ (upward projection),
𝑀. 𝐸 = 𝑃. 𝐸 + 𝐾. 𝐸 = 𝑚𝑔ℎ + 𝑚𝑣 2,
2 𝑢2= 𝑣 2+2𝑔ℎ =202+2×10×10=400+200,
1 1
𝑀. 𝐸 = 𝑚(𝑔ℎ + 𝑣 2), 12 = 0.25(10ℎ + ×42), 1
𝑢2=600, Initial kinetic energy= 𝑚𝑢2
2 2 2
12
10ℎ + 8 = =48, 10ℎ = 48−8 = 40, 1
Initial kinetic energy= ×0.1×600=30J. OR
0.25 2
40
ℎ = = 4m. Initial kinetic energy at the ground level equal
10
20. A body of mass 10.00kg falling freely attains total mechanical energy at height of 10m above
a velocity of 9.00ms-1 at a height of 20.00m above the ground. Initial 𝐾. 𝐸 = 𝑀. 𝐸 = 𝑃. 𝐸 + 𝐾. 𝐸 ,
1 1
the ground. Calculate the total energy at the Initial 𝐾. 𝐸 = 𝑚(𝑔ℎ + 𝑣 2 = 0.1(10×10+ ×202)
2 2
height A.1800J B.2000J C.2045J D.2405J E.2810J Initial 𝐾. 𝐸 = 0.1(100+200) = 30𝐽.
𝑚=10kg, 𝑣=9ms-1, ℎ=20m, 𝑀. 𝐸=?, 26. A stone of mass 1kg is dropped from a height
Total energy at that height above the ground, of 10m above the ground and falls freeky under
1
𝑀. 𝐸 = 𝑃. 𝐸 + 𝐾. 𝐸 = 𝑚(𝑔ℎ + 𝑣 2) gravity, its kinetic energy 5m above the ground is
2
1
𝑀. 𝐸 = 10(10×20+ ×92) = 10(200+40.5), equal to A.its kinetic energy on the ground
2 B.twice its initial potential energy C.its initial
𝑀. 𝐸 = 2405𝐽. potential energy D.half its initial potential
21. A diver steps off a diving platform that is 10m energy.
above water. If there is no air resistance during The kinetic energy of a body at half way point i.e
the fall, there will be a decrease in the diver’s 5m is equal to half its initial potential energy at
A.gravitational potential energy B.total 10m.
mechanical energy C.kinetic energy 27. A ball of mass 100g falls from a height of 5m
54
Demystified Series Physics Demystified by Dr Timothy
onto a floor and rebounds to a height of 3m. What 33. An object slides down an inclined plane from
energy is loss as a result of the impact on the an initial height of 30m. Its velocity at the foot of
floor? A.2J B.20J C.100J D.1000J E.2000J. the plane is 20ms-1.The percentage of its initial
Energy lost by a falling body falling from a height potential energy which is dissipated as heat is
𝐻 onto the floor and rebounds to a height ℎ is ; 𝟏 2
A.33 % B.50% C.66 % D.75%.
Energy lost = 𝑚𝑔(𝐻 − ℎ) , 𝑚=100g=0.1kg, 𝟑 3
mass of the body=𝑚, ℎ=30m, 𝑣=20ms-1,
g=10ms-2, 𝐻=5m, ℎ=3m,
Initial 𝑃. 𝐸 = 𝑚𝑔ℎ = 𝑚 ×10×30=300𝑚J,
Energy lost = 0.1×10(5−3) = 1(2) = 2J. 1
28. An object is moving with a velocity 5ms-1. At 𝐾. 𝐸 at the foot of the slope= 𝑚𝑣 2 ,
2
1
what height must a similar body be situated to 𝐾. 𝐸 = × 𝑚 ×202 = 200𝑚J. Energy dissispated
2
have a potential energy equal in value with the as heat = 300𝑚 −200𝑚 =100𝑚J.
kinetic energy of the moving body? (g=10ms-2) Percentage of the initial 𝑃. 𝐸 dissipated as
A.20.0m B.1.3m C.1.0m D.25.0m. 100𝑚 1
heat = × 100 = 33 %.
𝑣=5ms-1, 𝑚 is the same for both bodies, as they 300𝑚 2
1 34. Which of the following correctly describe the
are similar. 𝑃. 𝐸 = 𝐾. 𝐸, 𝑚𝑔ℎ = 𝑚𝑣 2
2 energy changes in the generation of light by a
1 𝑣2 52 25
𝑔ℎ = 𝑣 2 , ℎ = = = = 1.25m ≈1.3m. hydroelectric power station? A.Electrical –
2 2𝑔 2×10 20
29. An object of mass 2.0kg falls freely from a mechanical – potential – light B.Potential –
height of 100m onto muddy ground and comes to mechanical – electrical – light C.Mechanical –
rest in the mud. Calculate the distance the body sound – electrical – light D.Kinetic – mechanical
oenetrates if the average retarding force is – electrical – light.
20.000N A.0.1m B.1.0m C.2.0m C.10.00m The kinetic energy flowing water is converted
E.100.0m. into mechanical energy which is used to rotate
𝑚=2kg, ℎ=100m, 𝐹=20N, g=10ms-2, 𝑠=?, turbine which in turn generates a relative motion
𝑃. 𝐸 of the body equals the workdone in between a magnet and a coil to produce electrical
penetrating into the mud, 𝑃. 𝐸 = 𝑊, and light energy.
𝑚𝑔ℎ = 𝐹 × 𝑠, 2×10×100 = 20× 𝑠, 35. Two identical boxes are to be carried up a
2000 flight of steps.A man X carries one box up in two
𝑠= = 100m.
20 ,but another man Y who stopped twice to rest,
30. A force of 10N acting continuously increases completed the assignment in six mimutes.What
the kinetic energy of an object from 20𝐽 to 60𝐽. observation is correct? A.It is impossible to
The distance moved by the object is then A.400m determine the workdone because a machine was
B.200m C.60m D.20m E.4m. not used B.the power developed by X is more
𝐹=10𝑁, initial kinetic energy, 𝐾. 𝐸 1=20J, final than the average power developed by Y C.the
kinetic energy, 𝐾. 𝐸 2=60J, 𝐹=10N, 𝑠=?, energy developed by X is more three times that
Work done-energy conversion – expended by Y D.the workdone by X is three
Workdone=change in kinetic energy, times that done by Y E.the workdone by Y was
𝑊 = ∆𝐾. 𝐸 , 𝐹 × 𝑠 = 𝐾. 𝐸 2−𝐾. 𝐸 1, less than that done by X.
40
10× 𝑠 = 60−20 = 40 , 𝑠 = = 4m. The energy developed or workdone by both men
10
31. A constant force acting on a body of mass 4kg is the same as they moved through the same
𝟏
increses the velocity of the body from 20ms-1 to flight of steps. 𝑷 ∝ i.e the higher the power
𝒕
40ms-1. Calculate the workdone by the force. developed by a body the lesser the time taken to
A.1.2×103J B.2.4×103J C.3.6×103J D.4.8×103J. do a certain amount of work.
𝑚=4kg, 𝑢=20ms-1, 𝑣=40ms-1, 𝑊=?, 36. A body is pulled through a distance of 500m
1 1
𝑊 = ∆𝐾. 𝐸 = 𝑚(𝑣2−𝑢2) = ×4×(402−202) , by a force of 20N. If the power developed is
2 2
𝑊 = 2×1200 = 2400𝐽 = 2.4×103𝐽. 0.4kW, calculate the time for which the force acts.
32. A body rolls down a slope from a height of A.250.0s B.25.0s C.2.5s D.0.5s.
100m. Its velocity at the foot of the slope is 20ms- 𝑠=500m, 𝐹=20𝑁, 𝑃=0.4kW=400W, 𝑡=?,
1. What percentage of its initial potential energy
𝑊 𝐹×𝑠 𝐹×𝑠 20×500
𝑃= = , 𝑡= = = 25s.
𝑡 𝑡 𝑃 400
is converted into kinetic energy? A.40% B.35% 37. A car of mass 𝑀kg is accelerated uniformly
C.20% D.15% from rest by an engine of power 𝑃. The minimum
mass of the body=𝑚, ℎ=100m, 𝑣=20ms-1, 𝑀𝑉 𝑀𝑉
time it will take to attain a speed 𝑉 is A. B.
Initial 𝑃. 𝐸 = 𝑚𝑔ℎ = 𝑚 ×10×100=1000𝑚J 𝑃 2𝑃
1 𝑴𝑽𝟐 𝑀𝑉 2
𝐾. 𝐸 at the foot of the slope= 𝑚𝑣 2 C. D. .
2 𝟐𝑷 𝑃
1 1
𝑚𝑣 2 𝑚𝑣 2 𝑚𝑣 2 𝑀𝑉 2
𝐾. 𝐸 = × 𝑚 ×202=200𝑚J, 𝑃=
𝐾.𝐸
= = 2
, 𝑡= = .
2 𝑡 𝑡 2𝑡 2𝑃 2𝑃
Percentange of the initial 𝑃. 𝐸 converted to 38. Starting from rest, a car of mass 1000kg
𝐾.𝐸 200
𝐾. 𝐸 = × 100= ×100 = 20%. accelerates steadily to 20ms-1 in 10s. The
𝑃.𝐸 1000

55
Demystified Series Physics Demystified by Dr Timothy
average power developed in the time period is 44. A steam engine of efficiency 70% burns 20g
A.0.2kW B.40kW C.10kW D.15kW E.20kW. of coal to produce 10kJ of energy. If it burns 200g
𝑚=1000kg, 𝑣=20ms-1, 𝑡=10s, 𝑃=?, of coal per second, calculate the output A.70kW
𝑃=
𝐾.𝐸
=
𝑚𝑣 2
=
1000×202
= 20000𝑊 =20k𝑊. B.80kW C.100kW D.200kW E.300kW.
𝑡 2𝑡 2×10 The energy obtained by burning the coal i.e fuel,
39. A girl of mas 48kg runs up 25 steps, each of inside the steam engine is the energy input. 20g
height 0.2m to reach the first floor of a storey of coal give 10kJ of energy when burnt, but the
building. If the power expended by the girl is steam engine actually burns 200g of coal per
400W, calculate the time taken (g=10ms-2) A.2s second.
B.3s C.4s D.5s E.6s. The energy obtained when 200g of coal per
𝑚=48kg, number of steps=25, height of one second can be gotten by direct proportion.
step=0.2m, Total height, ℎ=number of steps × 20g of coal → 10k𝐽 or 10000𝐽 of energy
height of one step=25×0.2=5m, 𝑃=400𝑊, 200g of coal per second → 𝑥𝐽 of energy
𝑃.𝐸 𝑚𝑔ℎ 𝑚𝑔ℎ 48×10×5
𝑃= = , 𝑡= = = 6s. 𝑥=
200×10000
= 100000𝐽 per second.
𝑡 𝑡 𝑃 400
20
40. A pump lift water into an overhead tank at a 100000Js-1 or 100000W is the power input.
height of 12m at the rate of 5kgs-1. The power of 𝑃 𝑃
the pump is A.60W B.600W C.720W D.1200W. 𝜖 = 𝑜 × 100, 70= 𝑜 × 100,
𝑃𝑖 100000
𝑚 70×100000
ℎ=12m, mass per unit time = = 5kgs-1, g=10ms- 𝑃𝑜 = = 70000W = 70kW.
𝑡 100
2,𝑃=?, 𝑃 =
𝑚𝑔ℎ 𝑚
= 𝑔ℎ = 5×10×12 = 600W. 45. The area under a force-velocity graph is equal
𝑡 𝑡
41. A car travelling with a uniform velocity of to A.Energy B.Momentum C.Power
30ms-1 along a horizontal road overcomes a D.Acceleration.
constant frictional force of 600N. Calculate the Area under force-velocity time graph = F × 𝑣 ,
power of the engine of the car A.18kW B.20kW Power P = Force×velocity = F𝑣.
C.180kW D.200kW. 46. A car travelling at 30ms-1 overcomes a
𝐹=frictional force overcomed=600N, 𝑣=30ms-1 firctional force of 100N while moving. Calculate
𝑃 = 𝐹𝑣 = 600×30 = 18000𝑊 = 18k𝑊. the power developed by the engine
42. A pump is used to raise water from a depth of (1hp=0.75kW) A.0.23hp B.0.40hp C.4.00hp
30m to fill a reservoir of volume 1800m3 in 5 D.4.40hp.
hours. Calculate the power of the pump (density 𝐹=100N, 𝑣=30ms-1, 𝑃=?, 𝑃 = 𝐹𝑣 ,
of water=1000kgm-3,g=10ms-2) A.5kW B.10kW 𝑃 = 100×30 = 3000W = 3kW.
3𝑘𝑊×1ℎ𝑝
C.15kW D.20kW. 0.75kW → 1hp ∴ 3kW = = 4.00hp.
0.75𝑘𝑊
ℎ=depth of water raised=30m, 𝑉=volume of Jamb past questions on work, energy and
water=1800m3, 𝑡=5hrs=5×3600=18000s, power :
g=10ms-2, 𝜌=density of water=1000kgm-3, [1984/39,1983/17,1988/11,1992/6,1997/3,
𝑚
𝜌 = , 𝑚 = 𝜌𝑉, 𝑃 =
𝑚𝑔ℎ
=
𝜌𝑉𝑔ℎ
, 2000/5,2006/29,2008/40,2009/10,2013/1019
𝑉 𝑡 𝑡
1000×1800×10×30 84/43,1985/41,1989/6,2010/8,2014/13,1979/
𝑃= = 20000W = 20kW. 44,2011/2,2001/3,1994/9,1997/12,1999/3,
18000
43. A water pump of 1.2kW rating pumps 480kg 1991/7,2013/9,1986/5,1985/12,1987/9,2003/
of water into an overhead tank at a height of 5m 5,2004/12,2008/11,1984/22,1991/9,2002/5,1
in 30 seconds. The efficiency of the pump is 990/10,2003/1,2,2007/41,2013/13]
A.90.0% B.80.0% C.66.7% D.62.5%.
Power input=1.2kW=1200W, m=480kg, ℎ=5m,
𝑚𝑔ℎ
𝑡=30s, g=10ms-2, Power input= ,
𝑡
480×10×5
Power input= = 800W,
30
𝑃𝑜𝑤𝑒𝑟 𝑜𝑢𝑡𝑝𝑢𝑡 𝑃𝑜
Efficiency, 𝜖 = ×100 = × 100 ,
𝑃𝑜𝑤𝑒𝑟 𝑖𝑛𝑝𝑢𝑡 𝑃𝑖
𝑊𝑜𝑟𝑘 𝑜𝑢𝑡𝑝𝑢𝑡
Efficiency, 𝜖 = × 100
𝑊𝑜𝑟𝑘 𝑖𝑛𝑝𝑢𝑡
800
𝜖= × 100= 66.7%.
1200

56
Demystified Series Physics Demystified by Dr Timothy

CHAPTER 5 – FORCE FIELD AND GRAVITATIONAL FIELD


● Force field or field forces – These are forces The weight of an object on the moon is due to
which acts on a body in a region or space without gravitational effect of the moon on the object.
contact with the body. All field forces originates - Weightlessness – A state when a body feels no
from the centre and obey inverse square law e.g weight or in the absence of gravitational force. It
gravitational field forces, magnetic field forces occurs at infinite(great) height above the earth
and electric field forces. due to decreased force of attraction or
- Gravitational field forces are always acceleration in space , W ∝ 𝟐 .
𝟏

attractive only while Magnetic and electric 𝒓


- During weightlessness –
field forces are both attractive and repulsive.
I.The reaction on a body and the resultant force
- Conservative field – is a field where workdone
acting on a body is zero, the acceleration of the
by a force depends on the displacement, the
body equal to the gravitational acceleration , 𝑎=g.
workdone in moving a body around a closed
II.The centripetal force required by the moving
path is zero e.g gravitational field.
body is equal to or provided by the gravitational
- Non-conservative field or dissipative field –
force of attraction i.e 𝐹 C= 𝐹 g , as the weight is
is a field where the workdone by a force depends
equal to zero , W=0.
on distance covered by the force e.g friction and
- Factors affecting weight W or acceleration
viscosity.
due to gravity g (why they vary) :
- Gravitational field – is a region or space
I.Shape of the earth(earth is not spherical but
around a mass in which the gravitational force of
elliptical or bulge effect) – The value of g at the
the mass can be felt. Every massive body has a
poles is greater than at the eqauator
gravitational field around it.
gpoles>gequator, due to the shape of the earth as 𝑟
- Properties of gravitational field :
is smaller at the poles that at the equator. Thus, a
1.They act on massive bodies or planets.
body weighs more at the poles than the equator.
2.The field lines always ends at a mass.
II.Rotation of the earth on its axis(centripetal
3.They are usually attractive.
force effect) – The centripetal force required for
4.The force does not cause a deflection on the
the rotation of the earth is provided by part of
massive bodies in the field.
the total gravitational force, 𝐹𝑔 = 𝐹𝑐 + 𝑊,
5.The forces acts in the same direction with the
field. 𝐹𝑐 = 𝐹𝑔 − 𝑊
- Newton’s law of universal gravitation – force III.Variation (or Change) in latitude, longitude
exerted on two massive bodies 𝑚1 and 𝑚2 is and altitude (or height).
directly proportional to the product of their - The density of the earth does not affect the value
masses and inversely proportional to the square of g or W.
𝒎 𝒎 𝑮𝒎 𝒎 - The weight of a body depends on its mass 𝑚 and
of their distance apart 𝑟2 . 𝑭 ∝ 𝟏𝟐 𝟐 , 𝑭 = 𝟏𝟐 𝟐
𝒓 𝒓 acceleration due to gravity 𝑔. 𝑾 = 𝒎𝒈.
G=universal gravitational constant=6.67×10- - Weight of a body changes with the variation in
11Nm2kg-2.
acceleration due to gravity, 𝑾 ∝ 𝒈 (the mass is
- If both masses are double and the distance constant).
between them is doubled, the gravitational force - The weight of a body at different planets or
of attraction remains constant or the same. different point in the earth surface and
𝟏
- 𝑭 ∝ 𝟐 thus, force 𝐹 increases as distance 𝑟 acceleration due to gravity at that point is related
𝒓 𝑾 𝑾
reduces. 𝑭1𝒓21 = 𝑭2𝒓22 . by : 𝑾 ∝ 𝒈 , 𝟏 = 𝟐 .
𝒈𝟏 𝒈𝟐
- Graphical representation of the variation of - Mass of a body is constant at different
gravitational force 𝐹 between two objects and locations as it is independent of acceleration
the distance 𝑟 between their centres : due to gravity.
𝑭 𝑭 - The acceleration due to gravity on earth
depends on the mass of the earth 𝑴 (and not
the mass of the object) and the radius of the
𝟏/𝒓2 𝒓 earth 𝑹.
- A body revolving round the earth is kept on its 𝑮𝑴
- At the earth surface – 𝒈 = 𝟐 .
orbit by two opposite forces: force(acting 𝑹
towards the centre) and centrifugal forces - At an altitude or height h above the earth
𝑮𝑴
(acting away from the centre). surface – 𝒈1 = (𝑹+𝒉)𝟐 . 𝑀=mass of the earth,
- The force with which an object is attracted to 𝑅=radius of the earth=distance of a body from
the earth is Weight or force of gravity. the earth’s centre.
- The weight of an object on earth is due to the
gravitational effect of the earth on the object.
57
Demystified Series Physics Demystified by Dr Timothy
𝒈𝟏
- < 1 , 𝒈1 < 𝒈 hence, acceleration due to 𝒗𝒆 = √𝟐𝒈𝑹 = √
𝟐𝑮𝑴
=√
𝟐𝑷.𝑬
.
𝒈 𝑹 𝒎
gravity decrease with altitude/height (𝒈 ∝ 1/𝒉). - Satellite are small bodies which move in orbits
- The acceleration due to gravity on the earth round a large astronomical body. Natural
surface 𝑔 and acceleration due to gravity at satellites are the earth(or other planets) moving
height ℎ above the earth surface g1 are related by in orbits round the sun or the moon moving in
𝒈 𝑹+𝒉 𝒈𝑹𝟐
– 𝒈1= , 𝒅𝑹 𝟐 = OR 𝒈1 = (𝑹+𝒉)𝟐 . orbit round the earth. Artificial satellites are
𝒅𝑹 𝟐 𝑹
space crafts.
- The distance from the earth centre to the earth
- Orbital speed or speed of objects/satellite in
surface is equal to the radius of the earth 𝑅. If a 𝟐𝝅𝒓
body is at distance 𝑅 from the earth surface then, gravitational field is given by – 𝒗𝒔 = ,
𝑻
its distance from the earth centre is 2𝑅. 𝑟=orbital radius, 𝑇=orbital period,
- The acceleration due to gravity on moon is one- 𝑮𝑴 𝑷.𝑬
𝟏 𝒗𝒔 = √𝒈𝑹 = √ =√ .
sixth the value on earth. 𝒈moon= 𝒈earth. Hence, a 𝑹 𝒎
𝟔
body weighs more on earth than on moon. - The escape velocity of a satellite and the speed
- The acceleration due to gravity in different of the satellite round its obit are related by –
planets of mass 𝑀1 and 𝑀2 and radius 𝑅 1 and 𝑅 2 𝒗𝒆 = 𝒗𝒔 √𝟐.
𝒈𝟐 𝑴𝟐 𝑹𝟏 𝟐 𝟏 - With a velocity of about 8kms-1, a satellite
are related by – = ( ) [𝒈∝𝑴∝ ]. moves in a circular orbit close to the earth’s
𝒈𝟏 𝑴𝟏 𝑹𝟐 𝑹𝟐
- Gravitational field intensity or force surface.
intensity 𝑬𝑮 – at any point in a gravitational field - With a velocity greater than 8kms-1 but less than
is defined as the force per unit mass on a body at 11kms-1, a satellite moves in an elliptical orbit
𝑭
that point. 𝑬𝑮 = = 𝟐 .
𝑮𝑴 round the earth.
𝒎 𝒓 - With a velocity above 11kms-1, the satellite
- 𝑬1𝒓12 = 𝑬2𝒓22 , 𝑮𝑴 =constant.
completely escapes from the gravitational
- Gravitational field intensity is the same as
attraction of the earth.
acceleration due to gravity (force per unit
- Geosynchronous or Parking orbit – A
mass).
satellite is in a parking or geosynchronous
- When a massive body moves in the same
orbit if its period round its orbit exactly
(downward) or opposite (upward) direction of a
equals the period of rotation of the earth(as it
uniform gravitational force field, will experience
turns about its axis within 24hrs). The satellite
a constant or uniform acceleration and
will stay on the same place over the earth as the
deceleration of 9.8ms-2 respectively.
earth rotates, as they are at constant distance
- Gravitational potential 𝑽 – at a point is the
equal to the radius of the earth. The satellite is
workdone W in a taking a unit mass m from
said to be geostationary.
infinity to that same point. 𝑽 = 𝒈𝒓 = 𝑬G𝒓 .
- Parking orbits can be used to transmit
- Gravitational potential at the earth surface
𝑮𝑴 television programmes continuously from one
where 𝑟 =radius of the earth, 𝑅 is – 𝑽 = . part of the world to another.
𝑹
- The gravitational potential at any point is – - Period 𝑇 and speed of a satellite 𝑣𝑠 is
𝑽=−
𝑮𝒎
. The negative sign indicvates that the independent of size or mass of a body but
𝒓
depends on radius of the earth and acceleration
potential at infinity i.e zero, is higher than that
close to the mass producing the gravitational 𝟒𝝅𝟐 𝒓𝟑 𝒈𝑹𝟐
due to gravity. 𝑻 = √ , 𝒗𝒔 = √ .
𝒈𝑹𝟐 𝒓
field.
- Gravitational potential energy 𝑷. 𝑬 – It is the - Kepler’s Laws – Kepler’s law of planetary
workdone in taking a mass 𝑚 from infinity to the motion describe the orbits of objects about the
𝑮𝑴𝒎 sun.
earth surface of mass 𝑀. 𝑷. 𝑬 = −𝒎𝑽 = − .
𝒓 1st Law – The planets each travel along an ellipse
- Gravitational kinetic energy 𝑲. 𝑬 – This is half with the sun at one focus.
the gravitational potential energy but of opposite 2nd Law – The line joining the sun and the
𝑷.𝑬 𝒎𝑽 𝑮𝑴𝒎
sign. 𝑲. 𝑬 = = = . planets sweeps out equal areas in equal times
𝟐 𝟐 𝟐𝒓
- Work is done against gravity when a body is interval.
lifted or raised above the ground (energy is 3rd Law – The square of the period 𝑇 of
absorbed). Work is done by gravity when a revolution of the planets are proportional to the
body is brought to the ground from a height cubes of their mean distance 𝑟 from the sun i.e
(energy is released). 𝑇 2 ∝ 𝑟 3.
- Escape velocity 𝒗𝒆 - It is the minimum velocity
with which an object would just escape from the Examples :
influence of the earth’s gravitational field. 1. Which of the following are contact forces?
I.force of tension II.force of friction III.magnetic

58
Demystified Series Physics Demystified by Dr Timothy
force IV.force of reaction A.I,II and III B.I,II and C.6.3×108N D.1.0×1024N E.9.8×1025N.
IV C.I,III and IV D.II,III and IV E.I and IV . 𝑚1=10kg, 𝑚2=mass of the earth=6.0×1024kg,
Magnetic force, gravitational force and electric 𝑟=radius of the earth surface=6.4×106m,
force are field forces. 𝐹=
𝐺𝑚1 𝑚2
=
6.7×10−11 ×10×6.0×1024
=
40.2×1014
,
2. Which of the following statements is true of the 𝑟2 (6.4×106 )2 40.96×1012

law of universal gravitation? A.Any twobodies 𝐹= 0.98×102 = 9.8×101N.


attract each other with a force which is 7. A planet has a mass 𝑚1 and is at a distance 𝑟1
proportional to the difference of their masses from the sun. A second planet has mass 𝑚2=10𝑚1
B.Any two bodies attract each other with a force and is at a distance of 𝑟2=2𝑟1 from the sun.
which is inversely proportional to the square of Determine the ratio of the gravitational force
the distance between them C.Any two bodies experienced by the planets A.1:5 B.2:5 C.3:5
attract each other with a force which is D.4:5.
proportional to the product of their masses The gravitational force between 𝑚1 and the sun
𝐺𝑀𝑚1
and inversely proportional to the square of of mass 𝑀 is – 𝐹 1= 2 , 𝑟1
the distance between them D.Any two bodies The gravitational force between 𝑚2 and the sun
attract each other with a force which is directly 𝐺𝑀𝑚2 𝐺𝑀10𝑚 10𝐺𝑀𝑚1
of mass 𝑀 is – 𝐹 2= = (2𝑟 )2 1 = 2 ,
proportional to the square of the distance 2 𝑟2 1 4𝑟1
between them E.Any two bodies attract each The ratio of the gravitational forces,
𝐺𝑀𝑚1
other with a force which is proportional to the 𝐹1 (
𝑟1 2
) 𝐺𝑀𝑚1 4𝑟1 2 2
sum of their masses. 𝐹 1:𝐹 2 = = 𝐺𝑀10𝑚1 = × = =2:5.
𝐹2 𝑟1 2 𝐺𝑀10𝑚1 5
4𝑟1 2
3. Which of the following statements about the
8. A rocket is fired from the earh’s surface to a
universal gravitational constant, G is true? A.It
distant planet. By Newton’s law of universal
is a fundamental quantity B.It has dimension
gravitation, the force 𝐹 will A.increase as 𝒓
M-1L3T-2 C.Its S.I unit is Nm2kg-1 D.It is a vector
reduces B.increases as 𝐺 varies C.remains
quantity.
constant D.increases as 𝑟 increases.
The universal gravitational constant is a derived 1
quantity and a scalar quantity. Its unit is Nm2kg- 𝐹 ∝ 2 , the force 𝐹 increases as 𝑟 reduces.
𝑟
2. Its value is constant and does not vary. Universal gravitational constant 𝐺 does not vary,
4. The mass of the earth is 6.0×1024kg and that of it is constant.
the moon is 7.0×1022kg. If the distance between 9. Which of the sketched graphs below illustrates
them is 4.0×108m, calculate the force of the correct variation of the gravitational force 𝐹 g
attraction between them between two objects and the distance 𝑑 between
[𝐺=6.7×10-11𝑁m2kg-2] A.1.8×105N B.1.8×1010N their centres?
C.1.8×1015N D.1.8×1020N . A. 𝐹 g B. 𝑭g
𝑚1=6.0×1024kg, 𝑚2=7.0×1022kg, 𝑟=4.0×108m,
𝐺=6.7×10-11𝑁m2kg-2,
𝐺𝑚1 𝑚2 6.7×10−11 ×6.0×1024 ×7.0×1022 𝑑 𝒅
𝐹= = ,
𝑟2 (4×108 )2 C. 𝐹 g D. 𝐹 g
281.4×1035
𝐹= = 1.76×1020 ≈ 1.8×1020N.
16×1016
5. Two objects P and Q of masses 100kg and
200kg respectively are 1.2m apart on a 𝑑 𝑑.
horizontal surface. Calculate the net gravitational Gravitation force of attraction 𝐹 g betweeb two
force acting on Q (𝐺=6.67×10-11𝑁m2kg-2) objects is inversely proportional to the distance
A.1.11×10-7N towards P B.1.11×10-7N towards Q between them. Thus 𝐹 g decreases as 𝑟 increases
C.9.26×10-8N towards P D.9.26×10-8N towards as shown in the graph in option B.
Q. 10. If the distance between two suspended
𝑚1=100kg , 𝑚2=200kg , 𝑟=1.2m, 𝐺=6.67×10-11N masses 10kg each is tripled, the gravitational
𝐺𝑚1 𝑚2 6.67×10−11 ×100×200 force of attraction between them is reduced by
m2kg-2, 𝐹=?, 𝐹= = , A.one-quarter B.one-ninth C.one half D.one
𝑟2 (1.2)2

𝐹=
1.334×107
= 9.26×10-8N. The net gravitational third.
1.44 𝐹 1=𝐹, 𝑟1=𝑟, 𝐹 2=?, 𝑟2=3𝑟, 𝐹 1𝑟12= 𝐹 2𝑟22,
force on Q is same as the gravitational force 𝐹𝑟 2 𝐹
between P and Q. Gravitaional force is force of 𝐹 × 𝑟2= 𝐹 2× (3𝑟)2, 𝐹 2= 2 = , force is one-
9𝑟 9
attraction, hence Q is attracted to P or move ninth its initial value. Alternatively,
towards P. 1 1 1
𝐹 ∝ 2 , the distance 𝑟 is tripled, 𝐹 = 2 = , one-
𝑟 3 9
6. Calculate the force on a mass 10kg placed on ninth the initial value.
the earth’s surface. [Radius of the earth= 11. The force of 200N acts between two objects
6.4×106m, mass of the earth=6.0×1024kg, at a certain distance apart. The value of the force
𝐺=6.7×10-11𝑁m2kg-2) A.9.8N B.9.8×101N when the distance is halved is A.100N B.200N

59
Demystified Series Physics Demystified by Dr Timothy
C.400N D.800N. 𝑀E=6.0×1024kg) A.6.3ms-2B.3.7ms-2 D.0.4ms-2
𝐹 1=200N, 𝑟1=𝑟, 𝐹 2=?, 𝑟2=𝑟/2, 𝐹 1𝑟12= 𝐹 2𝑟22, D.9.8ms-2
200× 𝑟2= 𝐹 2× (𝑟/2)2 = 𝐹 2× 𝑟 2 /4 , 𝐺=6.67×10-11𝑁m2kg-2, radius of the earth,
𝐹 2=
200×𝑟 2 ×4
= 800N. Alternatively, 𝑟E=𝑅=6400km=6400×103m, height above earth
1
𝑟2
1 1 surface, ℎ=4000km=4000×103m, mass of the
𝐹∝ , the distance is halved, 𝐹 = (1/2)2 = , earth, 𝑀E=𝑀=6.0×1024kg,
𝑟2 1/4
𝐹 = 4 , four times the original value, 𝑔1= acceleration due to gravity at a height ℎ
𝐹 2 = 4×200 = 800N. above the earth’s surface,
6.67×10−11 ×6×1024
12. The acceleration due to gravity may be 𝐺𝑀
𝑔1 = (𝑅+ℎ)2 = (6400×103 ,
+4000×103)2
defined as the force A.of attraction of the sun on 40.02×1013 40.02×1013
the earth B.with which the earth revolves around 𝑔1 = = = 3.7ms-2.
(10400×10^3)2 1.0816×1014
the sun C.with which the earth attracts one- 18. If a particle of constant mass m moves in the
kilogram mass D.of the moon on the earth. direction of a uniform gravitational force field, its
13. A body moves in a circular orbit of radius 4𝑅 acceleration will A.increase B.decrease
round the earth. Express the acceleration of free C.uniform D.be constant.
fall due to gravity of the body in terms of 𝑔 When a massive body moves in the same
[𝑅=radius of the earth, g=acceleration of free fall (downward) or opposite (upward) direction of a
16 𝒈
due to gravity on the earth’s surface] A. B. uniform gravitational force field, will experience
𝑔 𝟏𝟔
4 𝑔 𝑔 a constant or uniform acceleration and
C. D. E. .
𝑔 4 8 deceleration of 9.8ms-2 respectively.
𝑔 𝑅+ℎ 4𝑅 𝑔 𝑔
𝑔1 = , 𝑑𝑅 2 = = =4, 𝑔1= = . OR 19. The acceleration of free fall due to gravity in
𝑑𝑅 2 𝑅 𝑅 42 16
1 a certain location is 9.707ms-2, calculate the
𝑔 ∝ 2 , 𝑔1𝑟12 = 𝑔2𝑟22, 𝑔1=𝑔, 𝑟1=𝑅, 𝑔2=?, 𝑟2=4𝑅, gravitational attraction a piece of cork of mass
𝑟
𝑔 × 𝑅 2 = 𝑔2×(4𝑅)2, 𝑔 × 𝑅 2 = 𝑔2×16𝑅 2, 50g at this location A.1.94×10-1𝑁B.4.85×10-1𝑵
𝑔𝑅 2
𝑔2 = , 𝑔2 = .
𝑔
C.1.94×102N D.4.85×102N
16𝑅 2 16
g=9.707ms¬2, 𝑚=50g=0.05kg, the gravitational
14. What is the acceleration due to gravity ‘𝑔’ on
the moon if g is 10ms-2 on the earth? A.0.10ms-2 attraction of the piece of cork is its weight,
B.0.74ms-2 C.1.67ms-2 D.10.0ms-2. 𝑊 = 𝑚𝑔 = 9.707×0.05=4.85×10-1𝑁.
1 1 20. The mass and weight of a body on earth are
𝑔moon= 𝑔earth= × 10 =1.67ms-2. 8kg and 80𝑁 respectively. Determine the mass
6 6
15. The earth is four times the size of the moon and weight of the body on a planet where the pull
and the acceleration due to gravity on the earth 1
of gravity is that on earth A.8kg, 8N B.1kg, 10N
is 80 times that on the moon. The ratio of the 8

mass of the mass of the moon to that the earth is C.64kg, 10N D.8kg, 10N.
Mass of a body is constant in all location in the
A.1:320 B.1:1280 C.1:80 D.1:4
𝑅 e=radius(size) of the earth, 𝑅 m=radius(size) of universe, it does not change. Pull of gravity is
the moon, 𝑔e=acceleration due to gravity on same as weight. Weight of the body on earth,
earth, 𝑔m=acceleration due to gravity on moon, 𝑊 e=80𝑁, Weight of the body on planet, 𝑊 p=
1 1
𝑅 𝑔 1
𝑅 e=4𝑅 m, 𝑒 =4, 𝑔e=80𝑔m, 𝑚 = , 𝑀𝑚 :𝑀𝑒 =?, 𝑊 e= ×80=10𝑁. The mass 𝑀=8kg, while its
8 8
𝑅𝑚 𝑔𝑒 80
weight 𝑊 P=10N.
𝑔2 𝑀2 𝑅1 2 𝑔𝑚 𝑀𝑚 𝑅𝑒 2 1 𝑀𝑚
= ( ) , = ( ) , = (4)2 , 21. The gravitational force of the moon is one
𝑔1 𝑀1 𝑅2 𝑔𝑒 𝑀𝑒 𝑅𝑚 80 𝑀𝑒
1 𝑀𝑚 𝑀𝑚 1 1 𝑀𝑚 1 sixth that of the earth. If a body weighs 6.0𝑁 on
= × 16 , = × , = ,
80 𝑀𝑒 𝑀𝑒 80 16 𝑀𝑒 1280 the moon, calculate its weight on the earth
𝑀𝑚 :𝑀𝑒 =1:1280. A.1.0𝑁 B.2.0𝑁 C.6.0𝑁 D.12.0𝑁E.36.0𝑵
1 1
16. The mass of the moon is 𝑀 and its radius is 𝑔e=𝑔, 𝑔m= 𝑔e , 𝑊 e=?, 𝑊 m=6.0N,
81 6
1 𝑊1 𝑊2 𝑊𝑒 𝑊𝑚
𝑅, where 𝑀 and 𝑅 are mass and radius of the 𝑊 ∝ g, = (constant mass), = ,
4 𝑔1 𝑔2 𝑔𝑒 𝑔𝑚
earth respectively. Calculate the acceleration due 𝑊𝑒 6
= 1 , 𝑊𝑒 =
6𝑔
=
6𝑔×6
= 36𝑁.
𝑔
to gravity on the surface of the moon (Take g on 𝑔
6
𝑔 6
𝑔
earth=9.8ms-2) A.1.94ms-2 B.7.56×10-3ms-2 22. The weight of an object on the earth is 600%
C.1.27ms D.9.8ms-2
¬2 of its weight on the moon. If the object weighs
1 𝑀 1
𝑀𝑚 = 𝑀, 𝑚 = , 𝑅𝑚 = 𝑅,
1 𝑅
= 4, 𝑔e=9.8ms-2, 60N on earth, determine its weight on the moon
81 𝑀 81 4 𝑅𝑚
A.0.1N B.6.0N C.10.0N D.36.0N
𝑔2 𝑀2 𝑅1 2 𝑔𝑚 𝑀𝑚 𝑅𝑒 2
𝑔m=?, = ( ) , = ( ) , 600 1
𝑊 e= 𝑊 m=6𝑊 m, 𝑊 m= 𝑊 e, 𝑊 e=60N,
𝑔1 𝑀1 𝑅2 𝑔𝑒 𝑀𝑒 𝑅𝑚 100 6
𝑔𝑚 1 1 1
= ×42, 𝑔𝑚 = 9.8 × × 16 = 1.94ms-2. 𝑊 m= ×60 = 10𝑁.
9.8 81 81 6
17. What is the value of the acceleration due to 23. A missle weighing 400N on earth’s surface is
gravity at a point 4000km above the earth’s shot into the atmosphere to an altitude if
surface? (𝐺=6.67×10-11𝑁m2kg-2, 𝑟E=6400km, 6.4×106m, taking the earth as a sphere of radius

60
Demystified Series Physics Demystified by Dr Timothy
6.4×106m and assuming the inverse square law 31. If 𝑀 and 𝑅 are the mass and radius of the
of universal gravitational , what would be the earth respectively and 𝐺 is the universal
weight of the missle at that altitude? A.100N gravitational constant, the earth’s gravitational
B.200N C.400N D.800N E.1600N. potential at an altitude H above the ground level
𝑊=400𝑁, ℎ=6.4×106m, 𝑅=6.4×106m, 𝑊 1=?, is A.−
𝐺𝑀
B.−
𝑮𝑴
C.−
𝐺𝑀
D.−
𝐺𝑀
.
𝑊 𝑅+ℎ 𝑊 𝐻 𝑹+𝑯 2𝐻 𝑅−𝐻
𝑊 1= 2 , 𝑑𝑅 2 = , 𝑊 1= (𝑅+ℎ/𝑅)2 𝐺𝑚 𝐺𝑀
𝑑𝑅 𝑅 𝑉=− , 𝑚=𝑀, 𝑟=𝑅 + 𝐻, 𝑉 = − .
2 2 𝑟 𝑅+𝐻
6.4×106
𝑊 1= 𝑊 (
𝑅
) = 400× ( ) , 32. In the eaerth’s surface. (Radius of the
𝑅+ℎ 6.4×106 +6.4×106
2 earth=6.4×106m, mass of the earth= 6.0×1024kg,
6.4×106 1 2 400
𝑊 1= 400× ( ) = 400× ( ) = , 𝐺=6.67×10-11Nm2kg-2] A.2.1×107Jkg-1
12.8×106 2 4
1
𝑊 =100𝑁. B.3.2×107Jkg-1 C.4.3×107Jkg-1 D.6.3×107Jkg-1.
24. In the absence of gravitational force, the 𝑅=6.4×106m,𝑀=6.0×1024kg,𝐺=6.67×10-11Nm2
weight of a body is A.its mass B.zero C.its density kg-2, 𝑉=?, Gravitational potential at the earth
𝐺𝑀 6.67×10−11 ×6×1024
D.its volume. surface, 𝑉= = = 6.3×107Jkg-1.
𝑅 6.4×106
25. I.The earth is not spherical but elliptical in 33. In the earth’s gravitational field A.work is
shape II.Variation in latitude and longitude done when a force moves its point of application
III.Rotation of the earth on its axis IV.Variation in around a closed path B.bodies repel themselves
the density of the earth. On which combination of when they come close C.energy is released when
the above does the weight of an object vary on the an object is raised above the ground D.a body
earth’s surface? A.I and II B.I,II and III C.II,III and experiences no decrease in deceleration
IV D.I,II,III and IV. when its moves freely upwards.
26. An astronaut experiences weightlessness in No work is done by a force around a closed path
space when he A.does not hold anything while in in a gravitational field(a conservative field) i.e
space B.is midway between the sun and the earth workdone is zero round a closed path.
C.is free from the earth’s gravitational field Gravitational force is a force of attraction, hence
D.walking on the moon bodies brought close to one another attracts
27. The gravitational field intensity I.has the unit themselves.
Nkg-1 II.is equal to the acceleration of free fall due When a body is raised above the ground in a
to gravity III.increases with altitude. Which of gravitational field, work is done against the field
the statements above is/are correct? A.I B.II C.I and energy is absorbed. When a body is brought
and II D.II and III E.I,II and III the ground from a height during free fall, work is
Gravitational field intensity is equal to the done by the field and energy is released.
acceleration of free fall due to gravity. A body moving upward in a gravitational field
𝐹
𝐸𝐺 = g = i.e force per unit mass (Nkg-1). experiences a constant deceleration of 9.8ms-2
𝑚
𝐸𝐺 and g decreases with altitude or height. while a body falling freely downward in a
28. The magnitude of the gravitational attraction gravitational field experiences a constant
between the earth amd a particle is 40N. If the acceleration of 9.8ms-2.
mass of the particle is 4kg, calculate the 34. A satellite of mass 𝑚 moves in an orbit
magnitude of the gravitational field intensity of around the earth of mass 𝑀. If the radius of the
the earth on that particle. A.10.0Nkg-1 orbit is 𝑟,the kinetic energy of the satellite can be
B.12.6Nkg-1 C.25.0Nkg-1 D.160.0Nkg-1 expressed as the product of the gravitational
𝐹 40 𝑀𝑚 𝑀𝑚 𝑀𝑚 𝑴𝒎 2𝑀𝑚
𝐹=40𝑁, 𝑚=4kg, 𝐸𝐺 =?, 𝐸𝐺 = = = 10𝑁kg-1. constant and A.− B.− C. 2 D. E. 2 .
𝑚 4 𝑟 2𝑟 𝑟 𝟐𝒓 𝑟
29. The gravitational field intensity at a location Gravitational kinetic energy is equal to half the
X in space is two-fifths of its value on the earth’s gravitational potential but of opposite sign.
𝐺𝑀𝑚
surface. If the weight of an object at X is 4.80N, 𝑃. 𝐸 = −𝑚𝑉 = − .
𝑟
what is its weight on the earth? A.1.92N B.2.88N 𝑚𝑉 𝐺𝑀𝑚 𝑀𝑚
𝐾. 𝐸 = = =𝐺× , hence the kinetic
C.6.72N D.8.00N E.12.00N. 2 2𝑟 2𝑟
Gravitational field intensity is same as energy of the satellite can be expressed as the
𝑀𝑚
acceleration due to gravity g.
2
𝑔𝑋 = 𝑔𝑒 , product of the gravitational constant 𝐺 and .
5 2𝑟
𝑊𝑒 𝑊𝑋 𝑊𝑒 4.8 35. The escape velocity of a rocket at location 𝑅
𝑊𝑋 =4.80N, 𝑊𝑒 =?, = , = 2 ,
𝑔𝑒 𝑔𝑋 𝑔
5
𝑔 meters above the surface of the earth of mass 𝑀
5×4.8×𝑔
𝑊𝑒 = = 12N. and radius 𝑅, is given by A.√2𝐺𝑀𝑅 B.√
2𝐺𝑀
2𝑔
𝑅
30. The gravitational potential at a point in a
𝐺𝑀 √2𝐺𝑀 𝑮𝑴
gravitational field is the workdone in taking a C.√ D. E.√ .
2𝑅 𝑅 𝑹
unit mass from A.zero to infinity B.infinity to
the point C.one point to another point D.a 𝑣𝑒 = √
2𝐺𝑀
, The distance of the rocket above the
𝑅
distance to a given point.

61
Demystified Series Physics Demystified by Dr Timothy
earth’s surface is 𝑅, hence its distance from the The period of revolution of the earth is
2𝐺𝑀 𝐺𝑀 independent of mass or size i.e no dependent on
earth’s centre= 𝑅 + 𝑅=2𝑅, 𝑣𝑒 = √ =√ . mass, but depends on the radius of the earth and
2𝑅 𝑅
36. The gravitational potential energy of a body acceleration due to gravity.
of mass 5kg , situated at a point within the earth’s 39. A satellite is in a parking orbit, if its period is
gravitational field is 3.25×108𝐽. Calculate the A.more than the period of the earth B.equal to
magnitude of the escape velocity of the body the period of the earth C.the square of the
A.5.50×104ms-1 B.1.12×104ms-1 period of the earth D.less than the period of the
C.6.25×10 ms
4 -1 D.3.60×103ms-1. earth.
2𝑃.𝐸 40. A geostationary satellite moves in an orbit of
𝑃. 𝐸=3.25×108𝐽, 𝑚=5kg, 𝑣𝑒 =? 𝑣𝑒 = √ , radius 6300km. Calculate the speed with which it
𝑚

2×3.25×108
moves in the orbit (𝜋=22/7) A.115ms-1 B.153ms-
𝑣𝑒 = √ = √1.3 × 108 = 1.12×104ms-1. 1 C.229ms-1 D.458ms-1.
5
37. The radius of the earth is 6.4×106m and the 𝑟=6300km=6300×103m, geostationary satellite
acceleration due to gravity is 10.0ms-2. The has a period is equal to the period of rotation of
escape velocity of a rocket launched from the the earth(24hrs), 𝑇=24hrs=24×3600=86400s,
2𝜋𝑟 2𝜋×6300×103
earth’s surface is A.4.2kms-1 B.4.0kms-1 𝑣=?, 𝑣 = = = 145.83𝜋 ,
𝑇 86400
C.11.3kms-1 D.25.3kms-1. 22
𝑣 = 145.ƒ × = 458ms-1 .
𝑅=6.4×106m, g=10ms-2, 𝑣𝑒 =?, 𝑣𝑒 = √2𝑔𝑅 7

𝑣𝑒 = √2 × 10 × 6.4 × 106 = √128 × 106 , Jamb past questions on gravitational field :


𝑣𝑒 = 11.3×103ms-1 = 11.3kms-1. [1995/8,2001/7,2004/17,2015/1,2013/2,2010
38. What would happen to the earth’s period of /6,2014/9,1993/7,1995/9,2003/13,2006/17,
revolution if its mass was three times as great as 2014/8,1997/8,2012/8,1998/6,2002/3,2005/
it is? A.increase 3 times B.decrease 3 times 32,2007/43,1980/1,1983/6,2015/5]
C.decrease 6 time D.remain the same

62
Demystified Series Physics Demystified by Dr Timothy

Chapter 6 – Equilibrium of forces


● Equilibrium of a body – A body is said to be about any point is equal to the sum of
in equilibrium when the resultant or net force anticlockwise moment about that same point i.e
acting on the body is zero and its acceleration is the algebraic sum or resultant moment is zero.
zero. - The moment is taken about a fixed point called
- Static equilibrium – Condition of a body when the fulcrum or pivot 𝑷.
the net force acting on the body is zero and the 𝑭1 𝑹 𝑭2
body is at rest. 𝒙1 𝒙2
- Dynamic equilibrium – Condition of a body
when the net force acting on the body is zero and 𝒙3 𝑷 𝒙4
the body is moving with a uniform velocity in a 𝑭3 𝑭4
straight line or rotating with a uniform angular I. Sum of upward forces=Sum of downward force
velocity. or Sum forces acting one direction=Sum of forces
- Moment of a force – This is the turning effect acting in the other direction. 𝑭1+𝑭2 = 𝑭3+𝑭4.
produced by the force. It is the product of force The algebraic sum of forces or resultant force is
and perpendicular distance in the line or point of zero, 𝑭1+𝑭2−𝑭3−𝑭4=0.
action of the force. Moment of a force, 𝑴 = 𝑭𝒅. II. Sum of clockwise moment (𝐹 1𝑥 1+𝐹 4𝑥 4) = Sum
- When the forces acting on a body are parallel of anticlockwise moment (𝐹 2𝑥 2+𝐹3𝑥 3).
to the distance at which they act, then the 𝑭1𝒙1+𝑭4𝒙4 = 𝑭2𝒙2+𝑭3𝒙3.
moment of the force is zero(0Nm), because III. The algebraic sum of moments or resultant
moment deals with perpendicular distance force is zero . 𝑭1𝒙1+𝑭4𝒙4−𝑭2𝒙2−𝑭3𝒙3=0.
not parallel distance. - 𝑹 is the reaction of the support at 𝑷 on the
- A force will produce zero difference in moment body. 𝑹 has no moment about 𝑷, as the no
when it acts on a body at different points but with perpendicular distance between 𝑹 and 𝑷 i.e
the same perpendicular distance. the distance between 𝑹 and 𝑷 =0.
- The resolved component of the distance - Centre of gravity C.G – of a body is the point
perpendicular to a force will produce moment of where its resultant weight(or resultant pull of
the force e.g if a force 𝐹 acts on a body at a the earth) is concentrated or appears to act.
distance 𝑑 inclined to the vertical or horizontal, - Position of the centre of gravity of regular
the component of the distance perpendicular to uniform or symmetrical bodies –
the force is 𝒅𝒔𝒊𝒏θ (vertical) and 𝒅𝒄𝒐𝒔θ I.The centre of gravity of a uniform meter rule is
(horizontal). at the midpoint i.e 50cm mark.
- Couple – It is the combination of two parallel, II.The centre of gravity of a uniform rod, lever or
equal and opposite forces, whose line of action do beam is at the midpoint.
not coincide e.g turning of car steering wheel, III.The centre of gravity of a uniform triangular
turning of a tap with our fingers. body is at the point of intersection of its medians.
- The resultant force acting on a couple is IV.The centre of gravity of a uniform circular or
zero. A couple produces rotational motion in ring-shaped body is at its centre.
a body. V.The centre of gravity of a uniform rectangular
- Moment of a couple also called Torque is the or square shaped body is at the point of
product of one of the forces and the intersection of its diagonals.
perpendicular distance (called arm of a couple) - The centre of gravity of a non-uniform or
from the line of actiong of the force. assymetrical body lies close to the larger side
Moment of a couple or Torque, 𝝉 = 𝑭𝒅. or section of the body.
- Factors that determine the torque are : I.The
magnitude of the force. II. The direction of the ● Equilibrium of non-parallel
force. III. The perpendicular distance of the coplanar forces :
force from the pivot. – Conditions of equilibrium of non-parallel
- Conditions for equilibrium of parallel coplanar forces –
coplanar forces acting on a body – I.They must be coplanar i.e they are not parallel
I.The algebraic sum or resultant of forces acting or not collinear(not in the same line).
on a body in any given direction is zero i.e sum of II.They must be concurrent i.e they all pass
upward forces equals the sum of downward through a common point or their line of action
forces. meet at a common point.
II.The system must obey the principle of III.The sum of the resolved component along
moment – Its states that if a body is in each direction is direction is equal to zero i.e the
equilibrium, then the sum of clockwise moment algebraic sum of components of forces in the

63
Demystified Series Physics Demystified by Dr Timothy
vertical direction is zero and the algebraic sum of ● Equilibrium of rigid bodies and
the components of forces in the horizontal stability of bodies –
direction is zero. 1.Stable equilibrium – A body is said to be in
IV.They can be represented by the three sides of stable equilibrium if it tends to return to its
a trinagle in magnitude and direction. original position when slightly displaced.
V.Any one force can be the equilibrant of the - A wide base and low centre of gravity(G)
other two forces and is equal in magnitude to the ensures stable equilibrium of a body e.g a cone
resultant of the other two forces.The equilibrant resting on its base, a bowl or sphere in the middle
force is equal and opposite resultant force. of a bowl, swinging pendulum bob.
𝑭1 Three non-parallel forces acting on a - Conditions for stable equilibrium:
body 𝑶 to keep it equilibrium. I.It returns to its original position(rest) when
𝑭2 slightly displaced i.e force tends to decrease the
𝑶
displacement when slightly displaced.
𝑭1 + 𝑭2 + 𝑭3 = 𝟎. II.Its potential energy increases after
displacement (initial P.E before displacement is
𝑭3 minimum).
𝐹 1 can be the equilibrant of 𝐹 2 and 𝐹 3 or 𝐹 2 the III.Wide base and low centre of gravity. Centre of
equilibrant of 𝐹 1 and 𝐹 3 or 𝐹 3 the equilibrant of gravity rises and falls inside the base of the body
𝐹 1 and 𝐹 3. after displacement.
- Methods of solving questions on equilibrium IV.It has restoring moment. Moment of weight is
of three non-parallel forces acting on a body ; greater about the base i.e moment of weight is
1. Lami’s theorem – It states that when three greater than moment of action about the edge of
concurrent forces acts on a body to keep it in tilt.
equilibrium, the ratio of each force and the sine 2.Unstable equilibrium – A body is said to be in
of the angle directly opposite it are equal. unstable equilibrium if when slightly displaced it
𝑭𝟏 𝑭 𝑭
Lami’s theorem : = 𝟐 = 𝟑 . tends to move further away from its original
𝒔𝒊𝒏𝜽𝟏 𝒔𝒊𝒏𝜽𝟐 𝒔𝒊𝒏𝜽𝟑
𝑭1 position.
- A narrow base and high centre of gravity(G)
ensures unstable equilibrium of a body e.g a
θ2 θ3 cone or egg resting on its apex or pointed end or
c tip, a ball or sphere resting on an inverted bowl,
θ1
𝑭3 𝑭2 a tight rope walker.
2.Resolution of forces – This involves the - Conditions for unstable equilibrium:
resolving of oblique or inclined forces into their I.It tip over over and does not return to its
perpendicular components.Then, equating the original position after displacement i.e force
opposite forces in either axis i.e sum of the tends to increase displacement when slightly
upward components equals sum of downward displaced.
forces and sum of leftward forces equals sum of II.Its potential energy decreases or lowered after
rightward forces. displacement (Initial P.E before displacement is
I. 𝑻𝒄𝒐𝒔𝜽 = 𝑾 = 𝒎𝒈 maximum).
𝜽 𝑻 II. 𝑻𝒔𝒊𝒏𝜽 = 𝑭. III.Narrow base and high centre of gravity.
𝑭 Centre of gravity falls and falls outside the base
of the body after displacement.
IV.No restoring moment. Moment of the weight is
𝑾 less about the base i.e moment of weight is less
3.Triangle law of forces – It states that if three than the moment of action about the edge of tilt.
forces are in equilibrium, they can be 3.Neutral equilibrium – A body is said to be in
represented by the three sides of triangle taken neutral equilibrium if when slightly displaced, it
in order. tends to come to rest in its new position.
- Each force maintains its original position in the - A flat base and constant centre of gravity (G)
triangle and each angle correspondsto its original ensures neutral equilibrium in a body e.g a
position or alternateswith its new position. cone or cylinder or egg resting on its side, a ball
Using the same diagram used in method 2 – or sphere on a smoth horizontal table.
Pythagora’s theorem – 𝑻2 = 𝑾2+𝑭2 . - Conditions for neutral equilibrium:
θ I.It remains in its new position when slieghtly
𝑾
𝑻 𝑭
𝒔𝒊𝒏 θ = , 𝒄𝒐𝒔 θ =
𝑾 displace or does not change position i.e force
𝑻
𝑭
𝑻 tends to keep the body in the same position when
𝒕𝒂𝒏 θ = . displaced.
𝑾
𝑭 II.Its potential energy is constant or does not

64
Demystified Series Physics Demystified by Dr Timothy
Change (Initial potential energy is zero). 4. In the diagram shown, the
III.Flat base and constant centre of gravity. 30° 10m moment of the 5N force about
Centre of gravity remains at the same position the point 𝑋 is
and does not change. 5N 𝑋
IV.Moment of weight is equal to the moment of A.25Nm B.50Nm C.25√3Nm D.50√3Nm.
action about the edge of tilt Perpendicular distance of the 5𝑁 force from 𝑋 is
V.Produces rolling movement on the body. 10𝑠𝑖𝑛30°, Moment about 𝑋, 𝑀 = 𝐹 × 𝑑,
- In summary – 1.A body is most stable when its 𝑀 = 5×10𝑐𝑜𝑠30° = 25Nm.
cnetre if gravity G is at the lowest point. 5. 𝑋● A force of 5𝑁 acts at a point
2.The higher the centre of gravity of a body the 2m 𝑌 on the rod 𝑋𝑌𝑍 as shown in
less stable it is and the lower the centre of gravity ●𝑌 the diagram. If 𝑋𝑌 is 2m, what
of a body the more stable it is. 5𝑁 is the moment of the force
3.A wide base favours stability as it leads to a low ●𝑍 about point 𝑋?
center of gravity. A.10Nm B.7Nm C.3Nm D.0Nm.
4.If slight displacement of an object raises its The moment of 5𝑁 about 𝑋 is 0𝑁m, as the 5𝑁
centre of gravity, then the object is stable. force is parallel to point 𝑋 or the line of action of
5.If slight displacement of an object lowers its the 5𝑁 force passes through point 𝑋. Moment
centre of gravity then the object is unstable. deals with the product of force and perpendicular
6.If slight displacement of an objecr neither distance not parallel distance.
raises or lowers its centre of gravity then the 6. P In the diagram, the difference
object is neutral equilibrium. 2.5m between the moment about P
1.5m of the 8𝑁 force and its moment
Examples : Q R about Q is
1. An object is acted upon by a system of parallel
forces.The condition(s) for static equilibrium of 8N
the object is/are that the I.algebraic sum of all the A.16Nm B.8Nm C.4Nm D.0Nm.
moments of the forces about a point is zero Perpendicular distance of the 8𝑁 force is 𝑄𝑅,
II.parallel forces must be equal in magnitude and PR2=PQ2+QR2, QR2=PR2−PQ2=2.52−1.52 ,
direction III.resultant of the parallel forces is zero QR2=6.25−2.25=4, QR= √4 = 2m,
A.I B.II C.III D.I and III E.II and III. Moment of the 8𝑁 force about 𝑃 =8×QR ,
For a body acted upon by parallel forces to be in M about P = 8×2 = 16Nm,
static equilibrium: i.It must obey principle of Moment of the 8N force about 𝑄 = 8×QR,
moment i.e The algebraic sum of all moments of M about Q= 8×2 = 16Nm. The difference
the forces about a point is zero or the sum of between the moments is zero (16−16=0).
clockwise moment is equal to the sum of 7. The term torque means A.the moment of a
anticlockwise moment ii.The resultant of the couple about an axis B.the resultant of several
parallel forces is zero or sum of in one forces acting on a body in equilibrium C.two
direction(upward forces) is equal to sum of equal and opposite forces whose line of action do
forces in the other direction(downward forces). not coincide D.two coplanar forces at right angles
- The forces must not be equal in magnitude to each other E.the equilibrant of two concurrent
and direction to produce static equilibrium. forces.
2. A technician applied a force of 250N at the end 8. Which of the following types of motion is
of a spanner of length 0.25m in order to loosen a produced by a couple? A.oscillatory B.rotational
nut.Determine the moment applied to the nut C.random D.translational.
A.1000.0Nm B.625.0Nm C.62.5Nm D.10.0Nm. 9. 10𝑁 In the figure above,
𝐹=250𝑁, 𝑑=length=0.25m, 𝑀=?, 0.25m calculate the magnitude
𝑀 = 𝐹 × 𝑑 = 250×0.25 = 62.5Nm. of the couple
3. 𝑃 Using the diagram below, 10𝑁 A.1.00Nm B.1.25Nm C.2.00Nm
12m calculate the moment of the D.2.50Nm E.5.00Nm.
30° force of 10𝑁 about the point Magnitude of a couple is same as moment of a
𝑃? couple or torque. 𝜏 = 𝐹𝑑 , 𝐹=one of the
forces=10𝑁, 𝑑=arm of the couple or distance
10𝑁 A.60Nm B.60√𝟑Nm C.120Nm between the forces=0.25m, 𝜏 =10×0.25=2.5Nm.
D.120√3Nm E.240Nm. 10. 𝐹 3 𝐹 2 The diagram above shows
Perpendicular distance of the 10𝑁 force from 𝑃 is three forces, 𝐹 1, 𝐹 2 and 𝐹 3
12𝑠𝑖𝑛30°, Moment about 𝑃, 𝑀 = 𝐹 × 𝑑, which keep the bar AB in
𝑀 =10×12𝑐𝑜𝑠30° = 10×12×
√3
=60√3𝑁m. horizontal equilibrium.
2 𝐹1
Which of the following equations is correct?

65
Demystified Series Physics Demystified by Dr Timothy
A.𝐹 3= 𝐹 1+𝐹 2 B.𝐹 2= 𝐹 1+𝐹 3 C.𝐹 1= 𝐹 2−𝐹 3 C.W moment =0.5×30,
D.𝑭1= 𝑭2+𝑭3 AC.W moment=0.2(20+10)+W×10,
For a system acted by a set of force to be in 0.5×30 = 0.2×30+10W, 15=6+10W,
equilibrium, the sum upwards forces equals the 9
10W = 15−6 = 9, W = = 0.9N.
10
sum of downward forces i.e 𝐹 2+𝐹 3= 𝐹 1 or the
15. A uniform meter rod of mass 1.5kg is pivoted
algebraic sum of the force equals zero i.e
at one end. A weight of 7N is placed at the centre
𝐹 2+𝐹 3−𝐹 1=0, 𝐹 2= 𝐹 1−𝐹 3 , 𝐹 3= 𝐹 1−𝐹2.
of the rod. The vertical force which should be
11. 2m 𝑥 2m
applied at the other end to maintain the rod in
equilibrium in equilibrium in the horizontal
position is A.22.0N B.11.0N C.10.5N D.8.5N.
32.5kgf 20kgf 15kgf
W = 𝑚𝑔 = 1.5×10 = 15N, 𝐹
The diagram above a see-saw with three boys of 0cm 50cm 100cm
weights 32.5kgf, 20kgf and 15kgf sitting on it. If
the see-saw is exactly horizontal and of negligible 50cm
weight, the value of 𝑥 is A.0.5m B.1.0m C.1.5m (15N+7N)=22N
D.2.0m. Both the weight of the meter rod(15N) and the
Taking moment about the pivot, 7N weight are acting at the centre of the rod.
Clockwise(C.W) moment=15(2+𝑥)+20𝑥, Taking moment about the pivot –
Anticlockwise(AC.W) moment=32.5(2), C.W moment = AC.W moment,
Sum of C.W moment=Sum of AC.W moment, 22×50 = 𝐹 ×100, 𝐹 =
22×50
= 11N.
15(2+𝑥)+20𝑥 =32.5(2), 30+15𝑥 +20𝑥 =65, 100
35 16. A uniform half meter rod AB, pivoted at a
35𝑥 =65−30=35, 𝑥 = = 1m. point 20cm from A, is balanced horizontally by
35
12. placing a 35g mass at A. Calculate the mass of the
Q 𝑇 =15N rod. A.140g B.170g C.233g D.350g
P R S A half-meter rod will have a length of 0.5m or
1.5m 0.5m 0.5m 50cm.
𝑊 0cm 20cm 25cm 50cm
The value of 𝑊 in the diagram shown above is A B
A.7.5𝑁 B.15.0𝑁C.20.0𝑵 D.30.0𝑁 20cm 5cm
Taking moment about point 𝑃, the pivot – 35g 𝑚
35×20
Clockwise (C.W) moment= 𝑊(1.5), 𝑚 × 5=35×20, 𝑚 = = 140g.
5
Anticlockwise(AC.W) moment= 𝑇(0.5+1.5), 17. A uniform meter rule is pivoted at its centre.
2𝑇
𝑊 ×1.5= 𝑇(0.5+1.5)= 𝑇(2), 𝑊 = , Loads of 15N and 20N are placed at the 10cm and
1.5
𝑇 = 15𝑁, 𝑊 =
2×15
= 20𝑁. 20cm marks respectively and the meter rule is
1.5 balanced horizontally by a load of 40N placed on
13. A uniform meter rule of mass 0.42kg is the other side of the pivot. Calculate the distance
balanced at the 60cm mark when a mass 𝑚 is of the 40𝑁 load from the pivot. A.15.0cm
placed at the 90cm mark. Calculate the value of 𝑚 B.20.0cm C.30.0cm D.80.0cm.
A.0.14kg B.0.42kg C.0.82kg D.0.84kg E.4.20kg 0cm 10cm 20cm 50cm 100cm
The meter rule is uniform, then its mass would be
concentrated at its mid-point i.e 50cm mark. 10cm 30cm 𝑥
Taking moment about the pivot, 15N 20N 40N
0cm 50cm 60cm 90cm 100cm 40× 𝑥 = 15(10+30)+20(30) = 15×40+20×30,
1200
40𝑥 = 600+600=1200, 𝑥 = = 30cm.
10cm 30cm 40
0.42kg 𝑚 18. A uniform bar 15m long, is balanced on a
C.W moment= 𝑚 × 30, AC.W moment=0.42×10 pivot at its mid-point. A boy of mass 55kg sits on
𝑚 ×30=0.42×10, 𝑚 ×30=4.2, 𝑚 =
4.2
= 0.14kg. one arm of the bar 5m away from the pivot. What
30 mass can be placed 2m away from the other end
14. Weights 0.2N and 0.5N are placed at the of the bar to keep the bar horizontal? A.5.0kg
30cm and 80cm marks respectively on a uniform B.6.9kg C.50.0kg D.55.0kg E.70.0kg.
meter rule. If the meter rule balances 19. 𝑃
horizontally on a knife edge at the 60cm mark, 30°
the weight of the meter rule is A.0.1N B.0.3N
C.0.4N D.0.9N. 40𝑁
0cm 30cm 50cm 60cm 90cm 100cm
20cm 10cm 30cm 20cm 30cm
Determine the magnitude of 𝑃 in the diagram
0.2N W 0.5N above. A.16N B.20N C.32N D.40N.
Taking moment about the pivot –

66
Demystified Series Physics Demystified by Dr Timothy
𝑃𝑠𝑖𝑛30° 𝑃 Sum of C.W moment=Sum of AC.W moment
1000(20)=500(20)+200𝑥,
30° 20000=10000+200𝑥, 200𝑥 =20000−10000,
10000
200𝑥 =10000, 𝑥 = = 50cm.
40𝑁 200

20cm 30cm
23. 𝐻 𝑃 𝐾
It is the vertical component of 𝑃 i.e 𝑃sinθ will
produce moment about the pivot or fulcrum.
40×20= 𝑃𝑠𝑖𝑛30°(30+20)= 𝑃(0.5)(50), A uniform beam 𝐻𝐾 of length 10m and weighing
800 200N is supported at both ends as shows.A man
800=25𝑃, 𝑃 = = 32𝑁.
25 weighing 1000N stands at a point 𝑃 on the
20. Masses 𝑚1 and 𝑚2 at the 20cm and 65cm beam.If the reactions at H and K are respectively
marks respectively of a uniform meter rule freely 800N and 400N,then the distance 𝐻𝑃 is A.4m
suspended at its centre of gravity. If the meter B.3⅓m C.3m D.6⅔m E.7m.
rule balances horizontally, determine the ratio 800N 400N
𝑚2:𝑚1 A.1:2 B.3:2 C.2:3 D.2:1. 10m
The metre rule being suspended at its centre of 𝐻 𝑃 5m 𝐾
gravity means it is balanced or pivoted at the
mid-point of the meter rule i.e 50cm.
0cm 20cm 50cm 65cm 100cm 1000N 200N
Taking moments about 𝐻 –
30cm G 15cm The force at 𝐻 will not produce any moment,
𝑚1 𝑚2 when moment is taken about 𝐻, as it don’t have
𝑚2 30 2
𝑚2×15= 𝑚1×30, = = , 𝑚2:𝑚1=2:1. any perpendicular distance.Same applies to the
𝑚1 15 1
21. A mass of 10g is attaced to the 0cm mark of a force of 𝐾,if moment was to be taken about 𝐾.
light meter rule while a mass of 40g is attached to Clockwise moment=200×5+1000𝐻𝑃
the other end,the 100cm mark. The centre of Anticlockwise moment=400×10
gravity of the system is located at the A.20cm Sum of CW moment=Sum of AW moment
mark B.60cm mark C.80cm mark D.mid-point of 1000+1000𝐻𝑃 =4000, 1000𝐻𝑃 =4000−1000 ,
3000
the meter rule. 1000𝐻𝑃 = 3000, 𝐻𝑃 = = 3m.
1000
The centre of gravity of the light meter rule of 24. A uniform beam of length 4m and mass 20kg
negligible weight, is the point were it balances is supported at both ends. A girl of mass 40kg
horizontally on the support or pivot. stands on the beam at a distance of 1.5m from
The centre of gravity is closer to the heavier one of the supports. The reactions at the supports
mass than the lighter mass. are A.150N, 250N B.200N, 200N C.250N, 350N
0cm 100cm
D.300N, 300N.
𝑥 100−𝑥 W=mg=20×10=200N, W=mg=40×10=400N,
10g 40g The reactions at the supports are 𝑃 and 𝑄
40(100−𝑥)=10𝑥, 4000−40𝑥 = 10𝑥, 𝑃 4m 𝑄
4000
4000=10𝑥 + 40𝑥 = 50𝑥, 𝑥 = = 80cm. 2m 2m
50
The center of gravity is 80cm from the 10g mass
and 20cm from the 40g mass or at the 80cm 1.5m
mark. 400N 200N
22. A weight of 1000g hangs from a lever 20cm Taking moment about point 𝑃 –
to the right of the fulcrum.At the left is 500g 200×2+400×1.5= 𝑄 ×4, 400+600=4𝑄,
1000
weight 20cm from the fulcrum.If a 200g weight is 4𝑄 = 1000, 𝑄 = = 250N,
4
Xcm away from the fulcrum,what is the value of Sum of upward force=Sum of downward forces,
X that will be make the lever balance? A.50cm 𝑃 + 𝑄 = 400+200, 𝑄=250N,
B.20cm C.10cm D.30cm E.70cm 𝑃 + 250=600, 𝑃 = 600−250 = 350N.
20cm 20cm 25. Three non-parallel collinear forces,
𝑥 represented both in magnitude and direction by
the three sides of a triangle taken in order, can
500g 200g 1000g A.move a body along a straight line B.rotate a
The 200g additional weight must be placed on body about about its axis C.make a body move
the lighter side inorder to balance the lever randomly D.keep a body in equilibrium
horizontally. D.make a body oscillate.
Taking moment about the pivot – 26. If three coplanar non-paralle forces act on a
Clockwise moment=1000(20) body to keep it in equilibrium, which of the
Anticlockwise moment=500(20)+200(𝑥) following statements is/are correct? I.the three

67
Demystified Series Physics Demystified by Dr Timothy
forces are collinear II.all the three forces must 10 𝑃
= , 𝑃=
10
= 20N.
𝑠𝑖𝑛30° 1 0.5
pass through the centre of gravity of the body
II. Resolution of forces : The oblique or inclined
III.the three forces can be represented by the
force 𝑃 is resolved into its perpendicular
three sides of a triangle A.I,II and III B.II C.II and
components and equate the forces in each axis.
III D.III E.I and II.
10N I. 10N = 𝑃𝑠𝑖𝑛30°,
Three non-parallel forces acting on a body to
II. 𝑃𝑐𝑜𝑠30° = 𝐹 ,
keep is in equilibrium – are concurrent i.e pass
𝑃𝑐𝑜𝑠30° 𝐹
through a common point, are not collinear i.e not
on the same line and they do not pass through the
𝑃𝑠𝑖𝑛30°
centre of gravity of the body. 10 10
27. When three coplanar non-parallel forces are 𝑃= = = 20N.
𝑠𝑖𝑛30° 0.5
in equilibrium I.they can be represented in III. Triangle law of forces : Each force will occupy
magnitude and direction by the three sides of a its original position, with its angle alternate or in
triangle taken in order II.their line of action meet it original position.
at a point III.the magnitude of any one force 𝐹
equals the magnitude of the resultant of the other 30° 𝑃
two IV.any of the forces is the equilibrant of the 10N OR 10N
𝑃 30°
other two. Which of the statements are correct?
A.I and II B.II and III C.II,III and IV D.I,II,III and 𝐹
10 10
IV E.I,II,III and IV. 𝑃 2=102+𝐹 2, 𝑠𝑖𝑛30° = , 𝑃= = 20𝑁.
𝑃 𝑠𝑖𝑛30°
When three non-parallel coplanar forces 𝐹 1, 𝐹 2 30. 10N
and 𝐹 3acts on a body in equilibrium,the resultant
of any two forces is equal in magnitude but 𝐹
opposite in direction to the third
150°
force(equilibrant force). 17N
28. 𝐹1 A particle 𝑃 is in equilibrium The value of 𝐹 in the figure above when in
θ1 under the action of three equilibrium is A.10N B.12N C.20N D.27N.
● θ2 forces 𝐹 1, 𝐹 2 and 𝐹 3 as shown I. Lami’s theorem ;
𝑃 𝐹 2 in the diagram. Which of 120° 10 𝐹 17
10N = = ,
𝐹3 the following equations are sin 150° sin 90° sin 120°
90° sin 150° = sin 30° ,
correct? I.𝐹 1𝑐𝑜𝑠θ1= 𝐹 2𝑐𝑜𝑠θ2 II.𝐹 1𝑠𝑖𝑛θ1= 𝐹 2𝑠𝑖𝑛θ2 150°
III.𝐹 3−𝐹1𝑐𝑜𝑠θ1−𝐹 2𝑐𝑜𝑠θ2=0 IV.𝐹 1+𝐹2+𝐹 3=0 A.I 𝐹 sin 120° = sin 60° ,
and II B.II and III C.II and IV D.II,III and IV. 17N sin 90° = 1 ,
10 10 17 17
Resolve the forces into their vertical and 𝐹= = = 20𝑁 OR 𝐹 = = ,
sin 30° 0.5 sin 60° 0.866
horizontal components – vertical components ; 𝐹 =19.6≈ 20N.
𝐹 1𝑠𝑖𝑛θ1 = 𝐹 2𝑠𝑖𝑛θ2 , horizontal components ; 𝐹 3 = III. Resolution of forces ;
𝐹 1𝑐𝑜𝑠θ1 + 𝐹 2𝑐𝑜𝑠θ2 , 𝐹 3− 𝐹 1𝑐𝑜𝑠θ1−𝐹2𝑐𝑜𝑠θ2=0. 10N 10N
29. 10𝑁
𝐹𝑐𝑜𝑠30° 17N
30°
𝐹
𝐹 150° 𝐹𝑠𝑖𝑛30°
30° 𝑂
17N
𝑃 10𝑁 = 𝐹𝑠𝑖𝑛30°, 𝐹𝑐𝑜𝑠30° = 17N,
The diagram above illustrates three forces acting 𝐹=
10
= 20N. 𝐹 =
17
= 19.6 ≈ 20N.
on an object at point 𝑂. If the object is in sin 30° cos 30°
equilibrium, determine the magnitude of the III. Triangle law of forces ;
force 𝑃. A.10.5N B.11.0N C.17.3N D.20.0N. 𝐹 2 = 172 + 102 ,
I. Lami’s theorem : The three forces acts 10𝑁 17N 𝐹 2=289+100=389,
concurrently on the body to keep it in θ to find 30° 𝐹 = √389 = 19.7N
the angle of of inclination of each force. 𝐹 ≈ 20N.
120°
Angle opposite the 10𝑁 force=180° −30° =150°
10N 31. The body 𝑋 shown in the diagram
has a mass of 6kg and is held in
𝑇 30°
equilibrium by a string and a
90° 90° 𝐹 𝑋 𝐹 horizontal force 𝐹.
30° The tension 𝑇 in the string is (g=10ms-2) A.4√𝟑N
150°
𝑃 B.12N C.40√3N D.120N.
10
=
𝐹
=
𝑃
,
10
=
𝑃
, I. Lami’s theorem ; W = 𝑚𝑔 =6×10=60N.
𝑠𝑖𝑛150° sin (90+30)° 𝑠𝑖𝑛90° 𝑠𝑖𝑛150° 𝑠𝑖𝑛90°
𝑠𝑖𝑛150° = 𝑠𝑖𝑛(180° −150°)= 𝑠𝑖𝑛30°, 𝑠𝑖𝑛90° = 1.

68
Demystified Series Physics Demystified by Dr Timothy
30° 𝑇 30° 34.
90° 𝐹 θ 12N
150° 90°
6√3N
W = 60N
𝑇 𝐹 60 𝑇 60
= = , = ,
sin 90° sin 150° sin(90+30)° sin 90° sin 120° 6N
𝑇=
60
=
60
= 60×
2 120
=
120
= ×
√3
, For what value of θ are the forces in the diagram
sin 60° √3
2
√3 √3 √3 √3 above in equilibrium? A.15° B.30° C.45° D.60°.
120√3 6√3 √3
𝑇= = 40√3N. sin θ = = ,
3 θ 12N
12 2
II. Resolution of forces ; 6N θ= sin −1 √3
= 60°.
𝑇𝑐𝑜𝑠30° 2
6 1
cos θ = = ,
12 2
𝑇𝑠𝑖𝑛30° 𝐹 6√3N θ = cos −1
1
= 60°.
2
6√3
60𝑁 𝑇𝑐𝑜𝑠30° = 60𝑁 , tan θ = = √3 , θ = sin −1
√3 = 60°.
6
𝑇=
60
=
60
= 60 ×
2
=
120
=
120
×
√3
, 35.
cos 30° √3 √3 √3 √3 √3
2 30° 𝑇
120√3
𝑇= = 40√3𝑁. 𝐹
3 60°
III. Triangle of forces :
𝑇2 = 602 + 𝐹 2 ,
W =60N
30° 𝑇 cos 30° = 60 , 𝑇 = 60 , From the diagram above, a weight W=60N is
60𝑁 𝑇 cos 30° suspended orizontally by a string attached to a
𝑇 =40√3N. wall. Find the tension 𝑇 on the string? A.60√3N
𝐹 B.20√3N C.40√𝟑N D.120N.
32. For what values of 𝐹 and θ will the firces 60 60 60
shown in the diagram be in equilibrium? cos 30° = , 𝑇 = = ,
30° 𝑇
𝑇 cos 30° √3/2
60N 𝑇 = 60 × =
2 120
=
120 √3
× ,
θ 40N √3 √3 √3 √3
120√3
𝐹 𝑇= = 40√3N.
3
𝐹 ● The reader should attempt question 33, 34 and
35 using the other two methods.
20N 36. 𝐹 A box 𝑋 of weight 𝑊 is
A.60N and 60° B.60N and 30° C.20√𝟑N and 60° 30° maintained in equilibirum
D.20√3N and30°. the action of two horizontal
Resolution of the forces into their perpendicular 4N 5N forces 5N and 4N
components – and a third force 𝐹 inclined at 30° to the vertical
40𝑐𝑜𝑠θ I. 40𝑐𝑜𝑠θ = 20N, in the diagram above. The value of W is A. N
√3
20 1 2
𝑐𝑜𝑠θ = = = 0.5, √3
40 2 B.√𝟑N C. N D.2√3N.
40𝑠𝑖𝑛θ 𝐹 θ = 𝑐𝑜𝑠 −1 0.5 = 60°. 3
𝐹 𝐹𝑐𝑜𝑠30°
II. 40𝑠𝑖𝑛θ = 𝐹, θ=60°, 30°
20N 𝐹 = 40𝑠𝑖𝑛60°,
√3
𝐹 = 40× = 20√3N. 4N + 𝐹𝑠𝑖𝑛30° 5N
2
33. If the system of forces
shown in the diagram W
30° 𝑇1 is in equilibrium, the Resolving the forces into the horizontal
𝑇2
𝑂 tension 𝑇2 is equal to component – 𝐹𝑠𝑖𝑛30° + 4 = 5, 𝐹𝑠𝑖𝑛30° = 5−4,
(g=10ms-2) 𝐹𝑠𝑖𝑛30° = 1, 𝐹 =
1
= 2N,
sin 30°
5kg Resolving the forces into the vertical component
A.100/√3N B.50/√𝟑N C.150/√3N D.50N . – 𝐹𝑐𝑜𝑠30° = 𝑊, 𝐹 = 2N,
Triangle law of forces – 𝑊 = 5×10 = 50N √3
𝑇2 2 = 502 + 𝑇1 2 , 𝑊 = 2𝑐𝑜𝑠30° = 2× = √3N.
2
30° 𝑇1 𝑡𝑎𝑛30° = 𝑇2 , 37. 𝑃 40𝑁 A particle is under the action of
50𝑁 50 three forces in equilibrium. Two
𝑇2 = 50𝑡𝑎𝑛30° = 50×1/√3 , of te forces are as shown. The
𝑇2 𝑇2 = 50/√3N. third force is
𝑄 40𝑁

69
Demystified Series Physics Demystified by Dr Timothy
A.80𝑁 along 𝑃 B.160N at 60° to 𝑃 C.40√𝟐N at 45° potential energy before displacement is
to either 𝑷 or 𝑸 D.0N E.40√2N at 30° to 𝑄. minimum.
The third force required for the body to be in The potential energy of body in neutral
equilibrium is the equilibrant force. The equilibrium is contant and does not change after
equilibrant force 𝐸 is equal in magnitude but displacement, but its initial potential energy
opposite in direction to the resultant force 𝑅. before displacement is zero.
𝑃 41. Which of the following is not in stable
θ θ equilibrium? A.a heavy weight suspended on a
40𝑁 𝑅 𝐸 40𝑁 string B.a cone resting on its slant edge C.a
θ heavy nased lamp table D.a beam of balance in
𝑄 use
40𝑁 40𝑁 A cone resting on its slant edge is neutral
𝐸 2 = 402+402 = 1600+1600 = 3200 , equilibrium as it produces rolling movement. All
40 other options are example of stable equilibrium.
𝐸 = √3200 = 40√2N, 𝑡𝑎𝑛θ = = 1 ,
40 42. A cone is in neutral equilibrium has its
θ = 𝑡𝑎𝑛−1 1=45°. The third force is 40√2N at 45° potential energy A.decreasedC.increased
to either 𝑃 or 𝑄 or 40√2N at S45°W , opposite to C.unchanged D.oscillating.
the direction of the resultant. 43. I. II. III. IV.
38. From the diagram below, if the forces are in .G . G
equilibrium, what is the value of thw tension 𝑇? .G G .
Which of the following bodies, each with centre
of gravity G, lying on a horizontal table, is/are
𝑇 𝑇 unstable equilibrium? A.II B.IV C.I and IV D.II and
45° 45° III E.III and IV.
IV represents a body in unstable equilibrium as it
has a high centre of gravity G. III represents a
100N body in stable equilibrium as it has low centre of
A.50.00N B.70.71N C.100.00N D.241.42N. gravity. I and II represents bodies in neutral
Resolving the forces vertically i.e as they are in equilibrium as they will produce rolling
equilibrium – 𝑇𝑠𝑖𝑛45° + 𝑇𝑠𝑖𝑛45° = W, movements continuously on the horizontal table.
100
𝑊 = 100𝑁 , 2𝑇𝑠𝑖𝑛45° = 100 , 𝑇 = , 44. I IV VII
2𝑠𝑖𝑛45°
100 II V VIII
𝑇= = 70.71N.
2×0.7071
- The tension 𝑻 increases as θ decreases. III VI IX
Tension 𝑻 is maximum when θ is 0° and
minimum when θ is 90°. The diagram above represents ball in an
39. Which of the following actions will increase undulating surface. Which of the following
the stability of a body I.increasing the base area options represents positions of stable
II.raising the centre of gravity of the body equilibrium? A.III,IV and VIII B.III,VI and IX
III.lowering the center of gravity of the body A.I C.I,IV and VII D.II,V and VIII
and II B.I and III C.II and III D.I,II and III. Balls at I,II,IV,V,VI and VII will not return to its
Wide base area and low centre of gravity original position after displacement i.e unstable
increases the stability of a body. equilibrium. Balls at III,VI and IX will return to its
40. A body was slightly displaced from its original position after displacement i.e stable
equilibrium position. Which one of the following equilibrium.
is a condition for its stable equilibrium A.an 45.
increase in the kinetic energy of the body B.an .G
increase in the potential energy of the body Q
C.a decrease in the potential energy of the body P
D.an increase in the weight of the body. The diagram above shows a solid figure with a
The potential energy of a body increases after base 𝑃𝑄 and centre of gravity G on an inclined
displacement in stable equilibrium due to plane. Which of the following statements is
increase in the height of the body, but its initial correct? A.The solid will fall over if the
potential energy of the body before displacement vertical line through G lies outside the base
is minimum. B.The solid will fall over of the vertical line
The potential energy of a body decreases after through G lies inside the base C.The solid will not
displacement in unstable equilibrium due to fall over if the vertical line through G lies outside
decrease in the height of the body, but its initial the base D.The solid can never fall over

70
Demystified Series Physics Demystified by Dr Timothy
If the the vertical line through the centre of
gravity G lies outside the base, the body will fall
over the inclined plane i.e unstable equilibrium.
If the vertical line through the centre of gravity G
lies inside the base, the body will not fall over the
inclined plane i.e stable equilibrium.
46. When a body is in stable equilibrium and is
slightly tilted A.the vertical through the centre
of gravity falls outside the base B.the vertical
through the centre of gravity is not disturbed
C.the moment of the weight is greater than the
moment of the action about the edge of tilt
D.the moment of the weight is less than the
moment of the action about the edge of tilt E.the
moment of the weight equals the moment of the
action about the edge of tilt.
A body is said to be in stable equilibrium when
the vertical through the centre of gravity rises
and falls inside the base of the body after
displacement and the moment of weight is
greater about the base i.e moment of weight is
greater than moment of action about the edge of
tilt.
47. When a body is slightly tilted, it is found that
its centre of gravity is slightly raised. What is the
state of equilibrium of the body? A.unstable
B.stable C.neutral D.cannot be determined.
48. The centre of gravity of a uniform triangular
lamina is the point of intersection of A.the
perpendicular bisectors of its sides B.the
bisectors of its three sides C.the medians D.the
altitudes E.all of the above.
49. The diagram shows a uniform
𝑁 circular object of centre 𝑀
𝑀 and diameter 𝐾𝑁 from which
𝐿 a circular section of 𝐿 and
diameter 𝐾𝑀 has been
𝐾 removed 𝐾𝐿 is a straight line.
The centre of gravity if the resulting body is
located A.at 𝐿 B.at 𝑀 C.between 𝐾 and 𝑀
D.between 𝑴 and 𝑵.
The centre of gravity before the removal of the
section 𝐿 of the object is at the centre 𝑀. On
removal of the section, the centre of gravity shift
to the heavier side of the body, between 𝑀 and 𝑁.
- When a cross-section of a circular body is cut
the centre of gravity shifts to the heavier side
(section not cut) of the body.

Jamb past questions on equilibrium of forces:


[1989/4,1994/10,1986/6,1999/5,2009/6,2011
/8,2010/7,1981/41,1982/46,1983/3,1990/3,
1991/6,1993/11,1995/4,1997/9,2000/10,2005
/7,1982/29,2008/8,2015/30,2016/34,1978/13
,1979/39,1984/2,1986/9,1985/10,11,1993/12,
1994/8,1995/10,1997/11,1998/8,2003/11,200
7/47,1990/7,8,1980/31,1992/7,2014/10]

71
Demystified Series Physics Demystified by Dr Timothy

Chapter 7 – Elastic Properties, Elasticity


● Elasticity – Elasticity is the ability of a material It is independent of the dimesion of the wire,
to return to its original shape and size after being stress and strain in the wire.
distorted by an external force. - Force constant 𝑘, young molulus 𝐸, area 𝐴 and
- Brittle materials – These are materials that 𝑬𝑨
length 𝐿 of an elastic material is given as : 𝒌 = .
break easily i.e before getting to the elastic limit 𝑳
- Force per unit length=force constant×strain,
e.g glass. 𝑭
- Ductile materials – These are materials that = 𝒌 × 𝑺.
𝑳
can be extended beyond the elastic limit before - Energy density of an elastic material or
breaking and can be drawn into thin wires before stretched wire is the ratio of its energy stored in
breaking e.g copper. the material to it’s volume.
- Hooke’s law states that the extension of any Energy density= (
𝒆𝒏𝒆𝒓𝒈𝒚 𝟏
) = (stress×strain)
𝒗𝒐𝒍𝒖𝒎𝒆 𝟐
elastic material is directly proportional to the
- Bulk modulus is the ratio of hydrostatic
applied force 𝐹/lod or weight, provided the
stress(pressure) to volumetric strain in liquids.
elastic limit is not exceeded. 𝑭 ∝ 𝒆, 𝑭 = 𝒌𝒆 ,
- Shear modulus or Rigidity modulus is the
𝑘=force or elastic constant or stiffness constant
𝑭 𝑭𝟏 𝑭 ratio of shear/tangetial stress to shear strain.
of the elastic material. 𝒌 = , = 𝟐. - Force/load-extension graph or stress-strain
𝒆 𝒆𝟏 𝒆𝟐
𝒆=extension=Final length−Original length, graph of an elastic material – A plot of
𝒆𝟏 = 𝑳𝟏 − 𝑳𝒐 , 𝒆𝟐 = 𝑳𝟐 − 𝑳𝒐 , load(force) against extension is a straight line
𝑭𝟏 𝑭
= 𝟐 , 𝐿1 the length produced by 𝐹1 , 𝐿2 is graph passing from the origin provided the
𝑳𝟏 −𝑳𝒐 𝑳𝟐 −𝑳𝒐 elastic limit is not exceeded. On further increase
the length produced by 𝐹2 , 𝐿𝑜 is the original of the load, a point is reached where the spring
length or natural length or unstretched length. no longer returns to its original length and the
- Hooke’s law can also be represented by – spring snaps.
𝑭𝟐 −𝑭𝟏 𝑳 −𝑳
= 𝟐 𝟏 , 𝑭𝒐 = 𝟎 , as no force is acting on Load or stress
𝑭𝟏 −𝑭𝒐 𝑳𝟏 −𝑳𝒐
an elastic material at rest, when 𝑳 = 𝑳𝒐 . 𝑩
- If three forces are acting on the same elastic 𝑬 𝑴
material causing a corresponding increase in the 𝒀
length of the material, they can be represented by
𝑭 −𝑭 𝑳 −𝑳 0 Extension or strain
– 𝟑 𝟐= 𝟑 𝟐.
𝑭𝟐 −𝑭𝟏 𝑳𝟐 −𝑳𝟏 Extension or strain
- Extension is same as change in length or 𝑩
increase in length by or extended by. 𝒀 𝑴
- Increased in length to or extended to is same 𝑬
as final length.
- Mass can be used in place of force in the 0
formulas above. Load or stress
● Modulus of elasticity of elastic materials – Elastic limit (𝑬) – It is the limit of force or the
- Tensile stress 𝑺T is the ratio of the force acting load beyond which the material does not return
on a unit cross-sectional area of an elastic to its original length when the load is removed.
material. Hooke's law is obeyed up to the elastic limit
𝑭𝒐𝒓𝒄𝒆 𝑭
Tensile stress , 𝑺𝑻 = = , 𝑨 = 𝝅𝒓𝟐 . i.e beyond the elastic limit the force or load is
𝑨𝒓𝒆𝒂 𝑨
- Tensile strain 𝑺is the ratio of the extension to no longer proportional to the extension.
the original length. It does not have a unit. Yield point (𝒀) – It is the point beyodnthe elastic
𝒆𝒙𝒕𝒆𝒏𝒔𝒊𝒐𝒏 𝒆 limit in which the elastic material has yielded all
Tensile strain 𝑺 = = .
𝒐𝒓𝒊𝒈𝒊𝒏𝒂𝒍 𝒍𝒆𝒏𝒈𝒕𝒉 𝑳 its elasticity permanently and has become
- Young modulus of elasticity 𝑬is the ratio of plastic. At yield point, a slight increase in load
tensile stress to tensile strain within the elastic produces a large extension.
region. Maximum load (𝑴) – It is the maximum load
𝑻𝒆𝒏𝒔𝒊𝒍𝒆 𝒔𝒕𝒓𝒆𝒔𝒔 𝑺
Yound modulus, 𝑬 = = 𝑻, which the elastic material is able to support.
𝑻𝒆𝒏𝒔𝒊𝒍𝒆 𝒔𝒕𝒓𝒂𝒊𝒏 𝑺
𝑭⁄𝑨 𝑭𝑳 Breaking or fracture point (𝑩) – It is the force
𝑬= = . or load at which the elastic material breaks into
𝒆⁄𝑳 𝑨𝒆
- Tensile stress and Young modulus has the same two (fractures).
unit and dimension as Pressure i.e Nm-2 or 𝑶𝑬 – Elastic region, 𝑬𝑩 – Plastic region.
Pascal(Pa). Tensile Strain has no unit. - The slope of the linear portion of a force-
- Young modulus depends only on the type of extension graph represents the force or
material from which an elastic wire is made. stiffness constant.

72
Demystified Series Physics Demystified by Dr Timothy
- Tensile stress ∝ Tensile strain. The slope of 𝑘=20Nm-1, first equilibrium length=original
the graph of a tensile stress vs tensile strain is length=𝐿𝑜 =0.25, 𝑚=100g=0.1kg, 𝐹=𝑚𝑔=0.1×10,
Young modulus. 𝐹=1N, The new equilibrium length= 𝐿1 =?,
● Workdone on an elastic material –The 𝐹 = 𝑘𝑒 = 𝑘(𝐿1 − 𝐿𝑜 ) , 1=20(𝐿1 −0.25),
workdone on an elastic material 𝑊 is given by; 1
𝐿1 − 0.25= = 0.05, 𝐿1 = 0.05+0.25 = 0.3m,
20
Workdone = Average force×extension,
𝐿1 = 30cm.
𝟏 𝟏 𝑭𝟐
𝑾 = 𝑭𝒆 = 𝒌𝒆𝟐 = . This is the same as the 6. A spiral spring extends by 5cm under a load of
𝟐 𝟐 𝟐𝒌
elastic potetial energy stored in it. 60N. When the load is replaced by a steel block,
- In a force-extension graph the area under the new extension is 7cm.The weight of the steel
the graph gives the workdone on an elastic block is A.12N B.43NC.84N D.96N.
material or the elastic potential energy 𝑒1 =5cm, 𝐹1 =60N, 𝑒2 =7cm, 𝐹2 =?,
𝐹 𝐹 𝐹 60 𝐹 60×7
stored in it. 𝑘= , 1= 2 , = 2 , 𝐹2 = = 84N.
𝑒 𝑒1 𝑒2 5 7 5
7. The extension in a spring when 5g wt hung
Examples : from it was 0.56cm. If Hooke’s law is obeyed,
1. Hooke’s law states that A.when a wire is what is the extension caused by a load of 20g wt?
loaded beyond the elastic limit the extension A.1.12cm B.2.14cm C.2.52cm D.2.80cm E.2.24cm
produced is inversely proportional to the tension 𝐹1 =5g wt, 𝑒1 =0.56cm, 𝐹2 =20g wt, 𝑒2 =?,
B.the extension produced in a wire is equal to the 𝐹 𝐹 𝐹 5 20 20×0.56
𝑘= , 1= 2 , = , 𝑒2 = = 2.24cm.
load applied C.the extension produced in a 𝑒 𝑒1 𝑒2 0.56 𝑒2 5
wire is proportional to the tension applied if 8. A 10g mass place on the pan of spring balance
the elastic limit is not exceeded D.the causes an extension of 5cm. If a 15g mass is
enxtension produced in a wire is inverlsey placed on the scale pan of the same spring
proportional to the tension applied E.when a balance the extension is A.3.3cm B.6.5cm
wire is loaded and unloaded it returns to its C.7.5cm D.10.8cm E.15.0cm
original length. 𝐹1 =𝑚1 =10g, 𝑒1 =5cm, 𝐹2 =𝑚2 =15g, 𝑒2 =?,
𝐹 𝐹 𝐹 10 15 15×5
- Hooke’s law can also be stated as stress is 𝑘= , 1= 2 , = , 𝑒2 = = 7.5cm.
𝑒 𝑒1 𝑒2 5 𝑒2 10
directly proportional to strain provided the
9. A load of 50𝑁 attached to the free end of a
elastic limit is not exceeded.
spiral spring stretches it from 0.30m to 0.36m.
2. Which of the following is Hooke’s law?
Calculate the length of the spring when a load of
A.Action and reaction are equal and opposite
70𝑁 is suspended from the end A.0.384cm
B.When a body is totally or partially immersed in
B.0.830cm C.0.960cm D.1.200cm.
a fluid it experiences an uothrust which is equal
𝐹1 =50𝑁,𝐿𝑜 =0.30cm,𝐿1 =0.36cm,𝐹2 =70𝑁,𝐿2 =?,
to the weight of fluid displaced C.For a fixed mass 𝐹 𝐹 𝐹 𝐹1 𝐹
of gas at constant temperature, the pressure is 𝑘= , 1= 2, = 2 ,
𝑒 𝑒1 𝑒2 𝐿1 −𝐿𝑜 𝐿2 −𝐿𝑜
inversely proportional to its volume D.The 50
=
70
,
50
=
70
,
extension of a material is proportional to the 0.36−0.3 𝐿2 −0.3
70×0.06
0.06 𝐿2 −0.3

force applied to it if the elastic limit is not 𝐿2 −0.3 = = 0.084 , 𝐿2 = 0.084+0.3,


50
exceeded. 𝐿2 = 0.384cm.
3. If a load of 1kg stretches a cord by 1.2cm, what 10. A force of 15N stretches a spring to a total
is the force constant of the cord? (g=10ms-2) length of 30cm. An additional force of
A.866Nm-1 B.833Nm-1 C.769Nm-1 D.667Nm-1. 10𝑁stretches the spring 5cm further. Find the
Load=force=𝐹=𝑚𝑔=1×10=10N, 𝑚=1kg, natural length of the spring A.25.0cm B.22.5cm
𝑒=1.2cm=1.2×10-2m, 𝐹 = 𝑘𝑒, C.20.0cm D.15.0cm.
𝐹 10
𝑘= = = 833Nm-1. 𝐹1 =15N, 𝐿1 =30cm, 𝐹2 =15+10=25N,
𝑒 1.2×10−2
4. A spiral spring extends from a length of 𝐿2 =30+5=35cm, 𝐿𝑜 =natural length=?,
𝐹1 𝐹 15 25 35−𝐿𝑜 25
10.00cm to 10.01cm, when a force of 20N is = 2 , = , = ,
𝐿1 −𝐿𝑜 𝐿2 −𝐿𝑜 30−𝐿𝑜 35−𝐿𝑜 30−𝐿𝑜 15
applied on it. Calculate the force constant of the 35−𝐿𝑜
= ,
5
3(35 − 𝐿𝑜 )=5(30 − 𝐿𝑜 ),
spring. A.2000Nm-1 B.1000Nm-1 C.500Nm-1 30−𝐿𝑜 3
D.250Nm-1. 105−3𝐿𝑜 = 150−5𝐿𝑜 , 5𝐿𝑜 −3𝐿𝑜 =150−105,
45
𝐹=20N, 𝐿1 =10.01cm, 𝐿𝑜 =10.00cm, 𝑘=?, 2𝐿𝑜 = 45, 𝐿𝑜 = = 22.5cm. Alternatively,
𝐹 𝐹 20 20 2
𝑘= = = = = 2000Nm-1. 𝐹𝑜 =0, 𝐹1 =15N, 𝐹2 =15+10=25N, 𝐿𝑜 =?, 𝐿1 =30cm,
𝑒 𝐿1 −𝐿𝑜 10.01−10 0.01
𝐹 −𝐹 𝐿 −𝐿 25−15 35−30
5. A spring with a force constant of 20Nm-1 𝐿2 =30+5=35cm, 2 1 = 2 1 , = ,
𝐹1 −𝐹𝑜 𝐿1 −𝐿𝑜 15−0 30−𝐿𝑜
hanging vertically has an equilibrium length of 10 5 2 5
= , = , 2(30 − 𝐿𝑜 )=5×3=15,
0.25m. A 100g mass is attatched to the spring and 15 30−𝐿𝑜 3 30−𝐿𝑜
45
carefully allowed to drop so that it comes to rest. 60−2𝐿𝑜 = 15, 2𝐿𝑜 = 60−15=45, 𝐿𝑜 = ,
2
The new equilibrium length of the spring is 𝐿𝑜 = 22.5cm.
A.30cm B.40cm C.50cm D.75cm (g=10ms-2). 11. A force of 100N stretches an elastic string to

73
Demystified Series Physics Demystified by Dr Timothy
a total length of 200cm. If an additional force of Load added Scale reading
100N stretches the string 5cm further, find the 1g 33.7cm
natural length of the string A.15cm B.12cm 3g 34.9cm
C.10cm D.8cm E.5cm. 7g ?
𝐹1 =100N, 𝐿1 =20cm, 𝐹2 =100+100=200N, A.35.5cm B.36.1cm C.37.3cm D.38.5cm E.39.1cm
𝐿2 =20+5=25cm, 𝐿𝑜 =natural length=?, First calculate the original length of the spring
𝐹1 𝐹
= 2 ,
100
=
200
,
25−𝐿𝑜
=
200 2
= , before calculating the length of the spring when
𝐿1 −𝐿𝑜 𝐿2 −𝐿𝑜 20−𝐿𝑜 25−𝐿𝑜 20−𝐿𝑜 100 1
the 3𝑁 is hung from it. 𝐹1 =1g, 𝐿1 =33.7cm, 𝐹2 =3g,
25−𝐿𝑜 = 2(20−𝐿𝑜 ), 25−𝐿𝑜 =40−2𝐿𝑜 ,
𝐿2 =34.9cm, 𝐿𝑜 =natural length=?,
2𝐿𝑜 − 𝐿𝑜 =40−25, 𝐿𝑜 = 15cm. Alternatively, 𝐹1 𝐹 1 3
𝐹𝑜 =0, 𝐹1 =100N, 𝐹2 =100+100=200N, 𝐿𝑜 =?, = 2 , = ,
𝐿1 −𝐿𝑜 𝐿2 −𝐿𝑜 33.7−𝐿𝑜 34.9−𝐿𝑜
𝐿1 =20cm, 𝐿2 =20+5=25cm, 34.9−𝐿𝑜 = 3(33.7−𝐿𝑜 )=101.1−3𝐿𝑜 ,
𝐹2 −𝐹1 𝐿 −𝐿
= 2 1 ,
200−100
=
25−20
,
100
=
5
, 3𝐿𝑜 − 𝐿𝑜 = 101.1−34.9 , 2𝐿𝑜 =66.2cm,
𝐹1 −𝐹𝑜 𝐿1 −𝐿𝑜 100−0 20−𝐿𝑜 100 20−𝐿𝑜 66.2
5 𝐿𝑜 = = 33.1cm.
1= , 20−𝐿𝑜 = 5, 𝐿𝑜 = 20−5=15cm. 2
𝐹1 𝐹2 𝐹3
20−𝐿𝑜
𝐹3 =7g , 𝐿3 =?, = = ,
12. The total length of a spring when a mass of 𝐿1 −𝐿𝑜 𝐿2 −𝐿𝑜 𝐿3 −𝐿𝑜
𝐹1 𝐹3 1 7 1 7
20g is hung from its end is 14cm, while its total = , = , = ,
𝐿1 −𝐿𝑜 𝐿3 −𝐿𝑜 33.7−33.1 𝐿3 −33.1 0.6 𝐿3 −33.1
length is 16cm when a mass of 30g is hung from 𝐿3 −33.1=7×0.6=4.2, 𝐿3 =4.2+33.1=37.3cm.
the same end. Calculate the unstretched length of Alternatively : 𝐹1 =1g, 𝐿1 =33.7cm, 𝐹2 =3g,
the spring assuming Hooke’s law is obeyed 𝐹 −𝐹 𝐿 −𝐿
A.9.33cm B.10.00cm C.10.66cm D.12.00cm 𝐿2 =34.9cm, 𝐹3 =7g, 𝐿3 =?, 3 2 = 3 2,
𝐹2 −𝐹1 𝐿2 −𝐿1
7−3 𝐿 −34.9 4 𝐿 −34.9 𝐿 −34.9
E.15.00cm. = 3 , = 3 , 2= 3 ,
3−1 34.9−33.7 2 1.2 1.2
𝐹1 =20g, 𝐿1 =14cm, 𝐹2 =30g, 𝐿2 =16cm, 𝐿3 −34.9=2×1.2=2.4, 𝐿3 =2.4+34.9=37.3cm.
𝐹1 𝐹
𝐿𝑜 =unstretched length=?, = 2 , 15. A string of length 4m is extended by 0.02m
𝐿1 −𝐿𝑜 𝐿2 −𝐿𝑜
20
=
30
,
20
=
14−𝐿𝑜
,
2
=
14−𝐿𝑜
, when a load of 0.4kg is suspended at its end.
14−𝐿𝑜 16−𝐿𝑜 30 16−𝐿𝑜 3 16−𝐿𝑜 What will be the new length of the string when
2(16−𝐿𝑜 )=3(14 − 𝐿𝑜 ) , 32−2𝐿𝑜 = 42−3𝐿𝑜 , the applied force is 15𝑁? (g=10ms-2) A.5.05m
3𝐿𝑜 −2𝐿𝑜 = 42−32 , 𝐿𝑜 = 10cm. B.6.08m C.4.05m D.4.08m.
Alternavtively : 𝐹𝑜 =0, 𝐹1 =20g, 𝐹2 =20g, 𝐿𝑜 =?, 𝐿𝑜 =original length=4m, 𝑒1 =0.02m, 𝑚=0.4kg,
𝐹 −𝐹 𝐿 −𝐿
𝐿1 =14cm, 𝐿2 =16cm, 2 1 = 2 1 , 𝐹1 =load=𝑚𝑔=0.4×10=4𝑁, 𝐹2 =15𝑁, 𝐿2 =new
𝐹1 −𝐹𝑜 𝐿1 −𝐿𝑜
𝐹 𝐹 𝐹 𝐹 𝐹
30−20
=
16−14
,
10
=
2
,
1
=
2
, length of the string, 𝑘 = , 1 = 2 , 1 = 2 ,
20−0 14−𝐿𝑜 20 14−𝐿𝑜 2 14−𝐿𝑜 𝑒 𝑒1 𝑒2 𝑒1 𝐿2 −𝐿𝑜
4 15 15×0.02
14−𝐿𝑜 = 2×2 = 4, 𝐿𝑜 = 14−4 = 10cm. = , 𝐿2 −4= = 0.075,
0.02 𝐿2 −4 4
13. The spiral spring of a spring alance is 25.0cm 𝐿2 = 0.075+4 = 4.075m = 4.08m.
long when 5N hangs on it and 30.0cm long, when 16. A spiral spring of natural length 20.00cm has
the weight is 10N. What is the length of the spring a scale pan hanging freely in its lower end. When
if the weight is 3N, assuming Hooke’s law is an object of mass 40g is placed in the pan, its
obeyed? A.15.0cm B.17.0cm C.20.0cm D.23.0cm. length vecomes 21.80cm. When the object is
First calculate the original length of the spring replaced with another of mass 60g, the length
before calculating the length of the spring when becomes 22.05cm. Calculate the mass of the scale
the 3𝑁 is hung from it. 𝐹1 =5N, 𝐿1 =25cm, 𝐹2 =10N, pan (g=10ms-2) A.104g B.208g C.312g D.416g.
𝐿2 =30cm, 𝐿𝑜 =natural length=?, The extension 𝑒 in a spring balance is produced
𝐹1 𝐹 5 10 5 25−𝐿𝑜
= 2 , = , = , by a force or load which is equal the mass of the
𝐿1 −𝐿𝑜 𝐿2 −𝐿𝑜 25−𝐿𝑜 30−𝐿𝑜 10 30−𝐿𝑜
1 25−𝐿𝑜 scale pan 𝑚𝑠 and the mass of the load 𝑚 placed in
= , 30−𝐿𝑜 = 2(25−𝐿𝑜 )=50−2𝐿𝑜 ,
2 30−𝐿𝑜 the scale pan.
2𝐿𝑜 − 𝐿𝑜 = 50−30, 𝐿𝑜 =20cm, 𝐿𝑜 =20.00cm, 𝐹1 =𝑚𝑠 + 𝑚1 =(𝑚𝑠 +40)g ,
𝐹1 𝐹 𝐹
𝐹3 =3N, 𝐿3 =?, = 2 = 3 , 𝐿1 =21.80cm, 𝐹2 =𝑚𝑠 + 𝑚2 =(𝑚𝑠 +60)g,
𝐿1 −𝐿𝑜 𝐿2 −𝐿𝑜 𝐿3 −𝐿𝑜 𝐹 𝐹 𝐹 𝐹1 𝐹
𝐹1 𝐹3 5 3 5 3 𝐿2 =22.05cm, 𝑘 = , 1 = 2 , = 2 ,
= , = , = , 𝑒 𝑒1 𝑒2 𝐿1 −𝐿𝑜 𝐿2 −𝐿𝑜
𝐿1 −𝐿𝑜 𝐿3 −𝐿𝑜 25−20 𝐿3 −20 5 𝐿3 −20 𝑚𝑠 +40 𝑚 +60 𝑚 +40 𝑚 +60
= 𝑠
, 𝑠 = 𝑠 ,
3 21.80−20.00 22.05−20.00 1.8 2.05
1= , 𝐿3 −20=3, 𝐿3 =3+20=23cm. 2.05(𝑚𝑠 +40)=1.8(𝑚𝑠 +60),
𝐿3 −20
Alternatively : 𝐹1 =5N, 𝐿1 =25cm, 𝐹2 =10N, 2.05𝑚𝑠 + 82 = 1.8𝑚𝑠 + 108,
𝐿2 =30cm, 𝐹3 =3N, 𝐿3 =?, 2.05𝑚𝑠 −1.8𝑚𝑠 = 108−82, 0.25𝑚𝑠 = 26,
26
𝐹3 −𝐹2 𝐿 −𝐿
= 3 2,
3−10 𝐿 −30
= 3 ,
−7 𝐿 −30
= 3 , 𝑚𝑠 = = 104g.
0.25
𝐹2 −𝐹1 𝐿2 −𝐿1 10−5 30−25 5 5
−7×5 17. A load of mass 45g produced an extension of
𝐿3 −30= = −7, 𝐿3 = 30−7 = 23cm. 6.5cm when placed on a scale pan attached to a
5
14. Complete the table below for a body which spring. If a 60g mass produced an extension of
obey’s Hooke’s law. 8cm on same spring, determine the mass of the

74
Demystified Series Physics Demystified by Dr Timothy
scale pan A.35g B.20g C.15g D.10g . known as A.modulus of rigidity B.modulus of
𝐹1 =𝑚𝑠 + 𝑚1 =(𝑚𝑠 +45)g, 𝑒1 =6.5cm, 𝐹2 =𝑚𝑠 + 𝑚2 elasticity C.bulk modulus D.Young’s modulus
= (𝑚𝑠 +60)g, 𝑒2 =8cm, E.shear modulus.
𝐹 𝐹
𝑘= , 1= 2, 𝑠
𝐹 𝑚 +45 𝑚 +60
= 𝑠 , Modulus of elasticity can be Young’s modulus,
𝑒 𝑒1 𝑒2 6.5 8
shear modulus or bulk modulus.
8(𝑚𝑠 +45)=6.5(𝑚𝑠 +60),
23. The general definition of elastic modulus is
8𝑚𝑠 + 360 = 6.5𝑚𝑠 + 390,
𝒔𝒕𝒓𝒆𝒔𝒔 𝑠𝑡𝑟𝑎𝑖𝑛 𝑠𝑡𝑟𝑒𝑠𝑠
8𝑚𝑠 −6.5𝑚𝑠 = 390−360, 1.5𝑚𝑠 = 30, A. B. C.𝑠𝑡𝑟𝑒𝑠𝑠 × 𝑠𝑡𝑟𝑎𝑖𝑛 D.√ .
30 𝒔𝒕𝒓𝒂𝒊𝒏 𝑠𝑡𝑟𝑒𝑠𝑠 𝑠𝑡𝑟𝑎𝑖𝑛
𝑚𝑠 = = 20g. Modulus of elasticity or elastic modulus generally
1.5
18. is the ratio of stress to strain.
1 1 24. In an experiment to determine Young’s
2 2 modulus for a wire, several loads are attached to
3 3 the wire and the corresponding extensions
4 4 measured. The tensile stress in each case
5 5 depends on A.load and the radius of the wire
𝑅 6 6 B.radius of the wire and extension C.load and the
𝑊1 7 𝑅 7 extension D.extension and the original length of
8 𝑊2 8 the wire.
9 9 Tensile stress is the ratio of the tensile force or
10 10 load to the area of the wire.Hence, tensile stress
In the figure above 𝑊 1=200g and 𝑊 2=450g. depends on the load or force and the cross-
Calculate the extension of the spring per unit load sectional area of the wire,which is given by its
A.6.0×10-5mN-1 B.2.5×10-4mN-1 C.6.0×10-3mN-1 radius. Tensile strain depends on the extension
D.2.5×10-2mN-1 (g=10ms-2). and ethe original length of the wire.
W1=450g, W1=200g, 𝐿1=5cm, 𝐿2=6.5cm, 25. A wire of diameter 2.8mm supports a weight
Increase in load= 𝑊 2−𝑊 1=450−200=250g, 𝑊. If the tensile stress in the wire is 2×107𝑁m-2,
𝐹 =load=250g=0.25kg=2.5𝑁, the value of W is (𝜋=22/7) A.123.2N B.98.4N
extension=𝐿2−𝐿1=6.5−5=1.5cm=1.5×10-2m C.56.0N D.49.2N.
2.5
Extension per unit load= =6.0×10-3mN-1. diameter, 𝑑=2.8mm=2.8×10-3m, Tensile stress,
1.5×10−2
19. A spring has a total length of 17.5cm under a 𝑆𝑇 =2×107𝑁m-2, Force=Weight=W,
𝐹𝑜𝑟𝑐𝑒 𝐹
load of 250g and 20.0cm under a load of 300g. Tensile stress, 𝑆𝑇 = = , 𝐴 = 𝜋𝑟 2 .
𝐴𝑟𝑒𝑎 𝐴
The extension per unit load is A.5×10-5mN-1 radius, 𝑟 = , 𝐴 =
𝑑 𝜋𝑑 2
, 𝑆𝑇 =
𝐹
=
4𝐹
,
𝜋𝑑2
B.8×10-5mN-1 C.5×10-2mN-1 D.8×10-2mN-1 2 4
4
𝜋𝑑 2
(g=10ms ). -2
2 × 107 = 22
4×𝑊
,
𝑊 1=300g, 𝑊 1=250g, 𝐿1=17.5cm, 𝐿2=20.0cm, 7
×(2.8×10−3 )2
Increase in load= 𝑊 2−𝑊 1=300−200=50g, 4× 𝑊 =2×10 × 7
22
×7.84×10-6 = 49.28×101 ,
7
𝐹 =load=50g=0.05kg=0.5N, 492.8
extension=𝐿2−𝐿1=20.0−17.5=2.5cm=2.5×10-2m 𝑊= =123.2N.
4
Extension per unit load=
0.5
= 5×10-2mN-1. 24. The tendon in a man’s leg 0.01m long, if a
2.5×10−2 force of 5𝑁 stretches the tendon by 2.0×10-5m,
- Extension per unit load is the reciprocal of calculate the strain on the muscle A.5×106
force or elastic or stiffness constant. B.5×102 C.2×10-3 D.2×10-7.
20. The property of a material that allows it to be Length,𝐿=0.01m, extension, 𝑒=2.0×10-5m, 𝐹=5𝑁,
stretched into a wire is called A.brittleness 𝑒𝑥𝑡𝑒𝑛𝑠𝑖𝑜𝑛 𝑒
B.malleability C.ductility D.fragibility. Tensile strain, 𝑆 = = ,
𝑜𝑟𝑖𝑔𝑖𝑛𝑎𝑙 𝑙𝑒𝑛𝑔𝑡ℎ 𝐿
21. Elasticity is the property which enables a 𝑆=
2×10−5
= 2×10-7.
material to A.return exactly to its original 0.01
25. An elastic spring of length 20cm stretches by
shape and size on the removal of a force
7mm under a load of 50𝑁. The strain in the string
B.retain its deformed shape after removal of an
is A.1.75×10-1 B.1.75×10-2 C.3.5×10-2
applied force C.be drawn out in length D.break
D.7.0×10-2.
without warning
𝐿=20cm, 𝑒=7mm=0.7cm, 𝑆=?,
- Plasticity is the ability of a material to remain in 𝑒 0.7
its new position or retain its deformed shape Tensile strain, 𝑆 = = =3.5×10-2.
𝐿 20
after distortion by an applied force. 26. A wire of length 2.00m and radius 1.0mm is
- Ductility is the ability of a material to be drawn stretched by 25.0mm on application of a force of
out in length. 103𝑁. Calculate the Young’s modulus for the wire
- Brittleness is the ability of a material to break A.5×1010Nm-2 B.2.5×1010Nm-2 C.5×108Nm-2
easily without warning. D.2.5×108Nm-2.
22. The ratio of tensile stress to tensile strain is 𝐿=2.00m, 𝑟=1.0mm=1×10-3m, 𝐹=103N,

75
Demystified Series Physics Demystified by Dr Timothy
𝑒=25.00mm=25×10-3m, Young’s modulus, 𝐸=? 𝐹
𝐹 = 𝑘𝑒 , extension, 𝑒 = , plastic specimen of
𝑇𝑒𝑛𝑠𝑖𝑙𝑒 𝑠𝑡𝑟𝑒𝑠𝑠 𝑆 𝑘
Yound modulus 𝐸 = = 𝑇, greater extension will have a smaller force
𝑇𝑒𝑛𝑠𝑖𝑙𝑒 𝑠𝑡𝑟𝑎𝑖𝑛 𝑆
𝐹 ⁄𝐴 𝐹𝐿 𝐹𝐿 𝐸𝐴
𝐸= = , 𝐴 = 𝜋𝑟 2 , 𝐸 = , constant 𝑘 than steel. 𝑘 = , force constant 𝑘
𝑒⁄𝐿 𝐴𝑒 𝜋𝑟 2 𝑒 𝐿
103 ×2 2000 increases as Young’s modulus 𝐸 increases, hence
𝐸 = 22 = 22 ,
7
×(1×10−3 )2 ×25×10−3
7
×1×10−6 ×25×10−3 plastic specimen has a higher Young’s modulus
𝐸 = 2.5×1010Nm-2. than steel.
27. A wire of cross-sectional area 6×10-5m2 and 32. When a force of 50N is applied to the free end
length of 50cm stretches by 0.2mm under a load of an elastic cord, an extension of 4cm is
of 3000N. The Young’s modulus for the wire is produced in the cord. Calculate the workdone on
A.8×1010Nm-2 B.1.25×1011Nm-2 C.2.5×1011Nm-2 the cord A.4.0J B.2.0J C.1.0J D.0.5J
D.5×1011Nm-2. 𝐹=50N, 𝑒=4cm=0.04cm, 𝑊=?,
1 1
𝐴=6×10-5m2, 𝐿=50cm=0.5m, 𝐹=3000N, W = 𝐹𝑒 = ×50×0.04 = 1.0J.
𝐹 ⁄𝐴 𝐹𝐿 2 2
𝑒=0.2mm=0.0002m, 𝐸=?, 𝐸 = ⁄ =
𝑒 𝐿 𝐴𝑒
33. An elastic string of 20cm extends to 24cm
3000×0.5 when it supports a weight of 100N. The energy
𝐸= = 1.25×1011Nm-2.
6×10−5 ×0.0002 stored in the string is A.1J B.2J C.5J D.10J.
28. A metallic bar 50cm long has a uniform cross- 𝐿o=20cm=0.2m, 𝐿1=24cm=0.24, 𝐹=100𝑁,
sectional area of 4.0cm2. If a tensile force of 35kN 1 1
𝑒=𝐿1−𝐿o, W = 𝐹𝑒 = 𝐹(𝐿1−𝐿o),
produces an extension of 0.25mm, calculate the 1
2 2
value of Young’s modulus A. 8.75×1010𝑁m-2 W = ×100(0.24−0.2) = 50×0.04 = 2J.
2
B.3.50×10 Nm11 -2 C.1.75×1011Nm-2 34.An elastic string of force constant 200Nm-1 is
D.2.75×10 Nm .
11 -2
stretched through 0.8m within its elastic limit.
𝐿=50cm=0.5m, 𝐹=35kN=35000N, 𝑒=0.25mm= Calculate the energy stored in the string A.64.0J
0.25×10-3m, 𝐴=4.0cm2=4×(10-2)2m=0.0004m, B.80.0J C.128.0J D.160.0J.
𝐹 ⁄𝐴
𝐸= ⁄ = =
𝐹𝐿 35000×0.5
−5 = 1.75×10 Nm .
11 -2 𝑘=200𝑁m-1, 𝑒=0.8m, 𝑊=?,
𝑒 𝐿 𝐴𝑒 0.0004×0.25×10 1 1
29. An elastic string of force constant 2000Nm-1 𝑊 = 𝑘𝑒2 = ×200×0.82 = 100×0.64 = 64.0J.
2 2
undergoes a strain of 0.02 under a load of 20N. 35. A spring of force constant 300Nm-1 is
The natural length of the string is A.50cm compressed such that its length shortens by 3cm.
B.75cm C.100cm D.200cm. Calculate the energy stored in the spring
𝑘=2000Nm-1, 𝑆=0.02, 𝐹=20N, 𝐿=?, A.0.270𝐽 B.0.405𝐽 C.0.135𝑱 D.0.540𝐽
Force per unit length=force constant×strain 𝑘=300𝑁m-1, compression=e=3cm=0.03m,
𝐹 20 20 1 1
=𝑘×𝑆, = 2000×0.02 , = 40, W = 𝑘𝑒 2= ×300×0.032=0.135𝐽.
𝐿 𝐿 𝐿 2 2
20
𝐿 = = 0.5m=50cm. 36. A spiral spring of natural length 30.0cm and
40 force constant of 20Nm-1 is compressed to
30. A metal rod of diameter 7mm and length
20.0cm. Calculate the energy stored in the spring
100mm is made from steel of Young’s modulus
A.0.1J B.1.0J C.10.0J D.100.0J.
2×1011Nm-2. The force constant for the rod is
Compression involves a decrease in the length of
A.7.7×106𝑁m-1 B.3.5×107Nm-1 C.2.8×107Nm-1
an elastic material, 𝐿o=30cm=0.3m, 𝑘=20𝑁m-1,
D.7.7×107Nm-1.
𝐿1=20cm=0.2m, 𝑒=𝐿o−𝐿1, W=?,
𝑑=7mm=0.007m, 𝐿=100mm=0.1m, 1 1 1
𝜋𝑑 2 W = 𝑘𝑒 2 = 𝑘(𝐿o−𝐿1)2 = ×20(0.3−0.2)2 ,
𝐸=2×1011𝑁m-2, 𝑘=?, 𝐴 = 𝜋𝑟 2 = , 2 2 2
4 W =10×0.12 = 1J.
𝜋𝑑2 22
𝐸𝐴 𝐸 4 𝐸𝜋𝑑 2 2×1011 × 7 ×0.0072 37. A spring of force constant 1500Nm-1 is acted
𝑘= = = = ,
𝐿 𝐿 4𝐿 4×0.1 upon by a constant force of 75N. Calculate the
𝑘 = 7.7×107Nm-1. potential energy stored in the spring A.5.0J B.3.8J
- Force constant 𝒌 increases as area 𝑨 C.3.2J D.1.9J.
𝑨
increases and length 𝑳 decreases i.e 𝒌 ∝ , 𝑘=1500Nm-1, 𝐹=75N, potential energy stored in
𝑳
Young’s modulus 𝑬=constant. the spring is elastic potential energy, 𝐸=?,
𝐹2 752
31. A steel sample and a plastic specimen of 𝐸= = = 1.9J.
2𝑘 2×1500
identical dimensions are subjected to the same
38. As the tension in an elastic string increases
tensile force. The plastic specimen shows a
from 100N to 180N, the string extends by 10cm.
greater extension I.plastic has a higher modulus
The work done in increasing the tension in the
of elasticity than steel II.steel has a higher
string is A.8J B.10J C.14J D.18J.
modulus of elasticity than plastic III.steel has
𝐹 1=100N, 𝐹 2=180N, 𝑒=10cm=0.1m,
higher force constant than plastic IV.stell has
Workdone=Average force× extension,
higher demsity than plastic. Which of the above 1 1
statementsis/are correct? A.I B.II C.I and IV D.II Average force= (𝐹 1+𝐹 2)= (100+180)=140N
2 2
and III. W = 140×0.1 = 14J.

76
Demystified Series Physics Demystified by Dr Timothy
39. A force of 2.8N is applied to an elastic string point,a slight increase in load produces a large
of length 1.0m. If the force causes the string to extension not proportional to the force.
increase in length by one-tenth of its original - Beyond the yield point,the material loses it
length. Calculate the elastic potential energy elasticity completely and is entirely plastic.
stored in the string A.0.14𝑱B.0.28𝐽 C.0.42𝐽 - Proportionality limit is the point up to which
D.0.56𝐽. equal increase in load causes equal extension.
1 1
𝐹=2.8𝑁, 𝐿=1.0m, 𝑒 = 𝐿 = ×1=0.1m, - Elastic point or limit is just beyond the
10 10
1 1 proportionality limit and it is the point beyond
𝑊 = 𝐹𝑒 = × 2.8×0.1=0.14𝐽. which a stretched material does not return to its
2 2
40. Calculate the workdone to stretch an elastic original shape and size.
string by 50cm, if a force of 12.5𝑁 produces an - Breaking or fracture or deformation point is the
extension of 5cm in it A.2.250J B.3.125J C.4.450J point where the material breaks into two.
D.31.250J E.44.450J. 44. An additional load to an elastic wire when the
In this question the 12.5N gives an extension of elastic limit is already reached will make the wire
5cm, hence we will have to calculate the force to A.break B.recoil C.vibrate at a high amplitude
constant, before we can determine the work done C.be strained permanently.
in causing an extension of 50cm. An additional load to an elastic material at the
𝐹 1=12.5N, 𝑒1=5cm=0.05m, 𝑘=?, elatic point or limit will extend the wire to the
𝐹 12.5
𝑘= 1= = 250Nm-1. yield point , where it becomes plastic and would
𝑒1 0.05
not return to its orginal shape or size when the
Work done to stretch the elastic string by 50cm,
1 load causing the distortion is removed.
𝑊 = 𝑘e22 , 𝑒2=50cm=0.5m, 45. A spring balance designed for a maximum
2
1
𝑊 = ×250×0.52 = 31.250J. load of 600N was used to weight a load of 820N.
2
41. A spring of length 2.0m requires a force of 6N After the 820N load has been removed the
to stretch it by 0.2cm. What work is done in pointer would not return to the zero mark on the
stretching it by 2cm, if the elastic limit is not scale because A.balance needed adjustment to
exceeded A.0.006J B.0.060J C.0.600J D.1.200J return the pointer B.820N has been removed
E.30.000J. C.pointer had stopped turning D.spring balance
This question is solved exactly the same way was of a cheap type E.elastic limit of the spring
question 40 was solved. material had been exceeded.
𝐿=2.0cm, 𝐹 1=6N, 𝑒1=0.2cm=0.002m, 𝑘=?, The elastic limitor elastic point of the spring is
𝐹 6 the maximum load it can carry (600N). If the
𝑘= 1= = 3000Nm-1,
𝑒1 0.002 spring balance is used to weigh a body (820N)
Workdone in stretching the spring by 2cm, greater than its elastic limit, the elastic limit will
1
W = 𝑘e22 , 𝑒2=2cm=0.02m, be exceeded and it becomes plastic, hence its
2
1 pointer would not return to the zero mark.
W = ×3000×0.022 = 0.6J = 0.600J.
2 46. A piece of wire fixed to a rigid horizontal
42. A body of mass 11kg is suspended from a beam is loaded progressively until its breaks.
ceiling by an aluminium wire of length 2m and Which one of the following graphs will
diameter 2mm. Calculate the elastic energy represents the variation of extension 𝑒 with load
stored in the wirel. [Young’s modulus of 𝐿 for the wire?
aluminium=7.0×1010𝑁m-2, g=10ms-2, 𝜋=3.14] 𝑒 𝑒
A.1.1×10-1J B.5.5×10-2J C.1.1×10-4J D.5.5×10-5J.
𝑚=11kg, 𝐿=2m, 𝑑=2mm=0.002m, g=10ms-2,
𝜋=3.14, 𝐸=7.0×1010Nm-2,
𝐹 = W = 𝑚g=11×10=110N, 0 𝐿 0 𝐿
𝐸𝐴 𝜋𝑑 2 𝐸𝜋𝑑 2
𝑘= , 𝐴 = 𝜋𝑟 2 = , 𝑘= , I. II.
𝐿 4 4𝐿
7×1010 ×3.14×0.0022 𝑒 𝑒
𝑘= = 109,900Nm-1 ,
4×2
𝐹2
𝑘=1.099×105Nm-1, 𝑊 = , 𝐹=110N,
2𝑘
1102
𝑊= = 0.055𝐽 = 5.5×10-2J. 0 𝐿 0 𝐿
2×1.099×105
43. The point at which the moleucles of a loaded III. IV.
wire begin to slide across each other resulting in 𝑒
a rapid increase in the extension is
A.proportionality limit B.elastic point
C.deformation point D.yield point.
Yield point is the point beyond the elastic limit 0 𝐿
where a loaded wire stretches rapidly.At yield V.

77
Demystified Series Physics Demystified by Dr Timothy
A.I B.II C.III D.IV E.V . Force constant 𝑘 = Slope of the graph=
120−0
,
6−0
Extension-Load(or force) graph initially is a 120
straigt line up to the elastic limit, then beyond the 𝑘= = 20Nm-1.
60
elastic limit it changes due to rapid increase in 51. The diagram below shows the force-
extension over a slight change in the load. extension curve of a piece of wire. The energy
47. Load(𝑁) 𝑇 stored when the wire is stretched from 𝐸 to 𝐹?
Force(𝑁)
𝑅 𝑆 R
0.2N Q
𝑄
𝑂 0.1N P
From the diagram above, the point that E F
represents the elastic limit is A.𝑄 B.𝑆 C.𝑹 D.𝑇 . 0.05m 0.10m Extension
Point 𝑅 represents the elastic limit. Hooke’s law A.1.5×10-2J B.7.5×10-1J C.7.5×10-3J D.2.5×10-3J.
is only valid up to elastic limit.Hooke’s law does Workdone or energy stored in an elastic
not hold beyond the elastic limit. Point 𝑆 material=Area of the figure under the force-
represents the yield point while point 𝑇 extension graph. The figure between 𝐸 and 𝐹 is a
1
represents the breaking or fracture point. trapezium. 𝐴 = (𝑎 + 𝑏)ℎ
2
48.The diagram below represents the stress- 1 1
𝑊 = (0.1+0.2)(0.1−0.05) = ×0.3×0.05,
strain graph of a loaded wire. Which of the 2 2
following is correct? 𝑊 = 7.5×10-3𝐽.
stress 𝐿 52. An object of mass 20kg is dropped on a spring
𝐽 𝐾 which is placed 5m below. If the spring has a
𝐼 force constant of 5×104𝑁m-1, the maximum
compression of the spring is A.1.0m B.0.5m
C.0.2m D.0.04m
Strain 𝑚=20kg, ℎ=5m, 𝑘=5×104𝑁m-1, compression, e=?,
A.At 𝐼, the wire becomes plastic B.𝑱 is the yield g=10ms-2 Potential energy of the object is
point C.𝐿 is the elastic limit D.At 𝐾, the wire converted into the elastic 𝑃. 𝐸 of the spring,
1 1
breaks. 𝑚𝑔ℎ = 𝑘𝑒2 , 20×10×5= ×5×104× 𝑒2 ,
2 2
𝐼 is the elastic limit, 𝐽 is the yield point, between 20×10×5×2 1 1 1
𝐽 and 𝐾 the wire is plastic, the wire breaks at 𝐿, 𝑒2 = = , 𝑒=√ = = 0.2m.
5×104 25 25 5
which is the breaking point. - Question drill – 1. Use the data in the table
49. Force below to determine the length 𝑋 of a wire
assuming Hooke’s law is obeyed.
Ans=810.0mm.
0 extension Force 0.0 5.0 10.0 50.0
The figure shown is a graph of force against applied/ N
extensiom for a spiral spring. The force constant Length of 800.0 801.0 802.0 X
of the spring is given by the A.slope of the linear wire/ mm
portion of the graph B.length of the linear 2. A spiral spring which obey’s Hooke’s law has
portion of the graph C.area under the linear an unstretched length of 20cm. If it stretches to
portion of the graph D.area under the entire 30cm when a mass of 500g is hung from one end,
graph. calculate the new length when a mass of 1000g
Force constant 𝑘 is given by the slope of the replaces the 500g mass A.60cm B.40cm C.35cm
𝐹
linear portion of the graph , 𝑘 = . D.20cm E.10cm.
𝑒
- The area of the entire graph gives the work done
or energy stored in an elastic material. Jamb past questions on Elastic properties :
50. The diagram below represents the graph of [1978/5,1983/32,1987/7,14,1982/50,1981/43,
the force(𝐹) applied in stretching a spiral against 1987/7,1989/10,1994/16,2004/35,2005/42,
the corresponding extension(𝑋). The force 2007/30,2012/15,1992/10,1998/13,1999/14,
constant of the spring is 2006/40,2011/14,2014/16]
(𝐹)
120
80
40
0
2 4 6 (𝑋)
A.40Nm-1 B.30Nm-1 C.20Nm-1 D.10Nm-1.

78
Demystified Series Physics Demystified by Dr Timothy

Chapter 8 – Simple machines


● A simple machine is a device which enable constant.
work to be performed faster and more - Lubricants e.g grease and ball bearings are
conveniently, where a small effort is used to used in machines to reduce friction, thereby
overcome a large load e.g a fishing rod. increasing 𝑴. 𝑨 and Effficiency but 𝑽. 𝑹
- Terms in simple machines : remains constant(as it is independent of
1.Load 𝑳 – It is the force overcomed. friction).
2.Effort 𝑬 – It is the force exerted to overcome - Classification of machines – Generally, the
the load. basic machine of which all other machines are
3.Mechanical advantage 𝑴. 𝑨 or force ratio – made of are hydraulic press, levers and
𝑳
It is the ratio of load to effort. 𝑴. 𝑨 = . inclined planes.
𝑬
● Levers – A small effort at a greater distance
4.Velocity ratio 𝑽. 𝑹 or speed or distance ratio
from the fulcrum is used to overcome a large load
– It is the ratio of distance moved by the effort(or
closer to the fulcrum, in order to obtain the
effort arm) 𝒙 to distance moved by the load(or
𝒙 greatest mechanical advantage. It works on the
load arm) 𝒚. 𝑽. 𝑹 = . Both 𝑀. 𝐴 and 𝑉. 𝑅 have
𝒚 principle of moment.
no units. - Classes of lever – There are three classes;
5.Efficiency 𝝐 – It is the ratio of the work output I.First class lever: The fulcrum 𝐹 is between the
to work input. 𝝐 =
𝑾𝒐𝒓𝒌 𝒐𝒖𝒑𝒖𝒕
×100. the load 𝐿 and the effort 𝐸[𝑳𝑭𝑬] e.g crow bar,
𝑾𝒐𝒓𝒌 𝒊𝒏𝒑𝒖𝒕
pliers,pincers,claw hammer,scissors,see saw and
Work output=Workdone by load or useful work
common balance. 𝑀. 𝐴 and 𝑉. 𝑅 is usually greater
done=Effort×distance moved by effort.
than 1 but could be less than or equal to 1.
Work input=Workdone by effort or workdone
II.Second class lever: The load 𝐿 is between the
on load or total work input=Load×distance
effort 𝐸 and the fulcrum 𝐹 or pivot[𝑬𝑳𝑭] e.g
moved by load.
bottle opener, wheel barrow and nutcracker.𝑀. 𝐴
- The efficiency of a machine is also given by the
and 𝑉. 𝑅 are always greater than 1.
ratio of the mechanical advantage to velocity
𝑴.𝑨 III.Third class lever: The effort 𝐸 is between the
ratio. 𝝐 = ×100. load 𝐿 and the fulcrum 𝐹 e.g forceps,sugar
𝑽.𝑨
- Work input is always less than work input due tongs,forearm of man,fishing rod and table
to work done against friction i.e knife.𝑀. 𝐴 and 𝑉. 𝑅 are always less than 1.
Work input=Work output+Work done - First and second class levers are force
against friction, 𝑾𝒊 = 𝑾𝒐 + 𝑾𝒇 . magnifiers while third class levers are
- Mechanical advantage 𝑀. 𝐴 is always less than distance magnifiers.
the velocity ratio 𝑉. 𝑅. - In a lever system, 𝑀. 𝐴 = 𝑉. 𝑅.
- Mechanical advantage 𝑴. 𝑨 and efficiency 𝝐 - A machine is said to be a force multiplier
affected by friction and load , 𝑴. 𝑨 ∝ 𝑳 (not when the load is greater than the effort.
linearly), hence the efficiency can never be ● Pulley systems – It is a machine used to raise
100%. The highest 𝑴. 𝑨 is obtained in a or lower heavy loads majorly in construction
frictionless machine. sites.
- Velocity ratio 𝑽. 𝑹 depends on the size and - The 𝑽. 𝑹 of a pulley system is equal to the
shape of the machine parts and not affected number of pulleys.
or independent of friction and load. - The 𝑽. 𝑹 can also be given by the number of
- Grapical relationships of mechanical advantage ropes or strings supporting the pulleys, when
𝑀. 𝐴, velocity ratio 𝑉. 𝑅, load 𝐿, effort 𝐸 and the number of pulleys are lesser.
efficiency 𝜖 – - Block and tackle pulley system:It consists of
𝑴. 𝑨 𝑽. 𝑹 fixed and movebale pulleys. 𝑽. 𝑹 of a block and
tackle system=number of moveable
𝑳 𝑳 pulleys+number of fixed pulleys.
𝑴. 𝑨 𝑳 - The greater the load to be lifted in a pulley
system the greater the efficiency, as the weight of
the pulleys attached to one load is less in
𝑽. 𝑹 𝑬 proportion to the load.
𝝐 - Efficiency increases as the mass(weight) or
number of the pulleys reduces, due to
decrease in friction.
𝑳 ● Inclined planes – It is a machine used to move
- 𝑀. 𝐴 initially increases with 𝐿 and then becomes heavy loads from a lower to a higher level up, by

79
Demystified Series Physics Demystified by Dr Timothy
sloping a plank e.g a plank sloping from the floor equal to the pitch 𝑷.
of a van or truck through which heavy loads like - Pitch 𝑷 =
𝒉𝒆𝒊𝒈𝒉𝒕 𝒓𝒂𝒊𝒔𝒆𝒅 𝒐𝒇 𝒕𝒉𝒆 𝒍𝒐𝒂𝒅 𝒉
= .
drums of oil can be raised to the floor of the van 𝒏𝒖𝒎𝒃𝒆𝒓 𝒐𝒇 𝒓𝒆𝒗𝒐𝒍𝒖𝒕𝒊𝒐𝒏𝒔 𝒏

or truck. - The distance moved by the effort=2𝝅𝒓.


- The effort distance or effort arm is equal to the 𝑟=radius of the handle or circle. or length of
length of the plane 𝐿, while the load distance or tommy bar.
𝟐𝝅𝒓 𝒓
load arm is equal to the height of the plane ℎ. - 𝑽. 𝑹 = . 𝑽. 𝑹 ∝ i.e 𝑽. 𝑹 increases as length
𝑷 𝑷
𝑳
𝑽. 𝑹 = . 𝑽. 𝑹 =
𝟏
, θ=angle of inclination of of tommy bar or radius of the circle 𝒓 i.e effort
𝒉 𝒔𝒊𝒏 𝜽
the plane to with the horizontal. arm, increases and the pitch 𝑷 i.e load arm,
- The efficiency of an inclined plane depends decreases. - In
on the degree of roughness of the of the a screw jack rotational motion of the effort is
inclined plane surface and not on the slope. converted into linear motion of the load.
- The velocity ratio 𝑽. 𝑹 of the inclined plane ● Wheel and axle –It is used to lift heavy loads
decreases as the slope or angle of inclination such as anchor ships and to lift water from deep
to the horizontal θ increases , 𝑽. 𝑹 decreases wells. The wheel and axle are connected such
as 𝒔𝒊𝒏θ increases. that they both have a common axis of rotation
- In a frictionless pulley, 𝑀. 𝐴 = 𝑉. 𝑅, as efficiency and rotated through one complete revolution at
is 100%. Therefore 𝑴. 𝑨 depends on the length the same time.
and height of the inclined plane. - The wheel and axle have the same period
- The effort 𝑬 in a rough inclined plane i.e friction and frequency i.e they make the same number
is not negligible, is given by – of revolutions per second.
𝑬 = 𝒎𝒈𝒔𝒊𝒏𝜽 + 𝑭𝑹 = 𝒎𝒈𝒔𝒊𝒏𝜽 + 𝝁𝒎𝒈𝒄𝒐𝒔𝜽 , - The radius of the wheel 𝑅 is the effort arm while
𝐹𝑅 =frictional force, 𝜇=coefficient of static friction the radius of the axle 𝑟 is the load arm.
𝑹 𝑹
of the inclined plane. - 𝑽. 𝑹 = . 𝑴. 𝑨 = , i.e in the absence of friction.
𝒓 𝒓
- The load 𝐿 moved up the plane 𝑳 = 𝒎𝒈. - The bicycle makes use of the wheel and axle
- The mechanical advantage 𝑀. 𝐴 is given by – principle.
𝑴. 𝑨 = =
𝑳 𝒎𝒈
=
𝟏
. ● Gear wheels –They are two tooted wheels
𝑬 𝒎𝒈𝒔𝒊𝒏𝜽+𝝁𝒎𝒈𝒄𝒐𝒔𝜽 𝒔𝒊𝒏𝜽+𝝁𝒄𝒐𝒔𝜽
which applies the wheel and axle principle
- Efficiency 𝜖 of the rough inclined plane –
𝑴.𝑨 𝒔𝒊𝒏𝜽 between the wheels of different radii connected
𝝐= ×100= ×100. by a belt amd rotating on separate shafts
𝑽.𝑹 𝒔𝒊𝒏𝜽+𝝁𝒄𝒐𝒔𝜽
● The Wedge – It is a combination of two -They are used to change speed, direction of
inclined planes used to separate bodies held motion(rotation), alter 𝑽. 𝑹. They also
together by strong and large cohesive forces e.g increase/magnify force and transmit motion.
𝒂𝒏𝒈𝒖𝒍𝒂𝒓 𝒔𝒑𝒆𝒆𝒅 𝒐𝒇 𝒔𝒎𝒂𝒍𝒍𝒆𝒓 𝒈𝒆𝒂𝒓 𝝎
splitting timbers. Examples of wedges are - 𝑽. 𝑹 = = 𝒔,
𝒂𝒏𝒈𝒖𝒍𝒂𝒓 𝒔𝒑𝒆𝒆𝒅 𝒐𝒇 𝒍𝒂𝒓𝒈𝒆𝒓 𝒈𝒆𝒂𝒓 𝝎𝑳
chisels,knives,axe e.t.c. 𝒏𝒖𝒎𝒃𝒆𝒓 𝒐𝒇 𝒕𝒆𝒆𝒕𝒉 𝒐𝒏 𝒍𝒂𝒓𝒈𝒆𝒓 𝒈𝒆𝒂𝒓 𝑵𝑳
𝒔𝒍𝒂𝒏𝒕 𝒉𝒆𝒊𝒈𝒉𝒕 𝒐𝒇 𝒘𝒆𝒅𝒈𝒆 𝒙 𝑽. 𝑹 = = .
- 𝑽. 𝑹 of a wedge= = . 𝒏𝒖𝒎𝒃𝒆𝒓 𝒐𝒇 𝒕𝒆𝒆𝒕𝒉 𝒐𝒏 𝒔𝒎𝒂𝒍𝒍𝒆𝒓 𝒈𝒆𝒂𝒓 𝑵𝒔
𝒕𝒉𝒊𝒄𝒌𝒏𝒆𝒔𝒔 𝒐𝒇 𝒘𝒆𝒅𝒈𝒆 𝒕𝒉
- The smaller gear(less toothed gear) is the
- Neglecting frictional forces, 𝑀. 𝐴 = 𝑉. 𝑅,
𝒙 driving gear and rotates faster than the larger
𝑴. 𝑨 = . Slant height of the edge=effort gear(more toothed gear) which is the driven
𝒕𝒉
distance effort arm, thickness of wedge=load gear.
distance or load arm. ● Hydraulic press/jack – It operates on Pascal’s
- A long(greater height), thin(lesser principle of pressure in fluids.
thickness) wedge has a higher 𝑴. 𝑨 than a - The pressure applied by the effort in the
short(lesser height), thick(greater thickess) smaller(pump) piston of radius 𝑟 and area 𝐴1 is
wedge. 𝑴. 𝑨 increases i.e becomes greater as equal to the pressure received by the load in the
the angle θ between the slant heights larger(ram) piston of radius 𝑅 and Area 𝐴2.
decreases i.e becomes smaller. 𝑨𝟐 𝑹𝟐
- 𝑴. 𝑨 = . 𝑽. 𝑹 = . 𝑟=radius of the pump
● ScrewJack – It is made of an inclined plane 𝑨𝟏 𝒓𝟐
wrapped round a cylinder, with the hypotenus of cylinder, 𝑅=radius of the arm cylinder. A2=area of
the inclined plane forming a spiral path which large piston(ram), A1=area of small
makes up the thread. It is used in screw jacks for piston(pump).
lifting motor car, nuts and bolts, screw driver,
spanner. Examples :
- The distance between two consecutive threads 1. Which of the following definitions about
is the pith 𝑷 of the screw i.e the load distance. simple machines is not correct? A.velocity ratio
𝑑𝑖𝑠𝑡. 𝑚𝑜𝑣𝑒𝑑 𝑏𝑦 𝑒𝑓𝑓𝑜𝑟𝑡 𝑖𝑛 𝑎 𝑔𝑖𝑣𝑒𝑛 𝑡𝑖𝑚𝑒
- The distance through which a screw moves is B.mechanical
𝑑𝑖𝑠𝑡. 𝑚𝑜𝑣𝑒𝑑 𝑏𝑦 𝑙𝑜𝑎𝑑 𝑖𝑛 𝑡ℎ𝑒 𝑠𝑎𝑚𝑒 𝑡𝑖𝑚𝑒
along its head when the head is turned 𝑙𝑜𝑎𝑑 𝑤𝑜𝑟𝑘 𝑔𝑜𝑡 𝑜𝑓𝑓
through one complete revolution i.e 360° is advantage is C.efficiency is
𝑒𝑓𝑓𝑜𝑟𝑡 𝑤𝑜𝑟𝑘 𝑝𝑢𝑡 𝑖𝑛

80
Demystified Series Physics Demystified by Dr Timothy
D.work put in is the product of load and energy conversion is A.
𝐸2
B.
𝑬𝟑
C.
𝐸3
D.
𝐸2 +𝐸3
.
𝐸1 𝑬𝟏 𝐸1 +𝐸2 𝐸1
distance moved by the load E.efficiency is
𝑚𝑒𝑐ℎ𝑎𝑛𝑖𝑐𝑎𝑙 𝑎𝑑𝑣𝑎𝑛𝑡𝑎𝑔𝑒 The energy input into the machine=Initial energy
. of water=𝐸1 , useful energy output from the
𝑣𝑒𝑙𝑜𝑐𝑖𝑡𝑦 𝑟𝑎𝑡𝑖𝑜
Work put in=Work input=Work done by effort machine=energy converted into electrical
=Work done on load=Effort×effort distance. energy=𝐸3 . 𝐸3 is wasted energy.
Work got off=Work output=useful work done Efficency=
𝐸𝑛𝑒𝑟𝑔𝑦 𝑜𝑢𝑡𝑝𝑢𝑡 𝐸
= 3.
𝐸𝑛𝑒𝑟𝑔𝑦 𝑖𝑛𝑝𝑢𝑡 𝐸1
=Work done by load=Load×load distance.
6. The variation of mechanical advantage with
Useless work done=Work done against friction.
load for a simple machine with a velocity ratio of
2. Which of the following statements about a
6 is correctly illustrated as
machine is not correct? A.It is a device for doing
A. 𝑀. 𝐴 B. 𝑀. 𝐴
work more conveniently B.All machines are
made of gear,wheels,levers,pulleys and
screws C.A machine often does work faster than
a man D.A machine is any device by means of
Load Load
which a force applied at one point is made to do
C. 𝑴. 𝑨 D. 𝑀. 𝐴
work at another point E.A fishing rod is a
machine
All machine are made of inclined planes,lever and
hydraulic press,which are the three basic
Load Load
machines.
7. The variation of velocity ratio with load for a
3. In practice, a machine can never be 100%
simple machine is correctly illustrated as
efficient. This implies that; A.the mechanical
A. 𝑉. 𝑅 B. 𝑽. 𝑹 C. 𝑉. 𝑅
advantage of any machijne is always less than 1
B.the velocity ratio of a machine is always more
than 1 C.the velocity ratio of a machine is never
greater than the mechanical advantage D.the
Load Load Load
useful work obtained from a machine is
D. 𝑉. 𝑅
always less than the work done by it E.the ratio
of the work output of a machine to the work input
is always greater than 1.
Useful work obtained or work output is always
Load .
less than the work input due to workdone in
8. A machine with a velocity ratio of 25 moves
overcoming frictional forces. 𝑀. 𝐴 is always less
load of 3300𝑁 when an effort of 165𝑁 is applied
than the 𝑉. 𝑅, as 𝑀. 𝐴 is affected by frictionwhile
to it. Calculate the efficiency of the machine
𝑉. 𝑅 is independent of friction.
A.80% B.60% C.50% D.25% E.20%
4. Which of the following statements defines 𝐿
correctly the efficiency of a machine? A.the load 𝑉. 𝑅=25, 𝐿=3300𝑁, 𝐸=165𝑁, 𝜖=?, 𝑀. 𝐴 = ,
𝐸
carried by the machine divided by the effort Efficiency, 𝜖 =
𝑀.𝐴
×100= ×
𝐿 1
×100,
𝑉.𝑅 𝐸 𝑉.𝑅
6required in carrying the load B.the distance 3300 1
moved by the load divided by the distance moved 𝜖= × ×100=80%.
165 25
by the effort C.useful work done bymachine 9. A machine of velocity ratio 6 requires an effort
divided by the total work put into it D.the of 400𝑁 to raise a load of 800𝑁 through 1m. Find
movement of the load divided by the movement the efficiency of the machine A.55.6% B.50.0%
of the effort about the same point E.the C.22.2% C.33.3%.
𝐿
temperature at the end of the application of the 𝑉. 𝑅=6, 𝐸=400𝑁, 𝐿=800𝑁, 𝑀. 𝐴 = , 𝜖=?,
𝐸
machine divided by the initial temperature. 𝑀.𝐴 𝐿 1
𝑤𝑜𝑟𝑘 𝑜𝑢𝑡𝑝𝑢𝑡 Efficiency, 𝜖 = ×100= × ×100,
Efficiency= , work output is useful 800 1
𝑉.𝑅 𝐸 𝑉.𝑅
𝑤𝑜𝑟𝑘 𝑖𝑛𝑝𝑢𝑡 𝜖= × ×100=33.3%.
workdone or workdone by load, work input or 400 6

total work input is work done by effort or work 10. A machine is used to lift a load of 20𝑁 through
done on load. a height of 10m. If the efficiency of the machine is
5. 40%, how much work is done? A.500𝑱 B.300𝐽
water with water with C.120𝐽 D.80𝐽.
energy 𝐸1 energy 𝐸2 𝐿=20𝑁, load distance, 𝑦=10m, 𝜖=40%, work done
by the machine=work input=work done by the
Electrical energy 𝐸3 effort=𝑊𝑖 ?, Work input 𝑊𝑜 =Work done by
The diagram above represents the energy flow load=load×load distance= 𝐿 × 𝑦,
𝑊
through a hydroelectric power. The efficiency of 𝑊𝑜 = 𝐿 × 𝑦 = 20×10=200𝐽, 𝜖 = 𝑜 × 100,
𝑊𝑖

81
Demystified Series Physics Demystified by Dr Timothy
40 =
200
×100 , 𝑊𝑖 =
200×100
= 500𝐽. mechanical advantage? (The number indicate
𝑊𝑖 40
their respective distances)
11. The efficiency of a simple machine is 75%. If A. 8cm 2cm B. 5cm 5cm
the machine does 2400J of work, the energy
spent to overcome friction is A.600J B.800J 𝐿 𝐸 𝐿 𝐸
C.1800J D.2400J E.3200J. C. 𝑬 D. 𝐸
𝜖=75%, work input, 𝑊𝑖 =2400J, Work done
against friction=energy spent to overcome
2cm 8cm 2cm 8cm
friction, 𝑊𝑓 =?, Work output, 𝑊𝑜 =?,
𝑊𝑜 𝑊𝑜 2400×75
𝜖= × 100, 75= ×100, 𝑊𝑜 = 𝑳 𝐿
𝑊𝑖 2400 100
𝑊𝑜 =1800𝐽. Work done against friction 𝑊𝑓 , In a lever system, 𝑀. 𝐴 = 𝑉. 𝑅.Greatest𝑉. 𝑅hence,
𝑊𝑓 = 𝑊𝑖 − 𝑊𝑜 = 2400−1800 = 600J. greatest 𝑀. 𝐴 is achived in lever system in option
C, where the load is closer to the fulcrum than the
12. A machine has an efficiency of 80%. If the
effort.
machine is required to overcome a load of 60N
- Option A and B are first class lever. Option C
with a force of 40𝑁, calculate its velocity ratio
is a second class lever. Option D is a third class
A.0.53 B.0.83 C.1.20 D.0.50 E.1.88.
lever.
𝜖=80%, 𝐿=60N, 𝐸=40𝑁, 𝑉. 𝑅=?,
𝑀.𝐴 𝐿 1 18. Which of the following machines does not
Efficiency, 𝜖 = ×100= × ×100 , apply the lever principle? A.Claw hammer
𝑉.𝑅 𝐸 𝑉.𝑅
60 1 60×100
80= × ×100, 𝑉. 𝑅 = =1.88. B.Wheel barrow C.Single pulley D.Sugar tong.
40 𝑉.𝑅 40×80
13. The efficiency of a machine is 80%. Calculate Claw hammer is a first class lever, wheel barrow
the workdone by a person using the machine to is a second class lever while sugar tong is a third
raise a load of 300kg through a height a of 4m class lever. Single pulley is an example of a pulley
(g=10ms-2) A.7500𝐽 B.15000𝑱 C.30000𝐽 system.
D.45000𝐽. 19. Which of the following instruments, in its
𝜖=80%, 𝐿=𝑊=300kg=3000𝑁, load distance normal use, serves as a first class lever? A.pair
𝑦=4m, Work output 𝑊𝑜 = 𝐿 × 𝑦 , Work done by of scissors B.a pair of sugar tongs C.a bottle
the person using the machine=Work input 𝑊𝑖 =?, opener D.a knife E.a nut cracker.
𝑊 3000×4 Pair of sugar tongs and knife are third class
𝜖 = 𝑜 × 100, 80= ×100,
𝑊𝑖 𝑊𝑖 levers. Bottle opener and nut craker are second
3000×4×100
𝑊𝑖 = =15000𝐽. class levers.
80
14. A machine of velocity ratio 6 is used to lift a 20. In wich of the following simple machines is
load of 1200N. If the efficiency of the machine is the effort applied between the load and the
80%, calculate the magnitude of the effort fulcrum? A.claw hammer B.pliers C.wheel
applied A.90N B.96N C.250N D.960N. barrow D.sugar tong E.nutcracker
𝑉. 𝑅=6, 𝐿=1200N, 𝜖=80%, 𝐸=?, Effort is applied between the load and the
𝑀.𝐴 𝐿 1 fulcrum in a third class lever e.g sugar tong. Claw
Efficiency, 𝜖 = ×100= × ×100, hammer and pliers are first class levers. Wheel
𝑉.𝑅 𝐸 𝑉.𝑅
1200 1 1200×100
80= × ×100, 𝐸 = = 250N. barrow and nutcracker are second class lever.
𝐸 6 6×80
15. A machine of velocity ratio 5 is used in lifting 21. Which of the following levers is not in the
a load with an effort of 500N. If the machine is same class as the others in respect of the
80% efficient, determine the magnitude of the locations of the load, the effort and the fulcrum
load A.2500N B.2000N C.900N D.625N. A.pincers B.see-saw C.pliers D.sugar tongs
𝑉. 𝑅=5, 𝐸=500N, 𝜖=80%, 𝐿=?, E.scissors.
𝑀.𝐴 𝐿 1 All the other options apart from sugar tongs are
Efficiency, 𝜖 = ×100= × ×100,
𝐿 1
𝑉.𝑅 𝐸 𝑉.𝑅
80×500×5
first class lever.
80 = × ×100, 𝐿 = = 2000N. 22. At what distance from the fulcrum will a boy
500 5 100
16. It is more convenient to use a lever when the of mass 40kg balance another boy of mass 60kg
A.effort is closer to the fulcrum than the load who is sitting 5.0m from the fulcrum on the other
B.load and the effort are at the same distance side? A.1.5m B.3.3m C.4.0m D.7.5m E.20.0m.
from the fulcrum C.effort is almost at the same 𝑚1=40kg, 𝑑 1=?, 𝑚2=60kg, 𝑑 1=5.0m,
distance from te fulcrum D.load is closer to the Lever system works on principle of moment.
fulcrum than the effort. Taking moment about the fulcrum,
Lever works on principle of moments. For a lever 𝐹 1𝑑 1= 𝐹 2𝑑 2 or 𝑚1𝑑 1= 𝑚2𝑑 2, 40× 𝑑 2=60×5,
300
to raise a large load buy a small effort, the load 𝑑 2= = 7.5m.
40
must be closer to the fulcrum than the effort, 23. An effort 𝑃 applied at one end of a crowbar
1
𝐹∝ . just overcome the resistance 𝑊 at the lid of a tin.
𝑑
17. Which of the following levers has the greatest The mechanical advantage of the crowbar is

82
Demystified Series Physics Demystified by Dr Timothy
expressed as A.𝑊 + 𝑃 B.𝑃 − 𝑊 C. D. E.𝑊𝑃
𝑾 𝑃 29. Lubricants used in machines to reduce
𝑷 𝑊
friction A.decrease the 𝑀. 𝐴 B.Increase the 𝑴. 𝑨
Effort, 𝐸=𝑃, Load, 𝐿=force overcomed= 𝑊,
𝐿 𝑊 C.increase the 𝑉. 𝑅 D.decrease the 𝑉. 𝑅.
𝑀. 𝐴 = = . Lubricants reduce friction inorder to increase
𝐸 𝑃
24. A lever of length 10m is used to lift a load of 𝑀. 𝐴 and efficiency of a machine. 𝑉. 𝑅 is not
mass 180kg as shown in the diagram below The affected by friction.
pivot is 2m from the load. If the efficiency of the 30. What is the velocity ratio of a pulley system
lever is 90%, the force 𝐹 required to lift the load having three pulleys in the fixed block and two in
is A.50𝑁 B.405𝑁 C.450𝑁 D.500𝑵. the movable block? A.2 B.3 C.4 D.5 E.6.
180kg 𝑉. 𝑅 in a block and tackle system is the total
number of pulleys=number of movable pulleys +
number of fixed pulleys.
2m 𝐹 Number of moveable pulleys=3, number of fixed
As efficnecy is not 100% hence, 𝑀. 𝐴 ≠ 𝑉. 𝑅 and pulleys=2, 𝑉. 𝑅 =3+2=5.
principle of moment is not used to find the effort. 31. The diagram above illustrates
𝜖=90%, 𝐿=180kg=1800𝑁, load distance 𝑦=2m, a block and tackle pulley
effort distance 𝑥 = 10−2=8m, 𝐸=F=?, system in which an effort 𝐸
𝑥 8 𝑀.𝐴
𝑉. 𝑅 = = = 4. Efficiency, 𝜖 = ×100 𝑬 supports a load of 100.0N.
𝑦 2 𝑉.𝑅
𝐿 1 1800 1 If the efficiency of the
𝜖= × ×100, 90= × ×100, machine is 75%, calculate
𝐸 𝑉.𝑅 𝐹 4
1800×100
𝐹= = 500N. the value of 𝐸
90×4
25. A block and tackle system has 6 pulleys. If the
efficiency of the machine is 60%, determine its
mechanical advantage A.12.0 B.10.0 C.3.6 D.1.8
𝑉. 𝑅=number of pulleys=6, 𝜖=60%, 𝑀. 𝐴=?,
𝑀.𝐴 𝑀.𝐴
Efficiency, 𝜖 = ×100, 60 = ×100,
𝑉.𝑅 6
60×6
𝑀. 𝐴 = = 3.6. 𝑳 A.22.2N B.38.0N C.67.0N D.75.0N.
100
26. A block and tackle system has six pulleys. A 𝑉. 𝑅=number of pulleys=6, 𝐿=100.0𝑁, 𝜖=75%,
𝑀.𝐴 𝐿 1
force of 50N applied to its lifts a load of weight W. 𝐸=?, Efficiency, 𝜖 = ×100= × ×100,
𝑉.𝑅 𝐸 𝑉.𝑅
If the efficiency of the system is 40%, calculate W 100 1 100×100
75= × ×100, 𝐸 = = 22.2N.
A.300N B.200N C.140N D.120N. 𝐸 6 6×75
𝑉. 𝑅=number of pulleys=6, 𝜖=40%, applied force 32. The diagram is a block and
𝐸=50𝑁, 𝐿=W=?, tackle system in which an
𝑀.𝐴 𝐿 1 effort of of 80N is used to
Efficiency, 𝜖 = ×100 = × ×100,
𝑉.𝑅 𝐸 𝑉.𝑅 80N lift a load of 240N. The
𝑊 1 40×50×6
40 = × ×100, W = = 120N. efficiency of the machine is
50 6 100
27. The figure shows a frictionless
pulley system in which a weight
in equilibrium with a weight of
of 40𝑁. Find the value of 𝑊

40N
𝑊 A.13.3N B.20.0N C.40.0N D.80.0N.
𝑉. 𝑅=number of pulleys=2, for a frictionless
pulley, 𝑀. 𝐴=𝑉. 𝑅=2, 𝐿=40N, 𝐸=W=?,
𝐿
𝑀. 𝐴 = , 𝐸 =
𝐿 40
= = 20N. 240.0N
𝐸 𝑀.𝐴 2 A.60% B.50% C.40% D.33%.
28. A load of mass 5kg is being lifted 𝐿=240.0N, 𝐸=80N, 𝑉. 𝑅=number of pulleys=6,
P vertically upwards using a pulley 𝑀.𝐴 𝐿 1
Efficiency, 𝜖 = ×100= × ×100,
system as shown. If the system is 𝑉.𝑅 𝐸 𝑉.𝑅
240 1
80% efficient, calculate the value 𝜖= × ×100 = 50%.
80 6
𝐸 of the applied effort 𝐸(g=10ms-2) 33. A machine of efficiency 80% is used to lift a
A.80.0N B.62.5N C.40.0N D.6.3N box. If the effort applied by the machine is twice
5kg E.4.0N . the weight of the box, calculate the velocity ratio
𝐿=5kg=50N, 𝑉. 𝑅=number of pulleys=1, 𝜖=80%, of the machine A.0.21 B.0.32 C.0.63 D.0.64
𝑀.𝐴 𝐿 1
𝐸=?, Efficiency, 𝜖 = ×100= × ×100, 𝜖=80%, 𝐿=weight of the box=𝑊, 𝐸 =2𝑊=2𝐿,
𝑉.𝑅 𝐸 𝑉.𝑅
𝑀.𝐴 𝐿 1
80 =
50 1
× ×100, 𝑊 =
100×50
= 62.5N. Efficiency, 𝜖 = ×100= × ×100
𝐸 1 80 𝑉.𝑅 𝐸 𝑉.𝑅

83
Demystified Series Physics Demystified by Dr Timothy
80 =
𝑊
×
1
×100, 𝑉. 𝑅 = ,
𝑊×100
Velocity ratio of an inclined plane depends on the
2𝑊 𝑉.𝑅 80×2𝑊
ratio of the length of the plane and vertical height
𝑉. 𝑅 =0.625=0.63. 𝐿
34. Which of the following actions will improve of the plane , 𝑉. 𝑅 = .

the efficiency of a pulley system? A.reducing the - Option C and E are only true for a frictionless
mass of the pulley B.increasing the frictional inclined plane (𝑴. 𝑨 = 𝑽. 𝑹).
force between the string and the pulley 40. An inclined plane used as a machine makes
C.increasing the mass per unit length of the string an angle of 30° with the horizontal. What is the
of the pulley D.increasing the mass of the pulley velocity ratio of the machine? A.
𝟏
B.sin 30°
𝐬𝐢𝐧 𝟑𝟎°
Friction increases as the mass or weight of the 1 1
pulleys increases i.e 𝐹 R ∝ 𝑚 𝑜𝑟 𝑊. The mass of C. D. E.cos 30° .
tan 30° cos 30°
1 1
the pulleys increases as the number of the pulleys 𝑉. 𝑅 of an inclined plane= = . θ=30°.
sin 𝜃 sin 30°
increases.Hence, reducing the mass of the pulley 41. The mechanical advantage (𝑀. 𝐴) of an
also reduces the frictional forces, thereby inclined plane depends on A.its length B.its
increasing the efficiency of the pulley. height C.the product of its length and height
35. A block and tackle system is used to raise a D.the ratio of its length to its height.
load of 250N through a vertical height of 30m. 𝐿
In a frictionless pulley, 𝑀. 𝐴 = 𝑉. 𝑅 = . Hence
What is the efficiency of the system if the ℎ
workdone against friction is 1500J? A.62.5% mechanical advantage, 𝑀. 𝐴 of an inclined plane
B.73.3% C.83.3% D.94.3% . depends on the ratio of its length to its height.
𝐿=250N, load distance=30m, Work output, 𝑊𝑜 or 42. In the diagram illustrates
Work done by load=250×30=7500J, Work done M above, a block of mass M
against friction, 𝑊𝑓 =1500J, Work input 𝑊𝑖 or is at rest on a plane
workdone by effort = 𝑊𝑜 + 𝑊𝑓 , θ inclined at an angle θ to
the horizontal. As θ increases, the velocity ratio
𝑊𝑖 = 𝑊𝑜 + 𝑊𝑓 = 7500+1500 = 9000J,
𝑊𝑜 7500 of the plane A.increases B.decreases C.remains
𝜖= × 100 = ×100 = 83.3%. the same D.decreases and later increases.
𝑊𝑖 9000
1 1
Use the diagram below to answer 36 – 38. 𝑉. 𝑅 = , 𝑉. 𝑅 ∝ , as the angle of inclination
sin 𝜃 𝜃
Support θ or sin 𝜃 increases, 𝑉. 𝑅 decreases.
- The diagram below shows a box X of weight W
Effort resting on a plank PQ that has its lower end P
hinged to a horizontal floor PR. The plank makes
an angle θ with the floor. Use this information to
Lower block of 3 light pulleys answer questions 43 to 45.
Q
Load 1020N X
The diagram above illustrates a block and tackle
system of pulleys which has a mechanical
advantage of 5. θ W
36. Determine the 𝑉. 𝑅 A.2 B.3 C.4 D.6. P R
42. As the end Q of the plank is raised, the
𝑉. 𝑅=number of strings in the pulley= 6 strings component of W normal to the plank will
(as the number of strings (6) is greater than A.increase B.decrease C.remain the same D.be
greater then the number of pulleys (5). slightly greater than W E.be independent of W.
37. Calculate the magnitude of the effort Component of the weight W normal to the plane
A.1020N B.340N C.204N D.170N . is Wcos θ. When the end Q of the plank is raised,
𝐿
𝑀. 𝐴=5, 𝐿=1020N, 𝐸=?, 𝑀. 𝐴 = , θ increases and cos θ deccreases thus, Wcos θ
𝐸
𝐸=
𝐿
=
1020
= 204N. decreases.
𝑀.𝐴 5 43. If θ is equal to 𝛼 at the time the box is just to
38. Calculate the efficiency of machine A.100.0%
slide down the plane, the coefficient of friction of
B.83.3% C.50.0% D.33.3%.
static friction between the plank and the box is
𝑀. 𝐴=5, 𝑉. 𝑅=6, 𝜖=?,
𝑀.𝐴 5 A.sin 𝛼 B.𝐭𝐚𝐧 𝜶 C.cos 𝛼 D.sec 𝛼 E.cot 𝛼.
Efficiency, 𝜖 = ×100= ×100=83.3%. When the box is about to slide down the plane,
𝑉.𝑅 6
39. A body is moved a distance 𝐿m up an inclined angle of fricton 𝛼 equals θ and the coefficient of
plane through vertical height of ℎm. Which of the friction 𝜇 = tan θ = tan 𝛼 .
following equtions is correct? 44. If the box is pulled from P to Q, which of the

A.efficiency= ×100 B.velocity ratio =
𝑳 following expresses the velocity ratio of the
𝐿
𝐿
𝒉
inclined plane? A.PR/PQ B.QR/PQ C.QR/PR
C.mechanical advantage = D.mechanical D.PQ/PR E.PQ/QR.

ℎ 𝑚𝑒𝑐ℎ𝑎𝑛𝑖𝑐𝑎𝑙 𝑎𝑑𝑣𝑎𝑛𝑡𝑎𝑔𝑒
advantage= E. =1. Velocity ratio is the ratio of the length of the
𝐿 𝑣𝑒𝑙𝑜𝑐𝑖𝑡𝑦 𝑟𝑎𝑡𝑖𝑜

84
Demystified Series Physics Demystified by Dr Timothy
inclined plane PQ to the height of the plane QR. 𝑉. 𝑅 =
2𝜋𝑟
, 𝑟=length or radius of effort handle or
𝑃
45. What is the velocity ratio of an inclined plane 𝑟
tommy bar, 𝑃=pitch. 𝑉. 𝑅 ∝ i.e velocity ratio
of length 2.6m, vertical height above the ground 𝑃
1.3m, and angle of inclination to the horizontal increases as the radius or length of effort arm or
30°? (sin 30° = 0.500, cos 30° = 0.866, tan 30° = tommy bar increases and decreases as the pitch
0.577) A.2.000 B.1.300 C.1.150 D.0.577. increases. Hence, using a jack with a smaller pitch
Length of the plane, 𝐿=2.6m, vertical height of the and increased length of the effort arm will
𝑊𝑒𝑖𝑔ℎ𝑡 𝑜𝑓 𝑡ℎ𝑒 𝑐𝑎𝑟
plane, ℎ=1.3m, θ=30°, 𝑉. 𝑅 = or
𝐿 1
, increase the 𝑉. 𝑅. 𝑀. 𝐴 = .
ℎ sin 𝜃 𝐸𝑓𝑓𝑜𝑟𝑡 𝑎𝑝𝑝𝑙𝑖𝑒𝑑

𝑉. 𝑅 = =
𝐿 2.6
=2. 𝑉. 𝑅 = = =
1
= 2.
1 1 𝑀. 𝐴 increases as the weight of the car or load
ℎ 1.3 sin 𝜃 sin 30° 0.5
increases and the effort decreases.
46. A plane inclined at angle θ has a velocity ratio
52. A spanner of length 20cm is used to turn a
of 10:1. The inclination of the plane to the
1 1 nut. If the spanner were replaced by another of
horizontal is given by A.tan 𝜃 = B.cot 𝜃 = length 15cm and held at its end with the same
10 10
C.cos 𝜃 =
1
D.𝐬𝐢𝐧 𝜽 =
𝟏
. effort as in the first case, the A.velocity ratio
10 𝟏𝟎
10 1 10 1 1 would increase B.the velocity ratio would
𝑉. 𝑅 =10:1= , 𝑉. 𝑅 = , = , sin 𝜃 = decrease C.mechanical advantage would
1 sin 𝜃 1 sin 𝜃 10
. increase but the velocity ratio would remain
48. From the diagram, find the constant D.mechanical advantage would
the mechanical advantage decrease but the velocity ratio would remain
20cm

of the system, if the plane constant E.mechanical advantage and velocity


30° is smooth ratio would remain constant.
A.0.50 B.0.86 C.1.15 D.2.00. A spanner works with the same principle as a
A smooth plane is a frictionless plane i.e a plane screw jack i.e screw system. 𝑉. 𝑅 =
2𝜋𝑟
,
with no friction. Hence, 𝑀. 𝐴 = 𝑉. 𝑅, 𝑃
𝐿 1 Using a spanner of length 20cm, 𝑟=20cm,
𝑀. 𝐴 = or , 𝐿=40cm, ℎ=20cm, θ=30°, 2𝜋×20 40𝜋

𝐿
sin 𝜃
40 1 1 1
𝑉. 𝑅 = = .
𝑃 𝑃
𝑀. 𝐴 = = = 2. 𝑀. 𝐴 = = = = 2. Using a spanner of length 15cm, 𝑟=15cm,
ℎ 20 sin 𝜃 sin 30° 0.5
49. A plane inclined at angke of 30° to the 𝑉. 𝑅 =
2𝜋×15
=
30𝜋
.
horizontal has an effieicny of 60%. A force of 𝑃 𝑃
40𝜋 30𝜋
magnitude 𝐹 is used to push a load of 15kg up the Hence, 𝑉. 𝑅 decreases as > OR 𝑉. 𝑅 ∝ 𝑟,
𝑃 𝑃
plane. Calculate 𝐹. A.45N B.75N C.125N D.360N velocity ratio increases when the radius or length
(g=10ms-2). of the screw increases and decreases when
𝐿=15kg=150N, 𝐸=𝐹, 𝜖=60%, θ=30°, radius or length of the screw decreases. Hence
𝑉. 𝑅 =
1
=
1
=
1
= 2, 𝑉. 𝑅(15cm)< 𝑉. 𝑅(20cm) i.e 𝑉. 𝑅 decreases. The
sin 𝜃 sin 30° 0.5
𝑀.𝐴 𝐿 1 mechanical advantage 𝑀. 𝐴 is constant, as the
Efficiency, 𝜖 = ×100= × ×100 effort is the same in both case and they act on the
𝑉.𝑅 𝐸 𝑉.𝑅
150 1 150×100
60 = × ×100 , 𝐹 = = 125N. same nut i.e the same load.
𝐹 2 2×60
50. The distance through which a screw moves 53. A screw jack with a Tommy bar of length
along its head when the screw head is turned 12cm is used to raise a car through a vertical
through 180° is A.the pitch B.half the pitch height of 25cm turning the Tommy bar through
C.one and half the pitch D.twice the pitch E.thrice 50 revolutions. Calculate the approximate
the pitch. velocity ratio of the jack [𝜋=3.14] A.453 B.302
The distance through which a screw moves along C.151 D.75.
its head when the head is turned through one Length of tommy bar, 𝑟=12cm, vertical height of
complete revolution i.e 360° is equal to the pitch load, ℎ=25cm, number of revolution, 𝑛=50, pitch,
ℎ 25
𝑃. Hence, the distance moved by the head of the 𝑃 = = = 0.5cm, 𝜋=3.14, 𝑉. 𝑅=?,
𝑛 50
screw when it rotates 180° i.e half of 360° is half 𝑉. 𝑅 =
2𝜋𝑟
=
2×3.14×12
=150.72=151.
the pitch. 𝑃 0.5

51. Which of these statements is correct abput a 54. Two spanners X and Y of lengths 15cm and
screw jack used in raising a car? A.velocityratio= 20cm respectively are used in turn to give a
𝑝𝑖𝑡ℎ 𝑜𝑓 𝑠𝑐𝑟𝑒𝑤 screw of pitch 2mm one complete rotation. If 𝑅 X
B.mechanicaladvantage= and 𝑅 Y are the respectively velocity ratios of the
2𝜋×𝑙𝑒𝑛𝑔𝑡ℎ 𝑜𝑓 𝑒𝑓𝑓𝑜𝑟𝑡 ℎ𝑎𝑛𝑑𝑙𝑒
𝑒𝑓𝑓𝑜𝑟𝑡 𝑎𝑝𝑝𝑙𝑖𝑒𝑑
C.velocity ratio can be increased spanners, what is the ratio 𝑅 X:𝑅 Y? A.2:3 B.3:2
𝑤𝑒𝑖𝑔ℎ𝑡 𝑜𝑓 𝑐𝑎𝑟
C.3:4 D.4:3
by using a jack with a smaller pitch D.velocity 𝑟X=15cm, 𝑟Y=20cm, 𝑃 X=𝑃 Y=2mm i.e they have the
ratio can be decreased by increasing the length of same pitch, 𝑉. 𝑅 X=𝑅 X, 𝑉. 𝑅Y=𝑅 Y,
the effort arm E.mechanical advantage can be 2𝜋𝑟 2𝜋
increased by increasing the effort. 𝑉. 𝑅 = , 𝑉. 𝑅 ∝ 𝑟 i.e = constant,
𝑃 𝑃
𝑉.𝑅𝑋 𝑉.𝑅𝑌 𝑅𝑋 𝑅𝑌 𝑅𝑋 15 3
Velocity ratio of a screw jack is given by – = , = , = = , 𝑅𝑋 :𝑅𝑌 =3:4.
𝑟𝑋 𝑟𝑌 15 20 𝑅𝑌 20 4

85
Demystified Series Physics Demystified by Dr Timothy
55. The handle of a screw jack of pitch 4mm turns done by load , 𝑊𝑜 = 𝑊𝑖 − 𝑊𝑓 = 𝑊𝑖 − 𝑊𝑖 = 𝑊𝑖 ,
1 4
5 5
through a circle of radius 21cm when the jack is 4
𝑊
22 𝑊𝑜 5 𝑖
used to raise a load. If 𝜋= , the velocity ratio of Efficiency, 𝜖 = × 100= ×100= 80% OR If
7 𝑊𝑖 𝑊𝑖
the jack is A.66 B.165 C.264 D.330 1
one-fifth(or of 100% i.e 20%) of the workdone
22 5
𝑃=4mm=0.4cm, 𝑟=21cm, 𝜋= , 𝑉. 𝑅=?, by the effort is expended in overcoming friction,
7
2𝜋𝑟 2×
22
7
×21 132 efficiency=80%
𝑉. 𝑅 = = = =330. 𝑅 50 𝑀.𝐴 𝐿 1
𝑃 0.4 0.4 𝑉. 𝑅 = = = 5, 𝜖 = ×100 = × ×100,
56. The pitch of a screw jack is 0.45cm and the 𝐿
𝑟 10
1
𝑉.𝑅
80×500×5
𝐸 𝑉.𝑅

arm is 60cm long. If the efficiency of the jack is 80 = × ×100, 𝐿 = = 2000N.


500 5 100
75
%. Calculate the mechanical advantage A.400 61. Gear wheels are used in machines for varying
𝜋
B.300 C.200 D.150 . I.mechanical advantage II.speed III.velocity ratio
75 A.I and II B.II and III C.I and III D.I,II and III
𝑃=0.45cm, 𝑟=60cm, 𝜖= % , 𝑀. 𝐴=?, E.none of the above.
𝜋
2𝜋𝑟 2𝜋×60 120𝜋 800𝜋
𝑉. 𝑅 = = = = , Gear wheels are used to change or vary speed,
𝑃 0.45 0.45 3
𝑀.𝐴 75 𝑀.𝐴 direction of motion(rotation) and velocity ratio.
Efficiency, 𝜖 = ×100, = ×100 ,
𝑉.𝑅 𝜋 800𝜋
3
They also increase or magnify force and transmit
𝑀. 𝐴 = ×
75
×
800𝜋
=200.
1 motion.
𝜋 3 100 62. A 20 toothed gear wheel drives a 50 toothed
57. In an ideal wheel and axle system, 𝑅 stands
one. If the angular speed of the smaller wheel is
for the radius of the wheel and 𝑟 is the radius of
120revs-1, the angular speed of the larger wheel
𝑹 𝑟 2
the axle. The mechanical advantage is A. B.( ) is A.3revs-1 B.40revs-1 C.360revs-1 D.2400revs-1.
𝒓 𝑅
𝑟 𝑅 2 Number of teeths in smaller(driving) gear 𝑁S=20,
C. D.( ) . number of teeths in larger(driven) gear 𝑁L=50,
𝑅 𝑟
An ideal machine is a mchine with 100% angular speed of smaller gear 𝜔S=120revs-1,
efficiency. Hence, in an ideal wheel and axle 𝜔
angular speed of larger gear 𝜔L=?, 𝑉. 𝑅 = 𝑠 = 𝐿
𝑁
𝑅 𝜔𝐿 𝑁𝑠
system, 𝑀. 𝐴 = 𝑉. 𝑅 = . 120 50 120×20
𝑟 = , 𝜔𝐿 = = 40revs-1.
58. The radius of a wheel is 30.0cm and that of its 𝜔𝐿 20 50

axle is 6.0cm. Calculate the effort required to lift 63. In a hydraulic press A.a small downward
a load of 120.0N using this machine, assuming force on the smaller piston produces a large
100% efficiency A.600.0N B.40.0N C.24.0N downward force on the larger piston B.a large
D.20.0N. upward force on the smaller piston produces a
Radius of wheel, 𝑅=30.0cm, radius of axle, large upward force on the larger piston C.a small
𝑟=6.0cm, 𝐿=120.0N, 𝐸=?, upward force on the smaller piston produces a
𝑀. 𝐴 = 𝑉. 𝑅,as the effieicny of the machine is small upward force on the larger pistonD.a small
𝐿 𝑅 𝐿 𝑅 downward force on the smaller piston
100%. 𝑀. 𝐴 = , 𝑉. 𝑅 = ∴ = ,
𝐸 𝑟 𝐸 𝑟 produces a large upward force on the larger
120 30 120×6
= , 𝐸= = 24.0N. piston E.a large downward force on the larger
𝐸 6 30
59. In a wheel and axle mechanism, the diameter piston produces a small upward force on the
of the wheel and axle are 40cm and 8cm larger piston.
respectively. Given that the machine is 80% In a hydraulic press, a small downward force
efficient, what effort is required to lift a load of called the effort, on the smaller piston produces
100N? A.20N B.25N C.50N D.80N. a large upward force called the load, on the larger
𝑅=diameter of the wheel=40cm, 𝑟=diameter of piston.
the axle=8cm, 𝜖=80%, 𝐿=100𝑁, 𝐸=?, 64. In a hydraulic press, a force of 40N is applied
𝑅 40
𝑉. 𝑅 = = = 5, Efficiency, 𝜖 =
𝑀.𝐴
×100 , to the smaller piston of area 10cm2. If the area of
𝐿
𝑟
1
8
100 1
𝑉.𝑅 the larger piston is 200cm2, calculate the force
𝜖= × ×100 , 80 = × × 100 , obtained A.800N B.500N C.80N D.50N.
𝐸 𝑉.𝑅 𝐸 5
𝐸=
100×100
= 25N. 𝐸=force applied in the smaller piston=40N,
80×5
𝐴1=area of the smaller piston=10cm2, 𝐿=force
60. The wheel and the axle of of a wheel-and- axle
obtained in the larger piston=?, 𝐴2=area of the
system have radii 50cm and 10cm respectively. If 𝐿 𝐴
one-fifth of the work done by the effort is used in larger piston=200cm2. 𝑀. 𝐴 = = 2 ,
𝐸 𝐴1
overcoming friction, an effort of 500N applied to 𝐿
=
𝐴2
,
𝐿
=
200
, 𝐿=
40×200
= 800N.
the wheel will raise a load of A.2500N B.2000N 𝐸 𝐴1 40 10 10

C.1000N D.500N. 65. A hydraulic press has a large circular piston


𝑅=50cm, 𝑟=10cm, 𝐸=500N, 𝐿=?, of radius 0.8m and a circular plunger of 0.2m. A
Workdone by effort=Work input 𝑊𝑖 , Work done force of 500𝑁 is exterted by the plunger. Find the
1 force exerted by the piston A.8000N B.4000N
against friction 𝑊𝑓 = 𝑊𝑖 , Work output or work
5 C.2000N D.31N.

86
Demystified Series Physics Demystified by Dr Timothy
Radius of piston=radius of larger section 𝑅=0.8m,
radius of plunger=radius of smaller section
𝑟=0.2m, force exerted by plunger= effort
𝐸=500N, force exerted by the piston =load 𝐿 =?,
𝐿 𝐴2 𝐿 𝜋𝑅 2
𝑀. 𝐴 = = , 𝐴2= 𝜋𝑅 2, 𝐴2= 𝜋𝑟2 , = ,
𝐸 𝐴1 𝐸 𝜋𝑟 2
𝐿 𝜋×0.82 500×0.64
= , 𝐿= = 8000N.
500 𝜋×0.22 0.04

Jamb past questions on simple machines :


[1979/2,1981/10,1982/14,1987/13,1988/8,13,
1994/13,1995/15,2001/8,2005/27,2006/15,
2015/36,2017/24,1983/36,1986/12,1989/9,
1992/9,1999/2,2000/10,2004/11,2007/46,200
8/14,2011/12,2015/8,1984/20,1991/8,1994/
12,1998/12,1999/4,2002/6,2014/15,2015/36,
1990/12,1993/9,2001/4,2009/12,2010/10,
2013/12,2014/14]

87
Demystified Series Physics Demystified by Dr Timothy

Chapter 9 – Equilibrium in liquids


[Density, Relative density and Upthrust]
● Density 𝝆 is the mass per unit volume of a - Upthrust on a body depends on the density
𝒎
substance. 𝝆 = . Its unit is Kgm-3 or gcm-3. of the fluid(liquid) 𝝆𝑳 and the volume of the
𝑽
- Relative density 𝑹. 𝑫 or specific gravity is the object or body immersed in the fluid or
ratio of the mass or weight of a given volume of a volume of the liquid displaced 𝑽𝒐 .
substance to the mass or weight of an equal - Upthrust is due to the difference in pressure
volume of water. It is also the ratio of the density between the upper and lower faces of the object,
of a substance to the density of water. with the pressure being greater at the lower face.
𝒎𝒂𝒔𝒔 𝒐𝒇 𝒂 𝒈𝒊𝒗𝒆𝒏 𝒗𝒐𝒍𝒖𝒎𝒆 𝒐𝒇 𝒂 𝒔𝒖𝒃𝒔𝒕𝒂𝒏𝒄𝒆 𝒎 Hence, upthrust increases with pressure, 𝑈 ∝ 𝑃.
𝑹. 𝑫 = = 𝒔,
𝒎𝒂𝒔𝒔 𝒐𝒇 𝒂𝒏 𝒆𝒒𝒖𝒂𝒍 𝒗𝒐𝒍𝒖𝒎𝒆 𝒐𝒇 𝒘𝒂𝒕𝒆𝒓 𝒎𝒘 - Upthrust is independent of temperature.
𝑹. 𝑫 =
𝒘𝒆𝒊𝒈𝒉𝒕 𝒐𝒇 𝒂 𝒈𝒊𝒗𝒆𝒏 𝒗𝒐𝒍𝒖𝒎𝒆 𝒐𝒇 𝒂 𝒔𝒖𝒃𝒔𝒕𝒂𝒏𝒄𝒆
=
𝑾𝒔
, - Archimede’s principle – It states that a body
𝒘𝒆𝒊𝒈𝒉𝒕 𝒐𝒇 𝒂𝒏 𝒆𝒒𝒖𝒂𝒍 𝒗𝒐𝒍𝒖𝒎𝒆 𝒐𝒇 𝒘𝒂𝒕𝒆𝒓 𝑾𝒘
𝒅𝒆𝒏𝒔𝒊𝒕𝒚 𝒐𝒇 𝒂 𝒔𝒖𝒃𝒔𝒕𝒂𝒏𝒄𝒆 𝝆𝒔 immersed(completely or partialy) in a fluid
𝑹. 𝑫 = = . experiences an upthrust(upward force) which is
𝒅𝒆𝒏𝒔𝒊𝒕𝒚 𝒐𝒇 𝒘𝒂𝒕𝒆𝒓 𝝆𝒘
- 𝑅. 𝐷is dimensionless or has no unit. equal to the weight of the fluid displaced.
- The density of water is 1gcm-3 or 1000kgm-3. - Real and Apparent weight – Real weight of a
- The density of a substance in gcm-3 is body is its weight in air. Apparent weight of a
numerically equal to its relative density e.g if the body is its weight when submerged or immersed
𝑅. 𝐷 of mercury is 13.6, its density is equal to in a fluid(liquid).
13.6gcm-3. 𝝆(gcm-3) = 𝑹. 𝑫. - An object tends to weighs less when in a
- The density of a substance in kgm-3 is equal to fluid(liquid) than in air, hence on immersion of a
its relative density times 1000 e.g 𝑅. 𝐷 of body attached to a string on water and
mercury is 13.6, its density is 13600kgm-3 or attempting to lift the body might break the string
13.6×103kgm-3. 𝝆(Kgm-3) = 𝑹. 𝑫 ×1000. as the weight in air (real weight) is greater than
- Density of a regular solid is determined by a its weight in water (apparent weight).
beam balance(mass) and measurement - A body appears to weigh lighter when
(volume). completely or partially immersed in water due to
- Density of an irregular solid is determined by a upthrust.
beam balance(mass) and volume of liquid - Upthrust in a liquid equal to the apparent loss
displaced (volume of the body). in weight of a body immersed in a fluid or the
- Density bottle is used for accurate weight of the fluid displaced.
measurement of the density/relative density Upthrust=Real weight – Apparent weight,
of a substance in the powder or granular real weight or weight in air = 𝑾𝑨 = 𝒎𝑨 𝒈,
form. apparent weight or weight of a body in fluid=
- If a liquid is heated i.e increase in 𝑾𝑨𝒑 = 𝒎𝑨𝒑 𝒈 , 𝑼 = 𝑾𝑨 − 𝑾𝑨𝒑 .
temperature its volume increase while the - Real weight or weight of the body in air,
density increases, while its mass remaining 𝑾𝑨 = 𝒎𝑨 𝒈 = 𝝆𝒐 𝑽𝒐 𝒈.
constant, then its density and volume before and - Apparent weight or weight of the body in a
after heating are related by – 𝝆1𝑽1= 𝝆2𝑽2 - If two fluid, 𝑾𝑨𝒑 = 𝒎𝑨𝒑 𝒈 = (𝝆𝒐 − 𝝆𝑳 )𝑽𝒐 𝒈.
substance have the same mass, their densities - Apparent mass , 𝒎𝑨𝒑 = (𝝆𝒐 − 𝝆𝑳 )𝑽𝒐 .
and volumes are related by – 𝝆1𝑽1= 𝝆2𝑽2 . - Bodies immersed or submerged in a fluid –
- The volume of a solid substance is determined The forces acting on a body immersed in a fluid
from its thickness or length and its area. (liquid) are : weight 𝑾 of the body acting
Volume=thickness or length×Area. downwards, upthrust or buoyancy force 𝑼
- If two substances of different densities are acting upwards. The magnitude of the two forces
mixed together, the density of the resulting determines how the body will behave in the fluid
𝒎
mixture is calculated as: 𝝆𝒎𝒊𝒙𝒕𝒖𝒓𝒆 = 𝒎𝒊𝒙𝒕𝒖𝒓𝒆, (liquid) :
𝑽𝒎𝒊𝒙𝒕𝒖𝒓𝒆
𝒎𝟏 +𝒎𝟐 1. If the weight is greater than the upthrust
𝝆𝒎𝒊𝒙𝒕𝒖𝒓𝒆 = , 𝒎𝟏 = 𝝆 𝟏 𝑽 𝟏 , 𝒎𝟐 = 𝝆 𝟐 𝑽 𝟐
𝑽𝟏 +𝑽𝟐 (𝑾 > 𝑼) or if density of object is greater than
OR 𝝆𝟏 𝑽𝟏 + 𝝆𝟐 𝑽𝟐 = 𝝆𝒎𝒊𝒙𝒕𝒖𝒓𝒆 𝑽𝒎𝒊𝒙𝒕𝒖𝒓𝒆 , 𝑽𝒎𝒊𝒙𝒕𝒖𝒓𝒆 = density of liquid (𝝆𝒐 > 𝝆𝑳 ), the body has a
𝒎 𝒎 𝒎 +𝒎
𝑽𝟏 + 𝑽𝟐 OR 𝟏 + 𝟐 = 𝟏 𝟐 . positive apparent weight and sinks in the liquid.
𝝆𝟏 𝝆𝟐 𝝆𝒎𝒊𝒙𝒕𝒖𝒓𝒆
● Upthrust and Archimede’s principle : 2. If a body is completely immersed and the
- Upthrust or buoyancy force or apparent loss weight is less than the upthrust (𝑾 < 𝑈) or
in weight 𝑼 is the upward force exerted by a (𝝆𝒐 < 𝝆𝑳 ), the body has a negative apparent
fluid on a body immersed completely or partially weight, hence rises and begins to float.
in the fluid. Upthrust 𝑼 = 𝝆𝑳 𝑽𝒐 𝒈. 3. If a body is partially immersed and the

88
Demystified Series Physics Demystified by Dr Timothy
weight of the body is equal to the upthrust fraction of the volume of the body immersed and
(𝑾 = 𝑼), the apparent weight of the body is zero decreasing the fraction of the volume above
and the body floats i.e principle of floatation of water.
floatation of a body. - Fraction of volume of an object submerged in a
- An addition force called tension 𝑻 also acts liquid is the ratio of the volume of the liquid
upward on a body immersed in a fluid if the body displaced to the volume of the object or the ratio
is suspended by a string or spring balance. of the density of the object to the density of the
𝑻 + 𝑼 = 𝑾 , 𝑻 = 𝑾 − 𝑼. liquid.
- For objects immersed completely in a Fraction of volume submerged =
liquidthe weight of object equals weight of 𝒗𝒐𝒍𝒖𝒎𝒆 𝒐𝒇 𝒍𝒊𝒒𝒖𝒊𝒅 𝒅𝒊𝒔𝒑𝒍𝒂𝒄𝒆𝒅
=
𝒅𝒆𝒏𝒔𝒊𝒕𝒚 𝒐𝒇 𝒐𝒃𝒋𝒆𝒄𝒕
,
fluid displaced and the volume of object 𝒗𝒐𝒍𝒖𝒎𝒆 𝒐𝒇 𝒐𝒃𝒋𝒆𝒄𝒕 𝒅𝒆𝒏𝒔𝒊𝒕𝒚 𝒐𝒇 𝒍𝒊𝒒𝒖𝒊𝒅
𝑽𝑳𝒅 𝝆𝒐
equals the volume of fluid displaced. Fraction submerged= = .
𝑽𝒐 𝝆𝑳
- The volume of the body immersed or volume of - Relative density of a substance based on
liquid displaced by the body decreases as the Archimede’s principle –
𝟏
density of the liquid increases [ 𝑽 ∝ ]. 1.Relative density of solids: A spring balance is
𝝆
- Volume of the body above liquid surface used to measure the weight of the solid in air or
increases as the volume of body immersed or true weight 𝑾𝑨 and its weight in water 𝑾𝒘 . The
volume of liquid displaced decreases. relative density of the solid is given by –
𝒘𝒆𝒊𝒈𝒉𝒕 𝒊𝒏 𝒂𝒊𝒓 𝑾 𝑾𝑨
- The weight of liquids displaced by a body 𝑹. 𝑫𝒔 = = 𝑨= .
𝒖𝒑𝒕𝒉𝒓𝒖𝒔𝒕 𝒊𝒏 𝒘𝒂𝒕𝒆𝒓 𝑼𝒘 𝑾𝑨 −𝑾𝒘
decreases as the relative density(specific gravity) - The relative density of the solid can also be
or density of the liquid increases. calculated from the extension produced by the
- Rise in volume of a liquid on complete or total weight in the spring balance, when in air 𝒆𝑨 and
immersion of an object in a fluid equals the 𝒆
water 𝒆𝒘 . 𝑹. 𝑫𝒔 = 𝑨 .
volume of the object immersed. 𝒆𝑨 −𝒆𝒘

- The volume of fluid displaced by a body on 2.Relative density of liquids – The weight of a
complete immersion is also equal to the volume solid as recorded by a spring balance in air 𝑾𝑨 ,
of the vessel. in water 𝑾𝒘 and in the liquid 𝑾𝑳 , whose reltive
- In complete immersion of bodies, upthrust is density is to be determined. The relative density
the same at all positions and depths within the of the liquid is given by –
𝒖𝒑𝒕𝒉𝒓𝒖𝒔𝒕 𝒊𝒏 𝒍𝒊𝒒𝒖𝒊𝒅 𝑼 𝑾 −𝑾
liquid as the volume of the liquid displaced at all 𝑹. 𝑫𝑳 = = 𝑳= 𝑨 𝑳.
𝒖𝒑𝒕𝒉𝒓𝒖𝒔𝒕 𝒊𝒏 𝒘𝒂𝒕𝒆𝒓 𝑼𝒘 𝑾𝑨 −𝑾𝒘
depths are the same. - The relative density of the liquid can also be
- In partial immersion of bodies, upthrust determined by the extension produced in air 𝒆𝑨 ,
varieswith the fraction of the body submerged i.e 𝒆 −𝒆
water 𝒆𝒘 and in the liquid 𝒆𝑳 . 𝑹. 𝑫𝒔 = 𝑨 𝑳 .
upthrust increasing with increase in the fraction 𝒆𝑨 −𝒆𝒘
submerged. ● Hydrometers – A hydrometer is used to
- Principle of floatation – It states that a floating measure the relative density of liquids e.g acids
body partially submerged will displace a in battery.
weight of the fluid equal to its own weight i.e a - It works on floatation principle.
body floats in a liquid when its weight is equal to - The weight of the hydrometer is equal to the
the upthrust acting on it by the liquid, 𝑾 = 𝑼. weight of the fluid displace by it i.e upthrust
- Factors affecting floatation of a body : on it when immersed in liquid or in water.
1.Density of the body – A body will float in a - The length of the test-tube submerged in a
fluid(liquid or gas) if the density of the body is liquid in a hydrometer is inversely proportional
𝟏
less than that of the liquid. to the density if the liquid i.e 𝒍 ∝ . Hence, the
𝝆
2.Shape of the body – A body denser than a higher the density of the liquid, the shorter the
liquid can still float if the body is shaped in a way length of the tube submerged or denser the
that its volume can displace its own weight of the liquid, lesser the hydrometer sinks and the lesser
liquid.The volume of the body will be so large the volume displaced.
compared to its weight. - The wide bulb of a hydrometer ensures that the
- A ship floats in water because its larger volume upthrust of any liquid is large enough to supports
displaces a large volume of water whose weight its weight.
balances the weight of the ship. Also a balloon - The narrow stem of a hydrometer ensures that
filled with a gas lighter than air will float if the the instrument is very sensitive to small changes
weight of the balloon and the and the gas in density.
contentequals the upthrust of air on the ballon. - The weighted end (lead shot) ensures that the
- When a body floats in a liquid, an increase in hydrometer floats upright and stable in the
temperature (by heating), will decrease the liquids.
density of the liquid, thereby increasing the - The density scale has unequal divisions and

89
Demystified Series Physics Demystified by Dr Timothy
decreases upwards. of petrol whose density is 7.2×102kgm-3 A.1.4m3
- Hydrometers are used to test the purity of liquid B.3.6m3 C.4.9m3 D.5.0m3 E.5.8m3.
whose density are known and the quality of milk. Volume of alcohol 𝑉𝑎 =?, density of alcohol,
- When a hydrometer is placed upright in a liquid, 𝜌𝑎 =8.4×102kgm-3, volume of petrol
it sinks freely to a certain depth 𝒉𝑳 in which it 𝑉𝑝 =4.2m3,density of petrol 𝜌𝑝 =7.2×102kgm-3. As
floats and the depth it sinks freely into water 𝒉𝒘 , they both have the same mass, 𝜌1𝑉 1= 𝜌2𝑉 2 , mass
the relative density of the liquid is given by – is constant, 𝜌𝑎 𝑉𝑎 = 𝜌𝑎 𝑉𝑎 ,
𝒅𝒆𝒑𝒕𝒉 𝒊𝒎𝒎𝒆𝒓𝒔𝒆𝒅 𝒊𝒏 𝒘𝒂𝒕𝒆𝒓 𝒉
𝑹. 𝑫𝑳 = = 𝒘. 8.4×102× 𝑉𝑎 = 7.2×102×4.2,
𝒅𝒆𝒑𝒕𝒉 𝒊𝒎𝒎𝒆𝒓𝒔𝒆𝒅 𝒊𝒏 𝒍𝒊𝒒𝒖𝒊𝒅 𝒉𝑳 7.2×102 ×4.2
- The relative density of the liquid can also be 𝑉𝑎 = = 3.6m3.
8.4×102
calculated as the ratio of the fraction of the 4. Colad oil of volume 600cm3 has a density of
volume submerged in water to the fraction 0.9gcm-3. When heated, the density of oil
submerged in liquid. decreases to 0.6gcm-3. The volume of the heated
𝒇𝒓𝒂𝒄𝒕𝒊𝒐𝒏 𝒐𝒇 𝒗𝒐𝒍𝒖𝒎𝒆 𝒔𝒖𝒃𝒎𝒆𝒓𝒈𝒆𝒅 𝒊𝒏 𝒘𝒂𝒕𝒆𝒓 𝑽
𝑹. 𝑫𝑳 = = 𝒘. oil is A.324cm3 B.400cm3 C.720cm3D.900cm3
𝒇𝒓𝒂𝒄𝒕𝒊𝒐𝒏 𝒐𝒇 𝒗𝒐𝒍𝒖𝒎𝒆 𝒔𝒖𝒃𝒎𝒆𝒓𝒈𝒆𝒅 𝒊𝒏 𝒍𝒊𝒒𝒖𝒊𝒅 𝑽𝑳
Increase in temperature i.e by heating, decreases
- Fraction of height or volume immersed in
the density of a liquid and increases the volume
water+Fraction of height or volume above
with the mass being constant.
water=1.
𝜌 1𝑉 1= 𝜌 2𝑉 2 , 𝜌1=0.9gcm-3, 𝑉 1=600cm3,
- A hydrometer reading is independent of
𝜌2=0.6gcm , 𝑉 2=?, 0.9×600 = 0.6× 𝑉 2 ,
-3
temperature.Hence, a hydrometer submerged in 0.9×600
water and cooling the water will not result in any 𝑉 2= = 900cm3.
0.6
rise or fall below the initial level as the volume 5. The relative density of gold,silver,copper and
change of the liquid compensate the expansivity zinc are given as 19.3, 10.5, 8.9 and 7.1
change of the tube. respectively. A piece of ornamental metal weighs
0.445kg and displaces 5×10-5m3 when
Examples : completely immersed in water. The metal is
1. The density of a solid is defined as the A.mass A.zinc B.copper C.silver D.gold .
of the solid compared to the mass of an equal Weight or mass of the ornament 𝑚=0.445kg ,
volume of water B.amount of water displaced volume of water displaced when totally
when a unit mass of solid is immersed in it immersed equals the volume of the ornament,
𝑚 0.445
C.weight per unit volume of the solid D.volume 𝑉=5×10-5m3, 𝜌 = = −5 = 8900kgm ,
-3
𝑉 5×10
per unit mass of solid E.mass per unit volume 𝑑𝑒𝑛𝑠𝑖𝑡𝑦 𝑜𝑓 𝑚𝑒𝑡𝑎𝑙 𝜌𝑚
of the solid . 𝑅. 𝐷 of metal = = ,
𝑑𝑒𝑛𝑠𝑖𝑡𝑦 𝑜𝑓 𝑤𝑎𝑡𝑒𝑟 𝜌𝑤
Density of substance is the mass per unit volume Density of water=1000kgm-3 or 1gcm-3 ,
of the substance. Option A defines relative 𝑅. 𝐷 =
8900
=8.9 i.e the metal is copper.
1000
density. 6. A tank of capacity 27m3 initially filled with
2. Density is a measure of the A.quantity of water is drained at the rate of 5kgs-1. If the
matter an object contains B.space an object density of water is 103kgm-3, the time it takes to
occupies C.compactness of matter with empty the tank is A.13.5min B.54min C.90min
respect to its size D.surface area of an object. D.135min .
3. What is the mass of 1.025 litres of water? 𝑚
𝑉=27m3, mass rate flow =5kgs-1, 𝜌=103kgm-3,
A.1025kg B.102.5kg C.10.25kg D.1.025kg 𝑡
𝑚 𝜌×𝑉 103 ×27
E.0.1025kg. mass rate flow = = ,5= ,
𝑡 𝑡 𝑡
Density of water 𝜌 =1gcm-3, volume of water 103 ×27 5400
𝑉=1.025litre=1.025(1000)cm3=1025cm3, 𝑡= = 5400s = = 90min
5 60
𝑚
𝜌 = , 𝑚 = 𝜌𝑉 =1×1025=1025g=1.025kg. 7. 1000 identical drops of oil of density 500kgm-
𝑉 3 have a total mass of 5×10-4kg. One of the drop
4. A rectangle metal block of density
2.5×103kgm-3 measures 10cm by 8cm by 5cm. form a thin film of area 0.5m2 on water. The
Calculate its mass A.4kg B.1kg C.0.4kg D.0.16kg thickness of the film is A.2×10-8m B.2×10-9m
C.2×10-7m D.3×10-9m
E.0.004kg.
Density of the block 𝜌=2.5×103kgm-3, dimension Density of oil 𝜌=500kgm-3, mass of 1000 drops of
of 10cm by 8cm by 5cm means, length oil=5×10-4kg, mass of one drop of oil 𝑚 =
5×10−4
𝐿=10cm=0.1m, breadth 𝑏=8cm=0.08m, height =5×10-7kg, Area of the film 𝐴=0.5m2,
1000
ℎ=5cm=0.05m, volume of the metal block= 𝐿𝑏ℎ , thickness of the film 𝑡ℎ=?,
volume of the metal block= volume=Area×thickness, 𝑉 = 𝐴 × 𝑡ℎ ,
0.1×0.08×0.05=0.0004m3=4×10-4m3, 𝑚 𝑚 5×10−7 5×10−7
𝜌= = , 500= , 𝑡ℎ = ,
𝑚 = 𝜌𝑉 =2.5×103×4×10-4=1kg. 𝑉 𝐴×𝑡ℎ 0.5×𝑡ℎ 0.5×500
5. What volume of alcohol with a density of 𝑡ℎ = 2×10 m .
-9 3

8.4×102kgm-3 will have the same mass as 4.2m3 8. Alcohol of mass 21g and density 0.7gcm-3 is
mixed with 10g of water. Determine the density

90
Demystified Series Physics Demystified by Dr Timothy
of the mixture [Density of water= 1.0gcm-3] 𝑅. 𝐷𝑄 =
𝑚𝑎𝑠𝑠 𝑜𝑓 𝑎 𝑔𝑖𝑣𝑒𝑛 𝑣𝑜𝑙𝑢𝑚𝑒 𝑜𝑓 𝑙𝑖𝑞𝑢𝑖𝑑 𝑄
=
𝑚𝑄
,
A.0.775gcm-3 B.0.780gcm-3 C.0.875gcm-3 𝑚𝑎𝑠𝑠 𝑜𝑓 𝑎𝑛 𝑒𝑞𝑢𝑎𝑙 𝑣𝑜𝑙𝑢𝑚𝑒 𝑜𝑓 𝑤𝑎𝑡𝑒𝑟
24
𝑚𝑤

D.0.880gcm .-3 𝑅. 𝐷𝑄 = = 0.8. 𝜌𝑄 = 0.8gcm-3 or 800kgm-3.


30
𝑚1 =21g, 𝜌1 =0.7gcm-3, 𝑚2 =10g, 𝜌2 =1gcm-3, 12. The mass of a specific gravity bottle is 15.2g
𝑚 𝑚 +𝑚
𝜌𝑚𝑖𝑥𝑡𝑢𝑟𝑒 = 𝑚𝑖𝑥𝑡𝑢𝑟𝑒 , 𝜌𝑚𝑖𝑥𝑡𝑢𝑟𝑒 = 1 2 , 𝑚1 =𝜌1 𝑉1 , when empty. It is 24.8g when filled with kerosene
𝑉𝑚𝑖𝑥𝑡𝑢𝑟𝑒 𝑉1 +𝑉2
𝑚 21 𝑚2 10 and 27.2g when filled wit distilled water.
𝑚2 =𝜌2 𝑉2 , 𝑉1 = 1 = = 30cm3, 𝑉2 = = , Calculate the relative density of kerosene A.1.25
𝜌1 0.7 𝜌2 1
𝑉2 = 10cm3 , 𝜌𝑚𝑖𝑥𝑡𝑢𝑟𝑒 =
21+10
=
31
= 0.775gcm-3 B.1.10 C.0.90 D.0.80.
𝑚1 𝑚2 𝑚1 +𝑚2
30+10
21 10
40
21+10 Specific gravity bottle is same as a relative
OR + = , + = , density bottle.
𝜌1 𝜌2 𝜌𝑚𝑖𝑥𝑡𝑢𝑟𝑒 0.7 1 𝜌𝑚𝑖𝑥𝑡𝑢𝑟𝑒
30+10 =
31
, 40 =
31
, Mass of empty bottle=15.2g, Mass of bottle when
𝜌𝑚𝑖𝑥𝑡𝑢𝑟𝑒
31
𝜌𝑚𝑖𝑥𝑡𝑢𝑟𝑒 filled with kerosene=24.8g, Mass of kerosene
𝜌𝑚𝑖𝑥𝑡𝑢𝑟𝑒 = = 0.775gcm-3. only 𝑚𝑘 =24.8−15.2=9.6g, Mass of bottle when
40
9. A liquid of volume 2.00m3 and density filled with water=27.2g, Mass of water only
1.00×103kgm-3 is mixed with 3.00m3 of another 𝑚𝑤 = 27.2−15.2=12.0g.
𝑚𝑎𝑠𝑠 𝑜𝑓 𝑘𝑒𝑟𝑜𝑠𝑒𝑛𝑒 𝑚 9.6
liquid of density 8.00×103kgm-3. Calculate the 𝑅. 𝐷𝑘𝑒𝑟𝑜𝑠𝑒𝑛𝑒 = = 𝑘= = 0.8.
𝑚𝑎𝑠𝑠 𝑜𝑓 𝑤𝑎𝑡𝑒𝑟 𝑚𝑤 12.0
density of the mixture (Assuming there is no
13. An empty relative density bottle has a mass
chemical reaction) A.5.20×102kgm-3
of 30g. When filled with paraffin, its mass is 70g.
B.8.80×102kgm-3 C.1.13×103kgm-3
Calculate the mass of the bottle when it is filled
D.5.20×103kgm-3.
with water. [Relative density of paraffin=0.8]
𝑉1 =2.00m3, 𝜌1 =1.00×103kgm-3, 𝑉2 =3.00m3,
𝑚 A.20g B.40g C.60g D.80g.
𝜌2 =8.00×103kgm-3, 𝜌𝑚𝑖𝑥𝑡𝑢𝑟𝑒 = 𝑚𝑖𝑥𝑡𝑢𝑟𝑒 , Mass of empty bottle=30g, Mass of bottle when
𝑉𝑚𝑖𝑥𝑡𝑢𝑟𝑒
𝜌𝑚𝑖𝑥𝑡𝑢𝑟𝑒 =
𝑚1 +𝑚2
, 𝑚1 =𝜌1 𝑉1 , 𝑚2 =𝜌2 𝑉2 , filled with paraffin=70g, Mass of paraffin
𝑉1 +𝑉2
only=70−30=40g, relative density of
𝑚1 = 𝜌1 𝑉1 =1.00×103×2.00=2.00×103kg,
paraffin=0.8,
𝑚2 = 𝜌2 𝑉2 =8.00×103×3.00=24.00×103kg, 𝑚𝑎𝑠𝑠 𝑜𝑓 𝑝𝑎𝑟𝑎𝑓𝑓𝑖𝑛 𝑚 40
2×103 +24×103 26×103 𝑅. 𝐷𝑝𝑎𝑟𝑎𝑓𝑓𝑖𝑛 = = 𝑝 , 0.8 = ,
𝜌𝑚𝑖𝑥𝑡𝑢𝑟𝑒 = = =5.2×103kgm-3 40
𝑚𝑎𝑠𝑠 𝑜𝑓 𝑤𝑎𝑡𝑒𝑟 𝑚𝑤 𝑚𝑤
2+3 5
OR 𝜌1 𝑉1 + 𝜌2 𝑉2 = 𝜌𝑚𝑖𝑥𝑡𝑢𝑟𝑒 (𝑉1 + 𝑉2 ) , 𝑚𝑤 = = 50g , Mass of water only=50g, Mass
0.8
𝜌1 𝑉1 +𝜌2 𝑉2 1×103 ×2+8×103 ×3 of bottle when filled with water=50+30=80g.
𝜌𝑚𝑖𝑥𝑡𝑢𝑟𝑒 = = ,
𝑉1 +𝑉2 2+3 14. When a body is completely immersed in a
2×103 +24×103
𝜌𝑚𝑖𝑥𝑡𝑢𝑟𝑒 = = 5.2×103kgm-3. liquid, it displaces an amount of the liquid that is
5
10. In an experiment to determine the relative equal to the I.volume of the body II.weight of the
density of cork, the following recordings were body III.mass of the body. Which of the above
made I.weight of sunker in water=𝑥, II.weight of is/are correct? A.I B.II C.III D.I and II E.I and III.
sinker in water and cork in air=𝑦, III.weight of When a body is completely immersed in a liquid,
both sinker and cork in water=𝑥. Which of the its displaces volume and weight of the liquid
following fractions gives the relative density of displaced are equal to its volume and weight
𝑦 𝑥 𝒚−𝒙 𝑧 𝑧−𝑦 respectively.
the cork? A. B. C. D. E.
𝑧−𝑥 𝑦−𝑧 𝒚−𝒛 𝑥−𝑦 𝑥−𝑧 15. An empty 60 litre petrol tank has a mass of
Weight of sinker in water= 𝑥, Weight of sinker in 10kg. Its mass when full of relative density 0.72
water and cork in air= 𝑦, is A.7.2kg B.33.2kg C.43.2kg D.53.2kg.
Weight of the cork in air 𝑊𝐴 = 𝑦 − 𝑥, Weight of mass of empty petrol tank=10kg, volume of
both sinker and cork in water= 𝑧, Weight of an petrol when full=volume of the tank=60litre=
equal volume of water= 𝑦 − 𝑧. 60(1000)cm3=60000cm3, relative density of the
𝑤𝑒𝑖𝑔ℎ𝑡 𝑜𝑓 𝑎 𝑔𝑖𝑣𝑒𝑛 𝑣𝑜𝑙𝑢𝑚𝑒 𝑜𝑓 𝑐𝑜𝑟𝑘 𝑦−𝑥
𝑅. 𝐷𝑐𝑜𝑟𝑘 = = . petrol 𝑅. 𝐷=0.72, density of the petrol
𝑤𝑒𝑖𝑔ℎ𝑡 𝑜𝑓 𝑎𝑛 𝑒𝑞𝑢𝑎𝑙 𝑣𝑜𝑙𝑢𝑚𝑒 𝑜𝑓 𝑤𝑎𝑡𝑒𝑟 𝑦−𝑧
𝜌=0.72gcm-3, mass of the petrol only, 𝑚 = 𝜌𝑉 ,
11. In an experiment using the relative density
𝑚 =0.72×60000 = 43200g=43.2kg. Mass of the
bottle, the following results were recorded:
petrol tank when full=mass of the empty
Mass of empty bottle+stopper=50g
tank+mass of the petrol=10+43.2=53.2kg.
Mass of bottle full of water+stopper=80g
16. Why does a a piece of cork float on water?
Mass of bottle full of liquid 𝑄 +stopper=74g
A.there is no upthrust on it when completely
Use these results to calculate the relative density
immersed in water B.the upthrust on it when
of liquid 𝑄 A.0.60 B.0.80 C.1.25 D.0.93 E.1.60
completely immersed in water is greater than
Mass of empty bottle 𝑚𝑏 + stopper 𝑚𝑠 =50g,
its weight in air C.upthrust on it when
Mass of bottle full of water (𝑚𝑏 + 𝑚𝑤 ) + stopper
completely immersed water is less than its
𝑚𝑠 = 80g, Mass of water 𝑚𝑤 = 80−50=30g ,
weight D.its specific gravity is greater than that
Mass of bottle full of liquid 𝑄 (𝑚𝑏 + 𝑚𝑄 ) +
of water E.its specific gravity is same as that of
stopper 𝑚𝑠 = 74g, Mass of liquid 𝑄 𝑚𝑄 =
water.
74−50=24g.

91
Demystified Series Physics Demystified by Dr Timothy
A body completely immersed in a liquid will float 21. The forces acting on a body which is
when the upthrust on it is greater than its weight suspended freely in still air from a support are
in air or when its density or specific gravity A.gravitational and tension only B.frictional and
(relative density) is greater than that of water. gravitational force only C.gravitational, tension
Option C is the condition necessary for a body to and fluid thrust only D.fluid thrust and tension
sink. only E.frictional, gravitational and tension only.
17. A piece of wood is floating on water. The The forces acting on a body suspended i.e by a
forces acting on the wood are A.upthrust and string, in still air are gravitational forces (weight)
reaction B.weight and reaction C.weight and tension(in the string) and fluid thrust (upthrust
upthrust D.upthrust and viscosity E.weight and in air).
viscosity. 22. The upthrust experienced by an object in a
The forces acting on a body floating on a liquid liquid depends on the I.density of the liquid
(water) are weight and upthrust. A body partially II.volume of the object III.volume of the liquid
immersed in a liquid will float when it weight displaced A.I B.I and II C.I and III D.II and III E.I,II
balances or equals the upthrust acting on it. and III.
18. A solid suspended by a piece of string is Upthrust of a liquid on an object, 𝑈 = 𝜌𝐿 𝑉𝑜 𝑔.
completely immersed in water. On attempt to lift Hence, upthrust depends on the density of the
the solid out of the water, the string breaks when liquid 𝜌𝐿 , the volume of the object 𝑉𝑜 or volume of
the solid is partly out of the water, this is because the liquid displaced by the body.
A.the tension in string decreases as the solid if 23. A block of ice floats with nine-tenth of its
lifted B.the mass of the solid is increased C.the volume submerged in water. In order to find its
solid apparently weighs less when completely relative density, one also need to know A.total
immersed in water than when partially volume of the ice block B.the actual volume of the
immersed D.part of the solid still in water is ice under water C.the weight of the ice block
exerting more force on the string E.the solid is no D.the density of the ice block E.nothing else.
𝜌
longer in equilibrium with water. 𝑅. 𝐷𝑖𝑐𝑒 = 𝑖𝑐𝑒 , the density of water is already
𝜌𝑤
A solid appears to weigh less when immersed
known, hence inorder to determine the relative
completely in liquids than when partially
density of ice we also need to know the density of
immersed or in air due to the upthrust exerted by
the ice block.
the liquid on the body.Hence, when the body is
24. A uniform solid cube material 10cm on each
partly out of water its weight exceeds the tension
side of mass 700g is submerged in water. Which
in the string and the string breaks.
of the following best described the behavior of
19. A balloon filled with hydrogen rises when
the cube in water? A.the cube will rest at an
released into the air because A.the upthrust of
equilibrium position in water B.The cube will
the balloon is less than the weight of the balloon
melt after a period of time C.The cube will sink in
B.the upthrust on the balloon is greater than the
water D.The cube will float.
weight of the balloon C.the upthrust on the
Length of the cube 𝐿=10cm, Volume of the cube
balloon equal to the weight of the balloon
𝑉 = 𝐿3=103=1000cm3, mass of the cube
D.the density of the balloon and hydrogen is 𝑚 700
greater than the density in air E.the pressure 𝑚=700g, density of the cube 𝜌𝑐𝑢𝑏𝑒 = = ,
𝑉 1000
below the balloon is greater than that above the density of the cube 𝜌𝑐𝑢𝑏𝑒 = 0.7gcm , density of
-3

balloon. the liquid(water) 𝜌𝑤𝑎𝑡𝑒𝑟 =1gcm-3. Since the


The ballon rises and floats in air because the density of the solid cube submerged in water is
upthrust of air on the balloon equal its weight , less than the density of water (𝜌𝑐𝑢𝑏𝑒 < 𝜌𝑤𝑎𝑡𝑒𝑟 ),
𝑈 = 𝑊. the solid cube will have a negative apparent
20. A 300g rubber balloon containing 200g of weight and will rise and begin to float in the
hydrogen moves vertically upwards with a water.
constant velocity. Neglecting air resistance, - The solid cube will sink if its density is
calculate the magnitude of the force pulling the higher that of water.
balloon upwards (g=10ms-2) A.0.5N B.1.0N 25. A body weighing 14𝑁 in air is partially
C.2.0N D.3.0N E.5.0N. immersed in water. If the mass of the water
Mass of the balloon=300g, mass of the hydrogen displaced in the process is 200g, calculate the
gas in the balloon=200g, Total mass of the upthrust on the body (g=10ms-2) A.2.0𝑵 B.3.0𝑁
balloon=mass of the balloon + mass of the gas C.3.5𝑁 D.7.0𝑁
content= 300+200=500g=0.5kg In partial or complete immersion of a body in a
The force pulling the ballon upward is the fluid(liquid), the weight of the fluid(liquid)
upthrust or fluid thrust and its equal to the displaced 𝑊𝑓𝑑 by the body is equal to the
weight of the ballon, as the balloon floats in air. upthrust 𝑈. Mass of water displaced
𝑈 = 𝑊 = 𝑚𝑔 = 0.5×10 = 5.0N. 𝑚𝑓𝑑 =200g=0.2kg, g=10ms-2, upthrust is equal to

92
Demystified Series Physics Demystified by Dr Timothy
the weight of the water displaced. 𝜌𝐿 =
2
=4000kgm-3=4.0×103kgm-3 .
500×10−6
𝑈 = 𝑊𝑓𝑑 = 𝑚𝑓𝑑 𝑔 = 0.2×10 = 2.0N.
32. A piece of iron weighs 250𝑁 in air and 200𝑁
26. A body of volume 0.046m3 is immersed in a in a liquid of density 1000kgm-3. The volume of
3
liquid of density 980kgm-3 with of its volume the iron is A.2.0×10-3m3 B.2.5×10-3m3
4
submerged. Calculate the upthrust on the body C.4.5×10 m D.5.0×10 m .
-3 3 -3 3

(g-10ms-2) A.11.27N B.33.81N C.112.70N 𝑊𝐴 =250𝑁, 𝑊𝐴𝑝 =200𝑁, 𝜌𝐿 =1000kgm-3, g=10ms-2,


D.338.0N. 𝑉𝑜 =?, 𝑈 = 𝜌𝐿 𝑉𝑜 𝑔 = 𝑊𝐴 − 𝑊𝐴𝑝 ,
Volume of body 𝑉𝑜 =0.046m3, density of liquid 1000× 𝑉𝑜 ×10 = 250−200 = 50 , 𝑉𝑜 =
50
,
𝜌𝐿 =980kgm-3, volume submerged=volume of 1000×10
3 𝑉𝑜 = 5.0×10-3m3.
liquid displaced 𝑉𝐿𝑑 = 𝑉𝑜 , upthrust 𝑈=?, 33. An object weighs 22kg in water and 30kg in
4
3 3
𝑈 = 𝜌𝐿 𝑉𝐿𝑑 𝑔 = 𝜌𝐿 𝑉𝑜 𝑔 = 980× ×0.046×10 air. What is the upthrust exerted by the liquid on
4 4
𝑈 = 338.10N. the object? A.520N B.220N C.80N D.20N
27. A solid has a weight of 8.0N in air and 2.0N in (g=10ms-2).
a certain liquid. Calculate the mass of the liquid 𝑚𝐴𝑝 =22kg, 𝑚𝐴 =30kg, g=10ms-2, 𝑈=?,
displaced by the solid (g=10ms-2) A.10g B.40g 𝑊𝐴 = 𝑚𝐴 𝑔, 𝑊𝐴𝑝 = 𝑚𝐴𝑝 𝑔, 𝑈 = 𝑊𝐴 − 𝑊𝐴𝑝 ,
C.600g D.1000g. 𝑈 = 𝑚𝐴 𝑔 − 𝑚𝐴𝑝 𝑔 = 𝑔(𝑚𝐴 − 𝑚𝐴𝑝 ),
Weight in air 𝑊𝐴 =8.0N, Weight in liquid or 𝑈 =10(30−22) = 80𝑁.
apparent weight 𝑊𝐴𝑝 =2.0N, 34. A test tube of radius 1.0cm is loaded to 8.8g,
Upthrust=Real weight−Apparent weight, if it is placed upright in water, find the depth to
𝑈 = 𝑊𝐴 − 𝑊𝐴𝑝 . Upthrust is equal to the weight of which it would sink A.2.8cm B.5.2cm C.25.5cm
the fluid displaced by the body, 𝑈 = 𝑊𝑓𝑑 = 𝑚𝑓𝑑 𝑔, D.28.0cm (g=10ms-2, density of water=1000kgm-
3 or 1gcm-3).
𝑚𝑓𝑑 𝑔 = 𝑊𝐴 − 𝑊𝐴𝑝 , 𝑚𝑓𝑑 × 10=8−2, 𝑚𝑓𝑑 × 10=6,
6 𝑚=8.8g, 𝑟=1.0cm, 𝜌=1gcm-3, g=10ms-2, ℎ=?,
𝑚𝑓𝑑 = = 0.6kg=600g. Upthrust on the test tube=Weight of water
10
28. An object of volume 1m3 and mass 2kg is displaced. 𝑈 = 𝑊, 𝑈 = 𝜌𝑉𝑔 = 𝜌(𝐴ℎ)𝑔, 𝑊 = 𝑚𝑔,
totally immersed in a fluid of density 1kgm-3. 𝜌(𝐴ℎ)𝑔 = 𝑚𝑔, 𝜌(𝐴ℎ) = 𝑚, 𝐴 = 𝜋𝑟 2 , 𝜋 = ,
22
Calculate its apparent weight A.20N B.10N C.2N 22 22 7×8.8
7

D.1N. 1( ×12× ℎ)=8.8, ℎ = 8.8, ℎ = = 2.8cm.


7 7 22
𝑚 2
𝑉𝑜 =1m3, 𝑚𝑜 =2kg, 𝜌𝑜 = 𝑜 = = 2kgm-3, 35. If a spherical metal bob of radius 3cm is fully
𝑉𝑜 1
immersed in a cylinder containing water and the
𝜌𝐿 =1kgm-3, 𝑊𝐴𝑝 = (𝜌𝑜 − 𝜌𝐿 )𝑉𝑜 𝑔,
water level rises by 1cm, what is the radius of the
𝑊𝐴𝑝 = (2−1)×1×10 = 10N. cylinder? A.12cm B.6cm C.3cm D.1cm.
29. A metal object of relative density 9.0 and Radius of the spherical bob 𝑟=3cm, height of
volume 2×10-4m3 is completely immersed in water rise ℎ=1cm, radius of the cylinder 𝑅=?,
water. Its apparent weight is A.18N B.16N C.4.5N Rise in volume of a liquid on complete or total
D.2.0N. immersion of an object in a fluid equals the
𝑅. 𝐷𝑜 =9.0, 𝜌𝑜 =9000kgm-3, 𝑉𝑜 =2×10-4m3, volume of the object immersed.The volume of
𝜌𝐿 =density of water=1000kgm-3 , fluid displaced by a body on complete immersion
𝑊𝐴𝑝 = (𝜌𝑜 − 𝜌𝐿 )𝑉𝑜 𝑔 , is also equal to the volume of the vessel. Thus,
𝑊𝐴𝑝 = (9000−1000)×2×10-4×10 = 16N. volume of the spherical bob displaced=volume of
30. If the relative density of a metal is 19. What the liquid displaced.
will be the mass of 20cm3 of the metal when 4
𝜋𝑟 3 = 𝜋𝑅2 ℎ ,
4 3 4
𝑟 = 𝑅2 ℎ , × 33 = 𝑅2 × 1,
immersed in water? A.400g B.380g C.360g 3
4
3 3

D.180g E.39g. 𝑅2 = ×27=36, 𝑅 = √36 = 6cm.


3
𝑅. 𝐷𝑜 =19, 𝜌𝑜 =19gcm-3, 𝑉𝑜 =20cm3, 𝜌𝐿 =1gcm-3, 36. A piece of wood of mass 40g and uniform
mass when immersed in water=apparent mass, cross-sectional area of 2cm2 floats upright in
𝑚𝐴𝑝 = (𝜌𝑜 − 𝜌𝐿 )𝑉𝑜 = (19−1)×20 = 360g. water of density 1gcm-3. The length of the wood
31. A body whose mass is 2kg and has a volume immersed is A.80cm B.40cm C.20cm D.2cm
of 500cm3 just floats when completely immersed Weight of the wood displaced=upthrust of water.
in a liquid. Calculate the density of the liquid 𝑊 = 𝑈, 𝑈 = 𝜌𝑉𝑔 = 𝜌(𝐴ℎ)𝑔, 𝑊 = 𝑚𝑔 , 𝑚 =
A.4.0×102kgm-3 B.4.0×103kgm-3 C.1.0×103kgm- 𝜌(𝐴ℎ), 𝑚=40g, 𝐴=2cm2, 𝜌=1gcm-3, length of the
3 D.1.0×106kgm-3. wood immersed ℎ=?,
40
𝑚𝑜 =2kg, 𝑉𝑜 =500cm3=500×10-6m3, 𝜌𝐿 =?, 40=1(2× ℎ), ℎ = = 20cm.
2
Upthrust=weight of the fluid displaced by it. The 37. A metal cube of volume 103mm3 is lowered
body is completely immersed hence, the weight into a measuring cylinder containing water. If the
of the fluid equal to the weight of the body. internal cross-sectional area of the cylinder is
𝑈 = 𝑊, 𝑈 = 𝜌𝐿 𝑉𝑜 𝑔 , 𝑊 = 𝑚𝑜 𝑔 , 𝜌𝐿 𝑉𝑜 𝑔 = 𝑚𝑜 𝑔 , 1.5×102mm2, by how much does the water level
𝜌𝐿 𝑉𝑜 = 𝑚𝑜 , 𝜌𝐿 ×500×10-6 = 2 ,

93
Demystified Series Physics Demystified by Dr Timothy
rise in the cylinder? A.6.67×100mm of density 900kgm-3. Determine the reading of
B.8.50×10 mm C.1.15×10 mm D.2.50×103mm
3 3 the spring balance (g=10ms-2) A.0.18N B.0.57N
E.1.50×105mm. C.0.75N D.0.93N.
Volume of the metal cube 𝑉=103mm3, internal 𝑉𝑜 =3×10-5m3, 𝜌𝑜 =2.5×103kgm-3=2500kgm-3,
cross-sectional area of the cylinder 2
volume of liquid displaced 𝑉𝐿𝑑 = 𝑉𝑜 , g=10ms-2,
3
𝐴=1.5×102mm2, height of water rise ℎ=?, volume
𝜌𝐿 =900kgm-3, reading of the spring balance is the
of the metal cube= volume of water displaced( i.e
apparent weight or weight of the body in the
the volume of the cylinder),
103
liquid. 𝑊𝐴𝑝 = 𝑊𝐴 − 𝑈 = 𝜌𝑜 𝑉𝑜 𝑔 − 𝜌𝐿 𝑉𝐿𝑑 𝑔 ,
𝑉 = 𝐴ℎ , 103=1.5×102× ℎ, ℎ = , 2 2
1.5×102 𝑊𝐴𝑝 = 𝜌𝑜 𝑉𝑜 𝑔 − 𝜌𝐿 𝑉𝑜 𝑔 = (𝜌𝑜 − 𝜌𝐿 ) 𝑉𝑜 𝑔 ,
3 3
ℎ = 6.67mm = 6.67×10 mm. 0
2
38. When a ship sails from salt water into fresh 𝑊𝐴𝑝 = (2500− ×900)×3×10-5×10 ,
3
water, the fraction of its water above the water 𝑊𝐴𝑝 = (2500−600) ×3×10-5×10 = 0.57N.
surface will A.increase B.decrease C.remain the 42. An object of mass 400g and density 600kgm-
same D.increase then decreases. 3 is suspended with a string so that half of it is

Sea or salt water has a higher density i.e denser, immersed in paraffin of density 900kgm-3.The
than fresh water. Volume of a body submerged tension in the string is A.1.0N B.3.0N C.4.0N
(or immersed) or volume of the liquid displaced D.5.0N.
increases as the density of the liquid decreases 𝑈 + 𝑇 = 𝑊, 𝑇 = 𝑊 − 𝑈, 𝑇=tension
1
𝑉 ∝ . The volume of the body above the liquid Mass of the object 𝑚𝑜 =400g=0.4kg,
𝜌
Weight of the object 𝑊 = 𝑚𝑔 =0.4×10=4N
increases as the volume of the body submerged
Upthrust 𝑈 =weight of the fluid displaced,
or volume of liquid displaced decreases. Hence,
𝑈 = 𝜌𝐿 𝑉𝐿𝑑 𝑔, 𝜌𝑜 =600kgm-3, Volume of object
when a ship sails from salt water to fresh water 𝑚 0.4 3
i.e decrease in density, the volume of the ship 𝑉𝑜 = 𝑜 = m , Volume of liquid displaced
𝜌𝑜 600
submerged increases and the volume above the 𝑉𝐿𝑑 =
𝑉𝑜
, 𝑈 = 𝜌𝐿 𝑉𝐿𝑑 𝑔 = 𝜌𝐿
𝑉𝑜
𝑔,
2 2
liquid decreases. 1 0.4
39. A solid object floats in liquid 𝑋 of relative 𝑈 = 900× × ×10 = 3N.
2 600
density 0.9 but sinks in liquid 𝑌 of relative 𝑇 = 𝑊 − 𝑈 = 4−3 = 1N. Alternatively,
1 0.4
density 0.7. It can be inferred that the A.volume 𝑻 = (𝒎𝒐 − 𝝆𝑳 𝑽𝑳𝒅 )𝒈 = (0.4−900× × )10
2 600
of 𝑋 displaced is less than that of 𝑌 B.volume of 𝑋 𝑇 = (0.4−0.3)10 = 1N.
displaced is greater than that of 𝑌 C.weight of 𝑋 43. A solid weighs 45N and 15N respectively in
displaced is less than that of 𝑌 D.weight of 𝑋 air and water. Determine the relative density of
displaced is greater than that of 𝑌. the solid A.0.33 B.0.50 C.1.50 D.3.00.
The upthrust in liquid 𝑋 or the weight of liquid 𝑋 Weight in air 𝑊𝐴 =45𝑁, Weight in water 𝑊𝑤 =15𝑁,
displaced by the solid is equal to its weight while relative density of the solid, 𝑅. 𝐷𝑠 =?
the upthrust in liquid 𝑌 or the weight of liquid 𝑌 𝑤𝑒𝑖𝑔ℎ𝑡 𝑖𝑛 𝑎𝑖𝑟 𝑊 𝑊𝐴
𝑅. 𝐷𝑠 = = 𝐴= ,
displaced by the solid is less than its weight. 𝑢𝑝𝑡ℎ𝑟𝑢𝑠𝑡 𝑖𝑛 𝑤𝑎𝑡𝑒𝑟 𝑈𝑤 𝑊𝐴 −𝑊𝑤
45 45
Hence, upthrust or weight of liquid 𝑋 displaced is 𝑅. 𝐷𝑠 = = = 1.50.
45−15 30
greater than the upthrust or weight of liquid 𝑌 44. A piece of metal of relative density 5.0 weighs
displaced. 60N in air. Calculate its weight when fully
40. A piece of wood floats inside water at room immersed in water A.4N B.5N C.48N D.60N.
temperature with a fraction of it above the liquid 𝑅. 𝐷𝑠 =5.0, 𝑊𝐴 =45N, 𝑊𝑤 =?,
surface. As the temperature of the water is raised, 𝑤𝑒𝑖𝑔ℎ𝑡 𝑖𝑛 𝑎𝑖𝑟 𝑊 𝑊𝐴
the temperature of the water is raised, the part if 𝑅. 𝐷𝑠 = = 𝐴= ,
𝑢𝑝𝑡ℎ𝑟𝑢𝑠𝑡 𝑖𝑛 𝑤𝑎𝑡𝑒𝑟 𝑈𝑤 𝑊𝐴 −𝑊𝑤
60 60
the wood above the liquid will A.Decrease 5.0 = , 60−𝑊𝑤 = = 12,
60−𝑊𝑤 5
because the density of water decreases with
𝑊𝑤 = 60−12 = 48N.
temperature B.Increases because the density of
45. A spiral spring loaded with a piece of metal,
water decreases with temperature C.Decreases
extends by 10.5cm in air. When the metal is fully
because the density of water increases with
submerged in water, the metal extends by 6.8cm.
temperature D.Increases because the density of
Calculate the relative density of the metal
water increases with temperature .
(assume Hooke’s law is obeyed) A.8.52 B.5.68
The density of water decreases with increase in
C.2.84 D.1.42.
temperature, hence the body sinks further and
Extension in air 𝑒𝐴 =10.5cm, Extension in water
the fraction of the body above the water surface
𝑒𝑤 =6.8cm, 𝑅. 𝐷𝑠 =?, Based on Hooke’s law – 𝐹(or
decreases.
𝑊) ∝ 𝑒 , hence extension can be used in place of
41. A block of volume 3×10-5m3 and density
force(or weight).
2.5×103kgm-3 is suspended from a spring 𝑒𝑥𝑡𝑒𝑛𝑠𝑖𝑜𝑛 𝑖𝑛 𝑎𝑖𝑟 𝑒 𝑒
2 𝑅. 𝐷𝑠 = = 𝐴 = 𝐴 ,
balance with of its volume immersed in a liquid 𝑢𝑝𝑡ℎ𝑟𝑢𝑠𝑡 𝑖𝑛 𝑤𝑎𝑡𝑒𝑟 𝑈𝑤 𝑒𝐴 −𝑒𝑤
3

94
Demystified Series Physics Demystified by Dr Timothy
𝑅. 𝐷𝑠 =
10.5
=
10.5
= 2.84. 𝑉𝑜 , volume of the liquid displaced 𝑉𝐿𝑑 . When the
10.5−6.8 3.7
cork floats, the weight of the cork equals the
46. An object weighs 60.0N in air, 48.2N in a
weight of the liquid displaced, 𝑊𝑜 = 𝑊𝐿𝑑 ,
certain liquid 𝑋 and 44.9N in water. Calculate the
𝜌𝑜 𝑉𝑜 = 𝜌𝐿 𝑉𝐿𝑑 , Fraction of volume submerged =
relative density of 𝑋 A.3.300 B.1.279 C.0.932 𝑉𝐿𝑑 𝜌𝑜 0.25×103 1
D.0.781. = = = = 0.2.
𝑉𝑜 𝜌𝐿 1.25×103 5
Weight in air 𝑊𝐴 =60.0N, Weight in liquid 51. A wooden object whose density is 0.06gcm-3
𝑋𝑊𝑋 =48.2N, Weight in water 𝑊𝑤 =44.9N, was dropped into water contained in a tall
relative density of the liquid 𝑋𝑅. 𝐷𝑋 =?, beaker. How much of the wooden object is in the
𝑢𝑝𝑡ℎ𝑟𝑢𝑠𝑡 𝑖𝑛 𝑙𝑖𝑞𝑢𝑖𝑑 𝑋 𝑈 𝑊 −𝑊
𝑅. 𝐷𝑋 = = 𝑋 = 𝐴 𝑋, water? A.One-quarter B.One-sixth C.Two-thirds
𝑢𝑝𝑡ℎ𝑟𝑢𝑠𝑡 𝑖𝑛 𝑤𝑎𝑡𝑒𝑟 𝑈𝑊 𝑊𝐴 −𝑊𝑤
60.0−48.2 11.8 D.A small fraction .
𝑅. 𝐷𝑋 = = = 0.781.
60.0−44.9 15.1 Density of the wooden object 𝜌𝑜 =0.06gcm-3,
47. A solid of weight 0.600N is totally immersed density of the liquid(water) 𝜌𝐿 =1gcm-3, volume of
in oil and water respectively. If the upthrust in oil the wooden object 𝑉𝑜 , volume of the
is 0.210N and the relative density of oil is 0.875. liquid(water) displaced 𝑉𝐿𝑑 . When the wooden
Find the upthrust in water A.0.600N B.0.360N block floats in water, the weight of the block
C.0.240N D.0.180N . equals the weight of the water displaced. 𝑊𝑜 =
𝑊𝐴 =0.600𝑁, Upthrust in oil 𝑈𝑜 =0.210N, relative 𝑊𝐿𝑑 , 𝜌𝑜 𝑉𝑜 = 𝜌𝐿 𝑉𝐿𝑑 ,
density oil 𝑅. 𝐷𝑜 =0.875, Upthrust in water 𝑈𝑤 =?, 𝑉
Fraction of volume submerged= 𝐿𝑑 = 𝑜 ,
𝜌
𝑢𝑝𝑡ℎ𝑟𝑢𝑠𝑡 𝑖𝑛 𝑜𝑖𝑙 𝑈 0.210 𝑉𝑜 𝜌𝐿
𝑅. 𝐷𝑜 = = 𝑜 , 0.875 = , 0.06
𝑢𝑝𝑡ℎ𝑟𝑢𝑠𝑡 𝑖𝑛 𝑤𝑎𝑡𝑒𝑟
0.210
𝑈𝑤 𝑈𝑤
Fraction of wooden block submerged= ,
1
𝑈𝑤 = = 0.240. 3
0.875 Fraction of wooden block submerged= , a
48. A copper cube weighs 0.25N in air, 0.17N 50
when completely immersed in paraffin oil and small fraction of the volume.
0.15N when completely immersed in water. The 52. A hydrometer is used for measuring the
ratio of upthrust in oil to upthrust in water is A.depth of water in a vessel B.relative density of
A.4:5 B.3:5 C.13:5 D.7:10 a liquid by method of floatation C.relative
Weight in air 𝑊𝐴 =0.25N, Weight in paraffin oil density of a liquid by finding the apparent weight
𝑊𝑜 =0.17N, Weight in water 𝑊𝑤 =0.15N, the ratio D.relative humididty in air.
of the upthrust in paraffin oil to upthrust in water Hydrometers are used for measuring relative
is equal to the relative density of paraffin oil. density. Its operation is based on principle of
𝑢𝑝𝑡ℎ𝑟𝑢𝑠𝑡 𝑖𝑛 𝑜𝑖𝑙 𝑈 𝑊 −𝑊 floatation.
𝑅. 𝐷𝑜 = = 𝑜= 𝐴 𝑜, 53.The relative density of the acid in a car battery
𝑢𝑝𝑡ℎ𝑟𝑢𝑠𝑡 𝑖𝑛 𝑤𝑎𝑡𝑒𝑟 𝑈𝑤 𝑊𝐴 −𝑊𝑤
𝑅. 𝐷𝑜 =
0.25−0.17
=
0.08 4
= = 4:5. can be measured with a A.hygrometer
0.25−0.15 0.1 5
49. A solid weighs 4.8g in air, 2.8g in water and B.manometer C.hydrometer D.barometer.
3.2g in kerosene. The ratio of density of the solid 54. Which of the following statements is/are
3 5 correct about the common hydrometer? I.It has
to that of kerosene is A.12 B.3 C.2 D. E. a long narrow stem II.The relative density values
2 4
Weight in air 𝑊𝐴 =4.8g, Weight in water 𝑊𝑤 =2.8g, marked on the stem increases down the the stem
Weight in kerosene 𝑊𝑘 =3.2g, relative density of III.It has a wide bulb IV.The bulb is loaded to
the solid 𝑅. 𝐷𝑠 , make the hydrometer float vertically A.II B.II and
𝑤𝑒𝑖𝑔ℎ𝑡 𝑖𝑛 𝑎𝑖𝑟 𝑊 𝑊𝐴
𝑅. 𝐷𝑠 = = 𝐴= , IV C.I,II and III D.II and IV E.I,II,III and IV.
𝑢𝑝𝑡ℎ𝑟𝑢𝑠𝑡 𝑖𝑛 𝑤𝑎𝑡𝑒𝑟 𝑈𝑤 𝑊𝐴 −𝑊𝑤
4.8 4.8 55. A loaded tube of uniform cross-sectional area
𝑅. 𝐷𝑠 = = = 2.4, density of the solid 4.0cm2 floats upright in water with 15.5cm of its
4.8−2.8 2
𝜌𝑠 =2.4gcm or 2400kgm-3
-3
length immersed and then in liquid 𝐿 with
relative density of kerosene 𝑅. 𝐷𝑘 , 18.0cm of its length immersed, if the density of
𝑢𝑝𝑡ℎ𝑟𝑢𝑠𝑡 𝑖𝑛 𝑘𝑒𝑟𝑜𝑠𝑒𝑛𝑒 𝑈 𝑊 −𝑊
𝑅. 𝐷𝑘 = = 𝑘 = 𝐴 𝑘 , water is 1.0gcm-3 is A.25.0g B.47.2g C.34.8g
𝑢𝑝𝑡ℎ𝑟𝑢𝑠𝑡 𝑖𝑛 𝑤𝑎𝑡𝑒𝑟 𝑈𝑊 𝑊𝐴 −𝑊𝑤
4.8−3.2 1.6 D.62.0g.
𝑅. 𝐷𝑘 = = = 0.8, density of kerosene Cross-sectional area of the loaded tube 𝐴=4.0cm2,
4.8−2.8 2
𝜌𝑘 =0.8gcm or 800kgm-3.
-3
depth of immersion in water ℎ𝑤 =15.5cm, depth
Ratio of the density of the solid to that of of immersion in liquid 𝐿ℎ𝐿 =18.0cm, 𝜌𝑤 =1.0gcm-3,
𝜌 2.4
kerosene= 𝑠 = = 3 or 3:1. The loaded test-tube as stated in the question is
𝜌𝑘 0.8
a hydrometer. In a hydrometer, by principle of
50. A piece of cork floats in a liquid. What fraction
loatation the weight of the tube is equal to the
of its volume will be immersed in the liquid
weight of the water displaced by it. Weight of the
(density of the cork=0.25×103kgm-3, density of
tube=Weight of water displaced, 𝑊 = 𝑊𝐿𝑑 ,
the liquid=1.25×10-3kgm-3) A.0.1 B.0.2 C.0.5
𝑚𝑔 = 𝜌𝑤 𝑉𝐿𝑑 𝑔, 𝑚 = 𝜌𝑤 𝑉𝐿𝑑 , volume of liquid
D.0.8.
displaced 𝑉𝐿𝑑 = 𝐴ℎ𝑤 , mass of the tube 𝑚=?,
Density of the cork 𝜌𝑜 =0.25×103kgm-3, density of
𝑚 = 𝜌𝑤 𝐴ℎ𝑤 =1×4×15.5 = 62.0g.
the liquid 𝜌𝐿 =1.25×10-3kgm-3, volume of the cork

95
Demystified Series Physics Demystified by Dr Timothy
- The relative density of the liquid 𝐿 can be gotten 60. A body of mass 25g floats in water of density
ℎ 15.5 1
by – 𝑅. 𝐷𝐿 = 𝑤 = = 0.86. Hence, the density 1.0×103kgm-3 with of its volume submerged.
ℎ𝐿 18.0 4
of 𝐿 is 0.86gcm-3 or 860kgm-3. Calculate the volume of the body above the water
56. A hydrometer of mass 3.60kg and volume surface A.75cm3 B.60cm3 C.50cm3 D.25cm3
1
6.00×10-5m3 floats in a liquid with of its volume E.6.25cm3
5 𝑚𝑜 =25g, 𝜌𝑤 =1.0×103kgm-3=1gcm-3, fraction of
above the liquid. Calculate the density of the volume submerged in water or volume of the
liquid A.4.50×105kgm-3 B.3.00×105kgm-3 1
C.1.50×105kgm-3 D.7.50×104kgm-3. liquid displaced 𝑉𝐿𝑑 = 𝑉𝑜 ,
4
Based on the principle of floatation, Floatation principle – Weight of the body =
Weight of hydrometer=Upthrust of the liquid (or Weight of the water displaced by it,
1 1
Weight of the liquid displaced by it). 𝑚 = 𝜌𝐿 𝑉𝐿𝑑 , 𝑚 = 𝜌𝑤 𝑉𝐿𝑑 = 𝜌𝑤 𝑉𝑜 , 25=1× 𝑉𝑜 ,
4 4
mass of hydrometer 𝑚=3.60kg, Volume of the 𝑉𝑜 =4×25=100cm3. Volume of body above
1
hydrometer=6.00×10-5m3, if of the volume of 3 3
water surface= 𝑉𝑜 = ×100=75cm3.
5 4 4
4
the hydrometer is above the liquid, then of its 61. A reactangular block of wood floats in water
5
volume is immersed. Volume of the liquid with two third of its volume immersed. When
displaced 𝑉𝐿𝑑 =
4
of the volume of the placed in another liquid it floats with half its
5
4 volume immersed. Calculate the relative density
hydrometer. 3.60= 𝜌𝐿 × ×6.00×10-5, of the liquid A.1.50 B.1.33 C.1.00 D.0.83 E.0.66.
5
3.60×5 2
𝜌𝐿 = =7.50×104kgm-3. Fraction of volume submered in water 𝑉𝑤 = ,
4×6.00×10−5 3
1
57. A hydrometer is placed in water and 6.4cm of Fraction of volume submerged in liquid 𝑉𝐿 = ,
2
its stem is immersed. What is the depth of relative density of the liquid 𝑅. 𝐷𝐿 =?,
immersion when it is placed in a liquid of relative 𝑓𝑟𝑎𝑐𝑡𝑖𝑜𝑛 𝑜𝑓 𝑣𝑜𝑙𝑢𝑚𝑒 𝑠𝑢𝑏𝑚𝑒𝑟𝑔𝑒𝑑 𝑖𝑛 𝑤𝑎𝑡𝑒𝑟
density 0.8 A.60cm B.70cm C.8cm D.9cm 𝑅. 𝐷𝐿 = ,
𝑓𝑟𝑎𝑐𝑡𝑖𝑜𝑛 𝑜𝑓 𝑣𝑜𝑙𝑢𝑚𝑒 𝑠𝑢𝑏𝑚𝑒𝑟𝑔𝑒𝑑 𝑖𝑛 𝑙𝑖𝑞𝑢𝑖𝑑
(density of water=1gcm-3). 𝑅. 𝐷𝐿 =
𝑉𝑤
=
2⁄3 2
= × 2 = 1.33.
1⁄2
Depth of immersion in water ℎ𝑤 =6.4cm, relative 𝑉𝐿 3

density of the liquid 𝑅. 𝐷𝐿 =0.8, depth of


immersion in liquid ℎ𝐿 =?, Jamb past questions on Eqilibrium in fluids
ℎ 6.4 6.4 (density, relative density and upthrust) :
𝑅. 𝐷𝐿 = 𝑤 , 0.8= , ℎ𝐿 = = 8cm. [1978/14,1979/1,1980/29,30,1981/50,1982/1
ℎ𝐿 ℎ𝐿 0.8
58. A plastic sphere floats in water with 50% of 5,20,45,1983/8,28,31,1984/4,1985/16,17,1986
its volume submerged. If it floats in glycerine /3,1987/15,1988/14,1989/12,1990/14,1991/
with 40% of its volume submerged, the density of 12,1992/12,1994/18,19,1995/20,1997/16,19,
the glycerine is A.1400kgm-3B.1250kgm-3 1998/16,1999/15,2000/4,2001/5,2002/10,
D.1000kgm C.500kgm-3 (density of water
-3 2003/3,2004/34,2005/46,2006/41,46,2007/34
=1000kgm-3). ,2008/18,2009/17,2010/17,2011/15,2012/16,
Fraction of volume submerged in water 𝑉𝑤 = 50% 18,2013/8,15,2015/13,27].
50
= =0.5, Fraction of volume submerged in
100
40
glycerine 𝑉𝑔 =40%= =0.4, 𝜌𝑔 =?,
100
𝑓𝑟𝑎𝑐𝑡𝑖𝑜𝑛 𝑜𝑓 𝑣𝑜𝑙𝑢𝑚𝑒 𝑠𝑢𝑏𝑚𝑒𝑟𝑔𝑒𝑑 𝑖𝑛 𝑤𝑎𝑡𝑒𝑟
𝑅. 𝐷𝐿 = ,
𝑓𝑟𝑎𝑐𝑡𝑖𝑜𝑛 𝑜𝑓 𝑣𝑜𝑙𝑢𝑚𝑒 𝑠𝑢𝑏𝑚𝑒𝑟𝑔𝑒𝑑 𝑖𝑛 𝑙𝑖𝑞𝑢𝑖𝑑
𝜌𝑔 𝑉 𝜌𝑔 0.5 0.5×1000
𝑅. 𝐷𝐿 = = 𝑤, = , 𝜌𝑔 = ,
𝜌 𝑤 𝑉 1000 𝐿 0.4 0.4
𝜌𝑔 =1250kgm-3 .
59. A uniform cylindrical block of wood floats in
water with one third of its height above the water
level. In a liquid of relative density 0.8, what
fraction of its height will be above the liquid
𝟏 1 1 4 5
level? A. B. C. D. E. .
𝟔 5 3 5 6
1
Height above water= ℎ , Height submerged in
3
2
water ℎ𝑤 = ℎ , Height above the liquid =𝑥, Height
3
submerged in liquid ℎ𝐿 =(1−𝑥)ℎ, relative density
of the liquid 𝑅. 𝐷𝐿 =0.8,
2 2
ℎ𝑤 ℎ
𝑅. 𝐷𝐿 = , 0.8= (1−𝑥)ℎ
3
= 3
,
ℎ𝐿 1−𝑥
2 1 2 10 5 5 1
1−𝑥 = × = × = , 𝑥 =1− = .
3 0.8 3 8 6 6 6

96
Demystified Series Physics Demystified by Dr Timothy

Chapter 10 – Pressure in solids and liquids,


Gas laws and Kinetic theory of matter
● Pressure in solids – 3.Bleeding of the nose at high altitude(height)
- Pressure 𝑷 is the normal or perpendicular 4.Maintaining the height or altitudes of
force 𝐹 per unit area 𝐴 acting on a surface. aeroplanes high above sea level.
𝑭
𝑷 = , 𝑭 = 𝑾 = 𝒎𝒈, 𝑨 = 𝝅𝒓𝟐 . - Pressure at the surface of any liquid is only
𝑨 atmospheric pressure i.e 𝑷atm or 𝑯mmHg or
- Pressure in solids depends on the force exerted 𝑯mmH2O.
and area of the body. - Pressure at a point ℎ above the atmospheric end
- Pressure on a body increases with an increase or above sea level –
in force acting on it i.e 𝑷 ∝ 𝑭. 𝑷 = (𝑷atm−𝝆𝒈𝒉)𝑵m-2 or (𝑯 − 𝒉)mmHg or
- Pressure decreases with an increase in area, mmH2O.
𝟏
𝑷 ∝ . Hence, a man standing on one toe of his - Pressure at a point below atmospheric end or
𝑨
foot (small area) exertes a larger pressure, below sea level or at the base of a liquid –
cutting objects with sharp knifes (small area) 𝑷 = (𝑷atm+𝝆𝒈𝒉)𝑵m-2 or (𝑯 + 𝒉)mmHg or
exerts a great pressure. mmH2O.
- Solid pressure is exerted downwars only. - The pressure exerted at the base of a liquid
- The units for pressure and their relationship: surface exposed to an enclosed air column is –
i.1Pascal(Pa)=1𝑁m-2ii.1Bar=105𝑁m-2 =105Pa. 𝑷 = Pressure due to air column+𝝆𝒈𝒉.
iii.1mmHg = 1Torr iv.760mmHg = - The pressure change in air of density 𝜌𝑎𝑖𝑟 and
1Atmospher(Atm). in a liquid of density 𝜌𝐿 or between different
liquids is given by – 𝑷𝒂𝒊𝒓 = 𝑷𝑳 , 𝝆𝒂𝒊𝒓 𝒈𝑯 = 𝝆𝑳 𝒈𝒉.
● Pressure in liquids or Hydrostatic - Measurment of atmospheric pressure –
Atmospheric pressure is measured by a
pressure – The pressure exerted in a liquid is
barometer. Types of barometer –
given by : 𝑷 = 𝝆𝒈𝒉. 𝜌=density of the liquid,
1.Simple or Torricellian barometer – Thick
𝑔=acceleration due to gravity, ℎ=depth below
walled glass tube filled with mercury and
liquid surface or height of liquid column.
inverted over a dish of mercury. A vacuum exist
- Pressure in liquids depends on density (relative
above the mercury level in the tube.The
density or specific gravity) of the liquid, depth or
atmospheric pressure supports a mercury
height of liquid column and acceleration due to
column of 760mmHg or 76cmHg.
gravity. It is independent of the shape of the
2.Fortin barometer– It is a simple barometer
container or total volume of liquid and cross-
where a leather bag is used as a reservoir of
sectional area or diameter of the vessel.
mercury instead of a mercury dish. It has an ivory
- Characteristics of liquid pressure –
pointer or index at the zero mark to indicate the
1.Pressure at all points at the same level within a
level of mercury and locate the zero mark of the
liquid is the same.
barometer scale.
2.Pressure in a liquid is exerted equally in all
3.Aneroid barometer – It has no barometric
directions at any point in a liquid.
fluid and measures slight variations of
3.Pressure in liquids icreases with depth of the
atmospheric pressure through a partially
liquid.
evacuated and sealed metal can. It can be used as
4.Pressure at the same level in different liquids
an altimeter to measure height or altitude) and
increases with their density.
in weather forcasts i.e high pressure indicates
● Atmospheric pressure – The atmosphere dry weather while low pressure indicates rainy
exerts some pressure on the earth surface and weather.
objects on the earth surface. This is also the - In a fautless barometer, the space above the
pressure exerted by air on objects. barometric liquid is a complete vacuum and
- Atmospheric pressure decreases with when tilted the vertical height of the mercury
increasing altitude (height) above sea level and column remains constant i.e 76cm 0r 760mm.
increases with increasing depth below sea level. - In a faulty barometer, the space above the
- Evidence of atmospheric pressure i.e barometric liquid is filled with air bubble and
experiments to show that the atmosphere when tilted the vertical height of the mercury
(air) exerts pressure – column falls below 76cm 0r 760mm.
1.Crushing of a can when the air in it is evacuated. - When a barometer is used as as an altimeter in
2.Cardboard paper maintaining its position to an determining the height(altitude) of mountains or
inverted beaker filled with water. aeroplanes 𝐻𝑃 above sea level, the relation used

97
Demystified Series Physics Demystified by Dr Timothy
𝝆𝑯𝒈
is – 𝑯𝑷 = (𝑯𝒐 − 𝑯𝟏 ). 𝐻𝑃 =height of mountain 8.Pressure cooker – It is an enclosed air-tight
𝝆𝒂𝒊𝒓
pot used to cook food at high temperature as
or aeroplane, 𝜌𝐻𝑔 =density of mercury,
pressure inside the cooker is increased above
𝜌𝑎𝑖𝑟 =density of air, 𝐻𝑜 =barometric height at sea atmospheric pressure. Water in a pressure
level (larger height) or pressure at sea level, cooker boils at a high temperature greater
𝐻1 = barometric height above sea level at 𝐻𝑃 than its normal boiling point at sea level i.e
(smaller height) or pressure above sea level. 100℃. Pressure cooker cooks food faster,
- Mercury is a better barometric liquid than thereby saving time and fuel.
water because its density is several times - Measurement of gas pressure – Gas pressure
greater than that of water, it needs no is measured by a manometer.
colouring, has a low vapour pressure i.e does - The pressure of a gas in a manometer –
not vaporize easily, and does not wet glass. 1.If the arm exposed to the atmosphere is higher
- The reading of pressure on a mercury than or at the same level the arm exposed to the
barometer depends on atmospheric pressure, gas, the gas pressure is given by –
density of mercury,acceleration due to gravity 𝑷 = 𝑷𝒂𝒕𝒎 + 𝝆𝒈𝒉 or 𝑯 + 𝒉.
and temperature of mercury.It is independent of 2.If the arm exposed to the atmosphere is lower
the cross-sectional area of the tube. than the arm exposed to the gas, the gas pressure
- Applications of atmospheric pressure – is given by – 𝑷 = 𝑷𝒂𝒕𝒎 − 𝝆𝒈𝒉 or
1.Hare’s apparatus and a U-tube – For 𝑯 − 𝒉. 𝐻=Height of liquid column due to
comparing the densities of two immiscible atmospheric pressure, ℎ=the difference in the
liquids or determining the relative density of a liquid level between the two arms of the
liquid. If a liquid 𝐿 of density 𝜌𝐿 and water of manometer.
density 𝜌𝑤 are introduced into a U-tube or Hare’s - If pressure is required in cm or mm of H2O and
apparatus, the relative density of the liquid 𝐿 is the atmospeheric pressure is in cm or mm of Hg.
𝝆 𝒉
given by – 𝑹. 𝑫𝑳 = 𝑳 = 𝒘 . ℎ𝐿 =height of liquid The atmospheric pressure should be converted
𝝆𝒘 𝒉𝑳
column or height of liquid atmospheric pressure to cm or mm of H2O using the equation –
supports, ℎ𝑤 =height of water column or height of 𝝆𝑯𝒈 𝒉𝑯𝒈 = 𝝆𝑯𝟐𝑶 𝒉𝑯𝟐𝑶 .
water atmospheric pressure supports. - Mercury is used for measuring high pressure
2.Siphon – This is a deviced used to remove while water is used measuring low pressure
liquids from receptacles or vessels which cannot due to their density.
be easily lifted or reached or emptied - Transmission of pressure – Pascal’s
converniently. For a siphon to function principle of transmission of pressure states
efficiently, the tube must be filled by the liquid, that the pressure applied to a fluid which is
the recipient vessel must be lower than the confined in a container will be transmitted
original vessel and the height of liquid column equally to all parts of the container.
must be less than the barometric height of the - Pascals principle is applied in hydraulic press
liquid. for lifting heavy loads, motor car-foot brakes or
3.Lift pumpor common pump – It is used to lift hydraulic brakes is slowing down or stopping
water from wells or lifting kerosene from a tall moving automobiles or cars and hydraulic jacks
tank. It raises water to a height less than 10m for lifting cars.
because atmospheric pressure cannot support - Based on Pascal’s principle, the pressure
water column above 10m high. exerted at a smaller piston of area 𝐴1 is equal to
4.Force pump – It raises water to a height that exerted in the larger piston 𝐴1 . Hence,
𝑭𝟏 𝑭
greater than 10m.It has an air chamber which 𝑷𝟏 = 𝑷𝟐 = 𝑷 , = 𝟐.
𝑨𝟏 𝑨𝟐
ensures a steady flow of water.
- In both a lift and force pump, upward stroke
Examples :
opens the lower valve and closes the upper
1. The stylus of a photograph record exerts a
valve while downward stroke closes the
force of 7.7×10-2N on a groove of radius 10-5m.
lower valve and open the upper valve
Compute the pressure exerted by the stylus on
5.Compression pump – Increases pressure by
the grove A.2.24×109Nm-2 B.4.90×108Nm-2
forcing air into an enclosure e.g bicycle pump and
C.2.45×108Nm-2 D.3.45×108Nm-2
syringe. 𝐹
6.Vacuum or evacuation pump – Decreasing or 𝐹=7.7×10-2𝑁, 𝑟=10-5m, 𝐴 = 𝜋𝑟 2 , 𝑃 = ,
𝐴
lowering pressure by evacuating air from an 7.7×10−2 7.7×10−2 7.7×108 7.7×108
𝑃= = = = ,
𝜋(10−5 )2 𝜋10−10
enclosure e.g crushing can experiment. 𝜋 3.142

– Lift pump, force pump, compression pump 𝑃 =2.45×108Nm-2


and vacuum pump are all mechanical pumps. 2. A rectangular water tank of weight 4.5×103𝑁
measures 2.0m by 1.5m by 1.2m. Calculate the
minimum pressure it can exert when resting on a

98
Demystified Series Physics Demystified by Dr Timothy
horizontal surface A.1.9×103𝑁m-2 and sealed. What is the maximum pressure thus
B.2.5×10 𝑁m C.3.0×10 𝑁m D.1.5×103𝑵m-2.
3 -2 3 -2 container can exert on a flat surface? (g=10ms -2)
𝐹=W=4.5×103N, minimum pressure is exerted A.9×104Nm-2 B.4×104Nm-2 C.3×104Nm-2
when the area is maximum. The maximum area D.2×10 Nm .
4 -2

is given by the dimension 2.0m by 1.5m, Maximum pressure is exerted when the vessel
𝐴 =2.0×1.5=3.0m2, 𝑃 = = ,
𝐹 𝑊
rest on its least area i.e 3m×2m, hence the height
𝐴 𝐴
4.5×103 ℎ=4m, g=10ms-2, 𝜌=1000kgm-3, 𝑃=?,
𝑃= =1.5×103Nm-2. 𝑃 = 𝜌𝑔ℎ = 1000×10×4=40000=4×104𝑁m-2.
3.0
3. A man exerts a pressure of 2.8×103Nm-2 on the 10. A liquid of mass 1.0×103kg fills a rectangular
ground and has 4×10-2m2 of his feet in contact tank of length 2.5m and width 2.0m. If the tank is
with the ground. The weight of the man is 4m high what is the pressure at the middle of the
A.140N B.112N C.102N D.70N. tank? (g=10ms-2) A.1.0×104Nm-2 B.2.0×103Nm-
𝑃=2.8×103𝑁m-2, 𝐴=4×10-2m2, 𝐹=W=?, 2 C.1.5×103Nm-2 D.1×103Nm-2.
𝐹 𝑊
𝑃 = = , W = 𝑃𝐴 = 2.8×103×4×10-2= 112N. Mass of the liquid 𝑚=1.0×103kg, length of the
𝐴 𝐴
tank 𝑙=2.5m, width or breadth of the tank
4. A rectangular block of dimensions
𝑏=2.0m, height of the tank ℎ=4m, Volume of the
2.0m×1.0m×0.5m weighs 200N. Calculate the
liquid=Volume of the tank, 𝑉 = 𝑙 × 𝑏 × ℎ
maximum pressure exerted by the block on the
𝑉 =2.5×2×4=20m3, density of the liquid,
horizontal floor A.100Nm-2 B.200Nm-2 C.300Nm- 𝑚 1×103
2 D.400Nm-2. 𝜌= = = 50kgm-3, height at the middle of
𝑉 20
𝐹=W=200N, maximum pressure is exerted when 1
the tank ℎ = (4)=2m, Pressure at the middle of
the area is minimum. The minimum area is given 2
the tank, 𝑃 = 𝜌𝑔ℎ = 50×10×2 = 1×103Nm-2.
by the dimension 1.0m×0.5m, 𝐴 =1.0×0.5 ,
𝐹 𝑊 200 11. Determine the pressure, due to water, at the
𝐴 =0.5m2, 𝑃=?, 𝑃 = = = = 400Nm-2. bottom of a tank which 25m deep and is four-
𝐴 𝐴 0.5
5. A man will exert the greatest pressure on a fifths full of water (Density of water= 1000kgm-3,
bench when he A.stands on the toes of one foot g=10ms-2) A.5×102Nm-2 B.4×102Nm-2
B.stands on one foot only C.stands on both feet C.2.5×104Nm-2 D.2×105Nm-2 E.2.5×105Nm-2.
D.lies flat on his back E.lies flat on his belly. ℎ=25m, depth of liquid in the tank= ℎ ,
4
The greatest pressure will be exerted when the 4
5

area is minimum i.e standing on the toes of one 𝜌=1000kgm-3, g=10ms-2, 𝑃 = 𝜌𝑔 ℎ ,


5
foot. 4
𝑃 = 1000×10× ×25 = 2×105Nm-2.
5
6. Which of the following statements is correct? 12. The pressure exerted by a liquid in a
A.pressure exerted is higher with a narrow
container is dependent on the A.mass of the
heel shoe than a flat heel shoe B.no pressure is
liquid B.cross-sectional area of the container
exerted in both narrow and flats heel shoes C.volume of the liquid D.density of the liquid
C.pressure exerted is lower with a narror heel
The pressure exerted by a liquid depends on the
shoes than a flat heel shoe D.pressure exerted is
density of the liquid 𝜌, acceleration due to gravity
the same in both narrow and flat heel shoes. g and depth of the liquid ℎ.
A narrow heel shoe has a small area than a flat
13. Which of the following about liquid pressure
heel shoes, hence will exert the greatest pressure.
is not correct? The pressure A.At a point is
7. The hydrostatic blood pressure diffenrence
proportional to the depth B.At any point in a
between the head and feet of a boy standing
liquid is the same at the same level C.Is exerted
straight is 1.65×104Nm-2. Find the height of the
equally in all directions at any point D.Of a liquid
boy (Density of the blood=1.1×103kgm-3,
at any point on the wall of its container acts in a
g=10ms-2) A.0.6m B.0.5m C.1.5m D.2.0m direction perpendicular to the wall E.At a
𝑃=1.65×104Nm-2, 𝜌=1.1×103kgm-3, g=10ms-2,
particular depth depends on the shape of the
height of the boy ℎ=?, 𝑃 = 𝜌𝑔ℎ,
vessel.
1.65×104 = 1.1×103×10× ℎ, Pressure in liquids at a particular depth is
1.65×104
ℎ= = 1.5m. independent of the shape of the vessel. All other
1.1×103 ×10
8. A reservoir is filled with a liquid of density options are true for liquid pressure.
2000kgm-3. Calculate the depth at which the 14. Which of these statements are correct for the
pressure in the liquid will be equal to 9100Nm-2 pressures in liquids? I.Pressure in a liquid at a
(g=10ms-2) A.0.262m B.0.455m C.0.664m point acts equally in all directions II.Pressure
D.0.819m. increases with depth III.Pressure at a depth
𝜌=2000kgm-3, 𝑃=9100Nm-2, g=10ms-2, ℎ=?, depends on the shape of the container
𝑃 = 𝜌𝑔ℎ , ℎ = =
𝑃 9100
= 0.455m. IV.Pressure at the same depth in different liquids
𝜌𝑔 2000×10 are proportional to the densities of any liquids
9. A weightless vessel of dimensions 4m×3m A.I,II and III B.I,II and IV C.I,III and IV D.II,III
×2m is filled with a liquid of density 1000kgm-3 and IV.

99
Demystified Series Physics Demystified by Dr Timothy
Pressure in liquids at a depth is independent of vapour pressure and need no colouring.
the shape of the container. 22. Mercury is suitable as a barometric fluid
15. I.Density of the liquid II.Acceleration due to because it A.expands uniformly B.is opaque C.is
gravity III.Type of container of the liquid IV.The several times denser than water D.is a good
constituents of the liquid. Which of the above conductor of heat
conditions will not affect the pressure of liquid? The high density of mercury is the major reason
A.I and III B.II and III C.III and IV E.I and II. why it is used a barometric fluid.
Pressure in liquids is affected by the density of 23. Which of the following statements is correct?
the liquid, acceleration due to gravity and depth The reading of pressure on a mercury barometer
below the surface of the liquid. is independent on A.the cross-sectional area of
16. Air was pumped out of sealed can and the can the tube B.the atmospheric pressure C.the
collapsed. Which of these statements is the best density of mercury D.the temperature of mercury
explanation for this? A.Pumping out of the air D.acceleration due to gravity.
weakened the can B.The material used in making Pressure in a mercury barometer depends on
the canw as very weak B.The remaining air density of mercury, acceleration due to gravity
inside the can condensed to form water D.The and height of mercury. The density of mercury is
air pressure inside the can became less than affected by temperature hence, temperature also
the pressure outside the can. has an effect on the reading
This is the crushing can experiment. Pumping air 24. A simple barometer reads 74.6cm when a
out of the sealed can reduces the pressure inside standard Fortin barometer reads 76.0cm of
it, hence the atmospheric pressure exceeds the mercury. This may be because the A.tube of the
pressure inside the can and causes it to collapse. simple barometer is not vertical B.simple
17. When cold water is poured on a can barometer contains some air in the space
containing hot water, the can collapse because above the mercury C.cross-sectional area of the
the A.steam condenses and occupies the partial tube of the simple barometer is greater than that
vacuum in the can B.external air pressure of the Fortin barometer D.cross-sectional rea of
counterbalances the pressure within the can the tube of the simple barometer is smaller than
C.steam expands to occupy the vacuum that of the Fortin barometer E.atmospheric air
remaining in the can D.external pressure exerts less pressure on the simple barometer
becomes greater than the pressure within the than on the fortin barometer
can. The height of mercury colu.mn supported by
18. The instrument used for measuring atmospheric pressure is 76.0cm. A faulty simple
atmospheric pressure is A.hydrometer barometer with air in the space above the
B.barometer C.manometer D.photometer mercury will record a height of mercury column
E.ratemeter less than 76.0cm i.e 74.6cm, due to the air
Barometer is used to measure atmospheric bubbles which decreases the height of mercury
pressure while manometer is used to measure column.
gas pressure. 25. The ivory pointer of a Fortin barometer is
19. Which of the following is suitable to use as an used to A.adjust the level of mercury in the
altimeter? A.a mercury barometer B.a Fortin leather bag B.read the scale of the barometer
barometer C.a mercury manometer D.an C.locate the zero mark of the scale D.adjust the
Aneroid barometer Vernier scale E.open and close the brass casing.
Aneroid barometer is used as an altimeter for 26. The height of the mercury column in a simple
measuring the altitude (or height) of bodies like barometer is ℎ. If the tube is slightly raised, which
mountains or air planes above the ground or sea of the following statement is tru? A.ℎ will change,
level. but the space above the mercury will remain
20. Which of the following devices are used to constant B.ℎ will remain constant, but the space
measure pressure? I.Aneroid barometer above the mercury is will change C.Both ℎ and the
II.Hydrometer III.Hygrometer IV.Manometer A.I space above the mercury will change D.Both 𝒉
and III B.I and IV C.II and III D.III and IV. and the space above the mercury will remain
21. Mercury is a suitable barometric fluid than constant E.None of the above is true.
water because I.It needs no colouring II.Its 27. Which of the following devices used to
density is many times greater than that of water measure pressure contain(s) no liquid? I.Simple
III.It does not wet glass IV.It has a high vapour barometer II.Fortin barometer III.Aneroid
pressure V.It has a high boiling point. Which of barometer IV.Manometer. A.I B.IV C.II and III D.I
the above statements are true? A.I and II B.I,II and II E.III.
and III C.III and IV D.IV and V E.III,IV and V 28. The diagram below is of a faultless simple
Mercury is a better barometric fluid than water barometer.
due to its high density, does not wet glass, low

100
Demystified Series Physics Demystified by Dr Timothy
Compression of the air increases the pressure in
the cylinder which offers resistance to the
motion of the piston 𝑃 as it is pushed into the
ℎ cylinder.
31. 𝑷 The diagram above illustrates
Mercury a simple barometer. Which
𝑸 distance measures the
Which of the following is not correct? A.A atmospheric pressure?
vacuum exists above the mercury in the tube 𝑹
B.The atmospheric pressure supports a column
of mercury ℎ C.The height of the column will not 𝑺
be the same if another loquid is used instead of A.𝑃𝑄 B.𝑸𝑹 C.𝑅𝑆 D.𝑄𝑆 .
mercury D.If the tube is tilted, the vertical The height of mercury column 𝑄𝑅 in the tube of a
height of the mercury column will change E.As simple barometer is the atmospheric pressure.
the barometer is taken up a mountain, ℎ 32. A faulty barometer reads 72.6cmHg when the
decreases. atmospheric pressure is 75.0cmHg. Calculate the
A fautless barometer has a vacuum above the atmospheric pressure when this barometer
mercury in th tube while a faulty barometer has reads 72.0cmHg A.79.4cmHg B.74.4cmHg
air above the mercury in the tube. C.74.2cmHg D.69.4cmHg.
The atmospheric pressure will support a column First barometer reading ℎ1 =72.6cm,
of mercury equal to ℎ and the height ℎ of the Atmospheric pressure at first reading
column will not change if the tube is tilted. The 𝐻1 =75.0cmHg, Second barometer reading
height of the column is different for different ℎ2 =72.0cmHg, Atmospheric pressure at second
liquids due to their density difference i.e ℎ is barometer reading 𝐻2 =?,
76cm for mercury and 1033.6cm for water. Atmospheric pressure 𝑯 is a measure of the
Pressure decreases with increase in altitude(ℎ), barometer reading or barometric height 𝒉,
hence barometer reading indicated by ℎ hence 𝑯 ∝ 𝒉 ,
𝑯𝟏 𝑯
= 𝟐.
75.0 𝐻
= 2 ,
𝒉𝟏 𝒉𝟐 72.6 72.0
decreases. 75.0×72.0
29. Mechanical pumps cannot A.move fluids in a 𝐻2 = = 74.38cmHg =74.4cmHg.
72.6
horizontal direction B.move fluids in a vertical 33. A simple barometer is constructed using a
direction C.compress gases in a container liquid 𝑄 of density 16.2gcm-3. Calculate the least
D.increase the atmospheric pressure length of the tube to be used to measure pressure
E.decrease the pressure in an enclosure of 1.5 atmosphere [Density of mercury=13.6gcm-
Atmospheric pressure cannot be increased or 3, 1 atmosphere=76.0cm of mercury] A.1358mm

decreased by a mechanical pump. Mechanical B.957mm C.760mm D.638mm


pumps can be used to move fluids in horizontal 𝜌𝑄 =16.2gcm-3, 𝜌𝐻𝑔 =13.6gcm-3, 𝑃=1.5atm,
direction when flow rate is low. Mechanical 1atm=76.0cmHg, ∴ 𝑃 =1.5×76.0=114cmHg,
pumps , lift and force pumps, can be used to move height of mercury in barometer ℎ𝐻𝑔 =pressure in
fluids in a vertical direction. Mechanical pumps height of mercury=114cmHg, height of liquid 𝑄
e.g vacuum or evacuation pump, can be used to inin barometer ℎ𝑄 =?,
decrease the pressure in an enclosure by Pressure in barometer when mercury or liquid 𝑄
evacuating air. Mechanical pressure e.g is used as the barometric liquid is the same.
compression pump(bicycle pump) can be used to 𝑃𝐻𝑔 = 𝑃𝑄 , 𝜌𝐻𝑔 𝑔ℎ𝐻𝑔 = 𝜌𝑄 𝑔ℎ𝑄 ,
increase the pressure in a vessel by compressing
𝜌𝐻𝑔 ℎ𝐻𝑔 = 𝜌𝑄 ℎ𝑄 , 13.6×114=16.2× ℎ𝑄 ,
air or gases into a container. 13.6×114
30. ℎ𝑄 = = 95.7cm = 957mm.
16.2
Air 𝑃 34. The figure above shows
an inverted U-tube with
cylinder the open end, 𝑂 of one
In the diagram above, the piston 𝑃 experiences a limb below the level 𝑊,
resistance to motion as it is pushed into the 𝑾 of water in a tank. In
cylinder because A.as the air is compressed its 𝑶 𝑷 order that water
pressure increases B.as the volume of the air is should begin to flow from
reduced its kinetic energy increases C.the the tank it is necessary that A.The U-tube is
increase in pressure increases the friction at the completely filled with water and 𝑃 should be
walls of the cylinder D.the decrease in the volume higher than 𝑊B.𝑷 should be lower than 𝑶 and
of the air increases its viscosity E.pressure is 𝑾 C.𝑃 should be lower than 𝑊 and 𝑂 should
transmitted equally in all directions. reach to the bottom of the veseel D.The U-tube is
The diagram above is a compression pump. completely filled with water and 𝑃 should be

101
Demystified Series Physics Demystified by Dr Timothy
lower than 𝑊 E.The U-tube is completely filled open surface of a pool of mercury of relative
with water and 𝑂 should reach the bottom of the density 13.6.If the barometric height is 76cm of
vessel. mercury,the pressure on the membrane is
The diagram is an example of a siphon. For water A.3.26×104Nm-2 B.1.03×104Nm-2
to flow out of 𝑃 – the U-tube must be completely C.1.36×10 Nm D.1.36×10 Nm-2
5 -2 7

filled with water due to suction pressure, 𝑃 Barometric height or height supported by
should be lower than 𝑂 and 𝑊 as liquid flows atmospheric pressure 𝐻 =76cm=0.76m
under pressure 𝜌𝑔ℎ, ℎ is the difference between depth below water surface ℎ=24cm=0.24m
levels of 𝑊 and 𝑃 , 𝑂 should be lower than 𝑊. 𝜌 = 𝑅. 𝐷 ×1000=13.6×1000=1.36×104kgm-3
- A siphon cannot function in space as Pressure on the membrane = Atmospheric
atmospheric pressure at the surface of the pressure +𝜌𝑔ℎ = 𝑃𝑎𝑡𝑚 + 𝜌𝑔ℎ=𝜌𝑔𝐻 + 𝜌𝑔ℎ,
liquid is zero. 𝑃 = 𝜌𝑔(𝐻 + ℎ) =1.36×104×10(0.76+0.24) ,
35. piston The diagram shows a 𝑃 =1.36×105×1 = 1.36×105Nm-2.
P lift pump with valves 40. The barometric reading at a place is
P and Q. During a 73.5cmHg. Calculate the pressure at a point 30m
downward stroke of below the surface of water contained in a
Q the piston, reservoir at the place (g=10ms-2, density of
mercury=1.3×104kgm-3, density of water=
1.0×103kgm-3) A.4.0×105Nm-2 B.3.0×105Nm-2
A.both valves are opened B.P is opened while Q C.2.0×105Nm-2 D.1.0×105Nm-2
is closed C.P is closed while Q is opened D.both Barometric reading at the place=atmospheric
valves are closed. pressure at that place=73.5cmHg=0.73mHg,
The two valves in lift and force pumps opens and height of mercury at the place corresponding to
clioses alternarively during the upward and atmospheric pressure ℎ𝐻𝑔 =0.73m, density of
downward stroke. In upward stroke, the upper mercury 𝜌𝐻𝑔 =1.3×104kgm-3, g=10ms-2, depth of
valve P closes while the lower valve opens. In waterℎ𝐻2𝑂 =30m, density of water
downward stroke, the upper valve P opens while 𝜌𝐻2𝑂 =1.0×103kgm-3 ,
the lower valve Q closes. Atmospheric pressure in 𝑁m-2= 𝜌𝐻𝑔 𝑔ℎ𝐻𝑔 ,
36. Normal atmospheric pressure at sea level is
Pressure due to depth of water = 𝜌𝐻2𝑂 𝑔ℎ𝐻2𝑂 ,
105Nm-2 and the acceleration due to gravity is
approximately 10ms-2. If the atmosphere has a ∴ Pressure at the point 30m below the surface of
uniform density of 1kgm-3. What is its height? water 𝑃 = 𝑃𝑎𝑡𝑚 + 𝜌𝐻2𝑂 𝑔ℎ𝐻2𝑂 ,
A.100m B.1000m C.10000m D.100000m 𝑃 = 𝜌𝐻𝑔 𝑔ℎ𝐻𝑔 + 𝜌𝐻2𝑂 𝑔ℎ𝐻2𝑂 ,
E.1000000m 𝑃 =1.3×104×10×0.735+1.0×103×10×30,
Atmospheric pressure 𝑃=105Nm-2, g=10ms-2, 𝑃 = 0.9555×105+3.0×105=(0.955+3.0)105,
𝜌=1kgm-3, ℎ=?, 𝑃𝑎𝑡𝑚 = 𝜌𝑔ℎ, 𝑃 =3.9555×105 = 4.0×105Nm-2.
105 41. A rectangular tank contains water to the
105=1×10× ℎ, ℎ = = 104m=10000m.
10 depth of 2m. If the base is 4m×3m, calculate the
37. Calculate the length of the liquid in a force on the base (density of water= 103kgm-3,
barometer tube that would support an g=10ms-2) A.2.4×105N B.2.4×104N C.2.0×104N
atmospheric pressure of 204000Nm-2, if the D.1.7×103N.
density if the liquid is 5200kgm-3 (g=10ms-2) ℎ=2m, base dimension 4m×3m, base area
A.0.75m B.0.76m C.3.92m D.39.23m 𝐴=4×3=12m2, 𝜌=103kgm-3, g=10ms-2, 𝐹=?,
Atmospheric pressure 𝑃=204000Nm-2, g=10ms-2, 𝐹
𝑃 = = 𝜌𝑔ℎ , 𝐹 = 𝐴(𝜌𝑔ℎ),
𝜌=5200kgm-3, ℎ=?, 𝑃𝑎𝑡𝑚 = 𝜌𝑔ℎ, 𝐴
𝑃 204000 𝐹 =12(103×10×2) = 24×104 = 2.4×105N.
ℎ= = = 3.92m.
𝜌𝑔 5200×10 42. The horizontal door of a submarine at a depth
38. A diver is 5.2m below the surface of water of 500m has an area of 0.4m2.Calculate the force
density 1000kgm-3. If the atmospheric pressure exerted by the sea water on the door at this
is 1.02×105Pa, calculate the pressure on the depth. (Relative density of sea water=1.03,
diver (g=10ms-2) A.1.02×105Pa B.1.54×105Pa density of pure water=1000kgm-3 Atmospheric
C.5.20×105Pa D.6.20×105Pa pressure=1.0×105𝑁m-2, g=10ms-2) A.6.3×106𝑁
𝑃𝑎𝑡𝑚 =1.02×105Pa, g=10ms-2, 𝜌=1000kgm-3, B.3.2×106𝑁 C.2.1×106𝑵 D.1.2×106𝑁
ℎ=5.2m, Pressure at a depth 5.2m below the ℎ𝑠𝑒𝑎 =500m, 𝐴=0..4m2, 𝑅. 𝐷 of sea=1.03,
water surface, 𝑃 = 𝑃𝑎𝑡𝑚 + 𝜌𝑔ℎ , 𝜌𝑠𝑒𝑎 =1.03×103kgm-3, 𝑃𝑎𝑡𝑚 =1.0×105𝑁m-2,
𝑃 =1.02×105+1000×10×5.2, ∴ Pressure at a depth of 500m in sea water ,
𝑃 =1.02×105+5.2×104=104(1.02×10+5.2), 𝑃 = 𝑃𝑎𝑡𝑚 + 𝑃𝑠𝑒𝑎 = 𝑃𝑎𝑡𝑚 + 𝜌𝑠𝑒𝑎 𝑔ℎ𝑠𝑒𝑎 ,
𝑃 =104(10.2+5.2)=104(15.4)=1.52×105Pa or 𝐹
𝑃 = = 𝑃𝑎𝑡𝑚 + 𝜌𝑠𝑒𝑎 𝑔ℎ𝑠𝑒𝑎 ,
1.52×105Nm-2. 𝐴
𝐹 = 𝐴(𝑃𝑎𝑡𝑚 + 𝜌𝑠𝑒𝑎 𝑔ℎ𝑠𝑒𝑎 ),
39. A circular membrane lies 24cm below the

102
Demystified Series Physics Demystified by Dr Timothy
𝐹 = 0.4(1.01×105+1.03×103×10×500), pressure at sea level 𝐻𝑜 =75cmHg , barometric
𝐹 = 0.4(1.01×105+51.50×105), height above sea level at 𝐻𝑃 (smaller height) or
𝐹 = 0.4×52.51×105 = 21.004×105 = 2.1×106N. pressure above sea level 𝐻1 =65cmHg,
𝜌𝐻𝑔
43. The hatch door of a submarine has an area of 𝐻𝑃 = (𝐻𝑜 − 𝐻1 ) =
13.0
(75−63),
𝜌𝑎𝑖𝑟 0.00013
0.5m2. The specific gravity of sea water is 1.03,
assume that g=10ms-2 and neglect the 𝐻𝑃 = 100000(12) = 1200000cm = 12000m.
atmospheric pressure, the force exerted by the 47. The height at which the atmosphere ceases to
exist is about 80km. If the atmospheric pressure
sea water on the hatch door at depth of 200m is
A.1.0×106Nm-2 B.1.03×104Nm-2 C.2.06×105Nm- on the ground level is 760mmHg, the pressure at
2 D.2.06×106Nm-2 E.1.03×103Nm-2. a height of 20km above the ground level is
𝐴=0.5m2, specific gravity or relative density of A.380mmHg B.570mmHg C.190mmHg
sea 𝑅. 𝐷=1.03, 𝜌𝑠𝑒𝑎 =1.03×103kgm-3, ℎ=200m, D.480mmHg (𝜌𝑚 =13.68gcm-3, 𝜌𝑎𝑖𝑟 =0.00013
g=10ms-2, Neglecting atmospheric pressure gcm-3).
𝐹 Pressure change for mmercury is equal to
𝑃𝑎𝑡𝑚 , 𝑃 = 𝑃𝑠𝑒𝑎 = 𝜌𝑠𝑒𝑎 𝑔ℎ𝑠𝑒𝑎 , 𝑃 = = 𝜌𝑠𝑒𝑎 𝑔ℎ𝑠𝑒𝑎 , pressure change for air. 𝑃𝐻𝑔 = 𝑃𝑎𝑖𝑟 ,
𝐴
𝐹 = 𝐴(𝜌𝑠𝑒𝑎 𝑔ℎ𝑠𝑒𝑎 ) = 0.5(1.03×103×10×200) 𝜌𝐻𝑔 =13.68gcm-3, atmospheric pressure at
𝐹 = 0.5(206×104) = 103×104 = 1.03×106Nm-2. ground level=760mmHg=76cmHg, pressure
44. Two divers 𝐺 and 𝐻 are at depths 20m and above the ground level 𝐻 1=?, height above the
40m repectively belowe the water surface in a ground 𝐻=20km=2×106cm, 𝜌𝐻𝑔 =13.68gcm-3 ,
lake. The pressure on 𝐺 is 𝑃 1 while the pressure
𝜌𝑎𝑖𝑟 =0.00013gcm-3,
on 𝐻 is 𝑃 2. If the atmospheric pressure is
𝜌𝐻𝑔 𝑔ℎ𝐻𝑔 = 𝜌𝑎𝑖𝑟 𝑔𝐻 , 𝜌𝐻𝑔 ℎ𝐻𝑔 = 𝜌𝑎𝑖𝑟 𝐻,
equivalent to 10m of water, the value of 𝑃 2/𝑃 1 is
13.68(76−𝐻 1)=0.00013×2×106
A.0.50 B.0.60 C.1.67 D.2.00 E.3.00. 0.00013×2×106
Depth of diver 𝐺 ℎ1=20m, depth of diver 𝐻 76−𝐻1 = = 19.0, 𝐻 1 = 76−19,
13.68
ℎ2=40m, Atmospheric pressure 𝐻=10m of water. 𝐻 1 = 57cmHg = 570mmHg. Alternatively,
𝑃 = 𝐻 + ℎ, 𝑃 1=10+20=30m, 𝑃 2=10+40=50m, 𝜌𝐻𝑔
𝐻𝑃 = (𝐻𝑜 − 𝐻1 ), height of plane above the
𝑃 2/𝑃 1 = 50/30 = 1.67. 𝜌𝑎𝑖𝑟

45. At a fixed point below liquid surface, the ground 𝐻𝑃 =12km=2×106cm, 𝜌𝐻𝑔 =13.68gcm-3,
pressure downward is 𝑃 1 and the pressure 𝜌𝑎𝑖𝑟 =0.00013gcm-3, barometric height at sea
upward is 𝑃 2. It can be deduced that A.𝑷1 = 𝑷2 level (larger height) or pressure at sea level
B.𝑃 1> 𝑃 2 C.𝑃 1 < 𝑃 2 D.𝑃 2> 𝑃1. 𝐻𝑜 =75cmHg , barometric height above sea level
Pressure at all points at the same level or a fixed at 𝐻𝑃 (smaller height) or pressure above sea level
13.68
points within a liquid is the same. Hence, 𝐻1 =?, 2×106 = (75−𝐻1 ),
0.00013
pressure downward 𝑃 1 is equal to pressure 0.00013×2×106
upward 𝑃 2 as both are at a fixed point or same (76−𝐻1 ) = =19 ,
13.68
level(or depth) below the liquid surface. 𝐻1 = 76−19 = 57cmHg = 570mmHg.
46. A pilot records the atmospheric pressure 48. In the Hare’s apparatus, water rises to a
outside the plane as 63cm of Hg while a ground height of 26.5cm in one limb. If a liquid rises to a
observer records a reading of 75cmHg for the height of 20.4cm in the other limb, what is the
atmospheric pressure on the ground. Assuming relative density of the liquid? A.0.8 B.1.1 C.1.2
that the density of the atmosphere is constant, D.1.3.
calculate the height of the plane above the height of water column ℎ𝑤 =26.5cm, height of
𝜌 ℎ
ground (Relative density of Hg=13.0 and of the liquid column ℎ𝐿 =20.4cm, 𝑅. 𝐷𝐿 = 𝐿 = 𝑤 ,
𝜌𝑤 ℎ𝐿
air=0.00013) A.12000m B.6300m C.7500m ℎ𝑤 26.5
D.13800m 𝑅. 𝐷𝐿 = = = 1.3.
ℎ𝐿 20.4
Pressure change for mercury is equal to pressure 49. Water and kerosene are drawn respectively
change for air. 𝑃𝐻𝑔 = 𝑃𝑎𝑖𝑟 , into two limbs of a Hare’s apparatus. The density
𝑅. 𝐷𝐻𝑔 =13.0, 𝜌𝐻𝑔 =13.0gcm-3, atmospheric of water is 1.0gcm-3 and the density of kerosene
pressure at the ground or sea level=75cmHg, is 0.80gcm-3. If the height of the water column is
pressure above sea level= 63cmHg, height above 20.0cm, calculate the height of the kerosene
the ground 𝐻=?, 𝑅. 𝐷𝑎𝑖𝑟 =0.00013, column A.1.6cm B.16.0cm C.20.0cm D.20.8cm
𝜌𝑎𝑖𝑟 =0.00013gcm-3, 𝜌𝐻𝑔 𝑔ℎ𝐻𝑔 = 𝜌𝑎𝑖𝑟 𝑔𝐻 , E.25.0cm.
𝜌𝐻𝑔 ℎ𝐻𝑔 = 𝜌𝑎𝑖𝑟 𝐻, 13.0×(75−63) = 0.00013× 𝐻, 𝜌𝑤 =1.0gcm-3, 𝜌𝑘 =0.80gcm-3, ℎ𝑤 =20.0cm, ℎ𝑘 =?
𝜌 ℎ 0.80 20.0 20.0
13.0×12 = 0.00013× 𝐻, 𝐻 =
13.0×12
, 𝑅. 𝐷𝑘 = 𝑘 = 𝑤 , = , ℎ𝑘 = ,
𝜌𝑤 ℎ𝑘 1.0 ℎ𝑘 0.80
0.00013
𝐻 = 1200000cm=12000m. Alternatively : ℎ𝑘 = 25.0cm.
𝜌𝐻𝑔 50. Atmospheric pressure supports height 𝑦 and
𝐻𝑃 = (𝐻𝑜 − 𝐻1 ), height of plane above the
𝜌𝑎𝑖𝑟 2𝑦 of water and liquid 𝑄 respectively. The
ground 𝐻𝑃 =?, 𝜌𝐻𝑔 =13.0gcm-3, 𝜌𝑎𝑖𝑟 =0.00013gcm- relative density liquid 𝑄 is A.4.0 B.2.0 C.1.0 D.0.5.
3, barometric height at sea level (larger height) or
Height of water atmospheric pressure supports

103
Demystified Series Physics Demystified by Dr Timothy
ℎ𝑤 =𝑦, height of liquid 𝑄 atmospheric pressure connected to a gas supply, the levels of mercury
ℎ 𝑦
supports ℎ𝑄 =2𝑦, 𝑅. 𝐷𝑄 = 𝑤 = = 0.5. in the two arms of the U-tube are as shown in the
ℎ𝑄 2𝑦
diagram above. If the atmospheric pressure is
51. 76.0cmHg, what is the gas pressure?
P A.62.6cmHg B.72.5cmHg C.79.5cmHg
D.85.9cmHg.
22.5cm 7.5cm
ℎ = difference in the level of liquids in both
arms=13.4cm−3.5cm=9.9cm, Atmospheric
Y X pressure 𝐻=76.0cm, Gas pressure in chamber G,
𝑃 = 𝐻 + ℎ =76.0+9.9=85.9cmHg.
The figure above shows the height of two liquids
55. Atmospheric pressure=70cm mercury
𝑋 and 𝑌 when some air is sucked out of the Gas supply
apparatus through the pump P. The diameter of
3cm
the tube in 𝑋 is twice that of tube in 𝑌. What is the
relative density of liquid 𝑋 with respect to liquid
𝑌 A.1/3 B.2/3 C.3 D.6.
The pressure is independent of the diameter or
area of the tube. The pressure at the surface of
The water manometer in the figure above is
both liquids are equal as they both exposed to the measuring the pressure of the gas supply. If the
atmosphere. 𝑃𝑌 = 𝑃𝑋 , 𝜌𝑌 𝑔ℎ𝑌 = 𝜌𝑋 𝑔ℎ𝑋 ,
𝜌 ℎ 22.5 specific gravity of mercury is 13.6 and the
𝜌𝑌 ℎ𝑌 = 𝜌𝑋 ℎ𝑋 , 𝑅. 𝐷𝑋 = 𝑋 = 𝑌 = = 3. atmospheric prerssure is 70cm of mercury, what
𝜌𝑌 ℎ𝑋 7.5
52. What is the pressure of the gas in the diagram is the total pressure of the gas supply in cm of
below? water? A.76cm B.73cm C.949cm D.952cm
𝑃 E.955cm.
Tap Pressure of the gas supply is the pressure at the
Liquid From gas liquid level in the left arm of the manometer.
supply Pressure at the left arm and the right arm of the
ℎ manometer are equal at the point where the
liquid level in both are the same. Atmospheric
pressure=70cmHg, the equivalent of
atmospheric pressure in cmH2O is required as
Density of the liquid in the manometer=𝑑 kgm-3, stated in the question. 𝜌𝐻𝑔 𝑔ℎ𝐻𝑔 = 𝜌𝐻2𝑂 𝑔ℎ𝐻2𝑂 ,
atmospheric pressure=𝑃 Nm-2, acceleration due 𝜌𝐻𝑔 ℎ𝐻𝑔 = 𝜌𝐻2𝑂 ℎ𝐻2𝑂 ,𝜌𝐻𝑔 =13.6gcm-3, ℎ𝐻𝑔 =70cm,
to gravity=g ms-2. A.(𝑃 + ℎ)Nm-2 B.(𝑃 − ℎ)Nm-2 𝜌𝐻2𝑂 =1gcm-3, ℎ𝐻2𝑂 =?, 13.6×70=1× ℎ𝐻2𝑂 ,
C.(𝑷 + 𝒉𝒅g)Nm-2 D.(𝑃 + ℎ𝑑)Nm-2 E.ℎ𝑑gNm-2. ℎ𝐻2𝑂 =952cm, Atmospheric pressure
The total gas pressure = Atmospheric 𝐻 =952cmH2O, ℎ=3cm, Total gas pressure
pressure+pressure due to liquid column 𝑃 = 𝐻 + ℎ =952+3=955cmH2O.
Total gass pressure= (𝑃 + ℎ𝑑g)Nm-2. 56.
53.
G

10cm
3cm

In the diagram above, if the atmospheric A girl stands on an air bag connected to a
pressure is 760mm, the pressure in the chamber manometer as illustrated in the diagram above.
G is A.660mm B.690mm C.830mm D.860mm. Determine the pressure exerted on the bag by the
ℎ = difference in the level of liquids in both girl [Atmospheric pressure = 75cmHg]
arms=10cm−3cm=7cm=70mm, Atmospheric A.10cmHg B.17cmHg C.70cmHg D.80cmHg.
pressure 𝐻=760mm, Pressure exerted by the girl is same as the total
Gas pressure in chamber G, 𝑃 = 𝐻 + ℎ , gas pressure 𝑃=?, atmospheric pressure
𝑃 =760+70=830mm. c
𝐻=75cmHg, difference in the level of mercury of
54. the arm exposed to the atosphere and that
To gas supply connected to the air bag ℎ =16−11=5cmHg
𝑃 = 𝐻 + ℎ =75+5 = 80cmHg.
13.4cm 57. When pressure is applied by means of a
3.5cm piston to the surface of a liquid contained in a
cylinder, the pressure transmitted to the liquid is
When one arm of a U-tube manometer is A.greater at the surface thanm the bottom of the

104
Demystified Series Physics Demystified by Dr Timothy
liquid B.all at the bottom of the liquid C.the same while molecules are capable of free existence
at every point in the liquid D.only at the surface 2.Energy is required to overcome and break the
of the liquid E.smaller at the surface than at the attractive forces between molecules of matter.
bottom of the liquid. - Evidence of molecular or kinetic theory of
Pressure applied at a point in a liquid is matter – Diffusion,change of state(melting,
transmitted equally in all directions in the liquid, boiling,evaporation),osmosis,heat transfer
(Pascal’s principle). (conduction and convection), Brownian
58. The areas of cross-section of two pistons in a motion,expansion.
hydraulic press are 2cm2 and 150cm2. If a - Evidence of intermolecular forces –
downward force of 20𝑁 is exerted on the smaller Friction,surface tension,cohesion and
piston, calculate the force om the larger one adhesion,capillarity, elasticity and viscosity.
A.112500N B.1500N C.150N D.15N E.3.75N. - Radiation cannot be explained by molecular or
𝐹1 =20𝑁, 𝐹2 =?, 𝐴1 =2cm2, 𝐴2=150cm2, kinetic theory of matter as it involves
𝐹1 𝐹
= 2,
20 𝐹
= 2 , 𝐹2 =
150×20
= 1500N. electromagnetic radiations in space and not
𝐴1 𝐴2 2 150 2
particles of matter.
- Temperature is a physical property and is a
● Gas laws and kinetic theory of measure of the average kinetic energy.
matter - Heat or Thermal energy is a form of energy and
- Kinetic theory of gases : is a measure of the total internal energy.
1.The temperature of a gas is a measure of the - The density of a gas depends on the
average kinetic energy of the gas molecules. separation between its molecules i.e
2.The volume of gas molecules is negligible intermolecular distance.
compared to the volume of the gas container. - Matter exists in three states namely solid, liquid
3.The attractive force between gas molecules are and gas. The three states of matter differ in their
negligible as they are far apart relative to their degree of movement,shape,
sizes (attraction between gas molecules is compressibility,intermolecular bonds and
negligible). energy of molecules.
4.The molecules of a gas move in straight lines - The major difference in solids, liquids and gases
colliding with one another and with the walls of is accounted for by the spacing of their molecules
the container. and intermolecular forces acting between the
5.The collisions of gas molecules are perfectly molecules.
elastic i.e kinetic energy and momentum of the Solids – I.Molecules of solids are in fixed
system are conserved. positions and closely packed i.e they have
6.The duration between the gas molecules is definite shapes. II.Solid molecules moves
negligible rather than the time between collision. vibrates or oscillates about a their mean position
- The collision of gas molecules with the walls and has the least kinetic energy. III.Molecules of
of the container constitute the pressure. a solid are bounded by the strongest
- Expansion of gases – Gases expands more than intermolecular forces. IV.Solids are not
liquids and solids for every rise in temperature. compressile.
Gas expansion is accompanied by a change in Liquids – I.Molecules of liquids are free to move
temperature,pressure,volume and density. within the liquid i.e assume the shape of its
- Collision of gas molecules : container and loosely packed. II.Liquid
1.The molecules of gases are in constant motion molecules move randomly or vibrate about
but the number of collisions is not constant i.e different mean positions within the liquid and
Number of collision increases as the volume has more kinetic energy than solids.
decreases. III.Molecules of liquids are bounded by stronger
2.Number of collision increases with increase in intermolecular forces but less than that of solids.
temperature. IV.Liquids are not compressible.
3.Number of molecules of gas does not increase Gases – I.Molecules of gases are farther apart
with increase in pressure. due to the large intermolecular distance between
4.Number of collision per second i.e rate of its molecules. II.Gas molecules move randomly
collision, increases with pressure. or disorderly colliding with one another and with
- Mean free path is the mean or average the walls of the container and has the highest
distance which a molecule travels between kinetic energy. III.The intermolecular forces
successive collisions. between gas molecules are relatively small or
- Kinetic theory of matter : negligible. IV.Gases are compressible.
1.Matter is made up of tiny paryicles called atoms - Diffusion – Diffusion is movement of
and molecules. molecules(solids,liquids or gases), from a more
- Atoms are not capable of free existence crowded region to a less crowded region, till

105
Demystified Series Physics Demystified by Dr Timothy
equilibrium is attained. Molecules diffuse from a barometric readings of the faulty barometer for
refion of higher concentration to a region of the 1st and 2nd days respectively. 𝑷𝟏 = 𝑯𝑨 − 𝑷𝑨 ,
lower concentration. 𝑷𝟐 =𝑯𝑩 − 𝑷𝑩 , 𝑽𝟏 =𝒍𝟏 = 𝒍 − 𝒍𝑨 , 𝑽𝟐 =𝒍𝟐 = 𝒍 − 𝒍𝑩 .
- The difference in the concentration of both Applying Boyle’s law – 𝑃1 𝑉1 = 𝑃2 𝑉2 ,
mediums is called diffusion of concentration 𝑷𝟏 and 𝑷𝟐 are the pressures of the trapped air for
gradient. Rate of diffusion increases as the the first and second days respectively.
diffusion or concentration gradient increases. 𝑽𝟏 and 𝑽𝟐 are the volume of trapped air for the
- Factors affecting the rate of diffusion 𝑹 : first and second day respectively.
𝟏 𝟏
1.size,mass or density, 𝑹 ∝ or 𝑹 ∝ , rate of 2. Capillary tubes(tubes with one end closed)
𝒎 𝝆
– The pressure exerted depends on the plane of
diffusion increases as the size, mass or density of inclination of the tube.
decreases. 2.Temperature, 𝑹 ∝ 𝑻 , the rate of a.Vertically, with open end facing upward or
diffusion increases as the temperature increases. closed end at the bottom –
3.Hydrostatic or osmotic pressure , pressure 𝑯
that increases the rate of diffusion or balances 𝑷𝟏 = 𝑯 + 𝒉
osmosis.
- The rate of diffusion of solids, liquids and gases
increases in the order – solids<liquids< gases. 𝒉 Mercury thread
Hence, solids has the least rate of diffusion while Trapped air or gas
gases has the highest rate of diffusion. 𝑽𝟏 =𝒍𝟏 𝑷𝟏
- Brownian motion is a random motion motion
of molecules(solid,liquid or gas). It gives b.Vertically, with open end facing downward
evidence of tiny particles of matter and they or closed end at the top –
are in constant state of random motion.
- Boyle’s law gives the relation between 𝑽𝟐 =𝒍𝟐 Trapped air or gas
pressure and volume of a fixed mass at constant 𝑷𝟐
temperature. It states that the pressure of a fixed
mass of a gas, is inversely proportional to its 𝒉 Mercury thread
volume provide the temperature is kept
𝟏
constant. 𝑷 ∝ , 𝑷𝑽 =constant, 𝑷𝟏 𝑽𝟏 = 𝑷𝟐 𝑽𝟐 .
𝑽
- Graphs of Boyle’s law : 𝑯 𝑷𝟐 = 𝑯 + 𝒉
𝑷 𝑷 c. Horizontally lying flat –

𝑽𝟑 =𝒍𝟑 𝒉 𝑷𝟑 = 𝑯
𝟏
𝑽
𝑽
𝑷𝟑 𝑯
𝑷𝑽
𝑷𝟏 , 𝑷𝟐 and 𝑷𝟑 are the respective pressures of the
trapped air. 𝑽𝟏 , 𝑽𝟐 and 𝑽𝟑 are the respective
volumes of the trapped air and is equal to their
respective length of air trapped by the capillary
𝑷 or 𝑽
tube i.e 𝒍𝟏 , 𝒍𝟐 and 𝒍𝟑 , as the tube is of uniform
- Applications of Boyle’s law :
cross-sectional area.
1.Simple barometer – The pressure in a simple
𝑯 =atmospheric pressure, 𝒉=height of mercury
baromter with air trapped above can be varied
thread in the capillary tube.
with the volume of air, represented as the length
Applying Boyle’s law – 𝑃1 𝑉1 = 𝑃2 𝑉2 = 𝑃3 𝑉3 .
of the air column.
NB – The length of entrapped air decreases as
𝑽𝟏 =𝒍𝟏 𝑷𝟏
the pressure increases.
𝑽𝟐 =𝒍𝟐 𝑷𝟐
3.Air bubbles in water bodies – The air bubbles
𝒍
just below the surface of water is under the
𝒍𝑨 =𝑷𝑨 𝑯𝑨 𝒍𝑩 =𝑷𝑩 𝑯𝑩
influence of atmospheric pressure only while air
bubbles at the bottom is under the influence of
atmospheric pressure and pressure due to depth
of water.
Mercury Mercury
𝑯 Atmospheric pressure
𝑷𝟏 = 𝑯,
First day (𝐴) Second day (𝐵)
Top 𝑷𝟐 = 𝑯 + 𝒉
𝒍 = length of the tube above mercury in the
𝒉 𝑷𝟏 𝑽𝟏
container(mercury reservoir), 𝑯𝑨 and 𝑯𝑩 are the
𝑷𝟐 𝑽𝟐
atmospheric reading of the 1st and 2nd days
Bottom
respectively, 𝑷𝑨 (or 𝒍𝑨 ) and 𝑷𝑨 (or 𝒍𝑨 ) are the
106
Demystified Series Physics Demystified by Dr Timothy
𝑷𝟏 =pressure at the top=atmospheric pressure 𝑯, θ, 𝑽𝒐 or 𝒍𝒐 =volume of gas or length of ga column
𝑷𝟐 =pressure at the bottom=atmospheric at 0℃, θ=temperature rise, 𝜸=cubic expansivity
pressure + pressure due to depth of water = 𝑯 + in ℃−1 or 𝐾 −1 .
𝒉 , 𝑽𝟏 =volume of air bubbles at the top, - Graphs on Charles’ law –
𝑽𝟐 =volume of air bubble at the bottom. 𝑽 𝑽
Applying Boyle’s law – 𝑃1 𝑉1 = 𝑃2 𝑉2 .
4.Manometer – The volume of air trapped at
one-arm of a manometer can be varied with 𝑻(𝑲)
pressure. 𝑽
273 0 𝑻(℃)
𝑻
𝑯 𝑯

𝑽 or 𝑻(𝑲)
𝒍𝟏 𝒉𝟏 𝒍𝟐 𝒉𝟐
● Pressure law gives the relation between
pressure and temperature of a fixed mass of gas,
at constant volume. It states that the pressure of
a fixed mass of gas is directly proportional to its
𝑷𝟏 = 𝑯 + 𝒉𝟏 , 𝑽𝟏 = 𝒍𝟏 , 𝑷𝟐 = 𝑯 + 𝒉𝟐 , 𝑽𝟐 =𝒍𝟐 .
absolute temperature provided its volume
- In a manometer, when the liquid in both
remains constant.
arms of the tube are at the same level, the 𝑷 𝑷 𝑷
pressure is atmospheric pressure only. 𝑷∝𝑻, = constant, 𝟏 = 𝟐 .
𝑻 𝑻𝟏 𝑻𝟐
5. Evacuation pump – Boyle’s law applies to an - Pressure expansivity or coefficient of
evacuation pump. 𝑷𝟏 𝑽𝟏 𝒏 = 𝑷𝟐 𝑽𝟐 𝒏 , 𝑷𝟏 𝑽𝟏 𝒏 = pressure expansion 𝜷 of a gas at constant
𝑷𝟐 (𝑽𝟏 + 𝑽𝑷 )𝒏 . 𝑷𝟏 =Initial pressure of air in the volume is the fractional increase in the pressure
vessel, 𝑽𝟏 =volume of vessel to be of a gas from 0℃ per kelvin or degree celcius rise
exhausted,𝑷𝟐 =final pressure of air on the in temperature.
vessel,𝑽𝟐 =volume of the vessel+volume of 𝜷=
𝒊𝒏𝒄𝒓𝒆𝒂𝒔𝒆 𝒊𝒏 𝒑𝒓𝒆𝒔𝒔𝒖𝒓𝒆 𝒇𝒓𝒐𝒎 𝟎℃ 𝑷 −𝑷
= 𝜽 𝒐.
evacuation pump=𝑽𝟏 + 𝑽𝑷 ,𝒏=number of strokes 𝒑𝒓𝒆𝒔𝒔𝒖𝒓𝒆 𝒂𝒕 𝟎℃×𝒕𝒆𝒎𝒑. 𝒓𝒊𝒔𝒆 𝑷𝒐 𝜽

of the pump. 𝑷𝜽 =pressure of gas at θ, 𝑷𝒐 =pressure of gas at


- The slope of a Boyle’s law graph i.e a graph 0℃, θ=temperature rise, 𝜷=pressure
of pressure plotted agains the reciprocal of expansivity in ℃−1 or 𝐾 −1 .
volume or the product of pressure and - Graphs on pressure law –
volume is equal to energy or workdone by an 𝑷 𝑷
expanding gas at constant pressure.
𝑾 = 𝑷∆𝑽 = 𝑷(𝑽𝟐 − 𝑽𝟏 ) .
- Pressure increases with depth from the top to 𝑻(𝑲)
𝑷
the bottom of a water body,hence the volume 273 0 𝑻(℃)
𝑻
decreases simultaneously.
● Charles’ law gives the relation between
volume and temperature of a fixed mass of gas,at 𝑷 or 𝑻(𝑲)
constant pressure. It states that the volume of a - An inexpnasible vessel has a constant
fixed mass of gas is directly proportional to its volume.
absolute temperature at constant pressure. - When solving questions in gas laws, always
Charles’ law can also be stated as, the volume of convert from ℃ to 𝑲 before solving.
1
a fixed mass of gas increases by of its volume 𝑻(𝑲) = 𝒕℃ +273.
273
at 0℃ per degree celcius rise in temperature, - General gas law is a combination of Boyle’s
provided its pressure remains constant. law, Charles’ law and pressure law. General gas
𝑷𝑽 𝑷 𝑽 𝑷 𝑽
𝑽∝𝑻,
𝑽 𝑽
= constant, 𝟏 = 𝟐 .
𝑽 equation is given as: = constant, 𝟏 𝟏 = 𝟐 𝟐 .
𝑻 𝑻𝟏 𝑻𝟐
𝑻 𝑻𝟏 𝑻𝟐
- Volume or cubic expansivity or coefficient of - The ideal gas equation is given as – 𝑷𝑽 = 𝒏𝑹𝑻,
cubic expansion 𝜸 of a gas at constant pressure 𝑃=pressure, 𝑉=volume, 𝑛=number or amount of
is the fractional increase in the volume a gas from moles, 𝑇=temperature, 𝑅=molar gas constant=
0℃ per kelvin or degree celcius rise in 8.313𝐽mol-1𝐾 -1.
temperature. - In summary, the equation of gas laws is
𝒊𝒏𝒄𝒓𝒆𝒂𝒔𝒆 𝒊𝒏 𝒗𝒐𝒍𝒖𝒎𝒆 𝒇𝒓𝒐𝒎 𝟎℃ 𝑽 −𝑽 represented by – 𝑃 𝑥 𝑉 𝑦 𝑇 𝑧 =constant 𝐾.
𝜸= = 𝜽 𝒐. 1.In Boyle’s law [𝑃𝑉 = 𝐾], 𝑥 =1, 𝑦 =1, 𝑧 =0.
𝒗𝒐𝒍𝒖𝒎𝒆 𝒂𝒕 𝟎℃×𝒕𝒆𝒎𝒑. 𝒓𝒊𝒔𝒆 𝑽𝒐 𝜽
- Cubic expansivity of a gas in a capillary tube can 𝑉
2.In Charles’ law [ = 𝐾], 𝑥 =0, 𝑦 =1,𝑧 = −1.
also be given in terms of the length of air column. 𝑇
𝑃
𝒍 −𝒍 3.In Pressure law [ = 𝐾],𝑥 =1,𝑦 =0, 𝑧 = −1.
𝜸= 𝜽 𝒐. 𝑇
𝑃𝑉
𝒍𝒐 𝜽
4.General gas law [ = 𝐾], 𝑥 =1, 𝑦 =1, 𝑧 = −1.
𝑽𝜽 or 𝒍𝜽 =volume of gas or length of gas column at 𝑇

107
Demystified Series Physics Demystified by Dr Timothy
- When an ideal gas expands at constant than that of gases E.molecues of gases move more
temperature, the volume, average separation randomly in their containers.
between molecules increases while pressure, Molecules of liquids move more freely than
average number of collisions per unittime, molecules of solids, as liquid molecules are far
average number per unit volume and momentum apart that those pof solids.
decreases. Average kinetic energy and average 8. Which of the following statements is correct
speed is constant as temperature is constant. about the movement of molecules in a liquid?
- The pressure of gas decreases when cooled I.all the molecules have the sae kinetic energy
(decrease in temperature) at constant volume, II.the number of molecules leaving a liquid
because thegas molecules collide less frequently surface is always the same as the number
with the walls of the container, as they are less entering the liquid III.near the surface molecules
energetic. with kinetic energy above average break away
- At constant pressure, density of a fixed mass of from the liquid and remain as molecules of
gas is inversely proportional to its temperature, vapour. A.I B.II C.III D.I and II E.II and III.
as volume varies inversely with density at Molecules of a liquid have different kinetic
constant mass. energy i.e some are more energetic that others.
- Standard temperature and pressure(S.T.P) – Molecules of liquid near the surface with kinetic
Standard temperature s.t is 0℃ or 273𝑲 while energy above the average, evaporates i.e break
standard pressure is 1atmosphere(1atm) or away from the liquid and remain as molecules of
760mmHg(76cmHg) or 1.013×105Pa or Nm-2. vapour. Statement II is only valid when the liquid
is in dynamic equilibrium with its own vapour i.e
Examples : saturated vapour.
1. The process whereby molecules of different 9. The differences observed in solids, liquids and
substances move randomly is called A.surface gases may be accounted for by A.their relative
tension B.diffusion C.capillarity D.osmosis. sizes B.the spacing and force acting between
2. A few grains of table salt were put in a cup of the molecules C.their melting point D.the
cold water, kept at constant temperature and left different molecules in each of them.
undisturbed.Eventually all the water tasted salty. 10. Which of the following is/are significantly
This action is due to A.convention B.osmosis different in solids, liquids and gases?
C.capillarity D.diffusion E.conductivity. I.the arrangement of the molecules in the entire
Salt molecules diffuse from an area of higher bulk of the material III.the motion of the
concentration to that of lower concentration, molecules III.the distances between the
until it is evenly distributed in the solution. molecules A.I B.II C.I and II D.I and III E.I,II and
3. I.Mass II.Density III.Temperature IV.Nature of III.
substance. Which of the above affect diffusion? Solids are more tightly or close packed than
A.I and II B.I,II and IV C.II,III and IV D.I,II,III and liquids and liquids are more closely packed than
IV. gases i.e difference in the arrangement of
4. When the surface of a piece of chalk is scraped, molecules.
the tiny particle that flakes off are known as Solids oscillate or vibrate about a fixed position.
A.atoms B.molecules C.elements D.matter. Liquids moves randomly or vibrate at different
Matter is made up of tiny particles called atoms mean positions i.e translational motion, within
and molecules. Molecules can exist alone e.g tiny the container. Gases move randomly colliding
particles of the chalk that flakes off, but atoms with one another and with the walls of its
cannot exist alone. container i.e difference in motion of the
5. Which of the following phenomena cannot be molecules.
explained by the molecular theory of matter? The intermolecule distance between molecule
A.Expansion B.Conduction C.Convention increases from solids-liquids-gases i.e difference
D.Evaporation E.Radiation. in ther distances between the molecules.
6. I.Change of state II.Diffusion III.Radiation 11. Which of the following statements about the
IV.Osmosis. Which of the processes above can be molecular theory of matter is not correct?
explained using the kinetic theory? A.I,III and IV A.gases exerts negligible force on one another
B.I,II and IV C.I,II,III and IV D.I,II,III. B.solids are closer together than those of liquids
7. Which of the following statements is not C.gases are in random motion D.liquids undergo
correct about matter? A.molecules of solid move only vibratory motion E.solids vibrates about
to and fro about a fixed point B.energy is required their mean position.
to break the forces of attraction between Liquids vibrates and oscillates about different
molecules of matter C.molecules of solid move mean points within the liquid.
more freely than molecules of liquids 12. Which of the following statements about
D.molecules of liquids are more closely packed kinetic theory of matter is not correct? A.when a

108
Demystified Series Physics Demystified by Dr Timothy
body is heated, the average kinetic energy of increasing pressure.
its molecules decreases B.the molecules of gas Molecules of a gas are in a state of constant
move more freely than tose of a liquid random motion but the number of collisions is
C.molecules of a liquid move more freely than not constant. Increase in pressure and decrease
those of a solid D.the molecules of matter are in volume increases the number of collisions.
always in motion E.matter is made up of very tiny Number of collisions per seconds increases with
particles called molecules. pressure, not number of molecules. Number of
When a body is heated, the temperature collisions also increases with temperature.
increases, hence the average kinetic energy of its 18. According to the kinetic theory of gases, the
molecules also increases. collision of the molecules of a gas with the walls
13. In which of the following media would the of the container is mainly responsible for the
average speed of the molecules of water be I.temperature of the gas II.viscosity of the gas
highest? A.ice B.water C.steam D.ice-water III.pressure of the gas A.I B.II C.III D.II and III
mixture E.water-steam mixture. E.I,II and III.
The average speed of molecules increases from 19. Which of the following statements about the
solids-liquids-gases, due to decrease in the gaseous state of matter is not correct? The
intermolecular forces binding the molecules. molecules of a substance in gaseous state A.are
Hence, the molecues of steam (gaseous water) in a constant state of motion B.have different
has the highestaverage speed. speeds C.have a temperature which is a measure
14. I.It is a rapid, constant and irregular motion of the average kinetic energy D.have a pressure
of tiny particles II.It gives evidence than any tiny which is determined by the number if
particles of matter called molecules exists III.It molecules present E.have no intermolecular
takes place only in gases IV.It gives evidence that attractive force.
molecules are in a constant state of random The pressure in gases is determined by the
motion. Which of the combinations above is collisions of the gas molecules with the walls of
correct about Brownian motion? A.I,II and III the container and not the number of molecules
B.I,II and IV C.I,III and IV D.II,III and IV. present.
Brownian motion takes place in solids, liquids 20. Which of the following quantities will
and gases. increase if a tight-fitting piston is pushed into a
15. From the kinetic theory of gases, temperature cylinder closed at one end? I.the number of
is a A.Form of energy and is proportional to the collisions per second between the air molecules
total kinetic energy of the molecules B.Form of II.the average distance distance between the air
energy and is proportional to the average kinetic molecules III.the number of collisions per second
energy of the molecules C.Physical property and of the air molecules against the walls of the
is proportional to the total kinetic energy of the cylinder IV.the volume of the air A.I and II B.I
molecules D.Physical property and is and III C.II and III D.I,II and III E.II,III and IV.
proportional to the average kinetic energy of Pushing the piston into the cylinder compresses
the molecules. the air in the cylinder.Hence the pressure
Heat is a form of energy and is proportional to the increases while the volume decreases. The
total kinetic energy of the molecules. number of collisions per second between the
16. Which of the following is not correct about molecules and the number of collisions per
the assumptions of the kinetic theory of gases? second between the molecules and the walls of
A.the attraction between the molecules is the container increases as pressure increases.
negligible B.the volume of molecules is negligible The average distance between the molecules
compared with the volume occupied by the gas decreases as the volume decreases.
C.the molecules of the gas behave like 21. What hapens when a gas expands at a
perfectly elastic spheres D.the duration of a constant temperature A.It pressure decreases
collision is negligible compared with the time B.The total momentum of its molecules remains
between collisions. constant C.Its pressure decreases and the total
It is the collision of gas molecules that is perfectly momentum of its molecules remains constant
elastic. D.Its pressure decreases and the total kinetic
17. One valid assumption of the kinetic theory of energy of its molecules decreases.
gases is that A.the molecules of a gas are Expansion of gas at constant temperature
constantly in a state of motion and the number of increases the volume, hence the pressure
collisions remain constant B.as the temperature decreases by Boyle’s law. The momentum change
increases, the number of collisions made by the per second decreases as pressure decreases. The
gas remains constant C.the molecules of a gas average kinetic energy is constant as
are all identical and very small in size D.the temperature is constant.
number of molecules of a gas increases with 22. When a fixed of an ideal gas expands at

109
Demystified Series Physics Demystified by Dr Timothy
constant temperature, which property of the gas the gas at constant temperature. The unit of the
molecules increases? A.Average number per unit slope is A.force B.force/m C.work D.force/m2
volume B.Average kinetic energy C.Average E.energy/m2.
speed D.Average separation. The slope is equal to work or energy and it unit is
Expansion of gas at constant temperature Joule.
increases the volume and decreases the pressure 28. Which of the following gas laws is equivalent
by Boyle’s law. The average separation between to the workdone? A.Pressure law B.Van der
the gas molecules increases as the volume waal’s law C.Charles’ law D.Boyle’s law .
increases. The average number per unit volume Boyle’s law constant is equivalent to workdone
and number of collision per second increases or energy , 𝑃𝑉 = 𝐾 = W.
with pressure The average speed and average 29. A product of 𝑃𝑉 where 𝑃 is pressure and 𝑉 is
kinetic energy are constant as temperature is the volume has the same unit as A.energy
constant. B.power C.acceleration D.force.
23. As the average molecular speed of a fixed 30. A gas with initial volume 2×10-6m3 is allowed
volume of a gas at a very low pressure increases, to expand to six times its initial volume at
its A.pressure increases B.pressure decreases constant pressure of 2×105Nm-2, what’s the
C.temperature decreases D.temperature remains workdone? A.12.0J B.4.0J C.2.0J D.1.2J
constant E.pressure remains constant. 𝑃=2×105Nm-2, 𝑉1 =2×10-6m3, 𝑉2 =6𝑉1 ,
Increase in the average molecular speed causes 𝑊 = 𝑃∆𝑉 = 𝑃(𝑉2 − 𝑉1 ) = 𝑃(6𝑉1 − 𝑉1 ) ,
the average kinetic energy to increase. Hence, the 𝑊 = 𝑃(5𝑉1 ) = 2×105(5×2×10-6) = 2.0J.
temperature also increases. Increase in 31. A gas at a volume 𝑉𝑜 in a container at pressure
temperature at constant or fixed volume brings 𝑃𝑜 , is compressed to one fifth of its volume. What
about increase in pressure by pressure law. will be its pressure if it maintains its original
24. If a given mass of gas at constant pressure 𝑃 4
temperature 𝑇? A. 𝑜 B. 𝑃𝑜 C.𝑃𝑜 D.5𝑷𝒐 .
5 5
obeys the relation, volume is proportional to the 1
absolute temperature, the gas is said to obey 𝑃1 =𝑃𝑜 , 𝑉1 =𝑉𝑜 , 𝑃2 =?, 𝑉2 = 𝑉𝑜 , At constant
5
A.Hooke’s law B.Boyle’s law C.Charles’ law temperature, Boyle’s law applies i.e 𝑃𝑉 = 𝐾.
1
E.Newton’s law. 𝑃1 𝑉1 = 𝑃2 𝑉2 , 𝑃𝑜 × 𝑉𝑜 = 𝑃2 × 𝑉𝑜 ,
5
Charles’ law gives the relation betweenthe 5𝑃𝑜 𝑉𝑜
𝑃2 = = 5𝑃𝑜 , five times the initial value.
volume and temperature of a fixed mass of gas , 𝑉𝑜
𝑉 ∝ 𝑇. 32. The volume of an air bubble increases from
25. A ballon inflated with helium gas at ground the bottom to the top of a lake at constant
level is released. As it rises through a constant temperature because A.atmospheric pressure
temperature atmosphere A.its pressure reduces acts on the surface of the lake B.pressure
but volume remains constant B.the product of increases with depth C.density increases with
pressure and volume remains constant C.both depth C.density remains constant with pressure
pressure and volume increase D.its pressure D.the bubble experiences an upthrust.
remains constant but volume decreases E.both Increase in the volume of air bubble from the
pressure and volume remains constant. bottom of the lake to the top proportionately
The pressure of the inflated ballon decreases as causes a decrease in the pressure at constant
it rises i.e pressure decreases with altitude. The temperature. Pressure at the bottom of the lake
volume of the inflated balloon increases as is greater than the pressure at the top, as
pressure decreases, at constant temperature by pressure increases with depth of lake.
Boyle’s law. Also the product of the pressure and 33. An air bubble of volume 4cm3 if formed 20cm
volume is constant according to Boyle’s law i.e under water. What will be its volume when it
1
𝑃 ∝ , 𝑃𝑉 =K. rises to just below the surface of the water, if the
𝑉 atmospheric pressure is equivalent to height of
26. If the volume of a fixed mass of gas is kept
10cm of water? A.2cm3 B.4cm3 C.6cm3 D.8cm3
constant, the pressure of the gas A.is inversely
E.12cm3.
proportional to its centigrade temperature B.is
Atmospheric pressure 𝐻=10cmH2O, depth of
inversely proportional to its absolute
water ℎ=20cm, pressure just below the surface
temperature C.is directly proportional to its
of water 𝑃1 =𝐻, volume of air bubble just below
centigrade temperature D.is directly
the surface of water 𝑉1 =?, pressure at the bottom
proportional to its absolute temperature
of water 𝑃2 =𝐻 + ℎ, volume of air bubble at the
E.remaibs constant also.
bottom of water 𝑉2 =4cm3,
The pressure of a fixed mass of gas is directly
Boyle’s law – 𝑃1 𝑉1 = 𝑃2 𝑉2 , (𝐻)𝑉1 = (𝐻 + ℎ)𝑉2 ,
proportional to its absolute kelvin temperature
10× 𝑉1 = (10+20)×4, 10× 𝑉1 =30×4=120,
at constant volume by pressure law , 𝑃 ∝ 𝑇. 120
27. In a gas experiment, the pressure of the gas is 𝑉1 = = 12cm3.
10
plotted against the reciprocal of the volume of 34. An air bubble rises from the bottom to the top

110
Demystified Series Physics Demystified by Dr Timothy
of a water dam which is 40m deep. The volume of of volume 1.0m3 at pressure of 2.0 atmospheres.
the bubble just below the surface is 2.5cm3. Find The piston of the cylinder is moved inwards,
its volume at the bottom of the dam, if reducing the volume of the gas to 0.50m3. What is
atmospheric pressure is equivalent to 10m of the pressure of the gas in this case if the
water A.10.0cm3 B.2.0cm3 C.1.6cm3 D.0.625cm3 temperature is constant? A.1.0 atmosphere B.1.5
E.0.5cm3. atmospheres C.2.0 atmospheres D.3.0
𝐻=10m, ℎ=40m, 𝑃1 =𝐻, 𝑉1 =2.5cm3, 𝑃2 =𝐻 + ℎ, atmospheres E.4.0 atmospheres.
𝑉2 =?, Boyle’s law – 𝑃1 𝑉1 = 𝑃2 𝑉2 , 𝑃1 =2.0atm, 𝑉1 =1.0m3, 𝑃2 =?, 𝑉2 =0.50m3,
(𝐻)𝑉1 = (𝐻 + ℎ)𝑉2 , 10×2.5= (10+40)𝑉2 , Boyle’s law applies at constant temperature,
25
25=50𝑉2 , 𝑉2 = = 0.5cm3. 𝑃1 𝑉1 = 𝑃2 𝑉2 , 2.0×1.0= 𝑃2 ×0.50,
50 2.0×1.0
35. An air bubble of radius 3cm rises from a 𝑃2 = = 4.0atm.
0.50
depth of 7.0m in a swimming pool. What will be 38. An air column 10cm in length
the radius of the bubble just before it reaches the is trapped into the sealed end
water surface? (Assume that the water 15cm of a capillary tube by a 15cm
temperature is uniform, atmospheric Hg column of mercury with the
pressure=10.34 metres of water) A3.6cm 10cm tube held vertically as shown
B.7.1cm C.5.0cm D.5.3cm. Air above. On inverting the tube, the
Air bubble is spherical. The volume of a sphere is air column becomes 15cm long. What is the
4
given by – 𝜋𝑟 3 , 𝑟=radius. 𝑃1 =10.34mH2O, 𝑟1 =?, atmospheric pressure during this experiment?
3
4 A.90cm B.76cm C.75cm D.60cm.
𝑉1 = 𝜋𝑟1 3 , 𝑃2 =10.34+7 =17.34mH2O, 𝑟2 =3cm, 𝑉2 =
3
4 4
𝜋𝑟2 3 = 𝜋 ×33, 𝑉2 =36𝜋cm3, 𝑃1 𝑉1 = 𝑃2 𝑉2 , 15cm
3 3
4 15cm
10.34× 𝜋𝑟1 = 17.34×36𝜋 =624.24𝜋,
3
3 15cm
3×624.24 3
𝑟1 3 = =45.28, 𝑟1 = √45.28 , 10cm
10.34×4
𝑟1 = 3.56cm = 3.6cm3 .
36. A narrow, uniform glass tube, closed at one I.Vertical II.Inverted
end, contains air which is sealed by a thread of Atmospheric pressure 𝐻=?, length of mercury
mercury. The tube is placed first horizontally and thread ℎ=15cm, length of air column 𝑙,
then vertically with the open end at the top. 1st diagram – 𝑃1 =𝐻 + ℎ=𝐻 +15, 𝑉1 =𝑙1 =10cm,
Which of the following statements is correct? 2nd diagram – 𝑃2 =𝐻 − ℎ=𝐻 −15, 𝑉2 =𝑙2 =15cm,
A.The length of the entrapped air in the 𝑃1 𝑉1 = 𝑃2 𝑉2 , (𝐻 +15)10 = (𝐻 −15)15,
horizontal position is less than that in the vertical 10𝐻 +150=15𝐻 − 225, 15𝐻 −10𝐻 =150+225,
375
position B.The length of the entrapped air in 5𝐻 = 375, 𝐻 = = 75cm.
5
the vertical position is less than that in the 39. 𝑃1 In the figure above, a thread
horizontal position C.The pressure of the of mercury 76cm long is used
entrapped air is atmospheric pressure in both to trap some dry air in a capill-
cases D.The length of the mercury thread 76cm ary tube of uniform cross-sec-
becomes shorter in the horizontal position than tion closed at one end.
in the vertical position E.The product of the 𝑃2 Calculate the ratio of the
pressure and the volume of the entrapped air is pressure 𝑃2 /𝑃1 [Atmospheric
not the same in both cases. pressure is 76cm of mercury] A.2 B.1 C.0.5 D.0.2.
The pressure of the entrapped air in the Atmospheric pressure 𝐻=76cmHg, length of
horizontal position is atmospheric pressure only mercury thread ℎ=76cmHg, 𝑃1 is the atmospheric
while the pressure of the entrapped air in the pressure, 𝑃1 =𝐻, 𝑃2 is the pressure exerted by the
vertical position is the atmospheric pressure and trapped air on the mercury thread, 𝑃2 = 𝐻 + ℎ,
the pressure due to the column of mercury 𝑃2 𝐻+ℎ 76+76 152
= = = = 2.
thread. Hence, pressure in the vertical position is 𝑃1 𝐻 76 76
greater than the pressure in the horizontal 40. The diagram below shows a horizomtal tube
position. The length of the entrapped air i.e the containing a column of mercury 76cm long.
volume of the entrapped air as 𝑉 ∝ 𝑙, in the Calculate the ratio of 𝑃2 /𝑃1 .
vertical position is less than in the horizontal
position by Boyle’s law. The length of the 𝑃2 𝑃1
mercury thread is always constant. The product 76cm
of the pressure and the volume of the entrapped (Atmospheric pressure is 76cm of mercury) A.4
air is the same or constant in both cases by B.3 C.2 D.1.
Boyle’s law. Atmospheric pressure 𝐻=76cmHg, length of
37. Some quatity of a gas is trapped in a cylinder mercury column or thread ℎ=76cmHg, 𝑃1 is the
atmospheric pressure, 𝑃1 =𝐻, 𝑃2 is the pressure
111
Demystified Series Physics Demystified by Dr Timothy
exerted by the trapped air on the mercury thread. The pressure initially acting on both arms are the
In the horizontal position pressure exerted by same, as the liquid level in both arms are the
the trapped air is equal to the atmospheric same. The arm above 𝑌 is exposed to the
𝑃 𝐻
pressure. Hence. 𝑃2 =𝐻, 2 = =1:1 or 1. atmosphere, hence the pressure acting on that
𝑃1 𝐻
arm is atmospheric pressure.
41. A little quantity of air is trapped on top of the
Atmospheric pressure 𝐻=75cmHg, 𝑃1 =𝐻=
mercury column in a barometer. When the
75cmHg, 𝑉1 =30cm3, on pouring mercury of 15cm
volume of the trapped air is 7.5cm3 and the
into the limb above 𝑋, the pressure increases.
atmospheric pressure is 76cm, the barometer
𝑃2 =75+15=90cmHg, 𝑉2 =?, 𝑃1 𝑉1 = 𝑃2 𝑉2 ,
reads 74.0cm. When the volume of the trapped 75×30
air is 10cm3 and the barometer reads 73.0cm, the 75×30=90× 𝑉2 , 𝑉2 = = 25cm3.
90
correct atmospheric pressure is A.75.7cm 44. A certain pump has a barrel whose volume is
B.75.0cm C.74.5cm D.74.0cm. 300cm3 and it is being used to exhaust a vessel of
7.5cm3 10cm3 volume 3dm3 which contains air initially at a
pressure of 71.50cm of mercury. Assuming that
74.0cm 76cm 73.0cm 𝐻 the operation takes place at a constant
temperature,what will be the pressure in the
vessel after one exhaust stroke of the pump
A.7.15cmHg B.65.0cmHg C.79.44cmHg
D.715.0cmHg E.786.5cmHgInitial pressure of air
𝐻=76cm, ℎ=74.0cm, 𝐻=?, ℎ=73.0cm, in thevessel 𝑃1 =71.50cm of mercury=7150cmHg,
𝑃1 =𝐻 − ℎ=76−74.0, 𝑃2 =𝐻 − ℎ=𝐻 −73.0, volume of vessel to be exhausted 𝑉1 =3dm3=
𝑃1 =2cm, 𝑉1 =7.5cm3, 𝑉2 =10cm3, 3000cm3, volume of pump 𝑉𝑃 =300cm3, 𝑉2 =𝑉1 +
𝑃1 𝑉1 = 𝑃2 𝑉2 , 2×7.5= (𝐻 −73)10, 𝑉𝑃 =3000+300=3300cm3, number of strokes 𝑛=1,
15=10𝐻 − 730, 10𝐻 = 15+730=745, 𝑃1 𝑉1 𝑛 = 𝑃2 𝑉2 𝑛 , 𝑃1 𝑉1 𝑛 = 𝑃2 (𝑉1 + 𝑉𝑝 )𝑛 ,
𝐻=
745
= 74.5cm. 71.5 × (3000)1 = 𝑃2 ×(3000+300)1,
10 214500
42.As a result of air at the top of a barometer, the 214500 = 𝑃2 × 3300, 𝑃2 = = 65cmHg.
3300
height of the mercury column is 73.5cm when it 45. A fixed mass of gas occupying a certain
should be 75.0cm, the volume of the space above volume has its pressure reduced to 25 percent of
the mercury is 8cm3. Calculate the correct its original value while the temperature is
barometric height when the barometer reads maintained constant. The ratio of the new
74.0cm and the volume of the space above volume to the original volume is A.2:1 B.1:2 C.1:4
mercury is 6.0cm3 A.72.0cm B.74.5cm C.75.1cm D.4:1.
D.76.0cm 25 1
𝑃1 =𝑃, 𝑃2 =25%𝑃= 𝑃 = 𝑃, 𝑉2 :𝑉1 =?, 𝑃1 𝑉1 = 𝑃2 𝑉2
100 4
8cm3 6cm3 1 𝑉2 4×𝑃
𝑃 × 𝑉1 = 𝑃 × 𝑉2 , = = 4 or 4:1.
4 𝑉1 𝑃
73.5cm 75.0cm 74.0cm 𝐻 46. A fixed quantity of gas is subjected to various
pressure 𝑃 and the corresponding volume 𝑉 is
measured at constant temperature. Which of the
following graphs best represents the results?
𝑃 I. 𝑃 II. 𝑃 III.
𝐻=75.0cm, ℎ=73.5cm, 𝐻=?, ℎ=74.0cm,
𝑃1 =𝐻 − ℎ=75.0−73.5, 𝑃2 =𝐻 − ℎ=𝐻 −74.0,
𝑃1 =1.5cm, 𝑉1 =8cm3, 𝑉2 =6cm3, 𝑉 1/𝑉 𝑉
𝑃1 𝑉1 = 𝑃2 𝑉2 , 1.5×8= (𝐻 −74.0)6, 𝑃 IV.
12=6𝐻 − 444 , 6𝐻 = 12+444 = 456, A.I, II B.II, III C.II, IV D.III, IV.
456
𝐻= = 76.0cm.
6
𝑉
43. In the J-tube below, 𝑌 and 𝑋 are on the same
47. At constant temperature the density of a fixed
horizontal level and 30cm3 of air is trapped
mass of gas is A.consatbt with temperature
above 𝑌 when the atmospheric pressure is
B.proportional to its volume C.inversely
75cmHg. Calculate the volume of air trapped
proportional to its temperature
above 𝑌 when 15cmHg is now poured into the
D.independent of its volume.
limb above 𝑋.
According to Charles’ law, volume is directly
proportional to absolute temperature at constant
𝑋 𝑌
pressure i.e 𝑉 ∝ 𝑇.
𝑚𝑎𝑠𝑠 𝑚
Density 𝜌 = = , at a fixed or constant
𝑣𝑜𝑙𝑢𝑚𝑒 𝑉
1 1
A.15cm3 B.25cm3 C.35cm3 D.45cm3. mass, 𝜌 ∝ . Hence 𝜌 ∝ i.e density is inverlsey
𝑉 𝑇

112
Demystified Series Physics Demystified by Dr Timothy
proportional to its absolute temperature. 𝜃=273−0=273℃, volume at 0℃ 𝑉𝑜 =273cm3,
48. The equation 𝑃 𝑥 𝑉 𝑦 𝑇 𝑧 = constant is Charles’ volume at 𝜃 𝑉𝜃 =?,
law, when A.𝑥=1, 𝑦=−1, 𝑧=1 B.𝒙=0, 𝒚=1, 𝒛=−1 𝛾=
𝑖𝑛𝑐𝑟𝑒𝑎𝑠𝑒 𝑖𝑛 𝑣𝑜𝑙𝑢𝑚𝑒 𝑓𝑟𝑜𝑚 0℃ 𝑉 −𝑉
= 𝜃 𝑜,
C.𝑥=1, 𝑦=0, 𝑧=1 D.𝑥=0, 𝑦=1, 𝑧=1. 1
𝑣𝑜𝑙𝑢𝑚𝑒 𝑎𝑡 0℃×𝑡𝑒𝑚𝑝. 𝑟𝑖𝑠𝑒
𝑉 −273
𝑉𝑜𝜃
273×273
𝑉
Charles’ law – 𝑉 ∝ 𝑇, = constant. It can also be = 𝜃 , 𝑉𝜃 − 273 = = 273,
273 273×273 273
𝑇
𝑃0 𝑉 1 𝑉𝜃 = 273+273 = 546cm . 3
written as 1 or 𝑃 𝑉 1 𝑇 −1 = constant.
0
55. A uniform capillary tube of negligible
𝑇
𝑃 𝑥 𝑉 𝑦 𝑇 𝑧 = 𝑃0 𝑉 1 𝑇 −1 , 𝑥=0, 𝑦=1, 𝑧=−1. expansion is sealed at one end, contains air
49. A fixed mass of gas of volume 546cm3 at 0℃ trapped by a pellet of mercury. The trapped air
is heated at constant pressure. What is the column is 13.7cm long at 0℃ and 18.7cm long at
volume of the gas at 2℃. A.1092cm3B.550cm3 100℃.Calculate the cubical expansivity of the air
C.548cm3 D.542cm3 E.273cm3 at constant pressure A.14.56×10-3𝐾 −1
𝑉1 =546cm3, 𝑇1 =0℃=0+273=273𝐾, 𝑉2 =?, B.10.95×10 𝐾 -3 −1
C.7.30×10 𝐾 −1 D.3.65×10-
-3

𝑇2 =2℃=2+273=275𝐾, Charles’ law holds at 3𝑲 −𝟏


.
𝑉
constant pressure – 1 = 2 ,
𝑉 546 𝑉
= 2 , Length at 0℃ 𝑙𝑜 =13.7cm, length at 𝜃 𝑙𝜃 =18.7cm,
𝑇1 𝑇2 273 275
𝑉 temperature rise 𝜃=100-0=100℃,
2 = 2 , 𝑉2 = 2×275 = 550cm3. 𝑙 −𝑙 18.7−13.7
275 Cubic expansivity 𝛾 = 𝜃 𝑜 = ,
50. At what temperature will the volume of a 5
𝑙𝑜 𝜃 13.7×100

given mass of air be 3 times its volume at 0℃ 𝛾= = 3.65×10-3𝐾 −1 .


1370
when the pressure is kept constant? A.546.0℃ 56. Which of the following graps is consistent
B.273.0℃ C.136.0℃ D.3℃ E.−273.0℃. with Charles’ law?
𝑉 𝑉
𝑉1 =𝑉, 𝑉2 =3𝑉, 𝑇1 =0℃=273𝐾, 𝑇2 =?, 1 = 2 , A. 𝑉 B. 𝑉
𝑇1 𝑇2
𝑉 3𝑉 3𝑉×273
= , 𝑇2 = = 819𝐾,
273 𝑇2 𝑉
𝑇2 = 819−273 =546.0℃. 𝑇℃ 𝑇℃
51. A gas occupies a certain volume at 27℃. At C. 𝑉 D. 𝑽
what temperature will its volume be doubled
assuming that its pressure remains constant?
A.54℃ B.60℃ C.108℃ D.327℃ E.600℃. 1/𝑇℃ 𝑻℃ .
𝑉1 =𝑉, 𝑉2 =2𝑉, 𝑇1 =27℃=27+273=300𝐾, 𝑇2 =?, 57. If the volume of a fixed mass of a gas is kept
𝑉1 𝑉
= 2,
𝑉 2𝑉
= , 𝑇2 =
2𝑉×300
=600𝐾, constant, the pressure of the gas A.is inversely
𝑇1 𝑇2 300 𝑇2 𝑉
proportional to its centigrade B.is inversely
𝑇2 = 600−273 = 327℃. proportional to its absolute temperature C.is
52. Dry hydrogen is trapped by pellet of mercury directly proportional to its centigrade
in a uniform capillary tube closed at one end. If temperature D.is directly proportional to its
the column of hydrogen at 27℃ is 1.0m, at what absolute temperature E.remains constant also.
temperature will the length be 1.20m? A.22.5℃ 58. A fixed mass of air at s.t.p is heated in such a
B.32.4℃ C.77.0℃ D.87.0℃ E.360.0℃. way that its volume remains unchanged.
𝑉 ∝ 𝑙 , as the capillary tube is of uniform cross- Calculate the temperature if its pressure is
sectional area. 𝑉1 =𝑙1 =1.0m, 𝑇1 =27℃, increased to 114cm of mercury A.0℃ B.1.5℃
𝑇1 =27+273=300𝐾, 𝑉2 =𝑙2 =1.2m, 𝑇2 =?, C.109.2℃ D.136.5℃ E.273℃.
𝑉1 𝑉 1 1.2
= 2, = , 𝑇2 = 1.2×300 = 360𝐾 , 𝑃1 =s.p=76cmHg, 𝑇1 =s.t=273𝐾, 𝑃2 =114cmHg,
𝑇1 𝑇2 300 𝑇2
𝑃 𝑃 76 114 114×273
𝑇2 = 360−273 = 87℃. 𝑇2 =?, 1 = 2 , = , 𝑇2 = ,
𝑇1 𝑇2 273 𝑇2 76
53. A column of air 10.0cm long is trapped in a 𝑇2 = 409.5𝐾, 𝑇2 = 409.5−273 = 136.5℃.
tube at 27℃. What is the length of the column at 59. The tyre of a car was found to be 150cmHg in
100℃? A.12.4cm B.13.7cm C.18.5cm D.37.0cm. the morning at a temperature of 27℃. Hard
𝑉 ∝ 𝑙 , as the capillary tube is of uniform cross- driving in the afternoon raised the temperature
sectional area. 𝑉1 =𝑙1 =10.0m, 𝑇1 =27℃, of the tyres to 57℃. The tyre pressure had by
𝑇1 =27+273=300𝐾, 𝑉2 =?, 𝑇2 =100℃=100+273, afternoon A.increased by 15.0cmHg
𝑉 𝑉 10.0 𝑉 373
𝑇2 =373𝐾, 1 = 2 , = 2 , 𝑉2 = = 12.4cm. B.decreased by 15cmHg C.increased by
𝑇1 𝑇2 300 373 30
54. The cubic expansivity of a certain gas at 22.50cmHg D.decreased by 22.50cmHg
constant pressure is
1
𝐾 −1 . If a given mass of E.remains constant.
273 𝑃1 =150cmHg, 𝑇1 =27℃=27+273=300𝐾, 𝑃2 =?,
the gas is held at constant pressure and its 𝑃 𝑃
volume at 0℃ is 273m3, determine the volume of 𝑇2 =57℃=57+273=330𝐾, 1 = 2 ,
𝑇1 𝑇2
the gas at 273℃ A.273m3B.546m3 C.819m3 150 𝑃
= 2 , 𝑃2 =
150×330
= 165cmHg, The tyre
300 330 300
D.1092m3.
1 pressure is increased by 15cmHg i.e 𝑃2 − 𝑃1 =
Cubic expansivity 𝛾= 𝐾 −1 , temperature rise 165−150 = 15cmHg.
273

113
Demystified Series Physics Demystified by Dr Timothy
60. The pressure of a fixed mass of an ideal gas at 65. A quantity of gas occupies a certain volume
27℃ is 3Pa. Tha gas is heated at a constant when the temperature is −73℃and the pressure
volume until its pressure is 5Pa. Determine the is 1.5 atmospheres. If the pressure is increased to
mew temperature of the gas A.100℃ B.227℃ 4.5 atmospheres and the volume is halved at the
C.273℃ D.500℃ same time, what will e the new temperature of
𝑃1 =3Pa, 𝑉1 =27℃=27+273=300℃, 𝑃2 =5Pa, 𝑇2 =?, the gas? A.573℃B.327℃ C.300℃ D.110℃E.27℃
𝑃1 𝑃
= 2,
3 5
= , 𝑇2 =
5×300
= 500𝐾, 𝑃1 =1.5atm, 𝑉1 =𝑉, 𝑇1 =−73℃=−73+273=200𝐾,
𝑇1 𝑇2 300 𝑇2 3 1 𝑃 𝑉 𝑃 𝑉
𝑇2 = 500−273 = 227℃. 𝑃2 =4.5atm, 𝑉1 = 𝑉, 𝑇2 =?, 1 1 = 2 2 ,
2 𝑇1 𝑇2
61. A motor tyre is inflated to a pressure of 1.5×𝑉 4.5× 𝑉
1
200×4.5× 𝑉
1

2.0×105Nm-2 when the temperature of air is = 2


, 𝑇2 = 2
= 300𝐾,
200 𝑇2 1.5×𝑉
27℃. What will be the pressure in it at 87℃ 𝑇2 = 300−273 = 27℃.
assuming that the volume of the tyre does not 66. A gas volume 700cm3 at a temperature of
change? A.2.6×105Nm-2 B.2.4×105Nm-2 30℃ has a pressure of 606mmHg. If the volume
C.2.2×10 Nm D.1.3×105Nm-2.
5 -2 of the gas is reduced by 300cm3 when the
𝑃1 =2.0×105𝑁m-2, 𝑇1 =27℃=27+273=300𝐾, 𝑃2 =?, temperature increases to 47℃. Calculate the
𝑇2 =87℃=87+273=360𝐾,
𝑃1 𝑃
= 2, pressure of the gas. A.280mmHg B.560mmHg
𝑇1 𝑇2
C.1120mmHg D.2240mmHg.
2.0×105 𝑃2 2.0×105 ×360
= , 𝑃2 = =2.4×105Nm-2. 𝑉1 =700cm3, 𝑇1 =30℃=30+273=303𝐾,
300 360 300
62. The pressure of a certain mass of gas at 𝑃1 =606mmHg, the volume is reduced by 300cm3,
constant volume increases from 250mmHg at 𝑉2 = 𝑉1 − 300=700−300=400cm3, temperature is
20℃ to 400mmHg at 50℃. Calculate the pressure reduced to 47℃ , 𝑇2 =47℃, 𝑇2 =47+273=320𝐾,
𝑃 𝑉 𝑃 𝑉 606×700 𝑃 ×400
expansivity of the gas. A.0.05℃−1 B.0.04℃−1 𝑃2 =?, 1 1 = 2 2 , = 2 ,
𝑇1 𝑇2 303 320
C.0.03℃−1 D.0.02℃−𝟏 . 606×700×320
𝑃2 = = 1120mmHg.
Pressure at 20℃ 𝑃𝑜 =250mmHg, pressure at 50℃ 303×400
𝑃𝜃 =400mmHg, temperature rise 𝜃=50−20=30℃, 67. The pressure of a given mass of a gas changes
pressure expansivity 𝛽=?, from 300Nm-2 to 120Nm-2 while the temperature
𝑃 −𝑃 400−250 150 drops from 127℃ to −73℃. What is the ratio of
𝛽= 𝜃 𝑜= = = 0.02℃−1 .
𝑃𝑜 𝜃 250×30 7500 the final volume to the initial volume? A.2:5
63. The pressure 𝑃, the volume 𝑉, and the B.5:4 C.5:2 D.4:5.
temperature 𝑇, of a given mass of air change 𝑃1 =300Nm-2, 𝑇1 =127℃=127+273=400𝐾,
simultaneously. Which of the following equations 𝑃2 =120Nm-2, 𝑇2 =−73℃=−73+273=200𝐾, the
𝑐𝑜𝑛𝑠𝑡𝑎𝑛𝑡 𝑃
is correct? A. 𝑃𝑉 = B. = 𝑐𝑜𝑛𝑠𝑡𝑎𝑛𝑡 × 𝑉 ratio of the final volume to initial volume 𝑉2 :𝑉1 =?,
𝑇 𝑇 𝑃1 𝑉1 𝑃 𝑉 300×𝑉1 120×𝑉2 𝑉 300×200
C. =
𝑃 𝑐𝑜𝑛𝑠𝑡𝑎𝑛𝑡
D. =
1 1 𝑽
E. =
𝒄𝒐𝒏𝒔𝒕𝒂𝒏𝒕
. = 2 2, = , 2= ,
𝑉 𝑇 𝑇 𝑃𝑉 𝑻 𝑷 𝑇1 𝑇2 400 200 𝑉1 400×120
𝑃𝑉 𝑉2 5
General gas law – = constant, Rearranging = or 5:4.
𝑇 𝑉1 4
the equation, = .
𝑉 𝑐𝑜𝑛𝑠𝑡𝑎𝑛𝑡 68. A given mass of ideal gas occupies a volume
𝑇 𝑃
𝑉 at a temperature 𝑇 and under a pressure 𝑃. If
64. When the volume of a given gas is halved and
the pressure is increased to 2𝑃 and the
its temperature doubled, the pressure A.remains 1
constant B.increased by a factor of 4 temperature reduced to 𝑇, then what is the
2
C.increases by a factor of 3 D.decreased by a percentage change in the volume of the gas?
factor is 3. A.0% B.25% C.75% D.300% E.400%.
1 1
𝑃1 =𝑃, 𝑉1 =𝑉, 𝑇1 =𝑇, 𝑃2 =?, 𝑉2 = 𝑉, 𝑇2 =2𝑇, 𝑃1 =𝑃, 𝑉1 =𝑉, 𝑇1 =𝑇, 𝑃2 =2𝑃, 𝑉2 =?, 𝑇2 = 𝑇,
2 2
1 1
𝑃1 𝑉1 𝑃2 𝑉2 𝑃×𝑉 𝑃2 ×2𝑉 𝑃×𝑉×2𝑇 𝑃1 𝑉1 𝑃2 𝑉2 𝑃×𝑉 2𝑃×𝑉2 𝑃×𝑉×2𝑇
= , = , 𝑃2 = 1 , = , = 1 , 𝑉2 = ,
𝑇1 𝑇2 𝑇 2𝑇 𝑇× 𝑉 𝑇1 𝑇2 𝑇 𝑇 𝑇×2𝑃
2 2
𝑃2 = 𝑃 × 2 × 2 = 4𝑃 , four times its initial value 1
𝑉2 = 𝑉 = 0.25𝑉. OR
𝑃𝑉
= constant. 𝑃 is doubled
4 𝑇
or increased by a factor of 4. Alternatively, 1 𝑇 1/2
𝑃𝑉 1 i.e 2, 𝑇 is halved i.e hence, 𝑉 = = =
= constant. 𝑉 is halved i.e , 𝑇 is douled i.e 2, 1 1 1
2 𝑃 2
𝑇 2
𝑇 2 × = = 0.5 i.e 𝑉2 = 0.5𝑉.
hence 𝑃 = = = 2×2=4 i.e four times its 2 2 4
𝑉1 −𝑉2
𝑉 1/2
Percentage change in volume %∆𝑉 = × 100
initial value or increased by a factor of 4. 𝑉1
𝑉−0.5𝑉 0.75𝑉
64. The volume of a given mass of gas 27℃and %∆𝑉 = × 100= × 100 = 75%.
𝑉 𝑉
800mmHg is 76cm3. Calculate its volume at s.t.p 69. At 27℃ and 2 atmospheres pressure, the
A.100.0cm3 B.72.8cm3 C.60.0cm3 D.36.4cm3. density of a gas is 0.40g per litres. What is its
𝑃1 =800mmHg, 𝑉1 =76cm3, 𝑇1 =27℃=27+273, density at 4 atmospheric pressure and 227℃.
𝑇1 =300𝐾, 𝑃2 =s.p=760mmHg, 𝑉2 =?, 𝑇2 =s.t=273𝐾, A.0.12g per litre B.0.24g per litre C.0.36g per litre
𝑃1 𝑉1 𝑃 𝑉 800×76 760×𝑉2 800×76×273
= 2 2, = , 𝑉2 = , D.0.48g per litre
𝑇1 𝑇2 300 273 300×760
𝑉2 = 72.8cm3. 𝑃1 =2atm, 𝑇1 =27℃=27+273=300𝐾,𝜌1 =0.40g per

114
Demystified Series Physics Demystified by Dr Timothy
litre, 𝑃2 =4atm, 𝑇2 =227℃=227+273, 𝑇2 =500𝐾,
𝑃𝑉 𝑚𝑎𝑠𝑠 𝑚
𝜌2 =?, = constant, density= , 𝜌= ,
𝑇 𝑣𝑜𝑙𝑢𝑚𝑒 𝑉
1 1 𝑷
𝜌∝ or 𝑉 = . Hence, = constant,
𝑉 𝜌 𝑻𝝆
𝑷𝟏 𝑷𝟐 2 4 300×0.40×4
= , = , 𝜌2 = ,
𝑻𝟏 𝝆𝟏 𝑻𝟐 𝝆𝟐 300×0.40 500×𝜌2 2×500
𝜌2 = 0.48g per litre.

Jamb past questions on Pressure in solids and


fluids :
[1978/8,1979/22,1981/5,1982/1,21,1983/9,37
,50,1985/14,15,1986/22,1988/15,1989/11,
1990/13,1991/1,13,1992/11,1993/10,1994/15
,17,1995/16,1997/18,1998/14,15,1999/12,
2000/6,2001/2,2002/8,2004/15,32,2005/40,
2006/50,2007/38,2008/16,17,2009/14,2010/
12,2011/10,2012/17,2013/14]

Jamb past questions in gas laws and kinetic


theory :
[1978/20,1979/17,38,1980/24,33,1981/6,36,
1982/5,41,1983/14,1984/28,1985/20,21,26,
1986/24,1987/19,20,1988/16,1989/14,15,17,
1990/17,1991/18,21,1992/16,1993/16,1994/
21,26,1995/17,21,1997/22,1998/19,23,1999/
10,13,2001/16,19,2002/18,2005/44,47,2006/4
7,2007/29,33,37,2008/21,2010/22,2011/19,24
,2012/21,2013/18,2014/21,2015/12,2016/11]

115
Demystified Series Physics Demystified by Dr Timothy

Chapter 11 – Temperature and its measurement


● Heat is a measure of the total internal energy expression below is used –
𝒕𝜽𝑷 −𝒕𝑳𝑷
=
𝒕𝜽𝑸 −𝒕𝑳𝑸
OR
of a body. It is a form of energy transferred from 𝒕𝑼𝑷 −𝒕𝑳𝑷 𝒕𝑼𝑸 −𝒕𝑳𝑸
one body to another due to difference in 𝒕𝜽𝑷 −𝒕𝑳𝑷
=
𝒕𝜽𝑸 −𝒕𝑳𝑸
, 𝑡𝜃𝑃 =temperature in scale 𝑃,
temperature between bodies. 𝑭.𝑰𝑷 𝑭.𝑰𝑸
● Temperature is the degree of hotness or 𝑡𝐿𝑃 =lower fixed point or ice point of scale 𝑃,
coldness of a body. It is a measure of the average 𝑡𝑈𝑃 =upper fixed point or ice point of scale 𝑃,
kinetic energy of the molecules of a body. 𝑡𝜃𝑄 =temperature in scale 𝑄, 𝑡𝐿𝑄 =lower fixed
Temperature determines the direction of heal point or ice point in scale 𝑄, 𝑡𝑈𝑄 =upper fixed
flow between two bodies in contact.
point or steam point in scale 𝑄,
- Temperature is accurately measured by a
𝐹. 𝐼𝑃 =fundamental interval of scale 𝑃,
thermometer.
𝐹. 𝐼𝑄 =fundamental interval of scale 𝑄.
- The substance whose physical property is used
in determining temperature is called a - Qualities of a good thermometer – A good
thermometer must have: low thermal capacity or
thermometric substance.
high heat conductivity to measure wide
- Fixed temperature – All thermometers have
two fixed points or reference points called upper temperature range, high sensitivity or short
response time, suitability i.e purpose of usage,
fixed temperature point and lower fixed
high accuracy, reproducibility i.e equivalent fixed
temperature point.
- Upper fixed temperature point or Steam points on different thermometer and consistency
point U.F.P or 𝑼 is the temperature of steam i.e agreement between different readings on the
same thermometer.
from pure boiling water at standard atmospheric
pressure of 760mmHg. Upper fixed point is - Types of thermometer, thermometric
determined using a hypsometer. property and thermometric substance used :
1.Optical or infrared thermometer – Variation
- Lower fixed temperature point or Ice point
in light intensity, colour or radiant heat with
L.F.P or 𝑳 is the temperature of a pure melting ice
at standard temperature of 760mmHg. Lower temperature. It is used for high temperature
measurement
fixed point is determined using a glass funnel and
2.Bimetallic thermometer – Change in
ice shavings or pure melting ice.
- Temperature Scales – Temperature scales are expansion(length) of two metals which forms a
bimetallic strip, with temperature. For bimetallic
used for temperature measurement. Types of 𝑳 −𝑳 𝜽−𝜽𝑳
temperature scales are namely ; Fahrenheit scale, thermometer ; 𝜽 𝑳 = .
𝑳𝑼 −𝑳𝑳 𝜽𝑼 −𝜽𝑳
Absolute or Thermodynamic or Kelvin scale, 3.Resistance thermometer – Change in
Centigrade or Celcius scale, Rhemur scale and electrical resistance of a platinum wire with
Rankin scale. temperature. For resistance thermoemeter ;
- Fundamental or Thermometric interval 𝑭. 𝑰 𝑹𝜽 −𝑹𝑳
=
𝜽−𝜽𝑳
.
is the difference or interval between the upper 𝑹𝑼 −𝑹𝑳 𝜽𝑼 −𝜽𝑳
fixed point (U.F.P) and lower fixed point (L.F.P) 4.Thermoelectric thermometer – Change in
of a scale. electromotive force E.M.F between two joined
1.Fahrenheit scale – U.F.P is 212℉, L.F.P is 32℃, dissimilar metals e.g copper and silver, which
fundamental interval 𝐹. 𝐼 is 180℉. forms a thermocouple, with temperature. The
2.Absolute or Themodynamic or Kelvin scale variation of E.M.F with temperature is referred to
– U.F.P is 373𝐾, L.F.P is 273𝐾, fundamental as See-beck effect. Thermoelectric
interval 𝐹. 𝐼 is 100𝐾 thermometer is the most convenient
3.Centigrade or Celcius scale– U.F.P is 100℃, thermometer for measuring high and
L.F.P is 0℃, fundamental interval 𝐹. 𝐼 is 100℃. varying(or rapidly changing) temperature e.g
4.Rhemur scale – U.F.P is 80°𝑅, L.F.P is 0°𝑅, temperature of the wings of an airplane. For
𝑬 −𝑬 𝜽−𝜽𝑳
fundamental interval 𝐹. 𝐼 80°𝑅. Thermoelectric thermometer ; 𝜽 𝑳 = .
𝑬𝑼 −𝑬𝑳 𝜽𝑼 −𝜽𝑳
5.Rankin scale – U.F.P is 672°𝑅, L.F.P is 492°𝑅, 5.Gas or Absolute thermometer – Change or
fundamental interval 𝐹. 𝐼 is 180°𝑅. variation in pressure at constant volume i.e
- A temperature change of 1℃ is the same as constant volume gas thermometer, or change or
1K change in temperature, as they have the varaition in volume at constant pressure i.e
same fundamental interval of 100. constant pressure gas thermometer.It is very
- A temperature change of 1℃ is the same as sensitive and accurate, hence used for more
9/5℉. accurate temperature measurement. All
- In conversion of temperature scale 𝑃 to another thermometer are calibrated against gas
temperature scale 𝑄 and vice-versa the thermometer or used as standard for

116
Demystified Series Physics Demystified by Dr Timothy
calibrating other thermometer. Thrmometric temperature reading.
substance is helium (widely used) and - A clinical theromometer is not sterilized in pure
hydrogen. For gas thermometer ; boiling water at normal atmospheric pressure of
𝑷𝜽 −𝑷𝑳
=
𝜽−𝜽𝑳
(constant volume gas 760mmHg because at 760mmHg water boils at
𝑷𝑼 −𝑷𝑳 𝜽𝑼 −𝜽𝑳
𝑽𝜽 −𝑽𝑳 𝜽−𝜽𝑳
100℃ and the range of temperature of a clinical
thermometer), = (constant pressure thermometer is below 100℃ and will break the
𝑽𝑼 −𝑽𝑳 𝜽𝑼 −𝜽𝑳
gas thermometer. glass due to excessive expansion of mercury.
6.Liquid-in glass thermometer – Change in - Six’s maximum and minimum thermometer
volume or length i.e expansion of liquid e.g – It is a liquid-in glass thermometer which is used
mercury or alcohol, with temperature. to measures the temperature of the weather or
- Suitability of a liquid as a thermometric the maximum and minimum temperature for a
liquid – I.The liquid must be easily seen in glass. day. It consists of two long arms region which
II.The liquid should expand or contract contains alcohol and a U-shaped arm region
uniformly with temperature. III.The liquid which contains mercury.
should have a high boiling point and low melting - The steel index in the minimum arm(left
point to ensure a wide range of temperature arm) moves upward while that in the
measurement. maximum arm (left arm) remains stationary,
- Mercury is preferred to alcohol as the when the temperature falls or at a low
thermometric liquid because – I.Mercury has temperature.
small heat capacity hence, high thermal - The steel index in the maximum arm (right
conductivity than alcohol. II.Mercury does not arm) moves upward while that in the
vapourize easily (it has a high boiling point than minimum arm (left arm) remains stationary,
alcohol). III.Mercury is opaque i.e can be easily when the temperature rises or at a high
seen, making it eeasy to read temperature while temperature.
alcohol is transparent, hence has to be coloured. - The steel indices are returned to their
IV.Mercury does not wet glass. V.Mercury original positions by a magnet.
measures a wide range and high temperature Maximum arm
due to its high boiling point(319℃). Minimum arm Air space
Alcohol
- Alcohol is preferred to mercury because –
Alcohol
I.Alcohol can measure low temperature due to its Steel index
low freezing point(-115℃) than mercury(-39℃). S2 Steel index S1
II.It expands more than mercury (alcohol Mercury
expands six times as more than mercury but not
uniformly as mercury). - Procedures for making a liquid-in-glass
- Water is not a good thermometric liquid thermometer –
because it exhibits anomalous expansion, 1.Bulb making – One end is sealed and blowed
colourless and wets glass, small range of for storage of the thermometric liquid
expansion due to small temperature interval 2.Liquid filling – The liquid is filled in steps by
(0℃ − 100℃). eliminating the air inside and allowing a large
- Factors increasing the sensitivity of a liquid quantity of liquid to move in. The filling process
in glass thermometer : I.Large and thin bulb for is done by: i.Inverting the tube in a glass jar
volume of liquid and conductivity i.e the bulb is containing the liquid with the bubl facing
made of thin glass so it can conduct heat quickly upwards or at the uppermost and heating gently.
and assume the heat of the surrounding quickly. ii.Removing the thermometer and boil the liquid
II.Thin walled stem bore and narrow tube for in the tube by heating the bulb. iii.Inverting the
visibility and capillary rise i.e the capillary tube is tube again in the glass jar form complete filling of
very narrow and of uniform bore so that small the tube.
changes in temperature can lead to large changes 3.End sealing – The thermometer is heated to a
in the length of the liquid. III.High expansivity of temperature above the maximum temperature
liquid i.e the thermometric liquid should have required and sealing of the open end.
high expansivity. - Absolute zero temperature is the
- Clinical thermometer –It is a form of liquid in temperature in the kelvin scale where thermal
glass thermometer which contains mercury as motion ceases corresponding to the complete
the thermometric liquid (a mercury in glass absence of translational motion of molecular
thermometer). It is used in hospitals for motion. It is the temperature at which the
measuring the human body temperature. It has a average kinetic energy of particles making up a
short temperature range(35℃ − 45℃), a kink body is zero. Absolute zero temperature is
or constriction (bent region) which prevents equal to 0𝑲 or −273℃.
the back flow of the liquid,therby altering the

117
Demystified Series Physics Demystified by Dr Timothy
Examples : Helium and hydrogen are used as the
1. In the study of physics, temperature and heat thermometric substance of an absolute or gas
are often confused with each other. Which of the thermometer. Alcohol and mercury are
following statements correctly defines these two thermometric substance used in liquid-in-glass
elements? A.Temperature is a measure of the thermometer. Platinum is used in a resistance
average kinetic energy of molecules B.Heat is a thermometer.
measure of the total kinetic energy of molecules 9. Which of the following statements about a gas
in a system C.Different materials require thermometer and a mercury in glass
different amounts of heat to cause a change in thermometer is correct? A.mercury
temperature D.all of the above E.A and B only. thermometer can not be calibrated against a
2. Temperature is a measure of the A.quantity of constant volume gas thermometer B.mercury
heat transferred into the molecules of an object thermometers are much more accurate than gas
B.mean kinetic energy of any individual thermometers C.gas thermometers has a wide
molecules of the object C.kinetic energy of any temperarure range than mercury
individual molecules of the substance D.amount thermometer D.gas thermometers are less
of workdone by the molecule of the object. cumbersome than mercury in glass
3. Which of the following properties cannot be thermometer.
utilized for temperature measurement? The Mercury thermometer and all other
A.Length of a liquid column B.Volume of a gas at thermometers are calibrated against a gas
constant pressure C.Pressure of a gas at constant thermometer. Gas thermometer are of higher
volume D.Resistance of a metallic wire E.Current accuracy and wide temperature range of
produced in a photoelectric effect. measurement than mercury in glass
Length of a liquid column is utilized in liquid-in - thermometer and all other thermometers. Gas
glass thermometer. Volume of a gas at constant thermometer are cumbersome hence they are
pressure and pressure of a gas at constant limited to industrial use only.
volume are utilized in gas/absolute 10. Which of the following about the constant
thermometer. Resistance of a metallic wire is volume gas thermometer are correct? The
utilized in resistance thermometer. thermometer is I.used as standard for calibrating
Photoelectricity is used for light intensity. other types of thermometer II.less convenient to
4.Which of the following physical properties is manipulate when compared with the use of the
not normally used to measure temperature? liquid in glass thermometer III.less sensitive than
Change in the A.length of a column of liquid liquid in glass thermometer A.I and II B.I and III
B.volume of a fluid C.mass of a solid D.pressure C.II and III D.I,II and III.
of a gas E.electrical resistance of a metal. Gas thermometer are more sensitive than liquid
5. The thermometric property of the in glass thermometer and are used as standard
thermocouple is that its A.pressure changes with for calibrating other themometers.
resistance B.e.m.f changes with temperature 11. I.reproducibility II.sensitivity III.high thermal
C.volume changes with temperature D.resistance capacity IV.high accuracy. The qualities of a good
changes with temperature. thermometer includes A.I,II and III B.I,II and IV
6. The variation in the thermometric property of C.I,III and IV D.II,III and IV.
an optical pyrometer depends on the A.radiant A good thermometer must be reproducible i.e
heat B.volume of the trapped gas C.pressure of gives equivalent fixed points in different
the trapped gas D.volume of the thermometric thermometers, sensitive i.e responds to
liquid. temperature changes quickly, low thermal
Optical pyrometer uses changes in light intensity, capacity or high thermal conductivity, high
radiant heat and colour for temperature accuracy i.e comparing the measured value to the
measurement. Volume of trapped gas is used in standard.
constant pressure gas thermometer, pressure of 12. I.High thermal capacity II.High sensitivity
trapped gas is used in constant volume gas III.Easy readability IV.Accuracy over a wide
thermometer. Volume of thermometric liquid is range of temperatures. From the statements
used in a liquid-in-glass thermometer. above, the qualities of a good thermometer is A.I
7. The thermometer whose reading is indicated and II B.I,III and IV C.II,III and IV D.I,II,III and IV
by a change in colour of its thermometric A good thermometer must have low thermal
property is the A.bimetallic strip thermometer capacity or high thermal conductivity to measure
B.platinum resistance thermometer C.optical a wide range of temperature, high sensitivity,
pyrometer D.thermocouple. easy readability and high accuracy over a wide
8. The thermometric substance of an absolute temperature range.
thermometer is A.alcoholB.mercury C.helium 13. The air in the stem of a mercury in glass
D.platinum thermometer should be expelled before

118
Demystified Series Physics Demystified by Dr Timothy
introducing mercury so that A.the mercury may (0℃ − 100℃).
not be oxidized B.the mercury may not evaporate 19. Which of the following features does not
C.enough mercury may enter the bulb D.the increase the sensitivity of a liquid-in-glass
mercury rise in the tue may not be subjected to thermometer? A.A large bulb B.A thick-walled
varying air pressure E.the glass of the bulb may tube C.A capillary tube with a narrow bore D.A
not crack if the mercury expands to compress the thin-walled bulb E.A liquid with a high
air. expansivity.
Air is excluded from the bulb used for storing Sensitivity of a liquid-in-glass thermometer
thermometric liquid, inorder to reduce the increases when a large and thin-walled bulb, thin
pressure and hence for more volume of the bulb capillary tube or stem with a narrow bore and a
to accommodate mercury. liquid of high expansivity.
14. Which of the following is not a reason for 20. In the construction of a liquid-in-glass
using mercury in preference to alcohol as a thermometer, the sensitivity increases if the
thermometric liquid? A.Mecury has a higher A.thermal conductivity of the liquid is low B.the
boiling point than alcohol B.Mercury has a boiling point of the liquid is high C.volume
higher expansivity than alcohol C.Mercury is a expansivity of the liquid is high D.diameter of
better conductor than alcohol D.Mercury does the capillary tube is large.
not wet glass E.Mercury is opaque. Sensitivity of a liquid-in-glass thermometer
Alcohol has a higher expansivity than mercury increases if the diameter of the capillary tube is
but the expansivity of mercury is uniform while small (thin-capillary tube) and volume
that of alcohol is irregular. expansivity of the liquid is high.
15. Which of the following properties of mercury 21. A short response time is obtained in a liquid-
is not a reason for using it as a thermometric in-glass thermometer when the A.bulb is large
liquid? A.It is visible B.It does not wet the glass and thick walled B.stem is long and thin C.bulb is
C.It allows for a wide temperature range to be small and thick-walled D.liquid is of high density
measured D.It has a high density E.It has a high and the bore is large E.bulb is thin-walled and
conductivity. the liquid is a good conductor of heat.
The density of mercury does not affects its usage A liquid-in-glass thermometer with a short
as a thermometric liquid. The high density of response time indicates a very high sensitive
mercury makes is suitable as a barometric liquid. thermometer. Sensitivity increases when the
16. Which of the following is not a reason why bulb is large and thin-walled, the stem or tube is
mercury is preferred to alcohol when used as a thin-walled and norrow bore,liquid of high
thermometric liquid? Mercury A.has a small volume expansivity and good conductivity.
thermal capacity B.does not vaporize C.is opaque 22. I.Use a liquid with a high melting point II.Use
and so easily seen D.is a good conductor of heat a liquid of high volume expansivity III.Use a
D.freezes at −39℃. capillary tube of large diameter. Which of the
Mercury is preferred to alcohol as a above best describes how the sensitivity of a
thermometric liquid because it has ; a low or liquid-in-glass thermometer can be enhanced?
small thermal capacity i.e high temperature A.I B.II C.I and III D.II and III.
range, does not vaporize easily i.e high boiling 23. Clinical thermometer are examples of
point, opaque i.e easily seen, good thermal A.constant pressure gas thermometer
conductivity. The disadvantages of mercury as a B.resistance thermometer C.alcohol
thermometric liquid are ; low expansivity i.e thermometer D.mercury-in-glass
alcohol expands more than mercury, and low thermometer.
freezing point −39℃. 24. A clinical thermometer is different from
17. One special advantage of alcohol over other mercury in glass thermometer owing to
mercury as a thermometric liquid is A.high A.its long stem B.the constriction on its stem
specific heat capacity B.low boiling point C.low C.its wide range of temperatures D.the grade of
freezing point D.low density. mercury use in it .
The low freezing point of alcohol i.e −115℃, Clinical thermometer has a constriction in its
hence makes it suitable in a thermometer for stem which prevents the mercury from falling
measuring very low temperature. back into the scale.
18. Water is a poor thermometric liquid because 25. The clinical thermometer differs from other
it A.has low vapour pressure B.is opaque C.is a mercury in glass thermometer because it has I.a
poor conductor D.wets glass. constriction II.a narrow bore III.a wide range IV.a
Water is not a good thermometric liquid because short range. Which of the above are correct A.I
it exhibits anomalous expansion (between 0℃ − and II B.I and IV C.II and III D.I,II and IV E.I,II,III
4℃), colourless and wets glass, small range of and IV.
expansion due to small temperature interval

119
Demystified Series Physics Demystified by Dr Timothy
Clinical thermometer has a constriction or kink of this scale C.the temperature of a mixture of
or bent region which prevents the back flow of ice and salt D.the temperature of pure melting
mercury into the scale thereby preventing the ice at a pressure of 760mm of mercury D.the
temperature read by it from falling back. It also freezing point of pure water at a pressure of
has a short temperature range (35℃ − 45℃). All 760mm of mercury
liquid-in-glass thermometers have narrow bore. Lower fixed point or ice point is the temperature
26. The diagram above shows a of a pure melting ice at a pressure of 760mmHg
𝑹 maximum and minimum or the freezing point of pure water at a pressure
𝑷 𝒀 thermometer divided into of 760mmHg. It is the zero mark of the Celcius
𝑿 three portions 𝑃, 𝑄 and 𝑅. scale i.e 0℃. The absolute zero in Celcius scale is
𝑸 Which of the following is −273℃, hence it is different from the lower fixed
true about the respective of poin which is 0℃.
contents of 𝑃, 𝑄 and 𝑅? A.Alcohol, mercury and 31. A hypsometer is used for determining
alcohol B.Air, alcohol and mercury C.Mercury, A.steam point B.relative humidity C.ice point
alcohol and mercury D.Air, mercury and alcohol D.relative density.
NB – The bulb above 𝑹 contains air. Hypsometer is used for determining the upper
27. fixed point or steam point.
32. The diagram below represents the process
for the determination of
Thermometer

Pure melting ice


The diagram above represents the Six’s
maximum and minimum thermometer. If the
temperature of the surrounding falls, which of
the following correctly states how the steel
indices in the thermometer would respond to the
change in temperature? A.S1 would be moved up, Water
S2 would not move B.S1 would not move but S2
would move upward C.both S1 and S2 would A.upper fixed point B.lower fixed point
move upwards D.S1 would move upwards and S2 C.melting point D.freezing point.
would move downwards E.S1 would move The diagram shows the use of a glass funnel and
downwards and S2 would move upwards pure melting ice in determining the lower fixed
When the temperature of the surrounding falls, point or ice point.
the steel index S2 in the minimum(or left) arm 33. A temperature of 20℃ is the same as A.36℉
moves upwards while the steel index S1 in the B.68℉ C.11.1℉ D.43.1℉
maximum(or right) arm would not move. 𝑡𝜃𝐶 −𝑡𝐿𝐶
=
𝑡𝜃𝐹 −𝑡𝐿𝐹
, temperature in Celcius scale
When the temperature of the surrounding rises, 𝑡𝑈𝐶 −𝑡𝐿𝐶 𝑡𝑈𝐹 −𝑡𝐿𝐹

the steel index S1 in the maximum(or right) arm 𝑡𝜃𝐶 =20℃, L.F.P in Celcius scale 𝑡𝐿𝐶 =0℃, U.F.P in
moves upwards while the steel index S2 in the Celcius scale 𝑡𝑈𝐶 =100℃, temperature in
minimum(or left) arm would not move. Fahrenheit scale 𝑡𝜃𝐹 =?, L.F.P in Fahrenheit scale
28. The absolute zero temperature is defined as 𝑡𝐿𝐹 =32℉, U.F.P in Fahrenheit scale 𝑡𝑈𝐹 =212℉,
the temperature at which A.the pressure of a real 20−0 𝑡𝜃𝐹 −32 20 𝑡𝜃𝐹 −32
gas is maximum B.the temperature of a gas is = , = ,
100−0 212−32 100 180
2
273℃ C.thermal motion ceases D.the volume 𝑡𝜃𝐹 −32= × 180=36, 𝑡𝜃𝐹 = 36+32=68℉.
10
of real gases is maximum E.ice melts. 𝐹𝜃 −𝐹𝐿
OR 𝜃 = × 100 , temperature in celcius
At absolute zero i.e 0𝐾 or −273℃, the volume of 𝐹𝑈 −𝐹𝐿
a gas is theoretically zero. scale 𝜃=20℃, L.F.P in Fahrenheit scale 𝐹𝐿 =32℉,
29. Absolute zero temperature can be defined as U.F.P in Fahrenheit scale 𝐹𝑈 =212℉, temperature
the temperature A.at which pure water and in Fahrenheit scale 𝐹𝜃 =?,
𝐹 −32 𝐹 −32
steam co-exist B.at which the average kinetic 20 = 𝜃 × 100, 20 = 𝜃 × 100,
212−32 180
energy of particles making up a body is zero 20×180
𝐹𝜃 −32 = = 36, 𝐹𝜃 = 36+32 = 68℉.
C.of zero degree Celsius scale D.at which pure 100
water changes to ice at standard atmospheric 34. A faulty thermometer registers 102.5℃ at
pressure. 100℃. If the thermometer has zero error, what
30. Which of the following statements about the temperature will it register at 55℃? A.54.6℃
lower fixed point of a thermometer using the B.55.0℃ C.56.0℃ D.56.4℃.
Celcius scale is not correct? It is A.different from The thermometer has U.F.P or steam point error
absolute zero of the temperature B.the zero mark but no L.F.P or ice point or zero error. Hence, for

120
Demystified Series Physics Demystified by Dr Timothy
the faulty thermometer, U.F.P or 𝑈=102.5℃, L.F.P 𝜃𝑈 =100℃, 𝜃=?, 𝜃 =
𝐿𝜃 −𝐿𝐿
× 100,
𝐿𝑈 −𝐿𝐿
or 𝐿=0℃, 𝜃𝑓 =?, for the fautless thermometer, 10
U.F.P or 𝑈=100℃, L.F.P or 𝐿=0℃, 𝜃=55℃, 𝜃 = × 100 = 40℃, 𝜃 = 40+273 = 313𝐾.
25
𝜃𝑓 −0
=
55−0
,
𝜃𝑓
=
55
, 40. Calculate the length which corresponds to a
102.5−0
55
100−0 102.5 100 temperature of 20℃. If the ice and steam points
𝜃𝑓 = × 102.5= 56.375℃ = 56.4℃. of an ungraduated thermometer are 400mm
100
35. The lower fixed point of a faulty thermometer apart A.20mm B.30mm C.60mm D.80mm
reads 2℃ while the upper fixed point is 100℃. 𝐿𝑈 − 𝐿𝐿 =400mm, 𝐿𝜃 − 𝐿𝐿 =𝐿=?, 𝜃𝐿 =0℃,
𝐿𝜃 −𝐿𝐿
What is the true temperature when the 𝜃𝑈 =100℃, 𝜃=20℃, 𝜃 = × 100,
𝐿𝑈 −𝐿𝐿
thermometer reads 51℃? A.52℃B.50℃ C.49℃ 𝐿 𝐿
D.46℃. 20 = × 100 = , 𝐿 = 20×4 = 80mm.
400 4
Hence, for the faulty thermometer: U.F.P or 41. The lengths of the mercury thread in a
𝑈=100℃, L.F.P or 𝐿=2℃, 𝜃𝑓 =51℃, for the fautless thermometer when its bulb is placed in melting
thermometer: U.F.P or 𝑈=100℃, L.F.P or 𝐿=0℃, ice and steam are 4.0cm and 29.4cm respectively.
51−2 𝜃−0 49 𝜃 49 If the length of the tread, when the bulb is place
𝜃=?, = , = , 𝜃 = × 100
100−2 100−0 98
𝑡𝜃 −𝑡𝐿
100 98
51−2
in a liquid is 19.0cm, calculate the temperature of
𝜃 = 50℃. OR 𝜃 = × 100, 𝜃 = × 100 the liquid A.74.8℃ B.65.5℃ C.59.1℃ D.40.9℃
𝑡𝑈 −𝑡𝐿 100−2
48 𝐿𝐿 =4.0cm, 𝐿𝑈 =29.4cm, 𝐿𝜃 =19.0cm, 𝜃𝐿 =0℃,
𝜃 = × 100 = 50℃. 𝐿 −𝐿
98
𝜃𝑈 =100℃, 𝜃=?, 𝜃 = 𝜃 𝐿 × 100,
36. A thermometer with an arbitrary scale of 𝐿𝑈 −𝐿𝐿
19.0−4.0 15.0
equal divisions registers −30°𝑆 at the ice point 𝜃= × 100 = × 100 = 59.055℃,
29.4−4.0 25.4
and +90°𝑆 at steam point. Calculate the Celsius 𝜃 = 59.1℃.
temperature corresponding 60°𝑆 A.25.0℃ 42. The heights of the mercury thread in a
B.50.0℃ C.66.7℃ D.75.0℃. mercury in glass thermometer when in melting
𝑡𝐿 =−30°𝑆, 𝑡𝑈 =+90°𝑆, 𝑡𝜃 =60°𝑆, 𝜃𝑈 =100℃, ice and then in steam are 3cm and 18cm
𝑡 −𝑡 𝜃−𝜃𝐿
𝜃𝐿 =0℃, 𝜃=?, 𝜃 𝐿 = , repectively. At a temperature 60℃ the height
𝑡𝑈 −𝑡𝐿 𝜃𝑈 −𝜃𝐿
60−(−30) 𝜃−0 60+30 𝜃 90 𝜃 would be A.7.5cm B.9.0cm C.10.8cm D.12.0cm
= , = , = ,
90−(−30) 100−0 90+30 100 120 100 E.12.6cm.
90
𝜃= × 100 = 75℃. OR 𝐿𝐿 =3cm, 𝐿𝑈 =18cm, 𝐿𝜃 =?, 𝜃𝐿 =0℃, 𝜃𝑈 =100℃,
120 𝐿 −𝐿 𝐿 −3
𝜃=
𝑆𝜃 −𝑆𝐿
× 100 =
60−(−30)
× 100 , 𝜃=60℃, 𝜃 = 𝜃 𝐿 × 100, 60 = 𝜃 × 100,
𝐿𝑈 −𝐿𝐿 18−3
𝑆𝑈 −𝑆𝐿 90−(−30) 𝐿 −3 60×15
𝜃=
60+30
× 100 =
90
× 100 = 75℃. 60 = 𝜃 × 100, 𝐿𝜃 −3 = = 9,
15 100
90+30 120
37. The ice and steam points on a mercury-in- 𝐿𝜃 = 9+3 = 12.0cm.
glass thermometer are 10cm and 30cm 43.
repectively. Calculate the temperature in degree
celcius, when the mercury meniscus is at the
14cm mark A.20℃ B.30℃ C.34℃ D.70℃.
𝐿𝐿 =10cm, 𝐿𝑈 =30cm, 𝐿𝜃 =14cm, 𝜃𝐿 =0℃,
𝐿 −𝐿 𝜃−𝜃𝐿 The diagram above illustrates the mercury-in-
𝜃𝑈 =100℃, 𝜃=?, 𝜃 𝐿 = , glass thermometer used for determining the
𝐿𝑈 −𝐿𝐿 𝜃𝑈 −𝜃𝐿
14−10
=
𝜃−0
,
4
=
𝜃
, 𝜃=
4
× 100, temperature of a room. Use the data on it to
30−10 100−0 20 100
𝐿𝜃 −𝐿𝐿
20
14−10 determine the room temperature A.3.3℃ B.7.0℃
𝜃 = 20℃. OR 𝜃 = × 100 = × 100 C.13.0℃ D.30.0℃.
𝐿𝑈 −𝐿𝐿 30−10
4
𝜃 = × 100 = 20℃. 𝐿𝑈 − 𝐿𝐿 =10cm, 𝐿𝜃 − 𝐿𝐿 =3cm, 𝜃𝐿 =0℃, 𝜃𝑈 =100℃,
20 𝐿 −𝐿 3
38. The length of mercury thread when it is at 𝜃=?, 𝜃 = 𝜃 𝐿 × 100, 𝜃 = × 100 = 30℃.
𝐿𝑈 −𝐿𝐿 10
0℃, 100℃ and at an unknown temperature 𝜃 is 44. A well lagged bar of 100cm has its ends
25mm, 225mm, 175mm respectively. The value maintained at 100℃ and 40℃ respectively, what
of 𝜃 is A.85.0℃ B.80.0℃C.75.0℃ D.70.0℃ is the temperature at a point 60cm from the
𝐿𝐿 =25mm, 𝐿𝑈 =225mm, 𝐿𝜃 =175mm, 𝜃𝐿 =0℃, hotter end? A.58℃ B.62℃C.64℃ D.76℃
𝐿 −𝐿
𝜃𝑈 =100℃, 𝜃=?, 𝜃 = 𝜃 𝐿 × 100, A point 60cm from the hotter end 100℃ is same
𝐿𝑈 −𝐿𝐿
175−25 150 as 40cm from the colder end 0℃.
𝜃= × 100 = × 100 = 75.0℃. 𝐿𝑈 − 𝐿𝐿 =100cm, 𝐿𝜃 − 𝐿𝐿 =40cm, 𝜃𝐿 =40℃,
225−25 200
39. The ice and steam points of a certain 𝜃𝑈 =100℃, 𝜃=?,
𝐿𝜃 −𝐿𝐿
=
𝜃−𝜃𝐿
,
40
=
𝜃−40
,
thermometer are 25cm apart. If the length of 𝐿𝑈 −𝐿𝐿 𝜃𝑈 −𝜃𝐿 100 100−40
40 𝜃−40 40
mercury thread is 10cm above the ice point mark, = , 𝜃 −40 = × 60 = 24,
100 60 100
calculate the temperature recorded by the 𝜃 = 24+40 = 64℃.
thermometer in kelvin A.252𝐾 B.313𝑲 C.382𝐾 45. The readings on the pressure scale at the
D.422𝐾 E.523𝐾. steam and ice points are 800mmHg and
𝐿𝑈 − 𝐿𝐿 =25cm, 𝐿𝜃 − 𝐿𝐿 =10cm, 𝜃𝐿 =0℃,
121
Demystified Series Physics Demystified by Dr Timothy
300mmHg respectively. Determine the 𝑅𝑈 =5.5Ω, 𝑅𝐿 =5Ω, 𝑅𝜃 =5.2Ω, 𝜃𝑈 =100℃, 𝜃𝐿 =0℃,
corresponding temperature in degree Celsius 𝑅 −𝑅
𝜃=?, 𝜃 = 𝜃 𝐿 × 100 =
5.2−5
× 100 ,
𝑅𝑈 −𝑅𝐿 5.5−5
when it reads 450mmHg A.15℃ B.30℃ C.45℃ 0.2
D.60℃ E.80℃ 𝜃 = × 100 = 40℃.
0.5
𝑃𝑈 =800mmHg, 𝑃𝐿 =300mmHg, 𝑃𝜃 =450mmHg, 51. Which of the following types of
𝑃 −𝑃
𝜃𝑈 =100℃, 𝜃𝐿 =0℃, 𝜃=?, 𝜃 𝐿 =
𝜃−𝜃𝐿
, thermometers can be used to measure a range of
450−300 𝜃−0 150
𝑃𝑈 −𝑃𝐿
𝜃
𝜃𝑈 −𝜃𝐿
temperatures from −50℃ to 50℃? I.Clinical
= , = , II.Mercury-in-glass III.Alcohol-in-glass A.I B.II
800−300 100−0 500 100
𝜃=
150
× 100 = 30℃. OR 𝜽 =
𝑷𝜽 −𝑷𝑳
× 100. C.III D.I and II
500 𝑷𝑼 −𝑷𝑳 The range of a clinical thermometer is from 35℃
46. The pressure on the gas of a constant volume to 43℃. Mercury freezes at −39℃, hence cannot
gas thermometer at ice point is 325mm of measure a temperature below −39℃ i.e 50℃, but
mercury and at the steam point is 875mm of boils at 3357℃. Alcohol freezes at −115℃ and
mercury. Find the temperature when the boils at 78℃, hence it is the most appropriate for
pressure of the gas is 490mm of mercury A.30𝐾 measure a range of temperature from −50℃ to
B.243𝐾 C.300𝐾 D.303𝑲. 50℃.
𝑃𝑈 =875mmHg, 𝑃𝐿 =325mmHg, 𝑃𝜃 =490mmHg, 52. A change in temperature of 45℃ in
𝑃 −𝑃
𝜃𝑈 =100℃, 𝜃𝐿 =0℃, 𝜃=?, 𝜃 = 𝜃 𝐿 × 100 equivalent to a change in temperature on the
𝑃𝑈 −𝑃𝐿
490−325 165 kelvin scale of A.25𝐾 B.45𝑲 C.81𝐾 D.318𝐾.
𝜃= × 100 = × 100 = 30℃, 𝜃=
875−325 550 53. The temperature of an object is raused by
30+273 = 303𝐾.OR 120℃. The resulting increase in its absolute
𝑃𝑈 =875mmHg, 𝑃𝐿 =325mmHg, 𝑃𝜃 =490mmHg, temperature is A.50𝐾 B.120𝑲 C.200𝐾 D.393𝐾.
𝑃 −𝑃 𝜃−𝜃𝐿
𝜃𝑈 =373𝐾, 𝜃𝐿 =273𝐾, 𝜃=?, 𝜃 𝐿 = , A temperature change of 45℃ is the same as 45K
𝑃𝑈 −𝑃𝐿 𝜃𝑈 −𝜃𝐿
490−325 𝜃−273 165 𝜃−273 change in temperature, as they have the same
= , = ,
875−325 373−273 550 100 fundamental interval of 100.
165
𝜃 −273 = × 100 = 30, 𝜃 = 30+273 = 303𝐾. 53. A change in temperature of 90℃ is equivalent
550
47. A thermometer record 680mmHg at steam to a change in temperature on the Fahrenheit
point and 440mmHg at ice point. The scale of A.50℉ B.90℉C.162℉D.194℉
9
temperature it records at 380mmHg is A.−25℃ A division on the Celcius scale is times the size
5
B.−20℃ C.20℃ D.25℃. of a division on the Fahrenheit scale.
𝑃𝑈 =680mmHg, 𝑃𝐿 =440mmHg, 𝑃𝜃 =380mmHg, A temperature change of 90℃ is the same as
𝑃 −𝑃
𝜃𝑈 =100℃, 𝜃𝐿 =0℃, 𝜃=?, 𝜃 = 𝜃 𝐿 × 100, 9
×90℉ =162℉.
𝑃𝑈 −𝑃𝐿 5
380−440 −60
𝜃= × 100 = × 100 = −25℃.
680−440 240
48. The gas in a gas thermometer occupies 54cm3 Jamb past questions on temperature and its
at 0℃ and 74cm3 at 100℃. What is the measurement :
temperature when it occupies 46cm3 if its [1978/17,1979/14,1980/7,1981/1,42,1983/35,
pressure is constant A.−45℃B.−40℃C.40℃ 1984/18,29,1987/16,1989/13,1990/15,1991/
D.45℃. 15,1992/13,1993/14,1994/20,1995/18,23,26,
𝑉𝑈 =74cm3, 𝑉𝐿 =54cm3, 𝑉𝜃 =46cm3, 𝜃𝑈 =100℃, 1997/20,1998/17,2000/21,2005/43,2006/48,
𝑉 −𝑉 2007/31,39,2008/1,2009/18,2010/15,2012/19
𝜃𝐿 =0℃, 𝜃=?, 𝜃 = 𝜃 𝐿 × 100,
𝑉𝑈 −𝑉𝐿 ,2013/16,2014/19]
46−54 −8
𝜃= × 100 = × 100 = −40℃.
74−46 20
49. A plantinum resistance thermometer has a
resistance of 4Ω at 0℃ and 10Ω at 100℃.
Assuming the resistance changes uniformly with
temperature, calculate the resistance of the
thermometer when the temperature is 45℃
A.14.0Ω B.6.7Ω C.3.2Ω D.2.7Ω.
𝑅𝑈 =10Ω, 𝑅𝐿 =4Ω, 𝑅𝜃 =?, 𝜃𝑈 =100℃, 𝜃𝐿 =0℃,
𝑅 −𝑅 𝑅 −4
𝜃=45℃, 𝜃 = 𝜃 𝐿 × 100, 45 = 𝜃 × 100
𝑅𝑈 −𝑅𝐿 10−4
𝑅 −4 45×6
45 = 𝜃 × 100, 𝑅𝜃 −4 = =2.7,
6 100
𝑅𝜃 = 2.7+4 = 6.7Ω.
50. A plantinum resistance thermometer wire
has a resistance of 5ohms at 0℃ and 5.5ohms at
100℃. Calculate the temperature of the wire
when the resistance is 5.2ohms A.80℃ B.60℃
C.40℃ D.10℃.

122
Demystified Series Physics Demystified by Dr Timothy

Chapter 12 – Thermal Expansivity


● Thermal expansivity – Thermal expansivity is a material are related by – 2𝜸 = 𝟑𝜷.
the increase in size or dimension of an object - Percentage in the volume of a solid after
when heat is applied to it. expansion %∆𝑽 =
∆𝑽 𝑽 −𝑽
× 100= 𝟐 𝟏 × 100 ,
- Thermal expansion is due to an increase in the 𝑽𝟏 𝑽𝟏

amplitude of vibration and motion of molecules %∆𝑽 = 𝜸∆𝜽 ×100.


of objects when heated. - In calculating for percentage change in length or
- Expansion in solids – Expansion in solids is area, replace volume 𝑉 with length 𝐿 or area 𝐴
regular i.e expands when heated and contracts and volume expansivity 𝛾 with linear expansivity
when cooled. Expansion is solids can be linear or 𝛼 or area expansivity 𝛽.
length, area or superficial and volume or cubic - Night time temperature is same as the lowest
expansion. temperature while day time temperature is same
1.Linear or length expansivity 𝜶 – This is the as the highest temperature.
increase in length per unit length per degree rise - Useful application of expansion in solids –
in temperature. I.Fitting of wheels on rims. II.Riveting of steel
𝒊𝒏𝒄𝒓𝒆𝒂𝒔𝒆 𝒊𝒏 𝒍𝒆𝒏𝒈𝒕𝒉 ∆𝑳 𝑳 −𝑳 plates. III.Bimetallic strips used in thermometers
𝜶= = = 𝟐 𝟏 . and thermostart(i.e fire alarms,compensation of
𝒐𝒓𝒊𝒈𝒊𝒏𝒂𝒍 𝒍𝒆𝒏𝒈𝒕𝒉×𝒕𝒆𝒎𝒑. 𝒓𝒊𝒔𝒆 𝑳𝟏 ∆𝜽 𝑳𝟏 (𝜽𝟐 −𝜽𝟏 )
∆𝐿=change in length or linear expansion of solid expansion in watches and clocks,laundry iron
= 𝐿2 − 𝐿1 , 𝐿2 =final length at 𝜃2 , 𝐿1 =initial length e.t.c). IV.Removal of tight glass stoppers.
at 𝜃1 . ∆𝑳 = 𝜶𝑳𝟏 ∆𝜽, 𝑳𝟐 = 𝑳𝟏 (𝟏 + 𝜶∆𝜽). V.Roller bearings in metal bridges
- When the temperature decreases and the length - Unuseful application of expansion in solids –
also decreases , 𝜃2 < 𝜃1 or 𝐿2 < 𝐿1 , the formula I.Bursting of water pipes in cold weather.
becomes – 𝜶 = 𝟏 𝟐 .
𝑳 −𝑳 II.Collapsing of metal bridges. III.Cracking of
𝑳𝟏 (𝜽𝟏 −𝜽𝟐 )
thick glasses due to uneven expansion i.e inner
- Safety gap or gap between metals is same as wall expanding more than the outer wall.
expansion = ∆𝑳 = 𝑳𝟐 − 𝑳𝟏 . IV.Sagging of telegraph wire. V.Buckling of rail
- Change in d iameter is a linear expansion not way tracks.
area or superficial expanision. - Thermostats – They are used to control
- The relationship between the linear expansivity temperature of laundry iron,aquaria for tropical
𝛼, Young modulus 𝐸, area 𝐴, force or tension 𝐹 fish,heated apartments and hot water storage
acting on a solid is given by – 𝑭 = 𝑬𝑨𝜶∆𝜽. tanks. It is not used in a pressure cooker.
2.Area or superficial expansivity 𝜷 – This is the - A bimetallic strip consists of two metals with
increase in area per unit area per degree rise in different expansivities i.e copper(or brass) and
temperature. steel(or iron). The metal with high expansivity
𝒊𝒏𝒄𝒓𝒆𝒂𝒔𝒆 𝒊𝒏 𝒂𝒓𝒆𝒂 ∆𝑨 𝑨 −𝑨
𝜷= = = 𝟐 𝟏 . (brass or copper) occupies the upper part
𝒐𝒓𝒊𝒈𝒊𝒏𝒂𝒍 𝒂𝒓𝒆𝒂 × 𝒕𝒆𝒎𝒑. 𝒓𝒊𝒔𝒆 𝑨𝟏 ∆𝜽 𝑨𝟏 (𝜽𝟐 −𝜽𝟏 )
∆𝐴=increase in area=𝐴2 − 𝐴1 , 𝐴2 =final area at while the other metal occupies the lower
𝜃2 , 𝐴1 =initial area at 𝜃1 . ∆𝑨 = 𝜷𝑨𝟏 ∆𝜽, part(steel or iron).
𝑨𝟐 = 𝑨𝟏 (𝟏 + 𝜷∆𝜽). - When the bimetallic strip is heated, the
- When the temperature decreases and the area metal with high expansivity occupies the
also decreases , 𝜃2 < 𝜃1 or 𝐴2 < 𝐴1 , the formula convex part and also occupies the concave
𝑨 −𝑨
becomes – 𝜷 = 𝟏 𝟐 . part when the strip is cooled.
𝑨𝟏 (𝜽𝟏 −𝜽𝟐 ) - The order of increasing expansivity in some
- Area expansivity of a material is twice its common solids is – iron→copper→silver→
linear expansivity. 𝜷 = 2𝜶. aluminium.
3.Volume or cubic expansivity 𝜸 – This is the - Expansion in liquids – Expansion in liquids
increase in volume per unit volume per degree unlike solids, is irregular. Liquids expands
rise in temperature. more than solids.
𝒊𝒏𝒄𝒓𝒆𝒂𝒔𝒆 𝒊𝒏 𝒗𝒐𝒍𝒖𝒎𝒆 ∆𝑽 𝑽 −𝑽
𝜸= = = 𝟐 𝟏 . - When a vessel containing a liquid is heated, the
𝒐𝒓𝒊𝒈𝒊𝒏𝒂𝒍 𝒗𝒐𝒍𝒖𝒎𝒆 ×𝒕𝒆𝒎𝒑. 𝒓𝒊𝒔𝒆 𝑽𝟏 ∆𝜽 𝑽𝟏 (𝜽𝟐 −𝜽𝟏 )
∆𝑉=increase in volume=𝑉2 − 𝑉1 , 𝑉2 =final volume liquid level first drops,due to the expansion of the
at 𝜃2 , 𝑉1 =initial volume at 𝜃1 . vessel,the liquid level later rises as it also
∆𝑽 = 𝜸𝑽𝟏 ∆𝜽, 𝑽𝟐 = 𝑽𝟏 (𝟏 + 𝜸∆𝜽). expands as more heat is supplied.
- When the temperature decreases and the - The expansion in liquids involves apparent
volume also decreases , 𝜃2 < 𝜃1 or 𝑉2 < 𝑉1 , the cubic expansion (expansion of the liquid only)
𝑽 −𝑽 and real cubic expansivity (expansion of liquid
formula becomes – 𝜸 = 𝟏 𝟐 . and container).
𝑽𝟏 (𝜽𝟏 −𝜽𝟐 )
- Volume expansivity of a material is thrice its - Apparent cubic expansivity 𝜸𝒂 of a liquid is the
linear expansivity. 𝜸 = 3𝜶. apparent increase in volume per unit volume per
- Volume expansivity and area expansivity of degree rise in temperature when the liquid is

123
Demystified Series Physics Demystified by Dr Timothy
heated in an inexpansible vessel. It is amount 4℃).
(mass or volume or weight) of liquid expelled to - Graphical representation of anomalous
the amount(mass or volume or weight) expansion of water –
remaining per degree rise in temperature. Volume Density
𝒂𝒑𝒑𝒂𝒓𝒆𝒏𝒕 𝒊𝒏𝒄𝒓𝒆𝒂𝒔𝒆 𝒊𝒏 𝒗𝒐𝒍𝒖𝒎𝒆
𝜸𝒂 = ,
𝒐𝒓𝒊𝒈𝒊𝒏𝒂𝒍 𝒗𝒐𝒍𝒖𝒎𝒆 ×𝒕𝒆𝒎𝒑. 𝒓𝒊𝒔𝒆
𝒂𝒎𝒐𝒖𝒏𝒕 𝒐𝒇 𝒍𝒊𝒒𝒖𝒊𝒅 𝒆𝒙𝒑𝒆𝒍𝒍𝒆𝒅
𝜸𝒂 = ,
𝒂𝒎𝒐𝒖𝒏𝒕 𝒐𝒇 𝒍𝒊𝒒𝒖𝒊𝒅 𝒓𝒆𝒎𝒂𝒊𝒏𝒊𝒏𝒈 ×𝒕𝒆𝒎𝒑. 𝒓𝒊𝒔𝒆
Amount of liquid expelled=Amount of liquid at
initial temp − Amount of liquid at the final temp. 0℃ 4℃ Temp 0℃ 4℃ Temp
Amount of liquid remaining is the amount of the
liquid at final temp. The amount of liquid can be Examples :
in mass or volume. 1. Which of the following is true about the
- Real or Absolute cubic expansivity 𝜸𝒓 of a expansion of matter when heated? A.The
liquid is the increase in volume per unit volume distances between the molecules decrease B.The
per degree rise in temperature in an expansible force of attraction between the molecules
vessel. increases C.The kinetic energy of the molecules
- Real cubic expansivity of a liquid is always increases without an appreciable increase in
greater than its apparent cubic expansivity their size D.The molecules decrease E.The
(𝜸𝒓 > 𝜸𝒂 ). molecules increase in size.
- Real cubic expansivity of a liquid is equal to the When a substance is heated , the kinetic energy
sum of the apparent cubic expansivity of the of the molecules increase and they overcome the
liquid and the cubic expansivity of the of the force of attraction between the molecules i.e the
vessel or container. 𝜸𝒓 = 𝜸𝒂 + 𝜸𝒄 . The cubic force of attraction decreases, and then move
expansivity of the vessel can be gotten from its further apart i.e distance between molecules
linear expansivity , 𝜸𝒄 = 3𝜶𝒄 . increases, resulting in an increase in size of the
- The density 𝝆 of substance can be used in place substance.
of volume in volume or cubic expansivity as they 2. The magnitude of the expansion or contraction
are inversely related – Hence, 𝝆𝟏 = 𝝆𝟐 (𝟏 + 𝜸∆𝜽). of a substance depends on the I.temperature
- Anomalous or Abnormal expansion of water change II.nature of the substance III.size of the
: Most liquids expand with temperature rise substance. Which of the statements above are
(heated) and contract with temperature fall correct? A.I and II B.I and III C.II and III D.I,II and
(cooled) over all ranges of temperatures. III.
𝑖𝑛𝑐𝑟𝑒𝑎𝑠𝑒 𝑖𝑛 𝑙𝑒𝑛𝑔𝑡ℎ ∆𝐿
- Water contracts from 0℃ to 4℃ (temperature 𝛼= = ,
𝑜𝑟𝑖𝑔𝑖𝑛𝑎𝑙 𝑙𝑒𝑛𝑔𝑡ℎ×𝑡𝑒𝑚𝑝. 𝑟𝑖𝑠𝑒 𝐿1 ∆𝜃
rise) and expands from 4℃ to 0℃ (temperature Expansion/contraction or increase in length, 𝑒 =
fall) (anomalous or abnormal expansion and ∆𝐿 = 𝐿1 𝛼∆𝜃, hence expansion or contraction
contraction).The density increases while the depends on temperature change or rise in
volume decreases between 0℃ to 4℃. Hence, temperature ∆𝜃, nature of the substance or linear
anomalous or abnormal behavior of water expansivity of the substance 𝛼 and original
occurs between 0℃and 4℃. length or size of the substance 𝐿1 .
- Below 0℃ and above 4℃, water behaves like all 3. A hollow steel piston fits exactly into a steel
other liquids i.e expands with temperature rise cylinder when both are at 60℃. Whe the
and contracts with temperature fall. temperature just falls to 19℃, A.The piston
- Water has its maximum or highest volume contracts and the cylinder expands B.Both
and minimum or least density at 0℃while it contract but the cylinder contracts more than the
has its minimum or least volume and piston C.Both contract but the piston contract
maximum or highest density at 4℃. more than the cylinder D.The piston still fits the
- Hope’s experiement using Hope’s apparatus cylinder exactly E.Only the cylinder contracts,
is used for the illustration and verification of the piston remains the same.
the anomalous behavior of water. The cylinder and the piston are both made of
- The anomalous behavior or expansion of water steel hence, they have the same linear
makes ponds, lakes or rivers to freeze from the expansivity. Therefore, when the temperature of
top surface rather than from the bottom.Hence, the cylinder and the piston falls from 60℃ to
marine lives can survive during winter since ice 19℃, they both contract to the same extent and
forms at the surface of the water while the the piston fits the cylinder exactly.
bottom of the water remains at 4℃, a 4. In which of the following devices is the
temperature warm enough for aquatic life. expansion of solid a disadvantage? I.Fire alarms
- The cubic expansivity of water is negative II.Thermostat III.Simple pendulum IV.Bimetallic
between 0℃ and 4℃ (the change volume is thermometer A.I B.II C.III D.I and IV.
negative when water is heated from 0℃ to

124
Demystified Series Physics Demystified by Dr Timothy
Expansion of solids is a disadvantage in simple expands and bends inorder to complete the
pendulum,balance wheel of a watch i.e makes it circuit hence, switches the flash light on.
to lose time,buildings,bridges,railway lines, 12. The design of the thermostat of an electric
electric transmission and telegraph wires. iron is based on the A.apparent expansion of a
Expansion of solids is an advantage in fitting steel body B.change in the resistance of a body on
tyres on steel wheels, rivets or riveted joint of heating C.thermo-electric effect D.change in the
steel plates,thermostat and fire alarm system. density of a body when heated E.expansion of
5. The expansion of solids can be considered a metals when heated.
disadvantage in the A.fire alarm system 13. A bimetallic strip of iron and brass was
B.balance wheel of a watch C.fitting wheels on heated. Which of the following diagrams
rims D.riveting of steel plates. accurately after heating?
6. Large metal bridges are often mounted in A. Brass Iron B . Brass Iron

roller bearings in order to A.prevent the bridges


from sinking into the ground B.prevent the C. Iron Brass D. Brass

bridges from changing theur lengths C.allow for Iron


the expansion and contraction of the bridges When a bimetallic strip is heated, the metal at the
D.allow movement of bridges from one side to convex or upper part i.e brass, of the curved
another. bimetallic strip is of higher expansivity strip is of
Roller bearings and contraction of bridges to higher linear expansivity that the other metal i.e
prevent the bending of bridges. iron.Option C best describes it
7. Which of the following is not an application Option D illuustrates a bimetallic strip when
ethermometer B.Temperature control in an cooled.
electric iron C.Sagging of telegraph wires 14. 𝑋
D.Compensated balance wheel of a watch. 𝑌
Sagging of telegraph wires is an undesirable The figure shows a composite bar of two metal
event which leads to snapping of telegraph wires strips 𝑋 and 𝑌 which has been heated. The
or electric transmission cables. curvature observed is as shown because A.the
8. During summer, the balance wheel of a clock two strips of metal expand in the same direction
expands. What effect does this have on the B.𝑋 expands more than 𝑌 for the same
accuracy of the clock? A.The clock gains time temperature C.the two strips become softer
B.The accuracy of the clock is not affected C.The when heated and the weight causes bending D.Y
clock loses time D.The clock stops working. expands more than 𝑋 at the same temperature
Increase in temperature increases the diameter E.𝑌 expands more than 𝑋 at different
of the balance wheel and weakens the elasticity temperatures.
of the hairspring, thus causing it to lose time. 𝑋 and 𝑌 are two different metals, hence their
9. One of the most important applications of coefficient of linear expansivity is also different
bimetallic strip is found in the construction of i.e expands differently.In a bimetallic strip, the
A.An altimeter B.A thermocouple C.A hygrometer metal in the upper or convex part has a greater
D.A thermostat. linear expansivity or expands more than the
10. Which of the following statements is not other metal at the lower part.
true? Thermostats are used to control the 15. Which of the following statements cearly
temperature of A.pressure cookers B.laundry describes the behaviour of thw fire alarm as
irons C.hot water storage tanks D.aquaria for shown below, given that the linear expansivities
tropical fishes E.heated apartments of copper and steel are 2.0×10-5℃−1 and 1.2×10-
Thermomstat is used when it is desirable to 5℃−1 ?

maintain a system at a certain steady Electric


temperature. Thermostats are not used in bell
pressure cookers.
11. The flasher unit of a motor vehicle works on Copper
the principle that A.metals are good conductors
of heat B.a bimetallic strip expands and bends 𝑃 Steel 𝑄
when heated C.the linear expansivity of one I.The bimetallic strip will not be able not close the
metal of a bimetallic strip changes while that of circuit when there is fire II.The bimetallic strip
the other remains constant when heated D.the will close the circuit when there is fire III.If the
linear expansivity of a bimetallic strip changes as copper and steel are interchanged, the circuit will
the temperature changes E.the linear expansivity close when there is fire A.I B.II C.III D.I and III
of two metals always add up to unity. E.II and III.
The flasher unit of a motor vehicle is made of a The bimetallic strip upon heating when there is
bimetallic strip and works on the principle that fire, will bend away from the key of the electric

125
Demystified Series Physics Demystified by Dr Timothy
bell, as copper has higher linear expansivity than 𝛼=
0.11
= 1.1×10-5℃−1 = 0.11×10-6℃−1 .
110.0×90
steel (copper expands more than steel). If the
22. A wire of length 20m is cooled from 45℃ to
copper and steel are interchanged (copper is
−5℃. Find the change in its length if its linear
outside while the steel inside), the bimetallic
expansivity is 0.000020𝐾 −1 . A.0.16m B.0.020m
strip will bend towards the key and close the
C.0.160m D.0.200m E.1.600m.
circuit upon heating.
𝐿1 =20m, 𝜃1 =45℃, 𝜃2 =−5℃, 𝛼=0.000020𝐾 −1 , 𝛼
16.The difference in length between two metal
=2.0×10-5𝐾 −1 , change in length ∆𝐿=?,
rods of different materials is constant at all
The temperature decreases i.e cooling, hence the
tempetures. Which of the following conclucisons 𝐿 −𝐿 ∆𝐿
is correct? A.The linear expansivities of the two body contracts. 𝛼 = 1 2 = ,
𝐿1 (𝜃1 −𝜃2 ) 𝐿1 (𝜃1 −𝜃2 )
metal rods are the same B.The increase in length ∆𝐿 = 𝛼𝐿1 (𝜃1 − 𝜃2 ) =2.0×10-5×20(45−(−5) ,
of both metal rods is the same at all temperatures ∆𝐿 = 40.0×10-5(50)=2.0×10-2m = 0.020m.
C.The difference in length is equal to the length of 23. A wire of length 100.0m at 30℃ has a linear
the shorter rod D.The sum of their lengths is expansivity of 2×10-5𝐾 −1 . Calculate the length of
always constant E.The increase in length at all the wire at a temperature of −10℃ A.100.08m
temperature is equal to the new length of the B.100.04m C.99.96m D.99.92m.
shorter rod. 𝐿1 =100.0m, 𝛼=2.0×10-5𝐾 −1 , 𝜃1 =30℃, 𝜃2 =−10℃,
Diffrence in length is same as increase or change 𝐿2 =?, The temperature decreases i.e cooling,
in length hence, the increase in length of both 𝐿 −𝐿
hence the body contracts. 𝛼 = 1 2 ,
metal rods is the same at all temperatures. 𝐿1 (𝜃1 −𝜃2 )

17. A rod of initial length 2m at a temperature of 𝐿1 − 𝐿2 = 𝛼𝐿1 (𝜃1 − 𝜃2 ), 𝐿2 = 𝐿1 − 𝛼𝐿1 (𝜃1 − 𝜃2 ),


25℃ is heated to 80℃. Calculate the increase in 𝐿2 = 100−2.0×10-5×100(30−(−10),
length of the rod if its linear expansivity is 𝐿2 = 100−2.0×10-5×100×40 = 100−0.08,
4.0×10-3𝐾 −1 A.0.26m B.0.44m C.0.53m D.0.84m. 𝐿2 = 99.92m.
𝐿1 =2m, 𝜃1 =25℃, 𝜃2 =80℃, 𝛼=4.0×10-3𝐾 −1 , 24. When a brass of length 100cm at 50℃ is
∆𝐿 heated, its length changes to 100.054cm at
increase in length ∆𝐿=?, 𝛼 = , ∆𝐿 = 𝛼𝐿1 ∆𝜃, [linear expansivity of brass = 0.000018𝐾 −1 ]
𝐿1 ∆𝜃
∆𝐿 = 𝛼𝐿1 (𝜃2 − 𝜃1 ) =4.0×10-3×2(80−25) A.51℃ B.54℃ C.72℃D.80℃ E.83℃.
∆𝐿 = 8.0×10-3×55 = 0.44m. 𝐿1 =100.0m, 𝜃1 =50℃, 𝐿2 =1000.54cm,
18. A metal rod 800mm I heated from 10℃ to 𝛼=0.000018𝐾 −1 =1.8×10-5𝐾 −1 , 𝜃2 =?,
95℃ expands by 1.36mm, the linear expansivity 𝛼=
∆𝐿 𝐿 −𝐿
= 2 1 , (𝜃2 − 𝜃1 ) = 2 1 ,
𝐿 −𝐿

of the metal is A.2.0×102𝐾 −1 B.2.0×10-2𝐾 −1 𝐿1 ∆𝜃 𝐿1 (𝜃2 −𝜃1 )


100.054−100 0.054
𝐿1 𝛼

C.5.0×10-3𝐾 −1 D.2.0×10-5𝑲−𝟏 . 𝜃2 − 50 = = = 30 ,
100×1.8×10−5 0.0018
𝐿1 =800mm, 𝜃1 =10℃, 𝜃2 =95℃, increase in length 𝜃2 = 30+50 = 80℃.
or expansion ∆𝐿=1.36mm, 𝛼=?, 25. An iron rod is 2.58m long at 0℃. Calculate the
𝛼=
∆𝐿
=
∆𝐿
=
1.36
= 2.0×10-5𝐾 −1 . length of a brass rod at 0℃, if the difference
𝐿1 ∆𝜃 𝐿1 (𝜃2 −𝜃1 ) 800(95−10)
between the lengths of the two rods must remain
19. A metal rod 80cm long lengthens by 0.09cm
the same at all temperature [linear expansivity of
when its temperature rises by 93.6℃. What is the
iron=1.2×10-5𝐾 −1 , linear expansivity of
linearv expansivity of the metal? A.11×10-6℃−1
brass=1.9×10-5𝐾 −1 ] A.1.61m B.1.62m C.1.63m
B.12×10-6℃−𝟏 C.15×10-6℃−1 D.9.6×10-6℃−1 D.1.64m.
E.10.4×10-6℃−1 . For iron – original length of iron 𝐿𝐼 =2.58m, linear
𝐿1 =80cm, ∆𝜃 =93.6℃, ∆𝐿=0.09cm, 𝛼=?, expansivity of iron 𝛼𝐼 =1.2×10-5𝐾 −1 , increase or
∆𝐿 0.09 0.09
𝛼= = = =12×10-6℃−1 . difference in length of iron ∆𝐿𝐼 , temperature rise
𝐿1 ∆𝜃 80×93.6 7488
∆𝐿
20. A brass rod is 2m long at a certain ∆𝜃, ∴ 𝛼𝐼 = 𝐼 ,
𝐿𝐼 ∆𝜃
temperature. What is the length for a For brass – original length of brass 𝐿𝐵 =?, linear
temperature rise of 100𝐾, if the linear expansivity of brass 𝛼𝐵 =1.9×10-5𝐾 −1 , increase or
expansivity of brass is 18×10-6𝐾 −1 A.2.0036m difference in length of brass ∆𝐿𝐵 , temperature
B.2.0018m C.2.1800m D.2.0360m ∆𝐿
𝐿1 =2m, ∆𝜃=100𝐾, 𝛼=18×10-6𝐾 −1 , 𝐿2 =?, rise ∆𝜃, ∴ 𝛼𝐵 = 𝐵 ,
𝐿𝐵 ∆𝜃
𝐿2 = 𝐿1 (1 + 𝛼∆𝜃) = 2(1+18×10-6×100) , Temperature rise ∆𝜃 for both iron and brass is
𝐿2 = 2(1+0.0018) =2×1.0018 = 2.0036m. the same, also the increase or difference in
21. What is the linear expansivity of a solid lengths of iron and brass are equal , ∆𝐿𝐼 = ∆𝐿𝐵 ,
whose temperature rises from 10℃ to 100℃ and ∆𝐿𝐼 = 𝛼𝐼 𝐿𝐼 ∆𝜃 , ∆𝐿𝐵 = 𝛼𝐵 𝐿𝐵 ∆𝜃,
whose length increases from 110.0cm to ∴ 𝛼𝐼 𝐿𝐼 ∆𝜃 = 𝛼𝐵 𝐿𝐵 ∆𝜃 , 𝛼𝐼 𝐿𝐼 = 𝛼𝐵 𝐿𝐵 ,
110.11cm? A.0.0011×10-6℃−1 B.0.011×10-6℃−1 𝐿𝐵 =
𝛼 𝐼 𝐿𝐼
=
1.2×10−5 ×2.58
= 1.63m.
C.0.11×10-6℃−1 D.1.1×10-6℃−1 E.11×10-6℃−1 𝛼𝐵 1.9×10−5

𝜃1 =10℃, 𝜃2 =100℃, 𝐿1 =110.00cm, 𝐿2 =110.11cm, 26. The length of an iron bar is 100cm at 20℃. At
∆𝐿 𝐿 −𝐿 110.11−110.0 what temperature will its length increase by
𝛼=?, 𝛼 = = 2 1 = ,
𝐿1 ∆𝜃 𝐿1 (𝜃2 −𝜃1 ) 110.0(100−10) 0.01%? [linear expansivity of iron=1.2×10-5℃−1 ]
126
Demystified Series Physics Demystified by Dr Timothy
A.48.0℃ B.38.0℃ C.28.3℃ D.23.0℃. 𝐿𝐶
=
1 2
= = 0.67, 𝐿𝐶 :𝐿𝐼 = 0.67:1 or 0.67.
𝐿𝐼 1.5 3
𝐿1 =100cm, 𝜃1 =20℃, 𝐿2 is increased by 0.01% of
0.01 31. The linear expansivity of a metal 𝑃 is twice
𝐿1 , 𝐿2 = 𝐿1 +0.01%𝐿1 = 𝐿1 + 𝐿1 , change in that of another metal 𝑄.When these materials are
100
0.01
length ∆𝐿 = 𝐿2 − 𝐿1 = 𝐿1 , heated through the same temperature change,
100
∆𝐿 ∆𝐿 their increase in length is the same. Calculate the
𝛼=2.0×10-5℃−1 , 𝛼 = = ,
𝐿1 ∆𝜃 𝐿1 (𝜃2 −𝜃1 ) ratio of the original length of 𝑃 to that of 𝑄 A.1:4
0.01
∆𝐿 100 1
𝐿 B.1:2 C.2:1 D.4:1 E.6:1.
(𝜃2 − 𝜃1 ) = , 𝜃2 − 20 = ,
𝛼𝐿1 1.2×10−5 ×100 For metal 𝑃 – ∆𝐿𝑃 = 𝛼𝑃 𝐿𝑃 ∆𝜃 , For metal 𝑄 –
0.01 1
𝜃2 − 20 = × 100 × = 8.3, ∆𝐿𝑄 = 𝛼𝑄 𝐿𝑄 ∆𝜃, the increase in length of both
100 1.2×10−5 ×100
𝜃2 = 8.3+20 = 28.3℃. metal 𝑃 and 𝑄 are equal. ∆𝐿𝑃 = ∆𝐿𝑄 ,
𝛼𝑄 ∆𝜃 𝛼𝑄
27. Stee; bars, each of length 3m at 28℃ are to be 𝛼𝑃 𝐿𝑃 ∆𝜃 = 𝛼𝑄 𝐿𝑄 ∆𝜃 ,
𝐿𝑃
= = , 𝛼𝑃 = 2𝛼𝑄 ,
𝐿𝑄 𝛼𝑃 ∆𝜃 𝛼𝑃
used for constructing a rail line. If the linear 𝐿𝑃 𝛼𝑄 𝐿𝑃 1
expansivity of the steel is 1.0×10-5𝐾 −1 , what is ∴ = , = , 𝐿𝑃 :𝐿𝑄 = 1:2.
𝐿𝑄 2𝛼𝑄 𝐿𝑄 2
the safety gap that must be left between 32. The ratio of the coefficient of linear
successive bars if the highest temperature 𝛼
expansion of two metals 1 is 3:4. If when heated
expected is 40℃? A.1.2×10-1cm B.7.2×10-2cm 𝛼2

C.6.0×10-2cm D.3.6×10-2cm E.1.8×10-2cm. through the same temperature change, the ratio
𝑒
𝐿1 =3m=300cm, 𝜃1 =28℃, 𝛼=1.0×10-5𝐾 −1 , of the increase in lengths of two metals 1 is 1:2,
𝑒2
𝜃2 =40℃, safety gap = expansion or change in the ratio of the original lengths
𝐿1 3
is A. B. C.
3 8
∆𝐿 𝐿2 2 8 3
length ∆𝐿. 𝛼 = , ∆𝐿 = 𝛼𝐿1 ∆𝜃 , 𝟐
𝐿1 ∆𝜃
D. .
∆𝐿 = 𝛼𝐿1 (𝜃2 − 𝜃1 ) =1.0×10-5×300(40−28) , 𝛼1
𝟑
3 ∆𝐿1 𝑒1 1 𝐿1
∆𝐿 = 3.0×10-3×12= 36×10-3cm = 3.6×10-2m. = 3:4 = , = = 1:2 = , =? ,
𝛼2 4 ∆𝐿2 𝑒2 2 𝐿2
28. A bridge made of steel is 600m long. What is ∆𝐿1 𝑒 𝛼 𝐿 ∆𝜃 𝛼 𝐿 𝛼1 𝐿1 1 3 𝐿1
= 1= 1 1 = 1 1 = × , = × ,
the daily variation in its length if the night time ∆𝐿2 𝑒2 𝛼2 𝐿2 ∆𝜃 𝛼 2 𝐿2 𝛼2 𝐿2 2 4 𝐿2
𝐿1 1×4 𝐿1 2
and day time temperatures are 10℃ and 35℃ = , = or 2:3.
𝐿2 2×3 𝐿2 3
repetively (linear expansivity of steel is 33. A metal sheet of area 100cm2 was heated
0.000012℃−1 ) A.0.18cm B.1.80cm C.18cm through 70℃. Calculate its new area if the linear
D.1800cm. expansivity is 0.000017𝐾 −1 A.100.06cm2
𝐿1 =600m, 𝛼=0.000012℃−1 =1.2.0×10-5℃−1 , B.100.12cm2C.100.24cm2 D.100.36cm2.
𝜃1 =10℃, 𝜃2 =35℃, daily variation in length = 𝐴1 =100cm2, ∆𝜃=70℃, 𝛼=0.000017𝐾 −1 , 𝛽=2𝛼,
increase in length ∆𝐿=?, 𝛽=2×0.000017=0.000034𝐾 −1 =3.4×10-5𝐾 −1 ,
∆𝐿
𝛼= , ∆𝐿 = 𝛼𝐿1 ∆𝜃 = 𝛼𝐿1 (𝜃2 − 𝜃1 ), 𝐴2 =?, 𝛽 =
∆𝐴 𝐴 −𝐴
= 2 1 ,
𝐿1 ∆𝜃
𝐴1 ∆𝜃 𝐴1 (𝜃2 −𝜃1 )
∆𝐿 =1.2×10-5×600(35−10) = 720×10-5×25, 𝐴2 = 𝐴1 (1 + 𝛽∆𝜃) = 100(11+3.4×10-5×70),
∆𝐿 = 0.18m = 18cm. 𝐴2 = 100(1+0.00238) = 100×1.00238,
29. A 10km railwayline is laid when the 𝐴2 = 100.238cm2= 100.24cm2.
temperature is 2℃. If the maximum temperature 34. A thin aluminium plate of surface area
in the region is 32℃, what should be the 1.500m2 at 20℃. What will be its surface area
minimum total length of all gaps between the when it is cooled to −20℃? (Take the linear
iron rails if the line is laid in strips? (Linear expansivity of aluminium to be 2.5×10-5𝐾 −1 )
expansivity of iron= 0.000012℃−1 ) A.7.68m A.1.503m2 B.1.500m2 C.1.498m2 D.1.497m2
B.4.08m C.3.84m D.3.6m E.0.24m E.1.490m2.
𝐿1 =10km=10000m, 𝜃1 =2℃, 𝜃2 =32℃, 𝐴1 =1.500m2, 𝜃1 =20℃, 𝜃2 =−20℃, 𝛼=2.0×10-
𝛼=0.000012℃−1 =1.2×10-5℃−1 , gap between 5𝐾 −1 , 𝛽=2𝛼, 𝛽=2×2.5×10-5=5.0×10-5𝐾 −1 ,
rails = expansion or increase in length ∆𝐿=?, temperature decreases and area also decreases.
∆𝐿
𝛼= , ∆𝐿 = 𝛼𝐿1 ∆𝜃 = 𝛼𝐿1 (𝜃2 − 𝜃1 ), 𝛽=
∆𝐴 𝐴 −𝐴
= 1 2 , 𝐴1 − 𝐴2 = 𝛽𝐴1 (𝜃1 − 𝜃2 ),
𝐿1 ∆𝜃
𝐴1 ∆𝜃 𝐴1 (𝜃1 −𝜃2 )
∆𝐿 =1.2×10-5×10000(32−2)=1.2×30=3.6m. 𝐴2 = 𝐴1 − 𝛽𝐴1 (𝜃1 − 𝜃2 )
30. The ratio of the linear expansivity of copper 𝐴2 = 1.5−5.0×10-5×1.5(20−(−20),
to that of iron iss approximately 1.5. A specimen 𝐴2 = 1.5−7.5×10-5×40 = 1.5−0.003= 1.497m2.
of iron and a specimen of copper expands by the 35. A thin square sheet of metal of sides 0.20m is
same amount per unit rise in temperature. The heated from 15℃ and 75℃. If the linear
ratio of their length is A.3 B.1.5 C.1.32 D.1 E.0.67 expansivity of the metal is 1.2×10-5𝐾 −1 find the
For copper – ∆𝐿𝐶 = 𝛼𝐶 𝐿𝐶 ∆𝜃 , For iron – ∆𝐿𝐼 = increase in its area A.6.05×10-5m2 B.5.76×10-
𝛼𝐼 𝐿𝐼 ∆𝜃, the expansion or increase in length of 5m2 C.5.05×10-5m2 D.3.75×10-5m2.
both copper amd iron are equal. 𝐿1 =0.20m, 𝐴1 =𝐿2=0.202= 0.04m2, 𝜃1 =15℃,
∆𝐿𝐶 = ∆𝐿𝐼 , 𝛼𝐶 𝐿𝐶 ∆𝜃 = 𝛼𝐼 𝐿𝐼 ∆𝜃 , 𝜃2 =75℃, 𝛼=1.2×10-5𝐾 −1 , 𝛽=2𝛼, 𝛽=2×1.2×10-5 =
𝐿𝐶 𝛼 ∆𝜃 𝛼 𝛼 𝛼 1
= 𝐼 = 𝐼 , 𝛼𝐶 :𝛼𝐼 = 𝐶 = 1.5 , 𝐼 = 2.4×10-5𝐾 −1 , change or increase in area ∆𝐴=?,
𝐿𝐼 𝛼𝐶 ∆𝜃 𝛼𝐶 𝛼𝐼 𝛼𝐶 1.5

127
Demystified Series Physics Demystified by Dr Timothy
∆𝐴 = 𝛽𝐴1 ∆𝜃 = 𝛽𝐴1 (𝜃2 − 𝜃1 ) , 0℃ is 8.5×103kgm-3 and its cubic expansivity as
∆𝐴 =2.4×10-5×0.04(75−15)= 9.6×10-8×60 5.7×10-5𝐾 −1 A.3.4×10-5m3 B.4.3×10-5m3
∆𝐴 =5.76×10-5m2. C.3.4×10 m D.3.4×10 m E.3.4×10-2m3.
-4 3 -3 3

36. A cube made of a metal of linear expansivity Mass m=170kg, density 𝜌=8.5×103kgm-3,
𝛼 is heated through a temperature 𝜃. If the initial original volume 𝑉1 = =
𝑚 170
0.02m3,
3 =𝜌 8.5×10
volume of the cube is 𝑉𝑜 , the expressiom for the
1 1 𝜃1 =0℃, 𝜃2 =30℃, 𝛾=5.7×10-5𝐾 −1 , ∆𝑉=?,
increase in volume of the cube A. 𝛼𝑉𝑜 𝜃 B. 𝛼𝑉𝑜 𝜃 ∆𝑉 = 𝛾𝑉1 ∆𝜃 = 𝛾𝑉1 (𝜃2 − 𝜃1 ) ,
3 2
C.2𝛼𝑉𝑜 𝜃 D.3𝜶𝑽𝒐 𝜽 . ∆𝑉 =5.7×10-5×0.02(30−0) ,
∆𝑉
Cubic expansivity 𝛾 = , 𝛾=3𝛼 , 𝑉1 =𝑉𝑜 , ∆𝜃=𝜃 , ∆𝑉 =5.7×10-5×0.02 ×30 = 3.4×10-5m3.
𝑉1 ∆𝜃
∆𝑉 43. When a metal balls is heated through 30℃, its
3𝛼 = , ∆𝑉 = 3𝛼𝑉𝑜 𝜃. volume becomes 1.0018cm3. If the linear
𝑉𝑜 𝜃
37. A blacksmith heated a metal whose cubic expansivity of the ball is 2.0×10-5𝐾 −1 , calculate
expansivity 6.3×10-6𝐾 −1 . The area expansivity is the original volume A.1.0000cm3 B.1.0020cm3
A.6.3×10-6𝐾 −1 B.4.2×10-6𝑲−𝟏 C.2.1×10-6𝐾 −1 C.1.0036cm3 D.1.0180cm3.
D.2.0×10-6𝐾 −1 . 𝑉2 =1.0018cm3, ∆𝜃=30℃, 𝛼=2.0×10-5𝐾 −1 ,
𝛾=6.3×10-6𝐾 −1 , 𝛽=?, 2𝛾 = 3𝛽, 2×6.3×10-6 =3𝛽, 𝛾=3𝛼=3×2.0×10 =6.0×10 𝐾 , 𝑉1 =?,
-5 -5 −1

2×6.3×10−6 𝑉2
𝛽= = 4.2×10-6𝐾 −1 . 𝑉2 = 𝑉1 (1 + 𝛾∆𝜃) , 𝑉1 = (1+𝛾∆𝜃) ,
3
1.0018 1.0018 1.0018
38. If 𝐿, 𝑆 and 𝑉 are the linear, area and volume 𝑉1 = = = = 1.0000cm3.
1+6.0×10−5 ×30 1+0.0018 1.0018
expansivities of a given metal respectively, which 44. The linear expansivity of brass is
of the following equation is correct? A.𝐿 − 𝑆 = 0 2.0×10-5℃−1 . If the volume of a piece of brass is
B.𝑉 − 2𝑆 = 0 C.𝑺 − 2𝑳 = 0 D.2𝑆 − 𝐿 = 0 15.00cm3 at 0℃, what is the volume at 100℃?
E.3𝑉 − 𝐿 = 0. A.16.03cm3 B.16.00cm3 C.15.09cm3 D.15.03cm3
Linear expansivity 𝛼 = 𝐿, area expansivity 𝛽 = 𝑆, 𝛼=2.0×10-5℃−1 , 𝛾=3𝛼=3×2.0×10-5, 𝛾
volume expansivity 𝛾 = 𝑉, =6.0×10-5℃−1 , 𝑉1 =15.00cm3, 𝜃1 =0℃, 𝜃2 =100℃,
𝛽=2𝛼, ∴ 𝑆 = 2𝐿, 𝑆 − 2𝐿 = 0. ∆𝑉 𝑉 −𝑉
𝑉2 =?, 𝛾 = = 2 1 ,
𝛾=3𝛼, ∴ 𝑉 = 3𝐿, 𝑉 − 3𝐿 = 0. 𝑉1 ∆𝜃 𝑉1 (𝜃2 −𝜃1 )
2𝛾 = 3𝛽, ∴ 2𝑉 = 3𝑆 , 2𝑉 − 3𝑆 = 0. 𝑉2 = 𝑉1 (1 + 𝛾(𝜃2 − 𝜃1 )) ,
39. If the cubic expansivity of a brass between 𝑉2 = 15(1+6.0×10-5(100−0)) ,
27℃ and 100℃ is 5.7×10-5𝐾 −1 , what is the linear 𝑉2 = 15(1+0.006) = 15×1.006 = 15.09cm3.
expansivity? A.2.85×10-5𝐾 −1 B.1.90×10-5𝑲−𝟏 45. A rectangular metal block of volume 10-6m3
C.1.86×10-5𝐾 −1 D.1.70×10-5𝐾 −1 E.1.62×10-5𝐾 −1 at 273𝐾 is heated to 573𝐾. If its coefficient of
𝛾
𝛾=5.7×10-5𝐾 −1 , 𝛼=?, 𝛾=3𝛼, 𝛼 = , linear expansion is 1.2×10-5𝐾 −1 , the percentage
3
5.7×10−5 change of its volume is A..1.5% B.1.1% C.0.4%
𝛼= = 1.90×10-5𝐾 −1 . D.0.1%.
3
40. Calculate the increase in volume when 𝑉1 =10-6m3, 𝜃1 =273𝐾, 𝜃2 =573𝐾, 𝛼=1.2×10-5𝐾 −1 ,
1500cm3 of steel is heated from 0℃ to 𝛾=3𝛼=3×1.2×10-5=3.6×10-5𝐾 −1 ,
40℃[linear expansivity of steel=1.2×10-5℃−1 ] 𝛾=
∆𝑉
=
∆𝑉
,
∆𝑉
= 𝛾(𝜃2 − 𝜃1 ) ,
A.0.12cm3 B.0.72cm3 C.1.44cm3 D.2.16cm3 𝑉1 ∆𝜃 𝑉1 (𝜃2 −𝜃1 ) 𝑉1
∆𝑉
E.1500.72cm3. Percentage change in volume ∆𝑉% = × 100,
𝑉1
𝑉1 =1500cm3, 𝜃1 =0℃, 𝜃2 =40℃, 𝛼=1.2×10-5℃−1 , ∆𝑉% = 𝛾(𝜃2 − 𝜃1 ) × 100 ,
𝛾=3𝛼=3×1.2×10-5=3.6×10-5℃−1 , ∆𝑉=?, ∆𝑉% =3.6×10-5(573−273) ×100 ,
∆𝑉 = 𝛾𝑉1 ∆𝜃 = 𝛾𝑉1 (𝜃2 − 𝜃1 ) , ∆𝑉% =3.6×10-5×300×100 = 1.08% = 1.1%.
∆𝑉 =3.6×10-5×1500(40−0) , 46. When very hot water is poured into a thin-
∆𝑉 =3.6×10-5×1500×40 = 2.16cm3. walled glass container, it is less likely to break
41. A solid metal cube of side 10cm is heated because A.thin glass does not expand fast B.glass
from 10℃ to 60℃. If the linear expansivity of the can withstand a very high temperature C.all
metal is 1.2×10-5𝐾 −1 , calculate the increase in its parts of the container get heated uniformly
volume A.0.6cm3 B.1.2cm3 C.1.8cm3 D.3.6cm3 D.the outer part of the container is cooler than
E.6.0cm3 the inner part E.the molecules of glass are too far
𝐿1 =10cm, 𝑉1 =𝐿1 3 =103=1000cm3, 𝜃1 =10℃, apart to cause breakage of the container.
𝜃2 =60℃, 𝛼=1.2×10-5𝐾 −1 , 𝛾=3𝛼=3×1.2×10-5, The inner and outer walls of a thin-walled glass
𝛾=3.6×10-5𝐾 −1 , ∆𝑉=?, container expands uniformly or equally as heat is
∆𝑉 = 𝛾𝑉1 ∆𝜃 = 𝛾𝑉1 (𝜃2 − 𝜃1 ) , supplied uniformly to all parts.
∆𝑉 =3.6×10-5×1000(60−10) , 47. Which of the following explains why a thick
∆𝑉 =3.6×10-5×1000×50 = 1.8cm3. glass cup cracks when water is poured into it?
42. A piece of brass of mass 170kg has its A.large increase in the heat capacity of the cup
temperature raised from 0℃ to 30℃. Calculate B.high density of water C.unequal expansion of
its increase in volume, given density of brass at the interior walls of the cup D.anomalous
128
Demystified Series Physics Demystified by Dr Timothy
expansion of water E.greater specific heat through 40℃,the mass reduces to 38g. Calculate
capacity of water compared with that of glass. the apprarent cubic expansivity of the liquid
A thick-walled glass cup or container A.6.25×10𝐾 −1 B.6.25×10-3𝑲−𝟏 C.2.50×10-3𝐾 −1
expandsunequally or non-uniformly when heat is D.2.50×10-4𝐾 −1 .
supplied i.e the interior wall expands more than Mass of density bottle=30g, mass of density
the exterior wall. bottle+liquid=40g, mass of density bottle+
48. Consider a thick glass thumbler and a thin liquid after heating=38g, mass of liquid expelled
glass thumbler which are made of the same kind 𝑀𝑒 = 40−38 = 2g, mass of liquid remaining(or
of glass. Some hot liquid is poured into them. left) 𝑀𝑟 = 38−30 = 8g, temperature rise
Which of the following statements is correct? ∆𝜃=40℃, 𝛾𝑎 =?,
A.The thick thumbler is less likely to crack 𝛾𝑎 =
𝑚𝑎𝑠𝑠 𝑜𝑓 𝑙𝑖𝑞𝑢𝑖𝑑 𝑒𝑥𝑝𝑒𝑙𝑙𝑒𝑑
=
𝑀𝑒
,
because it is stronger B.The thick thumbler is less 𝑚𝑎𝑠𝑠 𝑜𝑓 𝑙𝑖𝑞𝑢𝑖𝑑 𝑟𝑒𝑚𝑎𝑖𝑛𝑖𝑛𝑔 ×𝑡𝑒𝑚𝑝. 𝑟𝑖𝑠𝑒
2
𝑀𝑟 ×∆𝜃

likely to cracks because it insulatea the heat 𝛾𝑎 = = 0.00625𝐾 −1 = 6.25×10-3𝐾 −1 .


8×40
better, glass being a poor conductor of heat 53. A relative density bottle of volume 50cm3 is
C.Both thumblers are equally likely to crack completely filled with a liquid at 30℃. It is then
because they are made of the same kind of glass heated to 80℃ such that 0.75cm3 of the liquid is
D.The thick thumble is more likely to crack expelled. Calculate the apparent cubic
because of uneven expansion persists for a expansivity of the liquid A.0.00030𝑲−𝟏
longer time the thicker glass E.The thick B.0.00032𝐾 C.0.01970𝐾 D.0.02030𝐾 −1 .
−1 −1

thumbler is more likely to crack because it is Initial volume=50cm3, volume expelled


heavier. 𝑉𝑒 =0.75cm3, volume remaining 𝑉𝑟 = 50−0.75 ,
49. What is likely to happen if the glass of a 𝑉𝑟 =49.25cm3, 𝜃1 =30℃, 𝜃2 =80℃, 𝛾𝑎 =?,
thermometer expands more upon heating than 𝛾𝑎 =
𝑣𝑜𝑙𝑢𝑚𝑒 𝑜𝑓 𝑙𝑖𝑞𝑢𝑖𝑑 𝑒𝑥𝑝𝑒𝑙𝑙𝑒𝑑
,
the liquid inside? A.The glass will shrink B.The 𝑣𝑜𝑙𝑢𝑚𝑒 𝑜𝑓 𝑙𝑖𝑞𝑢𝑖𝑑 𝑟𝑒𝑚𝑎𝑖𝑛𝑖𝑛𝑔 ×𝑡𝑒𝑚𝑝. 𝑟𝑖𝑠𝑒
𝑉𝑒 𝑉𝑒 0.75
liquid will go down in the stem C.The liquid will 𝛾𝑎 = = = ,
𝑉𝑟×∆𝜃 𝑉𝑟 ×(𝜃2 −𝜃1 ) 49.25×(80−30)
rise up in the stem D.The will break 0.75
𝛾𝑎 = = 0.00030𝐾 . −1
Expansion of the glass makes its volume to 49.25×50
increase, hence the liquid level falls down the 54. A specific gravity bottle contains 0.055kg of a
stem to fill the increased volume of the glass. liquid at 20℃ and 0.050kg at 70℃. If the cubic
50. Which of the following statement is /are expansivity of the glass is 0.00003𝐾 −1 ,
correct? I.When a liquid expands its density falls determine the real cubic expansivity of the liquid
II.The apparent expansion of a liquid is greater A.0.00183𝐾 −1 B.0.00192𝐾 −1 C.0.00203𝑲−𝟏
than its real cubic expansion III.When a liquid is D.0.00205𝐾 −1 .
heated to a sufficiently high temperature it Mass of liquid at 20℃=0.055kg, mass of liquid at
changes into a solid A.I B.II C.III D.I and II E.II and 70℃=mass of liquid remaining 𝑀𝑟 =0.050kg,
III. mass of liquid expelled 𝑀𝑒 =0.055−0.050,
When a liquid expands, its volume increases and 𝑀𝑒 =0.005kg, 𝜃1 =20℃, 𝜃2 =70℃, 𝛾𝑎 =?,
𝑚𝑎𝑠𝑠 𝑜𝑓 𝑙𝑖𝑞𝑢𝑖𝑑 𝑒𝑥𝑝𝑒𝑙𝑙𝑒𝑑
its density decreases or falls.The apparent cubic 𝛾𝑎 = ,
𝑚𝑎𝑠𝑠 𝑜𝑓 𝑙𝑖𝑞𝑢𝑖𝑑 𝑟𝑒𝑚𝑎𝑖𝑛𝑖𝑛𝑔 ×𝑡𝑒𝑚𝑝. 𝑟𝑖𝑠𝑒
expansion of is less than its real cubic expansivity 𝑀𝑒 𝑀𝑒 0.005
𝛾𝑎 = = = ,
or the real cubic expansivion of a liquid is greater 𝑀𝑟 ×∆𝜃 𝑀𝑟 ×(𝜃2 −𝜃1 ) 0.050×(70−20)
0.005
than its apparent cubic expansion. When a liquid 𝛾𝑎 = = 0.002𝐾 −1 , 𝛾𝑟 = 𝛾𝑎 + 𝛾𝑐 ,
0.050×50
is heated to a sufficiently high temperature it 𝛾𝑐 = 0.00003𝐾 −1 , 𝛾𝑟 =0.00003+0.002,
changes into a gas. 𝛾𝑟 = 0.00203𝐾 −1 .
51. The cubic expansivity of mercury is 1.8×10- 55. The temperature of a glass vessel containing
4𝐾 −1 and the linear expansivity of glass is
100cm3 of mercury is raised from 10℃ to 100℃.
8.0×10-6𝐾 −1 , calculate the apparent cubic Calculate the apparent cubic expansion of the
expansivity of mercury in a glass container mercury. (Real cubic expansivity=1.82×10-4𝐾 −1 ,
A.1.00×10-4𝐾 −1 B.1.56×10-4𝑲−𝟏 C.1.72×10- cubic expansivity of glass=2.4×10-5𝐾 −1 )
4𝐾 −1 D.2.04×10-4𝐾 −1 .
A.0.52cm3 B.1.42cm3 C.1.87cm3 D.5.22cm3
Real cubic expansivity of mercury 𝛾𝑟 =1.8×10- E.1.22cm3.
4𝐾 −1 , linear expansivity of glass 𝛼𝑐 =8.0×10- Original volume 𝑉1 =100cm3 , 𝜃1 =10℃, 𝜃2 =100℃,
6𝐾 −1 , cubic expansivity of glass container 𝛾𝑟 =1.82×10-4𝐾 −1 , 𝛾𝑐 =2.4×10-5𝐾 −1 ,apparent
𝛾𝑐 = 3𝛼𝑐 = 3×8.0×10-6=24×10-6 = 0.24×10-4𝐾 −1 , cubic expansion=apparent increace in volume
apparent cubic expansivity 𝛾𝑎 =?, 𝛾𝑟 = 𝛾𝑎 + 𝛾𝑐 , 𝑎𝑝𝑝𝑎𝑟𝑒𝑛𝑡 𝑖𝑛𝑐𝑟𝑒𝑎𝑠𝑒 𝑖𝑛 𝑣𝑜𝑙𝑢𝑚𝑒 ∆𝑉
∆𝑉=?, 𝛾𝑎 = = ,
𝛾𝑎 = 𝛾𝑟 − 𝛾𝑐 =1.8×10-4−0.24×10-4 , 𝑜𝑟𝑖𝑔𝑖𝑛𝑎𝑙 𝑣𝑜𝑙𝑢𝑚𝑒 ×𝑡𝑒𝑚𝑝. 𝑟𝑖𝑠𝑒 𝑉1 (𝜃2 −𝜃1 )
𝛾𝑎 = 1.56×10-4𝐾 −1 . ∆𝑉 = 𝛾𝑎 𝑉1 (𝜃2 − 𝜃1 ) , 𝛾𝑎 = 𝛾𝑟 − 𝛾𝑐 ,
52. The mass of an empty density bottle is 30g. ∴ ∆𝑉 = (𝛾𝑟 − 𝛾𝑐 )𝑉1 (𝜃2 − 𝜃1 ) ,
When filled with a liquid, the mass of the bottle ∆𝑉 = (1.82×10-4−2.4×10-5)×100(100−10) ,
and the liquid is 40g. On heating the filled bottle ∆𝑉 = 1.58×10-4×100×90 = 1.42cm3.

129
Demystified Series Physics Demystified by Dr Timothy
56. Water shows anomalous behaviour A.below The graph above show the expansion of water as
0℃ B.between 0℃ and 4℃ C.at exactly 4℃ the temperature increases from 0℃. Which of the
D.between 4℃ and 100℃ E.above 100℃. following deductions from the graph are true?
57. In what range of temperature is the I.water has its maximum density at Q II.the
expansion of water anomalous? A.+208℃ to volume of water is greater at 0℃ than at 4℃
+212℃ B.−80℃ to −76℃ C.−40℃ to 0℃ III.the volume of water decreases uniformly
D.+96℃ to +100℃ E.0℃ to +4℃. when cooled from 100℃ to 0℃ IV.when water
58. Fishes can survive in the sea when the solidifies, its volume increases A.I and II B.I and
temperature of the atmosphere is below 0℃ III C.II and III D.I,II and III E.I,II and IV
because A.their body temperature can melt the Water has its minimum volume i.e 1.000cm3 at
ice B.only a thin layer of the water at the bottom 4℃ hence, its density will be maximum at 4℃.
freezes C.only a thin layer of the water at the The volume of water at 0℃ is 1.002cm3 while its
top freezes D.the water never freezes at all E.the volume at 4℃ is 1.000cm3 hence, its volume at
salts in sea water keep it warm. 0℃ is greater than its volume at 4℃. The volume
Anomlous behaviour of water makes water to of water decreases when cooled from 100℃ to
freeze from its surface downwards. As the water 4℃ and then increases from 4℃ to 0℃. When
cools from 4℃ to 0℃, it expands and its density water solidifies i.e changes to ice, its volume
decreases, hence it rises to the water surface increases.
where it freezes at 0℃ while the water below 64. The graph of density against temperature for
remains as liquid. Therefore aquatic life e.g water is correctly illustrated by
fishes, can survive in deep layers of water in sea. A. Density B. Density
59. When left in a freezer, a bottle full of water
cracks on freezing into ice because of the
A.decrease in the volume of water B.contraction
of the bottle C.expansion of the bottle D.increase
in the volume of water. 0 4 Temp 0 4 Temp
Water expands on freezing to ice hence, the
volume of the ice now becomes greater than the C. Density E. Density
volume of the bottle making it to crack.
60. At 4℃, the volume of a fixed mass of water is
A.constant B.maximum C.minimum D.zero
Water has a maximum density at and minimum
volume at 4℃ and has a minimum density and 0 4 Temp 0 4 Temp.
maximum volume at 0℃. The density of water increases from 0℃ to 4℃
61. If the temperature of water is gradually and decreases above 4℃, due to anomalous
increased from 0℃ to 4℃, the density water expansion as shown in option C.
within this range A.increases for a while and
then decreases B.decreases for a while and then Jamb past questions on thermal expansivity :
increases C.increase gradually D.decreases [1978/22,1979/9,34,1980/44,1982/22,1983/
gradually E.remains the same 22,1982/16,1983/42,1985/18,22,1986/18,
The density of water increases from 0℃ to 4℃ 1987/18,1990/16,1991/16,1992/14,15,1993/
and is maximum at 4℃. Above 4℃, water 15,1994/23,1995/19,1999/18,2000/21,25,
behaves like other liquids when heated i.e its 2004/33,38,2005/43,48,2006/44,2007/6,28,36
volume increases whileits density decreases. ,2008/19,2009/19,2010/16,2011/23,2013/17,
62. As water is cooled from 10℃ to 2℃, its 2014/20,2015/46]
density A.increases B.decreases C.first decreases
then increases D.first increases and then
decreases.
The density of water is maximum at 4℃, hence on
cooling from 10℃ its density increases till it
reaches 4℃. When cooled below 4℃, its density
decreases. Q
63. Vol(cm3)
1.008
1.006
1.004 P
1.002 Q
1.000

0 2 4 6 8 10 12 14 Temp(℃)

130
Demystified Series Physics Demystified by Dr Timothy

Chapter 13 – Thermal energy, Heat Capacity,


Latent heat and Vapours
● Thermal or heat energy 𝑯 – This is the transferred from the hot to cold body i.e heat lost
amount or quantity of heat required to raise or by hot body is equal to the heat gained by cold
reduce the temperature of a body. It is the energy body, until both attain an equilibrium or same
required to change the average position or temperature or thermal equilibrium.
kinetic energy of molecules of a substance. Heat lost by hotter body 𝐻𝐻 = Heat gained by
- Thermal or Heat capacity 𝑪 – is the quantity colder body 𝐻𝐶 , 𝒎𝑯 𝒄𝑯 ∆𝜽𝑯 = 𝒎𝑪 𝒄𝑪 ∆𝜽𝑪 ,
of heat required to raise the temperature of a 𝑚𝐻 =mass of hotter body, 𝑐𝐻 =specific heat
body by 1𝐾 or 1℃. Thermal or heat capacity is capacity of hotter body, ∆𝜃𝐻 = 𝜃𝐻 − 𝜃,
also the quantity heat lost by a body when its ∆𝜃𝐻 =temperature change of hotter body,
temperature falls by 1𝐾. 𝑯 = 𝑪∆𝜽 , 𝑪 =
𝑯
. 𝜃𝐻 =temperature of hotter body, 𝑚𝐶 =mass of
∆𝜽
colder body, 𝑐𝐶 =specific heat capacity of colder
𝐻=quantity of heat, 𝐶=heat or thermal capacity
body, ∆𝜃𝐶 = 𝜃 − 𝜃𝐶 , 𝜃𝐶 =temperature of colder
(𝐽𝐾 −1 ), ∆𝜃=temperature rise= (𝜃2 − 𝜃1 ).
body, 𝜃=equilibrium temperature or
- Specific heat capacity 𝒄 – is the quantity of heat
temperature of mixture or final temperature.
required to raise the temperature of a unit (1kg)
Hence, 𝒎𝑯 𝒄𝑯 (𝜽𝑯 − 𝜽) = 𝒎𝑪 𝒄𝑪 (𝜽 − 𝜽𝑪 ) .
mass of a body by 1𝐾 or 1℃. Specific heat
- Waterfall from a height – When water falls
capacity is also the quantity of heat lost by a unit
from a height in a dam, the potential energy of at
(1kg) mass of a body when its temperature falls
𝑯 the top and the kinetic energy of falling water is
by 1𝐾 or 1℃. 𝑯 = 𝒎𝒄∆𝜽 , 𝒄 = , 𝑐=specific converted to heat energy(internal energy) at the
𝒎∆𝜽
heat capacity (𝐽𝑘𝑔 𝐾 or 𝐽𝑔 𝐾 ), 𝑚=mass
−1 −1 −1 −1
bottom of the dam,hence the water becomes
of the body. slightly warm due to slight increase in
- The specific heat capacity is constant for temperature. 𝑷. 𝑬 = 𝑯, 𝑲. 𝑬 = 𝑯,
substances of the same material but differ for 𝟏
𝒎𝒈𝒉 = 𝒎𝒄∆𝜽 or 𝒎𝒗𝟐 = 𝒎𝒄∆𝜽. 𝑚=mass of
substance of different materials. 𝟐
water, 𝑔=acceleration due to gravity (10ms-2),
- Thermal capacity 𝐶 and specific heat capacity
ℎ=height of waterfall, 𝑣=velocity or speed of
are related by – 𝑪 = 𝒎𝒄.
waterfall.
- When equal masses or weights of bodies are
- A calorimeter is an apparatus used in
supplied with the same amount of heat, the body
experiments that involve exchange of heat
with a higher thermal capacity or specific heat
between two bodies at different temperature.
capacity have lower temperature difference or
- A good calorimeter should be made of a
temperature rise, as they are inversely
𝟏 𝟏 material e.g copper or aluminium, with high
proportional , 𝑪 ∝ or 𝒄 ∝ . thermal or heat conductivity and low specific
∆𝜽 ∆𝜽
- Thermal equilibrium is a condition where two heat capacity.
bodies in thermal contact are at the same - Methods of reducing heat loss in a
temperature and there is no net flow of heat calorimeter – I.Lagging the calorimeter i.e
between both bodies. surrounding it with an insulator e.g cotton wool
- Measurement of specific heat capacity is done or felt,to reduce heat loss by conduction. II.Air
by – Electrical method, Mixture method. insulating lid which prevents heat loss by
I. Electrical method or Electrical calorimetry : convention and evaporation. III.Polishing the
When an electrical device (heater or kettle) is outer surface to reduce heat loss by radiation.
used to supply heat to a liquid or object, the heat IV.Shielding the calorimeter from draught.
supplied by the heater equals the heat gained by - Calorific value of a substance is the therma;
the liquid or object. The heat supplied by the energy required per mass or volume of the
electrical device can be calculated using : source. Calorific value is in 𝐽𝑔−1 or 𝐽/m3.
𝑽𝟐 𝒕 𝑻𝒉𝒆𝒓𝒎𝒂𝒍 𝒆𝒏𝒆𝒓𝒈𝒚 𝒓𝒆𝒒𝒖𝒊𝒓𝒆𝒅
𝑷𝒕 or 𝑰𝑽𝒕 or or 𝑰𝟐 𝑹𝒕. Calorific value = ,
𝑹 𝒎𝒂𝒔𝒔 𝒐𝒓 𝒗𝒐𝒍𝒖𝒎𝒆 𝒐𝒇 𝒕𝒉𝒆 𝒔𝒐𝒖𝒓𝒄𝒆
Heat supplied by electrical heater = Heat gained Calorific value =
𝑯
=
𝒎𝒄∆𝜽
.
𝑽𝟐 𝒕 𝒎 𝒐𝒓 𝑽 𝒎 𝒐𝒓 𝑽
by substance. 𝑷𝒕 = 𝑰𝑽𝒕 = = 𝑰𝟐 𝑹𝒕 = 𝒎𝒄∆𝜽 . - 1 calorie (1cal) is equal to 4.2J.
𝑹
𝑃=electrical power or rate of heat supplied or
rating of the heater, 𝑡=time, 𝐼=current. ● Latent heat and change of state
𝑉=potential difference or voltage, 𝑅=resistance. - Latent heat also called hidden heat of a
II. Mixture method – When a hotter body (body substance is the heat gained or lost to change the
at high temperature) is added to or mixed with a state of that substance at the same temperature
colder body (body at low temperature), heat is (without a change in temperature). It is the heat

131
Demystified Series Physics Demystified by Dr Timothy
supplied or removed from a substance with no melting and expands(increase in volume) on
temperature difference. Latent heat is the energy freezing or solidification.
required to keep the molecules of a substance in - Density of substance increase on melting and
their mean position or their potential energy. decreases on freezing or solidification.
- Latent heat causes a change in the density and - Increase in pressure and presence of impurities
volume of a substance without any change in decreases the melting point or freezing point of a
mass. substance.
- Specific latent heat 𝑳 is the heat supplied or - The effect of pressure on the melting point of a
removed to change the state of a unit (1kg) mass solid is shown using Regelation or Refreezing
of a substance at a constant temperature. experiment. Regelation is the refreezing of
𝑯
𝑯 = 𝒎𝑳 , 𝑳 = (J𝒌𝒈−𝟏 ). asubstance after melting by the application of
𝒎
pressure.
- Latent heat of fusion is the quantity of heat
- When a ice cubes are introduced into a
required to convert a solid to its liquid state at
container filled to the brim and the ice
melting point at constant temperature.
floats,when the ice cube melts the water level
- Specfic latent heat of fusion 𝑳𝒇 is the heat
remain the same as the ice cube displaces its
required to change the state of 1kg mass of a solid
volume of water equal to its volume before
substance to its liquid state at its melting point at
floating which is now filled when it melts
constant temperature.
- When a container is filled with ice cubes to the
- Specific latent heat of vaporization 𝑳𝒗 is the
brim, when the ice melts, the water level drops
heat required to change the state of 1kg mass of
because ice occupies more volume than water
a liquid substance to its gaseous state at its
(anomalous expansion) and due to the fact that
boiling point at constant temperature.
the air spaces between them is completely filled
- Specific latent heat of fusion of a substance
with water when it melts.
is less than its specific latent heat of - Evaporation is a process where a liquid turns
vaporization and vice-versa, because the
spontaneously into vapour or gaseous state
intermolecular bonds in a solid is reduced when
below its boiling point. It is the escape of
transformed from solid to liquid but is
molecules at the liquid surface with energy above
completely broken when transformed from
the average kinetic energy from the liquid.
liquid to gas.
- Evaporation occurs at all temperatures.
- The apparatus involved in determining the
Evaporation is the change from liquid to
specific latent heat of vaporization includes:
unsaturated vapour and brings about cooling.
I.Boiling flask:Where water is heated to produce
- Rate of evaporation 𝑬 is affected by nature of
wet steam. II.Connecting tube:Wet steam passes
the liquid, wind or force convention of air over a
through it to the water or steam trap. Steam or liquid 𝑊, surface area of liquid exposed 𝐴,
water trap dries the steam and collects the
humidity or dryness of air 𝐻, temperature T,
water that condenses. 𝑾𝑨𝑻
- When heat is supplied from an electrical heater pressure 𝑃 and density of the liquid 𝜌, 𝑬 ∝ .
𝑷𝝆𝑯
to cause a change of state, then – Rate of evaporation of liquid increases as
𝑷𝒕 = 𝑰𝑽𝒕 =
𝑽𝟐 𝒕
= 𝑰𝟐 𝑹𝒕 = 𝒎𝑳. wind, area and temperature increases while
𝑹 pressure, density and humidity decreases.
- Change of state – Matter exists as solid, liquid - Blowing air over a liquid increases the
and gas. Change of state occurs at constant surface area. -
temperature and latent heat is either gained or Rate of evaporation is high in an unsaturated
lost. environment than a saturated environment,
- Change of state is accompanied by change in as the air can still accommodate more vapour.
density,volume and heat content without any - Rate of evaporation is not affected by
change in the mass and temperature. osmotic or hydrostatic pressure.
- During change of state from solid→liquid→ gas - When a volatile liquid e.g ether, methylated
– latent heat is absorbed or supplied. spirit or alcohol, is poured on the skin or hands,
- During change of state from gas→liquid→ solid latent heat of vaporization is drawn from the skin
– latent heat is removed or given off. making the person feel cool.
- Melting or fusion point of a substance is the - Volatile liquids have low boiling points and
temperatureat which the solid substance low latent heat.
changes into the liquid state. - Steam causes a more severe burning or
- When a solid substance is heated to its melting damage on the skin because : I.the heat content
point, the temperature increases before and after of steam or internal energy (thermal energy and
melting while temperature remains constant latent heat) is greater that of boiling
during melting. water(thermal energy only). II.Steam is at a
- Substance contracts(decrease in volume) on

132
Demystified Series Physics Demystified by Dr Timothy
higher temperature than boiling water. boiling point 𝑇𝐵 of the liquid.
Condition (i) is most acceptable. Section DE – It represents the gaseous or vapour
- Refrigerators works on the cooling effect of phase. Temperature changes when heated or
evaporation. The latent heat generated in a cooled.
refrigerator is removed by conduction, The heat supplied or removed in section OA, BC
convention and radiation to the surrounding. A and DE is thermal energy. The heat supplied or re
thermostat inside a refrigerator controls the rate supplied or removed in section AB and CD are
of vaporization and degree of cooling. latent heat of fusion and vaporization
- Boiling point or Ebullition point of a liquid is repectively.
the temperature at which the vapour pressure of
the liquid equals the atmospheric or external ●Vapours (Saturated and Unsturated)
pressure. - A liquid heated in a closed vessel, evaporates
- Boiling at higher temperature than the boiling and vapour is formed in the space above the
point means the atmospheric pressure is liquid. The vapour molecules exerts pressure on
increased, it has to beheated rapidly for the the liquid surface and the walls of the vessel
vapour pressure to equalize it. called vapour pressure.
- At normal atmospheric pressure(760mmHg or - When the space above the liquid is crowded
1atm),water boils or evaporates at 100℃. with vapour as the container is closed, initially
- Boiling point increases with pressure and the rate of vaporization exceeds the rate of
presence of impurities. Pressure increases with condensation, the vapour is said to be
liquid below sea level and decreases with liquid unsaturated at this point. A point is attained
above sea level. where the rate of vaporizaation equals the rate of
- Pressure cooker saves both time and fuel in condensation, the vapour is said to be saturated.
cooking because inside the cooker the boiling - Unsaturated vapour is a vapour that can still
point of water is raised due to increased hold more vapour molecules within a cofined
pressure. space. The number of molecules leaving a liquid
- Evaporation and Boiling both involves a change per second is more than the number of molecules
of state from liquid to gaseous state. rerturning to the liquid per second i.e rate of
- Evaporation occurs at any temperature while vaporization exceeds the rate of condensation.
Boiling occurs at a definite temperature. The vapour is not in dynamic equilibrium with its
Evaporation occurs at the surface of a liquid own liquid in a cofined space (vapour formed
(surface phenomenon) while Boiling involes the during evaporation).
whole volume or throughout the body of the - Saturated vapour is a vapour which can no
liquid. longer hold any more vapour molecules within a
- Evaporation depends on wind and surface area cofined space. The number of molecules escaping
of liquid while Boiling is independent of wind and per second equals the number of molecules
surface area of liquid. returning per second i.e rate of vaporization
- Heating and cooling curves – Curves which equals rate of condensation.The saturated
shows the variation of temperature with time of vapour is in in dynamic equilibrium with its own
a substance when heated or cooled to a change in liquid.
state. - Unsaturated vapours obey gas law (Boyle’s
Temperature Temperature law and Charles’ law) while Saturated
E E vapours do not obey gas laws (Boyle’s law and
𝑻𝑩 C 𝑻𝑩 D C Charles’ law).
D
- Saturated vapour pressure 𝑺. 𝑽. 𝑷 or
𝑻𝑴 A B 𝑻𝑴 B A equilibrium vapour pressure is the pressure
O exerted by molecules of a saturated vapour at at
O Time Time a definite temperature.
Section OA – It represents the solid phase. - Saturated vapour pressure is affected by
Temperature changes when heated or cooled. temperature only. Saturated vapour pressure
Section AB – It represents the melting or increases with temperature i.e 𝑺. 𝑽. 𝑷 ∝ 𝑻.
freezing region i.e a mixture of solid and liquid. - Saturated vapour pressure at any given
Temperature is constant here and is equal to the temperature is independent of the volume of
melting point 𝑇𝑀 . the vapour or external pressure.
Section BC – It represents the liquid phase. 𝑺. 𝑽. 𝑷(mmHg)
Temperature changes when heated or cooled.
Section CD – It is the boiling or liquefaction
region i.e a mixture of liquid and gas(vapour).
Temperature is constant and is equal to the Temp(℃)

133
Demystified Series Physics Demystified by Dr Timothy
- A liquid boils when its saturated vapour atmosphere or the mass of water vapour in a unit
pressure 𝑆. 𝑉. 𝑃 equals the external atmospheric volume of the air in the atmosphere.
pressure 𝐴. 𝑃. - A wet or moist air is referred to as a humid air.
- Boiling point or ebullition point of a liquid is - Relative humidity 𝑹. 𝑯is the ratio of the mass
the temperature at which the saturated of water vapour present in a certain volume of air
vapour pressure of the liquid equals the to the mass of water vapour required to saturate
external atmospheric pressure. the same volume of air ar at the same
- A liquid with high 𝑺. 𝑽. 𝑷 evaporates faster temperature. Relative humidity is the degree of
and reaches its boiling point quickly that a humidity of air or the nearness of the water
liquid with low 𝑺. 𝑽. 𝑷. vapour present in atmosphere to saturation.
- The pressure of water vapour in air can never - Relative humidity is expressed in percentage
exceeds the 𝑆. 𝑉. 𝑃. 𝑹. 𝑯 =
𝒎𝒂𝒔𝒔 𝒐𝒇 𝒘𝒂𝒕𝒆𝒓 𝒗𝒂𝒑𝒐𝒖𝒓 𝒊𝒏 𝒂 𝒈𝒊𝒗𝒆𝒏 𝒗𝒐𝒍. 𝒐𝒇 𝒂𝒊𝒓
,
- Sublimation is the change of solids substance 𝒎𝒂𝒔𝒔 𝒐𝒇 𝒘𝒂𝒕𝒆𝒓 𝒗𝒂𝒑𝒐𝒖𝒓 𝒓𝒆𝒒𝒖𝒊𝒓𝒆𝒅 𝒕𝒐 𝒔𝒂𝒕𝒖𝒓𝒂𝒕𝒆
𝑺.𝑽.𝑷 𝒐𝒇 𝒘𝒂𝒕𝒆𝒓 𝒂𝒕 𝒅𝒆𝒘 𝒑𝒐𝒊𝒏𝒕
directly to the gaseous state. 𝑹. 𝑯 = ,
𝑺.𝑽.𝑷 𝒐𝒇 𝒘𝒂𝒕𝒆𝒓 𝒂𝒕 𝒐𝒓𝒊𝒈𝒊𝒏𝒂𝒍 𝒂𝒊𝒓 𝒕𝒆𝒎𝒑𝒆𝒓𝒂𝒕𝒖𝒓𝒆
- Condensation is the change from the gas 𝒗𝒂𝒑𝒐𝒖𝒓 𝒑𝒓𝒆𝒔𝒔𝒖𝒓𝒆( 𝒑𝒂𝒓𝒕𝒊𝒂𝒍 𝒑𝒓𝒆𝒔𝒔𝒖𝒓𝒆) 𝒐𝒇 𝒂𝒊𝒓
𝑹. 𝑯 = ,
(vapour) to the liquid state. 𝑺.𝑽.𝑷 𝒂𝒕 𝒕𝒉𝒆 𝒔𝒂𝒎𝒆 𝒕𝒆𝒎𝒑𝒆𝒓𝒂𝒕𝒖𝒓𝒆 𝒐𝒇 𝒕𝒉𝒆 𝒂𝒊𝒓
- Condensation of water vapour in the - Relative humidity 𝑹. 𝑯 increases with 𝑺. 𝑽. 𝑷
atmosphere – The amount of water vapour air at dew point and decreases with 𝑺. 𝑽. 𝑷 at air
can hold decreases with a decrease in temperature.
temperature. Hence, cooling air below saturation - High humidity – Wet/moist atmosphere
or lowering of temperature decreases its ability during rainy or winter season.
to hold water, which results the formation of - Low humidity – Dry aymosphere during
water droplets in form of dew, mist, fog, frost and harmattan or summer season.
cloud. - Relative humidity is measured with a
1.Dew is moisture or water droplets formed on hygrometer e.g wet and dry bulb hygrometer.
when air is cooled below a critical temperature - Relative humidity is inversely proportional
due to variation in temperature between the day to temperature difference between the wet
𝟏
and night. Dew occurs due to land breezes that and dry bulb hygrometer, 𝑹. 𝑯 ∝ i.e relative
∆𝑻
occur at night. humidity 𝑹. 𝑯 is greatest when the difference
2.Mist is thin layer tiny water droplets close to in temperature between the dry and wet bulb
the earth surface formed by cooling air above but hygrometer ∆𝑻 is least.
near the earth.
3.Fog is a thick and dense mist which reduces Examples :
visibility. Car drivers put on their head lamps and 1. The amount of heat required to raise the
drives slowly due to impaired vision caused by temperature of a body is A.thermal capacity
fog. B.specific heat capacity C.thermal energy D.heat
4.Cloud is water droplets or ice crystals high up loss E.thermal conduction.
in the sky or atmosphere i.e floats in air due to 2. The amount of heat needed to raise the
the small size, formed by condensation of water temperature of 10kg of copper by1𝐾 is its
vapour in the atmosphere. A.internal energy B.heat capacity C.specific heat
5.Rain is water droplets which falls from large capacity D.latent heat.
clouds. Heat capacity is amount of heat required to raise
6.Frost is tiny crystals of ice formed on surfaces, the whole mass of a body by 1𝐾 or 1℃.
when water vapour changes directly to ice below 3. The quantity of heat required to raise the
the freezing point due to fall in air temperature. temperature of unit mass of a substance thrpugh
7.Hail is cloud of large water droplets which unit temperature is A.specific heat capacity
solidifies to ice pellets on falling to the ground. B.specific heat of sublimation C.specific heat of
- Dew point is the temperature at which the fusion D.specific heat of fission.
water vapour present in air is just sufficient 4. Equal masses of copper and rubber are raised
to saturate it. to the same temperature. After sometimes, the
- Dew point is dependent on the prevailing copper was observed to be at a lower
atmospheric conditions like temperature, temperature because A.the specific heat capacity
wind and the amount of water vapour in the of copper is lower than that of rubber B.copper
atmosphere. expand more than rubber C.the specific heat
- When air is cooled below dew point, water capacity of rubber is lower than that of
vapour condenses. Dew point is always less copper D.rubber expands more than copper.
than air temperature. Bodies with higher specific capacity has a lower
- Humidity is the amount of water vapour in the 1
temperature difference, 𝑐 ∝ . Hence, copper is
∆𝜃

134
Demystified Series Physics Demystified by Dr Timothy
at a lower temperature than rubber, because the water=4200J𝑘𝑔−1 𝐾 −1 ) A.4.57minutes
specific capacity of copper is higher than that of B.5.02minutes C.7.30minutes D.10.04minutes.
rubber or the specific heat capacity of rubber is 𝑚=1.5kg, 𝜃1 =30℃, 𝜃2 =80℃,𝐼=5𝐴, 𝑉=230𝑉,
lower than that of copper. 𝑐=4200J𝑘𝑔−1 𝐾 −1 , 𝑡=?,
5. When equal weights of iron and water are Heat supplied by electric kettle = Heat gained by
subjected to an equal supply of heat, it is found water. 𝐼𝑉𝑡 = 𝑚𝑐∆𝜃 = 𝑚𝑐(𝜃2 − 𝜃1 ),
that the piece of iron becomes much hotter than 5×230× 𝑡 = 1.5×4200(80−30),
water after a shorter time because A.the specific 1150𝑡 = 6300×50, 𝑡 =
6300×50
= 273.91s
1150
heat of iron is higher than that of water B.the 273.91
specific heat of water is higher than that of 𝑡= = 4.57min.
60
iron C.the specific heat of iron is infinite D.iron is 11. A electrical heater 220𝑉, 1000W is immersed
in solid form E.water is in liquid form into a bucket full of water.Calculate the mass of
Bodies with higher specific capacity has a lower the water if the temperature changes from 30℃
temperature difference, 𝑐 ∝
1
. Hence, iron is to 100℃ and current flows for 300seconds.
∆𝜃 (specific heat capacity of water=4200J𝑘𝑔−1 𝐾 −1 )
much hotter i.e at higher temperature, than
A.428kg B.42.86kg C.1.02kg D.7.14kg.
water, because the specific heat capacity of iron
𝑉=220𝑉, 𝑃=1000W, 𝜃1 =30℃, 𝜃2 =100℃,𝑡=300s,
is lower than that of water or specific heat
𝑐=4200J𝑘𝑔−1 𝐾 −1 , 𝑚=?,
capacity of water is higher than that of iron.
Heat supplied by heater = Heat gained by water.
6. How much heat is given out when a piece of
𝑃𝑡 = 𝑚𝑐∆𝜃 = 𝑚𝑐(𝜃2 − 𝜃1 ),
iron of mass 0.0500kg and specific heat capacity
1000×300 = 𝑚 ×4200(100−30),
460J𝑘𝑔−1 ℃−1 cools from 80.0℃ to 20.0℃?
300000 = 𝑚 ×4200×70 = 294000𝑚,
A.1380J B.2760J C.13800J D.23000J E.27600J. 300000
𝑚=0.0500kg, 𝑐=460𝐽𝑘𝑔−1 ℃−1 , 𝜃1 =20.0℃, 𝑚= = 1.02kg.
294000
𝜃2 =80.0℃, 𝐻=?, 𝐻 = 𝑚𝑐∆𝜃 = 𝑚𝑐(𝜃2 − 𝜃1 ), 12. A block of aluminium of mass 𝑚 is heated
𝐻 = 0.05×460(80−20) = 0.05×460×60=1380J . electrically by 25W heater. If the temperature
7. A copper ball of heat capacity 400𝐽𝐾 −1 is rises by 10℃ in 5minutes. Calculate the heat
heated fro, 20℃ to 100℃. Calculat ethe quantity capacity of aluminium A.1250J𝐾 −1 B.750J𝑲−𝟏
heat absorbed A.40.0kJ B.32.0kJ C.2.0kJ D.8.0k𝐽 C.125J𝐾 −1 D.50J𝐾 −1 .
E.3.2kJ. 𝑃=25𝑊, ∆𝜃=10℃, 𝑡=5min=300s, 𝐶=?,
𝐶=400𝐽𝐾 −1 , 𝜃1 =20℃, 𝜃2 =100℃, 𝐻=?, Heat supplied by heater = Heat gained by water.
𝐻 = 𝐶∆𝜃 = 𝐶(𝜃2 − 𝜃1 ) = 400(100−20) , 𝑃𝑡 = 𝐶∆𝜃, 25×300 = 𝐶 ×10,
25×300
𝐻 = 400×80 = 32000J= 32.0kJ. 𝐶= = 750J𝐾 −1 .
10
7. A body of mass 200g and specific heat capacity 13. An immersion heater rated 400𝑊, 220𝑉, is
0.4𝐽𝑔−1 𝐾 −1 cools from 37℃ to 31℃. Calculate used to heart a liquid of mass 0.5kg. If the
the quatiity of heat released by the body A.4800J temperature of the liquid increases uniformly at
B.1200J C.480J D.202J. the rate of 2.5℃ per second, calculate the specific
𝑚=200g, 𝑐=0.4J𝑔−1 ℃−1 , 𝜃1 =31𝐾, 𝜃2 =37𝐾, 𝐻=?, heat capacity of the liquid (Assume no heat is
𝐻 = 𝑚𝑐∆𝜃 = 𝑚𝑐(𝜃2 − 𝜃1 ) = 200×0.4(37−31), lost) A.1100J𝑘𝑔−1 𝐾 −1 B.320J𝒌𝒈−𝟏 𝑲−𝟏
𝐻 = 200×0.4×6 = 480J. C.200J𝑘𝑔 𝐾 D.176J𝑘𝑔−1 𝐾 −1 .
−1 −1
8. A liquid of mass 1000g requires 3.36×105Jof 𝑃=400𝑊, 𝑉=220𝑉, 𝑚=0.5kg, rate of temperature
heat to raise its temperature by 80𝐾. Calculate its ∆𝜃 ∆𝜃
specific heat capacity A.5.25×10-2J𝑔−1 𝐾 −1 increase =2.5℃s-1, 𝑐=?, 𝑃𝑡 = 𝑚𝑐∆𝜃 , 𝑃 = 𝑚𝑐
𝑡 𝑡
400
B.4.20×100J𝒈 𝑲 −𝟏 −𝟏
C.2.41×101J𝑔−1 𝐾 −1 400 = 0.5× 𝑐 ×2.5, 𝑐 = = 320J𝑘𝑔−1 𝐾 −1 .
0.5×2.5
D.3.25×101J𝑔 𝐾 E.2.69×105𝐽𝑔−1 𝐾 −1 .
−1 −1
14. An electrical heater dipped into 500g of
𝑚=1000g, 𝐻=3.36×105J, ∆𝜃=80𝐾, 𝑐=?, water in a plastic beaker raised the temperature
𝐻 3.36×105
𝑐= = = 4.2 or 4.20×100J𝑔−1 𝐾 −1 , from 20℃ to 50℃ in 3minutes. Determine the
𝑚∆𝜃 1000×80
9. A body of mass 𝑚, has a specific heat capacity rate at which heat is given out by the heater
𝑠 and a heat capacity 𝐶. If the temperature of the A.350Js-1 B.700Js-1 C.3500Js-1 D.7000Js-1
body changes by 𝜃℃ , which of the following (specific heat capacity of water =4.2 J𝑔−1 𝐾 −1 ).
equations is correct? A.𝑚𝑠𝜃 = 𝑚𝐶 −1 B.𝒎𝒔 = 𝑪 𝑚=500g, 𝜃1 =20℃, 𝜃2 =50℃, 𝑡=3min=180s,
C.𝑚𝑠 = 𝐶𝜃 D.𝑚𝑠 = 𝑠𝜃 . Heat supplied by heater = Heat gained by water,
Mass 𝑚, specific heat capacity 𝑐=𝑠 , thermal 𝐻 = 𝑚𝑐∆𝜃 = 𝑚𝑐(𝜃2 − 𝜃1 ) = 500×4.2(50−20)
capacity 𝐶, temperature change ∆𝜃=𝜃℃ , 𝐻 = 2100×30 = 63000J .
𝐻 63000
𝐶 = 𝑚𝑐, ∴ 𝐶 = 𝑚𝑠 . Rate of heat given out = Power = = ,
𝑡 180
10. Water of mass 1.5kg is heated from 30℃ to Rate of heat given out =350Js or 350W.
-1
80℃ using electric kettle which is rated 5𝐴, 230𝑉, 15. Study the data below and use it to answer
calculate the time taken to reach the final question 15.
temperature (specific heat capacity of

135
Demystified Series Physics Demystified by Dr Timothy
Mass of metal: 1.5kg, Rating of heating coil: 75W, 19. When two objects 𝑃 and 𝑄 supplied with the
Time of heating: 8minutes, Initial temperature: same quantity of heat, the temperature change in
27℃, Final temperature: 47℃. Calculate the 𝑃 is observed to be twivce that in 𝑄. If the masses
specific heat capacity of the metal of 𝑃 and 𝑄 are the same, calculate the ratio of the
A.2520.0J𝑘𝑔−1 𝐾 −1 B.1200.0J𝒌𝒈−𝟏 𝑲−𝟏 specific heat capacities of 𝑃 to 𝑄 A.1:1 B.1:2 C.1:3
C.250.0J𝑘𝑔−1 𝐾 −1 D.142.9J𝑘𝑔−1 𝐾 −1 D.1:4.
𝑚=1.5kg, 𝑃=75W, 𝑡=8min=480s, 𝜃1 =27℃, For 𝑃 – 𝐻𝑃 = 𝑚𝑃 𝑐𝑃 ∆𝜃𝑃 , 𝐻𝑃 =quantity of heat
𝜃2 =47℃, 𝑐=?, 𝑃𝑡 = 𝑚𝑐(𝜃2 − 𝜃1 ), supplied to 𝑃, 𝑚𝑃 =mass of 𝑃, 𝑐𝑃 =specific heat
75×480 = 1.5× 𝑐(47−27) , 36000 = 30𝑐, capacity of 𝑃, ∆𝜃𝑃 =temperature change in 𝑃,
𝑐=
36000
= 1200.0J𝑘𝑔−1 𝐾 −1 . For 𝑄 – 𝐻𝑄 = 𝑚𝑄 𝑐𝑄 ∆𝜃𝑄 , 𝐻𝑄 =quantity of heat
30
16. A 500𝑊 heater is used to heat 0.6kg of water supplied to 𝑄, 𝑚𝑄 =mass of 𝑄, 𝑐𝑄 =specific heat
from 25℃ to 100℃ in 𝑡1 seconds. If another capacity of 𝑄, ∆𝜃𝑄 =temperature change in 𝑄.
1000𝑊 heater is used to heat 0.2kg of water from 𝐻𝑃 = 𝐻𝑄 , ∆𝜃𝑃 =2∆𝜃𝑄 , 𝑚𝑃 =𝑚𝑄 ,
10℃ to 100℃ in 𝑡2 seconds, find 𝑡1 /𝑡2 A.50 B.5 𝑚𝑃 𝑐𝑃 ∆𝜃𝑃 = 𝑚𝑄 𝑐𝑄 ∆𝜃𝑄 , 𝑐𝑃 (2∆𝜃𝑄 ) = 𝑐𝑄 ∆𝜃𝑄 ,
C.5/4 D.1/5. 𝑐𝑃
=
∆𝜃𝑄 1
= , ∴ 𝑐𝑃 :𝑐𝑄 = 1:2.
Heat supplied by heater = Heat gained by water, 𝑐𝑄 2∆𝜃𝑄 2

𝑃𝑡 = 𝑚𝑐∆𝜃 = 𝑚𝑐(𝜃2 − 𝜃1 ), 20. Two metals 𝑋 and 𝑌 lose the same quantity of
500× 𝑡1 =0.6× 𝑐(100−25) =0.6𝑐 ×75 hear when their temperature dropped from 25℃
0.6𝑐×75 to 20℃. If the specific heat capacity of 𝑋 is hyalf
𝑡1 = = 0.09𝑐 ,
500 that of 𝑌, calculate the ratio of the mass of 𝑋 to
1000× 𝑡2 =0.2× 𝑐(100−10) =0.2𝑐 ×90 that of 𝑌 A.1:2 B.2:1 C.1:3 D.3:1.
0.2𝑐×90 𝑡 0.09𝑐
𝑡2 = = 0.018𝑐. 1 = = 5. 𝐻𝑋 = 𝑚𝑋 𝑐𝑋 ∆𝜃𝑋 , 𝐻𝑌 = 𝑚𝑌 𝑐𝑌 ∆𝜃𝑌 , the two metals
1000 𝑡2 0.018𝑐
17. A 1.2kW immersion heater is used to supply lose the same quantity of heat with the same
energy for 75s. The energy supplied is used to temperature change or drop. ∴𝐻𝑋 = 𝐻𝑌 ,
heat 800g of parrafin in a calorimeter of heat ∆𝜃𝑋 = ∆𝜃𝑌 . 𝑚𝑋 𝑐𝑋 ∆𝜃𝑋 = 𝑚𝑌 𝑐𝑌 ∆𝜃𝑌 ,
1 1
capacity 42.5J𝐾 −1 from 20℃ to 66℃. Calculate 𝑚𝑋 𝑐𝑋 = 𝑚𝑌 𝑐𝑌 , 𝑐𝑋 = 𝑐𝑌 , 𝑚𝑋 × 𝑐𝑌 = 𝑚𝑌 𝑐𝑌 ,
2 2
the specific heat capacity of paraffin 1
𝑚𝑋 = 𝑚𝑌 ,
𝑚𝑋 2
= , 𝑚𝑋 :𝑚𝑌 = 2:1.
A.1651J𝑘𝑔−1 𝐾 −1 B.2369J𝑘𝑔−1 𝐾 −1 2 𝑚𝑌 1

C.2393J𝒌𝒈−𝟏 𝑲−𝟏 D.2446J𝑘𝑔−1 𝐾 −1 21. A piece of copper of mass 0.1kg is heated to


E.2508J𝑘𝑔−1 𝐾 −1 . 100℃ and then trasnsferred to a well lagged
Heat supplied by heater = Heat gained by copper can of mass 0.05kg containing 0.2kg of
calorimeter + Heat gained by paraffin, water at 10℃. Calculate the final steady
𝑃𝑡 = 𝑚𝑐𝑝 (𝜃2 − 𝜃1 ) + 𝐶𝐶 (𝜃2 − 𝜃1 ), temperature of the mixture [Specific heat
𝑃=1.2kW=1200W, 𝑡=75s, mass of paraffin capacity of water=4.2×103J𝑘𝑔−1 𝐾 −1 , specific
𝑚=800g=0.8kg, heat capacity of calorimeter heat capacity of copper= 4.0×102J𝑘𝑔−1 𝐾 −1 ]
𝐶𝐶 =42.5J𝐾 −1 ,𝜃1 =20℃, 𝜃2 =66℃, specific heat A.7℃ B.14℃ C.28℃ D.42℃.
capacity of paraffin 𝑐𝑝 =?, Mass of copper 𝑚𝐶 =0.1kg, initial temperature of
copper 𝜃1 =100℃, mass of copper can 𝑀𝐶 =0.5kg,
1200×75 = 0.8× 𝑐𝑝 (66−20) + 42.5(66−20) ,
mass of water 𝑚𝑊 =0.2kg, initial temperature of
90000 = 36.8𝑐𝑝 + 1955, copper can and water 𝜃𝑜 =10℃ , specific heat
36.8𝑐𝑝 = 90000−1955 = 88045, capacity of copper 𝑐𝐶 =4.0×102𝐽𝑘𝑔−1 𝐾 −1 , specific
88045
𝑐𝑝 = = 2392.5 =2393𝐽𝑘𝑔−1 𝐾 −1 . heat capacity of water 𝑐𝑊 =4.2×103𝐽𝑘𝑔−1 𝐾 −1 ,
36.8
18. Two thermos flasks of volume 𝑉𝑋 and 𝑉𝑌 are final steady temperature of the mixture 𝜃=?,
filled with liquid water at an initial temperature Heat lost by copper = Heat gained by copper can
of 0℃. After some time, the temperature were + Heat gained by water,
𝑉 𝑚𝐶 𝑐𝐶 ∆𝜃1 = 𝑀𝐶 𝑐𝐶 ∆𝜃𝑜 + 𝑚𝑊 𝑐𝑊 ∆𝜃𝑜 ,
found to be 𝜃𝑋 and 𝜃𝑌 repectively. Given that 𝑋 =
𝑉𝑌 ∆𝜃1 = 𝜃1 − 𝜃 , ∆𝜃𝑜 = 𝜃 − 𝜃𝑜 ,
𝜃𝑋 1
2 and = . The rate of the rate of heat flow into 0.1×4.0×102(100− 𝜃)= 0.05×4.0×102(𝜃 −10)
𝜃𝑌 2
the flasks is A.1/4 B.1/2 C.1 D.2 E.1. + 0.2×4.2×103(𝜃 −10) ,
Heat flow in flask 𝑋, 𝐻𝑋 = 𝑚𝑋 𝑐∆𝜃𝑋 , Heat flow in 40(100− 𝜃) = (𝜃 −10)(20+840),
𝑚𝑎𝑠𝑠 40(100− 𝜃) = (𝜃 −10)860,
flow in flast 𝑌, 𝐻𝑌 = 𝑚𝑌 𝑐∆𝜃𝑌 , density = , 860
𝑣𝑜𝑙𝑢𝑚𝑒 (100− 𝜃) = (𝜃 −10) = (𝜃 −10)21.5,
as the liquid in both flask is water and its density 40
is 1gcm-3, volume of the flask can be used in place 100 − 𝜃 = 21.5𝜃 − 215,
of mass. Heat flow in flask 𝑋, 𝐻𝑋 = 𝑉𝑋 𝑐∆𝜃𝑋 , Heat 100 + 215 = 21.5𝜃 + 𝜃 , 315 = 22.5𝜃 ,
315
flow in flask 𝑌, 𝐻𝑌 = 𝑉𝑌 𝑐∆𝜃𝑌 , 𝜃= = 14℃.
22.5
𝐻 𝑉 𝑐∆𝜃 𝑉 𝑐(𝜃 −0)
Ratio of heat flow, 𝑋 = 𝑋 𝑋 = 𝑋 𝑋 = 22. Two liquids 𝑃 at a temperature of 20℃ and 𝑄
𝐻𝑌 𝑉𝑌 𝑐∆𝜃𝑌 𝑉𝑌 𝑐(𝜃𝑌 −0)
𝑉𝑋 𝜃 𝑋 𝑉𝑋 𝜃𝑋 1 at a temperature of 80℃ have specific heat
= × =2 × = 1. capacities of 1.0J𝑘𝑔−1 ℃−1 and 1.5J𝑘𝑔−1 ℃−1
𝑉𝑌 𝜃 𝑌 𝑉𝑌 𝜃𝑌 2

136
Demystified Series Physics Demystified by Dr Timothy
repectively. If equal masses of 𝑃 and 𝑄 are Heat supplied by water at 86℃ = Heat gained by
mixede in a lagged calorimeter, then the water at 30℃. 𝑚𝐻 (𝜃𝐻 − 𝜃) = 𝑚𝐶 (𝜃 − 𝜃𝐶 )
equilibrium temperature is A.44℃ B.50℃C.56℃ 𝑚𝐻 (86−44) = 𝑚𝐶 (44−30), 42𝑚𝐻 = 14𝑚𝐶 ,
D.60℃ E.70℃. 𝑚𝐻 14 1
= = , 𝑚𝐻 :𝑚𝐶 = 1:3.
𝑚𝐶 42 3
𝑚𝑃 =𝑚𝑄 =𝑚, 𝜃𝑃 =20℃, 𝜃𝑄 =80℃, 𝜃=?,
27. A tap supplies water at 25℃ while another
𝑐𝑃 =1.0𝐽𝑘𝑔−1 ℃−1, 𝑐𝑄 =1.5𝐽𝑘𝑔−1 ℃−1 ,
supplies water at 75℃. If a man wishes to bath
Heat lost by 𝑄 = Heat gainhed by 𝑃 , with water at 40℃, the ratio of the mass of cold
𝑚𝑄 𝑐𝑄 (𝜃𝑄 − 𝜃) = 𝑚𝑃 𝑐𝑃 (𝜃 − 𝜃𝑃 ) , water to the mass of hot water required is A.1:3
𝑚 × 1.5(80− 𝜃) = 𝑚 ×1(𝜃 −20), B.3:1 C.7:3 D.15:18
120 – 1.5𝜃 = 𝜃 − 20, 120+20 = 𝜃 + 1.5𝜃 , 𝜃𝐻 =75℃, 𝜃𝐶 =25℃, 𝜃=40℃, 𝑚𝐶 :𝑚𝐻 =?,
140
140 = 2.5𝜃 , 𝜃 = = 56℃. Heat supplied by water at 75℃ = Heat gained by
2.5
23. A metal of mass 200g at a temperature of water at 25℃. 𝑚𝐻 (𝜃𝐻 − 𝜃) = 𝑚𝐶 (𝜃 − 𝜃𝐶 )
100℃ is placed in 100g of water at 25℃ in a 𝑚𝐻 (75−40) = 𝑚𝐶 (40−25), 35𝑚𝐻 = 15𝑚𝐶 ,
𝑚𝐶 35 7
container of negligible heat capacity. If the final = = , 𝑚𝐶 :𝑚𝐻 = 7:3.
𝑚𝐻 15 3
steady temperature is 30℃, calculate the specific 28. Hot water is added to three times its mass at
heat cdapacity of the ,metal (specific heat 10℃ and the resulting temperature is 20℃ What
capacity of water is 4200J𝑘𝑔−1 𝐾 −1 ) is the initial temperature of the hot water?
A.150J𝒌𝒈 𝑲 B.300J𝑘𝑔 𝐾 C.320J𝑘𝑔−1 𝐾 −1
−𝟏 −𝟏 −1 −1
A.100℃ B.80℃C.50℃ D.40℃
D.1960J𝑘𝑔−1 𝐾 −1 . Mass of water at 10℃=three times its mass of hot
𝑚𝑚 =200g=0.2kg, 𝜃𝑚 =100℃, 𝑚𝑊 =100g=0.1kg water, 𝑚𝐶 =3𝑚𝐻 , 𝜃𝐶 =10℃, 𝜃=20℃, 𝜃𝐻 =?,
𝜃𝑊 =25℃, 𝑐𝑊 =4200𝐽𝑘𝑔−1 𝐾 −1 , 𝜃=30℃,𝑐𝑚 =?, Heat supplied by hot water = Heat gained by
Heat lost by metal = Heat gained by water, water at 10℃, 𝑚𝐻 (𝜃𝐻 − 𝜃) = 𝑚𝐶 (𝜃 − 𝜃𝐶 )
𝑚𝑚 𝑐𝑚 (𝜃𝑚 − 𝜃) = 𝑚𝑊 𝑐𝑊 (𝜃 − 𝜃𝑊 ) , 𝑚𝐻 (𝜃𝐻 −20) = 3𝑚𝐻 (20−10), 𝜃𝐻 −20 = 30 ,
0.2× 𝑐𝑚 (100−30) = 0.1×4200(30−25) , 𝜃𝐻 = 30+20 = 50℃.
2100
14𝑐𝑚 = 2100 , 𝑐𝑚 = = 150J𝑘𝑔−1 𝐾 −1 . 29. Water at 10℃ is added to 3 times its mass at
14
24. When 100g of liquid 𝐿1 at 78℃ was mixed 30℃, calculate the temperature of the mixture
with 𝑋g of liquid 𝐿2 at 50℃, the final temperature A.5℃ B.10℃ C.15℃ D.25℃
was 66℃.Given that the specific heat capacity of Mass of at water 30℃=3 times its mass at 10℃,
𝐿2 is half that of 𝐿1 , find 𝑋 A.50g B.100g C.150g 𝑚𝐻 =3𝑚𝐶 , , 𝜃𝐶 =10℃, 𝜃𝐻 =30℃, 𝜃=?,
D.200g . 𝑚𝐻 (𝜃𝐻 − 𝜃) = 𝑚𝐶 (𝜃 − 𝜃𝐶 ) ,
1
𝑚1 =100g, 𝜃1 =78℃, 𝜃2 =50℃, 𝑐2 = 𝑐1 , 𝜃=66℃, 3𝑚𝐶 (30− 𝜃) = 𝑚𝐶 (𝜃 −10), 90 −3𝜃 = 𝜃 −10,
2 100
𝑚2 =𝑋, Heat lost by 𝐿1 = Heat gained by 𝐿2 , 90+10 = 𝜃 +3𝜃, 100 = 4𝜃 , 𝜃 = = 25℃.
4
𝑚1 𝑐1 (𝜃1 − 𝜃) = 𝑚2 𝑐2 (𝜃 − 𝜃2 ) , 30. What mass of water at 100℃ should be added
1
100𝑐1 (78−66) = 𝑋 × 𝑐1 (66−50), to 15g of water at 40℃ to make thetemperature
1200
2
of the mixture 50℃? (Negelect heat losses to the
1200= 8𝑋 , 𝑋 = = 150g. surroundings) A.3.0g B.4.0g C.4.5g D.6.0g E.7.0g.
8
25. 100g of water at 70℃ is added to 200g of 𝑚𝐻 =?, 𝑚𝐶 =15g , 𝜃𝐻 =100℃, 𝜃𝐻 =40℃, 𝜃=50℃,
water at 10℃, and the mixture is well stirred. 𝑚𝐻 (𝜃𝐻 − 𝜃) = 𝑚𝐶 (𝜃 − 𝜃𝐶 ) ,
Neglecting the heat absorbed by the container, 𝑚𝐻 (100−50) = 15(50−40), 50𝑚𝐻 = 150,
calculate the final temperature of the mixture 𝑚𝐻 =
150
= 3.0g.
A.25℃ B.30℃ C.40℃ D.50℃ E.55℃ 50
31. The water at the bottom of a waterfall is
𝑚𝐻 =100g, 𝜃𝐻 =70℃, 𝑚𝐶 =200g, 𝜃𝐶 =10℃, 𝜃=?,
slightly warmer than that at the top because part
𝑐𝐻 =𝑐𝐶 , as both liquids are water.
of the A.potential energy raises the water
Heat supplied by water at 70℃ = Heat gained by
temperature B.watr absorbs latet heat from the
water at 10℃.
atmosphere C.kinetic energy is changed into
𝑚𝐻 𝑐𝐻 (𝜃𝐻 − 𝜃) = 𝑚𝐶 𝑐𝐶 (𝜃 − 𝜃𝐶 ) ,
internal energy D.dropping mass of water
Hence, 𝑚𝐻 (𝜃𝐻 − 𝜃) = 𝑚𝐶 (𝜃 − 𝜃𝐶 ) ,
increases the heat capacity E.A and C.
100(70− 𝜃) = 200(𝜃 −10), 70− 𝜃 =2(𝜃 −10),
When water falls through a height, the intial
70− 𝜃 = 2𝜃 −20, 70+20 = 2𝜃 + 𝜃, 90 = 3𝜃,
90 potential energy at that height and the kinetic
𝜃 = = 30℃. energy of falling water is converted to internal or
3
26. A tap supplies water at 30℃ while another heat energy at the bottom, hence water
supplies water at 86℃. If a man wishes to bath becomes slightly warmed at the bottom.
with water at 44℃, calculate the ratio of the mass 32. A waterfall is 420m high. Calculate the
of hot water to that of cold water required A.1:3 difference in temperature of the water between
B.3:1 C.3:7 D.7:3 the top and bottom of the waterfall. Neglect heat
𝜃𝐻 =86℃, 𝜃𝐶 =30℃, 𝜃=44℃, 𝑚𝐻 :𝑚𝐶 =?, losses (g=10ms-2, specific heat capacity of

137
Demystified Series Physics Demystified by Dr Timothy
water=4.20×103J𝑘𝑔−1 𝐾 −1 ) A.0.1℃ B.1.0℃ Latent heat is absorbed or taken in when ice
C.4.2℃ D.42.0℃ E.100.0℃. (solid) changes to water (liquid) at constant
ℎ=420m, g=10ms-2, 𝑐=4.20×103𝐽𝑘𝑔−1 𝐾 −1 , ∆𝜃=?, temperature.
𝑃. 𝐸 = 𝐻 , 𝑚𝑔ℎ = 𝑚𝑐∆𝜃, 𝑔ℎ = 𝑐∆𝜃 , 39. Which of the following occurs bwhen a
10×420 = 4.20×103× ∆𝜃 , ∆𝜃 =
10×420
, substance solidifies? A.The latent heat of fusion
4.2×103
is absorbed B.There is a rise in temperature
∆𝜃 = 1.0℃.
C.The latent heat of fusion is given off D.There
33. A body of specific heat capacity 450𝐽𝑘𝑔−1 𝐾 −1
is a fall in temperature E.The specific heat is
falls to the ground from rest through a vertical
given off.
height of 20m. Assuming conservation of energy,
When a substance solidifies of freezes it gives off
calculate the change in temperature of the body
latent heat of fusion but when a substance melts
on striking the ground level (g=10ms-2) A.0℃
2 𝟒 9 9 is absorbs latent heat of fusion.
B. ℃ C. ℃ D. ℃ E. ℃ . 40. What happens when a certain quantity of
9 𝟗 4 2
𝑐=450𝐽𝑘𝑔−1 𝐾 −1 , ℎ=20m, g=10ms-2, ∆𝜃=?, pure ice is completely changed to water at 0℃?
𝑃. 𝐸 = 𝐻 , 𝑚𝑔ℎ = 𝑚𝑐∆𝜃, 𝑔ℎ = 𝑐∆𝜃 , A.Latent heat is absorbed, the mass remains
200 4
10×20 = 450× ∆𝜃 , ∆𝜃 = = ℃. constant and the volume decreases B.Latent
450 9
34. A good calorimeter should be of A.low heat is given out, the mass remains constant and
specific heat capacity and low heat conductivity the volume decreases C.Latent heat is given out,
B.low specifc heat capacity and high the mass increases and the volume remains
conductivity C.high specific heat capacity and constant D.Latent heat is absorbed, the mass
low conductivity D.high specific heat capacity decreases and the volume increases.
and high conductivity D,dull surface and low heat When ice melts to water at a constant
conductivity. temperature of 0℃, latent heat is absorbed, the
A good calorimeter should respon quickly to mass remains constant, the volume decreases
temperature changes i.e high thermal and its density increases.
conductivity, and should be able to raise the 41. The specific latent hear of suion of lead is the
temperature of a body to a very high value hence, amount of heat required to A.Melt lead at its
must be made of low specific heat capacity. melting point B.Heat a unit mass of lead through
35. Which of the following processes does not 1℃ C.Change the state of a unit mass of lead at
reduce heat loss from a liquid in a calorimeter? its melting point D.Change the state of a unit
A.Lagging the calorimeter B.Using an insulator lid mass of lead at its boiling point.
C.Shielding the calorimeter from draughts 42. Which of the following statements is true of
D.Constantly stirring the liquid E.Polishing the the latent heat of fusion of a body? A.It is the heat
inner and outer surfaces of the calorimeter. required to change the temperature of the body
36. During the transformation of matter from the B.It is the energy required to breakdown the
solid to the liquid state,the heat supplied does not forces which hold the atoms in a regular
produce a temperature increase because A.the pattern without a change in temperature C.It
heat energy is quickly conducted away B.the heat is the energy required to bring the body to its
capacity has become very large as the substance melting point D.It is the energy required to
melts C.the heat gained is equal to the heat lost change the temperature of the body after melting
by the substance D.all the heat is used to break E.It is the energy required to break down the
the bonds holding the molecules of the solid forces which hold the atoms of a unit mass of the
together body in a regular pattern at a definite
During change of state, the heat supplied is used temperature.
to break the bond holding molecules together Option E gives the definition of specific latent
without any change in temperature of the body. heat of fusion.
37. Thelatent heat of fusion of a substance is the 43. An ice cube floats in a glass of water filled to
heat required to A.raise the temperature of unit the brim. What happens when the ice cube melts?
mass of the substance by 1℃ B.maintain unit A.The water level remain the same B.There is
mass of the substance a constant temperature a drop in the level of water in the glass due to
C.change unit mass of substance from solid to condensation outside C.The water level in the
liquid at constant temperature D.raise the glass overflows D.The level of water drops
temperature of unit mass by 100℃. because melted ice occupies less volume E.None
38. In the process of changing a piece of ice to of the above is correct.
water at 0℃ A.some quantity of heat is given out The ice cube displaces volume of water equal to
B.there is usually a temperatuyre change C.some its own volume before floating. Hence, after
quantity of heat is taken in D.the quantity of heat melting, it occupies the volume of water
given out is definite E.the quantity of heat taken originally displaced.
in varies with the temperature of the ice. 44. If a container is filled with ice to the brim,

138
Demystified Series Physics Demystified by Dr Timothy
what happens to the level of water when the ice high enough temperature, the vibrations
melts completely? A.The water in the glass become so violent that the molecules are
overflows B.The level of waterdrops C.The forced to leave the surface C.The temperature
level of water remains unchanged D.The level of of a solid is constant during melting because the
water goes up heat supplied is used up in keeping the molecules
The water level drops because ice occupies more in their mean positions D.The temperature of
volume than water (anomalous expansion) and liquid falls during evaporation because the more
due to the fact that the air spaces between them energetic molecules leave the liquid while the
is completely filled with water when it melts. less energetic one left behind E.The average
45. Changes of state is accompanied by a change kinetic energy of a liquid increases with
of A.temperature B.volume C.heat content temperature.
D.temperature and volume E.volume and heat Heating a solid to increase its temperature cuases
content. its molecules to vibrate at a high amplitude.
Change of state is accompanied by change in When a liquid is heated to high enough
density,volume and heat content without any temperature, the molecules changes to the
change in the mass and temperature. gaseous state. Evaporatiion which is the escape
46. The specific latent heat of vaporization of a of liquid molecules from a surface does not
substance is the heat needed to A.change a unit depend on heating, it takes place at all
mass of the substance at 100℃ to vapour without temperature and bring about a cooling effect i.e
any change at temperature B.raise the fall in temperature.
temperature of a unit mass of the substance by 50. If a weight is supported by a loop of copper
1℃ C.change the subtstance from liquid to wire which passes over a block of ice, the wire
vapour at its boiling point without any change in gradually passes through the ice but the ice still
temperature D.change the temperature of the remains in one piece. This phenomenon is known
substance from 0℃ to 100℃E.change a unit as A.freezing B.conduction C.melting
mass of the substance at its boiling point from D.regelation E.convention.
liquid to vapour without aby change in The experiment shows regelation which is the
temperature effect of pressure on the melting point of a solid.
Option C the definition of latent heat of 51. The following are some units I.𝑁s II.𝑁m
vaporization. III.Nm-2 IV.J𝐾 −1 V.J𝑘𝑔−1 . What are the units of
47. A steam trap is a component of a steam latent heat? A.I B.II C.III D.IV E.V.
generator that can be used to determine the 52. How much heat is required to convert 20g of
specific latent heat of vapourization of water. In ice at 0℃ to water at the same temperature?
the steady state, steam trap A.stores the steam [Specific latent heat of fusion of ice=336𝐽𝑔−1 ]
for future use B.prevents the steam from A.1.35×103J B.5.38×103J C.6.72×103J
escaping C.collects the water that condenses D.7.06×103J E.8.06×103J .
D.warms the steam up to 100℃ E.determine the 𝑚=20g, 𝐿=336𝐽𝑔−1 , 𝐻=?, 𝐻 = 𝑚𝐿 ,
quality of steam used 𝐻 = 20×336 = 6720J = 6.72×103J .
Steam or water trap dries steam so that only dry 53. The latent of fusion of ice is 80cal𝑔−1 . How
steam rather than condensed(wet) steam or much heat is required to change 10g of ice at 0℃
liquid water, passes through the delivery tube into water at the same temperature A.8cal
into the calorimeter. B.10cal C.80cal D.400cal E.800cal.
48. Which of the following observations is not 𝐿=80cal𝑔−1 , 𝑚=10g, 𝐻=?, 𝐻 = 𝑚𝐿,
correct? A.When water freezes its volume 𝐻 = 10×80 = 800cal.
increases B.When ice melts the mass of the 54. Ice of mass 50g at 0℃ was completely melted
water produced is less C.When ice melts, its by 70% of the total heat energy supplied.
volume decreases D.An increase in pressure Calculate the total energy supplied. [Specific
lowers the melting point of ice E.An increase in latent heat of fusion of ice=336J𝑔−1 ] A.2400J
pressure increases the boiling point B.6800J C.7251J D.24000J.
When water freezes into ice, its density 𝑚=50g, 70% of 𝐻 supplied to melt the ice,
decreases while its volume increases, the 70%𝐻 = 𝑚𝐿,
70
𝐻 = 𝑚𝐿 , 𝐻 =
100𝑚𝐿
,
100 70
opposite hold when ice melts. When ice melts the 100×50×336
mass of the water produced is same as that of the 𝐻= = 24000J.
70
ice. An increase in pressure increases the boiling 54. The melting point of a solid is given as 80℃.
point but decreases the melting point. If 105𝐽 of heat energy is required at this
49. Which of these statements is not correct? temperature to melt 10g of the solid, what is the
A.When the temperature of a solid is increased, specific latent heat of fusion of the solid?
each molecule in the solid increases its amplitude A.1.00×103J𝑘𝑔−1 B.1.25×105J𝑘𝑔−1
of oscillation B.When a liquid is heated to a C.1.00×107J𝒌𝒈−𝟏 D.8.00×108J𝑘𝑔−1 .

139
Demystified Series Physics Demystified by Dr Timothy
𝐻=105𝐽, 𝑚=10g=0.01kg, 𝐿=?, 𝐻 = 𝑚𝐿 , B.60J𝑔−1 C.120J𝒈−𝟏 D.1500J E.1500J𝑔−1 .
𝐻 105 𝑅=5Ω, 𝐼=2𝐴, 𝑡=30s, 𝑚=5g, 𝐿=? , 𝐼 2 𝑅𝑡 = 𝑚𝐿 ,
𝐿= = =1.00×107𝐽𝑘𝑔−1 .
𝑚 0.01 𝐼 2 𝑅𝑡 22 ×5×30
55. A 90W immersion heater is used to supply 𝐿= = = 120J𝑔−1 .
𝑚 5
energy for 5 minutes. The energt supplied is used 62. A quantity of ice at −10℃ is heated until the
to completely melt 180g of a solid at its melting temperature of the heating vessel is 90℃. Which
point. Neglecting energy losses to the of the following constants is not required to
surrounding, calculate the specific latent heat of determine the quantity of heat supplied to the
fusion of the solid A.0.5J𝑔−1 B.2.5J𝑔−1 C.15.0𝐽𝑔−1 vessel? A.specific heat capacity of ice B.specific
D.150.0J𝒈−𝟏 E.450.0J𝑔−1 . heat capacity of water C.specific latent heat of
𝑃=90𝑊, 𝑡=5minutes=300s, 𝑚=180g, 𝐿=?, fusion D.specific latent heat of vaporization.
𝑃𝑡 = 𝑚𝐿 , 90×300 = 180× 𝐿 , 𝐿 =
90×300
, Heating of ice from −10℃ to 90℃ involves –
180
I.Conversion of ice at −10℃ to 0℃(m.p of ice) i.e
𝐿 = 150.0J𝑔−1 .
𝐻1 = 𝑚𝑐𝑖 ∆𝜃𝑖 (specific heat capacity of ice).
56. What amount of current would pass through
II.Conversion of ice at 0℃ to water at 0℃ i.e
a 10Ω coil if it takes 21s for the coil to just melt a
𝐻2 = 𝑚𝐿𝑖 (latent heat of fusion of ice),
lump of ice of mass 10g at 0℃ if there are no heat
III.Conversion of water at 0℃ to water at 90℃ i.e
losses? [latent heat of fusion of ice=336𝐽𝑔−1 ]
𝐻3 = 𝑚𝑐𝑊 ∆𝜃𝑊 (specific heat capacity of water).
A.2𝐴 B.4𝑨 C.8𝐴 D.16𝐴.
Hence, latent heat of vaporizatiojn is inot
𝑅=10Ω, 𝑡=21s, 𝑚=10g, 𝐿=336𝐽𝑔−1 , 𝐼=?,
required, as the temperature of liquid water is
𝐼 2 𝑅𝑡 = 𝑚𝐿 , 𝐼 2 ×10×21 = 10×336,
10×336 not up to its boiling point(100℃).
𝐼2 = = 16 , 𝐼 = √16 = 4𝐴. 63. Ice of mass 10g at −5℃ was completely
10×21
57. An electric heater is used to melt a block of converted to water at 0℃. Calculate the quantity
ice of mass 1.5kg. If the heater is powered by a of heat used (Specific heat capacity of
12𝑉 battery and a current of 20𝐴 flows through ice=2.1J𝑔−1 𝐾 −1 , specific latent heat of fusion of
the coil, calculate the time taken to melt the block ice=336J𝑔−1 ) A.105J B.3255J C.3465J D.16821J.
of ice at 0℃ [Specific latent heat of fusion of 𝑚=10g, 𝑐𝑖 =2.1J𝑔−1 𝐾 −1 , 𝐿𝑖 =336J𝑔−1 , 𝐻=?,
ice=336×103𝐽𝑘𝑔−1 ] A.76.0min B.35.0min Total heat energy 𝐻 required to convert ice at
C.21.0min D.2.9min. −5℃ to water at 0℃ includes –
𝑚=1.5kg, 𝑉=12𝑉, 𝐼=20𝐴, 𝐿=336×103𝐽𝑘𝑔−1 , 𝑡=?, Conversion of ice at −5℃ to ice at ice at 0℃ : 𝐻1 =
𝐼𝑉𝑡 = 𝑚𝐿 , 20×12× 𝑡 = 1.5×336×103, 𝑚𝑐𝑖 ∆𝜃𝑖 (specific heat capacity),
1.5×336×103 Conversion of ice at 0℃ to water at 0℃ :
𝑡= = 2100s = 35min.
20×12
𝐻2 = 𝑚𝐿𝑖 (specific latent heat of fusion).
58. Calculate the quantity of heat released when
𝐻 = 𝐻1 + 𝐻2 = 𝑚𝑐𝑖 ∆𝜃𝑖 + 𝑚𝐿𝑖 = 𝑚(𝑐𝑖 ∆𝜃𝑖 + 𝐿𝑖 ) ,
100g of steam 100℃ condenses to water (Take
𝐻 = 10(2.1(0−(−5)+336) = 10(2.1(5) + 336) ,
the specific latent heat of vapourization of water
𝐻 = 10(10.5+336)= 3465J.
as 2.3×106 𝐽𝑘𝑔−1 ) A.2.3×101J B.2.3×102J
64. How much heat energy is needed to change
C.2.3×10 J D.2.3×10 J E.3.3×10 J.
4 5 7
3g of ice at 0℃ to steam? (Specific latent heat of
𝑚=100g=0.1kg, 𝐿=2.3×106J𝑘𝑔−1 , 𝐻=?,
fusion=336J𝑔−1 , specific latent heat of
𝐻 = 𝑚𝐿 , 𝐻 = 0.1×2.3×106 = 2.3×105J.
vaporization of water=2260J𝑔−1 , specific heat
59. A quantity of steam at 100℃ condenses to
capacity of water=4.2J𝑔−1 𝐾 −1 ) A.2268J B.3016J
water at the same temperature by releasing
C.7788J D.7800.6J E.9048J.
6.9×104J of energy. Calculate the mass of the
𝑚=3g , 𝐿𝑖 =336J𝑔−1 , 𝐿𝑠 =2260J𝑔−1 ,
condensed steam (Specific latent heat of
𝑐𝑊 =4.2J𝑔−1 𝐾 −1 , 𝐻=?,
vaporization of water=2.3×106J𝑘𝑔−1 ) A.3.0×10-
1kg B.3.0×10-2kg C.3.0×10-3kg D.3.0×10-4kg. Total heat energy 𝐻 required to convert ice at 0℃
to steam at 100℃ includes –
𝐻=6.9×104𝐽, 𝐿=2.3×106J𝑘𝑔−1 , 𝑚=?,
𝐻 6.9×104
Conversion of ice at 0℃ to water at 0℃ : 𝐻1 =
𝐻 = 𝑚𝐿 , 𝑚 = = = 3.0×10-2kg. 𝑚𝐿𝑖 (specific latent heat of fusion),
𝐿 2.3×106
60. A heater marked 50W will evaporate 0.005kg Conversion of water at 0℃ to water at 100℃ :
of boiling water in 50 seconds.What is the 𝐻2 = 𝑚𝑐𝑊 ∆𝜃𝑊 (specific heat capacity of water),
specific latent heat of vaporization of water in Conversion of water at 100℃ to steam at 100℃ :
𝐽/kg A.1.0×106B.2.5×106 C.2.5×105D.5.0×105 𝐻3 = 𝑚𝐿𝑠 (specific latent heat of vapourization).
E.5.0×106. 𝐻 = 𝐻1 + 𝐻2 + 𝐻3 = 𝑚𝐿𝑖 + 𝑚𝑐𝑊 ∆𝜃𝑊 + 𝑚𝐿𝑠 ,
𝑃=50𝑊, 𝑡=50s, 𝑚=0.005kg, 𝐿=?, 𝑃𝑡 = 𝑚𝐿 , 𝐻 = 𝑚(𝐿𝑖 + 𝑐𝑊 ∆𝜃𝑊 + 𝐿𝑠 ),
50×50 =0.005× 𝐿, 𝐿 =
50×50
=5.0×105 J/kg 𝐻 = 3(336+4.2(100−0)+2260),
0.005
𝐻 = 3(336+420+2260) = 9048𝐽.
61. All the heat generated in a 5Ω resistor by a
65. Calculate the amount of heat required to
2𝐴 flowing for 30 seconds is used to evaporate 5g
convert 2kg of ice at −2℃ to water at 0℃
of a liquid at its boiling point. What is the correct
(specific heat capacity of ice=2090J𝑘𝑔−1 ℃−1 ,
value of specifc latent heat of the liquid? A.120J

140
Demystified Series Physics Demystified by Dr Timothy
specific latent heat of fusion of ice= 333kJ𝑘𝑔−1 ) heat of ice=336J𝑔−1 , specific heat capacity of
A.666J B.8360J C.666000J D.674360J. water=4.2J𝑔−1 𝐾 −1 ) A.840J B.3360J C.3420J
𝑚=2kg, 𝐿𝑖 =333𝐾𝐽𝑘𝑔−1 =333000𝐽𝑘𝑔−1, D.4200J E.7560J.
𝑐𝑖 =2090𝐽𝑘𝑔−1 ℃−1 , 𝐻=?, 𝑚=10g, 𝐿𝑖 =4.2𝐽𝑔−1 𝐾 −1 , 𝑐𝑖 =336𝐽𝑔−1 , 𝐻=?,
Total heat energy 𝐻 required to convert ice at Total heat energy 𝐻 lost by boiling water –
−2℃ to water at 0℃ includes – Conversion of boiling water at 100℃ to water at
Conversion of ice at −2℃ to ice at ice at 0℃ : 𝐻1 = 0℃ : 𝐻1 = 𝑚𝑐𝑊 ∆𝜃𝑊 ,
𝑚𝑐𝑖 ∆𝜃𝑖 (specific heat capacity), Conversion of water at 0℃ to ice at 0℃ (freezing)
Conversion of ice at 0℃ to water at 0℃ : : 𝐻2 = 𝑚𝐿𝑖 . 𝐻 = 𝐻1 + 𝐻2 ,
𝐻2 = 𝑚𝐿𝑖 (specific latent heat of fusion). 𝐻 = 𝑚𝑐𝑊 ∆𝜃𝑊 + 𝑚𝐿𝑖 = 𝑚(𝑐𝑊 ∆𝜃𝑊 + 𝐿𝑖 ) ,
𝐻=𝐻1 + 𝐻2 = 𝑚𝑐𝑖 ∆𝜃𝑖 + 𝑚𝐿𝑖 , 𝐻 = 𝑚(𝑐𝑖 ∆𝜃𝑖 + 𝐿𝑖 ) 𝐻 =10(4.2(100−0)+336) = 10(4.2(100)+336)
𝐻 = 2(2090(0−(−2)+333000) , 𝐻 = 10(420+336) = 7560𝐽.
𝐻 = 2(2090(2) + 333000) = 2(4180+333000) 69. What is the difference in the amount of
𝐻 =674360J. heat given out by 4kg of steam and 4kg of water
66. How much heat energy is needed to convert when both are cooled from 100℃ to 80℃?
20g of ice at −15℃ to steam at 100℃? (specific (specific latent heat of steam= 2260000𝐽𝑘𝑔−1 ,
heat of ice=2.2𝐽𝑔−1 𝐾 −1 , specific heat of specific heat capacity of water=4200𝐽𝑘𝑔−1 𝐾 −1 )
water=4.18𝐽𝑔−1 𝐾 −1 , specific latent heat of A.4200J B.2260000J C.9040000J D.9380000J.
ice=0.334𝐾𝐽𝑔−1 , specific latent heat of 𝐿𝑠 =2260000𝐽𝑘𝑔−1 , 𝑐𝑊 =4200𝐽𝑘𝑔−1 𝐾 −1 , 𝑚=4kg ,
steam=2.260𝐾𝐽𝑔−1 ) A.77.28kJ B.60.90kJ Cooling steam from 100℃ to 80℃ involves –
C.60.32kJ D.52.55kJ E.51.92kJ. Conversion of steam at 100℃ to water at 100℃
𝑚=20g, 𝑐𝑖 =2.2𝐽𝑔−1 𝐾 −1 , 𝑐𝑊 =4.18𝐽𝑔−1 𝐾 −1 , (condensation) : 𝐻 = 𝑚𝐿𝑠 , Conversion of water
𝐿𝑖 =0.334𝐾𝐽𝑔−1 =334𝐽𝑔−1 , 𝐿𝑠 =2.260𝐾𝐽𝑔−1 = at 100℃ to water at 80℃ : 𝐻 = 𝑚𝑐𝑊 ∆𝜃𝑊 .
2260 𝐽𝑔−1 , Total heat given out by cooling steam =
Total heat energy 𝐻 required to convert ice at 𝑚𝐿𝑠 + 𝑚𝑐𝑊 ∆𝜃𝑊 ,
−15℃ to steam at 100℃ includes – While cooling water from 100℃ to 80℃, heat
Conversion of ice at −15℃ to ice at 0℃ : 𝐻1 = given out = 𝑚𝑐𝑊 ∆𝜃𝑊 . Hence difference in the
𝑚𝑐𝑖 ∆𝜃𝑖 (specific heat capacity of ice), amount of heat energy given when steam and
Conversion of ice at 0℃ to water at 0℃ : 𝐻2 = water are cooled from 100℃ to 80℃ =
𝑚𝐿𝑖 (specific latent heat of fusion), (𝑚𝐿𝑠 + 𝑚𝑐𝑊 ∆𝜃𝑊 ) − 𝑚𝑐𝑊 ∆𝜃𝑊 = 𝑚𝐿𝑠 ,
Conversion of water at 0℃ to water at 100℃ : Difference in heat energy = 4×2260000 ,
𝐻3 = 𝑚𝑐𝑊 ∆𝜃𝑊 (specific heat capacity of water), Difference in heat energy = 9040000J.
Conversion of water at 100℃ to steam at 100℃ : 70. 1kg of copper is transferred quickly from
𝐻4 = 𝑚𝐿𝑠 (specific latent heat of vapourization). boiling water to a block of ice. Calculate the mass
𝐻 = 𝐻1 + 𝐻2 + 𝐻3 + 𝐻4 , of ice melted, neglecting heat loss (specific heat
𝐻 = 𝑚𝑐𝑖 ∆𝜃𝑖 + 𝑚𝐿𝑖 + 𝑚𝑐𝑊 ∆𝜃𝑊 + 𝑚𝐿𝑠 , capacity of copper = 400J𝑘𝑔−1 𝐾 −1 , latent heat of
𝐻 = 𝑚(𝑐𝑖 ∆𝜃𝑖 + 𝐿𝑖 + 𝑐𝑊 ∆𝜃𝑊 + 𝐿𝑠 ) , fusion of ice= 333×103J𝑘𝑔−1 ] A.60g B.67g
𝐻=20(2.2(0−(−15)+334+4.18(100−0)+2260 C.120g D.133g.
𝐻 = 20(2.2(15)+334+4.18(100)+2260, 𝑐𝑐 =400J𝑘𝑔−1 𝐾 −1 , 𝐿𝑖 =333×103J𝑘𝑔−1 , 𝑚𝑐 =1kg,
𝐻= 20(33+334+418+2260) = 60900J= 60.90KJ. 𝑚𝑖 =?, ∆𝜃𝑐 =100−0=100℃,
67. A solid of mass 0.5kg initially at 50℃ melt at Heat lost by copper at 100℃ = Heat required to
60℃. Calculate the quantity of the heat required melt ice at 0℃. 𝑚𝑐 𝑐𝑐 ∆𝜃𝑐 = 𝑚𝑖 𝐿𝑖 ,
to melt it completely (Specific heat capacity of the 1×400×100 = 𝑚𝑖 ×333×103 ,
solid=2.0𝐾𝐽𝑘𝑔−1 𝐾 −1 , Specific latent heat of 𝑚𝑖 =
400×100
= 0.120kg = 120g.
333×103
fusion of the solid=1.0𝐾𝐽𝑘𝑔−1 )
71. When comparing the specific latent heats of
A.1500𝐽 B.10000𝐽C.10500𝑱 D.15000𝐽
vapourization 𝐿𝑣 of a substance to its specific
𝑚=0.5kg, 𝑐=2.0𝐾𝐽𝑘𝑔−1 𝐾 −1 =2000 𝐽𝑘𝑔−1 𝐾 −1 ,
latent heat of fusion 𝐿𝑓 , which of the following
𝐿=1.0𝐾𝐽𝑘𝑔 =1000𝐽𝑘𝑔−1 , 𝐻=?,
−1
statements is true A.𝐿𝑣 < 𝐿𝑓 B.𝐿𝑣 = 𝐿𝑓 C.𝑳𝒗 > 𝑳𝒇
Total heat energy 𝐻 required to melt the solid at
60℃ – D.𝐿𝑣 and 𝐿𝑓 can’t be measured.
Conversion of the solid at 50℃ to 60℃ : Specific latent heat of vaporization 𝐿𝑣 of a
𝐻1 = 𝑚𝑐∆𝜃 (specific heat capacity of solid), substance is always greater than its specific
Conversion of the solid at 60℃ to the liquid at latent heat of fusion 𝐿𝑓 .
60℃ (melting) : 𝐻2 = 𝑚𝐿 (specific latent heat of 72. A person suffers a more severe burn from
fusion). 𝐻 = 𝐻1 + 𝐻2 = 𝑚𝑐∆𝜃 + 𝑚𝐿 , steam than from boiling water because A.steam
𝐻 = 𝑚(𝑐∆𝜃 + 𝐿) = 0.5(2000(60−50)+1000), is at a higher temperature than boiling water
𝐻 =0.5(2000(10)+1000) =10500𝐽. B.steam spreads more easily over the skin than
68. Calculate the heat energy lost when 10g of boiling water C.steam penetrates more deeply
boiling water changes to ice at 0℃ (specific latent into the skin than boiling water D.steam posses

141
Demystified Series Physics Demystified by Dr Timothy
greater heat content per unit mass than Temp(℃)
boiling water. 100
The internal energy or heat content per unit mass 90
of steam is greater than that of boiling waterb as 80
steam posseses both thermal energy of boiling 70
60
water and latent heat of vaporization. Hence, it 5 10
causes more severe burn than boiling water. A.Cooling temperature B.Boiling point C.Melting
Steam is also at a higher temperature than point D.Vaporization point .
boiling water. The temperature 80℃ indicates the temperature
73. Steam from from boiling water causes more at which the molten naphthalene
damage on the skin than deos boiling water solidifies(freezes). Freezing point of a substance
because A.steam has latent heat of fusion B.the is equal to its melting point.
steam is at a higher temperature than water 78. The diagram below is the heating curve for a
C.steam brings heat more easily by convention pure solid , 𝑸𝑹 is the
D.water has a high specific heat . Temperature
Steam causes more damage on the skin than 𝑺
boiling water because it has latent heat of 𝑸
vaporization and is at a higher temperature than 𝑷 𝑹
boiling water. Temperature of steam can exceed Time
100℃ but but boiling water can not go above A.boiling region B.evaporation region C.melting
100℃. region D.sublimation region.
74. The melting point of a substance is equivalent Temperature during melting is constant and is
to its A.liquidification temperature represented by a horizontal line in the heating
B.solidification temperature C.solidification curve.
pressure D.vapour pressure. 79. A mass of ice at −10℃ is heated at a constant
Melting point of a substance is the same as its rate unitl it all melts and then boils away. Which
freezing point or solidification temperature. of the following temp[erature-time graphs
75. A glass of soft drink can be cooled more correctly represents the process?
effectively by adding melting ice than by adding A. Temp(℃) B. Temp(℃)
the same mass of water at 0℃ because A.ice
makes better thermal contacts with water 100 100
B.melting ice is at lower temperature than the
water C.water has higher specific heat capacity
than ice D.ice absorbs latent heat as it melts. 0 0
The melting ice asbsorbs the latent heat of fusion Time(s) Time(s)
needed to melt it from the soft drink, hence −10 −10
decreasing the temperature of the drink i.e C. Temp(℃) D. Temp(℃)
cooling it. The water and the melting ice are at the
same temperature i.e 0℃. 100 100
76. The statements below were made by some
students describing what happened during the
determination of the melting points of solids 0 0
1.The temperature of the solid was constanbt Time(s) Time(s).
until melting 2.The temperature pf the solid rose −10 −10
until melting started 3.During melting, the The temperature of the ice first increases from
temperature was rising 4.The temperature was −10℃ to 0℃. The ice at 0℃ then melts to water
constant 5.The temperature continued to rise at 0℃. The temperature of water increases from
after all the solid had melted 6.The temperature 0℃ to 100℃, wher it finally boils off as steam at
stopped rising after all the solid had melted. 100℃.
Which of the following gives the correct 80. Evaporation can be explained on the basis of
statements in the right order? A.2,4 and 5 B.2,3 the kinetic theory by the following statements
and 6 C.1,3 and 5 D.1,3 and 6 E.1,4 and 5. I.The fast-moving molecules of the liquid escape
Temperature of a solid rises before and after II.Some molecules have greater energy than the
melting but remains constant during melting. average III.The temperature of the remaining
77. In an experiment in which molten liquid falls IV.The average kinetic energy of the
naphthalene is allowed to cool. The cooling curve remaining molecules falls. What is the correct
in the figure below was obtained. The order in which the statements should be placed?
temperature 80℃ is known as the A.I,II,III,IV B.II,III,IV,I C.II,I,IV,III D,I,II,IV,III
E.I,III,IV,II.

142
Demystified Series Physics Demystified by Dr Timothy
Evaporation is the escape of vapour molecules the density of the liquid. It also depends on the
from the surface of a liquid below the boiling nature of the liquid.
point of the liquid. According to the kinetic 86. Which of the bfollowing does not affect the
theory, some molecules having kinetic energy rate of evaporation of a liquid? A.the surface area
greater than average kinetic energy move to the of the liquid B.force-convention of the air around
liquid surface and escape from the surface as the liquid C.the temperature of the surroundings
vapour. The average kinetic energy of the D.density of the liquid E.the osmotic pressure
remaining liquid molecules then drops hence of the liquid.
decrease in the temperature. Evaporation depends on the surface area of the
81. Evaporation of a liquid produces cooling of liquid, wind or force-convention on the liquid,
the liquid because the A.density of the liquid density of the liquid and temperature of the
decreases B.volume of the liquid decreases surroundings. It does not depend on the osmotic
C.vapour pressure above the liquid rises pressure of the liquid.
D.attraction between the vapour molecules and 87. Blowing air over a liquid aids evaporation by
the liquid molecules is great E.average kinetic A.decreasing its vapour pressure B,decreasing its
energy of the molecules of the liquid is density C.increasing its surface area
reduced. D.increasing its temperature
Evaporation is the escape of molecules with Blowing air or wind over a liquid surface sweeps
energy greater than the average kinetic energy away vapour from the liquids surface, hence
from the liquid surface, this results in a decrease increasing the exposed surface area for more
in the average kinetic energy of the remaining molecules to evaporate.
molecules in the liquid and gives a cooling effect 88. I.Increase in melting point of the liquid
due to reduction in temperature. II.Increase the boiling point of the liquid
82. Which of the following about evaporation is III.Decrease the melting of the liquid IV.Decrease
not correct? A.It is the escape of high energy the boiling point of the liquid, Which of the
molecules from the liquid surface B.It lowers the statements above about the effects of increase in
average internal energy C.It causes the pressure in a liquid are correct? A.II and III B.I
temperature of the liquid to fall D.It occurs only and II C.III and IV D.I and III.
at the boiling point of the liquid E.It occurs less Increase in pressure increases the boiling point
where the attractive forces of the molecules are of a liquid and decreases the melting or freezing
very great. point of a solid.
Evaporation occurs at any temperature below 89. Which of the following conditions will make
the boiling point of a liquid. It occurs less where water boil at a temperature of 100℃ when the
the attractive forces of the molecules are very atmospheric pressure is 750mmHg?
great, as the liquid molecules are strongly A.increasing the external pressure B.reducing
attracted to one another. the external pressure C.heat more readily at the
83. Which of the following is not a factor that can same pressure D.reduce the pressure by a
increase the rate of evaporation of water in a quarter.
lake? A.Increase in the pressure of the Water boils at 100℃ at normal atmospheric
atmosphere B.Rise in temperature C.Increase in pressure of 760mmHg. When the pressure is
the average speed of the molecules of water lowered to 750mmHg, water tends to boil at a
D.Increase in the kinetic energy of the energy temperature lower 100℃. Hence, the external
The rate of evaporation increases with increase pressure has to be increased for water to boil at
in temperature. Temperature increases as the 100℃.
average speed and kinetic energy increases. Rate 90. The boiling point of water exceeds 100℃
of evaporation decreases as the pressure of the when A.It is slowly heated B.It is rapidly heated
atmosphere increases, 𝐸 ∝
𝑊𝐴𝑇
. C.Its vapour pressure is greater than
𝐻𝑃𝜌
atmospheric pressure D.Its vapour pressure is
84. The process whereby a liquid turns
less than the atmospheric pressure.
spontaneously into vapour is called A.boiling
When water is rapidly heated it means the
B.evaporation C.regelation D.sublimation.
pressure is increased hence, the boiling point will
85. What does not affect the rate of evaporation
exceeds 100℃.
of a liquid? A.exposed surface area of the liquid
91. A quantity of water was found to boil at
B.humidity of the liquid C.nature of the liquid
103℃ when heated in a beaker. This was because
D.volume of the liquid E.temperature of the
the A.specifc heat capacity of the beaker was high
liquid.
B.water contained a quanity of dissolved salt
Rate of evaporation depends on wind, exposed
C.beaker contained distilled water D.water was
surface area,temperature,humidity, pressure and
being heate on top of a mountain E.specific heat

143
Demystified Series Physics Demystified by Dr Timothy
capacity of the mercury in the thermometer was temperature. Evaporation occurs ar the surface
high. only while boiling occurs throughout the whole
The presence of impurity e.g salt, increases the volume of the liquid. Evaporation is affecred by
boiling points of liquids. surface area while boiling is independent of
92. Food cook quicker in salt water than in pure surface area.
water because of the effect of A.Food nutrients 96. If a strong flow of air is passed through
on the thermal energy B.Salt on the thermal alcohol contained in a small beaker, water
conductivity of water C.Atmospheric pressure on droplets soon form and freezes on the outside of
the boiling point D.Dissolved substances on the the beaker. This is because alcohol has a A.high
boiling point. latent heat of vaporization B.low specific heat
Dissolved substance or impurities increases the capacity C.high specific heat capacity D.low
boiling point of liquids, latent heat of vaporization E.very high relative
93. What will happen to the boiling point of pure humidity.
water when it is heated in a place 30m below the Passing air or wind over alcohol increases its rate
sea level? A.It will be less than 100℃ B.It will still of evaporation. The latent heat of vaporization
be 100℃ C.It will still be fluctuating D.It will be needed to evaporate it is drawn from the air
more than 100℃. around the beaker hence the formation of water
Presssure increases with depth below sea level droplets around the beaker.
and decreases above sea level. Hence, at a place 97. Why does a body feel cold when splashed
30m below sea level pressure increases and the with petrol? A.is naturally cold B.removes heat
boiling point increases. from the body C.has a very low latent heat of
93. A red hot heated metal is quickly transferred vaporization D.obtains its latent heat of
into a beaker containing boiling water. What vaporization from the body E.has a low
effect will the hot metal have on the boiling freezing point
water? A.The rate of vaporization of the The splashed petrol draws the latent heat of
boiling water increases B.The boiling point vaporization from the body, hence decreasing the
rises sharply C.The boiling rises and then fall temperature of the body surface making feel cool.
D.The boiling point rises slightly. 98. Which of the following substances is the most
The water is already boiling at a fixed volatile at room temperature? A.water B.diesel
temperature(100℃) and the water is undergoing C.petrol D.kerosene
vaporization. Thus, transferring the red heated A volatile liquid is a liquid which evaporates
metal into the boiling water will increase the rate spontaneously without heat. Volatile liquids have
of vaporization of the boiling water, as the heat low latent heat of vaporization and low boiling
supplied to the boiling water becomes more point. Petrol is the most volatile than the other
intense. liquids given in the option.
94. Which of the following is common to 99. A vapour is said to be saturated when A.more
evaporation and boiling? They A.take place at molecules return to the liquid than leave it B.the
any temperature B.are surface phenomena number molecules leaving the liquid for the
C.involve change of state D.take place at a vapour are equal in number to those leaving
definite temperature. the vapour for the liquid C.higher number of
Both boiling and evaporation involves the change moleculesreturn to the vapour than the
of state from liquid phase into the gaseous phase. molecules leaving it for the liquid D.the vapour
Evaporation takes place at any temperature pressure equal the atmospheric pressure.
while boiling takes place at a definite Vapour is said to be saturated when the number
temperature. Evaporation is a surface of molecules leaving the liquid equals the
phenomena i.e occurs at the liquid surface while number of molecules returning to the liquid.
boiling occurs throughout the whole volume of 100. Vapour is said to be saturated on top of an
liquid. enclosed liquid if A.the rate of condensation is
95. Which of the following statements gives the equal to that of vaporization B.there is neither
true differences between evaporation and condensation nor vaporization of the liquid C.the
boiling? I.evaporation occurs at all temperatures rate of condensation is less than that of
while boiling occurs at fixed temperature for a vaporization D.the rate of condensation is
given pressure II.evaporation is a surface greater than that of vaporization.
phenomenon while boiling is an interior Vapour is said to be saturated when the rate of
phenomenon III.evaporation is affected by vaporization of liquid is equal to the rate of
surface area while boiling is not A.I and II C.I and condensation. Vapour is said to be unsaturated
III C.II and III D.I,II and III. when the rate of vaporization is greater than the
Evaporation occurs at all temperature whule rate of condensation or the rate of condensation
boiling or vaporization occurs at a definite is less than rate of vaporization.

144
Demystified Series Physics Demystified by Dr Timothy
101. A vapour is said to be saturated when humidity is
A.more molecules return to the liquid than leave 𝑠𝑎𝑡𝑢𝑟𝑎𝑡𝑒𝑑 𝑣𝑎𝑝𝑜𝑢𝑟 𝑝𝑟𝑒𝑠𝑠𝑢𝑟𝑒 𝑎𝑡 𝑑𝑒𝑤 𝑝𝑜𝑖𝑛𝑡
× 100%
it B.a dynamic equilibrium exists between the 𝑠𝑎𝑡𝑢𝑟𝑎𝑡𝑒𝑑 𝑣𝑎𝑝𝑜𝑢𝑟 𝑝𝑟𝑒𝑠𝑠𝑢𝑟𝑒 𝑎𝑡 𝑎𝑖𝑟 𝑡𝑒𝑚𝑝𝑒𝑟𝑎𝑡𝑢𝑟𝑒

molecules and the vapour molecules at a IV.The saturated vapour pressure of water
given temperature C.the vapour pressure is increases with temperature A.I B.II C.II and III
atmospheric D.the vapour pressure is D.III and IV E.II,III and IV.
atmospheric E.all molecules are moving with the The molecules of vapour(or gases) have varying
kinetic energy but their average kinetic energy is
same speed.
Saturated vapour is in dynamic equilibrium with the same, also their molecules are at a greater
its own liquid in a confined space. distance apart than a liquid.
102. A vapour whose molecules are in dynamic 108. Which of the following is not correct?
equilibrium with those of its own liquid is said to A.boiling occurs when the saturated vapour
be A.unsaturated B.gaseous C.saturated pressure of the liquid involved equals the
D.diffused. external pressure B.both the boiling point and
103. Which of the following statements is not the saturated vapour pressure depends on
correct? A.An unsaturated vapour is one that the external pressure C.saturated vapour
is in contact with its own liquid B.A liquid boils pressure rises with increase in temperature
D.the saturated vapour pressure is independent
only when its saturation vapour pressure is equal
to the external pressure C.The dew point is not of volume availiable for the vapour E.it is possible
fixed but merely describes the condition of the to boil water at a lower temperature than 100℃
atmosphere at a particular time D.Still air favours in altitude.
the formation of dew E.Relative humidity can be Boiling point depends on external pressure while
measured by using a hygrometer saturated vapour pressure is independent of
An unsaturated vapour is not in contact with its external pressure and volume of the vapour
own liquid as it does not tends to escape from the available. Pressure decreases with altitude or
vessel as in evaporation. height, hence at high altitude pressure decreases
104. The saturation vapour pressure of a liquid and the boiling point of water decreases below
100℃.
depends on its A.volume B.temperature C.mass
D.density E.pressure. 109. Water boils at a lower temperature when
Saturated vapour pressure of a liquid is affected heated at the top of a mountain than at sea level
because at the top of a mountain A.saturation
by temperature of the liquid only. It is
independent of the volume of the vapour or the vapour pressure of water is higher than at sea
external pressure. level B.sun adds more heat to the water
105. The sarurated vapour pressure of a liquid C.temperature is lower than at sea level
increases as the A.volume of the increases D.relative humidity of the atmosphere is lower
B.volume of the liquid decreases C.temperature E.pressure of the atmosphere is lower.
of the liquid increases D.temperature of the Pressure decreases with height above sea level.
liquid decreases. At the top of a mountain i.e high altitude, the
Saturated vapour pressure increases with the pressure is lowered which inturn lowers the
temperature of the liquid. boiling point of water.
110. The following is a table of the 𝑆. 𝑉. 𝑃 of
106. Which of the following correctly represents
the variation of the vapour pressure of water water from 10℃ to 50℃.
with temperature? Temp/℃ 10 20 30 40 50
A. cmHg B. cmHg 𝑆. 𝑉. 𝑃/mmHg 9.2 17.5 31.8 55.0 92.5
On a particular day, when the atmospheric
pressure is 78.85cm of Hg, some water vapour is
present in a barometer tube which reads
100℃ 100℃ 72.62cm of mercury. What is temperature of the
C. cmHg D. cmHg surrounding? A.10℃ B.30℃ C.40℃ C.45℃
D.55℃.
Atmspheric pressure 𝑃𝑎𝑡𝑚 =78.85cmHg,
barometric reading = height of mercury column
100℃ 100℃. ℎ=72.67cmHg. The total pressure is equal to
107. Which of the following statements is/are the atmospheric pressure which is given by
not true? I.A saturated vapour is one that is in the sum of the saturated vapour pressure due
equilibrium with its liquid in a closed space II.A to the water vapour and the pressure due to
vapour contains molecues with varying kinetic the height of the mercury column. 𝑷𝑻 =
energy and their distances apart are smaller than 𝑷𝒂𝒕𝒎 = 𝑺. 𝑽. 𝑷 + 𝒉 , 𝑺. 𝑽. 𝑷 = 𝑷𝒂𝒕𝒎 − 𝒉 ,
than those of the molecules of a liquid III.Relative 𝑆. 𝑉. 𝑃 = 78.85−72.67 = 3.18cmHg = 31.8mmHg.

145
Demystified Series Physics Demystified by Dr Timothy
The temperature which corresponds to the to cause cooling E.Has a relative of fifty percent.
saturated vapour pressure of water from the Dew point is the temperature or point at which
table is 30℃. water vapour or moisture in the atmospehere
111. A simple barometer contains a thin layer of first condenses into the liquid form.
water on top of the mercury column at 20℃. If the 119. If a room is saturated with water vapour,
atmospheric pressure is 75.8cm of mercury and the temperature of the room must be A.at 0℃
the saturation vapour pressure of water vapour B.at 100℃ C.above the dew point D.below or at
at 20℃ s 1.7cm of mercury, calculate the height the dew point.
of the mercury column A.77.5cm B.77.1cm Air is unsaturated with water vapour above dew
C.76.1cm D.74.4cm E.74.1cm. point. Air is be saturated with water vapour at
𝑃𝑎𝑡𝑚 =75.8cmHg, 𝑆. 𝑉. 𝑃=1.7cmHg, ℎ=?, dew point. Air is supersaturated with water
𝑃𝑎𝑡𝑚 = 𝑆. 𝑉. 𝑃 + ℎ , ℎ = 𝑃𝑎𝑡𝑚 − 𝑆. 𝑉. 𝑃 , vapour below dew point and at this temperature
ℎ = 75.8−1.7 = 74.1cmHg. dew is formed as the air cdan no longer hold
112. A pressure cooker saves both time and fuel water vapour causing it to condense into dew.
in cooking because inside the cooker the 120. At low humidity in an environment, the
A.boiling point of water is raised B.pressure is human skin is usually A.dry and smooth B.damp
constant C.heat is completely trapped D.volume and smooth C.damp and rough D.dry and rough.
of steam varies E.temperature is evenly At low humidity, the rate of evaporation is high
distributed. and the skin is dry and rough while at high
Pressure cooker is used to cook foods at high humidity, the rate of evaporation is low and the
altitudes where pressure lowered. The pressure skin is damp and smooth.
in the pressure cooker makes water to boil above 121. On a day when humidity of the air is very
its boiling point i.e 100℃, hence food is cooked at high, evaporation from a lake will A.be rapid
a higher temperature saving both time and fuel. B.be slow C.be nil D.increase with temperature
Mountain dwellers cook with pressure cooker as D.decrease with temperature.
atmospheric pressure is low at high altitudes e.g 122. When the relative humidity of the
mountain tops, making to water boil below atmosphere is described as being 100%, it
100℃. implies that A.the air in the atmosphere is
113. The main reason why rice cooks faster in a saturated with water vapour B.water can easily
pressure cooker than in cooking pot is that A.less evaporate into the atmosphere C.the atmosphere
heat escapes from the cooker B.the vapour contains very little water vapour D.the air in the
pressure in the cooker is constant C.the rice is not atmosphere is very dry.
affected by draught over the cooker D.the vapour Relative humidity is the degree of humidity or the
pressure in the cooker decreases E.the boiling degree of water vapour present in the
point of water in the cooker is raised. atmosphere. A relative humidity of 100% menas
114. Clouds formation is the direct result of theair in the atmosphere is 100% humid or wet
A.precipitation B.vaporization C.fusion and the air in the atmosphere is completely
D.sublimation E.condensation. saturated with water vapour.
115. The phenomenon whereby water droplets 123. The mass of water vapour in a given volume
in the atmosphere combine with dust particles to of air is 0.05g at 20℃, while the mass of water
reduce visibility is A.fog B.hail C.mist D.cloud. vapour required to satiurate it at the same
116. The small droplets of water that form on the temperature is 0.15g. Calculate the relative
grass in the early hours of the morning is A.hail humidity of the air A.3.33% B.5.55% C.33.33%
B.dew C.fog D.mist. D.55.55%.
Dew is condensed water vapour formed on object Mass of water vapour in air 𝑚=0.05g, mass of
in contact with the ground while mist is water vapour required to saturate it 𝑀=0.15g,
suspended water droplets in air. relative humidity 𝑅. 𝐻=?,
117. The temperature at which the water vapour 𝑅.𝐻 =
𝑚𝑎𝑠𝑠 𝑜𝑓 𝑤𝑎𝑡𝑒𝑟 𝑣𝑎𝑝𝑜𝑢𝑟 𝑖𝑛 𝑎 𝑔𝑖𝑣𝑒𝑛 𝑣𝑜𝑙. 𝑜𝑓 𝑎𝑖𝑟

present in the air saturates the air and begins to 𝑚𝑎𝑠𝑠 𝑜𝑓 𝑤𝑎𝑡𝑒𝑟 𝑣𝑎𝑝𝑜𝑢𝑟 𝑟𝑒𝑞𝑢𝑖𝑟𝑒𝑑 𝑡𝑜 𝑠𝑎𝑡𝑢𝑟𝑎𝑡𝑒
𝑚 0.05
condense is known as A.boiling point B.melting 𝑅. 𝐻 = × 100 = ×100 = 33.33%.
𝑀 0.15
point C.triple point D.dew point E.critical 124. On a certain day, the dew point is 10℃ when
temperature. the temperature of the air is 32℃. Calculate the
Dew point is the temperature at which the water relative humidity of the air [𝑆. 𝑉. 𝑃 of water at
vapour present in air is just sufficient to saturate 32℃ and 10℃ are 16.2mm and 9.8mm of
it. mercury respectively] A.10.4% B.12.8% C.25.2%
118. Dew point is the temperature at which D.60.4% E.74.1%.
water vapour in the atmosphere A.Turns into 𝑆. 𝑉. 𝑃 at dew point of 10℃=9.8mmHg, 𝑆. 𝑉. 𝑃 at
steam B.Soliddifes into ice pellets C.First air temperature of 32℃=16.2mmHg, relative
condenses into liquid form D.Is just sufficient humidity 𝑅. 𝐻=?

146
Demystified Series Physics Demystified by Dr Timothy
𝑅. 𝐻 =
𝑆.𝑉.𝑃 𝑜𝑓 𝑤𝑎𝑡𝑒𝑟 𝑎𝑡 𝑑𝑒𝑤 𝑝𝑜𝑖𝑛𝑡
× 100, the wet and dry bulbs of a hygrometer is high,
𝑆.𝑉.𝑃 𝑜𝑓 𝑤𝑎𝑡𝑒𝑟 𝑎𝑡 𝑎𝑖𝑟 𝑡𝑒𝑚𝑝𝑒𝑟𝑎𝑡𝑢𝑟𝑒
9.8 this indicates that A.the relative humidity is high
𝑅. 𝐻 = × 100 = 60.4%. B.the relative humidity is low C.it is about to
16.2
125. If the partial pressure of water vapour at rain D.there is plenty of sunshine
27℃ is 18mmHg and the saturated vapour Relative humidity is inversely proportional to the
pressure at the same temperature is 24mmHg temperature difference between the wet and dry
the relative humidity at this temperature is bulbs of a hygrometer 𝑅. 𝐻 ∝
1
. When the
∆𝑇
A.25% B.33% C.75% D.82%. temperature difference is high it indicates a low
Partial pressure of water vapour at 27℃= relative humidity and when the temperature
18mmHg, 𝑆. 𝑉. 𝑃 of water at 27℃=24mmHg, difference is low is indicates a high relative
relative humidity 𝑅. 𝐻=?, humidity.
𝑃𝑎𝑟𝑡𝑖𝑎𝑙 𝑝𝑟𝑒𝑠𝑠𝑢𝑟𝑒 𝑜𝑓 𝑎𝑖𝑟
𝑅. 𝐻 = × 100 , - Relative humidity is 100% when the
𝑆.𝑉.𝑃 𝑜𝑓 𝑡ℎ𝑒 𝑎𝑖𝑟
18
𝑅. 𝐻 = × 100 = 75%. temperature difference is zero i.e the wet and
24 dry bulb are at the same temperature.
126. The table above shows the saturation 131.
vapour pressure against temperatuire in a
Day 1 Day 2 Day 3
certain town.
Dry 30℃ 29℃ 25℃
Wet 22℃ 22℃ 21℃
The readings above are for three consecutive
days from wet and dry bulb hygrometer. It can be
If the vapour pressure in this town at 20℃ is
concluded that the relative humidity for the three
10mmHg, what is the relative humidity?
days is A.decreased steadily B.remained
A.170.0% B.57.0% C.17.5% D.10.0%.
unchanged C.increased steadily D.was least on
Vapour pressure 𝑉. 𝑃 of air at 20℃=10mmHg,
day 1.
𝑆. 𝑉. 𝑃 of air at 20℃=17.50mmHg,
𝑣𝑎𝑝𝑜𝑢𝑟 𝑝𝑟𝑒𝑠𝑠𝑠𝑢𝑟𝑒 𝑜𝑓 𝑎𝑖𝑟 Relative humidity is higher when the
𝑅. 𝐻 = × 100, temperature difference between the wet and dry
𝑆.𝑉.𝑃 𝑜𝑓 𝑡ℎ𝑒 𝑎𝑖𝑟
𝑉.𝑃 10 1
𝑅. 𝐻 = × 100 = × 100 = 57.0%. bulb is lower. 𝑅. 𝐻 ∝ .
𝑆.𝑉.𝑃 17.50 ∆𝑇
127. At which of the following temperatures will Day 1 – ∆𝑇 = 30−22 = 8℃.
the relative humidity of the air in a room be least Day 2 – ∆𝑇 = 29−22 = 7℃.
if the dew point in the room is 20℃ A.10℃ B.15℃ Day 3 – ∆𝑇 = 25−21 = 4℃.
C.20℃ D.25℃ E.30℃. Relative humidity is least at day 1 as the
Air temperature is always greater than the dew temperature is greatest at day 1 (8℃).
point or dew point is always less than the air
temperature. Hence, the air temperature can Jamb past questions on thermal energy,
either be 25℃ or 30℃ which are above the dew latent heat and vapours :
point 20℃. Saturated vapour pressure increases 1978/15,18,19,21,24,1979/5,10,42,1980/15,32,
with temperature 𝑆. 𝑉. 𝑃 ∝ 𝑇, therefore 𝑆. 𝑉. 𝑃 at 1981/2,3,16,37,1982/4,17,27,1983/38,44,1984
30℃ is greater than 𝑆. 𝑉. 𝑃 at 25℃. /13,17,19,27,1985/19,23,24,25,1986/18,19,20,
𝑹. 𝑯 ∝
𝑺.𝑽.𝑷 𝒐𝒇 𝒘𝒂𝒕𝒆𝒓 𝒂𝒕 𝒅𝒆𝒘 𝒑𝒐𝒊𝒏𝒕
, relative 21,25,1987/21,22,23,1988/17,18,19,1989/16,
𝑺.𝑽.𝑷 𝒐𝒇 𝒘𝒂𝒕𝒆𝒓 𝒂𝒕 𝒂𝒊𝒓 𝒕𝒆𝒎𝒑𝒆𝒓𝒂𝒕𝒖𝒓𝒆
18,1990/18,19,1991/17,18,20,1992/17,18,
humidity increases with 𝑆. 𝑉. 𝑃 at dew point and 1993/17,18,19,1994/22,24,25,1995/22,24,25,
decreases with 𝑆. 𝑉. 𝑃 at air temperature. 1997/21,23,24,25,26,1998/21,22,1999/11,16,
Relative humidity is least when 𝑆. 𝑉. 𝑃 at air 24,2000/15,24,26,2002/20,2003/14,17,22,25,
temperature is greatest or maximum i.e at 30℃. 2004/39,2005/41,45,50,2006/26,43,45,49,
128. Which of the following may be used to 2007/35,40,2008/22,23,24,2009/21,23,2010/
determine the relative humidity in a physics 19,20,21,2011/16,18,20,2012/22,23,24,2013/
laboratory? I.Manometer II.Wet and dry bulb 19,22,2014/22,23,2015/14,47,2016/45].
hygrometer III.Hair hygrometer IV.Hydrometer
A.I B.II C.II and III D.III E.II,III and IV.
129. Which of the following instruments is used
to determine the relative amount of water
vapour present in the atmosphere?
A.hypsometer B.hydrometer C.barometer
D.hygrometer E.manometer.
Relative humidity or relative amount of water
vapour present in the atmosphere is measured
by a hygrometer.
130. When the temperature difference between

147
Demystified Series Physics Demystified by Dr Timothy

Chapter 14 – Heat transfer


● Heat transfer or flow – Heat flows naturally in size. II.Better conductors tends to conduct
from a hot region (high temperature region) to a heat away from a body rapidly compared to
cold region (low temperature region). insulators, hence the body feels cold when in
- Heat flow between two bodies in thermal contact with such conductors. III.Ignition point
contact when there is a temperature gradient of gas e.g wire gaze,fire fighting suits,miner’s
(difference in temperature per unit distance safety lamp. IV.Calorimeter and refrigerator
between the two regions). lagging, plastic covering of hot water storage
- Heat transfer process continues until both tank.
bodies in thermal contact attain the same - Heat transfer by conduction is similar to
temperature (thermal equilibrium), where there wave motion because no permanent transfer
is no net flow of heat. of materials is involved and energy is
- Thermal equilibrium is a condition where two transferred.
bodies in thermal contact are at the same - Newton’s law of cooling – It states that the rate
temperature and there is no net flow of heat of heat loss of a body in air is directly
between both bodies. proportional to difference in temperature
- Modes of heat transfer – Conduction, between the body and its surrounding or excess
convection, radiation. Heat is also transferred temperature over the surrounding.
partly by evaporation and condensation. - Rate of heat loss
𝑑𝑄
depends on difference in
𝑑𝑡
1.Conduction – Conduction involves the transfer
temperature between the body and its
of heat through a material medium by the
surrounding or excess temperature over the
vibratory motion of the molecules about their
surrpunding, nature of surface i.e dull or
mean position with higher frequency without
polished, and exposed area of the surface.
movement of the medium. 𝒅𝑸
- It occurs in solids, liquids and gases but majorly Rate of heat loss = 𝑲𝑨(𝜽 − 𝜽𝑺 ). 𝐾=constant
𝒅𝒕
in solids. depending on the nature of the surface,
- Heat transfer by conduction is by the free 𝐴=exposed area of the surface, 𝜃 − 𝜃𝑆 = difference
electrons. in temperature between the body and its
- Heat cannot be conducted through a vacuum. surrounding or excess temperature over the
- Thermal conductivity is the extent to which a surrpunding, 𝜃=temperature of the body,
material can conduct heat.It is dependent on the 𝜃𝑆 =temperature of the surrounding.
type of material.The decreasing order of - Rate of heat loss can also be expressed in terms
conductivity for common substance is; of of thermal capacity and specific heat– Rate of
Silver→Copper→Aluminium→Brass→Iron heat loss
𝒅𝑸 ∆𝜽
= 𝑪 = 𝒎𝒄
∆𝜽
.
∆𝜽
= rate of
𝒅𝒕 𝒕 𝒕 𝒕
(or steel)→Wood→Glass. Glass has the least temperature fall.
thermal conductivity. 2.Convection – Convection is the heat transfer
- Rate of heat flow 𝑹 or heat flux by conduction through a material medium in fluids, due to
∆𝜽
varies directly with temperature gradient , translational movement of the medium or
𝑳
cross sectional area 𝑨 and the nature of the migration molecules of the medium from hot to
material or thermal conductivity 𝑲. colder region.
𝑹=
𝒅𝑸 ∆𝜽
∝𝑨 , 𝑹=
𝒅𝑸
= 𝑲𝑨 .
∆𝜽 - It takes place in fluids i.e liquids and gases, but
𝒅𝒕 𝑳 𝒅𝒕 𝑳 majorly/most rapidly in liquids than gases. It
𝑲 is the constant of proportionality called the
does not occur in solids.
thermal conductivity of the conductor. The
- Convection is heat transfer due to density
unit is Joule per second per meter per kelvin
and temperature gradient.
[𝑱s-1m-1K-1] or Watt per meter per kelvin [𝑾m-
1℃-1].The unit for rate of heat flow is 𝐽s-1 or 𝑊.
- Convection is depends or affected by :
I.Nature of the surface. II.Nature or type of fluid.
- Temperature gradient 𝑻𝑮 is the difference in
III.Fluid properties(volume of the fluid, density,
temperature between the hot and cold end per
∆𝜽 𝜽 −𝜽 fluid viscosity, specific heat capacity and thermal
the distance between both ends. 𝑻𝑮 = = 𝑯 𝑪 conductivity). IV.Fluid velocity/velocity of
𝑳 𝑳
Its unit is Kelvin per meter [Km-1]. . molecules. It is indepdent of nature and shape
- Application of conduction – I.Insulators or of the vessel.
poor conductors of heat(e.g plastic,wood, - Archimedes’ principle applies in convection
rubber) are used in handle of cooking utensils or as it involves fluids.
kitchen wares while the body is made of a metal - Convection cannot occur in a vacuum.
of good thermal conductivity as it responds to - Natural convection – When water is heated on
temperature change quickly rather than increase a heater, the waterin contact with the hot base or

148
Demystified Series Physics Demystified by Dr Timothy
bottom of the vessel rises to the top because it is - Painting a hot metal ball black will increase the
less dense than cold water, while the cold dense heat lost instead of preventing it, as black
water moves down and a convectional current surfaces are good radiators of heat.
is set up. Natural convection current are also set - Temperature and surface area effect on
up as land and sea breeze due to difference in the radiatiom is combined by Josef Stefan-
densities between warm and cold air. Boltzmann’s law. It states that the radiant flux 𝑯
- Sea breeze – Sea breeze occurs during the (rate of emission of radiant heat energy per unit
day. It is the cool air/breeze which blows from time), emitted from a body per unit area 𝑨 is
the sea to land during the day. Land is warmer directly proportional to emmisivity 𝜺, and fourth
than the sea during day time due to the lower power of the absolute Kelvin temperature 𝑻.
thermal capacity of land, hence warm less dense 𝑯
∝ 𝜺𝑻𝟒 , 𝑯 ∝ 𝑨𝜺𝑻𝟒 , 𝑯 = 𝝈𝑨𝜺𝑻𝟒 . 𝐻 in W or 𝐽s-1,
𝑨
air from land rises and is replaced by cool dense
𝑇 in K, 𝐴 in m2, 𝜎=stefans-Boltzmann’s constant
air from sea setting up convectional current i.e
=5.67×10-8𝑊m2K-4.
sea breeze.
- Emmisivity 𝜺 of a body ranges from 1 for a
- Land breeze – Land breeze occurs during the
perfectly black or dull surface to 0 for a
night. It is the cool air/breeze which blows from
perfectly white or polished body.
land to sea during the night. Sea retains much of
- The emission and absorption of radiant energy
its heat due to its high thermal capacity and is
can be investigated with a Leslie’s cube. Radiant
warmer than land at night, hence warm less
heat can be detected by a thermopile.
dense air from the sea rises and is replaced by
- Application of radiation : I.Silvered surface at
cool dense air from land setting up convectional
the base of iron and tea pot. II.Use of aluminium
current i.e land breeze.
paints(silvered) in the roofs of buildings.
- Forced convection – A hot fluid could be forced
III.Direct transfer of heat from the sun to the
to circulate to a cold region and a cold fluid also
earth or from a charcoal fire to an occupant in the
circulating to a hot region continuously through
room. IV.Greenhouse effect. V.Radiator of a
the action of a fan.
motor cooling itself as it is coloured black.
- Application of convection in liquids –
- Thermos flask or vacuum flask – is used to
I.Domestic water boilers. II.Cooling of motor car
keep its content at the suitable temperature
engine by a radiator. III.Land and sea breeze.
(keep hot liquids from cooling down and cold
- Application of convection in gases –
liquids form getting warm), by preventing heat
I.Ventilation of industrial buildings. II.Chimneys
loss and gain mainly by conduction, convention
in kitchens. III.Domestic hot water supply
and radiation and partly by evaporation.
system.
- Features for preventing heat loss/gain in a
3.Radiation – Radiation is the heat transfer from
thermos flask – I.Double wall or vacuum space
hotter to colder place without heating of the
by evacuating the space between the glass walls
intervening medium. It is the heat transfer
prevents heat loss/gain by conduction and
through a body without a material medium or
convection. II.Polished surface/silvered double
heat transfer through a vacuum.
wall prevents heat loss/gain by radiation.
- It is the heat transfer by electromagnetic waves
III.Insulaing cork at the top or cork stopper
in form of infrared waves.
prevents heat loss/gain by conduction. It also
- Radiation cannot be explained by molecular
prevents heat loss by evaporation and
or kinetic theory of matter, as it doesn’t
convection. IV.Insulating cork at the bottom or
require a material medium.
cork suppport of low thermal conductivity,
- Heat energy given out by radiation is radiant
prevents heat loss/gain by conduction.
heat or radiant energy.
- The main sources of heat loss in a thermos
- All surfaces with temperature greater than
flask are the thin glass walls at the neck and
abolute zero temperature (0𝑲 or −273℃),
the poorly conducting cork at the bottom.
emits radiant heat.
- The working principle of refrigerator utilizes
- The intensity of radiant heat energy
the cooling effect of evaporation. Heat is lost in
emmited by a surface depends on –
a refrigerator by conduction, convection and
I.Temperature of surface 𝑇 II.Surface area 𝐴
radiation. A thermostat in the refrigerator is
III.Nature/colour of surface [black(dull) or
used to control the rate of vaporization, hence
bright(polished) surface].
degree of cooling.
- Black or dull surfaces e.g a surface covered
- A hot cup of tea placed on a table loses heat by
with soot, are good radiators, absorbers and
conduction through the walls of the cup, by
emmiters of heat and are poor reflectors.
convection to air currents in contact with the
- Silvered or polished surface are good
outer surface of the cup and evaporation from
reflectors of heat but poor absorbers,
the exposed surface of the tea, and by radiation
emmiters and radiators of heat.
from all exposed surfaces of the cup and its
149
Demystified Series Physics Demystified by Dr Timothy
content. of iron move more slowly than those of wood
- A hot metal ball suspended in the open air loses D.iron is a better conductor of heat than wood
heat as it cools by conduction and radiation. E.iron has a higher expansivity than wood.
- Heat reaches the food which is being cooked in The iron bar conducts heat away from the hand
a pot or saucepan chiefly by conduction and than the piece of wood, being a better conductor
convection. of heat, hence the hands feels cool when holding
- When some ice cubes are added to a glass of the iron bar and warm when holding the piece of
warm water, the glass is cooled by conduction wood.
and convection. 8. The blade of a hoe feels colder to touch in the
- The human body loses heat by conduction morning than the wooden handle because the
through the skin, by convection and radiation A.handle contains stored energy in form of heat
from the surface of the skin. Heat is also lost via B.blade is a better conductor of heat that the
evaporation of sweat from the skin. handle C.blade is placed at a lower temperature
than the handle D.handle is a better conductor of
Examples : heat than the handle.
1. The mechanism of heat transfer from one point The blade of a hoe being a metal conducts heat
to another through vibration of the molecules of away from hence making the hands feel cool. The
the medium is A.convection B.conduction hande of a hoe is made of wood a poor conductor
C.radiation D.diffusion. of heat.
Conduction involves the molecular vibration of 9. A concerte floor feels colder than a rugged
nolecules of a substance. floor on a cold morning because A.rug loses heat
2. When heat is applied to one end of a metal rod, to the bare feet at a faster rate than the concrete
the molecules at the other end soon begin to B.rug loses heat to the bare feet at a slower rate
vibrate with greater amplitude than before than concrete C.rug is a better conductor of heat
because heat has been transferred by A.radiation than concrete D.concrete is a better conductor
B.convection C.conduction D.evaporation. of heat than rug.
The statement is the molecular explanation of Poor thermal conductors have high thermal
conduction using kinetic theory. capacities while good thermal conductors have
3. In which of the following are the substance low thermal capacities.
arranged in descending order of their thermal 10. The temperature gradient across a copper
conductivities? A.copper, steel, glass B.steel, rod of thickness 0.02m, maintained at two
copper, glass C.steel, glass, copper D.copper, temperature junctions of 20℃ and 80℃
glass, steel. respectively is A.3.0×102Km-1 B.3.0×103Km-1
Metals generally conduct heat than non-metals. C.3.0×104Km-1 D.5.0×103Km-1.
The best heat conductors in descending order are Thickness of the rod 𝐿=0.02m, 𝜃𝐻 =80℃ , 𝜃𝐶 =20℃
: silver, copper, gold, aluminium and steel. , temperature gradient=?,
4. Which of the following makes metal ideal for ∆𝜃
Temperature gradient 𝑇𝐺 = = 𝐻 𝐶 ,
𝜃 −𝜃

cooking utensils? A.high coefficient of expansion 80−20 60


𝐿 𝐿

B.good conduction of heat C.poor radiator of 𝑇𝐺 = = =3000℃m-1 ,


0.02 0.02
heat. 𝑇𝐺 =3.0×10 ℃m or 3.0×103Km-1.
3 -1

Metals being good conductors of heat transfer 11. A lagged steel bar of length 50cm has its ends
heat and respond to temperature changes immersed in melting ice and boiling water.The
quickly . temperature at a point 20cm from the hot end is
5. Cooking pots are usually made of metals A.40℃ B.50℃ C.60℃ D.80℃.
because metals A.have high coefficient of Temperature of the end immersed in boiling
expansion B.have low specific heat capacity C.are water 𝜃𝐻 =100℃ , temperature of the end placed
good conductors of heat D.are poor conductors in melting ice 𝜃𝐶 =0℃, length of the steel bar
∆𝜃 𝜃 −𝜃
of heat. 𝐿=50cm, Temperature gradient 𝑇𝐺 = = 𝐻 𝐶
𝐿 𝐿
6. Which of the following cannot be explained by 100−0
using the concept of conduction of heat? 𝑇𝐺 = = 2℃cm-1. The point 20cm from the
50
A.calorimeter lagging B.land and sea breezes hot end is same 30cm from the cold end. Hence,
C.the working of a miner’s safety lamp the temperature at that point, 𝜃 = 2℃cm-
1×30cm = 60℃ or 40℃ from the cold end.
D.refrigeration E.the use of fighting suits.
Land and sea breezes are natural conventional Alternatively – Given that the length of the lagged
currents and can be explained by convection. (well insulated), steel bar is 50cm and it ends are
7. An iron bar at 0℃ feels colder in the hand than immersed in melting ice at 0℃ and boiling water
a piece of wood at 0℃ because A.iron is heavier at 100℃. 0℃ 𝜃 100℃
than wood B.the molecules of iron vibrate more 𝑃 30cm 𝑄 20cm 𝑅
violently than the molecules of wood C.molecules 50cm

150
Demystified Series Physics Demystified by Dr Timothy
∆𝜃 𝜃 −𝜃 100−𝜃 𝜃
Temperature gradient = = 𝐻 𝐶 , 40×25π×10-6× =360×25π×10-6× ,
𝐿 𝐿 0.15 0.05
Temperature gradient 𝑻𝑮 is constant across 40(100−𝜃)
=
360𝜃
,
40(100−𝜃)
=
0.15
,
100−𝜃
=3,
any section along the length of a bar. Hence, 0.15 0.05 360𝜃 0.05 9𝜃
100−𝜃 = 3×9𝜃 = 27𝜃 , 100 = 27𝜃 + 𝜃 = 28𝜃 ,
temperature gradient of section 𝑃𝑄 is equal to 100
that of section 𝑃𝑅. 𝜃= = 3.57℃ ≈ 3.6℃.
28
𝑇𝐺 across section 𝑃𝑄 =
𝜃−0 𝜃
= , II. Rate of heat flow along the bar ?
30 30 𝑑𝑄 ∆𝜃 𝜃 −𝜃
100−0 100 𝑅= = 𝐾𝐴 = 𝐾𝐴 𝐻 𝐶 . It can be calculated
𝑇𝐺 across section 𝑃𝑅 = = = 2 Km-1, 𝑑𝑡 𝐿 𝐿
50 50
using the parameters of copper or iron.
𝑇𝐺 across section 𝑃𝑄 = 𝑇𝐺 across section 𝑃𝑅, 𝑑𝑄 3.6−0
𝜃 𝑅= = 360× 𝜋(5×10-3)2× ,
= 2 , 𝜃 = 2×30 = 60°. 𝑑𝑡 0.05
30 𝑑𝑄 3.6
12. What are the units of thermal conductivity 𝑅= = 360×25𝜋 ×10-6× = 0.648𝜋 ,
𝑑𝑡 0.05
𝑑𝑄 22
A.kg m sec2 B.Joule sec-1 m-1 K-1 C.kg m 𝑅= = 0.648× = 2.04Js-1 or 2.04W.
𝑑𝑡 7
D.Newton sec-1 m-1 K -1 E.m2 . 16. The time rate of loss of heat by a body is
13. An iron plate 2cm thick has a cross-sectional proportional to the A.difference in
area of 0.5m2 with temperature of 15℃ and temperature between the body and its
150℃ on both sides. Calculate the amount of heat surroundings B.temperature of its surrounding
that passes through the plate each second C.ratio of the temperature of the body to that of
(Thermal conductivity of iron= 59𝑊m-1𝐾 -1) its surroundings D.temperature of the body
A.18.4W B.36.9W C.184.4W D.368.8W. According to Newton’s law of cooling : the rate of
Thickness 𝐿=2cm=0.02m, 𝐴=0.5m2, 𝜃𝐶 =15℃, 𝑑𝑄
𝜃𝐻 =150℃, 𝐾=59𝑊m-1K-1, amount of heat passing heat loss of a body in air is directly proportion-
𝑑𝑡
𝑑𝑄
through the plate each second 𝑅 = =?, al to difference in temperature between the body
𝑑𝑄 ∆𝜃 𝜃𝐻 −𝜃𝐶
𝑑𝑡 and its surrounding or the excess temperature
𝑅= = 𝐾𝐴 = 𝐾𝐴 , over the surrounding ∆𝜃𝑆 . 𝑅 =
𝑑𝑄
∝ ∆𝜃𝑆 ,
𝑑𝑡 𝐿 𝐿
𝑑𝑄 150−15 125 𝑑𝑡
𝑅= = 59×0.5× = 29.5× , 𝑅=
𝑑𝑄
∝ (𝜃 − 𝜃𝑠 ).
𝑑𝑡 0.02 0.02
𝑑𝑄 𝑑𝑡
𝑅= = 184.375W = 184.4W. 17. On which of the following will the rate of heat
𝑑𝑡
14. If 1.2×106𝐽 of heat energy is given off in loss of a body to the surrounding? I.exposed
1sec from a vessel maintained at a temperature surface area of the body II.the shape of the body
gradient of 30Km-1, the surface area of the vessel III.the density of the body IV.the temperature
is [thermal conductivity of the vessel=400𝑊m- difference between the body and the
1𝐾 -1] A.1.0×102m2 B.9.0×102m2 C.1.0×103m2 surrounding A.I and II B.I and IV C.II and III D.III
D.9.0×104m2. and IV.
𝑄=1.2×106𝐽, 𝑡=1s, 𝑇𝐺 =30Km-1, 𝐾=400𝑊m-1K-1, Rate of heat loss of a body depends on the
𝐴=?, 𝑅 =
𝑑𝑄 ∆𝜃
= 𝐾𝐴 = 𝐾𝐴𝑇𝐺 , exposed surface area of the body, difference in
1.2×106
𝑑𝑡 𝐿
1.2×106
temperature between the body and its
= 400× 𝐴 × 30 , 𝐴 = = 1.0×102m2. surrounding or excess temperature over the
1 400×30
15. 𝜃 surrpunding and nature of surface(dull or
100℃ iron copper 0℃ shiining/polished surface). It does not depend on
The diagram above illustrates a composite bar of the shape or density of the body.
iron and copper. The bar is insulated along its 18. According to Newton’s law of cooling, the
sides and it has a diameter of 10mm. The length heat energy lost per unit area of a body depends
and thermal conductivity of the iron are 0.15m on the I.Surface area II.Nature of the surface
and 40Wm-1K-1 respectively and those of copper III.Temperature of the surface IV.Excess
are 0.05m and 360Wm-1K-1, respectively. If the temperature over the surroundings A.I and II B.II
free ends of the iron and copper are kept at 100℃ and IV C.I,II and III D.I, II and IV.
and 0℃ respectively, calculate the Based on Newton’s law of cooling : the rate of
I. Temperature θ at the interface between the heat loss depends in the excess temperature over
bars ? the surrounding, exposed surface area and the
𝐿 of iron=0.15m, 𝐾 of iron=40Wm-1K-1, 𝜃𝐻 of nature of the surface. It does not depend on the
iron=100℃, 𝜃𝐶 of iron=𝜃, 𝐿 of copper=0.05m, 𝐾 temperature of the surface.
of copper=360Wm-1K-1, 𝜃𝐻 of copper=𝜃, 𝜃𝐶 of 19. Thermal equilibrium between two objects
copper=0℃, diameter=10mm, radius 𝑟= =5mm
10 exists when A.the temperature of both objects
2 are equal B.the quantity of heat in both bodies
=5×10-3m, 𝜃=equilibrium temperature. As the are the same C.the heat capacities of both bodies
bar is insulated, the heat lost by the iron=heat are the same D.one object loses heat
𝑑𝑄 ∆𝜃 𝜃 −𝜃
gained by copper. 𝑅 = = 𝐾𝐴 = 𝐾𝐴 𝐻 𝐶 , continuously to the other.
𝑑𝑡 𝐿 𝐿
40×π(5×10-3)2×
100−𝜃
=360×π(5×10-3)2×
𝜃−0
, 20. Two bodies 𝑃 and 𝑄 are in thermal equilibri-
0.15 0.05

151
Demystified Series Physics Demystified by Dr Timothy
um. Which of the following statements about the conduction II.Convection occurs without any
bodies is correct? A.The temperature of 𝑄 is change in the average position of the particles of
higher than that of 𝑃 B.𝑃 and 𝑄 ave the same heat the medium concerned II.The principle of the
capacity C.𝑃 and 𝑄 have the same mass D.𝑷 and miner’s safety lamp is based on the fact that a
𝑸 are at the same temperature. metal guaze is a good conductor of heat A.I B.II
21. Two bodies 𝑃 and 𝑄 are in thermal contact. C.III D.I and II E.I and III.
Heat can flow from one body to another if A.the A hot water circulating system trasfers hear by
temperature of the bodies are in equilibrium convection due to difference in densities
B.the bodies have the same thermal conductivity between warm and cold water. Convection
C.a temperature gradient exists between the occurs with a change in the average position of
bodies D.the bodies have the same thermal the particles of the medium involved. Miner’s
capacity. safety lamp is based on heat transfer by
Heat can only flow between two bodies at conduction.
different temperatures i.e temperature gradient 27. In domestic water boilers, heat transfer is
exists between them. Heat flows from a body at mainly by A.conduction B.convection
higher temperature to a body at a lower C.transmission D.radiation.
temperature. No net flow of heat occurs when the Domestic water boilers are used to supply hot
temperature of bodies in thermal contact are in and cold water in homes. It is done by the rise of
equilibrium or same. hot less dense water and fall of cold dense waterb
22. Heat transfer conduction is similar to wave in the boiler.
motion A.transfer of materials is involved 28. Ice wrapped in guaze placed at the bottom of
B.there is increases in temperature C.no transfer a tube filled with water remains unmelted when
of materials is involved D.charged particles heat is applied to the top of the tube. This
vibrate at right angles. experiment shows that A.ice cannot sink in water
Heat transfer by conduction and wave motion are B.water is a bad conductor of heat C.water is a
similar because no transfer of materials is good medium to show convention D.ice is a solid
involved and energy is transferred in both. Heat that conducts heat.
transfer by conduction involves increase in Heat transfer in water and other liquids is by
temperature and chanrged particles vibrate but convection and not conduction. When the water
not at right angles to the direction of heat flow. molecules at the top of the test tube is heated,
Wave motion involves no increase in remains afloat at the top as it is less dense than
temperature and charged particles vibnrate at the cold liquid below. Hence, heat is not
right angles to the direction of wave motion i.e transferred to the bottom of the liquid and the ice
transverse wave. will not melt. If the test tube was heated from the
23. Heat transfer by convection in a liquid is due top rather than the top, the ice will melt as it is
to the A.translatory motion of the molecules heated directly and the warm less dense water
of the liquid B.increased vibratory of the molecules at the bottom rises to the top to
molecules of the liquid about their mean position replace the cold dense water molecules
C.expansion of the liquid as it is heated D.latent 29. Land breeze occurs A.during the day B.as a
heat of vaporization E.ability of the liquid to result of heat transfer by radidation C.as a result
evaporate at all temperature of heat transfer by convection D.as a result of
Convection involves the translational motion of heat transfer by conduction.
molecules of a the liquid. Convection is heat Land breeze occurs during the night while sea
transfer by molecular motion. breeze occurs during the night. Both land and sea
24. Heat transfer by convection in a liquid is due breeze are as a result of heat transfer by
to the A.increased vibration of the molecules of convection.
the liquid about their mean position B.variation 30. Cold breeze across a sandy beach in a sunny
of the density of the liquid C.expansion of the afternoon because A.the heat of the sun is
liquid as it is heated D.latent heat of vaporization absorbed by the sea B.the sea is warmer at that
of the liquid. time of the day C.cool air blows from the beach of
25. Heat transfer by convection will occur most the sea D.warm air on the beach is replaced by
rapidly in one of the following subatnces cooler air from the sea.
A.oxygen B.water C.wood D.mercury Cool breeze across a sandy beach in a sunny
Convection occurs in fluids i.e liquids and gases afternoon is sea breeze which occur during the
but majorly in liquids. Heat transfer in mercury is day. It is the conventional current set up when
by conduction, as mercury is a liquid metal and a warm less dense from the land /beach is replaced
conductor. by cool dense air from the sea.
26. Which of the following is/are not correct? I.A 31. In the formation of sea breeze, wind blows
hot water circulation system transfers heat by from A.sky to land B.sea to sky C.land to sea

152
Demystified Series Physics Demystified by Dr Timothy
D.sea to land. directly. The man is also heated partly by
32. The mode of heat transfer which does not convection as the charcoal fire warms air above
require a material medium is A.radiation it, which rises and circulates round the room by
B.propagation C.conduction D.convection. convectional currents.
33. Heat may be transferred by conduction, 42. Which of the following statements is not
convection and radiation. By which of these correct? A.Good emitters of radiant energy are
methods does heat travel through vacuum? also good absorbers B.Radiant heat can be
A.conduction only B.convection only C.radiation detected by a thermopile C.Infrared rays are heat
only D.all three methods E.None of the methods. rays D.Heat is transmitted through a vacuum by
34. Which of the following cannot be explained radiation E.Shiny surfaces are better
by the molecular theory of matter? A.conduction absorbers than dull surfaces.
B.convection C.evaporation D.expansion Good emitters of radiant energy are also good
E.radiation. absorbers and good radiators. Radiant heat can
35. The heat from the sun reaches the earth by be detected by a thermopile. Infra-red rays are
the process of A.insulation B.precipitation also called heat rays. Polished or shiny surfaces
C.conduction D.radiation E.convection are poor absorbers than dull or darkened
Heat from the sun reaches the earth by radiation surfaces.
as no material medium is needed. 43. A perfect emitter or absorber of radiant
36. A man standing 2m from a charcoal fire is energy is a A.white bodyB.red body C.black
warmed mainly by A.convection B.radiation body D.conductor.
C.refraction D.conduction. Black body are perfect emitter or absorber or
37. Heat is radiated by all hot objects in the form radiator. White body are perfect reflector.
of A.light energy B.solar energy C.infrared rays 44. Highly polished silvery surface are A.poor
D.x-rays. absorbers and poor emitters of radiation
38. Which of the following statements about heat B.good absorbers and good emitters of radiation
transfer is true? A.conduction occurs due to C.good absorbers but poor emitters of radiation
translational motion of the molecules of the D.poor absorbers but good emitters of radiation
medium B.fresh air can be made to circulate in a Highly polished silvery surface are poor emitters
room by the use of radiation process C.a hot and poor absorbers of radiation but good
water circulating system trasnfers heat by reflectors.
convection D.heat from the sun reaches the 45. It is advisable to wear white dresses in
earth by conduction process. tropics because white can A.absorb sweat
Conduction occurs due to vibrational motion of readily B.absorb all radiant heat C.conduct heat
the molecules of the medium about its mean away more readily from the body D.reflect
position. Fresh air can be made to circulate in a radiant heat E.absorb the rays from the sun.
room by forced convention e.g using a fan. Heat The temperature in tropics is very high, hence it
from the sun reaches the earth by radiation. is advisable to wear white dresses which are
39. The ground is always cold at night because good reflectors and poor absorbers of radiant
A.sun no longer shines at night B.atmosphere heat.
reflects sun’s energy at night C.atmosphere 46. Which of the following phenomena explains
absorbs the sun’s energy D.earth radiates heat the fact that a house whose roof is coated with
to the atmosphere at night. white paint will be cooler than one painted with
The earth absorbs radiant heat from the sun black paint? A.conduction B.convection
during the day and radiates the heat to the to C.reflection D.refraction.
back to the air at night, hence making the earth White paints are good reflector and poor
cool at night. absorber or radiator or emitterof radiant energy,
40. The radiator of a motor car is cooled by hence it reflects most the heat incident on the
A.radiation and conduction B.conduction roof, preventing the temperature of the house
C.radiation D.convection from being raised. Black paints are poor reflector
The radiator of a motor car being blackened is and good absorber or radiator or emitter of
cooled by radiation. The radiator cools the engine radiant energy, hence it absorbs most of the heat
of an automobile or motor car by convection. incident on the roof , raising the temperature of
41. A room is heated by means of a charcoal fire. the house.
A man standing away from the fire is warmed by 47. Which of the following statements about
A.conduction only B.conduction and convection radiant heat is/are correct? I.Radiant heat
C.convection only D.convection and radiation cannot travel through a vacuum II.Rough
E.radiation only surfaces emits radiant heat more than polished
The man is heated mainly by radiation as the fire surfaces III.Dark surfaces absorbs radiant heat
emits radiant heat and it reached the man better than bright surfaces A.I B.II C.III D.I and II

153
Demystified Series Physics Demystified by Dr Timothy
E.II and III. materials for hot weather D.Convection currents
48. Two similar kettles containing equal masses play a large part in the cooing process of a motor
of boiling water are placed on a table. If the car engine E.The vacuum space in a thermos
surface of one is highly polished and the surface flask helps to reduce heat losses by radation
of the other is covered with soot, which of the Air is a poor conductor or insulator, hence can be
following observations is correct A.the two used between a heated material and the lagging.
kettles will cool down at the same rate B.the Cotton materials tends to absorbs heat less than
polished kettle cools down more quickly by woolen materials making them suitable for hot
conduction C.the kettle covered with soot weather. The vacuum space in a thermos flast
cools down more quickly because it is a good helps to reduce heat loss by conduction and
radiator of heat D.the kettle covered with soot convection, as they require material medium.
cools down more quickly by the process of heat 54. Which of the following statements are
convection. correct? I.Land and sea breezes are natural
The kettle covered with soot(black surface) cools convection currents II.Convection may occur in
i.e its tempeeratrure falls down quickly, faster liquids or gases but not in solids III.The vacuum
than the highly polished kettle, as the kettle in a thermos flask prevents heat loss due to
covered with soot radiates or loses and absorbs convection only A.I and II B.I and IIII C.II and III
heat faster. D.I,II and III.
49. Two closed vessels contain equal volumes of 55. Which of the following statements is correct?
boiling water and are identical except that one is Silvered walls of a vacuum flask are used to
blackened on the outside while the other is prevent A.heat loss due to radiation B.heat loss
polished. Which of the following is correct? due to conduction C.vacuum loss D.heat entry
A.The polished vessel will cool faster because a into the inner flask due to convection E.heat loss
polished surface is a good radiator B.The due to convection.
polished vessel will cool faster because because The silvered wall surface prevents heat loss/ gain
it will absorb less heat from the room C.The black by radiation, as silvered surface are poor
vessel will cool faster because it is a better radiators or emitters or absorbers of heat.
conductor of heat D.The polished vessel will cool 56. Heat loss by radiation from a hot liquid in a
faster because it reflects heat better E.The black Thermos flask is reduced mainly by the
vessel will cool faster because a black surface A.vacuum between the glass walls B.silvering of
radiates heat faster than a polished one. the glass walls C.outer container D.insulating
50. Heat radiation occurs most rapidly from a cork cover E.glass which is transparent.
surface which is A.shinning black B.white and 57. A vacuum is needed between the two glass
black C.white and black D.dull and black. walls of a vacuum flask A.to prevent heat loss by
51. Which of the following is not a suitable radiation B.to prevent heat gained by radiation
method for reducing heat loss from a hot metal C.becasue air is a conductor of heat and will
ball? A.placing it in a vacuum B.painting it black conduct heat to the outside D.because vacuum is
C.placing it on a rubber support D.wrapping it a good absorber of heat E.to reduce heat losses
with cotton wool. by conduction and convection.
Placing the hot metal ball in a vacuum prevents 58. The thermos flask is designed to A.store hot
heat loss by conduction, as not material medium tea or water B.prevent heat loss or gain by
is present. Painting the metal block black will aid conduction, convection and radiation
heat loss by radiation and not prevent heat loss, C.prevent heat gain by conduction, convection
as black bodies are good radiators. Placing the and radiation D.prevent heat loss by conduction,
hot metal ball on a rubber support or wrapping it convection and radiation.
with cotton wool both of which are insulators, 59. The space between the double glass walls of
prevents heat loss by conduction. a thermos flask is evacuated and the two surfaces
52. Which of the following methods is not facing the evacuated space are silvered. The
suitable for reducing loss of heat from a piece of residual source of heat loss takes place by
hot iron A.wrapping it in cotton wool B.placing it A.convention B.radiation from the surfaces
on rubber supports C.painting it black C.conduction through the stopper and the
D.keeping it in a closed wooden box E.placing it glass D.conduction across the walls.
in a vacuum. The double glass walls with an evacuated space
53. Which of the following statements is not (vacuum) prevents heat loss by convection and
correct? A.The circulation of air which gives rise conduction. The silvered walls prevents heat loss
to sea breeze during the day is due to convection by radiation. Hence, heat can only be lost by
B.An air space can be used between a heated conduction through the cork stopper or
material and the lagging to improve insulation insulating cork at the top and the cork support or
C.Cotton materials are better than woolen insulating cork at the bottom through the glass to

154
Demystified Series Physics Demystified by Dr Timothy
the surrounding.
60. The main reason for making the cover of a
vacuum flask air tight is to prevent heat loss by
A.conduction B.evaporation C.radiation
D.convection.
A Thermos or vacuum flask not poorly closed or
not air tight will lead to evaporation of the liquid
molecules of its content, leading to the cooling or
the content.
61. In a good Thermos flask, the main cause of
heat loss is A.conduction through the cork
B.the plastic base of the Thermos flask C.the
silvered walls and shiny metals D.the outer cover
or jacket.
The main causes of heat loss in a Thermos flask
are the thin glass walls at the neck and the poorly
conducting cork at the bottom or cork stopper.
62. The coils and fins at the back of a refrigerator
are painted black so that the refrigerator A.can
produc e ice blocks B.may not lose its cool
temperature C.can absorbs air for effective
cooling D.can loose heat more rapidly by
radiation.
63. The intensity of heat radiated by a surface
depends on the following except A.surface area
B.absolute temperature C.thermal conductivity
D.nature of the surface
Radiation depends on nature of the surface,
surface area and absolute temperature. It is
independent of thermal conductivity.
64. Which of the following has no effect on
radiation A.Temperature B.Surface area
C.Nature of the substance D.Density.

Jamb past questions on heat transfer :


[1981/20,1982/43,1986/1,1987/24,1987/24,
25,26,1988/20,1990/20,1991/22,1992/19,
1994/27,1995/26,1999/26,2000/22,2002/25,
2003/26,2004/36,42,2007/32,2008/25,2009/
25,2010/18,23,2011/17,22,2013/24,2014/25]

155
Demystified Series Physics Demystified by Dr Timothy

Chapter 15 – Wave : Types, Propagation and Properties


● Wave is a disturbance which travels through a wave from rest or equilibrium or mean position.
medium or without a medium, transferring or - Phase – Particles or points are said to be in
propagating energy from one point to a other phase when they occupy similar positions or at
without any permanent displacement of the the same vertical distance from their equilibrium
medium. position or are moving in the same direction.
- A wave is a combination of both translational - Cycle is a complete oscillation of a vibrating
and oscillatory/vibratory motion. The particle.
particles of the medium vibrate or oscillate about - Wavelength  - is the distance between two
their equilibrium position while the wave itself consecutive crests or troughs or compressions or
spreads forward over a long distance i.e rarefractions. It is the distance between two
transaltional motion. successive points in phase or distance advanced
- Classification of waves – Waves are classified when a wave completes one cycle.
based on the requirement of a medium for - The distance between adjacent crests or
propagation i.e mechanical and electromagnetic troughs or compressions or rarefractions is equal
waves, direction of vibration of the particles of to one wavelength.The distance between
the medium relative to the wave motion i.e adjacent compressions and rarefractions or
transverse and longitudinal wave. adjacent crests and troughs is equal to half
- Mechanical waves – They require a material wavelength.
medium for propagation e.g water waves, sound - Crest and trough are respectively maximum
waves,wave in strings. upward and downward displacement in
- Electromagnetic waves – They don’t require a transverse waves while compressions and
material medium for propagation or they travel rarefractions are respectively maximum upward
through a vacuum e.g gamma rays, x- and downward displacement in longitudinal
rays,ultraviolet rays, light rays,infra red rays, waves.
radio waves and mircro-waves (GXULIRM). The -Frequency𝒇 of a wave is the number of waves
order of the electromagnetic spectrum GXULIRM, completed in one second. Frequency is the
shows an increasing wavelength and fundamental property of a wave which
decreasing frequency or penetration power depends only on the source of wave and not
hence, Gamma rays has the highest frequency or the medium of propagation hence, frequency
penetration power and lowest wavelength. They of wave equals the frequency of source
all moves with the same speed in a vacuum producing wave.
equal to the speed of light (3×108ms-1). - The frequency of devices with regularly ruled
- Transverse waves are waves with direction of lines or holes e.g stroboscope, gramophone
motion perpendicular to the direction of records, disc siren, and spoke of bicycles or
propagation e.g water waves, electromagnetic cart is given by – 𝒇 = 𝒏𝒗, 𝑓=frequency in Hertz,
waves. 𝑛=number of holes or regularly ruled lines,
- Longitudinal waves are waves with direction 𝑣=speed in revolution per second(rev/s)
of motion is parallel to the direction of - Period 𝑻 is the time taken for the wave for one
propagation e.g sound wave, wave in a spring. complete wave or time taken to move one
- Wave in strings is both mechanical and complete wavelength. 𝑻 = .
𝟏

transverse, wave in springs is both mechanical 𝒇

and longitudinal, sound wave is both mechanical -Wavefront is any line or section taken through
and longitudinal, and light wave is both an advancing wave in which all the particles are
electromagnetic and transverse. vibrating in the same phase or in the same state
- Water waves is both mechanical and transverse of disturbance. Wavefront shows how energy
wave. It can be studied in the laboratory by a is being transmitted from one point to
ripple tank. A stroboscope is used to make the another. The direction of propagation of a
water waves appear stationary. wave or the direction of energy flow in wave
- Water waves travels faster in deep water motion is drawn perpendicular or normal to
than in shallow water, 𝒗 ∝∝ depth. the wavefront.
- Terms used to describe wave – - Wave speed 𝒗 is the distance 𝒔 travelled by the
𝒔
- Pulse is the sudden increase in the magnitude wave in one second 𝒕 = 𝟏. 𝒗 = = 𝒇 = /𝑻.
𝒕
of a physical quantity shortly followed by a rapid - Particle velocity is the displacement of the
decrease.Pulse is seen mostly in water waves. A particles about their mean position in one
pulse consists of a single crest. second. 𝑽 = 𝝎𝑨, 𝜔=angular speed or angular
- Amplitude 𝑨 – is the maximum displacement of frequency of the particle, 𝐴=amplitude.

156
Demystified Series Physics Demystified by Dr Timothy
- Waves can be described based on their degree - The sign is negative, when the wave moves
of movement or restriction i.e progressive and from left to right i.e positive-𝑥 direction and
stationary wave. positive, when the wave moves from right to left
- Progressive or travelling waves are waves i.e negative-𝑥 direction.
that spreads out from a source transferring - Phase velocity, 𝒗 =
𝒂𝒏𝒈𝒖𝒍𝒂𝒓 𝒇𝒓𝒆𝒒𝒖𝒆𝒏𝒄𝒚
= .
𝝎

energy without any restriction. 𝒘𝒂𝒗𝒆 𝒏𝒖𝒎𝒃𝒆𝒓 𝒌


- Properties of wave – All waves exhibit the the
- Energy is the property of a wave that is
properties: reflection, refraction, diffraction and
propagated in a progressive or travelling
interference.
wave.
1. Reflection is the bouncing back of a wave from
- Standing or stationary waves are produced by
a surface in a different direction, that can’t be
the overlapping or interference two equal
penetrated or with minimum absorption. The
progressive of equal amplitude and frequency
velocity, frequency and wavelength of the wave
travelling in opposite direction. It is
is unchanged.
alsoproduced when progressive wave is
- Reflection principle is applied in determining
reflected from a surface.
the depth of a sea.
- Stationary waves do not transfer energy.
2. Refraction is the change in direction,velocity
- The amplitude of a progressive wave can be
and wavelength of a wave as it travels from one
constant while that of stationary wave is always
medium to the other with different densities.
constantly changing or varying.
Frequency of wave is the only property of
- The amplitude of the resultant wave is the
wave which is constant during refraction.
algebraic sum of the amplitude of both waves.
3. Diffraction is the spreading out or bending of
- The superimposition of both waves creates a
waves around corners or obstacles.
region of zero resultant disturbance as both
- Diffraction is pronounced and noticeable
waves are of the same amplitude and frequency
when: the width of the opening or aperture is
and are out of phase.
about the same order as the wavelength of the
- Nodes (N) are the points with minimum
waves and the opening is narrow i.e small
displacement or zero amplitude or at rest while
opening.
Antinodes (A) are points with maximum
- Diffraction does not takes place when the
displacement or amplitude in a stationary wave.
opening between two barriers or slit is large or
- The distance between two consecutive
has a wide aperture.
nodes NN or antinodes AA is half wavelength,
 - Sound waves requires a larger opening than
. The distance between a node and antinode water waves and water waves larger than light
𝟐
 waves due to their wavelengths i.e the
NA is quarter wavelength, .
𝟒
wavelength of sound waves is greater than that
- A stationary wave can be produced by both
of water waves, which in turn is greater than that
transverse and longitudinal waves.
of light waves, S>W>L.
- Equation of progressive wave – A wave can
- Examples of diffraction: I.Formation of
either travel in the positive 𝑥-direction or
blurred and brighter image in a pin hole camera
negative 𝑥-direction. The motion of a
with a large hole. II.Distributing of sound round
progressive wave is generally represented by the
the corners in a room through large openings.
equation : 𝒚 = 𝑨 𝒔𝒊𝒏(𝜽 ±), 𝑦=vertical
III.Bending of water waves through a narrow
displacement of particles, 𝐴=amplitude of wave,
opening.
𝜃=angular displacement= 𝜔𝑡, =phase angle or
4. Interference is the combination/overlapping
difference= 𝑘𝑥.
of identical waves of same amplitude,frequency
- Other equations derived from the general
and wavelength which are travelling in the same
equation above are given below –
or opposite direction.
1. 𝒚 = 𝑨 𝒔𝒊𝒏(𝝎𝒕 ± 𝒌𝒙), 𝜔=angular frequency or
𝟐𝝅 - Conditions for interference – I.Coherent
angular speed or velocity, 𝝎 =2𝝅𝒇 = , 𝑡=time, sources : sources emmiting waves of equal
𝑻
𝑘=wave constant or wave number, 𝒌 = ,
𝟐𝝅 frequency, wavelength and amplitude in same

phase or with a constant phase difference.
𝑥=displacement of the wave from origin.
𝟐𝝅𝒙 𝟐𝝅 𝟐𝝅𝒙 II.More than one vibrating source at a small
2. 𝒚 = 𝑨 𝒔𝒊𝒏(2𝝅𝒇𝒕 ± ), 𝒚 = 𝑨 𝒔𝒊𝒏( 𝒕 ± ) distance apart i.e close to each other.
 𝑻 
𝟐𝝅𝒙
- Phase difference or angle  = 𝒌𝒙 = . - Constructive interference or reinforcements

𝟐𝝅 𝟐𝝅 occurs when two waves meets in phase or step i.e
3. 𝒚 = 𝑨 𝒔𝒊𝒏 (𝒙 ± 𝒗𝒕) or 𝑨 𝒔𝒊𝒏 (𝒗𝒕 − 𝒙), crests meets with crests and troughs meets with
 
𝝎𝒕 𝒕
4. 𝒚 = 𝑨 𝒔𝒊𝒏( ± 𝝎𝒙) = 𝑨 𝒔𝒊𝒏 𝝎( ± 𝒙) , troughs, and reinforce to produce a larger wave
𝒗 𝒗
𝒕 with large amplitude. Phase difference
𝒚 = 𝑨 𝒔𝒊𝒏 𝟐𝝅𝒇( ± 𝒙) .
𝒗 between the waves is equal to one cycle or

157
Demystified Series Physics Demystified by Dr Timothy
360° or 1 wavelength, 1. dimensional films. III.Polarimeter in determining
- Destructive interference or cancellations the concentration of crystals e.g sugar solution.
occurs when two waves meets out of phase or - Reflection and refraction of light occur at the
step i.e crests meets with troughs to produce a sides of a glass not the front. Reflection and
wave of zero disturbance. Phase difference refraction of light at water or glass surface is
between the waves is equal to half cycle or always accompanied by partial polarization of
180° or half wavelength .
𝟏 light.
𝟐
- Polarization by reflection– Polarization of
- Nodal and antinodal lines are occurrence in
light can also be produced when light is
superimposition of two identical waves
incidented on a polished surface of a glass plate
travelling in the same direction i.e they occur in
at angle of incidence of about 57°. The reflected
interference of waves. Antinodal lines are the
light is said to be polarized. The reflected and
lines joining all points of constructive
refracted rays are at right angles to each
interference while Nodal lines are the lines
other i.e angle between them is 90°, and the
joining all points of distructive interference.
angle of incidence equals the polarizing angle.
- Examples of interference – I.Beats: regular
- The refractive index of the glass surface
rise and fall representing constructive and
producing the polarized light is calculated using
destructive interference. It occurs when two
Brewster’s law. It states that the refractive
sounds of nearly equal frequency are combined.
index of the glass material 𝒏 is equal to the tanget
II.The colours seen in the thin film of oil on the
of the angle of polarization 𝜽𝒑 . Brewster’s
road and in soap bubbles. III.Blooming of
lenses: production of different reflected beams law:𝒏 = 𝒕𝒂𝒏 𝜽𝒑 . 𝑛=refractive index of the glass
from a lens coated with fluoride salt. IV.Circular medium, 𝜃𝑝 =incident or polarizing angle.
colour rings formed when white light falls on a
converging lens of large radius of curvature Examples :
placed on a plane glass. 1. A stone is dropped into the middle of a pool of
- Superposition is the combination of waves of water. Which of the following statements is/are
different frequency. correct? I.Spherical waves are set up in water
5. Polarization or sidedness effect of a wave is II.The water moves outwards to the side of the
the restriction of an incident wave into vibrating pool III.Energy is transmitted outwards from the
in one plane or straight lines parallel to direction centre of disturbance A.I B.II C.III D.I and III E.II
of vibration.It is the production of a wave whose and III.
vibration occur in only in one plane. It is also a A stone dropped into a pool of water produces
procedure for changing incident wave of circular or spherical wave fronts.The water
constant intensity to an emergent wave of waves moves forward and outwards
varying intensity. transmitting energy over a long distance from the
- Polarization is exhibited by transverse centre disturbance.Hence,water waves does not
waves only. necessarily move to the side of the pool.
- Malu’s law of intensity of polarization states 2. Which of the following is not true about
that the transmission of light through a polarizer waves? A.waves transfer energy from one place
placed at an angle to the analyzer varies as the to another B.electromagnetic waves have the
square of the cosine of the angle. 𝑰 = 𝑰𝒐 𝒄𝒐𝒔𝟐 𝜽, same velocity in the same medium C.particles of
𝐼=intensity of incident unpolarised light in a medium vibrate if there is a wave motion in the
candela (cd), 𝐼𝑜 =intensity of emergent polarised medium D.the velocity of a sound wave depends
light in candela (cd), 𝜃=angle between the on the temperature of the air E.all waves
incident light and axis of polaroid. require a material medium for their
- Polarization of light – Polarized light can be propagation.
produced using polarizers [e.g tourmaline Energy is propagated in a travelling wave froom
crystal,quartz or a polaroid], selective absorption one point to another,while the particles vibrate
by crystals, double refraction, scattering through about a mean point without being displaced in
fine particles and by reflection. the medium. Only mechanical waves require
- Polarizers allows light vibrating in only one material medium for their propagation.
particular direction to pass through them and Electromagnetic waves do not require material
absorbs light vibrating in other directions. medium.
- Polaroids are used : I.sunglases,camera The velocity of sound wave is directly proportion
lens,side windows of a vehicle or plane and head to the square root of its absolute temperature ,
lamps to reduce the intensity of incident light and V∝ √𝑇.
reduces reflected light or glare. II.To produce 3- 3. Which of the following is not a mechanical
wave? A.wave produced in a stretched string

158
Demystified Series Physics Demystified by Dr Timothy
B.waves in closed pipes C.water waves D.radio due to the vibration of a air column in the
waves E.sound waves. medium are sound waves i.e longitudinal waves.
Radio waves are electromagnetic waves. 9. Which of the following is correct about light
4. Which of the following statements are correct and sound waves? A.Both of them are
of the production and propagation of waves? associated with energy transfer B.Both of them
I.vibration produces waves II.waves transmit need material media for propagation C.They are
energy III.the medium through which the wave both longitudinal waves D.Their velocities in air
travels does not travel with the wave IV.waves do are equal E.Both of them can be polarized.
not require any medium for transmission A.I and All waves transfer energy as they travel. Light
IV B.II and IV C.III and IV D.I,II and III E.II,III and waves are electromagnetic wave and transverse
IV. wave while sound waves are mechanical and
Vibration of a medium produces waves which longitudinal wave. Their velocity of liqght in air
transmit energy with the particles of the medium is about 3×108ms-1 while that of sound is about
vibrating and oscillating about a fixed point and 333ms-1. Only light wave which is travserse wave
does not travel with the wave. Some waves can be polarized. Sound wav, a longitudinal wave
require material medium i.e mechanical waves cannot be polarized.
while some do not i.e electromagnetic waves. 10. Which of the following is both mechanical
5. A transverse wave and a longitudinal wave and trasverse? A.Infra-red rays B.Gamma rays
travelling in the same direction in a medium C.Sound waves D.Water waves E.Microwaves.
differ essentially in their A.amplitude 11. In a wave, the maximum displacement of
B.direction of vibration of the particles of the particles from their equilibrium position is called
medium C.period of vibration of the particles of A.frequency B.amplitude C.period D.wavelength
the medium D.frequency. E.wave velocity.
A transverse wave travels in a direction 12. The fundamental property of wave which
perpendicular to the direction of vibration of the depends only on the source and not the medium
particles of the medium. A longitudinal wave of propagation is the A.wavelength B.harmonics
travels in a direction parallel to the direction of C.frequency D.velocity.
vibration of the particles of the medium. 13. Any line or section taken through an
6. Which of the following statements is/are advancing wave in which all the particles are in
correct? I.In a transverse wave, the particles of the same phase is called the A.wavelength
the medium vibrate perpendicularly to the B.wavefront C.wavetrough D.wave amplitude
direction of propagation of the wave II.In a E.wavecrest.
longitudinal wave , the particles of the medium Wave front is a line or surface in the path of an
vibrate parallel to the direction of propagation of advancing wave in which all the particles are in
the wave III.In a stationary wave, the particles of the same phase or state of disturbance. Wave
the medium do not vibrate but are stationary A.I length is the distance between two points in
and II B.I and III C.II and III D.III E.I,II and III. phase. Wavecrest is the maximum upward
Stationary or standing waves as the name displacement while wave trough is the maximum
implies, are waves which do not move or are downwards displacement of transverse wave.
stationary though the particles of the of the 14. Which of the following statements about a
medium vibrate about ther mean position. wave is/are correct? I.A wavefront is a line which
7. In which of the following cases is a transverse contains all particles whose vibrations are in
wave formed? I.A stone is thrown into a pool of phase II.The direction of propagation of a wave is
water II.A tuning fork is struck at the end III.A the line drawn parallel to the wavefront III.A
sonometer wire is plucked at the centre A.I B.II wavefront is a circle which is common to all
C.III D.I and II E.I and III. points that are in the same state of disturbance
A stone thrown into a pool of water produces A.I B.II C.III D.I and II E.I and III.
water waves which are transverse wave. A tuning The direction of propagation of a wave is the line
fork struck at the end produces sound waves drawn normal or perpendicular to the wavefront
which are longitudinal waves. A sonometer wire front. Statement ‘III’ describes a circular or
iss made of strings and when plucked at the spherical wave front.
centre produces transverse waves. 15. The sketch below represents a wave profile
8. Which of the following waves are longitudinal
waves? I.ripples on the surface of water II.wave
produced by a tuning fork vibrating in air III.light 𝑃 𝑄 𝑅 𝑆 𝑇
waves IV.wave produced by a flute A.I and II B.I
and III C.II and III D.II and IV E.III and IV. Which points are vibrating in phase? A.𝑃 and 𝑄
Ripples on the surface of water are water waves B.𝑷 and 𝑹 C.𝑃 and 𝑆 D.𝑅 and 𝑆 E.𝑆 and 𝑇.
i.e transverse waves. Wave produced by a flute Two or more points or particloes are said to be in

159
Demystified Series Physics Demystified by Dr Timothy
phase when they occupy similar positions and 20. Two wave trains will produce a standing
travelling in the same direction. Points 𝑃, 𝑅 and 𝑇 wave in a medium if they interfere with A.the
are in phase and their phase difference of phase same frequency, different amplitude and
angle is 0°, as they occupy similar positions and travelling in the same direction B.the same
are travelling in the same direction. Points 𝑄 and frequency, the same amplitude and travelling in
𝑇 are also in phase. the same direction C.the same frequency, the
16. 𝑦 B F same amplitude and travelling in opposite
direction D.different frequencies, the same
amplitude and travfelling in the same direction
A C E G θ E.different frequencies, different amplitudes abd
travelling in the same direction.
D H 21. Which of the following is a characteristic of
The diagram above represents a trasverse stationary waves? A.The antinode is a point of
travelling wave. Which two points are 180° out of minimum displacement B.The distance between
phase? A.A and E B.B and D C.C and F D.D and H. two successive nodes is one wavelength C.They
Two points are out of phase when they are not are formed by two identical waves travelling
occupying similar positions and not travelling in in opposite directions D.They can be trasverse
the same direction. Two points which are 180° or longitudinal.
out of phase have a phase difference of 180° or 𝜋 Antinode in a stationary wave is a point of
1
rad, half wavelength  or half cycle apart and maximum displacement. The distance between
2
two consecutive nodes or antinodes is half
their vertical displacement from equilibrium 1
position are equal in magnitude but opposite in wavelength . Sationary wave can be produced
2
direction. by a transverse or longitudinal wave
Points B and D and points C and F are 180° out of 22. Which of the following statements is/are not
phase. Points A and E and points D and H are in correct about a standing wave? I.It is formed by
phase. the superimposition of two progressive waves
17. I.Wavelength II.Medium of propagation travelling in the same direction II.all points on
III.Wave velocity IV.Frequency V.Energy. Which the wave are vibrating with maximum
of the above are used for characterizing waves? displacement III.it is formed by the superposition
A.I,II and IV B.I,III and V C.I and IV D.I,III and IV of two progressive waves of different amplitudes
Wavelength, wave velocity, frequency, period, travelling in opposite direction A.I B.II C.I and II
amplitude and wavefront are used for D.II and III E.I,II and III.
chnaracterizing waves. Sationary wave is formed by superposition or
18. 𝑌 interference of two identical progressive wave i.e
with the same amplitude, frequency, wavelenbth,
𝑂 𝑋 travelling in opposite direction. Some points are
with minimum disolacement or at rest i.e nodes
The above figure shows the motion of a while some points are with maximum
progressive wave along a string. The particle displacement i.e antinodes.
motion of the medium is in the direction 23. One end of a long wire is fixed while vibrator
A.parallel to 𝑂𝑋 B.parallel to 𝑶𝒀 C.60° to 𝑂𝑋 is attached to the other end. When the vibrator is
D.60° to 𝑂𝑌. energized, the types of waves generated in the
The figure above represents a transverse wave i.e wire are A.stationary and transverse
wave in a string. Transverse waves travel B.progressive and transverse C.stationary and
perpendicular to the direction of vibration of longitudinal D.progressive and longitudinal.
particles of the medium. Hence, the particle 24.
motion is parallel to 𝑂𝑌 and perpendicular or at
90° to 𝑂𝑋.
19. A string is sustaining a stationary wave when The diagram is an example of a A.beat pattern
A.the frequency of the wave is not proportional B.standing wave C.diffraction pattern
to the wavelength of the wave B.the amplitude of D.progressive wave.
vibration is always zero C.two equal waves are 25. Which of the properties of waves is/are
travelling in opposite directions D.the waves common to all waves? I.diffraction II.refraction
are longitudinal E.the frequency is directly III.interference A.I B.III C.I and II D.I,II and III
proportional to the length of the string. All waves exhibit reflection,refraction, diffraction
A stationary wave is produced when two and interference.
identical waves of the same wavelength, 26. Which of the following statements about
amplitude, frequency travelling in opposite electromagnetic waves is not correct? A.They
direction are superimposed. include X-rays B.They cannot be diffracted

160
Demystified Series Physics Demystified by Dr Timothy
C.Infra-red rays are electromagnetic D.They can decreased while the wavelength is unaltered
undergo interference E.Radio waves are C.Both the frequency of the wave and the
electromagnetic waves wavelength are decrease D.The frequency of
All waves can be diffracted or can undergo the wave is unaltered while the wavelength is
diffraction. decreased.
27. Ripples on water are similar to light waves in When a wave passes the boundary between to
that they both A.have the same wavelength media it undergoes refraction. Refraction is
B.longitudinal C.cannot be reflected D.travel at accompanied by a change in the direction of
the same speed E.can be refracted and wavefront, velocity and wavelength while the
diffracted. frequency is constant. Velocity is directly
Ripples on water are water waves. Water waves proportional to wavelength at constant
and light waves are both transverse, they can frequency i.e 𝑣 =𝑓 , 𝑣 ∝ (𝑓=constant). Hence,
both be reflected,refracted amd diffracted. The the wavelength of the wave decreases in the
wavelength of water wave is greater than that of medium as the velocity decreases.
light wave. 36. Which of the following remain unchanged as
28. Which of the following characteristics of light travels from one medium to the other?
wave is used in the measurement of the depth of I.Speed II.Wavelength III.Frequency A.I B.II C.III
the sea? A.Diffraction B.Interference D.I and II E.II and III.
C.Refraction D.Reflection. 37. The phenomenon of light bending round an
29. Transverse waves can be distinguished from obstacle is A.reflection B.polarization
longitudinal waves using the characteristics of C.refraction D.diffraction.
A.diffraction B.polarization C.reflection Diffraction grating is used to measure
D.refraction. wavelengths of light waves and to study the
Only trasverse waves udergoes polarization. structutr and intensity of spectra lines.
30. Vibration in a stretched spring cannot be 38.
polarized because they are A.transverse waves
B.mechanical waves C.stationary D.longitudinal Incident waves Emergent waves
waves
Longitudinal waves cannot be polarized, only Slit
transverse waves can polarized. The wave phenomenon demonstrated in the
31. Which of the following is the exclusive diagram is A.reflection B.deflection
property of a transverse wave? A.diffraction C.diffraction D.refrraction.
B.reflection C.compression D.polarization. The incident waves on passing through the slit
32. Which of the following is not a property of undergoes diffraction and spreads out as an
longitudinal waves? A.compression B.reflection emergent waves.
C.refraction D.polarization E.diffraction. 39. Which of the following is/are necessary for
33. The difference between sound and light the production of interference with two wave
waves is that sound waves A.are transverse trains? I.The sources must be close to each other
while light wave are longitudinal B.require a II.The waves must have the same frequency
medium to travel while light waves do not III.The waves must have the same amplitude
C.can be diffracted but light cannot D.cannot be IV.The waves must travel with the speed of light
reflected but light waves can. A.I and II B.I,II and III C.I,II an d IV D.I,III and IV
Sound waves are longitudinal while light waves E.II,III amd IV.
are transverse. Sound waves are mechanical i.e Intereference occurs between two waves when
require medium while light wave are their source are close to each other and are
electromagnetic i.e do not require medium. Both coherent i.e source that are always in phase or
sound and light waves can be diffracted and constant phase difference,have the same
reflected. amplitude, frequency and wavelength.
34. The change in the direction of a wave front 40. I.Coherence II.Same frequency III.Same
because of a change in the velocity of the wave in wavelength IV.Same intensity. Which of the
another medium is called A.reflection conditions above are necessary to produce
B.refraction C.diffraction D.interference interference fringes? A.I and II B.II and III C.I,II
E.dispersion. and III D.I,II and IV.
35. A wave disturbance travelling in air enters a 41. I.They should be identical B.They should
medum in which its velocity is less than that in originate from the same source III.They should
air. Which of the following statements is true be coherent IV.The should be monochromatic.
about the wave in the medium? A.The frequency From the statements above, the conditions for
of the wave is decreased while the wavelength is two waves ot interfere are A.I,II and III B.I,III
increased B.The frequency of the wave is and IV C.II,III and IV D.I,II,III and IV.

161
Demystified Series Physics Demystified by Dr Timothy
Interference occurs with identical waves i.e same Constructive interference occurs when two
wavelength,amplitude and frequency, same waves meets in phase i.e crest meets crest and
source or different source and the source must be troughs meets troughs, and it results to a higher
coherent. Monochromatic is when a light is of one resultant disturbance or larger amplitude.
colour or wavelength, hence it improves 46. The angle between two successive crests of a
interference in light wave only. wave is A.90° B.180° C.270° D.360°.
42. When two waves are superimposed on each The distance between two successive crest is
other, the occurences are possible, except equal to one wavelength (1) or one cycle which
A.stationary waves B.antinodal lines C.nodal is equivalent to an angular difference of 360° or
lines D.dispersion. 2𝜋 rad.
Superimposition of identical waves travelling in 47. The colours seen in the film of oil on the road
the same direction is interference of waves. and in soap bubbles are due to A.reflection
Nodal and antinodal lines represents destructive B.interference C.diffraction D.polarization.
and constructive interference respectively. Colours seen in the film of oil on the road and in
Dispersion is the separation of white light when soap bubbles are due to interference from the sky
passed through a prism, it does not involve or cloud.
superimposition of waves. 48. Whenever light waves wave are restricted to
43. Two identical waves travelling in the same a specific plane, they are said to be A.diffracted
direction are superimposed. What should be the B.refracted C.regularly reflected D.plane-
phase difference between the waves for polarized.
maximum destructive interference to occur A wave is plane-polarized if its vibration occurs
A.20° B.45° C.180° D.225° E.270°. only in one plane.
Destructive interference occurs for two waves 49. Which of the following are correct of plane-
out pf phase, with a phase difference or phase polarized light? I.Plane polarization of light is the
1
angle of 180° or 𝜋 rad or distance of  or half formation of hydrogen bubbles on the particles of
2
light II.Plane polarization of light is the splitting
cycle. Constructive intefrence occurs for two
of light into its spectra components III.Plane-
waves in phase angle of 360° or 2𝜋 or distance of
polarized light vibrates in one plane only
1 or 1 cycle.
IV.Polarization is characteristic of transverse
44.
waves A.I and II B.I and III C.I and IV D.II and III
String 𝐴 𝐼 𝐴
E.III and IV.
The diagram above illustratestwo waves of equal
50. A narrow beam of unpolarized light of
amplitude 𝐴 and frequencies aproching each
intensity 10.2cd is incident at an angle 30° to the
other. When the two waves meet at a point 𝐼
axis of the polaroid. What is the intensity of the
midways between them, the vertical
emergent polarized ray? A.3.83cd B.7.65cd
displacement of the resulting wave will be A.0
𝐴 𝐴 C.11.48cd D30.6cd.
B. C. D.2𝑨. Intensity of unpolarized light 𝐼𝑜 =10.2cd, angle
2 4
The two waves as shown in the diagram above between the incident light and polaroid axis
will interfere constructively, as both wave are of 𝜃=30°, Intensity of the emergent polarized light
the same frequency and amplitude and starts at ray 𝐼=?, Malu’s law : 𝐼 = 𝐼𝑜 𝑐𝑜𝑠 2 𝜃 ,
the same time. The waves will interfere with the √3
2
3
crest and trough on each wave metting each 𝐼=10.2× 𝑐𝑜𝑠 2 30°=10.2× ( ) =10.2× = 7.65cd.
2 4
other. The vertical displacement of the resulting 51. A parallel beam of unpolarized light is
wave is equal to the sum of their amplitude : incident on a plane glass surface at an angle of
𝐴 + 𝐴 = 2𝐴. 58° to the normal. If the reflected beam is
45. Particle displacement completely polarized, calculate the refractive
𝑃 index of the glass A.0.5 B.0.8 C.1.6 D.3.2.
𝑄 Angle of incidence or polarizing angle 𝜃𝑝 =58°,
refractive index of plane glass surface 𝑛=?,
𝑂
Distance Brewster’s law: 𝑛 = 𝑡𝑎𝑛 𝜃𝑝 = 𝑡𝑎𝑛 58° = 1.6.
52. If the polarizing angle for a given optical
The diagram above shows two wave forms 𝑃 and medium is 49°, calculate the refractive index of
𝑄 at a particular instant of time. The two waves the material A.1.75 B.1.15 C.0.75 D.0.66.
will interfere A.destructively to produce a wave 𝜃𝑝 =49°, 𝑛=?, Brewster’s law: 𝑛 = 𝑡𝑎𝑛 𝜃𝑝 ,
of a larger amplitude B.destructively to produce 𝑛 = 𝑡𝑎𝑛 49° = 1.15.
a wave of smaller amplitude C.constructively to 53.Which of the following statements is not
produce a wave of a larger amplitude correct about sound and light waves? A.The
D.constructively to produce a wave of a small wavelength of light waves is generally shorter
amplitude. than that of sound waves B.The frequency of

162
Demystified Series Physics Demystified by Dr Timothy
sound waves is generally lower than that of light A.0.05ms-1 B.0.20ms-1 C.5.00ms-1 D.50.00ms-1
waves C.Light is visible sound is not D.Sound E.200.00ms-1.
waves causes particles of the medium to vibrate 𝑓=1000Hz, =5.000cm=0.050m, 𝑣=?,
in the direction of propoagation of the waves 𝑣 = 𝑓 = 0.050×1000 = 50.00ms-1.
E.Light waves have the same speed as sound 61. If the wavelength of a wave with a velocity of
waves. 360ms-1 is 60m. The period of the wave is A.6s
54. The distance between successive crest and B.3.6s C.0.17s D.0.6s E.3s
trough of a wave is A.quarter wavelength B.twice  
𝑣=360ms-1, =60m, 𝑇=?, 𝑣 = , 𝑇 = ,
wavelength C.full wavelength D.half wavelength 60
𝑇 𝑣

The distance between two suceesive crests or 𝑇= = 0.166s = 0.17s.


360
troughs is equal to one full wavelength. The 62. A wave travcelling with a speed of 360ms-1
distance between successive crest and trough is has a wavelength of 60cm. The period of the wave
half wavelength. The distance between three in seconds is A.1.5×10-3 B.1.7×10-3 C.6.1×10-3
successive crests or troughs is equal to twice D.1.7×10-2 E.3.6×10-2.

wavelength. 𝑣=360ms-1, =60cm=0.6m, 𝑇=?, 𝑣 = ,
𝑇
55. What is the frequency of the sound made by  0.6
a siren having a discs with 32 holes and making 𝑇= = = 1.66×10-3 =1.7×10-3s.
𝑣 360
25 revolutions per second? A.80Hz B.600Hz 62.𝑦/m
C.800Hz D.1600Hz.
Number of holes 𝑛=32, speed in revolution per 0 0.01 0.02 0.03 𝑡/s
second 𝑣=25rev/s, frequency 𝑓=?,
𝑓 = 𝑛𝑣 = 32×25 = 800Hz. The diagram above illustrates a variation of the
56. A revolving gear wheel has 22 evenly spaced displacement 𝑦 of a wave particle with time 𝑡. If
teeth. A piece of card is held against the teeth the velocity of the wave is 250ms-1, calculate the
when the wheel is rotating at 360 revolutions per distance between two successive particles in
minute. Calculate the frequency of the note phase A.2.0m B.2.5m C.3.0m D.5.0m E.50.0m
emitted A.22Hz B.132Hz C.360Hz C.360Hz 𝑣=250ms-1, time taken to comlete 1 cycle is equal
D.1320Hz E.7920Hz. to the period 𝑇=0.02s, distance between two
Number of teeths 𝑛=22, speed in revolutions per points in phase is equal to the wavelength =?,

second 𝑣=360
𝑟𝑒𝑣
×
1𝑚𝑖𝑛
= 6rev/s, 𝑣 = ,  = 𝑣𝑇 ,  = 250×0.02 = 5.0m.
1𝑚𝑖𝑛 60𝑠 𝑇
𝑓 = 𝑛𝑣 = 22×6 = 132Hz. 64. The distance between two successive troughs
57. of a wave is 0.4m. if the frequency of the source is
2.0 825Hz, calculate the speed of the waves
1.0 A.165ms-1 B.330ms-1 C.412.5ms-1 D.825ms-1.
1.0 2.0 3.0 4.0 t(secs) Distance betweeb two successive troughs =
−1.0 wavelength =0.4m, frequency of source =
−2.0 frequency of wave𝑓=825Hz, 𝑣=?, 𝑣 = 𝑓 ,
The frequency of the wave in the diagram above 𝑣 = 0.4×825 = 330ms-1.
is A.0.25Hz B.0.17Hz C.1.05Hz D.1.57Hz 65. A boat is roked by waves of speed 30ms-1
Time take to complete 1 cycle=period 𝑇=4.0s , whose successive crests are 10m apart. Calculate
1 1
Frequency 𝑓 = = = 0.25s-1 or 0.25Hz. the rate at which the boat receives the waves
𝑇 4.0
58. D/cm A.30s-1 B.10s-1 C.3s-1 D.1s-1.
5 𝑣=30ms-1, distance between successive crest =
0 0.05 0.10 0.15 0.20 0.25 𝑡/s wavelength =10m, rate at which the boat
−5 receives wave is the frequency 𝑓 of the wave=?,
The diagram above represents the displacement 𝑣 = 𝑓 , 𝑓 = =
𝑣 30
= 3s-1 or 3Hz.

D versus time 𝑡, graph of a progressive wave. 10
66. A boat at anchor is rocked by waves whose
Deduce the frequency of the wave A.20Hz
B.10Hz C.5Hz D.4Hz. crests are 100m apart and whose velocity is
Time take to complete 1 cycle=period 𝑇=0.10s 25ms-1. At what interval does the wave crest
Frequency 𝑓 = =
1 1
= 10s-1 or 10Hz. reach the boat? A.0.24s B.4.00s C.75.00s
𝑇 0.10 D.2500.00s.
59. A wave has a frequency of 2Hz and a
Distance between crests =100m, 𝑣=25ms-1,
wavelength of 30cm. The velocity of the wave is
A.60.0ms-1 B.6.9ms-1 C.1.5ms-1D.0.6ms-1 interval at which the creast reaches the boat is
  100
𝑓=2Hz, =30cm=0.3m, 𝑣=?, 𝑣 = 𝑓 period 𝑇=?, 𝑣 = , 𝑇 = = = 4.00s.
𝑇 𝑣 25
𝑣 = 0.3×2 = 0.6ms-1. 67. A radio wave has a wavelength of 150m. If the
60. The velocity of wave having a frequency of velocity of radio waves in free space 3×108ms-1,
1000Hz and a wavelength of 5.000cm is

163
Demystified Series Physics Demystified by Dr Timothy
calculate the frequency of the radio wave The diagram above represents a transverse
A.4.5×1010Hz B.5.0×109Hz C.4.5×109Hz electromagnetic wave travelling with a speed
D.2.0×106Hz. 3.0×108ms-1.What is the frequency of the wave
=150m, 𝑣=𝑐=3×108ms-1, 𝑓=?, 𝑣 = 𝑐 = 𝑓 , A.3.0×107Hz B.9.0×107Hz C.1.0×109Hz
𝑐 3×108 D.3.0×10 Hz .
9
𝑓= = = 2.0×106Hz. 3 complete cyles = 3 wavelengths, 3 = 0.3m,
 150
68. Sixty complete waves pass a particular point =
0.3
= 0.1m, 𝑣=3.0×108ms-1, 𝑓=?,
3
in 4s. If the distance between three successive 𝑣 3×108
troughs of the waves is 15m, calculate the speed 𝑣 = 𝑓 , 𝑓 = = = 3.0×109Hz.
 0.1
of the waves A.300.0ms-1 B.225.0ms-1 73.
C.112.5ms D.75.0ms E.16.0ms-1.
-1 -1

Number of waves 𝑛=60, time taken 𝑡=4s,


𝑛𝑢𝑚𝑏𝑒𝑟 𝑜𝑓 𝑤𝑎𝑣𝑒𝑠 𝑛 60 𝑡 =0.6s
frequency 𝑓 = = = ,. 𝑥 =9m
𝑡𝑖𝑚𝑒 𝑡𝑎𝑘𝑒𝑛 𝑡 4
𝑓=15Hz, distance between three successive The diagram above illustrates a wave form.
troughs is equal to two wavelength. 2=15m, Determine the speed of the wave A.5.4ms-1
=
15
= 7.5m, 𝑣 = 𝑓 = 7.5×15 = 112.5ms-1. B.8.4ms-1 C.9.6ms-1 D.10.0ms-1.
2 From the diagram above, 0.6s is the time taken
69. Water waves are generated by dropping for 2 complete cycles. Time taken to complete 1
stones at regular intervals at a point in a pool of 0.6
cycle= period 𝑇 = = 0.3s. 3 complete
water.The first crest reaches another point 8m 2
cycles = 3 wavelengths, 3 = 9m,
away in 4s.If the distance between two 9  3
successive crests is 0.5m,the frequency of the  = = 3m, 𝑣 = = = 10ms-1.
3 𝑇 0.3
wave is A.2Hz B.4Hz C.8Hz D.16Hz. 74. Determine the distance between the
Distance 𝑠=8m, time 𝑡=4s, distance between two consecutive antinodes 𝑋𝑋. If the wavelength is
successive crest=wave length =0.5m, 𝑓=?, 60cm A.15cm B.30cm C.60cm D.120cm
𝑠 8 2 =60cm, distance between two consecutive
𝑣 = 𝑓 = , 0.5 × 𝑓 = = 2 , 𝑓 = = 4Hz. antinodes 𝐴𝐴 is half the wavelength,
𝑡 4 0.5
70. In a ripple tank experiment, a vibrating plate  60
𝐴𝐴 = 𝑋𝑋 = = = 30cm.
is used to generate ripples in the water. If the 2 2
75. A wave of frequency 10Hz forms a stationary
distance between two successive troughs is wave pattern in a medium where the velocity is
3.5cm and the wave travels a distance of 31.5cm 20cms-1. The distance between adjacent nodes is
in 1.5s, calculate the frequency of the vibrator A.1.0cm B.1.5cm C.2.0cm D.5.0cm
A.3.0Hz B.6.0Hz C.12.0Hz D.27.0Hz E.73.5Hz. 𝑓=10Hz, 𝑣=20cms-1, distance between adjacent
=3.5cm, 𝑠=31.5cm, 𝑡=1.5s, 𝑓=?, 
nodes 𝑁𝑁 is half the wavelength, 𝑁𝑁 = , ∴  =
𝑠 31.5 21 2
𝑣 = 𝑓 = , 3.5 × 𝑓 = = 21, 𝑓 = = 6.0Hz. 2(𝑁𝑁), 𝑣 = 𝑓 =2(𝑁𝑁) × 𝑓
𝑡 1.5 3.5
20
71. 𝑦/cm 20 = 2(𝑁𝑁) ×10, 𝑁𝑁 = = 1.0cm.
2×10

4cm 𝑋 76.
2cm Fixed Fixed
0 end end
𝑌 4m
The diagram above shows two wave forms 𝑋 and The diagram above illustrates a wave set up
𝑌. If the frequency of 𝑋 is 30Hz, what is the between two fixed ends 4.0m apart. If the speed
frequency of 𝑌? A.12Hz B.15Hz C.60Hz D.75Hz of the wave is 1.0ms-1, its wavelength and
The digram shows that wave form 𝑋 and 𝑌 starts frequency respectively are A.1m; 0.25Hz B.2m;
from the same point and completes their 0.25Hz C.2m; 0.50Hz D.4m; 0.50Hz.
oscillation at the same time 𝑡. The diagram above represents a stationary wave.
𝑛𝑢𝑚𝑏𝑒𝑟 𝑜𝑓 𝑐𝑦𝑐𝑙𝑒𝑠/𝑤𝑎𝑣𝑒𝑠 𝑛 The 4m is the distance between 4 nodes i.e 4𝑁𝑁,
Frequency 𝑓 = = , 
𝑡𝑖𝑚𝑒 𝑡𝑎𝑘𝑒𝑛
𝑓𝑋 𝑓𝑌
𝑡 distance between 2 consecutive nodes 𝑁𝑁 = ,
2
𝑓 ∝ 𝑛 (𝑡=constant), = , 𝑓𝑋 =30Hz, 𝑛𝑋 =1  4×2
𝑛𝑋 𝑛𝑌 ∴ 4𝑁𝑁 = 4m, 4 = 4m,  = = 2m,
cycle, 𝑛𝑋 =2½ cycle, 𝑓𝑌 =?, 2
𝑣
4
1
30 𝑓 1 5 𝑣=1.0ms-1, 𝑣 = 𝑓 , 𝑓 = = = 0.5Hz.
= 𝑌1 , 𝑓𝑌 = 30×2 = 30× = 75Hz.  2
1 2
2
2 2 77. The equation of trasverse wave travelling
72. along a string is given by 𝑦=0.3sin (0.5𝑥 −50𝑡)
where 𝑦 and 𝑥 are in cm and 𝑡 is in seconds, find
the maxium displacement of the particles from
0.3m the equilibrium position A.50.0cm B.2.5cm

164
Demystified Series Physics Demystified by Dr Timothy
C.0.5cm D.0.3cm. A.0.25×10-3rads-1 B.0.25×10-1rads-1
The maxium displacement of the particles from C.5.00×10 rads
2 -1 D.2.5×102rads-1.
equilibrium position is the amplitude 𝐴 of the Comparing 𝑦 = 0.25×10 sin(500𝑡 −0.025𝑥)
-3

wave. Comparing 𝑦=0.3sin (0.5𝑥 −50𝑡) with the with the equation 𝑦 = 𝐴 sin(𝜔𝑡 − 𝑘𝑥),
equation 𝑦 = 𝐴 𝑠𝑖𝑛(𝜔𝑡 − 𝑘𝑥), it shows that the Angular frequency or angular velocity or speed
amplitude 𝐴 = 0.3cm. 𝜔 = 500rads-1=5.00×102rads-1.
78. A wave is represented by the equation 83. A plane progressive wave is represented by
𝑦 = 𝐴 sin
2𝜋𝑥
. Which of the following statements the equation 𝑦 = 0.2 sin(200𝜋𝑡 −0.5𝑥), where 𝑥

and 𝑦 are measured in cm and 𝑡 in s. What is the
about the wave is not correct? A.𝐴 represents the
frequency of the wave? A.0.5Hz B.4.0Hz C.10.0Hz
maximum displacement B.= represents the
D.100.0Hz E.200.0Hz.
distance between two consecutive particles
Comparing 𝑦 = 0.2 sin(200𝜋𝑡 −0.5𝑥) with the
which are in phase C.𝑥 is the distance of the 2𝜋𝑥
𝟐𝝅𝒙 equation 𝑦 = 𝐴 𝑠𝑖𝑛(2𝜋𝑓𝑡 − ),
particles from origin D. represents the wave 
 200
number E.The unit of 𝑦 is the same as the unit of 2𝜋𝑓𝑡 = 200𝜋𝑡 , 𝑓 = = 100.0Hz.
2
𝑥. 84. The equation of a certain progressive wave
2𝜋𝑥 𝑡 𝑥
represents the phase difference or phase 𝑦 = 2 sin 2𝜋 ( − ) , where 𝑥 and 𝑦 are in cm
 0.01 30
2𝜋
angle. Wave number or wave constant 𝑘 = . and 𝑡 in seconds. Calculate the period of the wave

79. A travelling wave moving from left to right A.0.001s B.0.010s C.10.000s D.100.000s.
𝑡 𝑥
has an amplitude of 0.15m, a frequency of 550Hz Comparing 𝑦 = 2 sin 2𝜋 ( − ) with the
0.01 30
and a wavelength of 0.01m. The equation equation 𝑦 = 𝐴 𝑠𝑖𝑛(
2𝜋
𝑡−
2𝜋𝑥
),
describing the wave is 𝑇 
𝑡 𝑥 𝑡 𝑡
A.𝒚 = 0.15𝒔𝒊𝒏200𝝅(𝒙 −5.5𝒕) 𝑦 = 𝐴 sin 2𝜋 ( − ) , = , 𝑇 =0.010s.
𝑇  𝑇 0.01
B.𝑦 = 0.15𝑠𝑖𝑛200𝜋(0.01𝑥 −5.5𝑡) 85. The equation of a wave is
C.𝑦 = 0.15𝑠𝑖𝑛5.5𝜋(𝑥 −200𝑡) 𝑦 = 0.005 sin[𝜋(0.5𝑥 −200𝑡)], where 𝑥 and 𝑦 are
D.𝑦 =0.15𝑠𝑖𝑛𝜋(550𝑥 − 0.01𝑡). in meters and 𝑡 is in seconds. What is the velocity
Amplitude 𝐴=0.15m, frequency 𝑓=550Hz, of the wave? A.4000ms-1 B.400ms-1 C.250ms-1
wavelength =0.01m, the wave is moving from D.40ms-1.
left to right i.e positive-x direction, hence the sign Comparing 𝑦 = 0.005 sin[𝜋(0.5𝑥 −200𝑡)] with
is negative. Substituting into the equation: 2𝜋
the equation 𝑦 = 𝐴 𝑠𝑖𝑛 (𝑥 − 𝑣𝑡) ,
2𝜋 
𝑦 = 𝐴 𝑠𝑖𝑛 (𝑥 − 𝑣𝑡), 𝑣 = 𝑓 =0.01×550 , 2𝜋 200 2𝜋𝑥
 𝑣𝑡 = 200𝜋𝑡 , 𝑣 = = 100, = 0.5𝜋𝑥 ,
2𝜋  2 
𝑣= 5.5ms-1, 𝑦 = 0.15 sin (𝑥 −5.5𝑡) , 2𝜋𝑥
0.01 = = 4m, 𝑣 = 100 = 100×4 = 400ms-1.
𝑦 = 0.15 sin 200𝜋(𝑥 −5.5𝑡). 0.5𝜋𝑥

80. A wave has an amplitude of 4.0m, angular 86. The distance between two points 𝑃 and 𝑄,
1 2 along a wave is 0.05m. If the wavelength is 0.10m,
speed 𝜋 rads-1 and phase angle 𝜋 rad. The determin the phase angle between 𝑃 and 𝑄 in
3 3
displacement 𝑦 of the wave particle is given as radians A.0.1𝜋 B.𝜋 C.2𝜋 D.10𝜋.
𝝅 𝜋 2
A.𝒚 = 4𝐬𝐢𝐧 (𝒕 + 2) B. 𝑦 = 4sin (𝑡 + ) C.𝑦 = 𝑥=0.05m, =0.10m, Phase angle =?,
𝟑 3 𝜋 2𝜋𝑥 2𝜋×0.05
𝜋 𝜋
4sin (2𝑡 + 1) D.𝑦 = 4sin (𝑡 − 2) Phase angle = = = 𝜋rad or 180°.
3 3  0.10
Amplitude 𝐴=4.0m,angular speed 𝜔= 𝜋 rads-1
1 87. A progressive wave is represented by
𝜋𝑥
2
3
𝑦 = 10 sin(100𝜋𝑡 − ) , Two layers of the wave
phase angle = 𝜋 rad, Substituting into the 34
3 separated by 153cm have a phase difference of
equation: 𝑦 = 𝐴 sin(𝜃 ± ) = 𝐴 sin(𝜔𝑡 ± ) A.270° B.180° C.90° D.45°
1 2 𝜋
𝑦 = 4 sin ( 𝜋𝑡 ± 𝜋) = 4 sin (𝑡 ± 2). 𝜋𝑥
Compare 𝑦 = 10 sin(100𝜋𝑡 − ) with the
3 3 3 34
𝜋 𝜋𝑥
𝑦 = 4sin (𝑡 + 2). equation 𝑦 = 𝐴 sin(2𝜋𝑓𝑡 ± ) , = ,
3 34
81. The equation of a wave travelling in a 𝑥=153cm,  =
𝜋×153
=4.5𝜋, 𝜋=180°,
horizontal direction is expressed as 34
2𝜋  = 4.5×180 = 810°, =810° −(2×360°)=90°.
𝑦 = 15 sin (60𝑡 − 𝑥). What is the wavelength? 88. A progressive wave equation is represented
5
A.60m B.15m C.5m D.2m 𝜋𝑥
by 𝑦 = 𝐴 sin(150𝜋𝑡 − ). If the phase difference
2𝜋 4
Comparing 𝑦 = 15 sin (60𝑡 − 𝑥) with the is 45°, the value of 𝑥 in the equation is A.1cm
5
2𝜋 2𝜋 2𝜋
equation 𝑦 = 𝐴 𝑠𝑖𝑛 (𝑣𝑡 − 𝑥) , = ,  = 5m. B.2cm C.3cm D.4cm
  5 𝜋𝑥
82. The equation of a wave travelling along the Comparing 𝑦 = 𝐴 sin(150𝜋𝑡 − ) with the
4
𝜋𝑥
positive 𝑥-direction is given by equation 𝑦 = 𝐴 sin(2𝜋𝑓𝑡 ± ) ,  = ,
4
𝑦 = 0.25×10-3sin(500𝑡 −0.025𝑥). Determine the 𝜋𝑥 45°×4
=45°, 𝑥=?, 45° = , 𝑥= , 𝜋=180°,
angular frequency of the wave motion 4 𝜋

165
Demystified Series Physics Demystified by Dr Timothy
45°×4
𝑥= = 1cm.
180°

Jamb past questions on Waves : types,


propagation and properties :
[1978/49,1979/6,36,40,48,1981/4,22,1982/19,
1983/13,49,1984/45,1986/29,34,49,1987/27
,28,1988/35,1989/19,22,31,1991/24,1992/20,
23,1993/22,23,2000/36,2001/17,2002/17,26,
2004/40,2006/34,2007/4,8,2008/26,27,28,
2009/26,2730,2010/24,2011/21,2012/26,27,
2013/25,2013/26,2014/26,2015/37]

166
Demystified Series Physics Demystified by Dr Timothy

Chapter 16 – Sound waves


● Sound waves – Sound waves is produced by moving in the same direction with the sound and
vibrating bodies e.g gutitar strings or struked decreases when the wind is in opposite direction
tuning fork. as the sound.
- Sound wave is a mechanical wave, it requires a 2.Density 𝝆 : speed sound increases as the
material medium for its propagation. Hence, density of the medium decreases, 𝒗 ∝ .
𝟏

sound waves cannot travel through a vacuum. √𝝆

- Themedium through which sound waves - The speed of sound in solid,liquid and gas
travels are solid, liquid and gases. decreases from solid-liquid-gas. Hence, sound
- Sound waves in air comprises of regions with air travels faster through solid than in liquid and
pressure or densities above normal called faster in liquid than in air(gas) i.e 𝒗S > 𝑣L > 𝑣g.
compressions and regions with air pressure or 3.Elasticity of the medium i.e Young modulus
densities below normal called rarefractions. 𝑬 for solids : speed of sound increases as the
- Sound waves are longitudinal waves. elasticity of the medium increases, 𝒗 ∝ √𝑬.
- Sound waves have a longer wavelength than 4.Humidity 𝑯 : speed of sound increases with
light waves. humidity 𝒗 ∝ 𝑯.
- Sound waves and also light waves exhibits 5.Temperature 𝑻 : speed of sound increases as
Doppler effect. the temperature of the medium increases. The
- Doppler effect is the apparent change in the speed of sound is directly proportional to the
frequency/pitch of a source of wave due to the square root of its absolute Kelvin
𝒗𝟏 𝟐 𝒗𝟐 𝟐
relative motion between the source of wave and temperature. 𝒗 ∝ √𝑻,
𝒗𝟏
=
𝒗𝟐
, = .
√𝑻𝟏 √𝑻𝟐 𝑻𝟏 𝑻𝟐
the observer.
- The sudden drop in frequency/pitch of a train - Speed of sound is independent of pitch or
when sounding its whistle and just passes an frequency,air pressure and loudness or
observer, is a common example of Doppler effect. intensity.
- Application of Doppler effect – I.Radar used in - The speed of sound 𝑣 in solids is related to the
large airports for recording the height and Young modulus 𝐸 and density 𝜌 of the solid
𝑬
distance of incoming planes about to land. medium is given by – 𝒗 = √ .
𝝆
II.Police speed traps.
The apparent frequency 𝒇𝟏 in Doppler effect - During thunderstorm, we see the flash of
is given by : lightening before the sound of thunder because
1.When both the source and observer are moving the speed of light(about 3×108ms-1) is far greater
or heading towards each other i.e approaching than the speed of sound( about 340ms-1).
each other in opposite direction, then : - The distance 𝒔 between us and the point of
𝒗+𝒗 generation of the lightning and thunder or the
𝒇𝟏 = ( 𝒐 ) 𝒇. storm-centre is given by : 𝒔 = 𝒗𝒕, 𝑣=speed of
𝒗−𝒗𝒔
2.When both the source and observer are moving sound in air , 𝑡=time interval between the flash of
away from each other i.e chasing or receding in lightning and sound of thunder.
𝒗−𝒗
the same direction, then : 𝒇𝟏 = ( 𝒐) 𝒇. - Sound waves undergo reflection, refraction,
𝒗−𝒗𝒔
diffraction and interference. Sound waves do not
𝑓 1 =new or apparent frequency of source, exhibit polarization.
𝑓=initial frequency of source, 𝑣=speed of sound - Reflection of sound waves – The reflection of
in air, 𝑣𝑠 =speed of source, 𝑣𝑜 =speed of observer. sound waves from a plane surface gives rise to
- When the source is stationary, 𝒗𝒔 = 0. echo. Irregular or rough surfaces will cause
- When the observer is stationary, 𝒗𝒐 = 0. regular reflection of sound waves.
- According to Doppler effect, the frequency - Echo is the sound heard after reflection of
increases when the source moves towards the sound wave from a plane hard surface. It is also
observer and decreases when the source moves called reflected sound.
away from the observer. - Reverberation is the perserverance of sound
- Speed or velocity of sound – Speed of sound 𝑣 due to multiple reflection from a surface i.e
is the ratio of the distance traveled 𝑥 by sound multiple echo with diminishing intensity,after
𝒙
wave to the time taken 𝑡. 𝒗 = = 𝒇 . the source ceases. It is indistinct echo or sound
𝒕
- The speed of sound at 0℃ is about 33ms-1 in air, that cannot be differentiated from the original
1500ms-1 in water and 5000ms-1 in steel. sound.
- Factors affecting the speed or velocity of - A hall where reverberation occurs is said to
sound : have poor acoustical properties.
1.Wind : speed of sound increases with wind - Reverberation is reduced in halls by hanging
coming from the source to the observer or wind curtains round hall or lined with thick felt, having

167
Demystified Series Physics Demystified by Dr Timothy
more openings in the walls or covering boards - Noise is produced by vibrations or sound of
which absorbs the sound quickly, hence irregular frequency or pitch, which is unpleasant
minimize reflection of sound waves. to the ear.
- Application of Echoes – - Characteristics of musical note or sound –
1.Determination of the speed of sound in air – 1.Pitch – Pitch of a note is its position in a
𝟐𝒙
𝒗 = , 𝑣=speed of sound in air, 𝑥=distance musical scale. Pitch depends on frequency of
𝒕
sound i.e a low pitched note has a low frequency
between the source and the reflecting surface
while a high pitched note has high frequency.The
2𝑥=total distance moved by the sound before
human ear hear sound wave of frequency ranging
echo is heard due to reflection, 𝑡=time interval
from 20Hz to 20kHzonly. Sound of frequency
taken for the echo to be heard.
above hearing rang e i.e above 20Hz are called
2.Determination of the depth of a sea-bed.
ultrasonic sound.
3.Detection of submarine.
2.Intensity – Intensity of sound is the rate of flow
4.Exploration of earth for mineral deposits, gas
of energy per second per unit area perpendicular
and oil.
to the direction of propagation of sound wave.
- Refraction of sound waves – Sound waves are 𝑬 𝑷
refracted when they travel from to medium to 𝑰= = . It is measured in watt per square
𝒕×𝑨 𝑨
another of different densities. Sound coming meter (𝑾m-2).
from a distance is heard easily at night than in the - Intensity depends on the square of the
day due to refraction of sound waves towards the amplitude of the vibrating body 𝑰 ∝ 𝑨𝟐 .;
ground by cooler air at night than warmer air at 3.Loudness – Loudness is the magnitude of
day. sensation resulting from a sound reaching the
- Diffraction of sound waves – Sound waves are ear.
diffracted when they travel across an obstacle ot - Loudness depends on intensity𝑳 ∝ 𝑰, mass of
bend e.g distribution of sound round corners in a air which can be set into vibration𝑳 ∝ 𝑴𝒂𝒊𝒓 ,
room through a large opening. and square of the amplitude of the vibrating
- During refraction of sound waves, the frequency body 𝑳 ∝ 𝑨𝟐 .
is constant hence, speed of sound is directly - Intensity and loudness varies inversely with
propotional to its wavelength. Thus, when sound the square of distance 𝒓 from the source
moves from a cold air region to hot air region i.e 𝟏
𝑳 or 𝑰 ∝ 𝟐 .
increase in temperature, speed and wavelength 𝒓
- Amplitude of sound varies inversely with the
increase but frequency remains constant. 𝟏
-When sound waves moves from less dense (air) distance from the source of sound 𝑨 ∝ .
𝒓
to a more dense medium(water), the velocity and 4.Quality or Tone or Timbre – This is the
wavelength increases and vice-versa, but the characteristic note of a musical instrument which
frequency remains constant. differentiates it from another note of the same
- Inteference of sound waves – Superimposit- pitch and loudness produced by another
ion of sound waves gives rise to interference instrument.
effects. Constructive and destructive - A well made tuning fork has a pure tone i.e
interference of sound waves produces loud and single or fundamental frequency or with free
reduced or zero sound respectively. vibration and is not affected by quality.
- Beat is the alterations in loudness of sound - A musical note is a mixture of several
produced when two notes or sound waves of frequencies namely: fundamental frequency,
nearly equal frequencies are superimposed. harmonics and overtones.
Hence, beats are due to interference of two sound - Fundamental frequency 𝒇𝒐 is the lowest,
waves of close frequency. strongest single audible frequency of a vibrating
- Beat frequency 𝒇𝑩 is the number of intense body.
beats(sounds) heard per second. It is equal to the - Harmonics H are frequencies which are
difference in the frequency between the two multiple of the fundamental frequency, which is
𝒏
wave sources. 𝒇𝑩 = . the first harmonic. 1st harmonic = 𝑓𝑜 , 2nd
𝒕
𝒗 𝒗 𝟏 𝟏 harmonic=2𝑓𝑜 , 3rd harmonic =3𝑓𝑜 e.t.c.
𝒇𝑩 = 𝒇𝟏 − 𝒇𝟐 = − = 𝒗( − ). - Overtones are the next higher frequencies or
𝟏 𝟐 𝟏 𝟐
- Beat period 𝑻𝑩 is equal to the reciprocal of the mode of vibration above and multiple of
beat frequency. 𝑻𝑩 = .
𝟏 fundamental quantity. If, 𝑓𝑜 =5Hz, overtones are
𝒇𝑩
20Hz, 30Hz, 40Hz i.e multiple of 𝑓𝑜 . Also, 1st
- Musical note and Noise – Sound can either be overtone=2nd harmonic, 2nd overtone = 3rd
grouped as either musical note or noise. harmonic, 3rd overtone=4th harmonic.
- Musical note is produced by vibrations or - Overtones are quitter than the fundamental
sound of regular frequency or pitch, which is frequency.
pleasant to the ear. - Quality or timbre or tone depends on

168
Demystified Series Physics Demystified by Dr Timothy
number of harmonics and overtones. 𝟏 𝑻 𝒌 𝑻
𝒇 ∝ √ , 𝒇 = √ . 𝑘=constant which depends
- Musical intervals is the ratio of the frequencies 𝑳 𝝁 𝑳 𝝁
of two notes. on the mode of vibration or number of loops.
- Octave is a sound of frequency twice the - When two strings are of the same length,
fundamental frequency. If two musical notes frequency will be proportional to the square
have frequency in the ratio of 2:1, they are root of the mass , 𝒇 ∝
𝟏
.
said to differ by an octave. √𝒎

● Forced vibrations – Forced vibrations are - The frequency 𝑓𝑛 of 𝑛 loops or number of


those vibrations resulting from an external harmonics of a vibrating string is given by –
periodic force acting on a system and setting the 𝒇𝒏 =
𝒏
√𝝁 .
𝑻
𝟐𝑳
system vibrating at the same frequency as the
external periodic force. A louder sound is heard - Fundamental frequency 𝑓0 of a string is given by
due to the large area of surface in contact and the : 𝒇𝟎 =
𝟏 𝑻 𝟏 𝑻
√𝝁 =  √𝝁 = 𝟐𝑳 .
𝒗

large mass of air molecules set into vibration e.g 𝟐𝑳

vibration of a table when a vibrating tuning fork - The velocity or speed of the transverse wave set
is placed in contact with the table top. up along the stretched string is – 𝒗 = √ .
𝑻
- Resonance - Reesonance is a special form of 𝝁

forced vibration at which the frequency of the - Audible sound are sound that can be hearable
forced vibration is the same as or coincides with or sound that can be distinctly separated.
the natural frequency of the body set into Audible sound can be produced can be
vibration. It is characterised by maximum produced with lower frequencies. Any factor
amplitude of vibration and loudness. that lowers frequency, can be used to produce
● Vibrations in strings – When a string fixed at audible sound.
both ends is plucked at the middle, stationary ● Vibrations of pipes –Vibration of air column
transverse wave is produced in the string which in a tube or pipe produces a stationary and
in turn produces progressive longitudinal longitudinal waves along the length of the tube
sound waves in air. or pipe.It can be open or closed pipes.
- The strings can be made to vibrate in differents - The open end of the tube is an antinode (point
modes/loop which gives different wave forms of maximum displacement) while the closed end
with nodes 𝑁 and antinodes 𝐴 – or water surface is a node(point of zero or
𝑵 𝑨 𝑵 𝑵 𝑨 𝑵 𝑨 𝑵 minimum displacement).
1st position or loop 2nd position or loop - Open pipes: Pipes with both ends opened. Both
- The first and simplest mode of vibration is the open ends of an open pipe is an antinode. It has
fundamental frequency or first harmonic. Other all multiple of frequencies or harmonics just like
higher mode of vibration are all multiples of the a vibrating string hence, 1st harmonic=0
fundamental frequency or first harmonic. 1st overtone=𝑓𝑜 , 2nd harmonic =1st overtone=2𝑓𝑜 ,
harmonic=1 loop=𝑓𝑜 , 2nd harmonic=2 loops=1st 3rd harmonic=2nd overtone=3𝑓𝑜 , 4th
overtone=2𝑓𝑜 , 3rd harmonic=3loops=2nd harmonic=3rd overtone= 4𝑓𝑜 e.t.c.
overtone=3𝑓𝑜 e.t.c. - Frequency 𝑓𝑛 of 𝑛th overtone of open pipeis
𝒗
- Wavelenght 𝑛 of a 𝑛 harmonics or loops of a given by: 𝒇𝒏 = (𝒏𝒐 + 𝟏) , 𝑛𝑜 =number of
𝟐𝑳
𝟐𝑳
vibrating string is given by : 𝒏 = , 𝑛=number overtones, 𝐿=length of air column or length of
𝒏
of harmonics, 𝐿=length of string. pipe.
- Factors affecting the frequency of vibrating -Wavelength 𝑛 of 𝑛 harmonics of an open pipe is
𝟐𝑳
strings/a sonometer – given by : 𝒏 = , 𝑛=number of harmonics.
𝒏
1.Length 𝑳 : Frequency varies inversely with the - Close pipes: Pipes with one end closed and the
length of the string hence, frequency decreases as other end opened.The closed end is a node while
𝟏
length increases 𝒇 ∝ . the open end is an antinode. It has odd multiple
𝑳
2.Tension or force on string 𝑻 : Frequency of frequencies or harmonics only, 1st
varies directly with the square root of tension harmonic=0 overtone=𝑓𝑜 , 3rd harmonic=1st
hence, frequency increases when tension overtone=3𝑓𝑜 , 5th harmonic= 2nd overtone=5𝑓𝑜 ,
increases 𝒇 ∝ √𝑻 or 𝒇𝟐 ∝ 𝑻. 7th harmonic=3rd overtone=7𝑓𝑜 e.t.c.
3.Linear density or mass per unit length or - The frequency of the longitudinal wave in
thickness 𝝁 : Frequency varies inversely with the air column increases with the length of
the square root of linear density or mass per unit the air column.
𝟏 𝒎 𝟏 - Frequency 𝑓𝑛 of 𝑛th overtone of close pipe, is
length 𝒇 ∝ , 𝝁= , 𝒇∝ . 𝒗
√𝝁 𝑳 √𝒎⁄𝑳 given by: 𝒇𝒏 = (𝟐𝒏𝒐 + 𝟏) , 𝑛𝑜 =number of
𝟒𝑳
- Combining these relationships it gives – overtones, 𝐿=length of air column or length of
pipe.

169
Demystified Series Physics Demystified by Dr Timothy
- Wavelength 𝑛 of 𝑛 harmonics of closed pipe is can only be produced by vibrating objects.
𝟒𝑳
given by : 𝒏 = , 𝑛=number of odd harmonics. Sound wave is a mechanical and longitudinal
𝒏
wave. It cannot travel through a vacuum. Sound
- The strength of resonance in both open and
waves are produced by vibrating body. Sound
close, decreases as the length of the air column
𝟏 waves undergo reflection and refraction. Speed
increases, 𝑹 ∝ . Thus , the first position of of sound in solids e.g steel is greater that its speed
𝑳𝒂𝒊𝒓
resonance is stronger and louder than the in gases or air.
second position. 2. The sound of an electric bell dies down slowly
- Resonance tube experiment: A resonance when air is slowly pumped out from a bottle
tube (a closed pipe) is filled with water and the becasuse A.sound cannot pass through the
volume or length of air in the tube can be varied centre B.sound needs a material medium
lowering or raising the water level in the tube, C.sound can pass through a vacuum D.the
𝒗𝒂𝒊𝒓 or 𝑳𝒂𝒊𝒓 ∝
𝟏
. A tuning fork of frequency 𝑓, wavelength of sound becomes greater in the
𝑳𝒘𝒂𝒕𝒆𝒓
bottle.
which vibrates at the top of the tube until a loud Pumping air out of thr bottle creates a vacuum in
sound is heard,indicating the first position of it and sound waves cannot travel through a

resonance (fundamental note) 𝑳𝟏 = . Further vacuum hence, the sound dies down.
𝟒
increase in the length of air column by lowering 3. The sound from a bell in an enclosed jar
the water level produces another loud gradually faints away while the jar is being
sound,indicating the second position of evacuated. Which of the following explains this
resonance (first overtone), observation? A.the pressure is reduced B.the
𝟑
𝑳𝟐 = . 𝐿1 and 𝐿2 are the length of air column at sound waves are pumped out C.the waves are
𝟒 absorbed in the inner walls D.there is no more
the first and second resonanace position material medium E.the air is disturbed.
respectively. 4. Thunder is usually heard some seconds after
- The shortest air column which produces lightning is seen because A.the human eye is
resonance is equal to the first resonance more sensitive to light than the ear to sound

length , 𝑳 = . B.sound and light travel in different media
𝟒
- The difference of both position is used to C.thunder occurs after lightning D.sound travels
determine the wavelength  -  =2(𝑳𝟐 − 𝑳𝟏 ). more slowly than light E.sound travels in the
- The speed of sound in air from resonance tube form of waves while light dies not.
experiment is given by – 𝒗 =2𝒇(𝑳𝟐 − 𝑳𝟏 ). The speed of light in air is greater than that of
- End correction or end effect 𝒆 is the small sound in air, hence lightning(a form of light) is
distance at which the antinode extends beyond seen first before thunder(a form of sound) is
the edge of the tube. End correction is significat heard.
only in closed pipes(resonance tube) as it cancels 5. Which of the following media allow the
out at both ends of an open pipe. It is determined transmission of sound waves through them? I.air
by – 2𝒆 = 𝑳𝟐 −3𝑳𝟏 . II.liquid III.solids A.I and II B.I and III C.II and III
NB – The fundamental note of an open pipe is D.I,II and III.
an octave higher than that of a closed pipe of Sound waves can travel through solids, liquids
the same length. and gases(air). Sound waves travels faster in
- Musical instruments – They are classified as: solids, than in liquids and faster in liquids than in
i.Wind instrument – Air is vibrated in wind gases(air).
instrument e.g flute,organ pipe,trumpet, 6. The speed of sound travelling in various media
saxophone,whistle,clarinet trombone and bugle. increases in the order : A.air,mercury, water,iron
ii.Precussion instrument – A hard body is bar B.air, water, iron, mercury C.air, water,
vibrated against another body in percussion mercury, iron bar D.water, mercury, air, iron
instrument e.g bell,cymbal,drum, tuning fork and bar E.water, mercury, iron bar, air.
xylophone. The increasing order of the speed of sound is :
iii.String instrument – String is vibrated in gases–liquids–solid, 𝑣 g< 𝑣L< 𝑣S. Hence, speed of
string instrument e.g piano,harp,guitar,violin sound is greatest in solids.
and sonometer. 7. Two boys are communicating with each other
by steetching a string passing through a hole
Examples : punched in the bottom of each of two time cans.
1. Which of the following statements about sound The physical principle employed is that sound
is not true? A.Sound is a longitudinal wave travels A.mainly through air B.fainter in
B.Sound can be reflected and refracted like light stretched string C.faster through gases than in
C.Sound travels through a vacuum D.Sound solids and liquids D.with greater ease through a
moves faster in a steel wire than in air E.Sound string than in air E.none of the above is correct.

170
Demystified Series Physics Demystified by Dr Timothy
Sound travels faster with greater ease through a its absolute temperature is A.doubled B.halved
string (a solid) than in air (gas). C.constant D.quadrupled.
8. If a source of sound is moving, a stationary Velocity of sound is directly proportional to the
listener will hear a sound of different frequency. square of root of its absolute temperature 𝑣 ∝
This is called A.ultra-sound B.resonance √𝑇 . When 𝑣 is doubled then, 2 = √𝑇 , √𝑇 = 2,
C.Doppler effect D.rarefraction E.diffraction. 𝑇 = 22 = 4 i.e four times the original value or
Doppler effect is the apparent change in the quadrupled.
frequency of sound form a source when there is 15. The velocity of sound in air at 15℃ is 340ms-
relative motion between the source and an 1. Calculate the velocity at 47℃ A.790ms-1
observer. B.602ms C.358ms D.322ms .
-1 -1 -1
9. 𝑿 𝒀 𝒁 𝑇1 =15℃=15+273=288𝐾, 𝑣1 =340ms-1,
Suppose a car, travelling at a uniform speed, 𝑇2 =47℃=47+273=320𝐾, 𝑣2 =?,
sounds its horn at point 𝑌 while towards 𝑋 in the 𝑣1 𝑣2 𝑣1 2 𝑣2 2
𝑣 ∝ √𝑇 , = , = ,
above figure. Assuming there is no wind, which of √𝑇1 √𝑇2 𝑇1 𝑇2
the following observationas is correct? A.The 3402 𝑣2 2 340×340×320
= , 𝑣2 =2
= 128444.44,
man at point 𝑋 will receive a lower pitch B.The 288 320 288

driver at point 𝑌 will receive a lower pitch C.The 𝑣2 = √128444.44 = 358.39 = 358ms-1.
man at point 𝒁 will receive a lower pitch D.The 16. The velocity of sound in air of 16℃ is 340ms-
1. What will it be when the pressure is doubled
pitch will be the same at points 𝑋 and 𝑍 E.None of
the above. and its temperature rises to 127℃. A.4.000ms-1
According to Doppler effect, as relative motion B.400ms-1 C.160,000ms-1 D.8,000ms-1.
exists between the car i.e the source of sound, Velocity of sound is independent of pressure
and the man, there will be a change in frequency. hence, doubling the pressure will not affect the
The frequency decreases as the car is moving velocity.
away from the man. Hence, the frequency at point 𝑇1 =16℃=16+273=289𝐾, 𝑣1 =340ms-1,
𝑍 will be lower. The pitch at point 𝑍 is also 𝑇2 =127℃=127+273=400𝐾, 𝑣2 =?,
𝑣1 𝑣2 𝑣1 2 𝑣2 2 3402 𝑣2 2
lowered as pitch depends on frequency. 𝑣 ∝ √𝑇 , = , = , = ,
√𝑇1 √𝑇2 𝑇1 𝑇2 289 400
10. I.Temperature II.Density of air molecules 340×340×400
III.Pressure IV.Pitch. Which of the above will 𝑣2 2 = = 160000,
289
affect the velocity of sound in air? A.I and II B.II 𝑣2 = √160000 = 400ms-1.
and IV C.I,II and IV D.I,II,III and IV. 17. The velocity of sound wave at 27℃ is 360ms-
Speed or velocity of sound in air is affected by 1. Its velocity at 127℃ is A.120√3ms-1 B.240ms-1

temperature, density of air molecules, wind and C.240√𝟑ms-1 D.720√3ms-1.


elasticity. Pressure and pitch will not affect speed 𝑇1 =27℃=27+273=300𝐾, 𝑣1 =360ms-1,
of sound. 𝑇2 =127℃=127+273=400𝐾, 𝑣2 =?,
11. Which of the following factors affects speed 𝑣1 𝑣2 𝑣1 2 𝑣2 2 3602 𝑣2 2
𝑣 ∝ √𝑇 , = , = , = ,
of sound in air? I.temperature II.pressure √𝑇1 √𝑇2 𝑇1 𝑇2 300 400
III.frequency A.I B.II C.I and II D.II and III 𝑣2 2 =
360×360×400
= 172800,
300
Pressure and frequency will not affect the speed
𝑣2 = √172800 = 240√3ms-1.
of sound.
18. Given that Young’s modulus for aluminium is
12. Which of the following statements is not
7.0×1010𝑁m-2 and density is 2.7×103kgm-3, find
correct? A.The velocity of sound in air
the speed of the sound produced if a solid bar is
deceases with increase in temperature
struck at one end with a hammer A.5.1×103ms-1
B.Pressure has no effect on the velocity of sound
B.4.2×103ms-1 C.3.6×103ms-1 D.2.8×103ms-1.
in air C.Sound waves require a material medium
Young’s modulus 𝐸=7.0×1010𝑁m-2, density
for transmission D.The velocity of sound in water
𝐸
is greater than that in air E.Sound waves have 𝜌=2.7×103kgm-3, speed of sound 𝑣=?, 𝑣 = √ ,
𝜌
longer wavelengths than light.
The velocity of aouns in air increase with 𝑣=√
7.0×1010
= √2.59 × 107 = 5.1×103ms-1.
2.7×103
increase in temperature, 𝑣 ∝ √𝑇 .
13. Which of the following factors will affect the 19. In a sound wave in air the adjacent
velocity of sound? A.an increase in the pitch of rarefractions and compressions are separated by
the sound B.an increase in the loudness of the a distance of 17cm. If the velocity of sound wave
sound C.wind travelling in the same direction is 340ms-1 determine the frequency? A.10Hz
of the sound D.change in the atmospheric B.20Hz C.1000Hz D.5780Hz.
pressure at constant temperature The distance between adjacent compressions
Velocity of sound increases with wind travelling and rarefractions or adjacent crests and troughs
in the same direction of the sound. is equal to half wavelength.

14. The velocity of sound in air will be doubled if =17cm,  = 17×2=34cm=0.34cm, 𝑣=340ms-1,
2

171
Demystified Series Physics Demystified by Dr Timothy
𝑣 = 𝑓 , 𝑓 = =
𝑣 340
= 1000Hz. 27. Which of the following characteristics of a
 0.34
wave is used in the measurement of the depth of
20. A source of sound produces waves in air of
the sea? A.diffraction B.intefernce C.refraction
wavelength 1.65m. If the speed of sound in air is
D.reflection.
330ms-1, the period of vibration in second is
The depth of a sea can be determined by echo,
A.200s B.2.0s C.0.5s D.0.02s E.0.005s
  which is reflected sound waves.
=1.65m, 𝑣=330ms-1, 𝑇=?, 𝑣 = , 𝑇 = , 28. Which of the following statements about
𝑇 𝑣
1.65
𝑇= = 0.005s. echoes is not correct? They can be used A.in
330
21. Calculate the wavelength of a note which is underwater exploration for oil B.to measure the
one octave lower than a note of 512.0Hz in a speed of sound in air C.to differentiate between
certain medium in which the speed of sound is transverse and longitudinal waves D.to locate
384.0ms-1 A.0.375m B.0.670m C.1.500m shoals in fish E.to determine the depth of the sea
D.2.670m E.6.000m. bed.
512 Polarization is used to differentiate between
Frequency of the note 𝑓= = 256Hz i.e one
2 transverse and longitudinal waves not echoes.
octave lower than 512.0Hz, 𝑣=384.0ms-1, =?, 29. Which onr of the following is not a
𝑣 384
𝑣 = 𝑓 ,  = = = 1.500Hz. characteristic of a transverse wave? A.Pitch
𝑓 256
22. The speed of sound in air is 330ms-1. How far B.Reflection C.Refraction D.Interference
from the centre of a storm is an observer who E.Wavelength.
hears a thunderclap 2s after lightning flash? Pitch is perculiar to sound waves which are
(Neglect the time taken by light to travel to the longitudinal waves.
observer) A.1320m B.660m C.600m D.330m 30. The pheneomenon that makes sound persist
E.165m. when its source has been removed is known as
𝑣=330ms-1, time interval between lightning flash A.rarefraction B.acoustic vibration
and thunderclap 𝑡=2s, distance of the observer C.reverberation D.echo.
from centre of stor 𝑠=?, 𝑠 = 𝑣𝑡 = 330×2 = 660m. Reverberation is the persistence of sound waves
23. The speed of sound in air at sea-level is due to multiple echoes or reflection of sound.
340ms-1 while that of light is 300,000kms-1. How 31. During a thunderstorm, the sound is heard
far(to the nearest metre) from the centre of a over a long time. This phenomenon is referred to
thunder storm is an observer who hears a as A.refraction of sound B.reverberation
thunder 2s after a lighning flash? A.170m C.superposition D.diffraction of sound.
B.340m C.600m D.680m. 32. In large telecommunications auditorium,
𝑣=340ms-1, 𝑡=2s, 𝑠=?, 𝑠 = 𝑣𝑡 =340×2 = 680m. perforated absorbent materials are used to line
24. The physical property of sound waves can the ceiling so as to A.reduce the reverberation
best be described by A.Reflection and of sound in the hall B.reduce the height of the
diffraction B.Polarization and reflection ceiling from the floor C.increase the
C.Polarization and diffraction D.Polarization and reverberation of sound in the hall D.increase the
refraction. amount of echo in the hall.
Sound waves are longitudinal waves, hence The auditorium is perforated to absorbs sound
cannot be polarized. and reduce reflection of sound, thus reducing
25. Which of the following sets of characteristics reverberation.
is/are associated with sound waves? 33. The walls of a sound-recording studio are
A.Reflection,refraction and resonance usually lined with thick felt so that the sound
B.Refraction,interference and resonance waves can be A.evenly distributed B.easily
C.Interference,diffraction and resonance reflected C.absorbed to prevent reflection
D.Diffraction,reflection and resosnace E.All of D.reflected to prevent absorption E.rasily
the above. refracted in the whole room.
26. Which of these statements is/are correct? 34. A gun is fired in a ship some distance from a
I.Echoes are produced by the reflection of sound cliff. The echo is heard 0.500s later in the
on a hard surface II.Refraction of a wave is caused ship..What is the distance of the ship from the
by a change in the velocity of the wave cliff? (Velocity of sound in air=330ms-1) A.66.0m
III.Diffraction can be observed when a wave B.82.5m C.165.0m D.329.5m E.330.5m.
passed through a large slit A.I B.II C.III D.I and II 𝑡=0.500s, 𝑣=330ms-1, 𝑥=?, for echo or reflection
2𝑥 𝑣𝑡 330×0.5
E.II and III . of wave, 𝑣 = , 𝑥= , 𝑥= = 82.5m.
𝑡 2 2
Echoes are produced when sound waves are 35. A herdsman yelling out to fellow herdsman
reflected by a plane surface. Refraction of wave is heard his voice reflected by a cliff 4s later. What
caused by a change in direction of wavefront, is the velocity of sound in air if the cliff is 680m
velocity and wavelength. Diffraction will not away? A.170ms-1 B.136ms-1 C.340ms-1 C.680ms-
occur when a wave is passed thorugh a large slit. 1 E.510ms-1.

172
Demystified Series Physics Demystified by Dr Timothy
𝑡=4s, 𝑥=680m, 𝑣=?, 𝑣 = , 𝑣 =
2𝑥
= 340ms-1.
2×680
that instant (speed of sound in air= 340ms-1)
𝑡 4
A.0.119m B.0.238m C.0.476m D.0.952m
36. In order to find depth of the sea. A ship sends
The beep heard in car 𝐵 is the echo of the
out a sound wave and receives an ehco after one
ultrasonic sound of car 𝐵 after reflection by car
second. If the velocity of sound in water is
𝐴. 𝑡=5.6×10-3s, 𝑣=340ms-1, distance between the
1500ms-1, what is the depth of the sea? 2𝑥 𝑣𝑡
A.0.75km B.1.50km C.2.20km D.3.00km two cars 𝑥=?, 𝑣 = , 𝑥 = ,
𝑡 2
E.3.75km. 𝑥=
340×5.6×10−3
= 0.952m.
2𝑥 𝑣𝑡 2
𝑡=1s, 𝑣=1500ms-1, 𝑥=?, 𝑣 = , 𝑥 = , 43. A ship travelling towards a cliff receives the
𝑡 2
1500×1
𝑥= = 750m = 0.75km. echo of its whistle after 3.5s. A short while later,
2
37. The human ear can only distinguish between it receives the echo after 2.5s. if the speed of
sound if they occur at least 0.10s apart. If the sound in air under the prevailing conditions is
velocity of sound in air is 340ms-1, calculate the 250ms-1, how much closer is the ship to the cliff?
least distance from a reflecting surface at which a A.10m B.125m C.175m D.350m E.1000m.
listener can detect an echo A.3400m B.170m A B cliff 𝑥1 =initial distance of the
C.68m D.34m E.17m. ship from the cliff,
2𝑥 𝑣𝑡 𝑥2 𝑥2 =final distance of the
𝑡=0.10s, 𝑣=340ms-1, 𝑥=?, 𝑣 = , 𝑥 = ,
𝑡 2 𝑥1 ship from the cliff ,
340×0.10
𝑥= = 17m. 𝑡1 =3.5s, 𝑡2 =2.5s, 𝑣=250ms-1, “how much closer is
2
38. A fathometer is used to send a wave down to the ship to the cliff” is the difference between the
the sea bed. The reflected wave is received initial and final distance of the ship from the cliff,
𝑣𝑡 𝑣𝑡
0.5seconds. Calculate the depth of the sea (Speed 𝑥1 = 1 , 𝑥2 = 2 , The ship is closer to the cliff
2 2
of sound in water=1500ms-1) A.375m B.390m 𝑣𝑡1 𝑣𝑡2 𝑣
by – 𝑥 = 𝑥1 − 𝑥2 , 𝑥 = − = (𝑡1 − 𝑡2 ) ,
C.400m D.450m E.500m. 250
2 2 2
2𝑥
𝑡=0.5s, 𝑣=15000ms-1, 𝑣 = , 𝑥 = ,
𝑣𝑡 𝑥= (3.5−2.5) = 125×1 = 125m.
2
𝑡 2
1500×0.5 44. A sound pulse sent vertically downwards into
𝑥= = 375m.
2 the earth is reflected from two different layers of
39. A sound wave is produced from a source and the earth such that echoes are heard after 1.2s
an echo is heard 𝑡 seconds after wards. If 𝑑 ia the and 1.4s. Assuming the speed of the pulse is
distance of the reflecting surface from the source, 2000ms-1, calculate the distance between the
𝑣 the speed,  the wavelength and 𝑇 the period of layers A.200m B.400m C.2400m D.2600m
2𝑡 𝒕 2𝑇
the wave then A.𝑑 = B.𝒅 = C.𝑑 = E.2800m.
 𝟐𝑻 𝑡
𝑇 layer A pulse layer B
D.𝑑 = .
2𝑡 𝑥𝐴 𝑥𝐵
2𝑥  2𝑑  𝑡
𝑣 = = , 𝑥=𝑑 , = , 𝑑= . 𝑥
𝑡 𝑇 𝑡 𝑇 2𝑇
40. A man clapping his hands at regular intervals distance between the layer A and the pulse 𝑥𝐴 ,
observes that the echo of a clap coincides with time taken to hear echo after reflection at layer A
the next clap. If the reflecting cliff is 160m away 𝑡𝐴 =1.2s, distance between the layer B and the
and the speed of sound is 320ms-1, what is the pulse 𝑥𝐵 , time taken to hear echo after relfection
frequency of the clapping? A.1Hz B.2Hz C.4Hz at layer B 𝑡𝐵 =1.4s, distance between layer A and
D.8Hz B 𝑥=𝑥𝐴 + 𝑥𝐵 =?, 𝑣=2000ms-1,
𝑣𝑡 2000×1.2
As the echo of a clap coincides with the next clap, 𝑥𝐴 = 𝐴 = = 1200m,
2 2
the time taken is equal to the period. 𝑣𝑡 2000×1.4
2𝑥 2𝑥 𝑥𝐵 = 𝐵 = = 1400m, 𝑥 = 𝑥𝐴 + 𝑥𝐵 ,
𝑥=160m, 𝑣=320ms-1, 𝑣 = = , 𝑡 = 𝑇 , 2 2
𝑡 𝑇 𝑥 = 1200+1400 = 2600m. Alternatively ,
2𝑥 1 1 2𝑥 𝑣 320
𝑇= , 𝑇= , = , 𝑓= = = 1Hz. Total time taken for pulse to travel between both
𝑣 𝑓 𝑓 𝑣 2𝑥 2×160 𝑣𝑡
41. A man hears his echo from a nearby hill 2s layers 𝑡=𝑡𝐴 + 𝑡𝐵 =1.2+1.4=2.6s, 𝑥 = ,
2
after he shouted. If the frequency of his voice is 𝑥=
2000×2.6
= 2600m.
260Hz and the wavelength of his voice is 1.29m, 2
45. A hunter 412.5m away from a cliff, moves a
how far is the hill? A.330.0m B.335.4m C.660.0m
distance 𝑥 towards the cliff and fires a gun. He
D.670.8m.
2𝑥 hears the echo from the cliff after 2.2seconds.
𝑡=2s, 𝑓=260Hz, =1.29m, 𝑥=?, 𝑣 = = 𝑓 , Calculate the value of 𝑥 (speed of sound in air
𝑡
𝑓𝑡 1.29×260×2
𝑥= = = 335.4m. 330ms-1) A.49.5m B.99.0m C.363.0m D.412.5m
2 2
42. A car 𝐵 moves towards a stationary car 𝐴. If Cliff 𝑦=new distance btw the
𝐵 produces an ultrasonic sound at a point and it 𝑥 𝑦 hunter and cliff.
takes 5.6×10-3s for a beep to be heard in 𝐵, 412.5m 𝑡=2.2s, 𝑣=330ms-1,
𝑣𝑡 330×2.2
calculate the distance between the two cars at 𝑦= = = 363m, 𝑥 + 𝑦 = 412.5m,
2 2
𝑥 = 412.5−𝑦 = 412.5−363 = 49.5m.
173
Demystified Series Physics Demystified by Dr Timothy
46. A man standing between two cliffs, claps his temperature i.e 20℃. Wavelength increases with
hands and hears two echoes after 3.6s and 4.2s the velocity 𝑣 ∝ , hence wavelength will also
respectively. If the speed of sound in air is increase with temperature  ∝ √𝑇. Frequency is
330ms-1 calculate the distance between the cliffs constant during refraction, as it crosses the
A.594m B.693m C.1287m D.2574m. boundary speparating two media. All waves
cliff A cliff B including sound waves can be reflected.
𝑥𝐴 𝑥𝐵 50. If a sound wave goes from a cold-air region to
𝑥 a hot air region, its wavelength A.increase
𝑡𝐴 =3.6s, 𝑡𝐵 =4.2s, 𝑣=330ms-1, 𝑥=?, B.decrease C.decrease then increase D.remain
𝑣𝑡 330×3.6
𝑥𝐴 = 𝐴 = = 594m, constant .
2 2
𝑣𝑡
𝑥𝐵 = 𝐵 =
330×4.2
= 693m, 𝑥 = 𝑥𝐴 + 𝑥𝐵 , Wavelength increases with temperature  ∝ √𝑇,
2 2 hence when sound waves goes from cold-air
𝑥 = 594+693 = 1287m. Alternatively , region to hot-air region i.e increase in
𝑣𝑡
𝑡=𝑡𝐴 + 𝑡𝐵 =3.6+4.2=7.8s , 𝑥 = , temperature, its wavelength increases.
2
𝑥=
330×7.8
= 1287m. 51. The main difference between a musical note
2
and a noise is that A.the amplitude of the noise is
47. A man stands one-third of the way between
geater than that of the musical note B.the
two walls and claps his hands.He heard two
frequency of musical note is regular while
distinct echoes 1s apart.If the speed of sound is
that of noise is irregular C.both the amplitude
330ms-1,the distance between the walls is A.33m
and the frequency of noise is greater than those
B.110m C.220m D.330m E.495m.
of a musical noise D.the frequency of noise is
The two distinct echoes heard are due to a
𝑥 2𝑥 higher than that of a musical note E.the
distance of from one wall and from the other wavelength of a musical note is irregular while
3 3
wall. that of a noise is regular.
Wall A Wall B Musical note is a combination of sound of regular
𝑥 2𝑥
frequency while noise is a combination of sound
3 3
𝑥 of irregular frequency.
𝑥 𝑥 330×𝑡1 2𝑥 52. Which of the following is/are characteristic
For distance ; = , 𝑡1 = ,
3 3 2 990 of sound? I.pitch II.loudness III.quality IV.noise
2𝑥 2𝑥 330×𝑡2 4𝑥
For distance ; = , 𝑡2 = , A.I B.II C.I and II D.I,II and III E.I,II,III and IV.
3 3 2 990
Time interval between echoes= 𝑡2 − 𝑡1 =1s , Pitch,intensity,loudness and quality are all
4𝑥

2𝑥
= 1,
4𝑥−2𝑥
= 1,
2𝑥
= 1, 𝑥 =
990
, characteristics of sound.
990 990 990 990 2 53. The pitch of a sound note depends on
𝑥 = 495m. A.timbre B.harmonics C.quality D.frequency
48. When sound wave passes from air into water Pitch of a note depends on frequency.
its A.speed and frequency increase but its 54. The pitch of an acoustic device can be
wavelength remains constant B.speed and increased by A.increasing the frequency
wavelength increase but its frequency B.increasing the amplitude C.decreasing the
remains the same C.speed decreases D.speed loudness D.decreasing the intensity
remains the same but its frequency and Pitch of a note increases with frequency.
wavelength change E.speed increases but its 55.What changes(s) is/are observed in the sound
frequency and wavelength decrease produced by a siren when the speed of revolution
Speed of sound in water is greater than its speed is increased? I.The pitch becomes higher II.The
in air, hence when sound waves passes air into wavelength increases III.The velocity remains
water, its speed increases. During refraction constant A.I B.II C.III D.I and II E.I and III .
frequency is constant and speed is directly The frequency of a disc siren 𝑓 = 𝑛𝑣, 𝑣 is the
proportional to wavelength, 𝑣 ∝ . Hence, the speed in revolutions per second, hence the
wavelength also increases. frequency is directly proportional to the speed of
49. Which of the following is true of sound? revolution 𝑓 ∝ 𝑣. Increasing the speed of
A.sound travel faster in air at 20℃ than at 30℃ revolution increases the frequency and thus, the
B.the frequency of a given sound wave changes pitch becomes higher, as pitch depends on
when it crosses the boundary separating two frequency.
media C.the wavelength of a given sound wave in 56. The characteristic of a vibration that
air decreases as the temperature increased determines its intensity is the A.frequency
D.sound waves can not be reflected. B.overtone C.wavelength D.amplitude
None of the options are correct. Intensity depends on amplitude i.e intensity is
Speed of sound increases with absolute proportional to the square of the amplitude of a
temperature 𝑣 ∝ √𝑇, hence sound travels faster vibrating body 𝐼 ∝ 𝐴2 .
at higher temperature i.e 30℃ than at low 57. Loudness of a musical note depends on its

174
Demystified Series Physics Demystified by Dr Timothy
A.overtones B.frequency C.amplitude quality E.The velocity of sound in air does not
D.fundamental notes E.harmonics. depend on the intensity produing it.
Loudness of a musical note depends on its The speed of the wind affects the speed of sound
amplitude i.e loudness is proportional to the in air i.e speed of sound increases when wind
square of the amplitude 𝐿 ∝ 𝐴2 . travels in its direction.
58. Which of the following is true of the loudness 65. Which of the following is not correct? I.The
of sound? It A.depends on the square of the pitch of a sound note depends on the frequency
amplitude of the vibrating body B.is of vibrations II.The intensity of a sound note is
proportional to the distance of the observer from proportional to the amplitude of vibrations
the source of the sound C.is greater in vacuum III.Beats are produced by two sources sound
D.is independent of frequency. because one wave is travelling faster than the
Loudness depends on intensity and square of the other IV.When two sources of sound of
amplitude of the vibrating body. Loudness and frequencies 500Hz and 502Hz are sounded
intensity is inversely proportional to the square together, a beat frequency of 2Hz is observed
1
of the distance from a source 𝐿 ∝ 2 . Loudness of V.The first harmonic of a note has double the
𝑟
frequency of the fundamental note A.I and II B.II
sound is lowest in a vacuum as there is no
and III C.I and III D.III and IV E.IV and V.
material medium e.g air, for sound to travel, as
Pitch depends on frequency of vibration.
sound is a mechanical wave.
Intensity is proportional to the square of the
59. If the distance from a point source of sound is
amplitude. Beats are produced when two sources
doubled, by what factor does the intensity
of nearly equal frequency are sounded together.
decreases? A.4.00 B.2.00 C.0.50 D.0.25.
Beat frequency is the difference between the
Intensity of sound 𝐼 varies inversely with the
1 frequency of the two sources, 𝑓𝐵 = 502−500 =
square distance 𝑟 from the source 𝐼 ∝ 2 . 2Hz. The first harmonic of a note is same as the
𝑟
𝐼1 𝑟1 2 = 𝐼2 𝑟2 2 , 𝐼1 =𝐼, 𝑟1 =𝑟, 𝐼2 =?, 𝑟2 =2𝑟, fundamental frequency of the note. Hence,
𝐼 × 𝑟 2 = 𝐼2 × (2𝑟)2 , 𝐼𝑟 2 = 𝐼2 × 4𝑟 2 , statement “II,III and V” are not correct.
𝐼𝑟 2 𝐼 66. All of the following frequencies are overtones
𝐼2 = 2 = or 0.25𝐼 i.e the intensity decreases
4𝑟 4
of 320Hz except A.520Hz B.640Hz C.960Hz
by one-fourth of its original value or by a factor
D.1280Hz E.1600Hz.
of 0.25. Alternatively :
1 1 1 Overtones are multiples of frequency above the
𝐼 ∝ 2 , distance 𝑟 is doubled , 𝐼 = 2 = = 0.25. fundamental frequency. Hence overtones of
𝑟 2 4
60. If the same note is sounded on a piano, an 320Hz are multiples of frequency above it.
organ or any other musical instrument,the 1st overtone 𝑓1 = 2×320 = 640Hz,
source of sound can be recognized because of the 2nd overtone 𝑓2 = 3×320 = 960Hz,
characteristic of sound called A.pitch 3rd overtone 𝑓3 = 4×320 = 1280Hz,
B.frequency C.quality D.intensity E.wavelenght 4th overtone 𝑓4 = 5×320 = 1600Hz.
Quality/timbre/tone is a characteristic note of a 520Hz is not a multiple of 320Hz, hence is not an
musical instrument which differentiates it from overtone of 320Hz.
another note of the same pitch or frequency and 67. A note is the octave of another note if it has
loudness produced by another instrument. A.a frequency twice that of the first note B.a
61. Notes of the same frequency sounded on a frequency half of the first note C.the same
piano and a flute may differ in A.loudness fundamental frequency D.a frequency eight times
II.quality III.pitch A.I B.II C.I and II D.II and III that of the first note.
E.I,II and III. An octave is a sound of frequency twice its
62. The quality(timbre) of sound depends on fundamental frequency of frequency of its first
A,amplitude B.frequency C.harmonics D.velocity note.
E.wavelength. 68. A slight loading of a tuning fork has the effect
The quality(timbre) of sound depends on of A.decreasing its amplitude B.increasing its
overtones or harmonics. amplitude C.decreasing its frequency
63. What determines the quality of a note D.increasing its frequency.
produced by a vibrating string? A.The pitch of Loading e.g by placing a piece of plasticine on it
the note B.The tension of the string C.The or adding gums to the prong, so as to increase the
material of the string D.The presence of mass, decreases the frequency while the
overtones E.The thickness of the string. amplitude remains constant. The frequency of
64. Which of the following is not correct? A.The the unloaded tuning fork is greater than the
pitch of a note depends on the frequency of a frequency of loaded tuning fork.
vibrating source B.The speed of the wind has 69. Two sound waves have frequencies of 12Hz
no effect on the speed of sound in air C.The and 10Hz. Calculate their beat period A.0.5s
overtones present in a note determines its B.1.0s C.1.2s D.2.0s.

175
Demystified Series Physics Demystified by Dr Timothy
𝑓1 =12Hz, 𝑓2 =10Hz, beat period 𝑇𝐵 =?, 76. A guitar string is 75cm long. The wavelength
Beat frequency 𝑓𝐵 = 𝑓1 − 𝑓2 = 12−10 = 2Hz or 4 of its fundamental note is A.75cm B.150cm
1 1
beats per second. Beat period 𝑇𝐵 = = = 0.5s. C.37.5cm D.112.5cm E.50cm .
𝑓𝐵 2
Length of the string 𝐿=75cm, number of
70. Of two identical tuning forks with natural
harmonics 𝑛=1 i.e fundamental note, wavelength
frequency 256Hz,one is loaded so that 4 beats 2𝐿
per second are heard when they are sounded of a fundamental note 1 =? , 𝑛 = ,
𝑛
2×75
together.What is the frequency of the loaded 1 = = 150cm.
1
tuning fork? A.260Hz B.252Hz C.248Hz D.264Hz 77. A string is fastened tightly between two walls
E.258Hz. 24cm apart. The wavelength of the second
Loading a tuning decreases its frequency. The overtone is A.24cm B.16cm C.12cm D.8cm
loaded tuning fork has a lower frequency than Distance between the two walls=Length of the
the unloaded tuning fork. Frequency of unloaded string 𝐿=24cm, number of harmonics 𝑛=3 i.e 2nd
tuning fork 𝑓1 =256Hz, beat frequency 𝑓𝐵 =4 beats overtone or 3rd harmonic, wavelength of the 2nd
per second or 4Hz, frequency of loaded tuning 2𝐿 2×24
overtone 3 =?, 𝑛 = , 3 = = 16cm.
fork 𝑓2 =?, 𝑓𝐵 = 𝑓1 − 𝑓2 , 4 = 256 −𝑓2 , 𝑛 3
𝑓2 = 256−4 = 252Hz. 78. If a sonometer has fundamental frequency of
71. A sounding tuning fork is brought near the 450Hz, what is the frequency of the fifth
end of a pipe containing air column and the overtone? A.2700Hz B.456Hz C.444Hz D.75Hz.
loudness of the sound increases. This effect is due A sonometer is an instrument used for the
to A.diffraction B.echo C.resonance experimentation of factors affecting the
D.interference E.reverberation. frequency of a vibrating string. Fundamental
Resonance occurs when the natural frequency of frequency 𝑓𝑜 =450Hz, fifth overtone 𝑓5 =sixth
a vibrating body coincides or equals the harmonics= 6𝑓𝑜 = 6×450 = 2700Hz.
frequency of the external periodic force causing 79. The lowest note emitted by a stretched string
it to vibrate.It is characterized by a increase in has a frequency of 40Hz. How many overtones
loudness and amplitude. are there between 40Hz and 150Hz? A.4 B.3 C.2
72. When the bottom tip of vibrating tuning fork D.1.
is held in contact with a wooden box, a louder Frequency of lowest note=fundamental
sound is heard. This phenomenon is known as frequency 𝑓𝑜 =40Hz, 1st overtone 𝑓1 = 2𝑓𝑜 =
A.beats B.echoing C.resonance D.reverberation. 2×40 = 80Hz, 2nd overtone 𝑓2 = 3𝑓𝑜 = 3×40 =
73. Marching solidiers crossing a suspension 120Hz, 3rd overtone 𝑓3 =4𝑓𝑜 = 4×40 ,
bridge are usually advised to break their steps to = 160Hz. Hence, the number of overtones
avoid damaging the bridge owing to A.swinging between 40Hz and 150Hz are 2 i.e 80Hz and
B.vibration C.oscillation D.resonance. 120Hz.
When the soldiers match in step, the bridge is 80. A steel wire of length 0.5m is stretched
forced to vibrate in resonance at a frequency between two fixed points and its fundamental
same as the natural frequency of the soldiers, frequency is 200Hz. The speed of the wave in the
which thereby causes the bridge to collapse. wire is A.100ms-1 B.120ms-1 C.200ms-1
74. Which of the following statements is correct D.250ms-1.
2𝐿 2𝐿
about resonance? Resonance occurs when a body 𝐿=0.5m, 𝑓𝑜 =200Hz, 𝑛 = , 𝑛=1, 1 = ,
𝑛 1
A.vibrates with its natural frequency B.is forced 1 =2𝐿, 𝑣 = 𝑓 = 2𝐿𝑓𝑜 = 2×0.5×200 = 200ms-1.
to vibrates at its own natural frequency by 81. Find the frequencies of the first three
another body vibrating at the same frequency harmonics of a piano string of length 1.5m, if the
C.vibrates at the frequency of the body causing velocity of the waves on the string is 120ms-1
vibration D.vibrate with an irregular frequency. A.40Hz,80Hz,120Hz B.80Hz,160Hz, 240Hz
Option A and C describes free and forced C.180Hz,360Hz,540Hz D.360Hz,180Hz, 90Hz.
vibration respectively. 2𝐿
𝐿=1.5m, 𝑣=120ms-1, 𝑛 = , 𝑛=1, 1 = ,
2𝐿
75. Which of the diagrams below represents the 𝑣
𝑛
𝑣 120
1
second overtone of a vibrating string fixed at 1 =2𝐿, 𝑣 = 𝑓, 𝑓𝑜 = = = = 40Hz ,
 2𝐿 2×1.5
both ends? A. 𝑁 𝐴 𝑁 B. 𝑁 𝐴 𝑁 𝐴 𝑁 1st harmonic 𝑓𝑜 = 40Hz, 2nd harmonic 2𝑓𝑜 =
C. 𝑵 𝑨 𝑵 𝑨 𝑵 𝑨 𝑵 D. 𝑁 𝐴 𝑁 𝐴 𝑁 𝐴 𝑁 𝐴 𝑁 2×40 = 80Hz, 3rd harmonic 3𝑓𝑜 = 3×40 =
Second overtone is same as third harmonic or 120Hz.
third position or three loops,as indicated by the 82. What factors determined the frequency of a
digram in option C. Option A indicates the note emitted by a vibrating string? A.Amplitude
fundamental or first harmonic i.e one loop, of vibration,force constant of string and length of
option B indicates the first overtone or second string B.Amplitude of vibration,force constant of
harmonic i.e 2 loops, option D indicates third string and tension in string C.Mass per unit
overtone or fourth harmonic i.e four loops. length of string, tension in string and length

176
Demystified Series Physics Demystified by Dr Timothy
of string D.Force constant of string,tension in 87. Under constant tention and constant mass
string and length of string. per unit length, the note produced by a plucked
The frequency of a vibrating string is determined string is 500Hz when the length of the string is
by its length, linear density or mass per unit 0.90m. At what length is the frequency 150Hz?
length and tension or force in the string. A.3m B.4m C.5m D.6m.
83. Which of the following statements is true? 𝑓1 =500Hz, 𝐿1 =0.90m, 𝑓2 =150Hz, 𝐿2 =?,
The frequency of a vibration is A.proportional to 1
𝑓 ∝ at constant 𝑇 and 𝜇 , 𝑓1 𝐿1 = 𝑓2 𝐿2 , 500×0.9
𝐿
its length B.inversely propotional to the tension 500×0.9
within it C.proportional to the amplitude of = 150× 𝐿2 , 𝐿2 = = 3m.
150
vibration D.inversely propotional to its cross 88. When the tension in a vibrating string is
section E.proportional to the square root of increased, its frequency of vibration A.increases
the tension within it. as the square root of the tension B.decreases as
Frequency of a vibrating string is proportional to the square root of the tension C.increases as the
the reciprocal of its length,square of the tension square of the tension D.decreases as the square
within it and square root of the reciprocal of its of the tension E.is directly proportional to the
1 𝑇 tension.
linear density or mass per unit length , 𝑓 ∝ √ .
𝐿 𝜇 𝑓 ∝ √𝑇 , hence frequency is directly proportional
84. Under which of the following conditions will to the square root of tension.
a stringed instrument produce a high frequency 89. When the tension in a sonometer wire is
note. When the string is A.long,thick and loose doubled the ratio the new frequency to its initial
B.long,thin and loose C.long,thin and taut frequency is A.1/√2 B.1/2 C.√2 D.2
D.short,thick and taut E.short,thin and taut 𝑓 𝑓
𝑇1 =𝑇, 𝑇2 =2𝑇, 𝑓1 :𝑓2 =?, 𝑓 ∝ √𝑇 , 1 = 2 ,
√𝑇1 √𝑇2
1 𝑇
𝑓∝ √ hence, decrease in length i.e short 𝑓2 √𝑇2 𝑇2 2𝑇 2
𝐿 𝜇
= =√ =√ = √ = √2:1 or √2.
𝑓1 𝑇1 𝑇 1
string, decrease in thickness or linear density i.e √𝑇1

thin string and increase in tension i.e tout, will Alternatively, 𝑓 ∝ √𝑇 , when tension 𝑇 is
give a note with high frequency or pitch. doubled, then 𝑓 = √2 i.e frequency is increases
85. If the load at the end of a sonometer wire is by a factor of √2 or the ratio of its final to initial
immersed in a bucket of water,the original frequency is √2 or √2:1.
fundamental frequency of the wire could be 90. The fundamental frequency of a plucked wire
restored by A.decreasing the length of the under a tension of 400N is 250Hz. When the
wire B.increasing the length of the wire frequency is changed to 500Hz at constant
C.increasing the mass per unit length of the wire length, the tension is A.40N B.160N C.400N
D.changing the temperature of water D.1600N.
Immersion of the load in water, decreases the 𝑓1 =250Hz, 𝑇1 =400𝑁, 𝑓2 =500Hz, 𝑇2 =?,
weight of the load due to upthrust. Hence, weight 𝑓1 𝑓2 𝑓1 2 𝑓2 2
or tension 𝑇 is reduced. 𝑓 ∝ √𝑇 , = or = ,
√𝑇1 √𝑇2 𝑇1 𝑇2
1 𝑇 1 𝑇 𝑇 2502 5002 500×500×400
𝑓 ∝ ∝ √ , 𝑓2 ∝ 2 ∝ , 𝑓2 ∝ 2 , = , 𝑇2 = = 1600𝑁.
𝐿 𝜇 𝐿 𝜇 𝐿 𝜇 400 𝑇2 250×250

𝑇 ∝ 𝑓 2 𝐿2 𝜇, 𝑇 ∝ 𝐿2 𝜇, where 𝑓=constant, as 91. The note produced by a stretched string has


original fundamental frequency is to be restored. a fundamental frequency of 400Hz. If the length
𝑇 ∝ 𝐿2 𝜇or 𝑇 ∝ 𝐿2 ∝ 𝜇 i.e Tension 𝑇 is directly of the string is doubled while the tension in the
proportional to square of the length 𝐿 and string is increased by a factor of 4,the frequency
directly proportional to the linear density or is A.200Hz B.400Hz C.800Hz D.1600Hz
mass per unit length 𝜇. 1 𝑇 1
𝑓 ∝ √ , 𝑓 ∝ √𝑇 i.e when mass per unit length
Decreasing the weight or tension of the load , will 𝐿 𝜇 𝐿

cause a corresponding decrease in the length of or linear density 𝜇 is constant. Thus, when the
the wire and decrease in linear density or mass length 𝐿 is doubled while the tension 𝑇 increased
1 1
per unit length. by a factor of 4 , the frequency 𝑓 = √4 = × 2
2 2
86. When the length of a vibrating string is = 1.The ratio of the final to initial frequency
doubled, its frequency of vibration A.remains the 𝑓2 :𝑓1 = 1:1 i.e the frequency is constant or
same B.becomes half the former value unaltered. The new frequency 𝑓2 =400Hz.
D.becomes double former value E.becomes four 92. Which of the following actions is necessary in
times the former value order to tune a string to produce a note of an
Frequency is inversely proportional to the length octave higher than its fundamental? A.Doubling
1
of a string, 𝑓 ∝ . Hence, when the length 𝐿 is the length of the string B.Reducing the tension by
𝐿
1
doubled, 𝑓 = i.e frequency is reduced by half half C.Reducing the mass per unit length
2 D.increasing the tension four times
or becomes half the former value.

177
Demystified Series Physics Demystified by Dr Timothy
Doubling the length of the string decreases the 𝑓10 =10𝑓𝑜 = 10×50 =500Hz.
1 1
frequency by half i.e 𝑓 ∝ , 𝑓 = . 96. A sonometer wire of linear density 0.08kgm-
𝐿 2 1 subjected to a tension of 800𝑁 is plucked.
Reducing the tension by half decreases the
Calculate the speed of a pulase which moves from
1 1
frequency by a factor of √ , 𝑓 ∝ √𝑇 , 𝑓 = √ one end of the wire to the other A.179.0ms-1
2 2
B.100.0ms-1 C.15.8ms-1 D.8.0ms-1
Reducing the mass per unit length increases the
𝑇
1
frequency 𝑓 ∝ . 𝜇=0.08kgms-1, 𝑇=800𝑁, 𝑣=?, 𝑣 = √ ,
𝜇 𝜇
Increasing the tension four times increases the 800
frequency by a factor of 2 or the frequency is 𝑣=√ = √10000 = 100ms-1.
0.08
twice the fundamental i.e an octave higher than 97. A transverse wave is applied to a string
its fundamental, 𝑓 ∝ √𝑇 , whose mass per unit lemgth is 3×10-2kgm-3. If the
𝑓 = √4 = 2 or 2𝑓𝑜 . string is under a tension of 12𝑁,the speed of
93. Two strings of the same length and under the propagation of the wave is A.40ms-1 B.30ms-1
same tension give notes of frequency in the ratio C.20ms-1 D.5ms-1
4:1. The masses of the strings are in the 𝜇=3×10-2kgm-3, 𝑇=12𝑁, 𝑣=?, 𝑣 = √ ,
𝑇
corresponding ratio of A.2:1 B.1:2 C.1:4 D.1:16 𝜇

1 𝑇 1 𝑇 1 1 12
𝑓 ∝ √ or 𝑓 ∝ √ , thus, 𝑓 ∝ √ or 𝑓 ∝ , 𝑣=√ = √4 × 102 = 40ms-1.
𝐿 𝜇 𝐿 𝑚/𝐿 𝑚 √𝑚 3×10−2
when length 𝐿 and tension 𝑇 are constant, 98. Which of the following statements about the
𝑓1 :𝑓2 =4:1, 𝑚1 :𝑚2 =?, 3rd overtone of a vibrating air column of an open
𝑓1 𝑚2 𝑚2 4 𝑚2 pipe is correct? It has A.4 nodes B.5 nodes C.3
𝑓1 √𝑚1 = 𝑓2 √𝑚2 , =√ =√ , =√ , nodes D.4 antinodes.
𝑓2 √𝑚1 𝑚1 1 𝑚1
42 𝑚2 𝑚2 16 𝑚1 1 Sketch the diagram of the 3rd overtone or 4th
= , = , = ,
12 𝑚1 𝑚1 1 𝑚2 16 harmonic, using the fact that the wavelength
𝑚1 :𝑚2 = 1:16. Alternatively, increase by half ,
1
which is equivalent to two
1 1 1 2
𝑓∝ , √𝑚 ∝ or 𝑚 ∝ 2 , as frequency is in consecutive nodes inevery overtone. An open
√𝑚 𝑓 𝑓
the ratio of 4:1 or = 4, hence, 𝑚 = 2 =
4 1 1 pipe always have an antinodes at both open ends.
1 4 16
Therefore, the third overtone or 4th harmonic
or 𝑚1 :𝑚2 = 1:16.
has 4 nodes and 5 antinodes.
94. A piano wire 0.5m long has a total mass of
99. An organ pipe of length 40cm opened at both
0.01kg and is stretchyed with a tension of 800𝑁.
ends sounds its fundamental note. Neglecting
Calculate the frequency of the wire when it
end corrections, calculate the frequency of the
sounds its fundamental note A.200Hz B.100Hz
note (Speed of sound in air=340ms-1) A.213Hz
C.4Hz D.2Hz
B.272Hz C.380Hz D.425Hz E.544Hz
1 𝑇
𝐿=0.5m, 𝑚=0.01kg, 𝑇=800𝑁, 𝑓𝑜 =?,𝑓𝑜 = √𝜇 𝐿=40cm=0.4m, 𝑣=340ms-1, wavelength of an
2𝐿 2𝐿
open pipe 𝑛 = , 𝑛=1 for fundamental note,
1 𝑇 1 𝑇𝐿 1 800×0.5 𝑛
𝑓𝑜 = √𝑚/𝐿 = 2𝐿 √ 𝑚 = 2×0.5 √ , 2𝐿 𝑣 𝑣
2𝐿 0.01 1 = = 2𝐿 , 𝑣 = 𝑓𝑜 , 𝑓𝑜 = = ,
1  2𝐿
340
𝑓𝑜 = √
400
= √40000 = 200Hz. 𝑓𝑜 = = 425Hz. Alternatively, frequency of
0.01 2×0.4
𝑛𝑜 overtones of an open pipe is given by,
95. A string of length 1.0m vibrate in 10 loops. If 𝑣
the total mass of the string is 1.0×10-3kg and the 𝑓𝑛 = (𝑛𝑜 + 1) , 𝑓𝑛 =𝑓𝑜 , number of overtones
2𝐿
tension in it is 10N,calculate the frequency of the 𝑛𝑜 =0 overtones or first harmonic or
340
vibration A.50HzB.100Hz C.250Hz D.400Hz fundamental, 𝑓𝑜 = (0+1) = 425Hz.
2×0.4
E.500Hz. 100. An organ pipe of length 40cm opened at
number of loops 𝑛=10 loops, 𝐿=1m, 𝑇=10𝑁, both ends sounds its second overtone.Neglecting
frequency of vibration 𝑓𝑛 = 𝑓10 , end correttions,calculate the frequency of the
𝑓𝑛 =
𝑛 𝑇 𝑛
√𝜇 = 2𝐿 √𝑚/𝐿 = 2𝐿 √ 𝑚 ,
𝑇 𝑛 𝑇𝐿 note. [Speed of sound in air=340ms-1] A.213Hz
2𝐿
B.425Hz C.1275Hz D.850Hz
𝑓10 =
10

𝑇𝐿
=
10

10×1
= 5√
10
, 𝐿=40cm=0.4m, 𝑛𝑜 =2 i.e 2nd overtone, 𝑣=340ms-
2𝐿 𝑚 2×1 1×10−3 10−3 1, second overtone 𝑓 =?,
2
𝑣 340
𝑓10 =5√10 × 103 =5√104 =5×102 =500Hz. 𝑓𝑛 = (𝑛𝑜 + 1) , 𝑓2 = (2+1) ,
2𝐿 2×0.4
Alternatively, 𝑓𝑜 =
1 𝑇 1 𝑇
√𝜇 = 2𝐿 √𝑚/𝐿 = 2𝐿 √ 𝑚
1 𝑇𝐿
𝑓2 = 3×425 = 1275Hz.
2𝐿
101. An organpipe open at both ends has a
𝑓𝑜 =
1

10×1
=0.5√
10
= 0.5√10 × 103 fundamental frequency of 400Hz. The
2×1 1×10−3 10−3 frequencythe third overtone is A.400Hz
𝑓𝑜 =0.5√104 =0.5×102=50Hz, B.1600Hz C.800Hz D.1200Hz .

178
Demystified Series Physics Demystified by Dr Timothy
Open pipes has all multiples(odd and even) of the velocity of sound in air is 340ms-1 what is the
harmonics. 3rd overtone 𝑓3 =4th harmonic frequency of the fundamental note produced by
𝑓3 = 4𝑓𝑜 = 4×400 = 1600Hz. the pipe ? (Neglect end correction) A.188.89Hz
102. In a closed organ pipe producing a musical B.204.00Hz C.566.67Hz D.755.56Hz
note, an antinode is always at A.the closed end E.2266.67Hz.
B.the open end C.middle D.All the parts of the Length of closed pipe=60cm, length of water in
pipe. 3
the tube = ×60 =45cm, length of air column
4
At the open end of a closed pipe is an antinode
𝐿=60−45=15cm=0.15m, 𝑣=340ms-1, 𝑓𝑜 =?, 𝑛𝑜 =0,
while at the closed end is a node. 𝑣 340
103. An organ pipe closed at one end is 8cm long. 𝑓𝑛 = (2𝑛𝑜 + 1) , 𝑓𝑜 = (2×0+1) ,
4𝐿 4×0.15
Neglecting end corrections, calculate the 𝑓𝑜 = (0+1)566.67= 566.67Hz.
wavelength of its fundamental note A.2cm B.4cm 110. The air column as shown
C.8cm D.16cm E.32cm. in the figure vibrates by
𝐿=8cm, wavelength of 𝑛 harmonics of a closed the tuning fork. In this
4𝐿
pipe 𝑛 = , 𝑛=1 for fundamental note, 1 = state of resonance the
4𝐿
𝑛 waves in the air column will be A.stationary and
= 4𝐿 = 4×8 = 32cm. transverse B.stationary and longitudinal
1
104. The wavelength of the first overtone of a C.progressive and transverse D.progressive and
note in a closed pipe of length 33cm is A.44cm longitudinal.
B.33cm C.22cm D.17cm. The vibration of the tuning fork sets up
4𝐿
𝐿=33cm, 𝑛 = , 𝑛=3 for 1st overtone or 3rd stationary and transverse wave in the air column
𝑛
4×33 in the resonance tube.
harmonic(odd harmonics), 2 = = 44cm. 111.
3
105. From the digram of a closed pipe below, the Vibrating tuning Air
frequency produced by the wavelength of the fork
length 𝐿 is known as Water
Glass jar

𝐿 Resonance will occur in the above set=up when


A. fundamental B.second harmonic C.third the natural frequency of A.the water column is
harmonic D.fifth harmonic. the same as the frequency of the tuning fork B.the
The diagram of the closed pipe indicates the 3rd water column and the air column is the same as
overtone which is the 5th harmonic as closed the frequency of the tuning fork C.the air column
pipe form odd harmonic only. is higher that the frequency of the tuning fork
106. A pipe of length 45cm is closed at one end. D.the air column is the same as the frequency
Calculate the fundamental frequency of the of the tuning fork E.the air column is less than
sound generated in the pipe if the velocity of the frequency of the tuning fork.
sound in air is 360ms-1 (Neglect end corrections) 112. In the figure shows, a resonan-
A.5.5Hz B.148.5Hz C.200.0Hz D.550.0Hz. 𝐿 ce tube experiment is perform-
𝐿=45cm=0.45m, number of overtones 𝑛𝑜 =0 for ed using one tuning fork. As
fundamental frequency, 𝑣=360ms-1, 𝑓𝑜 =?, the water level is lowered the
𝑣 360
𝑓𝑛 = (2𝑛𝑜 + 1) , 𝑓𝑜 = (2×0+1) , first resonance is is obtained when the length of
4𝐿 4×0.45
𝑓𝑜 = (0+1)200 = 200Hz. the column 𝐿 = /4. The second resonance is
107. If the fundamental frequency of a closed obtained when 𝐿 equals A./2 B.3/4 C.
pipe organ on a day when the speed of sound is D.3/2.
340ms-1 is 170Hz, then the length of the pipe is The first resonance length is equivalent with the

A.50cm B.70cm C.100cm D.150cm E.200cm. fundamental note of a closed pipe, 𝐿1 =
4
𝑣=340ms-1, 𝑓𝑜 =170Hz, 𝑛𝑜 =0, 𝐿=?, The second resonance length is equivalent with
𝑣 𝑣 𝑣
𝑓𝑛 = (2𝑛𝑜 + 1) , 𝑓𝑜 = (2×0+1) = , the 1st overtone or 3rd harmonic of a closed pipe,
4𝐿 4𝐿 4𝐿
3
170 =
340
, 𝐿=
340
= 0.5m = 50cm. 𝐿2 = .
4𝐿 4×170 4
108. When vibration occur in an air column, the 113. If the first resonance position of resonance
distance between a node and an antinode is equal in a resoanace tube is 16.50cm from the open
to A.One-quarter of the wavelength B.One-half end, the distance from the open end to the next
the wavelength C.The wavelength D.Twice the position where resonance occurs will be
wavelength E.Four times the wavelength A.8.25cm B.22.00cm C.24.75cm D.33.00cm
The distance between a node and an antinode is E.49.50cm.
one-quarter the wavelength, 𝑁𝐴 = .
1 First position of resonance or first resonance
4 length 𝐿1 =16.50cm, second position of resonance
109. A closed pipe 60cm long is ¾ full of water. If

179
Demystified Series Physics Demystified by Dr Timothy
or second resonance length 𝐿2 =?, D.53×10-5ms-1 E.2500ms-1.
 𝑓=750Hz, 𝐿=0.3m, the diagram indicates the
𝐿1 = ,  = 4𝐿1 = 4×16.50 = 66.0cm
4 3 4𝐿
3 3×66.0 second position of resonance 𝐿 = ,  = ,
𝐿2 = = = 49.50cm. Alternatively, 4 3
4 4 4𝐿 4×750
  𝑣 = 𝑓 = × 𝑓 = × 0.3 = 300ms-1.
𝐿1 = , 𝐿2 =3( ) = 3𝐿1 = 3×16.5 = 49.50cm. 3 3
4 4
119. A tuning fork of frequency 340Hz is
114. In a resonance tube experiment, the
vibrated just above a cylindrical tube of height
effective length of the air column for the first
1.2m.If water is slowly poured into the tube,at
resonance is 20cm when set into vibration by a
what maximum height will resonance occur.
tuning fork of frequency 480Hz. Neglecting end
[Speed of sound in air=340ms-1] A.0.95m
effect, the velocity of sound in air is A.96ms-1
B.0.60m C.0.50m.
B.225ms-1 C.340ms-1 D.384ms-1.
First position of resonance is equal to the
First resonance length 𝐿1 =20cm=0.2m, 
frequency of vibrating air column 𝑓 equals fundamental note,𝐿 = ,𝐿=length of air
4
thefrequency of tuning fork at resonance, column,𝑣=340ms , 𝑓=340Hz.
-1
 𝑣 340
𝑓=480Hz, 𝑣=?, 𝑣 = 𝑓 , 𝐿1 = ,  = 4𝐿1 , 𝑣 = 𝑓 = 4𝐿 × 𝑓, 𝐿 = = = 0.25m,
4 4𝑓 4×340
𝑣 =4𝐿1 𝑓 = 4×0.2×480 = 384ms-1. 0.25m The maximum height of the
115. The diagram above shows a tube at which resonance will
𝐿 stationary wave of wavelength 1.2m will occur = height or length
40cm in a closed tube.The length 0.95m of water column 𝐿𝑤 .
of the resonating air column is Height of the tube ℎ=1.2m,
A.40cm B.10cm C.20cm D.30cm. Length of air column
The diagram shows the second position of 𝐿=0.25m, 𝐿𝑤 = ℎ − 𝐿, 𝐿𝑤 =1.2−0.25 =0.95m.
3
resonance 𝐿 = , =40cm, 𝐿 =
3×40
= 30cm. 120. In a resonance tube closed at one end, the
4 4
first two lengths of the air column that vibrate in
Alternatively, the closed tube has 1 antinode at
resonance with a tuning fork of frequency 316Hz
the top and 2 nodes i.e 1 at the middle and 1 at
  are 20.6cm and 73.1cm respectively.Calculate
the bottom. Thus, 𝐿 = 𝐴𝑁 + 𝑁𝑁 = + , the velocity of sound in air to 3 significant figures
4 2
+2
𝐿= = =
3 3×40
= 30cm. A.296ms-1 B.325ms-1 C.330ms-1 D.332ms-1
4 4 4
116. In a resonance tube experiment, if the E.338ms-1.
fundamental frequency of a vibrating air is 𝑓, the 𝑓=316Hz, 𝐿1 =20.6cm, 𝐿2 =73.1cm, 𝑣=?,
1 1 1 𝑣 = 2𝑓(𝐿2 − 𝐿1 ) = 2×316(73.1−20.6),
possible overtones are A. 𝑓, 𝑓, 𝑓 e.t.c B.2𝑓, 3𝑓, 𝑣 = 632(0.525) = 331.8 = 332ms-1.
2 4 8
4𝑓, 5𝑓, 6𝑓 e.t.c C.2𝑓, 4𝑓, 6𝑓, 8𝑓 e.t.c D.3𝒇, 5𝒇, 7𝒇, 121. The shortest length of the air column in a
9𝒇 e.t.c. resonance tube at resonance is 0.11m and the
Resonating tube is a close pipe and will produce next resonant length is 0.36m.Calculate the
only odd harmonics of overtones. frequency of vibration, given that the speed of
the tuning fork. sound in air is 340ms-1 A.340Hz B.347Hz
117. In a resosnace tube experiment, a tube of C.680Hz D.694Hz E.1360Hz
fixed length is closed at one end and several 𝑣= 340ms-1, 𝐿1 =0.11m, 𝐿2 =0.36m, 𝑓=?
tuning forks of increasing frequency used to 𝑣 = 2𝑓(𝐿2 − 𝐿1 ) , 𝑓 =
𝑣
=
340
obtain resonance at the open end. If the tuning 340
)
2(𝐿2 −𝐿1 2(0.36−0.11)

fork with the lowest frequency which gave 𝑓=


2×0.25
=680Hz.
resonance had a frequency of 𝑓1 and the next 122. Which of the following is a percussion
tuning fork which gave resonance had a instrument? A.Flute B.Guitar C.Piano D.Trumpet
frequency of 𝑓2 , find the ratio of 𝑓2 :𝑓1 A.8 B.3 C.2 E.Xylophone.
D.1/2 E.1/3 Flute and trumpet are wind instruments while
A resonance tube is a closed pipe. guitar and piano are string instruments.
First resonance position 𝑓1 = fundamental 123. Which of the following pairs of musical
frequency 𝑓𝑜 , Second resonance position 𝑓2 = instruments works on the principle of vibration
first overtone = third harmonic 3𝑓𝑜 , in air pipes? A.Guitar and piano B.Guitar and
𝑓2 3𝑓
= 𝑜 = 3:1 = 3. flute C.Trumpet and flute D.Organ and piano
𝑓1 𝑓𝑜
E.Violin and trumpet
118. 𝑓=760Hz In a resonance tube exper-
iment in the figure the Air column is vibrated in trumpet and flute,
string is vibrated in guitar,piano,violin and organ.
0.3m frequency of the tuning fork
124. Which of the following instruments has a
is 750Hz and the resonating
length of the air column is pure tone A.guitar B.vibrating strings C.tuning
0.3m as shown. The velocity fork D.siren.
A tuning fork has a pure tone or sound as it
of the wave is A.1850ms-1 B.225ms-1 C.300ms-1

180
Demystified Series Physics Demystified by Dr Timothy
produces sound with single or fundamental
frequency without being excited i.e free
vibration, vibrating at its own natural or
fundamental frequency.

Jamb past questions on sound waves :


[1982/6,47,1983/1,1984/14,15,1985/27,28,
1986/26,35,37,1987/29,30,31,1988/21,24,25,
26,2728,29,1989/20,21,1990/21,1991/30,31,
32,36,1992/22,1993/20,21,24,1994/29,30,31,
1995/30,31,1997/29,1998/24,25,26,27,29,
1999/12,19,21,2000/18,24,2001/12,18,2002/
13,15,29,2003/18,19,2004/4,2008/29,2009/28
,29,2011/26,31,2013/27,2014/27]

181
Demystified Series Physics Demystified by Dr Timothy

Chapter 17 – Light wave and Reflection of light


from plane and curved/spherical surface
● Light wave – Light wave is an electromagnetic or Umbra or black region is formed from a
wave i.e wave which do not require a material point source of light i.e small lightsource.
medium for its propagation or that can travel Partial shadow or Penumbra or grey region is
through a vacuum. formed from an extended light source i.e large
- Light wave is a transverse wave. Light is a source of light.
visible for for energy that enables vision. - The sharpness of the boundary of a shadow
- Sources of light – It can be natural e.g sun, isdetermined by the intensity of light stricking
stars,fire flies e..tc or artificial or e.g electric lamp, the object.
candle light e.t.c. 2.Eclipse – shadow formed when an opaque
- Luminous source – source that generate and celestial body (earth or moon) blocks a defined
emit light by itself. Natural or self luminous light source i.e the sun or the effect produced
sources are sun,stars,glowing worms,fire flies when one heavenly body cast its shadow on
and incandescent lamps. Artificial or man-made another.
luminous sources are candle light,torch light, car - Solar eclipse or eclipse of the sun – sun is
head lamps,matches and electric lamps. blocked by moon and the shadow of the moon is
- Non-luminous source – source that does not formed on earth.
generate light by itself but due to light falling on - Lunar eclipse or eclipse of the moon – sun is
it from another object that it reflects i,e they are blocked by earth and the shadow of the earth is
seen only when they reflect light from a luminous cast on the moon -
body. e.g moon,road signs, earth and pages of a Total eclipse(Umbra or total darkness):
book. formed where the extremes lines from the sun
- Transmission of light – Parts of the light touch the earth.
incident on a body is transmitted through the - Partial eclipse(Penumbra):formed where the
body while other parts are either reflected or intersecting lines from the sun touch the earth.
absorbed by the body. The amount of light - Annular eclipse – formed outside the earth
transmitted through a body depends on its when the extremes rays from the sun required
nature: transparent,transluscent and opaque. for total eclipse intersect before reaching the
- Transparent body – body that allows light to earth. It is seen beyond the vertex of the cone of
pass through it and that can be seen through e.g the moon produced (Umbra cone) and a bright
plane glass and clean water. ring of light or annulus is formed around the
- Translucent body – body that allows light to shadow of the moon.
pass through but cannot be seen through e.g 3.Pin-hole camera – It is made from an opaque
window panes,obscured/tinted glass sheets and box, one side of which is made of a translucent
tissue paper. sheet of paper and a tiny hole opposite it.
- Opaque body – body that do not allow light to - Images formed by a pinhole camera :
pass through and cannot be seen through e,g I.The image is real (as it can be caught on a
cardboard sheets,brick walls,metal sheets, wood screen) and inverted
or planks. II.With a pinhole or small tiny hole, the image will
- Ray of light is the direction or path taken by be sharp,clear and dim or not very bright due to
light energy. limited amount of light entering it.
- Beam of light is a collection of rays or stream III.With a large hole or many pinholes, the image
of light energy. will be blurred, overlapping and bright.
- Parallel beam is produced by a search light, IV.For large magnification(enlarged image), the
convergent beam is produced by a hand lens object distance must be small i.e the object
while divergent beam is produced by lamps. should be close to the pin hole and the image
- Rectilinear propagation of light is the distance should be large i.e the distance between
phenomenon of light rays travelling in straight the pinhole and the screen is increaded.
lines. - Hence, reducing the object distance will
- Diffraction of light is also called non- increase the height of the image.The image
rectilinear propagation of light. formed in this case is enlarged but less bright due
- Application of principle of rectilinear to ditribution of the light over a greater are of the
propagation of light: Formation of shadow, enlarged image. 𝑰𝒎𝒂𝒈𝒆 𝒉𝒆𝒊𝒈𝒉𝒕 ∝
𝒊𝒎𝒂𝒈𝒆 𝒅𝒊𝒔𝒕𝒂𝒏𝒄𝒆
,
eclipse and operation of a pin-hole camera. 𝒗
𝒐𝒃𝒋𝒆𝒄𝒕 𝒅𝒊𝒔𝒕𝒂𝒏𝒄𝒆

1.Shadow – images formed when source of light 𝑯𝑰 ∝ thus, bigger and brighter image.
𝒖
is blocked by an opaque material. Full shadow - When a pinhole camera is used in taking still

182
Demystified Series Physics Demystified by Dr Timothy
pictures the screen is replaced by a photographic 3.Image is at the same distance as the object
film. from the mirror – image distance 𝑣 is equal to
- External light is excluded from the pinhole the object distance 𝑢, 𝒗 = 𝒖.
camera by covering it with a dark cloth,inoder for 4.Image is the same size as object – image
a clearer image to be seen on the screen. height equals object height and magnification is
- Magnification produced by a pinhole camera unity, 𝒎 = 1 , 𝑯𝑰 = 𝑯𝑶 .
– Magnification 𝑚 is the ratio of the size(height) 5.Laterally inverted – inversion on the side
of the image to the size(height) of the object. only,the right hand of the object appears as the
𝒎=
𝒊𝒎𝒂𝒈𝒆 𝒉𝒆𝒊𝒈𝒉𝒕 𝑯𝑰
=
𝒊𝒎𝒂𝒈𝒆 𝒅𝒊𝒔𝒕𝒂𝒏𝒄𝒆 𝒗
, left hand on the mirror image. No change in the
𝒐𝒃𝒋𝒆𝒄𝒕 𝒉𝒆𝒊𝒈𝒉𝒕 𝑯𝑶
𝒍𝒆𝒏𝒈𝒕𝒉 𝒐𝒇 𝒄𝒂𝒎𝒆𝒓𝒂
𝒐𝒃𝒋𝒆𝒄𝒕 𝒅𝒊𝒔𝒕𝒂𝒏𝒄𝒆 𝒖
𝑳
vertical placement of the image, top is top and
𝒎= = . bottom is bottom.
𝒐𝒃𝒋𝒆𝒄𝒕 𝒅𝒊𝒔𝒕𝒂𝒏𝒄𝒆 𝒖
- Images in mirrors are formed due to
● Reflection of light waves reflection only. Multiple images are formed in
plane mirrors due to the thickness of the
- Reflection – Reflection of light occurs when
mirror.
light falls on an opaque body. It can either be
- Inclined plane mirrors – The number of
regular or diffused.
images formed when two formed when two
- Polished or smooth surfaces e.g mirrors,
mirrors are inclined at an angle 𝜃 to each other is
produces regular reflection. 𝟑𝟔𝟎
- Rough surface e.g a page of a book, produce given by; 𝒏 = −1.
𝜽
diffused or scattered reflection. - When two mirros are parallel to each other e.g
- Laws of reflection : mirrors in a barber’s shop, the angle between
1.The incident ray, the reflected ray and the them is 0° and they form infinite (multiple)
normal at the point of incidence, all lie in the images. 𝑛 =
360
−1= 𝛼 −1= 𝛼 i.e Infinite.
𝜃
same plane. - Perpendicularly inclined plane mirrors –
2.The angle of incidence 𝒊 with the normal is When two mirrors 𝑀1 and 𝑀2 are inclined
equal to the angle of reflection 𝒓 from the surface, perpendicularly i.e at 90° to each other, the angle
𝒊 = 𝒓. NB : Normal is not a ray. of incidence or reflection in 𝑀1 is equal to the
- The principle of reversibility of light states glancing angle of 𝑀2 while the angle of incidence
that the path of light ray is reversible. It is a or reflection in 𝑀2 is equal to the glancing angle
general principle in optics in 𝑀1 .
- Reflection in plane mirrors : A plane mirror is - The angle of deviation of the emergent ray
made when one side of a plane surface is coated. by two mirrors inclined at 90° to each other is
𝑰 𝑶 𝑹 𝐼𝑁 is the incidient ray, always 180°, regardless of the angle of
𝑅𝑁 is the reflected ray, incidence.
𝒊 𝒓 𝑂𝑁 isthe normal. - Plane mirror rotation – When a ray of light is
𝒈 𝑫 incident on a plane mirror and the mirror is
𝑵 rotated through an angle 𝜃, with the incident ray
- The angle of incidence 𝑖 is equal to the angle of being kept constant, the reflected ray is rotated
reflection 𝑟 , 𝒊 = 𝒓. through an angle 2θ i.e twice the angle of
-Angle between the incident ray and reflected rotation of the mirror.
ray= 𝒊 + 𝒓 =2𝒊 =2𝒓. - The rotation of the mirror can be either in an
- Glanging angle or angle of glance 𝒈 is the anticlockwise direction or clockwise direction
angle between the incident ray and the mirror or depending on the one described in a particular
the angle between the reflected ray and the question.
mirror. 𝒈 + 𝒊 = 90°, 𝒈 + 𝒓 = 90°. - For anticlockwise rotation of mirrors :
- Angle of deviation 𝑫 is the angle through 1.The new angle of incidence or reflection after
which the incident ray has deviated from its rotation = 𝒊 + 𝜽.
original path after being reflected. 2.The new angle between the incident and
𝑫 = 180° − (𝒊 + 𝒓) =180° −2𝒊 = 180° −2𝒓. reflected ray = 2(𝒊 + 𝜽).
- Angle of deviation is twice the glancing angle or - For clockwise rotation of mirrors :
angle of glance. 𝑫 = 2𝒈. 1.The new angle of incidence or reflection after
- If a ray is incidented perpendicularly or rotation = 𝜽 − 𝒊 .
normally on the mirror, the angle of incidence 2.The new angle between the incident and
and angle of reflection are 0°, as the ray reflected ray =2(𝜽 − 𝒊).
follows through the same path. - The angle between the new reflected ray after
- Images formed by a plane mirror: rotation and the normal before rotation in both
1.The image is virtual – behind the mirror and clockwise and anticlockwise rotation of the
cannot be caught on a screen. mirror = 𝒊 +2𝜽. 𝑖=initial incident angle, 𝜃=angle
2.Erect or upright.
183
Demystified Series Physics Demystified by Dr Timothy
of rotation of the mirror. magnified image.
- The displacement of the beam of light due to 5.Object at 𝐹(𝑢 = 𝑓) – image at infinity (𝑣 = ∞),
rotation of the mirror from its initial position on the nature of the image cannot be captured e.g
a screen is given by – 𝐭𝐚𝐧 𝟐𝜽 = 𝑩 ,
𝑫 car-head lamps, torchlights and searchlight.
𝑫𝑴𝑺
6.Object between 𝐹 and 𝑃 or mirror (𝑢 < 𝑓) –
𝐷𝐵 =Displacement of beam or spot on the screen, image is formed behind the mirror e.g dentist
𝐷𝑀𝑆 =distance between the mirror and screen. mirror, make-up or shaving mirror.
- Applications of plane mirrors –
- If the object is beyond 𝑪,the image is always
1.Simple periscope – Two plane mirrors are diminished (𝒎 <1).
placed parallel to each other i.e at 0° and each - If the object is at 𝑪,the image is same size as
of the mirrors is inclined at 45° to the wall. It is theobject (𝒎 =1).
used for observing above or below obstacles or - If the object is behind 𝑪,the image is always
normal view of vision. The major disadvantage of magnified (𝒎 >1).
a simple periscope is that it produces infinite or - Concave mirrors are used shaving mirrors
multiple images. when the object is placed close to the face or at a
2.Sextant – Used in navigation and in measuring distance less than the focal length.
the angle of elevation of the sun. - Parabolic mirrors are concave mirrors which
3.Kaleidoscope – Two plane mirrors are
produces parallel beam of constant intensity
inclined at 60° to each other. It is used in textile when a small light source is placed at its focus. It
industries for producing colour patterns and in is used in searchlights, car headlamps and torch
producing symmetrical patterns when pieces of lights.
coloured glasses acting as object are arranged by - A concave mirror used for shaving should be
shaking the tube. narrow while a concave mirror used for head
4.Mirror galvanometer – A galvanometer with lamps should be wide.
no physical pointer but with the principle of - Convex or Diverging mirror – The inner
rotation of reflected ray surface is coated with silver while the outer
5.Theatrical stage – Optical illusion e.g a man surface is the reflecting part.
walking with no head or a candle burning inside
- The closer the object to the mirror, the closer
a bottle filled water. the image to the mirror (𝑢 ∝ 𝑣).
- The image formed by a convex mirror is always
● Reflection from Spherical/curved virtual,erect and diminished (VED) and between
surface the principal focus and the pole of the mirror, at
- Concave or Converging mirror – The outer all object positions.
surface is coated with silver while the inner - Convex mirrors are used in driving mirros,
surface is the reflecting part. monitoring mirros and security mirrors in super-
- The nature of image formed depends on the markets, as it gives a wide field of view of objects
object position. As the object moves closer to the within a large angle and forms an erect image of
mirror or approaches the principal focus, the the object.
image becomes bigger and moves away from the - Rules for constructing ray diagram :
1 1
mirror (𝑢 ∝ , 𝑢 ∝ , 𝑣 ∝ 𝐻𝐼 ). 𝒊 = 𝒓, 𝒊 =q, 𝒓 =q
𝑣 𝐻𝐼 𝒊
𝒒𝒓
- Real images are formed on a screen and are 𝑪 𝑭
always inverted, are seen in the same side of the
mirror while Virtual image cannot be caught on a 1.Rays parallel to the principal axis are reflected
screen and are always erect or upright, are seen through are reflected through the principal focus.
behind the mirror. 2.Rays passing through the principal focus are
- 𝑭 = principal focus, 𝑪= centre of curvature, reflected parallel to the principal axis.
𝒇= focal length, 𝒓= radius of curvature, 3.Rays passing through the centre of curvature
𝑷= pole, 𝒎= magnification. are reflected back through the centre of
- Images formed by a concave mirror: curvature i.e at angle of incidence or reflection of
1.Object at infinity (𝑢 = ∞) – image at 𝐹(𝑣 = 𝑓), 0°.
real inverted and diminished image. 𝟏 𝟏 𝟏 𝒖𝒇
- Mirror formula : 1. + = . 2. 𝒗 = .
2.Object at 𝐶(𝑢 > 𝑟 or2𝑓) – image is between 𝐹 𝒖 𝒗 𝒇 𝒖−𝒇
𝒗𝒇 𝒓
and 𝐶(𝑓 < 𝑣 < 𝑟), real,inverted and diminished 3. 𝒖 = . 4. 𝒇 = .
𝒗−𝒇 𝟐
image.
- Magnification formula:
3.Object at 𝐶(𝑢 = 𝑟 or 2𝑓) – image at 𝐶(𝑣 = 𝒗 𝑯 𝒇 𝒓
𝑟 𝑜𝑟 2𝑓), real,inverted and same size. 1. 𝒎 = = 𝑰 . 2. 𝒎 = = .
𝒖 𝑯𝑶 𝒖−𝒇 𝟐𝒖−𝒓
4.Object between 𝐹 and 𝐶(𝑟 > 𝑢 > 𝑓) – image is 𝒗
3. 𝒎 = − 1 =
𝟐𝒗
– 1.
beyond 𝐶 (𝑣 > 𝑟 or2𝑓), real,inverted and 𝒇 𝒓
- Points to note when calculating any question

184
Demystified Series Physics Demystified by Dr Timothy
on mirrors: the focus.It is prevented by using a parabolic
1.Object distance 𝑢 is positive for both concave mirror.
and convex mirror.
2.Image distance 𝑣 is positive for real and Examples :
inverted images and negative for virtual and 1. A luminous object is one that A. gives off dim
erect images. blue-green light only in the dark B. gives out
3.Focal length 𝑓 of concave mirror is positive light of its own C. shines by reflected light only
while that of convex mirror is negative. D. glows only in the presence of light.
4.Magnification 𝑚 is positive for real and 2. Which of the following is not self luminous?
inverted images and negative for virtual and A.incandescent electric bulb B.the moon C.the
erect images. sun D.lighted candle E.incandescent fluorescent
5.The distance between an object and its image is tube
given by : 𝑫 = 𝒖 ± 𝒗. It is positive for virtual The moon is a non-luminous object, hence it
images and negative for real images. depends on light falling on it from a luminous
- Graphical illustration of mirror formulas object. Sun,incandescent electric bulb and
(this also apply to lenses) – This shows the fluorescent bulb are luminouse object.
graphical relationship between the variables in 2.Which of the following is not a luminous
mirror formula. object? A.the sun B.the moon C.a star D.a lighted
𝟏 𝟏 candle E.a lighted bulb.
𝒗 𝒖
3. I.Moon II.Sun III.Street light IV.Stars. Which of
the above is a natural source of light? A.II, III and
𝟏 𝟏 IV B.I,II and III C.III and IV D.II and IV
𝒖 𝒗 Moon is a non-luminous object. Street light is an
- The intercept on either axis, of both graphs artificial or man-made luminous source. Sun and
above is equal to the reciprocal of the focal length Stars are natural luminous sources.
𝟏
. Hence, the focal length is equal to the 4. Non-luminous objects can be seen because
𝒇
they A.are polished B.are near C.reflect light
reciprocal of the intercept.
D.emit light
𝒖𝒗 𝒎
Non-luminous objects are seen by the light they
reflect. Luminous objects emit light.
5. What happens to parallel beam of light?
𝒖+𝒗 𝒗
𝒇 A.diverge as they travel B.they meet at infinity
−1
C.they intersect D.they converge as they travel
- The slope of the graph of 𝑢𝑣 against 𝑢 + 𝑣, is
The light rays of a parallel beam are parallel to
equal to the focal length 𝒇, 𝒖𝒗 = 𝒇(𝒖 + 𝒗).
one another and they do not meet or intersect.
- The slope of the graph of 𝑀 against 𝑣 is equal to
𝟏 The light rays of a convergent beam
the reciprocal of the focal length , the intercept converge(meets) at a point as they travel while
𝒇
on 𝒎-axis is equal to −1, while the intercept of that of a divergent beam diverge(spread out from
𝒗-axis,is equal to focal length. the source) as they travel.
- Comparison between plane and convex 6. Which of the following statements supports
mirror in driving mirrors : the assumption that light travels in a straight
Plane mirror Convex mirror lines? A.light can be diffracted B.a source of light
1.Image of same size Diminished image produces distinct shadows of opaque objects
as the object i.e image are smaller C.a source of light produces interference patterns
(Advantage) than object on suitably placed screen D.light can be refracted
(Disadvantage). Rectilinear propagation of light is a phenomenon
2.Lateral inversion No lateral inversion which states that light travels in a straight line. It
(Disadvantage) (Advantage). is applied in the formation of shadows and
3.Correct distance of Correct distance of eclipse and in the operation of a pinhole camera.
image can be image can’t be 7. Shadow and eclipse result from the
estimated i.e image estimated,as the A.Refraction of light B.Reflection of light
distance equals image seems farther C.Rectilinear propagation of light D.Diffraction
objects distance way (Disadvantage). of light.
(Advantage) 8. Light travels in straight lines. In which of the
4.Erect image Erect image. following is this principle manifested? I.pin hole
- Spherical aberration in mirrors is the inability camera II.formation of shadows III.diffraction
of a curved mirror to reflect wide beam or rays of iv.occurence of eclipse A.I and III B.II and III C.I,II
light that are far from the principal axis through and III D.I,II and IV E.I,II,III and IV .
a common focus but through a point nearer than 9. The sharpness of the boundary of the shadow

185
Demystified Series Physics Demystified by Dr Timothy
of an object is determined by the A.rays of light the moon B.in the umbra produced by the earth
passing through the object B.intensity of light C.in the penumbra produced by the moon
striking the object C.opacity of object D.nature D.beyond the vertex of the umbra cone of the
of the object moon.
A bright extended source of light will gives less Annular eclipse is seen on earth beyond the
sharp and brighter shadow while a small or point intersectiong of the extreme lines required for
source of light gives a sharper shadow. total eclipse or umbra i.e beyond the the vertex of
10. I.Total internal reflection of light the umbra cone.
II.Conservation of light energy III.Relative 17. The image in a pinhole camera is A.erect and
motion of the earth,sun amd moon IV.Rectilinear formed by refraction through a lens B.virtual and
propagation of light. Which of the above is a formed by dispersion C.erect and gets sharper as
phenomenon of total solar eclipse? A.I and IV B.II the hole becomes larger D.inverted and formed
and IV C.I and III D.III and IV by the light from each point travelling in a
Eclipse is as a result of rectilinear propagation of straight line
light. Eclipse of the sun or moon is based on the The image in a pinhole camera is real and
position of the sun,moon and earth relative to inverted. It is formed due to rectilinear
one another. propagation of light.
11. An eclipse of the sun may occur when A.the 18. If the size of the table of a pinhole camera is
sun passes between the moon snd the earth increased, the image formed becomes
B.the moon passes between the earth and the A.brighter and blurred B.brighter and larger
sun C.the earth passes between the moon and the C.brighter and sharper D.blurred and duller
sun D.the moon and the earth rotate together E.a E.blurred and larger
part of the earth rotates away from the sun Increasing the holes of a pinhole camera
Solar eclipse or eclipse of the sun occurs when increases the amount of light entering it hence
the moon passes between the earth and the sun, the brightness increases leading to overlapping
such that the shadow of the moon is cast on the images. Thr image then becomes brighter,less
earth. sharp and blurred.
12. The eclipse of the sun occurs when the 19. The pin-hole camera is less sharply defined
A.moon is note completely hidden in the earth’s image when the A.pin-hole is larger
shadow B.moon’s umbra falls on some part of the B.illumination is less C.screen is further from the
earth C.moon is between the sun and the earth pin-hole D.object is further from the pin-hole.
D.earth is between the sun and the moon. Less-sharply defined or blurred image is due to
13. Total eclipse of the sun occurs when the increased brightness when pin-hole is larger .
A.earth is between the moon and the sun B.sun is 20. What is the effect of the increase in the size of
between the moon and the earth C.moon is the hole of a pin-hole camera on the image? It
between the sun and the earth D.ozone layer A.gives a blurred image B.corrects chromatic
threathened. aberration C.magnifies the image D.brings the
14. Moon image into a sharper.
Sun Earth 21. As an object moves towards a pin-hole
𝑋 camera,the image becomes A.brighter and
When the sun,the moon and the earth are as smaller B.less brighter and bigger C.brighter but
showm in the diagram above, an observer remains the same size D.less bright and
standing at 𝑋 is in A.penumbra region and sees a diminishing in size E.brighter and bigger
partial eclipse B.penumbra region and sees a Object moving towards or closer to the pinhole
total eclipse C.umbra and sees a partial eclipse camera decreases the object distance hence the
D.umbra and sees a total eclipse. image height increases i.e becomes larger as
Partial eclipse is seen at the edges or penumbra 𝑖𝑚𝑎𝑔𝑒 ℎ𝑒𝑖𝑔ℎ𝑡 ∝
𝑖𝑚𝑎𝑔𝑒 𝑑𝑖𝑠𝑡𝑎𝑛𝑐𝑒 𝑣
, 𝐻𝐼 ∝ . Object
region while total eclipse is seen at the centre or 𝑜𝑏𝑗𝑒𝑐𝑡 𝑑𝑖𝑠𝑡𝑎𝑛𝑐𝑒 𝑢

umbra region. moved towards the pin hole reduces the


15. Annular eclipse occurs when the A.earth is sharpness of the image and increases the
between the sun and the moon B.moon is brightness.
between the sun and the earth C.extreme rays 22. If the distance between the object of a pinhole
of light from the sun intersect after reaching the camera is reduced by half, the size of the image of
earth D.sun is between the moon and the earth. the object A.is doubled B.is quadrupled
In annular eclipse the extreme rays of the sun C.remains the same D.is halved
𝑖𝑚𝑎𝑔𝑒 𝑑𝑖𝑠𝑡𝑎𝑛𝑐𝑒 𝑣 1
required for total eclipse intersect before 𝑖𝑚𝑎𝑔𝑒 ℎ𝑒𝑖𝑔ℎ𝑡 ∝ , 𝐻𝐼 ∝ , ∴ 𝐻𝐼 ∝ ,
𝑜𝑏𝑗𝑒𝑐𝑡 𝑑𝑖𝑠𝑡𝑎𝑛𝑐𝑒 𝑢 𝑢
reaching the earth not after reaching the earth. when the object distance is halved, then image
16. An annular eclipse of the sun is seen by an size or height 𝐻𝐼 =
1
= 2 i.e doubled.
observer at a point A.in the umbra produced by 1/2

186
Demystified Series Physics Demystified by Dr Timothy
22. A man 1.5m tall is standing 3m in front of a 𝑟=25°, angle of deviation 𝐷=?, 𝐷 = 180° − 2𝑟,
pinhole camera whose distance between the hole 𝐷 = 180° − 2×25° = 180° − 50° = 130°.
and the screen is 0.1m. What is the height of the 30. A ray of light incident on a plane mirror is
man on the screen? A.0.05m B.0.15m C.0.30m inclined to the mirror at an angle of 50°. What is
D.1.00m the angle of deviation of the ray? A.50° B.80°
object height 𝑂𝐻 =1.5m, object distance 𝑂𝐷 =3m, C.90° D.100° .
image distance 𝐼𝐷 =0.1m, image height 𝐼𝐻 =?, 𝑔=50°, 𝐷 = 2𝑔 = 2×50° = 100° . Alternatively,
𝑚=
𝑖𝑚𝑎𝑔𝑒 ℎ𝑒𝑖𝑔ℎ𝑡 𝐻𝐼
=
𝑖𝑚𝑎𝑔𝑒 𝑑𝑖𝑠𝑡𝑎𝑛𝑐𝑒 𝑣
, 𝑖 = 90° − 𝑔 = 90° −50° = 40°, 𝐷 = 180° − 2𝑖 ,
𝐻𝐼
𝑜𝑏𝑗𝑒𝑐𝑡 ℎ𝑒𝑖𝑔ℎ𝑡 𝐻𝑂
𝑣 𝐻𝐼 0.1
𝑜𝑏𝑗𝑒𝑐𝑡 𝑑𝑖𝑠𝑡𝑎𝑛𝑐𝑒 𝑢
0.1×1.5
𝐷 = 180° − 2×40° = 180° − 80° = 100°.
= , = , 𝐻𝐼 = = 0.05m. 31. An engineer intends to deviate a light ray
𝐻𝑂 𝑢 1.5 3 3
23. An object of height 4cm is placed in front of a from its path 120° through reflection from plane
cuboid pinhole camera of size 6cm. If the image mirror. Calculate the angle of incidence A.20°
formed is 2cm high, how far is the object from the B.30° C.40° D.60°.
pinhole? A.3.0cm B.8.0cm C.12.0cm D.16.0cm 𝐷=120°, 𝑖=?, 𝐷 = 180° − 2𝑖 , 2𝑖 = 180° − 𝐷 ,
60°
𝐻𝑂 =4cm, 𝐿=6cm, 𝐻𝐼 =2cm, 𝑢=?, 2𝑖 = 180° − 120° = 60° , 𝑖 = = 30°.
𝑖𝑚𝑎𝑔𝑒 ℎ𝑒𝑖𝑔ℎ𝑡 𝐻𝐼 𝐿𝑒𝑛𝑔𝑡ℎ 𝑜𝑓 𝑐𝑎𝑚𝑒𝑟𝑎 𝐿 2
𝑚= = , 32. 𝐼 The diagram above shows a ray of
𝑜𝑏𝑗𝑒𝑐𝑡 ℎ𝑒𝑖𝑔ℎ𝑡 𝐻𝑂 𝑜𝑏𝑗𝑒𝑐𝑡 𝑑𝑖𝑠𝑡𝑎𝑛𝑐𝑒 𝑢
𝐻𝐼 𝐿 2 6 6×4 light 𝐼𝐾 incident on plane mirror at
= , = , 𝑂𝐷 = = 12cm.
𝐻𝑂 𝑢 4 𝑂𝐷 2 𝐾. Calculate the angle of deviation
24. An object 2.00m high is placed at a distance Plane mirror 𝐾 on of the ray after reflection
of 5.00m from a pin-hole camera.If the image A.35° B.55° C.70° D.105° E.145°.
height is 4.00cm,what is the distance of the film 𝑔=35°, 𝐷 = 2𝑔 = 2×35° = 70°.
from the pin hole? A.2.50cm B.5.00cm 33. Which of the following statements is not true
C.10.00cm D.20.00cm E.100.00cm . of the image in a plane mirror? The image is A.the
𝐻𝐼 =4cm, 𝐻𝑂 =2m=200cm, 𝑢=5m=500cm, 𝑣=?, same size as the object B.laterally inverted
𝐻 𝑣 4 𝑣 4×500
𝑚= 𝐼 = , = , 𝑣= = 10.00cm. C.virtual D.magnified E.the same distance
𝐻𝑂 𝑢 200 500 200
behind the mirror as the object is in front
25. Which of the following cannot be explained
The image formed by a plane mirror is virtual,
by using the laws of reflection? A.Formation of
erect or upright,laterally inverted,same size as
images by plane mirrors B.Formation of images
the object and the same distance behind the
by curved mirrors C.Action of the periscope
mirror as the object in front. The image of a plane
D.Formation of shadows E.None of the above
mirror is same size as the object, hence can not
Formation of shadows is explained by rectilinear
magnified.
propagation of light and not by reflection.
34. Which of the following does not describe the
26. A ray of light makes an angle of 35° with a
image formed by a plane mirror? A.Erect
plane mirror. What is the angle of reflection?
B.Laterally inverted C.Same distance from the
A.55° B.35° C.70° D.65°.
mirror as object D.Magnified E.Virtual.
The angle a ray of light makes with the mirror is
35. Which of the following diagrams shows the
the glancing angle or angle of glance 𝑔.
correct image of the letters 𝐿𝐹 as seen through a
𝑔=35°, angle of reflection 𝑟=?, 𝑖 + 𝑔 = 90° or
plane mirror.
𝑟 + 𝑔 = 90°, 𝑟 = 90° − 𝑔 = 90° − 35° = 55°.
A.LF FL B.LF ꟻL C.LF ˩ꟻ D.LF ꟻ˩
27. A reflected ray of light makes an angle of 27°
Images formed by a plane mirror is laterally
with the surface of a plane mirror, what is the
inverted i.e truned through 180° or the left side
angle of incidence of the ray? A.27° B.54°C.63°
of an object turns to the right and the right turns
D.117° E.153°
to the left.
𝑔=27°, angle of incidence 𝑖=?, 𝑖 = 90° − 𝑔,
36. S 45° PQ is a thin rod on a horizon-
𝑖 = 90° − 27° = 63°.
T table,RS is plane mirror incli-
28. A ray of light strikes a plane mirror at a
P Q ned at 45° to the horizontal
glancing angle of 50°. Calculate the angle
R as shown in the figure. The
between the incident and reflected rays A.20°
image of PQ as seen in the mirror by the eye at T
B.40° C.50° D.80° E.100°.
is A.Horizontal B.Parallel to the mirror C.At
𝑔=50°, angle between the incident and reflected
infinity D.Vertical E.Highly magnified.
rays 𝜃=?, 𝑖 = 90° − 𝑔 = 90° − 50° ,
On drawing the ray diagram the distance of each
𝑖 = 40°, 𝑖 = 𝑟 = 40° , angle between the incident
point of the object in front of the mirror is same
and reflected rays 𝜃 = 𝑖 + 𝑟 ,
as the the distance from its respective image
𝜃 = 40° + 40° = 80°.
behind the mirror.
29. A ray of light is incident on a plane mirror S 45° Hence, the virtual image of
such that the angle of reflection is 25°. What is the
T the rod same size as the
angle of deviation of the ray after reflection from
object, appears vertical.
the mirror? A.25° B.50° C.65° D.115°E.130°
P Q R
187
Demystified Series Physics Demystified by Dr Timothy
37. The magnification of an object 2cm tall when when the mirror moves 1m towards the man, the
placed 10cm in front of a plane mirror is A.6.0 new object distance 𝑢 = 4−1=3m. New image
B.1.0 C.0.7 D.0.6. distance 𝑣=3m. The distance between the man
Object height 𝐻𝑂 =2cm, object distance 𝑢=10cm, and his image = 𝑢 + 𝑣 = 3+3 = 6m.
In a plane mirror, object height equals image 43. A man at the back of a crowd watches a
height and object distance equals image distance. parade by holding a plane mirror just above his
Thus, image height 𝐻𝐼 =2cm, image distance head. The parade passes 6m behind his head and
𝐻
𝑣=10cm. Magnification 𝑚 = 𝐼 = = 1.
𝑣
the mirror is 0.25m in front of the man. How far
𝐻𝑂 𝑢
does the image in the mirror appear to be from
38. A boy looks at the image of an object in a
the man? A.6.00m C.6.25m C.6.50m D.6.75m
plane mirror. He observes two images, a main
bright one and the other faint. The observed 6m 0.25m 0.25m 6m
image result from A.Reflection only P M MI PI
B.Refraction only D.Reflection and refraction P=parade, M=man, MI=man’s image, PI=parade’s
D.Diffraction and interference. image. The distance of the image of the parade in
Images formed in mirror are due to reflection. the crowd from the man = PI to M =
Multiple images are formed in mirrors due to the 6+0.25+0.25 = 6.50m.
thickness of the mirror. Refraction takes place in 44. If a boy walks away from a plane mirror at a
lenses and glasses. Reflection and refraction can speed of 2ms-1, his image moves from him at a
take place in thick glasses, hence the reason for speed of A.4ms-1 B.3ms-1 C.2ms-1 D.1ms-1
multiple images.
39. Which of the following descriptions of the 𝑥 𝑥
image of a real object formed by a plane mirror is O2 O1 I1 I2
not correct? It is A.same size as the object If the body walks away from the mirror from O1
B.laterally inverted C.virtual D.at the same to O2 at 𝑡s his distance from the mirror , 𝑥 = 2𝑡m,
distance behind the mirror as the object in front distance moved by image from I1 to I2, 𝑥 = 2𝑡m.
E.formed on a screen Distance moved by the the image from the man =
Plane mirrors forms virtual images. A virtual 2𝑡 + 2𝑡 = 4𝑡m.
image is one through which light rays do not Speed at which the image moves way from him =
actually pass through but is still visible to the eye 4𝑡
= 4ms-1.
and cannot be formed on a screen. A real image is 𝑡
one through which light rays actually pass 45. An object 8cm in front of a plane mirror
through and can be caught on a screen. moves at a speed of 6cms-1 away from the mirror.
40. A barber, standing at a distance of 2m from a What will be the distance separating the object
large plane mirror, walks 1m farther way from and the image after 2s? A.8cm B.12cm C.20cm
the mirror. How far is his image now from him D.28cm E.40cm
A.2m B.3m C.4m D.5m E.6m With a speed of 6cms-1, distance moved after 2s
= 𝑣𝑡 = 6×2 = 12cm.
1m 2m 2m 1m
F.P I.P M I.PI F.PI 12cm 8m 8cm 12cm
F.P=final position, I.P=initial position, M=mirror, Distance separating the object and the image =
I.PI=Initial position’s image, F.PI=final position’s 12+8+8+12 = 40cm.
image. Object distance in a plane mirror is equal 46. An object is placed in front of two plane
to the image distance. mirrors inclined at 60° to each other. Determine
The distance of the barber from his image = F.PI the number of images formed A.2 B.3 C.5 D.7
360° 360°
to F.P = 1+2+2+1 = 6m. 𝜃=60°, 𝑛=?, 𝑛 = −1= − 1 = 6−1 ,
𝜃 60°
41. An object is placed 10cm in fornt of a plane 𝑛 = 5 images.
mirror. If it is moved 8cm further away from the 47. An object is positioned between two plane
mirror, determine the distance of the final image mirros inclined at right angles to each other.The
from the mirror A.2cm B.9cm C.16cm D.18cm object is 1 unit distance from each mirror.The
number of images formed is A.1 B.3 C.2 D.5 E.4
8cm 10cm 10cm 8cm 360°
F.O I.O M I.I F.I 𝜃=90° i.e at right angles, 𝑛=?, 𝑛 = −1,
𝜃
360°
Distance of the final image from the mirror = F.I 𝑛= − 1 = 4−1 = 3 images.
90°
to M = 8+10 = 18cm. 48. A man standing between two parallel mirrors
42. A man stands 4m in front of a plane mirror If in a barber’s shop will see the following number
the mirror is moved 1m towards the man, the of his own image A.Eight B.Two C.Four D.One
distance between him and his new image is A.3m E.Infinite
B.5m C.6m D.10m. 360°
Initial object distance from the mirror=4m, 𝜃=0° i.e parallel, 𝑛=?, 𝑛 = −1,
𝜃

188
Demystified Series Physics Demystified by Dr Timothy
𝑛=
360°
− 1 = 𝛼 − 1 = 𝛼 i.e infinite. 54. Y In the fugure an incident

ray XY makes an angle of
49. Two plane mirrors are inclined to each other
15° 15° with a fixed lines AY which
such that an object placed between them has 11
is normal to the surface of a
images. Determine the angle of inclination A.30°
plane mirror. If the mirror is
B.45° C.60° D.90°
360° 360° X A turned as indicated by the arrow
𝑛=11 images, 𝜃=?, 𝑛 = − 1 , 11 = −1, through 40°, what angle will the reflected make
𝜃 𝜃
360° 360°
= 11+1 = 12, 𝜃 = , 𝜃 = 30°. with AY? A.25° B.40° C.15° D.55° E.95°
𝜃 12
50. An incidenet ray is made to travel in the The arrow indicates anticlockwise rotation.
opposite direction parallel to itself by means of Angle of incidence before rotation 𝑖=15°, angle of
two mirrors 𝑀1 and 𝑀2 set at an angle 𝜃 as shown rotation of the mirror 𝜃=40°, For anticlockwise
in the diagram below rotation: Angle the reflected ray after rotation
𝑀1 makes with the normal AY before rotation = 𝑖 +
2𝜃 = 15° + 2×40° = 15° + 80° = 95°.
𝜃 55. In the diagram below, a ray of light is incident
on a plane mirror at an angle of 30°,
direction of rotation
𝑀2
What is the magnitude of 𝜃? A.30° B.45° C.60° Calculate the new angle of
D.90° E.120° . 30°
incidence if the mirror is
𝑀1 𝜃 + 𝜃 + 𝜃 = 180° , rotated through 10°
𝜃 3𝜃 = 180° , A.10° B.20° C.30° D.40° E.60°.
𝜃 𝜃 = 180/3 = 60°. For anticlockwise rotation of mirror :The new
𝜃
𝜃 angle of incidence or reflection = 𝑖 + 𝜃, 𝑖=30°,
𝜃 𝜃=10°, New angle of incidence = 30° + 10° = 40°.
𝑀2 56. A ray of light is incident normally on a plane
51. Two mirrors 𝑚1 , 𝑚2 are mirror. If the incident ray is kept fixed while the
inclined at right angles as mirror is rotated through an angle of 30°,
shown above. Calculate the determine the initial and final angle of reflection
30° the angle of reflection of respectively. A.0° and 60° B.90° and 30° C.0° and
the ray at 𝑚2 30° D.90° and 60°.
A.30° B.45° C.60° D.90° . When a ray is incidented normally or
Angle of incidence of 𝑚1 , 𝑖1 =30°. For two mirrors perpendiculaly on a plane mirror, the angle of
inclined at right angles, the angle of incidence of incidence or the angle of reflection is 0°, 𝑖 = 𝑟 =
𝑚1 is equal to the glancing angle of 𝑚2 while the 0°. When the mirror is rotated through 30°, the
angle of incidence of 𝑚2 is equal to glancing angle new angle of reflection is equal to 2×30° = 60°.
of 𝑚1 . 𝑖1 = 𝑔2 = 30°, 𝑟 = 90° − 𝑔 , 𝑟2 = 90° − 𝑔2 Thus, the initial and final angle of reflection are
𝑟2 = 90° − 30° = 60°. 0° and 60° respectively.
52. P 57. A ray OP is incident at an angle of 25° on a
plane mirror as shown in the diagram below.
O P
40° Q
N
25°
Two mirrors of the same length are arranged as O
shown in the diagram below. A ray of light NO If the mirror is turned through 40° in the
strikes the system at O and emerges along PQ. direction indicated, the angle between the
The emergent ray has been deviated through. incident ray and the new relected ray is A.30°
A. 230° B. 220° C. 210° D. 200° E. 180°. B.50° C.80° D.130°.
The angle of deviation of the emergent ray by two For clockwise rotation of a mirror: The new angle
mirrors inclined at 90° to each other is always of incidence or reflection = 𝜃 − 𝑖 and The angle
180°, regardless of the angle of incidence. between the incident ray and the new reflected
53. A ray of light strikes a plane mirror at an ray = 2(𝜃 − 𝑖), 𝜃=40°, 𝑖=25°.
angle of 35°. If the mirror is rotated thrpugh 10°, The angle between the incident ray and the new
through what angle is the reflected ray rotated? reflected ray = 2(40° − 25°) = 2(15°) = 30°.
A.70° B.45° C.25° D.20° 58. Y In the diagram above, an
If a mirror is rotated through angle 𝜃, the incident ray AY makes an
reflected ray is rotated through an angle 2𝜃. 30° angle of 30° with the nor-
Angle of rotation of the mirror 𝜃=10°, mal XY. If the mirror is
The reflected ray is rotated through angle = 2𝜃 = A Y rotated anticlockwise about Y
2×10 = 20°.

189
Demystified Series Physics Demystified by Dr Timothy
through an angle of 20°, while AY is fixed, what usually placed A.At angle of 60° B.Parallel to one
angle will the reflected ray now make with the another C.Perpendicular to one another D.At an
incident ray? A.70° B.80° C.100° D.120°. angle of 45°
For anticlockwise rotation: The angle between A kaleidoscope consists of two plane mirrors
the incident ray and new reflected ray =2(𝑖 + 𝜃), inclined at 60° to each other. It is used in textile
𝑖=30°, 𝜃=20° , The angle between the incident industries for obtaining coloured patterns.
ray and the new reflected ray = 2(30° +20°) = 63. The instrument used by designers to obtain
2(50) = 100°. different colour patterns is called A.episcope
59. A plane mirror gives a spot of light on a B.periscope C.kaleidoscope D.sexant.
screen which is 3m from a mirror. The screen is 64. The instrument used for measuring the angle
perpendicular to the initial ray of light. When the of elevation of the sun is called A.Altimeter
mirror is rotated,the spot of light moves a B.Kaleidoscope C.Periscope D.Sexant
distance of 4m across the screen. Calculate the E.Telescope.
angle of rotation of the mirror. A.26.6°B.53.1° Sexant is used in navigation for measuring the
C.36.9° D.18.5° angle of elevation of the sun. Its operation is
The reflected gives a spot of light on the based on the principle of rotation of plane
screen and on rotation of the mirror the spot of mirrors.
light is slightly displaced. 65. Which one of the following is common to both
𝜃 the sextant and the periscope? They both
A.depend on the laws of refraction B.employ
2𝜃 converging lenses of short focal lengths C.use
3m totally reflecting prisms D.depend on law of
4m Screen reflection of light E.are used to measure the
Displacement of beam or spot on the screen angle of elevation.
𝐷𝐵 =4m, distance between the mirror and screen Sexant and periscope are both made of plane
𝐷 4
𝐷𝑀𝑆 =3m, tan 2𝜃 = 𝐵 = = 1.333, mirrors, hence they depend on law of reflection
𝐷𝑀𝑆 3
53.12 of light. Sexant is used to measure the angle of
2𝜃 = 𝑡𝑎𝑛−1 1.333 = 53.12°, 𝜃 = = 26.6° elevation of the sun while periscope is used for
2
60. An incident ray is reflected normally by a viewing objects above or below an obstacle.
plane mirror onto a screen where it forms a 66. The inside portion of part of a hollow metal
bright spot.The mirror and screen are parallel sphere of diameter 20cm is polished.The portion
and 1m apart.If the mirror is rotated through will therefore form A.concave mirror of focal
5°,calculate the displacement of the spot A.8.7cm length 5cm B.concave mirror of focal length
B.10.0cm C.15.4cm D.17.6cm. 10cm C.convex mirror of focal length 5cm
𝜃 = 5° D.convex mirror of focal length 20cm
Diameter of the sphere 𝑑=20cm, radius of
2𝜃 𝑑 20
curvature of the sphere 𝑟 = = = 10cm.
2 2
1m
The focal length of the mirror obtained from the
𝐷𝐵 Screen 𝑟
sphere is half the radius of curvature. 𝑓 = =
Distance between the mirror and screen 2
10
𝐷𝑀𝑆 =1m=100cm, angle of rotation of the mirror = 5cm. The mirror is a convex mirror as the
2
𝜃=5°, displacement of beam or spot on the screen inner potion is polished.
𝐷
𝐷𝐵 =?, tan 2𝜃 = 𝐵 , 𝐷𝐵 = 𝐷𝑀𝑆 tan 2𝜃 , 67. An image formed on a screen is always
𝐷𝑀𝑆
𝐷𝐵 = 100tan 2(5°) = 100tan 10° = 100×0.176 , A.inverted B.magnified C.upright D.virtual
𝐷𝐵 = 17.6cm. Real images can be caught on a screen and they
61. The simple periscope is an optical instrument are always inverted. Virtual images are cannot be
which I.comprises two parallel plane mirrors, caught ona screen and are always erect or
suitably arranged II.magnifies images of objects upright. Realand virtual images can both either
III.is used for viewing behind obstacles. Which of be magnified or diminished.
the statements above is/are correct? A.I,II and III 68. Which of the following is applicable to a real
B.I and III C.I and II D.II E.I. image formed by a concave mirror? I.It can be
A simple periscope is made of plane mirrors, observed on a screen II.It is always inverted and
whose magnification is unity i.e not a magnifier. in front of the mirror III.It only seems to exist
The two plane mirros are parallel i.e at 0° to each IV.It is formed by the actual converging of rays of
other. It is used for viewing objects over or light A.I and II B.I and III C.I,II and III D.I,II and
behind barriers/obstacles. Simple periscopes are IV E.All four statements
used by soldiers in armoured tanks. Real images can be observed or caught on a
62. Two plane mirrors in a kaleidoscope are screen, are always inverted and in front of the
mirror i.e same side of the mirror with the object.

190
Demystified Series Physics Demystified by Dr Timothy
They are formed by the actual intersection of distance equal to the focal length or half the
light rays. radius of curvature will produce parallel beam of
69. Which of the following statements is/are light, so that the image is at infinity.
correct for light reflected from a spherical 73. At what position will an object be placed in
mirror? I.incident rays parallel to the principal front of a concave mirror in order to obtain an
axis pass through the centre of curvature after image at infinity? A.At centre of curvature
reflection II.rays through the centre of curvature B.Between the principal focus and the centre of
are returned along the same path III.rays through curvature C.At the pole of the mirror D.At the
the principal focus are sent parallel to the principal focus.
principal axis A.I B.I and III C.II and III D.I and II When an object is placed at the principal focus of
E.I,II and III a concave mirror the image is formed at infinity.
Rays of light incident parallel to the principal axis 74. A parallel beam of light can be produced by
are reflect through the principal focus. Rays of placing a small bright source of light A.at the
light incident through the principal focus are principal focus of a converging mirror B.at the
reflected parallel to the principal to the principal principal focus of a diverging mirror C.at the
axis. Rays of light incident through the centre of centre of curvature of a converging mirror D.at
curvature are reflected along the same path. the centre of curvature of a diverging mirror
70. A ray of light which passes through the E.one diameter away from a circular plane
centre of curvature of a concave mirror is mirror.
reflected by 75. Where on the principal axis of a concave
the mirror at an angle of A.0°B.45° C.90° D.180° mirror must an object be placed to obtain its
A ray of light passing through the centre of image at the centre of curvature A.At the centre
curvature is reflected along the same path i.e at of curvature B.Between the centre of curvature
angle 0°. and the principal focus C.At the principal focus
6. After reflection from the concave mirror, rays D.Between the focus and the pole E.At infinity.
of light from the sun converges A.at the radius of When an object is placed at the centre of
curvature B.at the focus C.beyond the radius of curvature of a comcave mirror a real, inverted
curvature D.between the focus and the radius of image same size of the object is formed at the
curvature. centre of curvature. Magnification is unity 𝑚=1 in
Rays of light from the sun are at infinity, hence this condition.
they converge at the principal focus of the 76. If an object stands at the centre of curvature
concave mirror after reflection. of a converging mirror, the image produced will
71. Heat from the sun is to be used to burn a piece be A.inverted and virtual C.inverted and real
of paper with the aid of a concave mirror of C.erect and virtual D.magnified and real
radius of curvature 32cm.To enable the paper to The image formed if an object is at the centre of
start burning in the building in the shortest curvature is real, inverted, same size as the object
possible time,the paper is coloured X and and at the centre of curvature.
positioned at a distance Y from the mirror.What 77. When an object is placed between the centre
are X and Y respectively? A.black and infinity of curvature and the focus of a converging mirror,
B.white and 32cm C.black and 16cm D.black and the image formed is A.real, dimisnihsed and
32cm E.white and 16cm Colouring the paper erect B.real, magnified and inverted C.real,
black will allow the sun to burn the paper as magnified and erect D.virtual, diminished.
black body are good absorber and radiator of An object placed between the centre of curvature
heat. Heat from the sun are parallel rays of light and the principal focus of a concave mirror will
from infinity and pass through the principal focus form an image beyond the centre of curvature
𝑟
, focal length of the mirror, 𝑓 = = =16cm.
32
which is real, inverted and magnified.
2 2
78. The image formed by a concave mirror is real,
Thus, the rays of light from the sun converges at
inverted and magnified, when the object is placed
the focus and the heat from it is absorbed by the
A.at the focus B.at the centre of curvature
paper painted black and is burnt.
C.beyond the centre of curvature D.between the
- Colouring the paper white will prevent the heat
centre of curvature and the focus.
from penetrating the paper,as white bodies are
79. When an object is placed very close to the
good reflectors.
pole of a concave mirror, the virtual image
72. In order to produce a parallel beam of light, a
obtained is A.Diminished and upright
lamp is placed at a distance from the concave
B.Diminished and inverted C.Enlarged and
mirror equal to A.the radius of curvature B.half
inverted D.Enlarged and upright.
the radius of curvature C.half the focal length
A virtual,erect or upright and magnified or
D.twice the focal length E.twice the radius of
enlarged image is formed by a concave mirror
curvature.
when the object is placed between the focus and
Placing a lamp at the principal focus i.e at a

191
Demystified Series Physics Demystified by Dr Timothy
the pole of the mirror. construction of a searchlight is the A.concave
80. An object O lies at a distance m in fornt of a mirror B.convex mirror C.spherical mirror
concave mirror of focal length 𝑓. If m< 𝑓, then the D.parabolic mirror.
final image obtained will be A.Virtual and Parabolic mirror produced wide parallel beam of
diminished B.Magnified and erect C.Real and light.
inverted D.Diminished and erect. 87. What type of mirrors are capable of
When object is placed at a distance m from a producing parallel beams of light such as those
concave mirror which is less then its focal length arising from the headlamps of a car?
𝑓 i.e between the principal focus and the mirror, A.cylindrical mirrors B.parabolic mirrors
a virtual,erect and magnified image is formed C.spherical mirrors D.plane mirrors.
behind the mirror. 88. 𝑋 The image of the object
81. A pencil is placed vertically between a 𝑂𝑋 formed by the mirror
concave mirror and its focal point. The image in 𝐶 𝐹 𝑂 𝑃 in the figure shown is
the mirror will be A.Real, inverted and A.real, upright and
diminished B.Virtual, inverted and enlarged diminished B.virtual, upright and magnified
C.Real, erect and enlarged D.Virtual, erect and C.virtual, upright and diminished D.real, inverted
enlarged. and magnified.
82. A real object object is placed at a distance Object placed between principal focus 𝐹 and the
𝑢cm in front of a curved mirror of focal length mirror, will form a virtual, erect or upright,
𝑓cm. If the image is upright and magnified, then magnified image which is behind the mirror or
the mirror is A.Convex and 𝑢 = 2𝑓 B.Concave the right of the pole 𝑃.
𝑓
and 𝑢 = 2𝑓 C.Convex and 𝑢 = D.Concave and 89. 𝑌 The image of the object
2
𝒇 𝑂𝑌 formed by the mirror
𝒖= . 𝐶 𝑂 𝐹 𝑃 in the figure shown is
𝟐
An upright or erect and magnified image is located A.between 𝐹 and
formed by a concave mirror when the objects 𝑃 B.to the right of 𝑃 C.to the left of 𝑪 D.between
distance is less than the focal length of the mirror 𝐶 and 𝐹 .
i.e between the focus and the pole. An object Object placed between the centre of curvature 𝐶
𝑓
distance 𝑢 = is less than the focal length 𝑓. and the principal focus 𝐹,will form real, inverted
2
83. A boy on looking into a mirror observed that and magnified image which is beyond 𝐶 i.e to the
his face appeared to have grown bigger. The boy left of 𝐶.
must have been looking at a A.convex mirror 90. In the diagram below, an incident ray, parallel
with his face at the focus B.concave mirror with and close to the prin cipal axis of a converging
his face between the focus and the mirror mirror, passes thrpugh the focus 𝐹 after
C.concave mirror with his face at the focus reflection. 𝑅 is the centre of curvature of the
D.convex mirror with his face between the focus mirror
and the mirror 𝑄 𝑃
𝑝
Object placed between the focus and the concave 𝑞
mirror will form a virtual, erect and magnified 𝑠 𝑟
image.
84. A concave mirror used as a shaving mirror Which of the following is correct? A.𝑞 = 𝑠 B.𝑝 =
should A.have a virtual focal length B.be narrow 𝑠 C.𝑟 = 𝑠 D.𝑝 = 𝑞 = 𝑠 E.𝒑 = 𝒒 = 𝒓 .
C.have a small curvature D.have a long focal Angle of incidence 𝑖 = 𝑝, angle of reflection 𝑟 =
length 𝑞. Law of reflection : angle of incidence equal
Concave mirror used as shaving or dressing angle of reflection, 𝑖 = 𝑟. ∴ 𝑝 = 𝑞. 𝑝=𝑟
mirror is narrow while concave mirror used for (alternate angles). ∴ 𝑝 = 𝑞 = 𝑟. 𝑝 + 𝑞 = 𝑠.
search light and car head lamps os wide. 91. Which of the following is true for the image
85. Where can a man place his face to get an formed by a convex mirror? I.The image is always
enlarged image when using a concave mirror to virtual II.The image is always erect III.The image
shave? A.Between the centre of curvature and lies between the pole and focus IV.The image is
the principal focus B.At the principal focus never magnified V.The focal length is negative A.I
C.Between the principal focus and the pole B.I and II C.II and III D.I,II,III and IV E.All five.
D.At the centre of curvature. The image formed by a convex mirror is always
Shaving,dressing or make-up mirror is made of virtual, erect and diminished i.e cannot be
concave mirror. The person’s face position magnified. It is formed between the pole abd the
during shaving should be between the focus and focus. The focal length of a convex mirroir is
the pole i.e within the focal length in order to negative.
obtain a virtual, erect and magnified image. 92. Which of the following best describes the
86. The most suitable type of mirror used for the images formed by a convex mirror? A.Virtual,

192
Demystified Series Physics Demystified by Dr Timothy
erect and diminished B.Real, erect and 99. An object is placed 12cm in front of a concave
diminished C.Virtual, inverted and magnified mirror of radius 32cm.The image formed is
D.Real, inverted and magnified E.Virtual, A.virtual and 48cm behind the mirror B.real
inverted and diminished. and 48cm in front of the mirror C.virtual and
93. Convex mirrors are used as driving mirriros 19.2cm behind the mirror D.real and 19.2cm in
because images formed are A.erect, virtual and front if the mirror
diminished B.erect, real and dimisnised C.erect, Object distance 𝑢=12cm, radius of curvature
virtual and magnified D.inverted, virtual and 𝑟=32cm,
1 1 1
+ = , 𝑣=
𝑢𝑓
, 𝑓=
𝑟

diminished. 𝑟 32
𝑢 𝑣 𝑓 𝑢−𝑓
𝑢𝑓 12×16
2
192
94. A convex mirror is used as a driving mirror 𝑓= = = 16cm, 𝑣 = = = ,
2 2 𝑢−𝑓 12−16 −48
because I.Its image is erect II.It has a large field 𝑣 = −48cm. The image is virtual, as image
of view III.It has a long focal length. Identify the distance is negative i.e 48cm behind the mirror.
correct statement(s) A.I B.I and II C.I and III D.II 100. An object is placed 10cm in front of a
and III E.I,II and III. concave mirror of focal length 15cm. What is the
Convex mirror is used as a car driving mirror position and nature of the image formed?
because its forms an upright or erect image of A.30cm and virtual B.6cm and real C.6cm and
objects and has a wide or large field of view. virtual D.30cm and real.
95. Which of the following properties make the 1
𝑢=10cm, 𝑓=15cm, + = , 𝑣 =
1 1 𝑢𝑓
,
convex mirrors useful as driving mirror? I.the 𝑢 𝑣 𝑓 𝑢−𝑓
10×15 150
image is real II.the image is erect III.It has a wide 𝑣= = = −30cm, the image is 30cm
10−15 −5
field of view IV.the image is magnified A.I and III from the lens as virtual as the image distance is
B.II and III C.I,II and III D.I,II and IV. negative.
96. Which of the following mirrors can be used as 101. An object is placed 10cm in front of a
a driving mirror? I.A plane mirror II.A concave mirror of focal length 15cm. The image
converging mirror III.A diverging mirror A.I B.II formed is A.virtual, erect and magnified B.real,
C.I and II D.I and III E.II and III inverted and magnified C.virtual, erect and
A plane mirror and a convex or diverging mirror diminished D.virtual, inverted and magnified
can be used as driving mirror but convex or E.real, inverted and diminished.
diverging mirrors are mostly used due to its 1 1
𝑢=10cm, 𝑓=15cm, + = , 𝑣 =
1 𝑢𝑓
,
wider field of view.. 𝑢 𝑣 𝑓 𝑢−𝑓
10×15 150
97. Which of the following is a advantage of the 𝑣= = = −30cm, the image is virtual as
10−15 −5
diverging mirror over the plane mirror when the image distance is negative. Virtual images are
used as a driving mirror? A.the field of view is always erect. Virtual image in a concave mirror is
wider B.the images are magnified C.the correct 𝑣
magnified or magnification 𝑚 = = − = − 3
30
distance of the car behind can be estimated D.the 𝑢 10
image is virtual E.all the images formed are erect i.e magnified. Magnification is negative as the
A convex or diverging mirror has a wider field of the image is virtual
view than a plane mirror. A convex or diverging 102. An object which is 5cm high is placed
mirror forms a diminished image, hence the vertically 10cm in front of a concave mirror. If
correct position or size of the object cannot be this object produces an image 40cm from the
estimated i.e a disadvantage of convex mirror. mirror, the height of the image is A.0.75cm
Plane mirrors form images of same size or B.4.00cm C.8.00cm D.12.00cm.
distance as the object, hence the correct position Object height 𝐻𝑂 =3cm, 𝑢=10cm, 𝑣=40cm, image
𝐻 𝑣
or size of an object can be estimated i.e an height 𝐻𝐼 =?, 𝑚 = 𝐼 = ,
𝐻𝑂 𝑢
advantage of plane mirror. Images in plane 𝐻𝐼 40
= , 𝐻𝐼 =
40×3
= 12cm.
mirror are laterally inverted i.e a disadvantage of 3 10 10
plane mirror. Both convex and plane mirror 103. A concave mirror of radius of curvature
forms erect images. 40cm forms a real image twice as large as the
98. 𝑌 object. The object distance is A.60cm B.40cm
C.30cm D.10cm.
𝑟 40
𝑂 𝑃 𝐹 𝐶 𝑟=40cm, 𝑓 = = = 20cm, 𝑚=2, 𝑢=?,
2 2
𝑓 20 20
𝑚= , 2= , 𝑢 − 20 = = 10,
The image of the object OX formed by the convex 𝑢−𝑓 𝑢−20 2

mirror in the figure shown is A.at C B.between F 𝑢 = 10+20 = 30cm. Alternatively,


𝑟 40 40
and C C.between P and F D.between O and P 𝑚= , 2= , 2𝑢 − 40 = = 20,
2𝑢−𝑟 2𝑢−40 2
The image formed by a convex mirror is always 2𝑢 = 20+40 = 60, 𝑢 = = 30cm.
60

virtual,erect or upright and diminished and is 2


104. Which of the following expressions gives
always formed between the principal focus 𝐹 and
the linear magnification produced by a concave
the pole 𝑃, at all positions of the object.
mirror of radius of curvature 𝑟, if 𝑢 and 𝑣 are the
193
Demystified Series Physics Demystified by Dr Timothy
𝑣
object and image distance respectively? A. − 1 111. The image of an object is located 6cm
𝑟
𝟐𝒗 𝑢 2𝑢 behind a convex mirror. If its magnification is 0.6,
B. − 1 C. − 𝑟 D. −1
𝒓 𝑟 𝑟 calculate the focal length of the mirror? A.3.75cm
𝑣 𝑟 𝑣 2𝑣
𝑚 = −1, 𝑓 = , 𝑚 = −1= −1. B.6.60cm C.10.00cm D.15.00cm.
𝑓 2 𝑟/2 𝑟
105. A dentist obtains a linear magnification of 4 𝑣=−6cm(virtual image), 𝑚=−0.6 (virtual image),
𝑣 6
of a hole in a tooth by placing a concave mirror at 𝑓=?, 𝑚 = − 1 , −0.6 = − − 1 ,
𝑓 𝑓
a distance of 2.0cm from the tooth. The radius of 6 6
− = −0.6+1, − = 0.4, 𝑓 = −
6
,
curvature of the mirror is A.5.3cm B.3.2cm 𝑓 𝑓 0.4
C.2.7cm D.1.6cm. 𝑓 = −15cm, focal length of a convex mirror is
𝑚=4, 𝑢=2.0cm, 𝑟=?, 𝑚 =
𝑟
, 4=
𝑟
, negative.
𝑟
2𝑢−𝑟 2×2−𝑟 112. A convex mirror produces an image which
4= , 4(4−𝑟) = 𝑟 , 16−4𝑟 = 𝑟 , is one-quarter the size of an object placed in front
4−𝑟
𝑟 + 4𝑟 = 16, 5𝑟 = 16, 𝑟 = = 3.2cm.
16
of it. If the radius of curvature of the mirror is
5
106. A real image three times the size of an object 32cm,the distance between the object and its
is formed 24cm from a converging mirror. What image is A.60cm B.48cm C.32cm D.12cm
1 𝑟 32
is the focal length of the mirror? A.8cm B.16cm 𝑚=− (virtual image), 𝑟=32cm, 𝑓 = = ,
4 2 2
C.6cm D.12cm 𝑓=−16cm (convex mirror), 𝑢=?, 𝑣=?,
𝑣 24 𝑓 1 16 1 16
𝑚=3, 𝑣=24cm, 𝑓=?, 𝑚 = − 1 , 3 = − 1 , 𝑚= , − =− , − =− ,
𝑓 𝑓 𝑢−𝑓 4 𝑢−(−16) 4 𝑢+16
24 24
= 3+1 = 4, 𝑓 = = 6cm. −1(𝑢 + 16) = 4 × −16 = −64,
𝑓 4
−𝑢 −16 = −64, −𝑢 = −64+16 = −48,
107. An object placed 10cm from a concave 𝑣 1
mirror of focal length 5cm would have its image 𝑢 = 48cm. 𝑚 = , 𝑣 = 𝑀𝑢 = − × 48, 𝑣 =
𝑢 4
A.at infinity B.10cm from the focus C.at the −12cm (virtual image), 𝑣 =12cm.
centre of curvature D.at the radius of curvature. Distance between an object an its image, 𝐷 =
𝑢=10cm, 𝑓=5cm, 𝑣=?, 𝑣 =
𝑢𝑓
=
10×5
= ,
50 𝑢 ± 𝑣, it is positive for virtual image and negative
𝑢−𝑓 10−5 5 for real image. 𝐷 = 𝑢 + 𝑣 =48+12 =60cm.
𝑣 = 10cm. Image distance equals object distance, 113. A candle is placed 20cm from a concave
𝑣 = 𝑢 = 10cm. Hence the object is placed at the mirror of focal length 10cm. The distance
centre of curvature and the image will also be between the candle and its image os A>40cm
formed at the centre of curvature. B.20cm C.10cm D.0cm.
108. An object of height 5cm is placed at 20cm 𝑢𝑓 20×10
from a concave mirror of focal length 10cm. The 𝑢=20cm, 𝑓=10cm, 𝑣=?, 𝑣 = = ,
𝑢−𝑓 20−10
200
image height is A.20cm B.15cm C.10cm D.5cm 𝑣= = 20cm. Distance between the candle
10
Object height 𝐻𝑂 =5cm, 𝑢=20cm, 𝑓=10cm, image and its image 𝐷 = 𝑢 − 𝑣 (real image) ,
𝐻 𝑓
height 𝐻𝐼 =?, 𝑚 = 𝐼 = , 𝐷 = 20−20 = 0cm.
𝐻𝑂 𝑢−𝑓
𝐻𝐼
=
10 𝐻
, 𝐼 = = 1, 𝐻𝐼 = 5cm.
10 114. An object is placed at a distance of 15cm
5 20−10 5 10 from a concave mirror of focal length 10cm. The
109. Calculate the position of an object placed in distance between the object and its image is
front of a convex mirror having focal length of A.15cm B.30cm C.45cm D.infinite.
12cm such that an image is formed 6cm away 𝑢𝑓 15×10
𝑢=15cm, 𝑓=10cm, 𝑣=?, 𝑣 = = ,
from the mirror A.3.0cm B.4.0cm C.9.0cm 𝑢−𝑓 15−10
150
D.12.0cm. 𝑣= = 30cm. Distance between the object and
5
𝑓=−12cm (negative focal length), 𝑣=−6cm its image 𝐷 = 𝑣 − 𝑢 (real image and 𝑣 > 𝑢), 𝐷 =
(convex mirror forms virtual image), 𝑢=?, 30−15 = 15cm.
𝑣𝑓 −6×−12 72 72
𝑢= = = = = 12cm.
𝑣−𝑓 −6−(−12) −6+12 6
110. The driving mirror of a car has a radius of Jamb past questions on Light wave and
curvature of 1m. A vehicle behind the car is 4m reflection of light :
from the mirror. Find the image distance behind [1978/45,1979/16,18,32,1981/12,1982/7,18,
8 𝟒 9 4 23,32,36,1984/6,1985/30,32,1986/33,1988/31
the mirror A. m B. m C. m D. m
7 𝟗 2 7 ,32,1989/26,27,1990/23,24,25,1991/25,26,
Car driving mirror is a convex mirror. 𝑟=1m , 1992/24,25,26,1993/25,26,1995/35,1997/34,
𝑟 1
𝑓 = = = −0.5m (negative focal length) , 1997/34,1998/29,30,31,2001/11,14,2002/19,
2 2
𝑢=4m, 𝑣 =
𝑢𝑓
=
4×−0.5
=
−2
=
−2
, 2003/21,23,2004/2,2005/34,37,2006/35,37,
−20
𝑢−𝑓
4
4−(−0.5) 4+0.5 4.5 2007/5,2008/30,32,33,2009/31,32,33,2100/29
𝑣= = − ,image distance is negative as the ,2011/30,2012/30,31,32,33,2013/29,2014/33,
45 9
image is virtual. Image distance behind the 2015/49]
4
mirror 𝑣 = m.
9

194
Demystified Series Physics Demystified by Dr Timothy

Chapter 18 – Refraction of light


● Refraction of light rays from plane incidence is greater than the angle of refraction
surface e.g a ray of light moving from water (𝑛 = 1.33) to
- Refraction is the bending of light (change in glass (𝑛 = 1.5).
direction) when it passes through a transparent 2.When a ray of light moves from a denser
media due to difference in speed and wavelength medium to a less dense medium or higher
as it passes from one medium to another. refractive index to that with lower refractive
Frequency is constant during refraction. index, it bends towards the normal and the angle
- Laws of refraction: of incidence is less than the angle of refraction e.g
1.The incident ray,the refracted ray and the a ray of light moving from glass (𝑛 = 1.5) to air
normal at the point of incidence, all lie in the (𝑛 = 1).
same plane. 3.If the incident ray is incident normally i.e at
2.Snell’s law – The ratio of sine of angle of angle 90° to the side of the block the angle of
incidence to sine of angle of refraction is constant incidence is zero (𝑖 = 0°), it passes straight
for a given pair of media.This constant is the through the glass, emerging at the other face
refractive index 𝒏 of the medium. without a change in direction or displacement.
𝐬𝐢𝐧 𝒊 - The diagrams below illustrates the change in
Snell’s law : 𝒏 = direction of light after refraction at the interface
𝐬𝐢𝐧 𝒓
- Refractive index 𝒏 of a medium is the extent of or boundary between glass and air –
refraction of light in that medium with respect to 1. 𝒊 2.
another medium. 𝒊
𝐬𝐢𝐧 𝒊 𝒔𝒑𝒆𝒆𝒅 𝒐𝒇 𝒍𝒊𝒈𝒉𝒕 𝒊𝒏 𝒎𝒆𝒅𝒊𝒖𝒎 𝟏 𝒗
𝒏= = = 𝟏.
𝐬𝐢𝐧 𝒓 𝒔𝒑𝒆𝒆𝒅 𝒍𝒊𝒈𝒉𝒕 𝒊𝒏 𝒎𝒆𝒅𝒊𝒖𝒎 𝟐 𝒗𝟐
𝒓
𝒗 ∝  , as frequency 𝒇 is constant thus,
𝒘𝒂𝒗𝒆𝒍𝒆𝒏𝒈𝒕𝒉 𝒐𝒇 𝒍𝒊𝒈𝒉𝒕 𝒊𝒏 𝒎𝒆𝒅𝒊𝒖𝒎 𝟏  Less dense to dense(𝒊>𝒓). 𝒓
𝒏= = 𝟏. Denser to less dense(𝒊<𝒓).
𝒘𝒂𝒗𝒆𝒍𝒆𝒏𝒈𝒕𝒉 𝒐𝒇 𝒍𝒊𝒈𝒉𝒕 𝒊𝒏 𝒎𝒆𝒅𝒊𝒖𝒎 𝟐 𝟐
- For a light moving from air to glass : The 3.
refractive index of glass 𝑛𝑔 is given by – Incident ray along the normal
𝒔𝒑𝒆𝒆𝒅 𝒊𝒏 𝒂𝒊𝒓 and emerges without being
𝒏𝒈 = 𝒏(air-glass) = 𝒏(𝒂−𝒈) = , displaced.
𝒔𝒑𝒆𝒆𝒅 𝒊𝒏 𝒈𝒍𝒂𝒔𝒔
𝒏𝒈 = 𝒏(𝒂−𝒈) =
𝒗𝒂
. - Refraction through a glass block –
𝒗𝒈
1.The angle of refraction 𝑟 increases
- For a ray of light travelling from medium of proportionately with the angle of incidence 𝑖.
refractive index 𝑛1 to another medium of 2.The angle of incidence 𝑖 at the first face is equal
refractive index 𝑛2 , the refractive index of to the angle of emergence 𝑒 at the opposite face.
medium 1 in respect to medium 2 𝑛(1−2) is given 3.The angle of refraction 𝑟 at the first face is equal
𝐬𝐢𝐧 𝒊 𝒗𝟏 𝒏𝟐
by – 𝒏(𝟏−𝟐) = = = . to the angle of incidence at the second or
𝐬𝐢𝐧 𝒓 𝒗𝟐 𝒏𝟏
- Based on the principle of reversibility of light – opposite face 𝑖2 .
𝟏 4.The incident ray is parallel to the emergent
𝒏(𝟏−𝟐) = . ray, hence the emergent is not deviated from
𝒏(𝟐−𝟏)
- When a ray of light is refracted from one the incident ray and the total angle of
medium to another with parallel boundaries or deviation of is 0°.
interface e.g (air-glass-water), then the product - The angle of deviation/displacement 𝑫 after
the refractive index and the sine of angle of refraction in the first face, D = 𝒊 − 𝒓.
incidence of the light in each media are equal or 5.The incident ray changes from the original path
constant. 𝒏 𝐬𝐢𝐧 𝒊 = constant. at any of the faces and it is called Lateral
∴ 𝒏𝒂 𝐬𝐢𝐧 𝒊𝒂 = 𝒏𝒈 𝐬𝐢𝐧 𝒊𝒈 = 𝒏𝒘 𝐬𝐢𝐧 𝒊𝒘 . deviation or displacement 𝒅𝑳 , 𝒅𝑳 =
𝒕𝒉 𝐬𝐢𝐧(𝒊−𝒓)
.
𝐜𝐨𝐬 𝒓
- The higher the optical density or refractive 𝑡ℎ=thickness of the block, 𝑖=angle of incidence,
index of a medium, the lower the speed of light 𝑟=angle of refraction.
(and wavelength) in that medium. Hence when 𝒊
light travels between two media of different 𝒊 = 𝒆 , 𝒓 = 𝒊𝟐
densities, it is either refracted away or towards
the normal depending on the densities or 𝒓 𝒊 𝒕𝒉
refractive index of the media. Thus : 𝟐
1.When a ray of light moves from a less dense
medium to a denser medium or lower refractive 𝒆
index to that with higher refractive index, it - Real and Apparent depth – It is observed that
bends towards the normal and the angle of a mark on a piece of paper under a glass block

195
Demystified Series Physics Demystified by Dr Timothy
appears nearer to top of the glass block when diviation 𝑫𝒎𝒊𝒏 the angle of incidence equals
viewed. The actual depth of the glass block is the angle of emergence (𝒊 = 𝒆). At maximum
called real depth while the depth as it appears to deviation 𝑫𝒎𝒂𝒙 the angle of incidence and
the eye is called the appararent depth. angle of emergence are equal to 90°.
𝑹𝒆𝒂𝒍 𝒅𝒆𝒑𝒕𝒉 𝑹 𝒅
𝒏= = . Real depth of a glass
𝑨𝒑𝒑𝒂𝒓𝒆𝒏𝒕 𝒅𝒆𝒑𝒕𝒉 𝑨
𝑫𝒎𝒂𝒙
block is equal to its thickness, 𝑹 = 𝒕𝒉.
- The apparent depth is displaced from the real 𝑫𝒎𝒊𝒏
depth by a certain value which is equal to the
Lateral or appararent displacement 𝒅. 𝒊=90° 𝒊=𝒆 𝒆=90° 𝒊
𝑹 𝑹
𝑹 = 𝑨 + 𝒅, 𝑨 = 𝑹 − 𝒅. Thus, 𝒏 = = , - Critical angle and Total internal reflection –
𝑨 𝑹−𝒅
𝒅=
𝑹(𝒏−𝟏)
= 𝑹 (𝟏 − ) .
𝟏 - When a ray travels from a less dense to a dense
𝒏 𝒏 medium,the ray is always reflected and refracted.
- Percentage of the lateral or apparent - When a ray travels from a dense to a less dense
𝒍𝒂𝒕𝒆𝒓𝒂𝒍 𝒅𝒊𝒔𝒑𝒍𝒂𝒄𝒆𝒎𝒆𝒏𝒕
displacement = × 100 , medium – I.with a small angle of incidence,the
𝑹𝒆𝒂𝒍 𝒅𝒆𝒑𝒕𝒉
𝒅
%𝒅 = × 100 =
𝑹−𝑨
× 100. ray is strongly refracted and weakly reflected i.e
𝑹 𝑹 decrease in brightness or intensity or amplitude.
- Refraction through triangular block/prism – II.as the angle of incidence, the ray becomes
A ray of light passing through is refracted twice weakly refracted and strongly reflected. III.when
i.e air-prism interface and opposite prism-air the angle of incidence is increased such that the
interface, and the emergent ray is deviated. angle of refraction is 90°, the angle of incidence is
𝑨 called critical angle and the ray is strongly
𝒊 𝒅 refracted and weakly reflected. IV.with angle of
𝒓𝟏 𝒓𝟐 𝒆 incidence greater than the critical angle,the ray is
not refracted but strongly reflected inside the
Emergent ray dense medium.
Refracted ray - Critical angle 𝑪 is the angle of incidence at
- 𝒊 – angle of incidence, 𝒓 – angle of refraction,𝑨 which the angle of refraction is 90° when light
– refracting angle. A=2𝒓. For an equilateral passes from denser to less dense medium.
traingular prism, 𝑨 =60°. - The refractive index of the medium is given by :
- There is always an angle between the incident 𝒏 = 𝐬𝐢𝐧 𝑪 (When light travel from dense to less
and emergent rays, which is the angle of 𝟏
dense medium) OR 𝒏 = (When light travel
deviation 𝒅. For equilateral prism, total angle of 𝐬𝐢𝐧 𝑪
deviation. 𝑫 = 𝒅𝟏 + 𝒅𝟐 , 𝒅𝟏 = (𝒊𝟏 − 𝒓𝟏 ), from dense to less dense medium).
𝒅𝟐 = (𝒊𝟐 − 𝒓𝟐 ) , 𝑫 = (𝒊𝟏 − 𝒓𝟏 ) + (𝒊𝟐 − 𝒓𝟐 ) , - When the emergent ray grazes the surface of the
𝒓𝟏 + 𝒓𝟐 = 𝑨, 𝑫 = (𝒊 + 𝒆) − 𝑨. prism it means the ray makes an angle of 90° with
- The deviation of a ray of light passing from air the normal (angle of refraction is 90°). Thus, the
into triangular prism is determined by – i.angle the angle of incidence equals the critical angle.
of incidence𝑖ii.refracting angle(A) of the prism - Total internal reflection is the reflection of an
iii.refractive index of the material of the prism. incident ray at the interface between the medium
- At minimum deviation 𝑫𝒎𝒊𝒏 : of incidence and another medium of lower
1.The light rays passes symmetrically through refractive index when the angle of incidence
the prism i.e the rays within the prism are exceeds the critical angle.
parallel to the base of the prism. - For glass of refractive index 1.5, total internal
2.Angle of incidence equals the angle of reflection occurs when the angle of of incidence
emergence, 𝒊 = 𝒆. is greater than 42°, its critical angle.
3.Angle of refraction in the first face 𝑟1 is equal to - For total internal reflection to occur –
the angle of incidence in the second face 𝑟2 . 1.Light must pass from a dense medium to a less
4.Refracting angle 𝑨 = 𝒓𝟏 + 𝒓𝟐 = 2𝒓. dense medium.
- Minimum deviation is same as the angle 2.The angle of incidence in the denser medium
through which a ray is least deviated. must be greater than the critical angle.
- The refractive index of the glass prism 𝑛 – - Application of Total internal reflection;
𝟏
𝒊 = (𝑨 + 𝑫𝒎𝒊𝒏 ), 𝑫𝒎𝒊𝒏 = 2𝒊 −2𝒓 = 2𝒊 − 𝑨 , 1.Submarine periscope – Right-angled
𝟐
𝟏
isosceles prisms (reflecting prism) are used to
𝒔𝒊𝒏𝟐(𝑨+𝑫𝒎𝒊𝒏)
𝟏
𝒓 = 𝑨, 𝒏=
𝐬𝐢𝐧 𝒊
= . replace plane mirrors in simple periscopes, to
𝑨
𝟐 𝐬𝐢𝐧 𝒓 𝒔𝒊𝒏
𝟐 prevent occurence of multiple images, tarnishing
- As the angle of incidence increases, the angle of and deterioration associated with plane mirrors.
deviation first decreases to a minimum value 2.Optical fibres – Thin flexible glass tube e.g
called minimum deviation and then increases silica glass,fluoride glass,phosphate glass or
giving rise to a curved graph. At minimum chalcogenide glass,of high refractive index
196
Demystified Series Physics Demystified by Dr Timothy
through which light signals can travel over a long normally on the hypothenuse PR of a prism, the
distance by means of total internal reflection.It ray is totally reflected twice, at the interface PQ
used in telecommunication to transmit signals and QR and emerge normally through the
over longer distances at higher band widths(data hypothenus. P
base) than metal wires which were previously Incident ray
used.
D coating F H
B core Reflected ray Q
𝑖
A E coating G R
In the optical fibre digragram above, ray AB is
refracted as BD, to meet the interface between ● Refraction from spherical surfaces/
the core and coating at an angle freater than the
critical angle. Total internal reflection occurs
lenses
repeatedly and emerge as ray GH. - Convex or Converging lens is thickest in
3.Appearance of shinning object e.g diamond middle and thinnest in edges.
4.Prism binoculars oe short telescope – Device - Concave or Diverging lens is thinnest in
middle and thickest in edges.
for looking at small distant object by magnifying
the objects uising lenses for eye. - Real images in lens is formed on opposite
5.LASER(Light Amplification by Stimulated side of the lens i.e behind the lens while
Virtual image is formed in same side of the
Emission of Radiation) – Device which emits
light through a process of optical modulation/ lens with the object i.e infront of the lens.
amplification based on a stimulation of - Images formed by a convex lens :
1.Object at infinity – image at 𝐹, real,inverted and
eledtromagnetic signal. Uses of Laser :
i.Communication:hologram production,optical diminished e.g burning glass.
fibres,space communication,data transfer,VCD 2.Object beyond 2𝐹 – image between 𝐹 and 2𝐹,
real,inverted and diminished e.g lens camera.
and DVD playing. ii.Medicine:Servical cancer
3.Object at 2𝐹 – image at 2𝐹,real,inverted and
surgery,boring holes in skull,monitoring glucose
level for diabetic patient iii.Militray/security: same size e.g copying or duplicating camera.
generation of isotopes fro nuclear weapons, 4.Object between 𝐹 and 2𝐹 – image beyond 2𝐹,
real,inverted and magnified e.g slide projectors
aircrafts and satellites,guidiance system missles.
- Total Internal reflection in nature – and objective lens of microscope.
1.Fish’s eye view – Fishes or a swimmer moving 4.Object at 𝐹 – image at infinity, real, inverted
and magnified e.g theatre and stadium spot
on his back under water,has its field of view
widened and can see everything above water lights.
surface at a cone range twice the critical angle, 5.Object between 𝐹 and opical centre 𝐶 of the
lens – image is beyond object i.e same side of the
while objects in water and outside this range are
seen by total internal reflection. lens with object, virtual,erect or
2.Mirrage – An optical illusion caused by a upright,magnified e.g simple microscope or
combined effect of refraction(majorly) and total magnifying glass.
- Images formed by concave lens :
internal reflection,whereby shining pool of water
is seen on a tarmac road on a sunny day and 1.Image is always virtual,erect or upright,
disappears on getting close to the spot. diminished (VED), at all objects positions.
2.Image is always between the principal focus
- Totally reflecting prism – They are right
angled isosceles triangular prism. and the lens.
1.Turning of incident ray through 90° : When - Graphical representation, lens formula and
magnification formula is same as that of mirrors
a ray is incident normally (at right angles) on one
side of the prism PQ (but not the hypotenuse), (Page 207).
the ray is totally internally replected as it strikes - The focal length of convex lens is positive while
the hypothenuse PR at angle 45° greater than the that of concave lens is negative.
- Image distance is positive for real images
critical angle of glass,42° and emerges normally
through QR. P (inverted) and negative for virtual images (erect
or upright).
- Distance between an object and its image is
Incident ray 45°
ray given by – 𝑫 = 𝒗 ± 𝒖. It is positive for real,
Q 45°
R inverted images and negative for virtual,
erect images. 𝑫 = 𝒖 − 𝒗(for concave lens).
Reflected ray
2.Turning of incident ray through 180° or - Spherical aberration in lenses – The inability
inverting a beam of light: When a ray is incident of a lens to refract wide beam or rays of light that

197
Demystified Series Physics Demystified by Dr Timothy
are far from the principal axis through a common same as in vacuum C.is higher than in 𝑦D.𝒚 is
focus but through a point nearer than the focus. higher than in 𝒙 .
It can be minimized by allowing light to pass The speed of light is high is lessss dense medium
through the optical centre 𝑪 (centre of the lens) (low refractive index) than in dense medium
only. (high refractive index), optical density or
- Rules for constructing ray diagrams in lens: refractive index varies inversely with the speed
1.A ray from the object, parallel to the principal of light. Hence, speed of light in 𝑦 (𝑛 = 1.33) is
axis emerges through the principal focus after higher than in 𝑥 (𝑛 = 1.51).
refraction by the lens. 5. Light of velocity 3.0×108ms-1 is incident on a
2.A ray from the object through the principal material of refractive index 𝑛. If the velocity is
focus emerges parallel to the principal axis after reduced to 2.4×108ms-1 in the material, what is
refraction by the lens. 𝑛? A.2.33 B.2.25 C.1.33 D.1.25
3.A ray from the object passing through the Velocity in air 𝑣𝑎 =3.0×108ms-1, velocity in
optical centre of the lens passes through medium 𝑣𝑚 =2.4×108ms-1, refractive index of the
𝑣
undeviated. medium 𝑛𝑚 =?, 𝑛𝑚 = 𝑛(𝑎−𝑚) = 𝑎 ,
𝑣𝑚
- The existence conjugate points one on either 3.0×108
side of a lens is based on principle of 𝑛𝑚 = 𝑛(𝑎−𝑚) = = 1.25.
2.4×108
reversibility of light. 6. If the speed of light in water is 2.26×108ms-1
- The magnifying power of a lens depends on its and the velocity of light in air is 3.0×108ms-1.
focal length. Calculate the refractive index of water A.1.33
- The shorter the focal length the greater the B.1.23 C.1.45 D.1.56
power of the lens and the longer the focal 𝑣𝑤 =2.26×108ms-1, 𝑣𝑎 =3.0×108ms-1, 𝑛𝑤 =?,
length lesser the power of the lens. 𝑛𝑤 = 𝑛(𝑎−𝑤) =
𝑣𝑎
=
3.0×108
= 1.327 = 1.33.
- Power of lens 𝑷 is equal to the reciprocal of its 𝑣𝑤 2.26×108
focal length in meters. Power of a lens is in 7. A ray of light passes through glasss of
𝟏 𝒖+𝒗 refractive index 1.50. If the speed of light in
Dioptres 𝑫. 𝑷 = = .
𝒇 𝒖𝒗 vacuum is 3.00×108ms-1, determine the spoeed
- Power of a convex lens is positive while that of light in glass A.1.50×108ms-1 B.2.00×108ms-1
of a concave lens is negative. C.2.25×108ms-1 D.4.50×108ms-1
𝑣𝑎 =3.00×108ms-1, 𝑛𝑔 =1.50, 𝑣𝑔 =?,
Examples : 𝑣𝑎 3.00×108
1.When a ray of light passes from water to glass,it 𝑛𝑔 = 𝑛(𝑎−𝑔) = , 1.50 = ,
𝑣𝑔 𝑣𝑔
is A.bent away from the normal B.bent towards 3.00×108
𝑣𝑔 = =2.00×108ms-1.
the normal C.totatally internally reflected 1.50
D.absorbed by the glass E.broken into pure 8. The speed of light in vacuum is 3.0×108ms-1 If
spectrum. the refractive index of transpartent liquid is 4/3,
When light passes from less dense medium or then the speed of light in the liquid is
medium with low refractive index(water) to a A.0.44×108ms-1 B.2.25×108ms-1 C.3.0×108ms-1
dense medium or medium with high refractive D.4.0×108ms-1 E.4.33×108ms-1
index(glass),it bends towards the normal and the 𝑣𝑣 =3.0×108ms-1, 𝑛𝐿 =4/3, 𝑣𝐿 =?,
𝑣𝑣 𝑣𝑣 3.0×108 3.0×108 ×3
angle of incidence in water is greater than the 𝑛𝐿 = , 𝑣𝐿 = = = ,
𝑣𝐿 𝑛𝐿 4/3 4
angle of refraction in glass. 𝑣𝐿 =2.25×108ms-1.
2. A ray of light which strikes a glass slab normal 9. Light of frequency 6.0×1014Hz travelling in air
incidence passes through the slab A.undeviated is transmitted through glass of refractive index
and undisplaced through the slab B.deviated 1.5m, calculate the frequency of light in the glass
and undisplaced at a lower speed C.deviated and A.4.0×1014Hz B.6.0×1014Hz C.7.5×1014Hz
displaced at a lower speed D.undeviated and D.9.0×10 Hz.
14
displaced at a faster speed Refraction occurs when light travels through
A ray incidente normally on a glass slab and will different media of different densities or
pass through undeviated and undisplaced refractive indices. Frequency of light in both
emerging normally at the other face. media is the same, as frequency is constant
3. When a ray of light is incident normally on air- during refraction. Thus, 𝑓𝑎 = 𝑓𝑔 =6.0×1014Hz.
glass interface, its angle of refraction is A.0°
10. A light wave of frequency 5×1014Hz moves
B.42° C.45° D.60° E.90°
through water which has a refractive index of
When a ray of light is incident normally, the angle
4/3. Calculate the wavelength in water if the
of incidence and the angle of refraction is 0°.
vcelocity of light in air is 3.0×108ms-1 A.6.0×10-
4. When light passes through two media 𝑥 and 𝑦 7m B.4.5×10-7m C.2.2×10-6m D.1.7×10-6m
of refractive index 1.51 and 1.33 respectively, the
Frequecy of light in air is same as that in water.
speed of light in A.𝑥 is same as in 𝑦 B.𝑥 and 𝑦 is
𝑓𝑎 =𝑓𝑤 = 5×1014Hz, 𝑛𝑤 =4/3, 𝑣𝑎 =3.0×108ms-1,

198
Demystified Series Physics Demystified by Dr Timothy
𝑣𝑎 𝑣𝑎
𝑤 =?, 𝑛𝑤 = = , 𝑛=?, 𝑛 =
sin 𝑖
=
sin(90−𝛼)
, sin(90 − 𝛼) = cos 𝛼 ,
𝑣𝑤 𝑤 𝑓𝑤 sin 𝑟 sin 𝛽
4 3.0×108 3.0×108 ×3 cos 𝛼
= , 𝑤 = =4.5×10-7m. 𝑛= .
3 5×1014 𝑤 5×1014 ×4 sin 𝛽
11. Light of wavelength 5000×10-8cm travels in 16. C O B
free space with a velocity of 3×108ms-1. What is 50° X
its wavelength in glass of refractive index 1.5?
A.3333×10-8cm B.5000×10-8cm C.6666×10- A
8cm D.7500×10-8cm The diagram above shows an incident ray AO
𝑎 =5000×10-8cm, 𝑣𝑎 =3.0×108ms-1, 𝑛𝑔 =1.5, 𝑔 =?, inclined at angle of 50° to the interface CB. The
𝑣𝑎 𝑎 5000×10−8 refracted ray OB is found to lie along the surface.
𝑛𝑔 = = , 1.5 = ,
𝑣𝑔 𝑔 𝑔 What is the refractive index of the medoium X
5000×10−8 sin 50° sin 40° sin 90° 𝐬𝐢𝐧 𝟒𝟎°
𝑔 = =3333×10-8cm. repect to air? A. B. C. D.
1.5 sin 40° sin 50° sin 50° 𝐬𝐢𝐧 𝟗𝟎°
sin 90°
12. A glass plate 0.9cm thick has a refractive E. .
sin 40°
index of 1.5. How long does it take for a pulse of C O B
light to pass through the plate? 50° X
A.3.0×10-11s B.4.5×10-11s C.3.0×10-10s 𝑖
D.4.5×10 s (𝑐=3.0×10 ms )
-10 8 -1
50° + 𝑖 =90°, 𝑖 = 90° − 50° = 40°, 𝑟 = 90°,
𝑡ℎ=0.9cm=0.009m, 𝑛𝑔 =1.5, 𝑣𝑎 =3.0×108ms-1, sin 𝑖 sin 40°
𝑡ℎ 𝑛= , 𝑛= .
𝑣𝑔 = , 𝑡=time taken for the pulse to pass sin 𝑟 sin 90°
𝑡
𝑣𝑎 𝑣𝑎 17. If the refractive index of glass is √2 , calculate
through the glass plate. 𝑛𝑔 = , 𝑣𝑔 = , the angle of incidence of a ray of light travelling
𝑣𝑔 𝑛𝑔
𝑑 𝑡ℎ 𝑣𝑎 0.009 3.0×108 0.09×1.5 from air to glass when the angle of refraction is
𝑣𝑔 = = = , = , 𝑡= ,
𝑡 𝑡 𝑛𝑔 𝑡 1.5 3.0×108 1 √2 √3
30° (sin 30° = , sin 45° = , sin 60° = ,
𝑡 = 4.5×10-10s. 2 2 2

13. A ray of light passes from air to water to glass sin 90° = 1) A.0° B.30° C.45° D.60° E.90°
sin 𝑖
to air. Given that the refractive index for light 𝑛𝑔 =√2 , 𝑟=30°, 𝑖=?, 𝑛𝑔 = ,
sin 𝑟
4 sin 𝑖 1
passing from air to water is and air to glass is √2 = sin 30° , sin 𝑖 = √2 sin 30° , sin 𝑖 = √2 × 2 ,
3
3
, calculate the refractive index of glass relative sin 𝑖 =
√2
, 𝑖 = sin−1
√2
= 45°.
2 2 2
to water A.0.50 B.0.67 C.0.75 D.1.13. 𝐜𝐨𝐬 𝜶 sin 𝛼 cos 𝛽 sin 𝛽
4 3 A. B. C. D. .
𝑛(𝑎−𝑤) =𝑛𝑤 = , 𝑛(𝑎−𝑔) =𝑛𝑔 = , refractive index of 𝐬𝐢𝐧 𝜷 sin 𝛽 sin 𝛼 sin 𝛼
3 2 18.
glass relative to water 𝑛(𝑔−𝑤) =?,
30°𝑛 3/2 3 3 9
30°
𝑔
𝑛(𝑤−𝑔) = = = × = = 1.13. M 60°
𝑛𝑤 4/360°2 4 8
14. The absolute refractive indices of glass and
water are 3/2 and 4/3 repsectively. The The refractive index of the medium M in the
refractive index at the interface when a ray diagram above is A.2/√3 B.1/√3 C.2√3 D.√𝟑 .
travels from water to glass is A.1/2 B.8/9 C.9/8 𝑖 = 90−30 = 60°, 𝑟 = 90−60 = 30°, 𝑛𝑀 =?,
D.17/12 E.2 . 𝑛𝑀 =
sin 𝑖
, 𝑛𝑀 =
sin 60°
=
√3/2
=
√3
× 2 = √3.
3 4 𝑛𝑔 sin 𝑟 sin 30° 1/2 2
𝑛𝑔 = , 𝑛𝑤 = , 𝑛(𝑤−𝑔) =?, 𝑛(𝑤−𝑔) = , 19. 𝑖
2 3 𝑛𝑤
𝑛𝑔
𝑛(𝑤−𝑔) = =
3/2 3
= × = .
3 9
air
𝑛𝑤 4/3 2 4 8
medium
15.
𝛼 Air
From the digarm above, the correct equation for
Medium
the refractive index 𝑛 of the medium is
𝛽 sin 𝑖 sin 𝑖
If a ray travelling in air is incident on a A.𝒏 = 𝐭𝐚𝐧 𝒊 B.𝑛 = C.𝑛 =
sin 90° cos(90°−𝑖)
transparent medium as shown in the diagram D.𝑛 = sin 90° .
the refractive index of the medium is given as The light ray in thediagram is polarized, as the
A.
𝐜𝐨𝐬 𝜶
B.
sin 𝛼
C.
cos 𝛽
D.
sin 𝛽
. reflected ray is perpendicular or at right angles
𝐬𝐢𝐧 𝜷 sin 𝛽 sin 𝛼 sin 𝛼
to the refracted ray.
𝜃
𝑖 𝑖
𝛼 Air
air
Medium
medium
𝛽
𝑟
By law of reflection, the angle of incidence equals
Angle of incidence𝑖 = 𝜃 = 90−𝛼, angle of
the angle of reflection. 𝑟=angle of refraction. 𝑖 +
refraction 𝑟 = 𝛽, refractive index of the medium
90° + 𝑟 = 180° , 𝑟 = 90° − 𝑖 ,

199
Demystified Series Physics Demystified by Dr Timothy
𝑛=
sin 𝑖
=
sin 𝑖
, sin(90° − 𝑖) = cos 𝑖 , 3.0×108ms-1 and 1.8×108ms-1 respectively.
sin 𝑟
sin 𝑖
sin(90°−𝑖)
Calculate the sine of angle of incidence that will
𝑛= = tan 𝑖 (Brewster’s law). produce an angle of refraction of 30° of ray of
cos 𝑖
20. A ray of light is incident on air-glass boundary light incident on glass A.1.2 B.1.0 C.0.8 D.0.6
at an angle 𝜃. If the angle between the partially E.0.3.
reflected ray and the refracted ray is 90°, 𝑣𝑎 =3.0×108ms-1, 𝑣𝑔 =1.8×108ms-1, 𝑟=30°, sin 𝑖=?,
calculate 𝜃, given that the refractive index of 𝑛𝑔 =
𝑣𝑎
=
sin 𝑖
,
3.0×108
=
sin 𝑖
,
5
=
sin 𝑖
,
glass is 1.50 A.28.2 B.46.5 C.56.3 D.64.7 𝑣𝑔 sin 𝑟 1.8×108 sin 30° 3 0.5
5×0.5
The partially reflected ray and refracted ray are sin 𝑖 = = 0.833 = 0.8.
3
at right angles i.e 90° to each other, thus the light 26. sin 𝑖
ray is polarized. 𝑛=1.5, 𝑖=𝜃=?,
𝑛 = tan 𝑖 , 1.5 = tan 𝜃 , 𝜃 = tan−1 1.5 = 56.3°. 1.0
21. 0.8
60°
0.6
Air 0.4
Glass 0.2
0.0 0.2 0.4 0.6 0.8 1.0 sin 𝑟
In the figure above,a ray of light in air strikes a The graph above shows the relationship between
glass plate at an angle of incidence of 60°.The sin 𝑖 and sin 𝑟 for a ray of light in air incident on
reflected ray is observedbe perpendicular to the a liquid surface. If 𝑖 and 𝑟 are respectively angles
refracted ray.What is the refractive index of of incidence and refraction deduce the refractive
glass? A.1.73 B.1.50 C.0.87 D.0.57. index from the graph. A.1.63 B.1.50 C.1.33
The light ray is polarized, as the reflected ray is D.1.25 E.0.80.
perpendicular or at right angles to the refracted Refrcative index 𝑛 is give by the slope of the
ray. 𝑖=60°, 𝑛 = tan 𝑖 = tan 60° , 𝑛 = 1.73 . graph of sin 𝑖 against sin 𝑟. 𝑛 = slope of the graph.
22. Incident ∆ sin 𝑖 1−0 1
𝑛= = = = 1.25.
𝑖 ∆ sin 𝑟 0.8−0 0.8
Glass(𝑛=1.52) 27. A measuring cylinder has a depth of 4.2cm
Water(𝑛=1.33) when filled with a liquid. A coin dropped inside it
From the diagram above, calculate the incident appears to be at a depth of 3.5cm when viewed
angle A.41° B.49° C.55°D.61° from above. Calculate the refractive index of the
𝑛𝑔 =1.52, 𝑛𝑤 =1.33, 𝑟=90°, 𝑖=?, liquid A.2.20 B.1.83 C.1.20 D.0.83 E.0.70
𝑛𝑤 sin 𝑖 1.33 sin 𝑖 sin 𝑖 Real depth 𝑅=4.2cm, Apparent depth 𝐴=3.5cm,
𝑛(𝑔−𝑤) = = , = = , 𝑅 4.2
𝑛𝑔 sin 𝑟 1.52 sin 90° 1 𝑛=?, 𝑛 = = = 1.2.
1.33 𝐴 3.5
sin 𝑖 = = 0.875, 𝑖 = sin−1 0.875 = 61°. 28. A nail is placed at the bottom of a bucket
1.52
23. which is filled with water to a depth of 20cm. At
30° Medium X what depth will the nail appear to be when
Refractive index 4/3 viewed from the top? (Refractive index of
Medium Y water=1.33) A.5.0cm B.7.5cm C.10.0cm
θ Refractive index 3/2 D.12.5cm E.15.0cm.
The diagram above shows ray of light passing Real depth 𝑅=20cm, Apparent depth 𝐴=?,
from medium X to medium Y. Calculate angle 𝜃 to 𝑛=1.33=4/3, 𝑛 = , 𝐴 = =
𝑅 𝑅 20 3
= 20 × ,
𝐴 𝑛 4/3 4
the nearest degree A.18° B.22°C.26° D.30°
𝐴 = 15.0cm.
𝑖=30°, 𝑛𝑋 =4/3, 𝑛𝑌 =3/2, 𝑟=𝜃=?,
𝑛 sin 𝑖 3/2 sin 30° 3 3 0.5 29. A coin is placed at the bottom of a cube of
𝑛(𝑋−𝑌) = 𝑌 = , = , × = , glass 𝑡cm thick. If the refraction index of the glass
𝑛𝑋 sin 𝑟 4/3 sin 𝜃 2 4 sin 𝜃
9
=
0.5
, sin 𝜃 =
8×0.5
, sin 𝜃 = 0.444 , is 𝜇, how high does the coin appear to be raised
8 sin 𝜃 9
to an observer looking perpendicularly into the
𝜃 = sin 0.444 = 26.38° = 26°.
−1
1 𝒕(𝝁−𝟏) 1 𝑡
24. A ray of light travelling from glass into ethyl glass? A. B. C.𝑡(1 + ) D. E.𝜇𝑡 .
𝑡−𝜇 𝝁 𝜇 𝜇
alcohol is incident at the boundary at angle of Real depth 𝑅=Thickness 𝑡, 𝑛= 𝜇, height appear to
incidence 30°. Calculate the angle of refraction. be raised =apparent displacement 𝑑,
[Refractive index of glass=1.5 ; refractive index of 𝑛= =
𝑅 𝑅
, 𝑛(𝑅 − 𝑑) = 𝑅 , 𝑛𝑅 − 𝑛𝑑 = 𝑅,
ethyl alcohol=1.36] A.27.0° B.51.7° C.33.4° 𝐴 𝑅−𝑑
𝑅(𝑛−1)
D.72.8°. 𝑛𝑅 − 𝑅 = 𝑛𝑑 , 𝑅(𝑛 − 1) = 𝑛𝑑 , 𝑑 = ,
𝑛
𝑛 sin 𝑖 𝑡(𝜇−1)
𝑛𝐸 =1.36, 𝑛𝑔 =1.5, 𝑖=30°, 𝑟=? 𝑛(𝑔−𝐸) = 𝐸 = , 𝑑= .
𝑛𝑔 sin 𝑟 𝜇
1.36
=
sin 30°
=
0.5
, sin 𝑟 = = 0.551 ,
1.5×0.5 30. A travelling microscope is focused on a mark
1.5 sin 𝑟 sin 𝑟 1.36
on a table. If a glass block of thickness 18.0cm is
𝑟 = sin−1 0.551 = 33.4°.
placed on the mark, by how many centimetres
25. The velocity of light in air and glass are
200
Demystified Series Physics Demystified by Dr Timothy
will the microscope be moved upwards so as to to glass. (refractive index of glass = 1.5) A.10.5°
focus the mark once again? (Refractive iondex of B.19.5° C.20.0° D.21° E.49.5° .
glass=1.5) A.27cm B.24cm C.9cm D.6cm 𝑖=30°, 𝑛𝑔 =1.5, angle of deviation 𝑑=?,
𝑅(𝑛−1) sin 𝑖 sin 𝑖 sin 30° 0.5
𝑅=𝑡ℎ=18.0cm, 𝑛=1.5, 𝑑=?, 𝑑 = , 𝑛𝑔 = , sin 𝑟 = = = ,
𝑛 sin 𝑟 𝑛𝑔 1.5 1.5
18(1.5−1) 18(0.5)
𝑑= = = 6cm. sin 𝑟 = 0.333 , 𝑟 = sin 0.333 = 19.5°. −1
1.5 1.5
31. An object is placed directly below a glass 𝑑 = 𝑖 − 𝑟 = 30−19.5=10.5°.
block of thickness 3.0cm. Calculate the lateral 37. A ray of light is incident on one side of a
displacement of the object if the refractive index parallel-sided glass block of refractive index 1.50
of the glass is 1.5 A.1.0cm B.1.5cm C.2.0cm and emerges from the other side after refraction
D.2.5cm. through the block. If the angle of incidence is
𝑅(𝑛−1) 40°,the angular deviation of the emergent ray is
𝑅=𝑡ℎ=3.0cm, 𝑛=1.5, 𝑑=?, 𝑑 = ,
𝑛 A.60° B.39° C.0° D.28°.
3(1.5−1) 3(0.5)
𝑑= = = 1cm. The incident ray passing through a rectangular
1.5 1.5
32. The difference between the apparent and real block is parallel to the emergent ray,hence the
depths of a swimming pool filled with water is emergent ray is not deviated and angular
1.5m.Calculate the depth of the swimming pool if deviation is 0° .
the refractive index of water is 1.33? A.4cm 38. Two rays of light which are equally inclined
B.3cm C.5cm D.6cm to the vertical emanate from a point below the
𝑑 = 𝑅 − 𝐴 = 1.5m, 𝐴 = 𝑅 − 𝑑, 𝑛=1.33, surface of water(refractive index=4/3).If the two
𝑛 = =
𝑅 𝑅
,𝑅 = 𝑛(𝑅 − 𝑑), 𝑅 = 𝑛𝑅 − 𝑛𝑑 , rays are inclined to each other at angle of 80° in
𝐴 𝑅−𝑑
𝑛𝑑
the water,the angle between the rays when they
𝑛𝑅 − 𝑅 = 𝑛𝑑, 𝑅(𝑛 −1)= 𝑛𝑑 , 𝑅 = , emerge into air is A.59° B.105°C.118°D.160°
𝑛−1
𝑅=
1.33×1.5
=
1.995
= 6.05cm = 6cm. 𝑟 𝑟
1.33−1 0.33 Air
33. The apparent depth of stone viewed from the
80° Water
top of a rectangular glass block was found to be 40° 40° 40° 40°
10.5cm. Calculate the percentage displacement of
the stone, when displaced 3.5cm from the base of
Since the rays are equally inclined to the vertical
the block A.25% B.33% C.66% D.75%
and are inclined at 80° to each other, each ray will
𝐴=10.5cm, 𝑑=3.5cm, 𝑅 = 𝐴 + 𝑑, percentage
𝑑 be inclined at 40° to the vertical.
displacement of the stone = × 100 , The ray travels from water to air :
𝑅
𝑑 3.5 3.5 1 1 sin 𝑖
%𝑑 = × 100 = × 100 = × 100, 𝑛𝑤 = 𝑛(𝑎−𝑤) , 𝑛(𝑤−𝑎) = = , 𝑛(𝑤−𝑎) =
𝐴+𝑑 10.5+3.5 14 𝑛(𝑎−𝑤) 𝑛𝑤 sin 𝑟
%𝑑 = 25%. 1 sin 40° 4
= , sin 𝑟 = × 0.643 = 0.857,
34. A submarine is observed to rise from a real 4/3 sin 𝑟 3
depth of 80m to 60m in water. Calculate the 𝑟 = sin 0.857 = 59°. The angle between the
−1

change in apparent depth. (Refractive index of rays when they emerge into the air will be the
water=4/3) A.60m C.15m C.80m D.45m sum of their angle of refraction. Their angle of
𝑅1 =80m, 𝑅2 =60m, 𝑛=4/3, change in apparent refraction are equal. 𝜃 = 2×59° = 118°.
𝑅 𝑅
depth ∆𝐴=?, 𝑛 = , 𝐴 = , ∆𝐴 = 𝐴1 − 𝐴2 , 39. If the refractive index of water relative to air
𝑅1 𝑅2 1
𝐴 𝑛 is 1.33, what is the critical angle for ray travelling
∆𝐴 = − = (𝑅1 − 𝑅2 ) , from water to air? A.39° B.41° C.42° D.49°.
𝑛 𝑛 𝑛
∆𝐴 =
1 3
(80−60) = × 20 = 15cm. 𝑛(𝑎−𝑤) =𝑛𝑤 =1.33, 𝐶=?, Air to water is less dense to
4/3 4 1 1
35. Which of following statement about a ray of dense, 𝑛𝑤 = , 1.33 = ,
sin 𝐶 sin 𝐶
1
light which passes through a rectangular glass sin 𝐶 = = 0.752, 𝐶 = sin 0.752 , −1
1.33
block and emerges from the opposite faces are 𝐶 = 48.75° = 49°.
correct? I.The angle of incidence at the first face 40. Calculate the critical angle for medium whose
= the angle of emergence from the opposite face refractive index is 1.25 A.41° 21’ B.42° 18’
II.The angle of refraction at the first face = the C.49° 0’ D.52° 6’ E.53° 8’ .
angle of incidence at the opposite face III.The 1 1
incident is parallel to emergent ray IV.The angle 𝑛=1.25, 𝐶=?, 𝑛 = , 1.25 = ,
sin 𝐶 sin 𝐶
1
of refraction at the first face = the angle of sin 𝐶 = = 0.8 , 𝐶 = sin−1 0.8 = 53.13°
1.25
emergence from the opposite face A.I,II and III 𝐶 = 53.13° = 53° + 0.13(60’) = 53° 8’.
B.I,II and IV C.I,III and IV D.II,III and IV E.I,II,III 41. Calculate the critical angle in glass for light
and IV . travelling from glass to water [Refractive index of
36. A ray of light from air is incident at angle of water=1.33, refractive index of glass=1.50]
30.0° on one face of a glass prism.What is the A.41.8° B.48.6° C.51.4° D.60.0°E.62.5°
angle of deviation of theray as it passes from air 𝑛𝑤 =1.33, 𝑛𝑔 =1.50, 𝐶=?, A ray of light moving glass

201
Demystified Series Physics Demystified by Dr Timothy
to water is dense to less dense medium thus, and III D.I and III E.II and IV.
𝑛(𝑔−𝑤) = sin 𝐶 , angle of refraction at critical Total internal reflection occurs when light moves
𝑛
angle=90°, sin 90°=1. 𝑛(𝑔−𝑤) = 𝑤 = sin 𝐶 , from an optically dense to optically less dense
𝑛𝑔
1.33
medium and the angle of incidence os greater
= sin 𝐶 , sin 𝐶 = 0.8867, 𝐶 = sin−1 0.8867 than the critical angle(or critical angle is less than
1.5
𝐶 = 62.46° = 62.5°. the angle of incidence).
42. A ray of light incident at an angle 𝜃 in a 47. Total internal reflection will not occur when
rectangular prism grazes the surface of the light travels from A.water into glass B.glass into
prism on emerging from the prism.Determine the air C.glass into water D.water to air
value of 𝜃. [Absolute refractive index of the Total internal reflection occurs when light travels
material of the prism is 1.5] A.41.8° B.48.2° from optically dense to optically less dense
C.60.8° D.90.0°. medium i.e glass to water, glass to air, water to
When emergent ray grazes the surface of the air e.t.c, and not the reverse like air to glass.
prism, it means it makes angle 90° with the 48. 𝛽 In the above diagram, a ray of
normal (angle of refraction is 90°). Angle of Air light passes from glass into
refraction is 90° at critical angle. Glass air. If the angle of incidence
𝛼
Angle of incidence 𝜃 = critical angle 𝐶, is 𝛼 and the angle of refraction
𝑛=
1
=
1
, sin 𝜃 = =
1 1
= 0.667, in air is 𝛽, which of the follow-
sin 𝐶 sin 𝜃 𝑛 1.5 ing is true? A.𝛼 = 𝛽 B.the refractive index of
𝜃 = sin−1 0.667 = 41.8° . 𝐬𝐢𝐧 𝜷
43. If the refractive index of glass is 1.5,what is glass is C.the ray suffers total internal
𝐬𝐢𝐧 𝜶
the critical angle at the air-glass interface? reflection if 𝛽 is greater than the critical angle
A.sin−1 (1/2) B.𝐬𝐢𝐧−𝟏 (2/3) C.sin−1 (3/4) D.sin−1 D.the ray will suffer total internal reflection, if 𝛼
(8/9) . is less than the critical angle E.the refractive
1 sin 𝛼
𝑛𝑔 = 𝑛(𝑎−𝑔) = , 𝑛𝑔 = 1.5 , index of glass is .
sin 𝐶 sin 𝛽
1 1 10 2 2
sin 𝐶 = = = = , 𝐶 = sin−1 ( ) . When light travels from glass to air i.e dense to
𝑛 1.5 15 3 3
44. The velocity of light in air is 3×108ms-1. less dense medium, it is refracted away from the
Calculate the velocity of light in glass if the critical normal and the angle of refraction 𝛽 is greater
angle for light travelling from glass to air to 42°. than the angle of incidence 𝛼 , 𝛽 > 𝛼. The
A.3.00×108ms-1 B.2.70×108ms-1 C.2.45×108ms-1 refractive index of glass is the refractive index
sin 𝛽
D.2.01×108ms-1 E.1.40×108ms-1 from air to glass not glass to air thus, 𝑛 = or
sin 𝛼
𝑣𝑎 =3×108ms-1, 𝐶=42°, 𝑣𝑔 =?, glass to air is dense 1
=
sin 𝛼
. Total internal internal reflection occurs
𝑣𝑔 𝑛 sin 𝛽
to less dense. 𝑛(𝑔−𝑎) = = sin 𝐶 , when the angle of incidence in the denser
𝑣𝑎
𝑣𝑔
= sin 42°, 𝑣𝑔 = sin 42° ×3×108 , medium is greater than the critical angle, 𝛼 > 𝐶.
3×108
𝑣𝑔 = 0.6691×3×108=2.01×108ms-1. 49. In which of the following optical instruments
is used made of total internal reflection?
45. Which of the following is/are necessary for
I.sextant II.kaleidoscope III.prism binoculars
the phenomenon of total internal relfection to
IV.submarine periscope A.I and II B.II and III C.III
occur? I.Light must travel from an optically
and IV D.I and IV E.I,II and III.
dense to an optically less dense medium II.The
Prism binoculars are shorts telescopes with two
angle of incidence in the optically dense medium
converging lens for refraction and totally
must be greater than the critical angle between
reflecting prism for total internal reflection. It
the two media III.The refracted ray must travel in
produces an upright image and achieve a high
the dense medium at 90° to the normal. A.I B.II
magnification of an object. Submarine periscope
C.I and II D.I and III E.II and III.
are made of reflecting prism which reduces the
Total internal reflection occurs when a ray of
distortion and formation of multiple images
light moves from a denser medium to a less
associated with plane mirrors in a simple
denser or rarer medium and the anlge of
periscope.
incidence must be greater than the critical angle.
50. The operation of optical fibre is based on the
46. Which of the following conditions are
principle of A.dispersion of light B.interference
necessary and sufficient for total internal
of light C.refraction of light D.polarization of
reflection to take place at the boundary between
light .
two media? I.light is passing from optically dense
Optical fibre is operated based on total internal
medium to optically less dense medium II.light is
relfection majorly but refraction also occurs at
passing from optically less dense medium to
the face of incidence.
optically dense medium III.angle of incidence is
51. Which of the following is not caused by
greater than critical angle IV.angle of incidence is
refraction alone? A.a meter rule appears bent
less than critical angle A.I and II B.II and III C.I
when immersed in water B.there appear to be

202
Demystified Series Physics Demystified by Dr Timothy
pool of water on tarred road on hot days C.a 𝑋
pool of water appears to be shallower than it
really is D.light passing through a triangular 𝜃3 A.𝜃1 B.𝜃2 C.𝜽𝟑 D.𝜃4 .
𝜃2
prism is bent towards the base E.a parallel beam 𝜃1
of light is made to converge to a point by a convex 𝜃4
lens. X Y
The appearance of pool of water on a tarred 56. A ray of light undergoes minimum deviation
broads on hot or sunny days is called mirage at 40° when its is incident on an equilateral
which can be explained by refraction and total triangular glass prism. Calculate the refractive
internal reflection.A meter rule which appears index of the prism A.148 B.1.50 C.1.53 D1.67
bent when immersed in water,pool of water Minimum deviation 𝐷𝑚𝑖𝑛 =40°, refracting angle of
appearing shallower and images formed by a an equilateral prism 𝐴=60°, 𝑛=?,
triangular prism and lens are explained by 1
𝑠𝑖𝑛 (𝐴+𝐷𝑚𝑖𝑛 )
1
𝑠𝑖𝑛 (60°+40°)
1
sin (100°)
refraction only. 𝑛= 2
𝐴 = 2
60° = 2
,
𝑠𝑖𝑛 𝑠𝑖𝑛 sin 30°
2 2
52. Which of the following may be used to explain sin 50° 0.766
𝑛= = = 1.53.
mirage? I.layers of air near the road surface have sin 30° 0.5
varying refractive indices in hot weather II.road
surfaces sometimes become good reflectors in
hot water III.light from the sky can be reflected 57. Calculate the refractive index
upwards after coming close to the road surface
60° of the material for the glass
30°
A.II B.II and III C.I and III D.I,II and III. prism in the diagram A.3/2
Mirage doesn’t occur due to the nature of the B.4/3 C.√2 /2 D.√𝟐 .
road but due to decreasing refractive index or
1
increasing temperature of the media through 𝑠𝑖𝑛 2(𝐴+𝐷𝑚𝑖𝑛 )
𝐴=60°, 𝐷𝑚𝑖𝑛 =30°, 𝑛=?, 𝑛 = ,
which light from the sky passes. 𝑠𝑖𝑛
𝐴
2
Refraction concept – mirage is the progressive 1
𝑠𝑖𝑛2(60°+30°)
1
sin2(90°) sin 45° 1/√2
bending of light as it travels through warm layers 𝑛= 60° = = = ,
𝑠𝑖𝑛 sin 30° sin 30° 1/2
2
of decreasing refractive index unitl it becomes 1 2 2 2 √2 2√2
parallel to the ground. 𝑛= × = = × = = √2 .
√2 1 √2 √2 √2 2
Reflection concept – mirage is the progressive 58. Calculate the angle of minimum deviation for
bending of light from the sky as it passes through a ray which is refracted through an equiangular
successive layers of optically less dense air of prism of refractive index 1.4 A.29° B.60° C.90°
increasing temperature near the hot ground until D.99°.
it is fnally totally internally reflected. 𝐴=60°(equiangular prism), 𝑛=1.4, 𝐷𝑚𝑖𝑛 =?,
53. Which of the following phenomena explains 1
𝑠𝑖𝑛2(𝐴+𝐷𝑚𝑖𝑛 )
1
𝑠𝑖𝑛 2(60°+𝐷𝑚𝑖𝑛)
the formation of mirage? I.reflection II.refraction 𝑛= 𝐴 , 1.4 = 60° ,
𝑠𝑖𝑛 𝑠𝑖𝑛
2 2
III.diffraction A.I and II B.II and III C.I and III 1
𝑠𝑖𝑛 2(60°+𝐷𝑚𝑖𝑛)
1
𝑠𝑖𝑛2(60°+𝐷𝑚𝑖𝑛 )
D.I,II and III. 1.4 = = ,
𝑠𝑖𝑛 30° 0.5
54. Which of the following diagrams below 1
𝑠𝑖𝑛 (60° + 𝐷𝑚𝑖𝑛 ) = 1.4×0.5 = 0.7,
correctly illustrates the path of a ray of light 1
2

through a glass prism? (60° + 𝐷𝑚𝑖𝑛 ) = sin−1 0.7 = 44.43° ,


2
A. B. (60° + 𝐷𝑚𝑖𝑛 ) = 44.43° × 2 = 88.86° ,
𝐷𝑚𝑖𝑛 = 88.86° − 60° = 28.86° = 29°.
𝐴 60°
Alternatively, 𝐴 = 2𝑟 , 𝑟 = = = 30°, 𝑛=1.4,
2 2
sin 𝑖
𝑖=?, 𝑛 = , sin 𝑖 = 𝑛 sin 𝑟 , sin 𝑖 = 1.4 × sin 30°
sin 𝑟
C. D. sin 𝑖 = 1.4×0.5 = 0.7, 𝑖 = sin−1 0.7 = 44.43° ,
𝐷𝑚𝑖𝑛 = 2𝑖 − 2𝑟 = 2×44.43° − 2×30° ,
𝐷𝑚𝑖𝑛 =88.86° −60° = 28.86° = 29°. OR
𝐷𝑚𝑖𝑛 = 2𝑖 − 𝐴 = 2×44.43° − 60° = 88.86° −60°
𝐷𝑚𝑖𝑛 = 28.86° = 29°.
The ray of light refracts towards the normal on 59. A ray of light is incident on an equilateral
the incident face as it moves from air to glass i.e glass prism of refractive index 3/2.Calculate the
less dense to dense. It refracts away from the angle through which the ray is minimally
normal on emerging at the other face from glasss deviated in the prism. A.30.0° B.37.2°C.42.0°
to air i.e dense to less dense (option D). D.48.6°.
55. In the diagram below, which of the angles 𝜃1 , 𝐴=60°(equilateral prism), 𝑛=3/2, 𝐷𝑚𝑖𝑛 =?,
1 1
𝜃2 , 𝜃3 and 𝜃4 , is the angle of deviation of a ray of 𝑠𝑖𝑛2(𝐴+𝐷𝑚𝑖𝑛 ) 3 𝑠𝑖𝑛 2(60°+𝐷𝑚𝑖𝑛)
𝑛= , = ,
light passing through glass prism XYZ? 𝑠𝑖𝑛
𝐴
2
2 𝑠𝑖𝑛
60°
2

203
Demystified Series Physics Demystified by Dr Timothy
3
1
𝑠𝑖𝑛 2(60°+𝐷𝑚𝑖𝑛)
1
𝑠𝑖𝑛 2(60°+𝐷𝑚𝑖𝑛) with angle of reflection of 60° towards the face
= = , BC where it is incident again normally and
2 𝑠𝑖𝑛 30° 0.5
1 3
𝑠𝑖𝑛 (60° + 𝐷𝑚𝑖𝑛 ) = ×0.5 = 0.75, emerges normally i.e at 90° to the face BC.
2 2
1 63. A 45° triangular glas prism can be used as
(60° + 𝐷𝑚𝑖𝑛 ) = sin−1 0.75 = 48.59° ,
2 reflector of light because A.refraction never
(60° + 𝐷𝑚𝑖𝑛 ) = 48.59° × 2 = 97.18° , takes place in such a prism B.the angle of
𝐷𝑚𝑖𝑛 = 97.18° − 60° = 37.18° = 37.2°. reflection equals the angle of incidence C.the
60. A ray of light is incident at an angle of 30° on refractive index of glass is less than 1 D,it is
a glass prism of refractive index 1.5. Calculate the transparent E.the critical angle for glass is less
angle through which the ray is minimally than 45°.
deviated in the prism A.10.5° B.19.5° C.21.1° A ray of light that strikes one face of the
D.38.9° E.40.5° triangular prism normally or perpendicularly
sin 𝑖 sin 𝑖
𝑖=30°, 𝑛=1.5, 𝐷𝑚𝑖𝑛 =?, 𝑛 = , sin 𝑟 = , meets the other face at angle of incidence 45°
sin 𝑟 𝑛
sin 𝑟 = =
sin 30° 0.5
= 0.333, 𝑟 = sin−1 0.333 which is greater than the critical angle of glass
1.5 1.5 42°, thus the ray is totally internally reflected.
𝑟 = 19.45° , 𝐷𝑚𝑖𝑛 = 2𝑖 − 2𝑟 , 64. In the day time, it is possibles to see under
𝐷𝑚𝑖𝑛 = 2×30° − 2×19.45° = 60° −38.9° , shady areas such as under a tree, because light
𝐷𝑚𝑖𝑛 = 21.1°. has undergone A.internal reflection B.refraction
61. Deviation(degrees) C.diffraction D.diffuse reflection
Diffraction of light is the property of light to bend
22 round obstacles or spread out through tiny gaps.
20 Thus, light rays from the sun spreads out through
18 the tiny gaps in trees by diffraction such that one
16 can see under the shady area in the tree.
14 Refraction and total internal reflection is
15 20 25 30 35 θ(degrees) incorrect as only one media is involved in this
The figure above shows a plot of angles through case.
a glass prism when light is incident at 𝜃 degrees 65. A ray incident on a glass prism undergoes
on the prism. The incident angle that produces minimum deviation when the A.incident angle is
the minimum deviation is A.25° B.18° C.35° equal to angle of refraction B.refraction angle
D.20° . equals 90° C.incident angle equals 90°D.incident
From the graphs above, at minimum deviation of angle equals angle of emergence.
18°, angle of incidence is 25° 66. In a ray diagram for a thin converging lens, a
62. A A ray of light XY is incident normally ray that is not parallel to the optic axis but passes
X on face 𝐴𝐵 of an equilateral glass through the optic centre will A.pass through
prism ABC of refractive index 1.50. undeviated B.pass through the centre of
The ray will curvature after refraction C.emerge parallel to
𝐵𝐶 the principal axis D.pass through the principal
A.emerge on the face AC,deflected away from the focus after refraction.
normal B.be totally internally reflected at the Ray of light pssing through the optical centre of a
face AC and emerge normal to the face BC lens pass through undeviated i.e angular
C.emerge on face AC,deflected towards the difference of 0°. Ray of light parallel to principal
normal D.be totally internally reflected at the axis of a lens will be refracted through the
face AC and emerge deflected away from the principal focus. Ray passing through the
normal on face BC. principal focus will be refracted parallel to the
The prism is an equilateral triangular prism with principal axis.
a refracting angle of 60°. 67. In an experiment with a convex lens, a graph
A As the ray is incident normally, of the reciprocal of image distance (1/𝑣) is
60° it will pass through the face AB plotted against the reciprocal of the object
60° undeflected and undeviated. distance (1/𝑢) as shown in the figure. The focal
60° length is given by the
B C 1
A.slope of the line AB
𝑣

𝑛=
1 1
, sin 𝐶 = , 𝑛=1.5, sin 𝐶 = ,
1 A B.reciprocal of either OA and OB
sin 𝐶 𝑛 1.5 C.length of OA or OB
sin 𝐶 = 0.667, 𝐶 = sin−1 0.667 = 41.83° , D.ratio of OA/OB.
𝐶 = 42°. The light ray strikes the face 𝐴𝐶 within 1
O B
the prism at angle of incidence of 60°. Since the 𝑢
1 1
angle of incidence 60°, is greater than the critical The focal length of a graph of vs , is equal to
𝑣 𝑢
angle 42°, the ray is totally internally reflected

204
Demystified Series Physics Demystified by Dr Timothy
the reciprocal to the intercept i.e OA or OB, on object is moves from the centre of curvature
1 1
either axis ( – axis or – axis). towards the focus of a converging lens, its image
𝑣 𝑢
A.moves closer to the lens B.remains stationary
68. The correct shape of the graph of 𝑢𝑣 against
C.becomes larger D.becomes smaller E.remains
(𝑢 + 𝑣) for an object distance 𝑢 and image
the same size.
distance 𝑣 in an experiment to find the focal
73. If a document is to be produced using a
length of a convex lens is given as
duplicating camera, where should the camera be
A. 𝒖𝒗 B. 𝑢𝑣
placed in order to obtaib an exact copy? A.at the
point which is twice the focal length of the
lens of the camera C.between the principal
(𝒖 + 𝒗) (𝑢 + 𝑣)
focus and a point twice the focal length of the lens
C. 𝑢𝑣 D. 𝑢𝑣
of the camera D.Beyond a poibt twice the focal
length of the lens of the camera.
At point 2𝐹 i.e the point which is twice the focal
(𝑢 + 𝑣) (𝑢 + 𝑣) .
length of the lens, an image same size as the
The graph of 𝑢𝑣 against (𝑢 + 𝑣) is a straight line
object is formed on 2𝐹 at the opposite side of the
graph with a positive slode equal to the focal
lens. Thus, it can be used as a duplicating or
length 𝑓, 𝑢𝑣 = 𝑓(𝑢 + 𝑣) , as shown in option A.
copying camera when an object is placed at the
69. In an experiment ysing a converging lens to
centre of curvature 2𝐹 or at a point twice the
produce real imagees on a screen, the linear
focal length.
magnification 𝑚 is plotted against the image
74. A converging lens of focal length 𝑓 will act as
distance 𝑣 as illustrated in the diagram below.
a simple microscope if the object is placed
The distance OP represents the
A.beyond 2𝑓 fromn the lens B.at 2𝑓 from the lens
𝑚 A.focal length of the lens
C.between 2𝑓 and 𝑓 from the lens D.at 𝑓 from the
B.radius of curvature
lens E.between 𝒇 and the lens.
C.thickness of the lens
A converging or convex lens functions as a simple
𝑣 D.diameter of the lens.
microscope when the object is placed between
O P
𝑣 1 the focal length and the lens, to produce a virtual,
𝑚 = − 1 = (𝑣) − 1 , the slope of the graph is erect and magnified image.
𝑓 𝑓
equal to the reciprocal of the fical length
1
, the 75. When an object is placed between the
𝑓
principal focus and the optical centre of a convex
intercept on the 𝑣-axis 𝑂𝑃 is equal to the focal
lens, it could be used as a A.reflecting lens
length 𝑓, the intercept on the 𝑚-axis is equal to
B.compound microscope C.projector D.simple
−1.
microscope.
70. O
When an object is placed between the principal
focus and the optical centre of a convex lens, the
A P B
lens functions as a simple microscope.
76. A converging lens of focal length 5cm forms
An object 𝑂 is placed in front of a convex lens of
an image which is 10cm from the lens and on the
focal length AP=PB in the figure above. The
same side of the lens as the object.How far from
image is formed A.between A and P B.between P
the lens is the object? A.0.3cm B.3.3cm C.5.0cm
and B C.beyond A D.beyond B .
D.10.0cm E.15.0cm
Object between principal focus A and optical
𝑓=5cm, 𝑣=−10cm(virtual image are on the same
centre P,forms a virtual,erect and magnified
side of the lens as the object,hence image
image(VEM) and is formed beyong the object i.e
distance is negative), 𝑢=?,
beyond 𝐴. 𝑣𝑓 −10×5 −50 10
71. As an object approaches the focus of a 𝑢= = = = = 3.3cm.
𝑣−𝑓 −10−5 −15 3
converging lens from infinity, its image I.move 77. An object is placed 14cm in front of a
closer to the lens II.moves away from the lens converging lens of focal length 10cm. The image
III.remains the same IV.becomes smaller. Which 5
is formed A.5 cm on the same side as the object
of the above is/are true? A.II B.III C.IV D.I and III 5
6

E.III and IV . B.5 cm on the opposite side from the object


6
1 1
In a conex or converging lens, 𝑢 ∝ ∝ , as the C.24cm on the opposite side from the object
𝑣 𝐻𝐼
D.35cm on the ssame side as the onbject E.35cm
object approaches the focus from infinity i.e on the opposite side from the object
decreasing object distance, the images moves 𝑢𝑓
away from the lens i.e increasing image distance, 𝑢=14cm, 𝑓=10cm, 𝑣=?, 𝑣 = ,
𝑢−𝑓
and the images becomes larger in size i.e 𝑣=
14×10
=
140
= 35cm. The image formed is a
14−10 4
increasing image size/height. real image and is formed on the opposite side of
72. Which of the following is correct? As an the lens from the object.
205
Demystified Series Physics Demystified by Dr Timothy
78. An object is placed 5.0cm from a thin D.40cm.
converging lens of focal length 10.0cm. The 𝑓=20cm, 𝑚=−2(virtual image), 𝑢=?,
image formed will be A.3.3cm from the lens, 𝑚=
𝑓
, −2 =
20
, 𝑢 − 20 =
20
,
𝑢−𝑓 𝑢−20 −2
erect, virtual and on the same side of the lens
B.3.3cm from the lens, inverted, real and on the 𝑢 − 20 = −10, 𝑢 = −10+20 = 10cm.
other side of the lens C.5.0cm from the lens, erect, 84. An object 15cm from a converging lens forms
virtual and on the same side of the lens D.10.0cm a real image of magnification 3. Calculate the
focal length of the lens A.1.33cm B.1.76cm
from the lens, erect, virtual and on the same
side of the lens E.10.0cm from the lens, erect, C.3.75cm D.11.25cm E.22.50cm.
𝑓 𝑓
real and the other side of the lens. 𝑢=15cm, 𝑚=3, 𝑓=?, 𝑚 = , 3= ,
𝑢−𝑓 15−𝑓
𝑢𝑓
𝑢=5.0cm, 𝑓=10.0cm, 𝑣=?, 𝑣 = , 3(15−𝑓) = 𝑓 , 45−3𝑓 = 𝑓 , 45 = 𝑓 + 3𝑓 ,
𝑢−𝑓 45
𝑣=
5×10 50
= = −10cm, the image distance is 45 = 4𝑓 , 𝑓 = = 11.25cm.
4
5−10 −5
85. An object is placed 10cm from a converg-ing
negative, thus the image is virtual, erect and on
lens of focal length 15cm.Calculate the
the same side of the lens with the object.
magnification of the image formed A.3.0 B.1.5
79. An object placed 50cm away from the focus
C.0..6 D.0.3
of an converging lens of focal length 15cm, 𝑓 15
produces a focused image on a screen. Calculate 𝑢=10cm, 𝑓=15cm, 𝑚=?, 𝑚 = = ,
𝑢−𝑓 10−15
the distance between the object and the screen 15
𝑚 = = − 3 , the negative sign indicates that
A.42.3cm B.65.6cm C.76.5cm D.84.5cm. −5

Distance of the object from the principal focus of the image is virtual.
the lens=50cm, focal length 𝑓=15cm, object 86. A converging lens of focal length 10cm forms
𝑢𝑓 an erect image three times the size of a real
distance 𝑢=50+15=65cm, 𝑣=?, 𝑣 = , object. Calculate the distance between the object
𝑢−𝑓
𝑣=
65×15
=
975
= 19.5cm. Distance between the and the image A.6.6cm B.11.1cm C.13.3cm
65−15 50
D.20.0cm.
object and the screen = 𝑢 + 𝑣 = 65+19.5
Object distance from the screen = 84.5cm. 𝑓=10cm, 𝑚=−3(erect image are virtual), 𝐷=?,
𝑣 𝑣 𝑣
80. An object is placed 30.0cm from a converging 𝑚 = − 1 , −3 = − 1, = −3+1 = −2,
𝑓 10 10
lens. If the real image is formed 90.0cm from the 𝑣 = −2×10 = −20cm (virtual image),
object, calculate the focal length of the lens 𝑣
𝑚 = , −3 =
−20
, 𝑢=
−20
= 6.67cm,
A.20.0cm B.22.5cm C.40.0cm D.60.0cm. 𝑢 𝑢 −3
𝑢𝑣 Disatnce between the object and its virtual image
𝑢=30.0cm, 𝑣=90.0cm, 𝑓=?, 𝑓 = , 𝐷 = 𝑣 − 𝑢 = 20 − 6.67 = 13.3cm.
𝑢+𝑣
30×90 2700
𝑓= = = 22.5cm. 87. A lens of focal length 12.0cm forms an upright
30+90 120
81. A converging lens of focal length 5cm forms a image three times the size of a real object.The
virtual image which is 20cm from the lens. How distance between the object and the image is
far fron the lens is the object? A.2.0cm B.4.0cm A.16.0cm B.24.0cm C.8.0cm D.32.0cm.
C.6.7cm D.20.0cm E.25.0cm. Only a convex lens can produce an upright (erect)
𝑓=5cm, 𝑣=−20cm(virtual image), 𝑢=?, and magnified image.Concave lens form erect
𝑣𝑓 −20×5 −100 and diminished image. Erect or upright images
𝑢= = = = 4.0cm.
𝑣−𝑓 −20−5 −25 are virtual.
17. An object 1cm high placed on the axis 15cm 𝑚=−3, 𝑓=12cm, 𝐷=?, 𝑚 =
𝑓
,
from a converging lens forms an image 30cm 12 12
𝑢−𝑓

from the lens. The size of the image is A.0.5cm −3 = , 𝑢 − 12 = = −4, 𝑢 = −4+12,
𝑢−12 −3
B.1.5cm C.2cm D.2.5cm E.3cm 𝑣
𝑢 = 8cm, 𝑚 = , 𝑣 = 𝑚𝑢 = −3×8 = 24cm
𝐻𝑂 =1cm, 𝐻𝐼 =?, 𝑢=15cm, 𝑣=30cm, 𝑢
𝐻 𝑣 𝐻 30 Distance between the object and its virtual image
𝑚 = 𝐼 = , 𝐼 = , 𝐻𝐼 = 2cm. 𝐷 = 𝑣 − 𝑢 =24−8=16cm.
𝐻𝑂 𝑢 1 15
82. An object placed on the principle axis of a 88. An object is placed 5.6×10-2m in front of a
convex lens of focal length 10cm, produces a real converging lens of focal length 1.0×10-1m.The
image of double magnification. The image image formed is A.real,erect and magnified
distance from the lens is A.30cm B.25cm C.20cm B.virtual,erect and magnified C.real,inverted
D.15cm E.10cm. and magnified D.virtual,erect and diminished.
𝑣
𝑓=10cm, 𝑚=2, 𝑣=?, 𝑚 = − 1 , 2 = −1,
𝑣 𝑢=5.6×10-2m=0.056m, 𝑓=1×10-1m=0.1mk, 𝑣 =
𝑓 10 𝑢𝑓 0.056×0.1 0.056
𝑣 = = = −1.273cm ,
= 2+1 = 3, 𝑣 = 3×10 = 30cm. 𝑢−𝑓 0.056−0.1 0.044
10
83. An object is placed in front of a converging Image distance is negative, thus it is a virtual.
lens of focal length 20cm.The image is virtual and Virtual images are always erect. Convex
has a magnification of 2. What is the distance of (converging lens) forms virtual images that is
the object from the lens A.5cm B.10cm C.30cm magnified. Alternatively,

206
Demystified Series Physics Demystified by Dr Timothy
𝑚=
𝑓
=
0.1
=
0.1
= −2.3 95. Which of the following correctly describes
𝑢−𝑓 0.056−0.1 0.044
the image of an object 4cm from a diverging lens
Image is virtual as magnification is negative, of focal length 12cm? A.The image is virtual, 3cm
virtual images are always erect. The image is also
in front of the lens B.The image is real, 6cm
magnified as magnification 𝑀=2.3. behind the lens C.The image is virtual, 6cm in
89. A converging lens and a screen are placed front of the lens D.The image is real, 3cm in front
20cm and 80cm respectively from an object in a of the lens.
straight line so that a sharp image of the object is 𝑢𝑓
formed on the screen. If the object is 3cm high, 𝑢=4cm, 𝑓=−12cm(concave lens), 𝑣=?, 𝑣 = =
𝑢−𝑓
calculate the height of the image formed A.1cm 4×−12
=
−48
=
−48
= −3cm. The image is
4−(−12) 4+12 16
B.9cm C.12cm D.15cm.
virtual image. Virtual image is formed in fornt of
Object distance from the the lens 𝑢=20cm,
the lens. Alternatively, without solving, image
distance of object from the screen=80cm, image
formed by a concave or diverging lens is always
distance from the lens 𝑣=80−20=60cm,
𝐻 𝑣 𝐻 60 virtual and closer to the lens than the object.
𝐻𝑂 =3cm, 𝐻𝐼 =?, 𝑚 = 𝐼 = , 𝐼 = , 96. An object is placed 15cm from a diverging
𝐻𝑂 𝑢 3 20
𝐻𝐼 = 3×3 = 9cm. lens of focal length 12cm. The image of the object
90. The magnification of an object produced with formed by the lens is A.Real and 6.67cm from the
a converging lens of focal length 15cm is 5. If the lens B.Virtual and 6.67cm from the lens C.Real
image is real, what is the separation between the and 60.00cm from the lens D.Virtual and 60.00cm
object and the image? A.13cm B.18cm C.72cm from the lens
D.90cm E.108cm. 𝑢=15cm, 𝑓=−12cm, 𝑣=?, 𝑣 =
𝑢𝑓
=
15×−12
𝑣 𝑣 𝑢−𝑓 15−(−12)
𝑓=15cm, 𝑚=5, 𝐷=?, 𝑚 = − 1, 5 = − 1, −180 −180
𝑣 𝑣
𝑓 15 𝑣= = = −6.67cm (virtual image).
15+12 27
5+1 = , 6= , 𝑣 = 6×15 = 90cm,
15 15 Alternatively, image formed by a concave or
𝑣 𝑣 90
𝑚 = , 𝑢 = = = 18cm, diverging lens is always virtual and closer to the
𝑢 𝑚 5
Distance between the object and its real image lens than the object.
𝐷 = 𝑣 + 𝑢 = 90 +18 = 108cm. 97. An object is placed at a distance 2𝑓 from a
91. The image formed by a diverging lens is diverging lens of focal length 𝑓.The image
always A.magnified, virtual and erect distance is A.2𝑓 B.2𝒇/3 C.𝑓 D.𝑓/4.
𝑢𝑓 2𝑓×−𝑓
B.diminished, real and inverted C.diminished, 𝑢=2𝑓 , 𝑓 = −𝑓, 𝑣=?, 𝑣 = = ,
𝑢−𝑓 2𝑓−(−𝑓)
virtual and erect D.diminished, virtual and −2𝑓 2 −2𝑓 2 −2𝑓
erect. 𝑣= = = (virtual image).
2𝑓+𝑓 3𝑓 3
The image formed by a concave or diverging lens [Option A would have been correct,if was a
is always virtual,erect or upright and diminished convex or converging lens].
and is formed between the object and the optical 98. A diverging lens of focal length 30cm
centre of the lens. produces an image 20cm from the lens.
92. The image of a real object formed by a Determine the object distance A.10cm B.12cm
diverging lens is always A.inverted B.real C.50cm D.60cm.
C.situated between the object and the lens 𝑓=−30cm, 𝑣=−20cm, 𝑢=?, 𝑢 =
𝑣𝑓
,
D.larger than the object. −20×−30 600 600
𝑣−𝑓

93. An object is placed at a point 𝑋 between the 𝑢= = = = 60cm.


−20−(−30) −20+30 10
focal point 𝐹 and the optical centre 𝐶 of a 99. A concave lens of focal length 15cm forms an
diverging lens. If 𝐹1 is the focal point on the other image 1/3 of the size of the object. The object
side of the lens, the image of the object is formed distance is A.10cm B.20cm C.30cm D.60cm.
between A.𝐹 and 𝑋 B.𝑿 and 𝑪 C.𝐶 and 𝐹1 D.𝐹1 1
𝑚=− (magnification is negative for concave
and 2𝐹1 . 3
𝑓 1 −15
The image formed by a concave or diverging lens lens), 𝑓=−15cm, 𝑢=?, 𝑚 = , − = ,
𝑢−𝑓 3 𝑢−(−15)
is always between the object 𝑋 and optical centre 1
− =
−15
,
𝐶 of the lens, regardless the position of the object. 3 𝑢+15
94. Which of the following instruments can be −1(𝑢 + 15) = 3 × −15, −𝑢 −15 = −45,
used to form a virtual image of an object? −𝑢 = −45+15 = −30, 𝑢 = 30cm.
I.Converging lens II.Diverging lens III.Concave 100. A thin converging lens has a power of 4
mirror A.I B.II C.III D.II and III E.I,II and III. dioptres. Determine its focal length A.0.25m
A converging lens forms a virtual image when the B.0.03m C.2.50m D.5.00.
object is placed between the focus and the lens.A 𝑃=4𝐷(positive for converging or convex lens),
1 1 1
diverging lens forms virtual image always. A 𝑃 = , 𝑓 = = =0.25m =25cm.
𝑓 𝑃 4
concave mirror forms a virtual image when the 101. A thin lens is placed 50cm from an
object is placed between the focus and the pole of illuminated object. The image produced has
the mirror.

207
Demystified Series Physics Demystified by Dr Timothy
1
linear magnification of . Calculate the power of
4
the lens in diopters A.2.5𝐷 B.5.0𝐷 C.10.0𝑫
D.25.0𝐷
1 𝑓 1 𝑓
𝑢=50cm, 𝑚= , 𝑃=?, 𝑚 = , = ,
4 𝑢−𝑓 4 50−𝑓
50
50−𝑓 = 4𝑓 , 50 = 4𝑓 + 𝑓 = 5𝑓 , 𝑓 = ,
5
1 1
𝑓 = 10cm=0.1m, 𝑃 = = = 10.0𝐷.
𝑓 0.1
102. A convex lens of power 5 dioptres forms an
erect image of magnification 3 of an object. Find
the object and image distances respectively.
A.13.33cm and 20cm B.13.33cm and 40cm
C.26.66cm and 20cm D.26.66cm and 40cm.
1
𝑃=5𝐷, 𝑚=−3(erect images are virtual) , 𝑃 = ,
𝑓
1 1
𝑓= = =0.2m =20cm.
𝑃 5
𝑣 𝑣 𝑣
𝑚 = − 1, −3 = − 1 , −3+1 = ,
𝑓 20 20
𝑣
−2 = , 𝑣 = −2×20= −40cm.
20
𝑣 𝑣 −40 𝑓
𝑚= , 𝑢= = = 13.3cm. 𝑚 = ,
𝑢 𝑚 −3 𝑢−𝑓
20 20
−3 = , 𝑢 − 20 = = −6.67,
𝑢−20 −3
𝑢 =20−6.67=13.33cm
103. A converging lens has a focal length of 5cm.
Dtermine its power A.+20.0𝐷 B.+0.2𝐷 C.−20.0𝐷
D.−0.2𝐷
1 1
𝑓=5cm=0.05m, 𝑃=?, 𝑃 = = = +20.0𝐷.
𝑓 0.05
104. A diverging lens has a focal length of 10cm.
Calculate its power A.−0.1𝐷 B.+0.1𝐷 C.−10.0𝑫
D.+10.0𝐷.
1 1
𝑓=−10cm=−0.1m, 𝑃=?, 𝑃 = = ,
𝑓 −0.1
𝑃 = −10𝐷 or −10 dioptres.
105. An object 20cm from a convex lens has an
image 25cm away from the lens.Calculate the
power of the lens A.45𝐷 B.9𝑫 C.20𝐷 D.5𝐷.
𝑢=20cm=0.2m, 𝑣=25cm=0.25m, 𝑓=?, 𝑃=?
1 𝑢+𝑣 0.2+0.25 0.45
𝑃= = = = = 9𝐷.
𝑓 𝑢𝑣 0.2×0.25 0.05

Jamb past questions on refraction of light :


[1978/42,1979/8,42,1980/14,25,35,1981/26,
32,40,44,1982/8,24,1983/7,46,1984/32,1985/
31,33,1986/28,1987/34,35,1988/30,33,1990/
26,27,1991/27,1991/29,1992/29,1993/29,
1994/34,36,1995/33,1997/35,38,1998/33,
1999/34,2000/16,19,2001/15,2002/14,16,
2003/16,29,31,38,2004/1,5,7,2005/39,2006/
38,2008/34,35,2009/25,34,2010/30,2011/32,
2012/34,35,2013/31,2015/7,16]

208
Demystified Series Physics Demystified by Dr Timothy

Chapter 19 – Optical instruments, application of light waves


● Simple microscope or magnifying glass or 𝑣𝑜 =image distance of objective lens, 𝑢𝑜 =object
hand lens – It consists of a single convex lens distance of objective lens, 𝑣𝑒 =image distance of
used to magnify near objects i.e make it eye piece lens, 𝑢𝑒 =object distance of eye piece
appear bigger.The object 𝑂is placed between lens.
the principal focus F and the lens, the image 𝐼 - Image distance of the eye-peiece 𝑣𝑒 is equal to
formed is virtual,erect and magnified (VEM). least distance of vision 𝐷, 𝒗𝒆 = 𝑫.
- 𝒎𝒐 is positive as it forms real image while 𝒎𝒆
is negative as it forms virtual images.
𝒗 𝒗
Alternatievly, 𝒎𝑳 = 𝒎𝑨 = ( 𝒐 − 𝟏) ( 𝒆 + 𝟏)
𝐼 2𝐹 𝐹 O 𝐹1 2𝐹1 𝒇𝒐 𝒇𝒆
- Distance of separation between the lenses:
- Simple microscope is used for studying or to 𝑫 = 𝒗𝒐 + 𝒖 𝒆 .
view biological specimens and for reading small ● Astronomical telescope – It consists of two
prints. convex lenses ; the objective lens of long focal
- The magnification of a simple microscope is length and eyepiece lens( or ocular) of short
𝑫 focal length (𝒇𝒐 > 𝒇𝒆 ), for viewing distant
given by – 𝒎 = −1, 𝐷=least distance of vision object, celestial or heavenly objects e.g
𝒇
and it is negative. moon,stars e.t.c, making them appear larger
For a normal eye the least distance is 25cm. and closer.
𝒗
𝒎 = −1 =
𝒇
. 𝒎 and 𝒗 are negative,as a Objective Eye-piece
𝒇 𝒖−𝒇
𝑓𝑜
virtual image is formed, while 𝒖 and 𝒇 are
𝑓𝑒
positive.
𝐼2 𝐹𝑒 𝐹𝑜 𝐹𝑒
- Simple microscope gives both linear and
𝐼1
angular magnification.
● Compound microscope – It consists of two
converging(convex lens) lenses; an objective lens
which is closer to the object and an eyepiece lens(
Abnormal use
or ocular) which is close to the observer’s eye.
- Distant object e.g stars, forms an image 𝐼1 at the
Both lens have short focal lengths with the focal
focus 𝑓𝑜 ,of the object lens which is real, inverted
length of the eye piece 𝒇𝒆 being greater than
and diminished(RID). The objective image 𝐼2
that of the objective 𝒇𝒐 (𝒇𝒆 > 𝒇𝒐 ) or (𝒇𝒐 < 𝒇𝒆 ,
which lies within the focus 𝑓𝑒 of the eye piece,
𝒇𝒐 ●𝒇𝒆 ). Eyepiece
acts as the eyepiece object.Hence,a final
Objective
virtual,inverted and magnified image (VIM) is
formed at the near point (25cm from the
𝐼2 𝐹𝑜 𝐹𝑒 𝐼1 𝐹𝑒
eyepiece) of the observer.
𝑂
- Distance of separation between lenses or length
of an astronomical telescope abnormal
adjustment: 𝑫 = 𝒇𝒐 + 𝒖𝒆 .
- The magnifying power of astronomical
telescope is due to the fact that the angle
- The object 𝑂 is placed between 𝐹𝑜 and 2𝐹𝑜 (𝑢 >
substended at the eye by the final image at
𝑓𝑜 ), of the objective lens. The image formed 𝐼1 -
infinity is very much greater than that susbended
(object of the eye piece) is real,inverted and
by the distant object.
magnified (RIM) and is beyond 2𝐹𝑜 , which is less
- The inverted image formed by the astronomical
than the focal length of the eye piece,hence the
telescope is a disadvantaged when viewing the
final image 𝐼2 is virtual,inverted and magnified
earth but can be tolerated when viewing
(VIM) and is formed beyond the object.
heavenly bodies eg stars.
-The eye piece acts as a magnifier or
- In normal adjustment or normal use–
magnifying glass.
1.The focus of the objective𝑓𝑜 coincides with the
- Compound microscope can produce both linear
focus of eyepiece𝑓𝑒 . 2.The final image is virtual,
𝒎𝑳 and angular magnification 𝒎𝑨 .
inverted and magnified but formed at infinity.
- Magnifying power of a compound microscope,
3.The distance of separation between the lenses
𝑴 = 𝒎𝑳 = 𝒎 𝑳 = 𝒎 𝒐 × 𝒎 𝒆 .
𝒗 or length of the telescope: 𝑫 = 𝒇𝒐 + 𝒇𝒆 . 4.The
magnification by the objective lens 𝒎𝒐 = 𝒐 .
𝒖𝒐 magnifying power of the astronomical telescope
𝒗𝒆
magnification by the eye piece lens 𝒎𝒆 = . is equal ratio of the focal length of the objective
𝒖𝒆

209
Demystified Series Physics Demystified by Dr Timothy
to the focal length of the eyepiece: 𝑴 =
𝒇𝒐
. diminished(RID) image at the objective focus
𝒇𝒆
which coincides with the centre of curvature
Objective Eye-piece 2𝑭of the inverting lens,to produce another
𝑓𝑜 𝑓𝑒 image(real,re-inverted to become erect and same
size) at the same centre of curvature 2F. The 2𝑭of
𝐹𝑜 the inverting lens coincides with the focus of the
𝐹𝑒 eyepiece, hence the final image is virtual,erect
𝐼1 and maginified(VEM) image at infinity.
- The distance between the objective and
eyepiece lens: 𝑫 = 𝒇𝒐 +4𝒇 + 𝒇𝒆 .
Normal adjustment - The advangtage of a terrestrial telescope is
● Galilean or Galileo’s telescope or Opera the production of erect(upright) image, hence
glass – It consists of a converging lens of long can be used on earth. Its disadvantage is that
focal length as the objective and a diverging lens the inverting lens extends the length of the
of short focal lengthas the eye piece. telescope by 4𝒇, making it too long.
- Normal adjustment – The objective lens ● Slide Projector or Projection lantern – This
collects parallel rays from a distant object to form is an instrument for producing a magnified or
a real,inverted and diminished image at objective enlarged image of a transparent object (slide) on
focus.The parallel rays are however intercepted a screen.
by the eyepiece diverging lens before they - It consists of – i.The power light source
converge at the objective focus, which causes the situated at the centre of curvature the concave
rays to diverge,producing a final virtual,erect and mirror, which illuminates the object(slide).
magnified image (VEM), at infinity. ii.The concave mirror directs and reflects the
- Abnormal use or adjustment – The Galilean light onto the condenser. iii.The condenser of
telescope is adjusted such that the final image is two plano convex lenses, converge the light
formed at near point. through the slide to produce a bright image. It
- Distance of separation between the lenses or usually has an infra red filter between the
length of a Galilean telescope is equal to the lenses to prevent the passage of heat to the slide.
digfference in the focal length of the lenses. iv.The slide(object) is between 𝒇 and 2𝒇(𝒇 <
𝑫 = 𝒇o−𝒇e. 𝒖 < 2𝒇)of the adjustable projection lens thus, a
- Magnifying power of a Galilean telescope is real, inverted and magnified image is formed
given by the ratio of the focal length of the on the screen which is placed beyond 2𝒇 of the
𝑓
objective to the that of the eyepiece. 𝑀 = 𝑜 lens.
𝑓𝑒
Objective Eyepiece - Magnification in a projector 𝑚 is given by ;
𝒊𝒎𝒂𝒈𝒆 𝒉𝒆𝒊𝒈𝒉𝒕 𝑯𝑰 𝒊𝒎𝒂𝒈𝒆 𝒃𝒓𝒆𝒂𝒅𝒕𝒉 𝑩𝑰
𝑓𝑜 𝒎= = =
𝒐𝒃𝒋𝒆𝒄𝒕 𝒉𝒆𝒊𝒈𝒉𝒕 𝑯𝑶 𝒐𝒃𝒋𝒆𝒄𝒕 𝒃𝒓𝒆𝒂𝒅𝒕𝒉 𝑩𝑶
𝑓𝑒 𝐹𝑒 𝒊𝒎𝒂𝒈𝒆 𝒍𝒆𝒏𝒈𝒕𝒉 𝑳𝑰 𝒗
𝐹𝑜 . 𝒎 = −𝟏.
𝒐𝒃𝒋𝒆𝒄𝒕 𝒍𝒆𝒏𝒈𝒕𝒉 𝑳𝑶 𝒇
- Length of a hall where a slide projector is
used=distance between the slide(object) and
- Galilean telescope can be used on earth as it the screen= 𝒖 + 𝒗.
forms erect image i.e its advantage over ● Human eye – Parts of the human eye ;
astronomical telescope. It has small field of i.Cornea -: Transparent membrane covering the
view and poor magnification. eye in front. ii.Pupil -: Adjustable opening in the
- Galilean telescope is smallest in size and iris through which light passes into the eye.
shortest in length i.e its advantage over other iii.Iris -: Muscular coloured part of the eye which
telescope. controls the size of the pupil iv.Converging lens
- Galilean telescope in abnormal use can be -: An elastic lens of variable thickness or focal
used to view actors on the stage in a theatre. length for focusing object on the retina. v.Retina
● Terrestrial telescope – It consists on extra -:Photosensitive part of the eye where image is
convex lens(inverting lens) placedbetween the formed.
objective and eyepiece lens.The extra lens helps - Near and Far points – Near point is the nearest
to re-invert the image to form a final erect distance of clear vision while the Far point is the
image. farthest distance of clear vision. In a normal
Objective lens Inverting lens Eyepiece eye,the near point is 25cm whilefar point is at
infinity(∞).
- Accomodation – It is the ability of the eye lens
𝑓𝑜 2𝑓 2𝑓 𝑓𝑒 to focus on near and far objects by alteration of
its focal length.
- The object lens forms a real,inverted and - To focus on near objects,the ciliary muscle

210
Demystified Series Physics Demystified by Dr Timothy
relax,buldges the lens and shorten its focal controlling the amount of light entering the
length. To focus on far objects, the ciliary muscles camera. iv.Shutter: It opens to expose the film to
contracts,stretches the lens and increases the light for a short period of time v.Photographic
focal length. The ciliary muscles is primarily film: Light-sensitive sheet where the image is
responsible for accommodation. formed.
- Persistence of vision or persistent vision – Comparison of the human eye and lens
This is the retention of an impression of vision of camera
image on the retina for a period of time after Human eye Lens camera
object has been removed i.e making light seen 1.Flexible biological Rigid non-biological
appear to have costant intensity.This lens lens
phenomenon is used in the showing and 2.Iris Diaphragm
production of motion pictures in cinema 3.Retina Photographic film
films. 4.Pupil Aperture
- Binocular or Stereoscopic vision -: Blending 5.Distance between Distance between lens
or overlapping of two images formed by both lens and retina is and film is
eyes to form one distinct image in the brain.It is fixed adjustable/variable
imposible with one eye or when the field of 6.Eye lid Shutter
view don’t overlap. 7.Focal length of lens Focal length of lens is
- Eye defects – is variable fixed
1.Astigmatism – This occurs due to loss of - Both lens camera and the human eye form a
power of accommodation.It is the ability to see of real,inverted and dimished image.
objects clearly(blurring) in one direction than in
one direction than the other or inability of the eye Examples :
to focus on both far and near objects.It can be 1. The optical instrument for viewing near
corrected using a cylindrical lens. objects is the A.telescope B.sextant C.microsope
2.Presbyopia – The inability of the eye to focus D.binoculars.
near objects or the loss of power of Simple microscope is used magnifying near
accommodation due to loss of elasticity in the objects in order to view them.
ciliary muscle due to old age or hardenening/ 2. The magnified and virtual image of a near
weakening of the eye lens.It can be corrected object is produced by A.Prism binoculars
using a bifocal lens(combination of concave B.Astronomical telescope C.Periscope D.Simple
and convex lens). microscope.
3.Myopia (Short-sightedness) – Ability of the The image formed by a simple microscope is
eye to see near objects clearly and inability to see virtual, erect and magnified.
distant object clearly,due to the eye ball being too 3. A man uses a converging lens of focal length
long or strong eye lens(short focal length).Image 10cm as a magnifying glass. How far away from
of distant object is focused in front of the retina.It the lens must an object be placed form him to see
can be corrected using a concave (diverging) its image at 25cm from the lens? A.6.7cm
lens. B.7.1cm C.12.9cm D.15.0cm.
4.Hypermetropia or Hyperopia (Long- 𝑓=10cm, 𝑣=−25cm(virtual image), 𝑢=?,
sightedness) – Ability of the eye to see distant 𝑢=
𝑣𝑓
=
−25×10
=
−250
= 7.1cm.
objects clearly and inability to see near objects 𝑣−𝑓 −25−10 −35

clearly,due to the eye ball being too short or weak 4. A converging lens of focal length 3.0cm is used
eye lens(long focal length).Image of near object is as a simple microscope. If the final image is
focused in behind the retina.It can be corrected formed at a distance 30cm from the lens,
using a convex (converging) lens. calculate the linear magnification A.0.1 B.11.0
For a short-sighted person – Image distance= C.1.0 D.9.0 E.10.0.
distant object or distance that cannot be seen 𝑓=3.0cm, 𝑣=−30cm, 𝑚=?,
𝑣 30
clearly. Image distance is negative and a 𝑀 = –1 = − −1 = −10−1 = −11
𝑓 3
concave lens of focal length equal to the image 𝑚=11(the image is virtual, as magnification is
distance(far point of the sufferer) is needed. negative).
- For a long-sighted person – Image distance= 5. A simple magnifying glass is used to view an
closer object=distance cannot be seen clearly. object. At what distance from the lens must the
Image distance is also negative. object be placed so that an image 5 times the size
- Lens camera – It consists of : i.Lens: of the object is produced 20cm from the lens?
convexlens whose distance from the film is A.2.0cm B.3.3cm C.4.0cm D.5.0cm E.10.0cm.
adjustable ii.Aperture: opening through which 𝑣
𝑚=−5, 𝑣=−20cm, 𝑢=?, 𝑚 = , 𝑢 = ,
𝑣

light enters the camera iii.Diaphragm: It −20


𝑢 𝑚

regulate the size of the aperture, therby 𝑢= = 4.0cm.


−5

211
Demystified Series Physics Demystified by Dr Timothy
6. A magnifying glass is used to view a biological microscope objective? A.−4mm B.−4cm
specimen.The distance between the lens and the C.+4mm D.+4cm.
specimen is 10cm. If the image is formed at a The converging lenses in compound microscope
distance of 25cm from the lens, calculate the have short focal lengths,with that of the objective
magnification A.2/5 B.0.4 C.2.5 D.1. lens being shorter than the eye piece. The focal
𝑢=10cm, 𝑣=−25cm, 𝑚=?, 𝑚 = =
𝑣 −25
= −2.5. length will be +4mm(convex lens have positive
𝑢 10
focal length) and is the smallest.
8. A simple microscope forms an image 10cm
15.
from an eye close to the lens.If the object is 6cm
O
from the eye, calculate the focal length of the lens
𝐹𝑜 𝐹𝑒 𝐼1
A.15.00cm B.3.75cm C.4.00cm D.16.00cm
𝑢𝑣 6×−10 𝐼2
𝑣=−10cm, 𝑢=6cm, 𝑓=?, 𝑓 = = ,
𝑢+𝑣 6+(−10)
−60 −60
𝑓= = = 15cm.
6−10 −4
What does the diagram above represents?
9. An observer with normal eyes views an object
A.microscope in normal use B.telescope in
with a magnifying glass of focal length 5cm.The
abnormal use C.microscope in abnormal use
angular magnification is A.−6 B.−5 C.5 D.6
D.telescope in normal use.
𝑓=5cm, 𝐷=least distance of vision=−25cm(for a
𝐷 −25 The object is closer to the lens i.e near object and
normal eye) , 𝑚 = − 1 = − 1=−5−1 = −6. a microscope is used for near objects. In
𝑓 5
10.. A compound microscope makes use of A.two abnormal use of a compound microscope,the
diverging lenses to produce an enlarged image of image is formed within the lens i.e 25cm from the
a very distant object B.two converging lens to eye. In normal use of a compound
produce a diminished image C.two diverging microscope,the image distance of the
lenses to produce a diminished image of a near objective lens coincides with the focal length
object D.two converging lenses to produce an of the eye piece and the final image is formed
enlarged,inverted image of a near object E.a at infinity.
combination of converging and diverging lenses 16. Which of the following relations about the
to produce the image of both distant and near focal length 𝑓𝑜 of the objective and focal length 𝑓𝑒
objects. of the eyepiece of a compound microscope is
Compound microscope uses two converging correct? A.𝑓𝑜 = 𝑓𝑒 B.𝒇𝒐 < 𝒇𝒆 C.𝑓𝑜 > 𝑓𝑒 D.𝑓𝑜 = 2𝑓𝑒 .
lenses of short focal lengths to produce a real, In a compound microscope the focal length of the
inverted and magnified image for the objective eyepiece is greater than that of the objective or
and virtual,inverted and magnified image in the the focal length of the objective is less than that
eye piece,of anear object placed close to the of the eyepiece. 𝑓𝑒 > 𝑓𝑜 or 𝑓𝑜 < 𝑓𝑒 .
objective. 17. A compound microscope has lenses of focal
11. A compound microscope makes use of A.two length 1.0cm and 5.0cm. An object is placed
diverging lenses to produce an enlarged image of 1.2cm from the objective lens,if a virtual image is
a very distant object B.two converging lenses to formed 25cm from the eye piece,calculate the
produce a diminished and erect image of a distance between the two lenses? A.10.2cm
distant object C.two diverging lenses to produce B.12.3cm C.11.5cm D.10.5cm.
a diminished image of a near object D.two Object distance of objective lens 𝑢o=1.2cm, focal
converging lenses to produce an enlarged length of objective lens 𝑓o=1cm, image distance
inverted image of a near object E.a of objective lens 𝑣𝑜 =?
combination of converging and diverging lenses 𝑢 𝑓
𝑣𝑜 = 𝑜 𝑜 =
1.2×1
=
1.2
= 6cm.
𝑢𝑜 −𝑓𝑜 1.2−1 0.2
to produce the images of both distant and near
objects. Object distance of eye piece lens 𝑢𝑒 =?, focal
12. If 𝐹 represents the principal focus of the length of eye piece 𝑓𝑒 =5cm, image distance of eye
piece lens 𝑣𝑒 =−25cm (the image of eye piece is
objective lens of a compound microscope, the 𝑣 𝑓 −25×5 −125
object to be viewed is placed A.at 𝐹B.between 𝑭 virtual), 𝑢𝑒 = 𝑒 𝑒 = = ,
𝑣𝑒 −𝑓𝑒 −25−5 −30
and 2𝑭 C.between 𝐹 and the objective lens D.at 𝑢𝑒 = 4.16cm = 4.2cm. Distance of separation
2𝐹 E.beyond 2𝐹. between lenses in a compound microscope is
The object is placed between 𝐹 and 2𝐹 of the equal to the sum of the image distance of the
objective lens, and the image is formed beyond objective lens and object distance of the eye
2𝐹 of the objective. piece. 𝐷 = 𝑣𝑜 + 𝑢𝑒 =6+4.2 =10.2cm.
13. In a compound microscope, the objective and 18. If the linear magnification of the objective
the eyepiece focal lengths are A.long B.short and eyepiece convex lenses of a compound
C.the same D.at infinity. microscope are 4 and 7 repectively, calculkate
14. Of the four lenses whose focal lengths are the angular magnification of the microscope A.2
provided below, the most suitable for use as a B.3 C.11 D.28.

212
Demystified Series Physics Demystified by Dr Timothy
Magnification of objective lens 𝑚𝑜 =4, The final image of a compound microscope is
magnification of eyepiece 𝑚𝑒 =7, anular inverted and within the lenses or at near point
magnification or magnifying power of compound from the eye. The final image of an astronomical
microscope 𝑀=?, 𝑀 = 𝑚𝑜 × 𝑚𝑒 = 4×7 = 28. telescope is always inverted. The objective of a
19. In a compound microscope, the initial image compound microscope has a shorter focal length
is twice the object while the final image than the eyepiece.
is four times the object. Calculate the magnifying 24. An astronomical telescope is said to be in
power of the microscope A.0.5 B.2.0 C.4.0 D.6.0 normal adjustment when the A.Final image is at
E.8.0. infinity B.Final image is at the near point of eye
The final image is four times the object (not the C.Focal length of the objective is longer that that
initial image, the magnifiying power, 𝑀 = 4.0. of eyepiece D.Eye is accommodated.
4
Alternatively, 𝑚𝑜 =2, 𝑚𝑒 = = 2, as the final image 25. What optical instrument can best be
2
constructed with converging lens of focal lengths
is four times the object and the initial image is
twice the object. 𝑀 = 𝑚𝑜 × 𝑚𝑒 = 2×2 = 4.0. 50cm and 5cm? A.Compound microscope
B.Terrestrial telescope C.Astronomical
20. The focal length of the objective and eye piece
telescope D.Galileo’s telescope.
in a compound microscope are +0.50cm and
Astronomical telescope uses two converging
+2.00 cm respectively.The instrument is focused
on an object 0.52cm from the objective lens and lenses: the objective with long focal length
(50cm) and eyepiece with short focal length
a virtual image is viewed by the eye at a distance
(5cm).
of 25cm.Calculate the magnifying power of the
microscope. A.313 B.334 C.364 D.338. 26. An astronomical telescope has objective and
eyepiece lenses of focal lengths 3.5m and 5cm
𝑓𝑜 =0.5cm, 𝑓𝑒 =2cm, 𝑢𝑜 =0.52cm, 𝑣𝑒 =𝐷=25cm
𝑢 𝑓 0.52×0.5 0.26 respectively. Determine the magnifying power of
𝑣𝑜 = 𝑜 𝑜 = = =13cm. the telescope when in normal adjustement
𝑢𝑜 −𝑓𝑜 0.52−0.5 0.02

𝑀 = 𝑚𝑜 × 𝑚𝑒 = (
𝑣𝑜 𝑣
− 1) ( 𝑒 + 1) , A.70.0 B.17.5 C.7.0 D.0.7.
𝑓𝑜 𝑓𝑒 𝑓
13 25 𝑓𝑜 =3.5m=350cm, 𝑓𝑒 =5cm, 𝑀=?, 𝑀 = 𝑜 ,
𝑀 = ( − 1) ( + 1) = (26−1)(12.5+1) , 350
𝑓𝑒
0.5 2
𝑀 = (25)(13.5) =337.5 =338. 𝑀= = 70.0.
5
21. Which of the following is not true about an 27. An astronomical telescope has an eyepiece of
astronomical telescope in normal adjustment focal length 5cm. If the angular magnificat- ion in
A.the focal length of the objective lens is greater normal adjustment is 10, what is the distance
than that of the eye piece B.the distance between between objective and eyepiece in cm? A.110cm
the objective lens and the eyepiece is the sum of B.55cm C.50cm D.45cm.
their focal lengths C.its final image of a star is 𝑓𝑒 =5cm, 𝑀=𝑚𝐴 =10, 𝑓𝑜 =?
𝑓
virtual,erect and at infinite distance D.its final 𝑀 = 𝑜 , 𝑓𝑜 = 𝑀𝑓𝑒 = 10×5 = 50cm.
𝑓𝑒
image of a star is real,erect and at an infinite
Distance between lenses in normal adjustment
distance E.its angular magnification is the ratio
𝐷 = 𝑓𝑜 + 𝑓𝑒 =50+5 =55cm.
of the focal length of the objective to that of the
28. An astronomical telescope has an angular
eyepiece.
magnification of 7. If the distance between the
In normal adjustment, the final image of a
lenses in normal adjustment is 45cm, calculate
heavenly body e.g start, is virtual, inverted
the focal length of the lenses.
magnified but at infinity or infinite distance.
𝑀=𝑚𝐴 =7, 𝐷=45cm, 𝑓𝑜 =?, 𝑓𝑒 =?,
22. A telescope is said to be in normal adjustment 𝑓
when the A.focal length of objective is greater 𝑀 = 𝑜 , 𝑓𝑜 = 𝑀𝑓𝑒 = 7× 𝑓𝑒 = 7𝑓𝑒 ,
𝑓𝑒
than that of the eyepiece B.objective focal point 𝐷 = 𝑓𝑜 + 𝑓𝑒 , 45 = 7𝑓𝑒 + 𝑓𝑒 = 8𝑓𝑒 ,
coincides with that of the eyepiece C.focal 45
𝑓𝑒 = =5.625cm, 𝑓𝑜 = 𝑀𝑓𝑒 = 7× 𝑓𝑒 = 7𝑓𝑒 ,
length of the eye piece is greater than that of the 8
𝑓𝑜 = 7×5.625 = 39.375cm.
objective D.focal length of the eyepiece is equal to
- Question Drill – 1.The focal length of the lenses
that of the objective.
of an atronomical telescope are 5.0cm and 100cm
In normal adjustment the focus of the objective
respectively.Calculate the length of the telescope,
coincides with that of the eyepiece.
if the object is at infinity and the final image is
23. Which of the following statements about the
formed 25.0cm from the eye. Ans=104cm.
compound microscope and astronomical
Hint: It is a case of abnormal adjustment,as the
telescope is.are correct? I.The final image of a
final image is formed at near point i.e 25cm for a
compound microscope is located at infinity II.The
normal eye. For normal adjustment the final
final image of the astronomical telescope is
image is formed at infinity. 2.An astronomical
always erect III.The objective of a compound
telescope in normal use has an angular
microscope has a very short focal length A.I B.II
magnification of 4 and the lenses are at a distance
C.III D.I and III E.II and III.

213
Demystified Series Physics Demystified by Dr Timothy
of 25cm, calculate the focal length of the 𝑣= distance of the screen from lens=10m
eyepiece. Ans=5cm. 𝐿 𝐵 𝑣
=1000cm, 𝑢=?, 𝑓=? 𝑚 = 𝐼 = 𝐼 = ,
300
=
𝐿𝑂 𝐵𝑂 𝑢 7.62
29. In a Galilean telescope,the objective and 1000 7.62×1000
eyepiece have focal lengths of 60cm and 10cm , 𝑢= = 25.4cm.
𝑢 300
respectively.What is the separation of these 𝑓=
𝑢𝑣
=
25.4×1000
=
25400
= 24.7cm.
𝑢+𝑣 25.4+1000 1025.4
lenses when the telescope is used in normal
40. A projection lantern is used to give the image
adjustment? A.6cm B.10cm C.50cm D.60cm
of a slide on a screen. If the image is 24 times as
E.70cm.
large as the slide and the screen is 72.0m from
𝑓𝑜 =60cm, 𝑓𝑒 =10cm, Distance of separation of
the projecting lens, what is the position of the
lenses in a Galilean telescope 𝐷 = 𝑓𝑜 − 𝑓𝑒 ,
slide from the lens? A.4.0m B.3.5m C.3.0m
𝐷 = 60−10=50cm.
D.0.3m.
30. An optical instrument was made with a 𝑣 𝑣 72.0
convex lens of focal length 50cm and a concave 𝑚=24, 𝑣=72.0m, 𝑢=?, 𝑚 = , 𝑢 = = = 3.0.
𝑢 𝑚 24
lens of focal length 5cm.This instrument must be 41. The use of lenses is not applicable in the
A.a Galilean telescope B.an astronomical A.projector B,human eye C.periscope
telescope C.a slide projector D.a compound D.telescope.
microscope. 42. If the focal length of a camera lens is 20cm,the
31. The Galilean telescope has an advantage over distance from the film at which the lens must be
other telescopes in that it A.produces an erect set to produce a sharp image of an object 100cm
image B.is shortest in length C.has the best away is A.17cm B.20cm C.25cm D.100cm
magnification D.produces the sharpest image 𝑓=20cm(lens of a camera is a convex lens),
32. Which of the following statements about the 𝑢=100cm, distance of the film from the lens=
𝑢𝑓 100×20
Galilean telescope is not correct? A.The final image distance=𝑣=?, 𝑣 = = ,
𝑢−𝑓 100−20
image is inverted B.It is shorter than terrestrial 2000
telescope C.The final image is erect D.It has a 𝑣= = 25cm.
80
small field of view. 43. The following are parts of the eye I.retian
33. The terrestrial telescope has one extra II.pupil III.iris. The correct equivalent in the
convex lens more than astronomical telescope. camera in the same order are A.film, aperture,
The extra lens is for A.magnification of the image diaphragm B.film, diaphragm, aperture
B.erection of the image C.improving the C.diaphragm, aperture, film D.aperture,
sharpness D.creating an inverted image. diaphragm, film.
34.Which of the following makes use of a concave Retina is similar to the film,pupil is similar to the
mirror? A.camera B.kaleidoscope C.periscope aperture and iris is similar to the diaphragm.
D.simple microscope E.slide projector 44. The eye controls the amount of light reaching
35. Which of the following is not a component of the retina by adjusting A.Iris B.Optic nerve
a slide projector? A.concave mirror B.screen C.Cornea D.Retina.
C.condenser D.convex mirror E.light source. 45. Certain similarities between the eye and the
36. The purpose of the condenser in a film camera is/are listed below.Which of the
projector is to A.cast an image of the film on the following statements is/are not true? I.both the
screen B.prevent the image from being blurred camera and the eye have a light sensitive screen
C.make the image brighter D.invert the image II.inverted images are formed on the screen in
laterally E.turn the image upright on the screen. both III.the distance between the lens and the
37. At which of the following distances from the screen is fixed in both IV.the amount of light
lens should a slide be placed in a slide projector entering both can be adjusted A.I B.II C.III D.II
if 𝑓 is the focal length of the projection lens A.less and III E.I and IV.
than 2𝑓 B.greater than 𝒇 but less than 2𝒇 The distance between the lens and the screen is
C.greater than 2𝑓 D.equal to 𝑓 E.equal to 2𝑓. fixed in the human eye but can be adjusted in a
38. In a projection lantern of focal length 𝑓, the camera.
object distance 𝑢 is such that A.𝑢 > 2𝑓 > 𝑓 B.𝑢 < 46. The image of an object on the retina of the
𝑓 < 2𝑓 C.𝑢 = 𝑓 < 2𝑓D.𝒇 < 𝑢 < 2𝒇. human eye is A.virtual and diminished
39. A lantern gives an image 3m square of slide B.inverted and real C.erect and diminished
7.26cm square on a screen. If the screen is 10m D.erect and real.
from the projection lens of the lattern,calculate 47. The ability of the human eye to focus objects
the focal length of the lens A.40.3cm B.26.1cm on the retina is referred to as the power of
C.24.7cm D.0.7cm E.0.3cm. A.interference B.diffraction C.accommodation
Image 3m square: image length 𝐿𝐼 =3m= 300cm, D.superposition.
image breadth 𝐵𝐼 =3m= 300cm, Object (slide) 48. The property of the eye knon as its power of
7.26cm square: object length 𝐿𝑂 =7.26cm, object accommodation is controlled by the A.pupil
breadth 𝐵𝑂 =7.26cm, B.vitreous humour C.iris D.ciliary muscles.

214
Demystified Series Physics Demystified by Dr Timothy
49. The range of accommodation of a normal point, to 25c in front of the eye i.e near point.
human eye is A.0m to 0.1m B.0m to 0.25m Longsighted people have short eye-ball and near
C.0.10m to 0.25m D.0.10 t0 infinity E.0.25m to objects are focused behind retina.
infinity. Longsightedness is corrected by converging or
Range of vision for the human eye is from near convex lens. Shortsighted people can see near
point i.e 25cm(0.25m) to far point i.e infinity(∞). objects clearly but cannot see far or distant
50. Prebyopia is a defect of the eye resulting from objects clearly.
A.weak ciliary muscle B.short eye ball C.loss of 59. Which of the following statements about
sphericity of the lens D.long eye ball Presbyopia defects of vision is/are correct? I.for a long-
is due to the weakening or loss sphericity of the sighted person,close object appear blurred II.for
eye lens or loss of elasticity in the ciliary muscle a short-sighted person,distant object appear
at old age. blurred III.short-sight is corrected using a pair of
51. The lack of power of accommodation which converging lens A.I B.II C.I and II D.II and III E.I,II
is mainly due to hardening of th eye lens and is and III.
usual with the onset of old age is called A.myopia Short-sight is corrected using a diverging lens
B.hypertropia C.presbyopia D.astigmatism. (concave) while Long-sight is corrected using a
52. When the human eye loses its power of converging lens(convex).
accommodation, the effect is known as 60. Which of the following statements is correct
A.astigmatism B.long-sightedness C.short- about a long sighted boy who does not put on
sightedness D.presbyopia. glasses A.he cannot see distant object clearly
53. Which of the following eye defects can be B.rays of light from close object are focused in
corrected using a cylindrical lens? front of the retina C.his eye ball is too long
A.Astigmatism B.Presbyopia C.Chromatic D.parallel rays of light are focused behind his
aberration D.Myopia. retina (None of the options are correct).
54. For a short sighted person, light rays from a A long-sighted person can see distant object
point on a very distant object is focused A.In clearly and cannot see near objects clearly i.e
front of the retina B.On the retinal by a they appear blurred,eye ball is too short,rays of
converging lens C.Behind the retina by a light form a close object are focused behind the
diverging lens D.In front of the retina at distance retina while parallel rays of light from distant
2𝐹 from the lens. object falls on the retina. The opposite holds for a
A short-sighted person can focus near objects short-sighted person.
clearly on the retina but cannot focus distant 61. A far sighted person cannot see objects that
object. Distant object are focused in front of the are less than 100m away. If this person wants to
retina and can be made to focus on the retina by read book 25cm away,what type and focal length
a diverging or concave lens. of lens does he need? A.convex, 33cm B.convex,
55. Which of the following is used for the 20cm C.concave, 33cm D.concave, 20cm.
correction of short-sightedness? A.Concave lens 𝑢=25cm, 𝑣=−100cm(image distance is negative)
B.Concave mirror C.Convex mirror D.Convex 𝑓=? , 𝑓 =
𝑢𝑣
=
25×−100
,
𝑢+𝑣 25+(−100)
lens. −2500 −2500
56. For the correction of myopic defects in the 𝑓= = = 33.3cm
25−100 −75
human eye we reguire A.a convex lens B.a A converging or convex lens of focal length 33cm
concave lens C.a combination of concave and is required.
convex lenses D.a prism E.a concave mirror. 62. A man wears convex lens glasses of focal
57. For correcting long sight defects in the length 30cm in order to correct his eye defect.
hu7man eye we require a A.Converging lens Instead of optimum 25cm, his least distance of
B.Diverging lens C.Microscope D.Periscope vision is A.14cm B.28cm C.75cm D.150cm.
E.Plane glass sheet. 𝑓=30cm, 𝑢=25cm, 𝑣=?, 𝑣 =
𝑢𝑓
=
25×30
,
58. Which of the following is not correct? A.The 750
𝑢−𝑓 25−30

average range of distinct vision for normal eye is 𝑣= = −150cm. The image will appear to be
−5
from the far distance(infinity) up to about 25cm at 150cm which will be clearly seen by the man.
in front of the eye B.Longsighted people have 63. A certain far-sighted person cannot see
difficulty in making the eye lens sufficiently object that are closer to the eye than 50cm
powerful to focus nearby objects C.Shortsighted clearly. Determine the power of the converging
people cannot accommodate distant objects lens which will enable him to see at 25cm
D.Long sighted people need diverging A.0.03𝐷 B.0.04𝐷 C.0.06𝐷 D.0.02𝑫.
spectacles lenses E.A person with long sight has 𝑢=25cm, 𝑣=−50cm, 𝑓=?, 𝑃=?
his eye-ball too short and the image of an object 𝑃= =
1 𝑢+𝑣
=
25+(−50)
=
−25
= 0.02𝐷.
is therefore formed behind the retina. 𝑓 𝑢𝑣 25×−50 −1250

The range of distinct vision if from infinity i.e far 64. A patient with a sight defect has a least

215
Demystified Series Physics Demystified by Dr Timothy
distance of distinct vision of 150cm.For him to be
able to read a material placed at a distance
25cm,what is the focal length of the glasses he
should wear? A.15.0cm B.30.0cm C.17.6cm
D.21.4cm.
𝑢𝑣
𝑢=25cm, 𝑣=−150cm, 𝑓=?, 𝑓 = ,
𝑢+𝑣
25×−150 −3750
𝑓= = = 30cm.
25+(−150) −125
65. A person can focus an object only when it lies
within 200cm from him. Which spectacles should
be used to increase his maximum distance of
distinc vision to infinity? A.Concave lens
B.Plane glasses C.Binoculars D.Convex lens.
The person is shortsighted, as he cannot focus
image of objects greater than 200cm, thus his
maximum distance of distinc vision can be
increased from 200cm to infinity by concave lens.
66. A near sighted person cannot see objects
clearly when they are beyond 70cm from his
eyes.What is the focal length of the correcting
lens he should use? A.−25cm B.25cm C.−70cm
D.70cm.
A concave lens is used to correct short-
sightedness,as it makes object placed at infinity,
𝑢=∞, appear to be at the persons far point,
1 1 1 1 1 1
𝑣=−70cm. + = , + = ,
𝑢 𝑣 𝑓 ∞ −70 𝑓
1 1 1
= 0, = , 𝑓 = −70cm ( concave lens)
∞ −70 𝑓
In myopic(short-sighted) persons generally,a
concave mirror of focal length equal to the
persons far-point or image distance is needed.
𝑓 = 𝑣 = −70cm(from the question above).
67. A myopic man with a far point of 2m.What
will be the prescription for the eye glasses he will
use? A.−0.1 diopter B.−0.5 diopter C.0.5
diopter D.1.0 diopter.
𝑣=−2m (far point), 𝑢=∞, 𝑓 = 𝑣 = − 2m,
1 1
𝑃 = = = −0.5𝐷.
𝑓 −2

Jamb past questions on optical instruments


and application of light waves :
[1979/11,1984/31,1986/30,1987/37,1999/31,
2006/33,2014/31,32]

216
Demystified Series Physics Demystified by Dr Timothy

Chapter 20 – Dispersion of light, Electromagnetic Spectrum


● Dispersion – Dispersion is the separation or
splitting of a beam of white light into its Red
components colours or spectra.
- Angular dispersion of light by a prism is due to Violet
difference in colour (wavelength), speed and - When a second prism is inverted, and placed
refractive indices of the light. between the first prism and the screen, a spot of
- Wavelength is the characteristic of light that white light is formed on the screen due to
determines its colour. A monochromatic light recombination of the spectrum.
is a light of one colour or wavelength. e.g White light
green light. When a monochromatic light is
passed through a prism, refraction occurs
without dispersion.
Green light is a monochromatic - A spectrometer is used to produce a pure
light. spectrum. Pure spectrum is that in which the
colours are clearly separated or distinct from
each other i.e they do not overlap.
-A spectrometer consists of:
- All colours of light travels with same speed in a 1.A divergent beam of light behind a narrow slit
vacuum. placed at the principal focus of the first
-The component colours or spectrum of white converging lens in order to produce parallel rays.
light are red,orange,yellow,green,blue,indigo and The slit produces a series of narrow coloured
violet (ROYGBIV). images which minimizes overlapping of
- When white light is passed into a glass prism,it colours.
is separated into a spectrum on a screen 2.The converging lens which produces parallel
consisting of red,orange,yellow,green, rays that are incident on the prism.
blue,indigo and violet (ROYGBIV). 3.A 60° prism that disperses the white light into
- Red is least deviated,has the fastest speed in coloured spectrum of light.
glass,highest wavelength,least refractive index 4.The second converging lens which collects the
and least change of speed. The opposite holds for parallel spectrum of light and brings them to
violet. focus on the screen,which is at the focal plane of
the lens.
Red R - Inorder to produce a pure spectrum, a slit,
Orange O
Yellow Y two converging or convex lens and triangular
Green G prism is required.
Blue B Converging lens Converging lens
Indigo
n I Slit
Violet V
- The deviation increases in the order ROYGBIV, R
with red being least deviated while violet is most
deviated. The variation can be determined using V
the expression below: Light source Prism pure spectrum
𝒏∆𝒗 - Recombination of colours – The spectrum of
𝑫∝ , 𝐷=deviation, 𝑛=refractive index,
𝒗 white light (ROYGBIV) can be recombined to give
∆𝑣=change in speed, 𝑣=speed in glass, white light by : 1.Inverting prism 2.Plane mirror
=wavelength of light. 3.Newton’s colour disc -
Thus, as deviation increases from red to violet, Colour of objects –The colour of an opaque
their refractive index and change in speed of the object is the colour of light it reflects while the
light colours increases while their speed in glass colour of a transparent object is the colour of
and wavelength decreases. light it transmits.
- Rainbow is formed by the dispersion of - An object will appear white if all colours of
sunlight by spherical raindrops floating in the white light are reflected and will appear black if
air which acts like tiny prisms. no colour of white light is reflected i.e its absorbs
- The spectrum of white light produced by one all the clours.
prism is an impure spectrum as the adjacent - Classification of colours of lights(pure
colours overlap. colours) and pigments(impure colours):
- To obtain a wider spectrum of colours, 1.Primary colours – Colours that can be
another prism identical to the other is placed combined to form another colour e.g red,green
between it and the screen. and blue in light and red,yellow and blue in

217
Demystified Series Physics Demystified by Dr Timothy
pigments. - The spectrum consists of gamma rays,x-rays,
2.Secondar colours – Colours obtained by ultraviolet rays,light ray(visible light),infra red
mixing any two primary colours e.g ray,radio waves and microwaves(GXULIRM).
In lights: In pigments or paints: - The frequency, penetration power and energy
Red+Green=Yellow Red+Yellow=Orange decreases while wavelength decreases in the
Red+Blue=Magenta Red+Blue=purple or order GXULIRM. Thus, gamma rays has the
violet shortest wavelength & highest frequency while
Blue+Green=Cyan Blue+Yellow=Green mircowaves has the longest wavelength & lowest
3.Complementary colours – Two colours which frequency.
give a white colour when mixed e.g - Visible rays, infra red ray and ultra violet
Red+Cyan=White, Green+Magenta=White, rays can be detected in the sun’s spectrum.
Blue+Yellow=White,Red+Blue+Green=White.
- Colour mixing in pigments or paints – Examples :
Pigments or paints absorbs certain colours from 1. Dispersion of whoite light is the ability of white
white light and reflect the rest.They are impure light to A.penetrate air,water and glass B,move
colours. E.g yellow paint(a mixture of yellow,red in a straight line C.move around corners
and green colours) absorbs blue light and reflects D.separate to its component colours.
red and green light while a blue paint(a mixture 2. Which of the following statements is not
of blue and green light) absorbs red and yellow correct? A.dispersion is a phenomenon by
lights but reflects green light.A mixture of yellow which light bends round corners B.dispersion
and blue paint thus reflects green light and results from the difference in speed of light
absorbs blue,red and yellow light. i.e yellow paint components within a material medium C.the
+blue paint =green paint. violet component of white light has the lowest
- Colour filters – are transparent objects used in speed in any refracting medium D.all
analysis of colours in a transmitted light. They components of white light travel with the same
allow light of one colour to pass through them speed in vacuum
while absorbing light of other colours e.g If a Diffraction is the bending of light round obstacles
white light is incident on a red filter, red light is not dispersion. Dispersion is as a result of the
transmitted while other colours are absorbed. difference in speeds of the various components of
● Electromagnetic Spectrum – It consists of light in glass. All colours of white light (ROYGBIV)
electromagnetic waves which are also transverse travel with same speed in vacuum.
waves. Deviation ∝
𝑛∆𝑣
,violet which is the most deviated
𝑣
- The table below shows the types,source and
colour in the spectrum, has the greatest
properties of electromagnetic spectrum:
refractive index and change in speed, and lowest
Spectrum Source Properties speed and wavelength.
1.Gamma Radioactive Highly 3.Dispersion of light by a glass prism is due to the
rays materials. penetrating. A.Different hidden colours of the glass
2.X-rays X-ray tube, fast Penetrating B.Different speeds of the various colours in
moving screen and glass C.Defects in the glass D.High density of
electrons stopped by glass.
stopped by a bones. 4. A narrow beam of white light can be split into
target. diffrerent colours by a glass prism. The correct
3.Ultra Sunlight,quartz Skin explanation is that A.White light is an
violet rays lamps,gas pigmentation, electromagnetic wave B.The prism has all the
discharge Fluorescence. colours of white light C.White light has
tubes. undergone total internal reflection in the prism
4.Light Sunlight,lamps. Vision. D.Different colours of white light travel with
rays different speeds in glass E.White light consists
(visible Heat of yellow, green and red colours.
ray) sensation, for 5. The angular dispersion of a prism depends on
5.Infra red Heat radiated taking A.the angle of incidence as well as the index of
rays by hot bodies photographs refraction B.the index of refraction only C.the
at night or in angle of incidence only D.the thickness of prism
haze E.the prism angle
condition. Dispersion of a prism depends on refractive
6.Radio Transmitter Radio signals index(or index of refraction) of the prism.
waves 6. Rainbow formation is a natural phenomenon
- They travel in vacuum with the same speed i.e illustrating that white light A.splits into three
speed of light(3×108ms-1). components after rainfall B.is dispersed by glass
218
Demystified Series Physics Demystified by Dr Timothy
prisms C.splits into its constituent colours before 14. The angle of deviation of light of various
rainfall D.is dispersed into its constituent colours passing through a triangular prism
colours by raindrops. increases in the order A.red→green→blue
7. A rainbow is formed when sunlight is incident B.green→violet→blue C.blue→red→green→
on water droplets suspended in the air due to D.blue→green→red
A.diffraction B.refraction C.dispersion Deviation of the spectrum increases in the order
D.interference – ROYGBIV: red,orange,yellow,green, blue,indigo
8. The spectrum of white light consists of and violet.
coloured lights arranged in the following order 15. The angle of deviation of light of varous
A.Blue,red,green,yellow,indigo,violet,orange colours passing through a glass prism decreases
B.Red,orange,yellow,green,blue, indigo,violet in the order of A.blue,orange and red B.red,
C.Red,yellow,indigo,green,blue,violet blue and orange C.blue,red and orange D.red,
D.Indigo,green,blue,violet,yellow,red,orange orange and blue E.orange,blue and red
E.yellow,blue,green,violet,orange, indigo,red. Deviation of the spectrum increases in the order
The spectrum of white is given by the acronym – ROYGBIV: red,orange,yellow,green, blue,indigo
ROYGBIV, ranging from red to violet. and violet. Thus, deviation decreases in the
9. The visible part of light consists consists of the order: VIBGYOR.
following colours: A.red,indigo,infra red, 16. A spectrum of white light is not a pure
violet,yellow,green and blue B.red,green, spectrum when A.the different colours in it
blue,violet,indigo,orange and yellow C.blue, overlap B.it is no red colour in it C.it is made up
ultraviolet, infra red,red,yellow,indigo,violet, of only three colours D.it does not contain all the
green and orange D.infra red and ultra violet colours
E.red,orange, yellow,green,blue,indigo and infra The spectrum is said to be impure when some of
red . the colours in the middle overlap one another.
The visible part of light ranges from red to violet 17. A spectrum of sunlight is said to be impure
(ROYGBIV). The invisible part are infra red and when A.It is made of only three colours B.It has
ultra violet. no red colour in it C.The different colours
10.When white light is dispersed by a overlap D.It does not contain white light.
spectrometer, the component having the 18. Which of the following arrangement in the
shortest wavelength is A.Orange B.Green sequence shown can be used to obtain a pure
C.Violet D.Red. spectrum of white light is A.Source,slit,prism,
Deviation ∝
𝑛∆𝑣
, deviation increases in the order diverging lens,screen B.Source,slit,converging
𝑣
lens,prism,converging lens,screen C.Source,
: ROYGBIV. Violet is most deviated hence will
converging lens,prism,diverging lens,screen
have the shortest wavelength.
D.Source,slit,diverging lens,screen
11. Which of the following paurs of colours gives
The slit provides a narrow beam,which is
the widest separation in the spectrum of white
collimated, i.e made parallel by converging lens
light? A.Red and violet B.Green and yellow
before it is incident on the prism.After refraction
C.Red and indigo D.Yellow and violet
by the prism,the spectrum is brought to a focus
Red and violet are most widely separated in the
on a screenplaced at the focal plane of the second
spectrum(ROYGBIV) than the other pairs given.
converging lens.
12. Which of the following pairs of light rays
19. The production of pure s[ectrum could easily
show the widest separation in the spectrum of
be achieved using a A.triangular prism only
white light? A.Green and yellow B.Indigo and
B.triangular prism with two concave lenses
violet C.Orange and red D.Blue and red Red and
C.triangular prism with two convex lenses
blue are most widely separated in the
D.glass prism with a pin.
spectrum(ROYGBIV) than the other pairs given.
20. In the production of pure spectrum of white
13. A beam of light consisting of orange and blue
light using a spectrometer, the function of the
colours is incident on a glass prism,which of the
converging lens in a collimator is to A.produce a
following is(are) correct? I.Orange is more
convergent beam which is incident on the prism
deviated than blue light II.Blue is more deviated
B.produce a parallel beam of light which is
than orange light III.The refractive index of glass
incident on the prism C.diverge the incident
is higher for orange than for blue light IV.The
beam a bit before dispersion D.focus the
refractive index is higher for blue than for orange
dispersed beam on the screen.
light A.I B.II C.I and III D.II and IV
𝑛∆𝑣 21. A source of light,a glass prism, and two
Deviation ∝ , deviation increases in the order converging lenses are suitably arranged to
𝑣
: ROYGBIV. Blue light is more deviated than produce a pure spectrum of white light on a
orange light, hence will have a higher screen. The source of white light is then replaced
refractive index of glass than orange light. with a sodium vapour lamp. The screen will now

219
Demystified Series Physics Demystified by Dr Timothy
show A.the colours of the rainbow with dark options are correct
lines crossing them B.a large patch of yellow The red rose flower will absorb the blue light,
light C.a narrow line of yellow light D.a red line thus it appears black.
and yellow line E.a white patch of light 30. When light is incident on an object which is
The set up will produce a pure spectrum of light magenta in colour, which of the following colours
from a source. Sodium vapour lamp emits light of will be absorbed? A.Red and blue B.Green only
yellow colour(a monochromatic light), thus C.Red and green D.Red only
when passed through the set up a pure spectrum Magenta is a secondary light colour of red and
of yellow light widely separated is obtained. blue light. Thus, when light is incident on the
22. Which of the following ecplains why a green magenta coloured object, red and blue lights are
leaf appears green in bright daylight? I.It absorbs reflected while green light is absorbed.
only the green componene t of sunlight II.It 31. A red pen is placed on a yellow table mat.In
absorbs all colours in sunlight except green III.I It green light, the pen and the mat will appear
reflects only green component of sunlight A.I B.II repectively A.red and yellow B.black and green
C.I and III D.II and III. C.black and white D.blue and green
The colour of the green leaf i.e opaque body, In green light, the red pen absorbs green and
appears green because it absorbs all other appear black while the yellow table mat,which
colours from sunlight but reflects thje green can reflect red and green light, appears green.
component. 32.
23. A red flower appears red in sunlight because
it A.transmits red light B.refracts red light White object E F G
C.absorbs red light D.reflects red light 𝐸,𝐹 and 𝐺 are yellow, red and cyan transparent
The colour of opaque objects e.g flower, is seen light filters repectively. The colour of a white
by the colour it reflects. The colour of a opaque object when viwed through the
transparent objects e.g glass,light filter, is seen by arrangement of the filters abvove is A.Yellow
the colour it transmits. B.Red C.Black D.Blue
24. In sunlight, a blue flower looks blue because Yellow light filter 𝐸 transmits yellow(a mixture
we see the flower by the light it A.receives from of red and green) from the white object
our eye B.disperses C.arbsorbs D.refracts (ROYGBIV). The red light filter 𝐹 transmits only
E.reflects. red and absorbs green. The cyan light filter 𝐺
25. When a yellow card is viewed in blue light, it (transmits only blue and green), absorbs the red
will appear A.black B.orange C.purple D.red light and appears black.
E.white 33.
A yellow card appears yellow in white light
because it reflects only yellow light(a mixture of White object G Y
red and green light) and absorbs light of other In the diagram above, a white object is observed
colours. Pure blue light is absorbed when through green(G) and yellow(Y) light filters.The
incident on a yellow card and the card appears colour seen is A.black B.yellow C.blue D.green
black. If a green light was used the card will The green light filter G transmit green from the
appear green,as it can reflect green from the white object(ROYGBIV). The yellow filter Y
card. If a red light was used the card will transmit the green colour which is seen by the
appear red,as it can reflect red from the card. observer.
26. When a yellow card is observed through a 34.
blue glass, the colour would appear A.Black Screen
B.Green C.Red D.White.
27. What is the apparent colour of a red shirt
when viewed in pure green light? A.Red B.Green
C.Yellow D.Black E.Blue White light prism Green
The green light is absorbed by the red short as source light filter
they have no colour in common, thus the shirt In the diagram above light from a white light
appears black as no light is transmitted. source is incident on a glass prism. The colour
28. A white screen is illuminated simultaneously that will appear on the screen on passing through
by red and green light beams. The colour seen is the green light filter is A.black B.white C.yellow
A.yellow B.blue C.white D.black D.green.
The red and green light both combine to form The white on passing through the prism is
yellow light on the screen. separated into the spectrum of white light
29. When a red rose flower is observed in blue ROYGBIV, which is then incident on the green
light, what colour does the observer see? filter. The green light filter absorbs all other
A.Magenta B.Yellow C.Red D.Blue None of the colours except green light which it transmits to

220
Demystified Series Physics Demystified by Dr Timothy
the screen. Frequency decreases in the order – GXULIRM.
35. If the Nigerian flag(green, white, green) is Radio waves have the lowest frequencby from
viewed in pure yellow light, which of the the options given.
following set of colours would be observed on the 44. Which of the following electromagnetic
flag? A.green,yellow,green B.red,yellow, red waves has the highest frequency? A.Infra-red
C.black,yellow,black D.green,white,green rays B.X-rays C.Ultraviolet rays D.Radio waves
Green part reflect green component from the Frequency decreases in the order – GXULIRM. X-
yellow light and absorbs the red component rays has the highest frequency from the options
while the white part reflects the yellow light. given.
36. Secondary colours of light A.red,green and 45. Which of the following electromagnetic
blue B.green,white and red C.cyan,magenta and waves is least energetic? A.Infra-red rays B.X-
yellow D.yellow,blue and magenta rays C.Ultra-violet rays D.Gamma rays
The secondary colours of pigemnts or paints are The energy in the spectrum decreases in the
orange, green and purple or violet. order – GXULIRM, hence gamma rays is most
37. Which of the following is a secondary colour? energetic while Infra-red is least energetic in the
A.Blue B.Orange C.Red D.Green options.
Green is a secondary colour in pigments or paints 46. Which of the following statements is true of
but a primary colour in light. Orange is a ultraviolet, visible and infra-red rays? A.They are
secondary colour in paints/pigments. all electromagnetic waves with the same
38. Which of the following is not part of the wavelength B.Ultraviolet rays have shorter
electromagnetic spectrum? I.Radio waves II.Beta wavelengths than infra red rays and produce
rays III.Gamma rays IV.Alpha rays A.I and II B.I fluorescence C.Infra-red rays have shorter
and III C.II and IV E.III and IV. wavelength than visible light and produce
Alpha rays and beta rays are not electromagnetic sensation of heat D.The wavelengths increase in
waves, they are radioactive radiations. sequence: visible,ultraviolet,infra- red
39. Of the following which is different from the They are all electromagnetic waves with
others? A.x-rays B.gamma rays C.cathode rays different wavelengths. Their wavelength
D.ultraviolet rays E.infrared rays increases in the order:ultraviolet rays,visible
All other options apart from cathode rays are light and infra red. Thus, the ultraviolet has a
electromagnetic waves. Cathode rays are streams shorter wavelength than visible light and infra-
of fast moving electrons. red while visible light has a shorter wavelength
40. Which of the following electromagnetoic than infa-red. Ultraviolet rays causes
waves has the shortest wavelength? A.Radio fluorescence in bodies while onfra-red causes
waves B.X-rays C.Infra red D.Blue light heat sensation.
E.Ultraviolet 47. Which of the following are true about infra-
Wavelength increases in the order – GXULIRM red radiation? I.It is invisible II.It is called heat
Thus, X-rays has the shortest wavelength from ray III.Its frequency is higher than that of blue
the options given. light IV.It travels as a transverse A.I,II,III and IV
41. In which of the following arrangements is the B.I,II and IV C.I,III and IV D.II,III and IV
wavelength in an increasing order? A.Gamma Only light rays(visible light) are visible in the
rays,infra red rays,x-rays,radio waves B.Gamma electromagnetic spectrum,others invisible i.e
rays,x-rays,infra-red rays, radio waves infra-red ray(also called heat ray) is invisible.
C.Radio waves,x-rays,gamma rays,infra-red rays Frequency decreases in the order: GXULIRM, i.e
D.Infra-red rays,radio waves,x-rays,gamma rays Blue light(light ray) has higher frequency than
Wavelength increases in the order – GXULIRM infra-red ray. All electromagnetic rays are
i.e gamma rays,x-rays,ultraviolet rays,light transverse.
rays,infra-red rays,radio waves and micro waves. 48. Radio carrier waves are generated by causing
42. The range of wavelengths of the visible electrons to oscillate rapidly in A.a microphone
spectrum is 400nm–700nm. The wavelength of B.a telephone camera C.an antennaC.coaxial
gamma rays is A.Longer than 700nm B.Shorter cables E.a transmitter.
than 700nm but longer than 400nm C.550nm
D.Shorter than 400nm E.Infinite Jamb past questions on dispersion of light
Wavelength increases in the order – GXULIRM and electromagnetic spectrum :
The wavelength of gamma rays is shorter than [1979/40,1980/37,1983/13,1984/40,1985/34,
that of light waves or visible spectrum, thus it 1986/29,1988/35,1989/29,1990/30,1992/30,
should be shorter than its lower limit (400nm) 1993/31,1994/39,2010/32,2011/29,2012/37,
43. Which of the following electromagnetic 2013/28].
waves have the lowest frequency? A.radio
waves B.infra-red C.ultra-violet D.x-rays

221
Demystified Series Physics Demystified by Dr Timothy

Chapter 21 – Electrostatics, Electric field, Capacitors


● Electrostatics – This involves the aspect of 3.Earthing e.g by touching with the finger,to
electricity that deals with charges at rest. conduct some charges to the earth.Then removal
- Electric Charges – It represents the amount or of the finger.
quantity of electricity in a body. There are two 4.The charged rod(inducing charge) is removed,
types of charges : positive and negatice charges. while the induced charges of opposite sign
Positive charge means deficit of electrons distributes itself in the conductor.
while negative charges means excess of
electrons. B C B C C
- The quantity or magnitude of any charge 𝑸
is a multiple 𝒏 of the charge of an electron 𝒆.
𝑸 = 𝒏𝒆, 𝒆 =1.6×10-19C.
- Charge carriers in solids are electrons.
- Charge carriers in liquids are ions.
- Charge carriers in gases are electrons and ions.
- Fundamental law of electrostatics states that In summary – Placement of charged body →
like charges repel while unlike charges attracts. Earthing → Separation(unearthing) →
- Charging or Electrification of bodies – In Removal of charged body.
charging or electrification, charges are neither - If a positively charged rod is used, a negative
created nor destroyed but can only be charge is induced in the conductor and if a
transferred or induced. negatively charged rod is used, a positive
- Methods of charging bodies – By friction, charge is induced in the conductor.
contact or electrostatic induction. - Charging two bodies in contact by induction
1. Charging by friction – For charging insulators 1.A charged rod is brought close to the one end of
and conductors. Rubbing two uncharged bodies both conductors.
together results the to exchange of equal 2.Charges of opposite kinds is induced in both
magnitude of electrons to produce opposite conductors with the conductor close to the
charges. A glass rod or cellulose acetate strip charged rod acquiring opposite sign as it, while
rubbed with silk becomes positively charged that farther from the charged rod acquires same
while an ebonite or polythene rod rubbed sign as the charged rod.
with fur becomes negatively charged. The 3.Separation of the conductors through short
glass rod losses electrons to silk to become distance with the charged rod still still in place.
positive and the silk negative.The ebonite gains 4.The charged rod is removed.
electrons from fur to become negative and fur - The end close to the charged rod acquires
positive. opposite charges by inductionwhile the end
- No charge is produced when glass rod is far from the charged rod acquires same
rubbed with fur or ebonite rod rubbed with charge as the charged rod.
silk. - Gold leaf electroscope – It is used to detect and
2. Charging by contact – For charging test for small charges.
conductors only. Transferring of chargesof the Metal plate
same sign but of less quantity from an Insulation
insulated charged body to an uncharged body.
3. Charging by electrostatic induction – For Metal rod
charging conductors only. A charged body is Gold foil/leaf
brought close to an uncharged body placed on an Earthed metal case
insulating base(both bodies do not touch), an
induced charge of equal magnitude but - A gold leaf electroscope can be charged by
opposite sign is formed on the uncharged body. contact or induction.
- The charge in the charged rod(inducer) is Charging by induction – i.A charged object is
preserved. No charges are lost nor placed near the metal cap ii.The charged rod still
transferred between both bodies. in placed,the cap of the electroscope is earthed
- Charging a single body by induction – temporarily. iii.After breaking the earthing, the
1.A charged rod(inducing charge) 𝑩 is brought charged rod is withdrawn iv.The gold leaf and
close to a conductor 𝑪 which sits on an insulating metal rod acquire similar charge and they
base. diverge from each other.
2.Electrons are repelled to the other end of the - Gold leaf electroscope is used for –
conductor i.e distribution of opposite charges. 1.Comparing the insulating property of

222
Demystified Series Physics Demystified by Dr Timothy
different materials/distinguishing between friction. Lightning flash results from a sudden
conductors and insulators : When the metal flow of charge between the cloud and the earth.
cap of a charged electroscope is touched with a - Lightning conductor – It consists of an iron rod
body ,a rapid collapse of the leaves indicates the with spikes or copper strip with a sharply
body is a conductor, gradual or slow collapse pointed end.It is used for protecting a building
indicates the body is a poor conductor while no from being destroyed by lightning.
change in divergence/collapse indicates the body - If a positively charged cloud passes over the
is an insulator. building,negative electric charges are induced at
2.Comparing the sign and magnitude of the copper strip i.e the tip, of the
charges : Increase in divergence of a charged lightning,leading to a dipole with the cloud. The
electroscope is seen when a charged rod of the atoms of the air undergo ionization due to high
same charge is placed near it e.g the electroscope electric field between the cloud and tip of the
is positive and the charged rod is also positive. conductor, with negative ions drifting towards
Decrease in divergence of a charged electroscope the cloud and neutralizes the positive charge in
is seen when a charged rod of opposite charge or the cloud.
a neutral(uncharged) rod is placed near it. - Coulumb’s law – It states that the force of
Increase in divergence is the true test of sign attraction or repulsion 𝐹 between two point
of a charge. charges 𝑞1 and 𝑞2 is proportional to the product
- Bringing opposite charges to a charged of their charges and inversely proportional to the
electroscope – divergence increases at a square of their distance 𝑟 apart.
lesser height (close to it) while divergence 𝒒 𝒒 𝑲𝒒 𝒒
𝑭 ∝ 𝟏𝟐 𝟐, 𝑭 = 𝟏𝟐 𝟐 , 𝑭 =
𝟏 𝒒 𝒒
× 𝟏𝟐 𝟐 .
decreases at a greater height (far from it). 1
𝒓 𝒓 𝟒𝝅𝜺 𝒓

- The degree of divergence of the leaves of the 𝐾= =9.0 × 109Nm2C-2,𝜀=permittivity of


4𝜋𝜀
electroscope is a measure of the relative medium=8.854×10-12C2N-1m-2 or 8.854×10-12
magnitudes of the charges on the bodies. Farad per meter.
- The leaf in an electroscope can be made of thin - Coulomb’s law is valid only for point charges
sheet of gold or aluminium,as they can diverge i.e charges whose value is much smaller
easily. compared to the distance between them.
- Charge density – It is charge per unit area of a - The greater the force the smaller the
surface. deflection or distance between the charges.
- Charge density is highestat the -The force acting on the charges increases as
sharpestpoint of a conductor, as the sharpest the distance between the charges decreases.
𝟏
point has the smallest area and greatest 𝑭 ∝ 𝟐 , 𝑭𝟏 𝒓𝟏 𝟐 = 𝑭𝟐 𝒓𝟐 𝟐 .
𝒓
concentration of charges. - Electric field – It is a region or space round a
- Charges are mostly concentrated at the charged body where electric force or a region
outermost part(not innermost part) of hollow where electrostatic induction takes place when a
conductor. conductor is placed within. - Electric
flux  – These are imaginary lines that represents
the direction of the electric field.
- Electric force and electric field are both
vector quantities.
- Faraday’s butterfly net is used to determine - Electric flux spreads radially outwards for
the distribution of charges between the positive charges and spreads radially inwards
innermost and outermost part of a hollow (towards) negative charges.
conductor.
- Proof plane – For transferring charges by
contact (conduction).
- Electrophorus – It is referred to as the
simplest electrostatic generator. It is used to Negative charge Positive charge
produce charges(by induction), store or secure radially inwards radially outwards
charges and transfer charges.
- Discharging bodies – Charges in a body can be
removed by earthing,neutraliziation by opposite
charges from surrounding air(atmospheric
ionization), by point action and ionization of
flames. Attraction Repulsion
- Lightning – Lightning is caused by a storm - The total electric flux through a closed
cloud which is higly charged as a result of the the surface due to any charge not enclosed by the
cloud rubbing against the air i.e charging by surface is zero.

223
Demystified Series Physics Demystified by Dr Timothy
- Electric flux density 𝑬 – The electric flux  per electric field do not point in different direction at
unit area 𝐴 perdendicular to the direction of the the same point.
 3.Lines of force starts only on positive charges
electric field. 𝑬 = .
𝑨
and end only on negative charges,with the
- Electric flux density is same as electric field
number of field lines starting or ending being
intensity.
proportional to the magnitude of the charge.
- Electric field intensity or electric field
4.The closer the field lines are together, the
strength𝑬 – This is the force per unit positive
stronger the electric field at that point. i.e the
charge at any point or potential difference across
greater the force,the less the distance or
a unit distance. It represents the magnitude of
𝑭 𝑲𝑸 𝑸 𝑽 deflection or the lesser the force,the greater the
the electric field. 𝑬 = = 𝟐 = 𝟐 = , deflection or distance.
𝑸 𝒓 𝟒𝝅𝜺𝒓 𝒓
𝐸=electric field intensity=𝑁C-1 or 𝑉m-1, 𝑄=point 5.The force on charges can be repulsive or
charge, 𝑉=potential difference. attractive.
- When two charges of different magnitude are 6.The force is exerted in same direction with the
placed at the same distance each other in the field and it deflects charges, change the speed or
same medium but different electric field kinetic energy of the charges.
intensities, the electric field intensity and the
charges are related by – 𝐸 =
𝑄
, 𝜀 is constant Examples :
4𝜋𝜀𝑟 2
1. Which of the following statements is correct?
in the same medium, 𝑟 is constant at the same
𝑬 𝑬 A.charges generated on glass rubbed with silk are
distance, thus : 𝟏 = 𝟐 . called negative B.charges in various media can be
𝑸𝟏 𝑸𝟐
- Electric potential 𝑽 – It is the workdone in carried by protons detached from their atoms
bringing a unit charge from infinity to a point in C.the magnitude of any charge is a multiple of
an electric field. A positively-charged body has the charge of an electron D.the intensity of an
a positive potential, a negatively-charged electric field is a scalar quantity E.if 500 Joules
body has a negative potential while a neutral work are required to move 2 coulombs charge
body (e.g the earth) has a zero potential. between two points, the potential difference
- Electric potential difference between two points between the two points is 100V
is the workdone 𝑊in taking a unit charge 𝑞,from Glass rod rubbed with silk acquires positive
other point to the other. charge. Charges in various media are not carried
𝑾 = 𝑸𝑽, 𝑽 = =
𝑾 𝑲𝑸
=
𝑸
. by protons i.e electrons in solids, ions in liquids,
𝑸 𝒓 𝟒𝝅𝜺𝒓 electrons and ions in gases. The intensity or
- For an isolated positive or negative charge strength of all fields (electric, magnetic and
the potential energy is zero, as it not placed in gravitational) are vector quantity. Potential
an external electric field. 𝑊𝑜𝑟𝑘𝑑𝑜𝑛𝑒 𝑊 500
- When two like charges are brought together, difference= ,𝑉= = = 250 Joule
𝐶ℎ𝑎𝑟𝑔𝑒 𝑄 2
work is done against the field,as like charges per coulomb or 250 volts. The magnitude of any
repel each other.So work has to be done against charge is a multiple of the charge of electrons i.e
the field to bring them together. 𝑄 = 𝑛𝑒.
- When two opposite charges are borught 2. Which of the following is true of an electrical
together, work is done by the field, as unlike or charge? A.positive charge means deficit of
opposite charges attract each other. So work is electrons B.negative charge means excess of
done by the field to bring them together. electrons C.electric current means movement of
- If work is done against the electric field,the electrons D.all of the above E.A and B only
electric potential is positive and the potential Positive charge means excess of protons and
energy increases. deficit of electrons. Negative charges means
-If work is done by the electric field, the deficit of protons and excess of electrons.
electric potential is negative and the potential 3. The production and type of charge of a
energy decreases. body can only be accounted for by the
- Electric current is due to the flow of charges i.e A.equilibrium in protons and neutrons
positive charges or negative charges (electrons). B.deficiency or surplus of electrons
Positive charges flows from higher to lower C.deficiency or surplus of protons D.deficiency or
potential (i.e direction of electric field) while surplus in neutrons
negative charges (electrons) flow from lower to Surplus of electrons indicates negative charge
higher potential(i.e opposite to the electric field). while deficiency of electrons indicates positive
- Properties of electric field – charge.
1.In a uniform field, the lines of force are straight, 4. An electric charge could be transmitted
parallel and uniformly spaced. through A.wood,rubber and stone B.paper, clay
2.Lines of force do not cross each other i.e as and plastic C.glass,acid and cloth D.the human

224
Demystified Series Physics Demystified by Dr Timothy
body,water and metals E.all of the above. Charging an uncharged conductor by induction
5. When a biro pen rubbed on a dry silk cloth is produces charges of opposite sign and equal
moved very closed to a piece of paper on a dry magnitude as the inducing charge.
table, the pen is found to pick up the paper. This Charging by friction also produces equal
is because A.Both the pen and the cloth are magnitude of charges but of opposite signs.
magnetized B.The pen is magnetized but the Charging by contact produces charges of same
cloth is not C.The pen is charged whule the cloth sign but of less magnitude.
is magnetized D.Both the pen and the cloth are 10. What is the correct sequence of steps for
charged. charging an insulated conductor by induction?
The biro rubbed on the dry silk cloth is charged I.bringing a charged body near to the conductor
by friction and on moving it closed to a piece of II.removing the charged body III.touching the
paper it induces opposite charges on the paper, conductor with a finger IV.removing the finger
thus it attracts it and picks it up. A.I,II,III,IV B.I,III,IV,II C.II,I,III,IV D.III,IV,I,II
6. When an ebonite rod is rubbed with fur, it has E.I,III,II,IV .
A.No charge at all B.A negative charge C.A Placement of charged body → Earthing (touching
positive charge D.Negative and positive charges with finger) → Separation (removing the finger
Ebonite rod rubbed with fur acquires negative i.e unearthing) → Removal of the charged body.
charge. 11. M N In the figure, M and N are
7. An insulated rod when rubbed with a material P insulated metal spheres
acquires A.a negative charge if it is made of glass in contact. A negatively
rubbed with silk B.no charge if it is made of charged ebonite rod P is
glass and rubbed with fur C.no charge if it is is brought near M, when M
made of copper and rubbed with silk D.a positive and N are separated and the rod is taken away,
charge if it is made of copper rubbed with fur. which of the following statement is correct?
Glass(an insulator) rubbed with silk acquires A.Both M and N have acquired negative charges
positive charge. Ebonite(insulator) rubbed with B.Both M and N have acquired positive charges
fur acquires negative charge. No charge is C.M has acquired negative charges and N positive
acquired when glass is rubbed with fur or ebonite charges D.M has acquired positive charge and
rubbed with silk. Copper is a conductor not an N negative charge.
insulator,as stated in the question. The end close to the charged rod acquires
8. An ebonite rod rubbed with fur attracts a glass opposite charges by inductionwhile the end far
rod rubbed wth silk because A.Ebonite has a from the charged rod acquires same charge as the
negative charge while glass has a positive charged rod.Thus, 𝑀 is positively charged while
charge B.Ebonite has a positive charge while 𝑁 is negatively charged.
glass has a negative charge C.Both have negative 12. The magnitude of charges on two bodies are
charges D.Both have positive charges. to be compared. Which of the following
Ebonite rod acquires negative charges when instruments would be suitable? A.Gold leaf
rubbed with fur while glass rod acquires positive electroscope B.Capacitor C.The electrophorus
charges when rubbed with silk. Unlike charges D.Ebonite rod.
attract, thus negatively charged ebonite rod 13. The sign of the charge on a charged glass rod
attracts the positively charged glass rod. may be determined with A.an electrometer B.a
9. A short chain is usually attached to the rear charged electroscope C.a galvanometer D.an
side of a petrol tanker trailing behind it to ensure uncharged electroscope.
that A.the heat generated in the engine is Increase in divergence is a true test of sign.
conducted to the earth B.the charges generated 14. A positively charged rod 𝑋 is brought near an
by friction in the tanker are conducted to the uncharged metal 𝑌 and is then touched by a
earth C.the tanker does not exceed a certain finger with 𝑋 still in place. When the finger is
speed limit D.the chain vibrates in resonance removed, the result is that 𝑌 hasd A.A negative
with the tanker engine. The friction due to the charge and a positive potential B.No charge and
rubbing action between the petrol tanker and air zero potential C.A negative charge and a
results in the generation of electric charges that negative potential D.A positive charge and a
collects on the petrol tankers body, which are zero potential.
conducted to the earth through the chain. The positively charged rod induces negative
10. In the process of electrification by induction, charge in the uncharged metal. Negative charge
the amount of induced charges in relation to the is at a negative potential.
inducing charge is A.same sign and quantity 15. A negatively-charged rod is brought near the
B.same sign but greater C.opposite sign but less cap of a gold leaf electroscope.The cap is earthed
D.opposite sign and greater E.opposite sign and momentarily with the rod near to it. The rod is
equal. then removed. Which of the following is correct?

225
Demystified Series Physics Demystified by Dr Timothy
A.the cap will a positively charged and the 20. A charged ball is placed in contact with the
negatively charged leaves will diverge B.the cap inner wall of a hollow conducting sphere. The
will be negatively charged and the positively resulting electric field is A.inside the sphere B.on
charged leaves will diverge C.the cap and the the surface of the ball C.inside and outside the
leaves will be positively charged D.the cap and sphere D.outside the sphere.
the leaves will be negatively charged E.the leaves Excess charges are concentrated on the
will not diverge because the cap has been outside(surface) of the sphere and the electric
earthed. field inside the hollow sphere is zero.
Negative charged rod induces positive charges 21. The field that results from the contact of a
on the cap and leaf of the electroscope. Earthing positively charged ball with the inner wall of an
conducts excess electrons away from the leaf, uncharged hollow conducting sphere is A.Inside
thus divergence of the leaf increases. the sphere B.On the surface of the ball C.Inside
16. What happens when a positively charged and outside the sphere D.Outside the sphere.
insulator is placed near the disc of a positively Charges are concentrated in the outer part of a
charged gold leaf electroscope, and is then hollow conductor.
removed? A.the leaf falls, and then rises when 22. Which of the following statements are true of
the body is removed B.the leaf rises further, an insulated charged body carrying bpositive
and remains in that position when the body is charges? I.It contains excess positive charges II.It
removed C.the leaf falls, and remains in that has an electric field III.It possesses potential
position when the body is removed D.the leaf energy IV.It carries an electric current A.I and II
rises further and falls to its original position B.I,II and III C.II and III D.I,II,III and IV.
when the body is removed E.nothing happens Positively charged bodies contains more protons
Leaf diverges(rises) further when a positively than electrons. The excess positive
charged rod is brought close to a positively charge(protons) creates an electric field. For an
charged electroscope and remains in this isolated positive or negative charge the potential
position when the body is removed.The leaf can energy is zero, as it not placed in an external
be made to return to its original position by electric field.
earthing. 23. Two unequal insulated charged spheres
17. As a negatively charged rod is brought nearer carrying unlike charges are connected together
to the cap of a positively charged gold leaf with a short wire.Charges will flow from one
electroscope,the divergence of the leaf will sphere to the other until both spheres A.carry
A.decreases to zero B.steadily increase equal charges of opposite signs B.carry equal
C.decrease to zero and then increase D.remain charges of the same sign C.are at the same
constant E.increase to a maximum and then potential D.become uncharged
decrease. Charges/current always flow between two
Divergence of the leaf decreases when a points of different electric potentials until their
negatively charged rod is brought close to a potential equalizes i.e equipotential.
positively charged electroscope. Option B would 24. The instrument used for securing a large
have been correct if both the electroscope and number of similar charges by induction is called
charged rod bears same the charge. A.electroscope B.proof plane C.electrophorus
18. As a positively charged rod is brought nearer D.capacitor
to the top cap of a positively charged gold leaf Electrophorus stores (secures), separate,
electroscope, the divergence of the levaes will produce and transfers charges by induction.
A.decrease to zero B.steadily increase Proof plane transfers charges by conduction
C.decrease to zero and then increase D.remain (contact). Capacitor stores charges when a
constant E.increase to maximum and then cell(battery) is connected across its terminal.
decrease. Electrophorus is the simplest electrostatic
Divergence increases as the charged rod and the generator.
gold leaf electroscope are positively charged. 25. When a conductor is mounted on an
19. A source emitting alpha particlesis brought insulating stand is charged and left for sometime
near the cap of a negatively charged gold-leaf the conductor eventually losses all of its charges.
electroscope. The gold leaf will A.diverge and This is because the A.electric charges evaporate
remain there B.fall and remain in its new from the surface of the conductor B.charges on
position C.fall and then rises up again to its the conductor are neutralized by opposite
initial position D.not be affected. charges from the surrounding air C.chrges
Alpha particles are positively charged. Thus, ionizes the surrounding air D.charges are
when brought close to a negatively charged conducted to the earth.
electroscope, the divergence decreases and the 26. Which of the following statements about
gold leaf fall. lightning conductor is not correct? A.It initiates

226
Demystified Series Physics Demystified by Dr Timothy
point action B.The spikes gain an induced charge Value.
opposite to that of the cloud C.It consists of a 𝐹1 =𝐹, 𝑟1 =𝑟, 𝐹2 =?, 𝑟2 =4𝑟, 𝐹 ∝ 2,
1
𝑟
thick metal strip connecting sharp points to a
𝐹1 𝑟1 2 = 𝐹2 𝑟2 2 , 𝐹 × 𝑟 2 = 𝐹2 × (4𝑟)2 ,
metal plate D.The metal strip receives charges 𝐹𝑟 2 𝐹
and discharges them to the air through the 𝐹𝑟 2 = 𝐹2 × 16𝑟 2 , 𝐹2 = = i.e 1/16 of its
16𝑟 2 16
sharp points former or original value. Alternatively,
27. Which of the following statements are correct 1 1 1
𝐹 ∝ 2, 𝑟 is doubled thus, 𝐹 = 2 = i.e 1/16 of
𝑟 4 16
about lightning conductor? I.they prevent tall
its orginal or former value.
buildings from being damaged by lightning
31. The electric force between two unlike
II.they each consist of a sharp pointed rod at one
charges, each of magnitude 𝑞, separated by a
end connected to earth through a thick cable 𝑞2
III.charges opposite to those in the thunder cloud distance 𝑑in air of permittivity 𝜀𝑜 is A.
4𝜋𝜀𝑜 𝑑
are induced in the lightning conductors IV.the 𝑞2 𝑞2 𝒒𝟐
B.− C. D.−
induced charge flows down the cable and 4𝜋𝜀𝑜 𝑑 4𝜋𝜀𝑜 𝑑 2 𝟒𝝅𝜺𝒐 𝒅𝟐
1 𝑞1 𝑞2
neutralizes the charge in the cloud A.I and II B.II 𝑞1 =𝑞 , 𝑞2 =−𝑞, 𝑟=𝑑 , 𝐹 = × ,
4𝜋𝜀 𝑟2
and III C.I,II and III D.I,II and IV E.I,III and IV. 1 𝑞×−𝑞 𝑞2
Ligthning conductors consist of a sharp pointed 𝐹= × =− .
4𝜋𝜀𝑜 𝑑2 4𝜋𝜀𝑜 𝑑 2
rod or strip made of copper, at the top and is 32. The ratio of electrostatic force 𝐹𝐸 to
connected to the earth by a thick iron metal gravitational forcen 𝐹𝐺 between two protons
cable.They prevents tall buildings from each of charge 𝑒 and mass 𝑚, at a distance 𝑑 is
collapsing by lightning. Charges opposite to the 𝑒 𝑒2 𝐺𝑚2 𝒆𝟐
A. B. C. D. .
cloud are induced in the lightning conductors. 4𝜋𝜀𝑜 𝐺𝑚 𝐺𝑚2 4𝜋𝜀𝑜 𝑒 2 𝟒𝝅𝜺𝒐 𝑮𝒎𝟐
𝑒2
The induced charge i.e the charge at the sharp Electrostatic force of repulsion 𝐹𝐸 = ,
4𝜋𝜀𝑜 𝑑 2
point, is conducted to the earth.It is the charge 𝐺𝑚2
produced from the ionization of air that Gravitational force of attraction 𝐹𝐺 = ,
𝑑2
neutralizes the charge in the cloud not the 𝑒2
𝐹𝐸 4𝜋𝜀𝑜 𝑑2 𝑒2 𝑑2 𝑒2
induced charge. 𝐹𝐸 :𝐹𝐺 = = 𝐺𝑚2
= × =
𝐹𝐺 4𝜋𝜀𝑜 𝑑 2 𝐺𝑚2 4𝜋𝜀𝑜 𝐺𝑚2
28. The diagram below show lines of force in 𝑑2
various electric fields. In which of the diagrams 33. The force of repulsion between two point
would a positive test charge experience less force positive charges 5𝜇C and 8𝜇C separated at a
as it moves from 𝑃 to 𝑄. distance of 0.02m apart is A.1.8×10-10N
A. 𝑸 B. 𝑄 C. D. 𝑄 B.9.0×10-8N C.9.0×102N D.4.5×103N.
1
𝑷 ( = 9.0 × 109Nm2C-2)
4𝜋𝜀
𝑃 𝑃 𝑄 𝑞1 =5𝜇𝐶=5×10-6C, 𝑞2 =8𝜇𝐶=8×10-6C, 𝑟=0.02m
𝑃 𝐾𝑞 𝑞 1 𝑞 𝑞
𝐹 = 12 2 = × 122 ,
Curved lines represents non-uniform fields 𝑟 4𝜋𝜀 𝑟
9.0×109 ×5×10−6 ×8×10−6 360×10−3
(fields that change directions from one place to 𝐹= = ,
0.022 4×10−4
the other). The greater deflection of the charge or 𝐹 = 900𝑁 = 9.0×102N.
distance moved by the charge indicates a lesser 34. Two bodies each carrying a charge of
force applied to the test charge (𝐹 ∝ 1/𝑟2) . The 2.00×10-10C are 5cm apart. Calculate the
test charge deflects through a larger angle in magnitude of the force between the charges.
option A. 1
( =9.0 × 109Nm2C-2) A.1.44×10-7N
29. If the distance between two stationary 4𝜋𝜀
charged particles is doubled, the magnitude if the B.7.20×10-11N C.1.44×10-11N D.7.20×10-9N.
electrostatic force between them will be 𝑞1 =𝑞2 =𝑞= 2×10-10C, 𝑟 = 5cm = 0.05m,
𝐾𝑞 𝑞 1 𝑞 𝑞
A.doubled B.halved C.a quarter of its former 𝐹 = 12 2 = × 122 ,
𝑟 4𝜋𝜀 𝑟
value D.unchanged E.four times the former value 9×109 ×(2×10−10 )2 9×109 ×4×10−20
1 𝐹= = ,
𝐹1 =𝐹, 𝑟1 =𝑟, 𝐹2 =?, 𝑟2 =2𝑟, 𝐹 ∝ 2, 0.052 2.5×10−3
𝑟 𝐹 = 1.44×10 N. -7
𝐹1 𝑟1 2 = 𝐹2 𝑟2 2 , 𝐹 × 𝑟 2 = 𝐹2 × (2𝑟)2 , 35.Two identical charges situated 20cm apart in
𝐹𝑟 2 𝐹
𝐹𝑟 2 = 𝐹2 × 4𝑟 2 , 𝐹2 = 2 = i.e quarter of its vacuum repel each other with a force of 1.0N.
4𝑟 4
former or original value. Alternatively, Calculate the value of each charge. [ (4π𝜀 0)-1 =
1 1 1
𝐹 ∝ 2, 𝑟 is doubled thus, 𝐹 = 2 = i.e quarter 9×109N𝑚2𝐶 -2] A.3.6𝜇C B.4.0𝜇C C.2.1𝝁C D.4.4𝜇C.
𝑟 2 4 𝑞1 =𝑞2 =𝑞=?, 𝑟=20cm=0.2m, 𝐹=1.0N,
of its orginal or former value. 1 𝑞 𝑞
(4π𝜀 0)-1 = 9×109Nm2C-2, 𝐹 = × 122 ,
30. If the distance between two point charges is 4𝜋𝜀𝑜 𝑟
increased by a factor of four, the magnitude of the 9×109 ×𝑞 2 9×109 ×𝑞 2
1.0 = , 1.0 = ,
0.22
electrostatic force between them will be A.1/2 of 0.04
0.04
its former value B.1/16 of its former value 0.04 = 9×109× 𝑞2, 𝑞2= =4.44×10-12,
9×109
C.1/4 of its former value D.4 times of its former 𝑞 = √4.44 × 10−12 = 2.1×10-6C = 2.1𝜇C.
227
Demystified Series Physics Demystified by Dr Timothy
36. Calculate the electric field strength at a point 42. Calculate the electric potential at a distance
0.05m from a point charge of magnitude of 20.0cm from a point charge of 0.015C placed in
3.14×10-6C in a medium of permittivity 2.5×10- air of permittivity 𝜀 0 A.3.40×109V B.6.75×108V
11Fm-1. (π=3.14) A.2.0×105𝑁C-1 B.4.0×106NC-1 C.6.75×106V D.3.40×105V E.8.30×10-12V
C.1.1×107NC-1 D.2.2×108NC-1. (take;
1
=9.0×109Nm2C-2).
4𝜋𝜀𝑜
𝑄=3.14×10-6C, 𝑟=0.05m, 𝜀=𝜀 0=2.5×10-11Fm-1, 𝐾𝑄 𝑄
𝑄 3.14×10−6 𝑄=0.015C, 𝑟=20cm=0.2m, 𝑉 = = ,
π=3.14, 𝐸 = = 𝑟 4𝜋𝜀𝑟
4𝜋𝜀𝑜 𝑟 2 4×3.14×2.5×10−11 ×(0.05)2 9×109 ×0.015
3.14×10−6 𝑉= =6.75×108𝑉.
𝐸= = 4×106NC-1. 0.2
4×3.14×2.5×10−11 ×2.5×10−3 43. Find the workdone in moving a 2C charge
37. A charge of 4𝜇C is placed 3m away from between two points X and Y in an electric field if
appoint a point P, calculate the magnitude of the the potential difference is 100 volts A.50J B.100J
electric field intensity, due to the charge at the C.200J D.400J.
point P A.8.1×104𝑉m-1 B.2.7×10-4𝑉m-1 𝑊
𝑄=2C, 𝑉=100𝑉, 𝑉 = , 𝑊 = 𝑄𝑉 ,
C.1.2×10 𝑉m D.4.0×104𝑽m-1
4 -1 𝑄
1
( =9.0×109Nm2C-2). 𝑊 = 2×100 = 200J.
4𝜋𝜀
1
44. A point charge of magnitude 2.5𝜇C is moved
𝑄=4𝜇C=4×10-6C, 𝑟=3m, =9.0×109Nm2C-2, through a distance of 0.04m against a uniform
4𝜋𝜀
𝐸=
𝐾𝑄
=
𝑄
=
9×109 ×4×10−6
, electric field of intensity 15𝑉m-1. Calculate the
𝑟2 4𝜋𝜀𝑟 2 32 workdone on the charge A.1.5×10-6J B.1.0×10-
9×109 ×4×10−6
𝐸= = 4×104𝑉m-1 . 6J C.3.5×10-6J D.4.0×10-6J E.6.0×10-6J.
9
38. A charge of 2.0×10-5C experiences a force of 𝑄=2.5𝜇C=2.5×10-6C, 𝑟=0.04m, 𝐸=15Vm-1
80N in a uniform electric field. Calculate the Workdone on charge=Force×distance
magnitude of the electric field intensity 𝑊 = 𝐹 × 𝑟, 𝐹 = 𝑄𝐸, 𝑊 = 𝑄𝐸𝑟
A.8.0×106NC-1 B.4.0×106NC-1 C.4.0×10-4NC-1 𝑊 = 2.5×10-6×15×0.04 = 1.5×10-6J.
D.2.0×104NC-1. 45. Why is it impossible for the lines of force of
𝐸=?, 𝐹=80N, 𝑄=2.0×10-5C, 𝐸 = ,
𝐹 an electric field to cross one another? A.like
𝑄 charges repel B.a charged particle is deflected by
80
𝐸= =4.0×106NC-1. an electric field C.like charges are arranged on
2.0×10−5
38. Calculate the force acting on an electron of the same lines of force D.a positively charged
charge 1.6×10-19C placed in an electric field of particle can travel in only one direction at
intensity 108𝑉m-1 A.1.6×10-11N B.1.6×10-14N anytime E.none of these is correct.
C.1.6×10-16N D.1.0×10-16N. 45. When two like charges are brought together,
𝑄=1.6×10-19C, 𝐸=108𝑉m-1, 𝐹=?, 𝐸 = ,
𝐹 the nature potential energy of the system formed
𝑄 will A.increase because work is done against
𝐹 = 𝐸𝑄 = 108×1.6×10-19= 1.6×10-11N. the field B.increase because work is done by the
39. Two parallel plates at a distance of 9×10-3m field C.decrease because work is done by the field
700V. The electric field strength between them is D.decrease because work is done against the field
A.6.30×100Vm-1 B.1.26×10-4Vm-1 When two like charges are brought together,
C.7.78×10 Vm1 -1 D.6.30×103Vm-1 work is done against the field,as like charges
E.7.78×10 Vm 4 -1
repel each other.So work has to be done against
𝑉 700
𝑉=700𝑉, 𝑟=9×10-3m, 𝐸=?, 𝐸 = = , the field to bring them together.
𝑟 9×10−3
𝐸 =7.78×10 𝑉m .4 -1 If work is done against the electric field,the
40. A charge of 50𝜇C has an electric field strength electric potential is positive and the potential
of 360NC-1 at a certain point. The electric field energy increases. If work is done by the electric
strength due to another charge 120𝜇C kept at the field,the electric potential is negative and the
same distance apart and in the same medium is potential energy decreases.
A.18NC-1 B.144NC-1C.864NC-1 D.150NC-1 46. When a charge moves through an electric
𝑄 circuit in the direction of an electric force,it
2 , 4π𝜀𝑟 =constant i.e same medium
𝐸= 2
4𝜋𝜀𝑟
1 𝐸
A.gains both potential energy and kinetic energy
𝜀 and same distance 𝑟 apart. 𝐾 = = , B.gains potential energy and lose kinetic energy
4𝜋𝜀𝑟 𝑄
𝐸1
=
𝐸2
, 𝐸1 =360NC-1, 𝑄1 =50𝜇C, 𝐸2 =?, 𝑄2 =120𝜇C , C.loses potential energy and gains kinetic
𝑄1
360
𝑄2
𝐸2 360×120
energy D.loses both potential and kinetic energy
= , 𝐸2 = = 864NC-1. Charges moving in the direction or perpendicular
50 120 50
41. The expression for the potential at a point, to an electric field or force, gains kinetic
distance 𝑟 from a charge 𝑞, in an electric field is energy(speed) and loses potential energy.
𝑞2 𝑞2 𝑞 𝒒
A. B. C. D. .
4𝜋𝜀𝑜 𝑟 2
𝑄
4𝜋𝜀𝑜 𝑟
𝑞
4𝜋𝜀𝑜 𝑟 2 𝟒𝝅𝜺𝒐 𝒓 ● Capacitors
𝑉= = . - Capacitor – A capacitor is a device used to store
4𝜋𝜀𝑟 4𝜋𝜀𝑜 𝑟
electric charges and energy. It consists two

228
Demystified Series Physics Demystified by Dr Timothy
conductors with equal and oppositecharges - Charging a capacitor – The plates of the
which are separated by an insulating material capacitor are connected to a cell (battery) for
called the dielectric. some time,so that electrons are attracted by the
- The equal and opposite charges are called positive terminal of the cell and drifted towards
dipoles. The ability of a capacitor to store one of the plates leaving opposite charges(±𝑄)
charges is called Capacitance. on both plates. The energy stored in the capacitor
-The dielectric material must be an insulator is half of the total energy supplied by the cell,
e.g air, glass, paraffin wax, paper foil e.t.c. while the remaining half is converted to heat
- Parallel plate capacitor – Common type of energy.
capacitor with two parallel plates separated by - Discharging a capacitor – The plates are
an insulating medium(dielectrics). Charges are connected or in contact by wires, which ensures
stored due to the decrease in the potential the continuous back flow of electrons round the
difference and increase in the capacitance of one circuit and neutralise the positive charged plate
of the conductors (charged) when another and charges on the plates.
conductor(uncharged but earthed) is brought - Arrangement of capacitors :
close to it. 1.Series arrangement – When capacitors 𝐶1 , 𝐶2
- Capacitance 𝑪 – Capacitance of a capacitor is and 𝐶3 are connected in series, as shown below :
the ratio of the charge on either conducor to the 𝑪𝟏 𝑪𝟐 𝑪𝟑
potrential difference between the conductors.
𝑸
𝑪 = , 𝑸 = 𝑪𝑽.
𝑽
𝑬
- Graphical relationship with the charge 𝑄 stored
by a capacitor and the potential difference 𝑉
I.Same charge flows through each of them i.e
across it : Charges stored by a capacitor is
𝑸𝟏 = 𝑸𝟐 = 𝑸𝟑 = 𝑸.
directly proportional to the potential difference,
II.Voltage across each capacitor is different and
𝑸∝ 𝑽.
their arithmetic sum is equal to the total voltage
𝑸
Slope=Capacitance 𝑪. or E.M.F of the cell. 𝑬 = 𝑽𝑻 = 𝑽𝟏 + 𝑽𝟐 + 𝑽𝟑 .
III.The effective or total capacitance is equa; to
𝑽 the reciprocal sum of the capacitance of each
𝟏 𝟏 𝟏 𝟏
- The slope the graph above is equal to the capacitor – = + + .
𝑪𝑻 𝑪𝟏 𝑪𝟐 𝑪𝟑
capacitance across the capacitor. 𝑪𝟏 𝑪𝟐
- For two capacitors in series – 𝑪𝑻 = .
- The area of the graph above is equal to the 𝑪𝟏 +𝑪𝟐
workdone in stroring charges or energy - If the capacitance of 𝑛 capacitors are of equal
𝑪
stored by a capacitor. magnitude, then – 𝑪𝑻 = .
𝒏
- Factors affecting capacitance of a capacitor: 𝑸
IV.Their individual voltage is given by – 𝑽𝟏 = ,
1. Areaof the plates – Capacitance increases with 𝑪𝟏
𝑸 𝑸
increase in area of the plates (𝑪 ∝ 𝑨). 𝑽𝟐 = , 𝑽𝟑 = . Alternatievly, for two
𝑪𝟐 𝑪𝟑
2. Distance of separation between the plates –
capacitors in series, 𝑄 is the same or constant,
Capacitance decreases as the distance of 𝟏
𝟏 hence; 𝑽 ∝ i.e the smaller capacitance will
separation between the plates increases (𝑪 ∝ ). 𝑪
𝒅 have the highest voltage.
3.Shape of plates/conductors. 𝑪 𝒐𝒇 𝒐𝒕𝒉𝒆𝒓 𝒄𝒂𝒑𝒂𝒄𝒊𝒕𝒐𝒓
4.Dielectrics (insulating medium) between 𝑽(across 𝑪) = × 𝑽𝑻
𝒔𝒖𝒎 𝒐𝒇 𝒄𝒂𝒑𝒂𝒄𝒊𝒕𝒂𝒏𝒄𝒆
the plates – Thickness or density of the medium. 𝑽𝟏 (voltage across 𝑪𝟏 ) =
𝑪𝟐
× 𝑽𝑻 .
Capacitance increases as the thickness or density 𝑪𝟏 +𝑪𝟐
𝑪𝟏
of the dielectrics increases. (𝑪 ∝ 𝜺), e.g 𝑽𝟐 (voltage across 𝑪𝟐 ) = × 𝑽𝑻 .
𝑪𝟏 +𝑪𝟐
capacitance with air is less than that with - When capacitors are stored in series, the
𝜺𝑨 𝜺𝑨
paper. 𝑪 ∝ , 𝑪= , 𝐶=capacitance, equivalent capacitance is less than the
𝒅 𝒅
𝑑=distance, 𝐴=area, 𝜀=dielectric constant or individual capacitance and even the small
permittivity of the medium. 𝜀 = 𝜀 0 i.e in individual capacitance.
vacuum(free space) or air. - Parallel arrangements – When capacitors 𝐶1 ,
- Relative permittivity 𝜺𝒓 is the ratio of the 𝐶2 and 𝐶3 are in parallel as shown below :
permittivity in a medium 𝜀𝑚 to the permitivity in 𝐶1
vacuum/air 𝜀𝑜 . It is also expressed as the ratio of
the capacitance in a medium 𝐶𝑚 to the 𝐶2
𝜺 𝑪
capacitance in vacuum/air 𝐶𝑜 . 𝜺𝒓 = 𝒎 = 𝒎 .
𝜺𝒐 𝑪𝒐 𝐶3
- The capacitance of an isolated sphere of radius
𝑟 which carries a charge 𝑄 is – 𝑪 = 𝟒𝝅𝜺𝒐 𝒓 . 𝐸

229
Demystified Series Physics Demystified by Dr Timothy
I.Voltage or potential difference across each of 2×10-6×12 = 𝐼 × 0.02, 𝐼 =
2×10−6 ×12
,
the capacitors is the same and equal to the E.M.F. 0.02

𝑽 𝟏 = 𝑽𝟐 = 𝑽 𝟑 = 𝑽 𝑻 = 𝑬 . 𝐼 = 1.20×10 𝐴.-3

II.The effective or total capacitance is the 3. The purpose of dielectric material in a parallel
arithmethic sum of each of the capacitance – plate capacitor is to A.increase the capacitance
𝑪𝑻 = 𝑪𝟏 + 𝑪𝟐 + 𝑪𝟑 . B.dcrease its capacitance C.insulate the plates
III.The total or effective charges is given by the from each other D.increase the magnetic field
arithmetic sum of the individual charges – 𝑸𝑻 = between the plates.
𝜀𝐴
𝑸𝟏 + 𝑸𝟐 + 𝑸𝟑 = 𝑪𝑻 𝑽 . 𝐶 ∝ , Capacitance increases as permittivity
𝑑
IV.Their individual charges is given by – 𝑸𝟏 = of the medium𝜀 increases, area of of the plates 𝐴
𝑪𝟏 𝑽, 𝑸𝟐 = 𝑪𝟐 𝑽, 𝑸𝟑 = 𝑪𝟑 𝑽. Alternatively, for increases and distance between the plates 𝑑
capacitors in parallel, 𝑉 is same or decreases. Permitivity of the medium increases
constant,hence; 𝑸 ∝ 𝑪. i.e the higher as the thickness or density of the dielectric
capacitance will store highest charge. material increases.
𝑸(in 𝑪)=
𝑪 𝒐𝒇 𝒕𝒉𝒆 𝒄𝒂𝒑𝒂𝒄𝒊𝒕𝒐𝒓
× 𝑸𝑻 4. When a piece of rectangular glasss block is
𝒔𝒖𝒎 𝒐𝒇 𝒄𝒂𝒑𝒂𝒄𝒊𝒕𝒂𝒏𝒄𝒆
𝑪𝟏 inserted between two parallel plate capacitor, at
𝑸𝟏 (charge in 𝑪𝟏 ) = × 𝑸𝑻 . constant plate area and distance of separation,
𝑪𝟏 +𝑪𝟐
𝑸𝟐 (charge in 𝑪𝟐 ) =
𝑪𝟐
× 𝑸𝑻 the capaciatance of the capacitor will
𝑪𝟏 +𝑪𝟐 A.Increase B.Decrease C.Decrease then increase
- Workdone in charging a capacitor or energy D.Remain constant
stored by a capacitor –The energy stored in a 𝜀𝐴
𝐶= , at constant area 𝐴 and distance of
capacitor is electrostatic potential energy. 𝑑
𝟏 𝟏 𝑸𝟐 separation, 𝐶 ∝ 𝜀 i.e capacitance increases as the
𝑾 = 𝑸𝑽 = 𝑪𝑽𝟐 = . permittivity of the medium increases. Inserting a
𝟐 𝟐 𝟐𝑪
- The ratio of energy stored in two parallel rectangular block with high density and
capacitors is equal to the ratio of their thickness between the parallel plate capacitor,
capacitance. increases the permittivity and thus, increasing
𝑸𝟐 the capaciatance.
- Workdone on charges –𝑾 = 𝑸𝑽 = 𝑪𝑽𝟐 = .
𝑪 5. How would the capacitance of a parallel plate
- Othertypes of capacitors includes – paper
capacitor be affected if the distance of separation
capacitor, variable air capacitor and eletctrolytic
of its plates is decreased. It will A.increase in
capacitor.
value B.decrease slightly C.remain unchanged
- Uses/application of capacitors –1.Tuning in
D.drop to zero
radio circuits. 2.Ignition system of a motor 1
vehicle. 3.Cancels sparking in switches or 𝐶 ∝ , capaciatance increases as the distance of
𝑑
circuits with inductance e.g induction coil. separation between the plates decreases.
𝜀𝐴
4.Blocking noise in a.c amplifiers. 5.Storing 6. If the capacitance of a capacitor is 𝐶 = ,
𝑑
electrical energy and large quantities of charge. which of the following parameters is varied when
6.Filters out alternate current(A.C) from direct a variable capacitor is used for tuning radio sets?
current(D.C) i.e having high resistance for D.C A.Area and distance B. constant (𝜀) only
and low resistance for A.C. C.Constant (𝜀) and araea D.Distance .
In a variable capacitor, the permittivity of the
Examples : medium is constant while the area and distance
1. If the charges on an object is measured as is varied or adjusted.
4.0×10-18C, how many excess electrons does the 7. Which of the following materials is not used as
object posses, given that the charge of an electron a dielectric in capacitors? A.Glass B.Brass
is 1.6×10-19C? A.18 B.19 C.25 D.37 C.Paper foil D.paraffin wax
𝑄=charge on object=4.0×10-18C, 𝑒=electronic Dielectric substance in a capacitor are insulators
charge or charge of an electron=1.6×10-19C, Brass is a conductor hence, cannot be used as a
𝑄
𝑛=number of electrons, 𝑄 = 𝑛𝑒, 𝑛 = , dielectric. Water is not a good dielectric
𝑒
4.0×10−18 substance, because it can function as a good
𝑛= = 25.
1.6×10−19 insulator only when it is pure.
2. A capacitor of capacitance 2𝜇F is charged from 8. A parallel capacitor has a common plate area
a 12V battery and discharged through a of 5×10-8m2 and plate separartion of 2×10-3m.
milliammeter in 0.02s. Determine the current Assuming free space, what is the capacitance?
recorded by the milliammeter A.4.80×10-7𝐴 A.2.25×10-17F B.4.55010-17F C.2.25×10-16F
B.8.33×10-6𝐴C.1.20×10-6𝐴D.1.20×10-3𝑨 D.4.50×10-16F (𝜀𝑜 =9.0×10-12C2N-1m-2)
𝑄 = 𝐼𝑡, 𝑄=charge, 𝐼=current, 𝑡=time=0.02s 𝐴=5×10-8m2, 𝑑=2×10-3m, 𝜀𝑜 =9.0×10-12C2𝑁-1m-2,
𝑄 = 𝐶𝑉, 𝐶=capacitance=2𝜇F=2×10-6F, 𝜀𝑜 𝐴 9×10−12 ×5×10−8
𝑉=voltage=12𝑉, 𝑄 = 𝐶𝑉 = 𝐼𝑡, 𝐶=?, 𝐶 = = = 2.25×10-16F .
𝑑 2×10−3

230
Demystified Series Physics Demystified by Dr Timothy
9. A parallel plate capacitor of plate area 10cm 2 Calculate the effective capacitance between the
in vacuum has a capapcitance of 10-2𝜇F. What is points P and Q in the diagram above A.0.25𝜇F
the distance between the plates? (𝜀𝑜 =9.0×10- B.0.75𝝁F C.1.33𝜇F D.1.66𝜇F E.4.00𝜇F.
12Fm-1) A.9×10-13m B.9×10-7m C.9×10-3m The first three 1𝜇F capacitors are in parallel,
D.9×10 m.
7 Total capacitance = 1+1+1 = 3𝜇F.
𝐴=10cm2=10×10-4m2, 𝐶=10-2𝜇F=10-8F, The 3𝜇𝐹 is now in series with the last 1𝜇F,
𝜀 𝐴 𝜀 𝐴 3×1 3
𝜀𝑜 =9.0×10-12Fm-1, 𝑑=?, 𝐶 = 𝑜 , 𝑑 = 𝑜 , Effective capacitance, 𝐶 = = =0.75𝜇F.
𝑑 𝐶 3+1 4
𝑑=
9×10−12 ×10×10−4
= 9×10-7m
. 15. 𝐶
10−8
10. A parallel plate capacitor consists of two 𝐶
plates each of area 9.6×10-3m2, separated by a
dielectric of thickness 2.25×10-4m and dielectric 𝐶 𝐶
constant 900. Calculate the capacitance of the Four capacitors, each of capacitance 𝐶, are
capacitor. (𝜀 0=permittivity of free space= connedted as shown in the diagram above. The
8.85×10-12𝐹m-1) A.3.8×10-10F B.3.8×10-7F equivalent capacitance is A.𝐶/4 B.5𝐶/2 C.2𝑪/5
C.3.4×10 F D.3.4×10 F .
-7 -10
D.4𝐶
𝐴=9.6×10-3m2, 𝑑=thickness of the dielectric= Total capacitance of the parallel connection, 𝐶𝑃 =
2.25×10-4m, relative permittivity or dielectric 𝐶 + 𝐶 =2𝐶. This 2𝐶 is now in series with the
constant 𝜀𝑟 =900, permittivity of free space/ air other two 𝐶 capacitor, the equivalent
𝜀
𝜀𝑜 =8.85×10-12𝐹m-1, 𝜀𝑟 = 𝑚 , 𝜀𝑚 = 𝜀𝑟 𝜀𝑜 . 1 1 1 1 1+2+2 5
𝜀𝑜 capacitance; = + + = = ,
𝜀𝑚 𝐴 𝜀𝑟 𝜀𝑜 𝐴 𝐶𝑇 2𝐶 𝐶 𝐶 2𝐶 2𝐶
permitivity of the medium 𝜀𝑚 =?, 𝐶 = = 2𝐶
𝑑 𝑑 𝐶𝑇 = .
900×8.85×10−12 ×9.6×10−3 5
𝐶= = 3.4×10-7F. 16. Three 2𝜇𝐹 capacitors are to be connected in
2.25×10−4
11. The capacitance of a parallel plate capacitor such a ways as to give an effective capacitance of
is 20𝜇F in air and 60𝜇F in the presence of a 3𝜇𝐹. This can be done by connecting A.two
dielectric. What is the dielectric constant? A.6.0 parallel capacitors in series with the third B.two
B.2.0 C.0.3 D.3.0. series capacitors in parallel with the third
Capacitance with a medium 𝐶𝑚 =60𝜇F, capacitor C.all three capacitors in series D.all
Capacitance in air/free space or vacuum three capacitors in parallel.
𝐶𝑜 =20𝜇F, 𝜀𝑟 =relative permittivity or dielectric In option B, two 2𝜇𝐹 are in series,
𝐶 60 2×2 4
constant=? 𝜀𝑟 = 𝑚 = = 3. Total capacitance= = = 1𝜇F.
𝐶𝑜 20 2+2 4
12. The third 2𝜇F in parallel with 1𝜇F, equivalent
capacitance, 𝐶 = 2+1 = 3𝜇F.
In option A, two 2𝜇F are in parallel, Total
In the circuit shown above, each capacitor has a capacitance=2+2 = 4𝜇F. The third 2𝜇F in series
2×4 8
capacitance of 2𝜇F. The equvalent capacitance is with 4𝜇𝐹, equivalent capacitance, 𝐶 = = =
2+4 6
A.4/3𝝁F B.3/2𝜇F C.3𝜇F D.6𝜇F. 4
𝜇F. In option C, the three 2𝜇F in series,
Parallel capacitor : Equivalent capacitance= 3
2
arithmetic sum of individual capacitance, equivalent capacitance 𝐶 = 𝜇F. In option D, the
3
Total capacitance=2+2=4𝜇𝐹. This is now in three 2𝜇F are in parallel, equivalent capacitance,
series with the first capacitor, the equivalent 𝐶=2+2+2=6𝜇F.
𝐶 𝐶 2×4 8 4
capacitance, 𝐶𝑇 = 1 2 = = = 𝜇𝐹. 17. The plates of a 0.5𝜇F uncharged capacitor are
𝐶1 +𝐶2 2+4 6 3
connected to a 10V battery. The charge on the
13.
2𝜇F 1𝜇F 3𝜇F
capacitor after a long time is A.5.0𝝁C B.10.0𝜇C
C.20.0𝜇C D.50.0𝜇C.
2μF 𝐶=0.5𝜇F=0.5×10-6F, 𝑉=10V, 𝑄=?, 𝑄 = 𝐶𝑉 ,
The equivalent capacitance for the circuit shown 𝑄 = 0.5×10-6×10 = 5×10-6 = 5𝜇C.
above is A.8.0𝜇F B.4.5𝜇F C.0.8𝝁F D.0.5𝜇F. 18. Two 50𝜇F parallel plate capacitors are
The 1𝜇F and 3𝜇F capacitors are in parallel connected in series. The combined capacitor is
arrangement, Total capacitance=1+3=4𝜇F. The then connected across a 100V battery. The
4𝜇F is now in series with the two 2𝜇F, equivalent charge on each plate of the capacitor is
capacitance: A.5.00×10-5C B.2.5×10-3C C.1.25×10-3C
1 1 1 1 2+2+1 5 4 D.1.00×10 C.
-2
= + + = = , 𝐶 = = 0.8𝜇F.
𝐶 2 2 4 4 4 5 The charges on each of the two capacitors are
14. 1μF equal and is equal to the total charge,as they are
1μF in series. If the capacitors are of equal
P Q
1μF 1μF

231
Demystified Series Physics Demystified by Dr Timothy
magnitude, then their effective capacitance, Which of the above statements are correct? A.I
𝐶
𝐶𝑇 = , number of capacitors 𝑛=2, and IV B.II and III C.I and III D.II and IV.
𝑛 𝜀𝐴
50 𝐶∝ , Capacitance varies inversely with the
𝐶𝑇 = = 25𝜇F = 25×10-6F, 𝑉=E.M.F of battery 𝑑
2
distance of separation of the plates. 𝑄 = 𝐶𝑉,
=100𝑉, 𝑄𝑇 = 𝐶𝑉 = 25×10-6×100 = 2.5×10-3C.
𝑉=constant, ∴ 𝑄 ∝ 𝐶. i.e Charge varies directly
19. 𝑃 5𝜇𝐹
with the capacitance. Capacitance increases as
the distance of separation of the plates decreases
𝑄 10𝜇𝐹
hence, charge on each plates also increases.
23. The energy stored in a capacitor of
capacitance 10𝜇F carrying a charge of 100𝜇C is
20𝑉
A.4×10-3J B.5×10-4J C.5 ×104J D.4×102J.
The diagram above shows two capacitors 𝑃 and
𝐶=10𝜇𝐹=10×10-6𝐹, 𝑄=100𝜇C=100×10-6C
𝑄 of capacitances 5𝜇F and 10𝜇F. Find the charges 𝑄2 (100×10−6 )2 (10−4 )2 10−8
stored in 𝑃 and 𝑄 respectively. A.200𝜇C and 𝑊= = = = ,
2𝐶 2×10×10−6 2×10−5 2×10−5
100𝜇C B.100𝝁C and 200𝝁C C.4𝜇C and 2𝜇C 𝑊 =5×10 J. -4
D.2𝜇C and 4𝜇C. 24. 2×10-4J of energy is stored in a 0.01𝜇F
𝑉𝑃 =𝑉𝑄 =𝑉=20𝑉(parallel capacitors), 𝐶𝑃 =5𝜇𝐹= capacitor. The charge on each plates is A.2×10-
5×10-6𝐹, 𝐶𝑄 =10𝜇𝐹=10×10-6𝐹, 6𝜇C B.2𝝁C C.2×103𝜇C D.2×108𝜇C.

𝑄𝑃 = 𝐶𝑃 𝑉 =5×10-6×20=100×10-6C=100𝜇C. 𝑊=2×10-4J, 𝐶=0.01𝜇F=0.01×10-6F, 𝑄=?,


𝑄𝑄 = 𝐶𝑄 𝑉 =10×10-6×20=200×10-6C=200𝜇C 𝑄2
𝑊= , 𝑄2 =2𝑊𝐶 = 2×2×10-4×0.01×10-6,
2𝐶
Alternatively, 𝑄 ∝ 𝐶 (parallel capacitors). Thus,
the higher capacitance will store the highest 𝑄2 =4×10-12 , 𝑄 = √4 × 10−12 = 2×10-6C ,
charge. 𝐶𝑇 =10+5 =15𝜇𝐹 𝑄 = 2 𝜇C.
𝑄𝑇 = 𝐶𝑇 𝑉 =15𝜇 ×20=300𝜇C. 25. A capacitor 8𝜇𝐹, is charged to a potential
𝐶 5 difference of 100V. The energy stored by the
𝑄𝑃 = 𝑃 × 𝑄𝑇 = ×300𝜇 =100𝜇C capacitor is A.1.0×104J B.8.0×10J C.1.25 ×10J
𝐶𝑃 +𝐶𝑄 5+10

𝑄𝑄 =
𝐶𝑄
× 𝑄𝑇 =
5
×300𝜇 =200𝜇C. D.4×10-2J.
𝐶𝑄 +𝐶𝑃 5+10 𝐶=8𝜇𝐹=8×10-6𝐹, 𝑉=100𝑉, 𝑊=?
20. 5𝜇F 15𝜇F 1 1
𝑊 = 𝐶𝑉 2 = ×8×10-6×(100)2 ,
20𝜇F 2 2
𝑊 = 4×10-6×104 = 4×10-2J.
26. The diagram below shows two capacitors 𝑃
24𝑉 and 𝑄 of capacitances 2𝜇F and 4𝜇F respectively
In the network shown,determine the potential connected to a d.c source.The ratio of the energy
difference across the 5𝜇F capacitor A.3V B.6V stored in 𝑃 to 𝑄 is
C.12V D.18V. 𝑃 2𝜇𝐹
Voltage 𝑉(across 20𝜇F)=Voltage 𝑉(across 5𝜇F
and 15𝜇F)=𝑉𝑇 =24𝑉, as they are in parallel. 𝑄 4𝜇𝐹
5𝜇F and 15𝜇F are connected in series –
5×15 75
𝐶= = = 3.75𝜇𝐹.
5+15 2 𝑉 A.4:1 B.1:2 C.1:4 D.2:1.
The charge in the series capacitors – 1
𝐶𝑃 =2𝜇𝐹, 𝐶𝑄 =4𝜇𝐹, 𝑊 = 𝐶𝑉 2 ,
𝑄 = 𝐶𝑉 = 3.75𝜇 ×24 = 90𝜇C. 2
1
𝑄(5𝜇F)=𝑄(15𝜇F), series capacitors. 𝑉 = ,
𝑄 Energy stored in 𝑃, 𝑊𝑃 = 𝐶𝑃 𝑉 2 ,
2
𝐶 1
𝑉(across 5𝜇F)=
90𝜇
= 18𝑉. Alternatively, Enegry stored in 𝑄, 𝑊𝑄 = 𝐶𝑄 𝑉 2,
2
3.75𝜇 𝑊𝑃
1 Ratio of energy stored in 𝑃 to 𝑄 = ,
𝑉∝ (series capacitors) thus, the smaller 𝑊𝑄
𝐶 1
𝐶 𝑉2
capacitance will have the highest 𝑊𝑃
= 2 𝑃
=
𝐶𝑃 2
= =
1
, 𝑊𝑃 :𝑊𝑄 =1:2.
1
𝐶 𝑊𝑄 𝐶 𝑉2 𝐶𝑄 4 2
voltage.𝑉𝑇 =24V, 𝑉(across 5𝜇F) = 15 × 𝑉𝑇 , 2 𝑄

15
𝐶5 +𝐶15
Alternatively, the ratio of energy stored in two
𝑉(across 5𝜇F)= ×24 = 18𝑉. parallel capacitors is equal to the ratio of their
5+15
21. The plates of a parallel-plate capacitor with capacitance.
air separating the plates are connected to a 27. 2𝜇F
battery so that the potential difference between
the plates is constant. If the separation between 3𝜇F
the plates is decreased I.the capacitance of the
capacitor decreases II.the capacitance of the 100𝑉
capacitor increases III.the charge on each plate In the circuit above, the potential across each
decreases IV.the charge on each plate increases capacitor is 100V. The total energy stored in the

232
Demystified Series Physics Demystified by Dr Timothy
two capacitors is A.3.0×104J B.3.0×102J
C.2.5×10 J D.6.0×10 J
-2 -3 .

Effective capacitance 𝐶= 2+3=5𝜇F=5×10-6F


1 1
𝑊 = 𝐶𝑉 2 = ×5×10-6×1002 =2.5×10-2J.
2 2
28. 2𝐶

𝐶
𝐾
𝑉
Two capacitors of capacitances 𝐶 and 2𝐶, are
connected across a voltage as shown in the
diagram. With the key 𝐾 open, the energy stored
by the 2𝐶 capacitor is 𝐸. If the key is now closed
and the system attains equilibrium, the total
energy stored in the capacitors is A.2𝐸/3 B.𝐸
C.3𝑬/2 D.3𝐸.
With the key open, the capacitor 𝐶 is cut off from
the connection, Energy stored in 2𝐶 = 𝐸 =
1
×2𝐶 × 𝑉 2 = 𝐶𝑉 2 .
2
With the key closed, both capacitors 2𝐶 and 𝐶,
are involved in the connection. Total
capacitance=2𝐶 + 𝐶 =3𝐶 .
Energy stored at equilibrium in capacitors
1 3 3𝐸
combined = ×3𝐶 × 𝑉 2 = 𝐶𝑉 2 = .
2 2 2
Therefore, the energy stored when the key is
closed=3𝐸/2 or 1.5𝐸.

Jamb past questions on electrostatics, electric


field and capacitors :
[1978/29,1980/9,21,1982/26,32,1983/47,1984
/7,35,1987/40,1988/42,1989/32,1990/36,
1991/39,1992/39,41,1995/37,43,1997/39,
2001/25,26,30,2000/38,2004/50,2005/17,19,
2006/32,2008/38,2009/39,2010/33,34,2013/
36,37,2014/33,2014/34,37,2015/11,15]

233
Demystified Series Physics Demystified by Dr Timothy

Chapter 22 – Electric cells, Conduction of electricity


through liquids and gases
● Electric cells – These are cells which produce E.M.F of the cell and an opposing or back or E.M.F
current that flows in one direction i.e direct to the original E.M.F.The internal resistance of
current, by the conversion of chemical energy to the cell also increases and reduces the amount of
electric energy. It consists of two dissimilar current produced.
electrodes and an electrolyte. - Polarisation can be prevented by adding a
- The electrolyte dissociates into positive and depolariser e.g manganese dioxide, which
negative ions. On passage of current, positive oxidises hydrogen to water or by brushing the
ions are attracted to the negative electrode, copper electrode frequently.
where they are neutralized by accepting 2. Local action – Dissolution of zinc electrode in
electrons while the negative ions are attracted to the electrolyte to liberate hydrogen, due to
the positive electrode where they are neutralized impurities such as iron and carbon,which set up
by losing electrons. tiny local cells at zinc surface.
- Current is generated due to the flow of ions. - Local action can be prevented by cleaning in
- Current flows from the positive to the negative sulpuric acid and rubbing mercury on the zinc
electrode in the external conductor and from (i.e amalagamating the zinc electrode) or by
negative to the positive electrode in the using a pure zinc electrode.
electrolyte. - Daniel cell – Copper is the positive electrode in
- Cells are classified based on the E.M.F i.e copper sulphate solution(copper plating), zinc is
strength of the cell,amount of current produced the negative electrode in sulphuric acid
and internal resistance of the cell. (electrolyte). The copper sulphate solution is the
- Electromotive force E.M.F or 𝑬of a cell – It is depolariser.It has a low E.M.F of 1.1V and
the total energy reuired to send a unit positive provides only a small/low current for a short
charge round a complete circuit.It is the driving time.It keeps a steady current and can be
force for the current in a circuit or the potential used as a standard cell.
difference between two electrodes when no - Leclanche cell (wet and dry cell) – Carbon is
current is flowing. the positive electrode, zinc is the negative
- Internal resistance 𝒓 of a cell – It is the electrode, ammonium chloride(solution for wet
opposition to the flow of current in the cell type and paste for dry type) is the electrolyte. The
producing the current. depolariser is a mixture of powdered carbon and
- Types of electric cells – manganese dioxide.
1.Primary cells: Cells that the process through - It keeps an unsteady current and is used for
which electric current is generated is irreversible supplying intermittent current in
e.g simple voltaic cell, Daniel cell, Lechlanche battery(cell) used in torch-lights and
cells(wet and dry). transistor radios.
2.Secondary cells ( or Accumulators): Cells in - Lead acid accumulator – Lead dioxide(PbO2)
which the chemical process through which is the positive electrode,lead is the negative
current is generated is reversible i.e it can be electrode and sulphuric acid is the electrolyte.
recharged by passing current into the cell in a - The relative density of the acid is 1.25 and the
reverse direction e.g acid cell(Lead-acid cell), E.M.F is about 2.2V when fully charged and drops
alkaline cell(nickel-iron cell). as it discharges. It is used in car batteries, phone
-The internal resistance of Daniel cell (𝐷) , simple batteries e.t.c.
voltaic cell (𝑆), Leclanche cell (𝐿) and - Discharging (supplying current) – The
accumulator (𝐴), increases in the order – 𝒓𝑨 < strength of the electrolyte diminishes i.e relative
𝒓𝑫 < 𝒓𝑳 < 𝒓𝑺 . Thus, accumulators have the density drops, and the E.M.F also drops, while the
lowest internal resistance and will produce ammeter reading rises. Lead sulphate is formed
the largest amount of current. on the surface of electrodes.
- Simple Voltaic cell – Copper(Cu) is the positive - Recharging – Current is passed in opposite and
electrode(anode), zinc(Zn) is the negative reverse direction by a direct current(D.C) source
electrode(cathode), while sulphuric acid(H2SO4) or alternate current(A.C) source with a rectifier
is the electrolyte. (it converts the A.C to D.C).The ammeter reading
- All primary cells are affected by polarisation falls as the E.M.F and relative density of the
and local action. electrolyte rises to their original value.
1. Polarisation – Accumulation of hydrogen - Maintenance/Care for acid cells(batteries) –
bubbles on the copper plate electrode or anode 1.Topping up charge by regular recharging of the
as current is supplied.It causes a reduction in the cell or removal of electrolyte and drying the cell

234
Demystified Series Physics Demystified by Dr Timothy
when not in used. electrolyte is unchanged. e.g copper electrodes in
2.Avoiding shorting of terminals i.e connecting a copper sulphate solution.
wire directly across the terminals of a cell or - Faraday’s laws of electrolysis :
improper terminal connections during 1. Faraday’s first law of electrolysis – The mass
recharging(positive pole of a cell to the negative 𝑚 of a substance liberated during electrolysis is
terminal of the D.C source),as it produces a directly proportional to the quantity of electricity
relatively low resistance and large current which 𝑄 passing through the circuit. 𝒎 ∝ 𝑸, 𝒎 = 𝒁𝑸,
might be harmful to handle. 𝑸=𝑰𝒕, ∴ 𝒎 = 𝒁𝑰𝒕. 𝑍=electrochemical equivalent
3.Avoiding over discharging of the cell for a long of the substance (e.c.e)= mass of the substance
period of time. liberated by 1coulumb of electricity.
4.Cleaning battery terminals with water and 𝒎 = 𝝆𝑽, 𝑽 = 𝑨 × 𝒕𝒉, 𝜌=density of the substance,
smeared with grease to prevent current leakage 𝑉=volume of the substance, 𝐴=area of the
5.Regular topping of electrolyte level i.e substance, 𝑡ℎ=thickness of the substance.
maintaining the electrolyte level with water or 2. Faraday’ssecond law of electrolysis – If the
compensating the loss of water through same quantity of electricity is passed through
evaporation.Acid should not be added to the different electrolytes, the masses 𝑚 of elements
cell except when spillage occurs. liberated are proportional to
- Nickel-Iron (NIFE) accumulator – Nickel(Ni) their chemical equivalent 𝐶.𝑒.
or nickel hydroxide is the positive electrode, iron 𝒎 ∝C.e, C.e =
𝑹𝑨𝑴
or mass deposited by
𝒗
(Fe) is the negative electrode and caustic potash
1Faraday. 𝑹𝑨𝑴=relative atomic mass,
(or potassium hydroxide solution) is the
v=valency of the substance.
electrolyte.
- Application of electrolysis –
- The NIFE accumulator supply larger current
1.Electroplating – The metal to be electroplated
(used for heavy duty vehicle) and last
is the cathode, the coating or electroplating metal
longerthanlead-acid accumulator. They don’t
is the anode and the electrolyte is a salt of the
need special maintenance as they can be left
anode metal.
discharged for a long time without spoiling. -
2.Extraction of metals e.g sodium.
Capacity of an accumulator or battery
3.Purification of metals.
capacity is measured in Ampere-hours 𝑨𝑯.
4.Calibration of ammeter – The ammeter
Capacity of an accumulator or battery
connected in series with a voltameter measures
capcaicty = Current 𝑰 × recharge time 𝒕.
the current 𝑰𝑨 , flowing through the circuit for a
Current is in amperes 𝐴 while time is in hours.
known time 𝒕and mass deposited 𝒎is measured.
- Electrolytes –These are substance that
The exact current value 𝑰is calculated from
dissociates ito positive ions(cations) and
faraday’s first law.
negative ions(anions) in aqueous solution.
Error in ammeter reading= 𝑰 − 𝑰𝑨 .
- Strong electrolytes are either strong acids,
Percentage error in ammeter reading =
alkali or salts e.g sulphuric acid(H2SO4), caustic 𝑰−𝑰𝑨
soda(NaOH), brine(NaCl) e.t.c. × 100.
𝑰
- Weak ekectrolytes are weak acids,alkalis and
their salts e.g ethanoic acid, grape juice e.t.c ● Conduction in gases
- Non-electrolytes are covalent substance e.g - Discharge through gases – Under normal
glucose(sugar), alcohol, hydrocarbons e.t.c. conditions gases are insulators.
- Gases conduct electricity better at high
● Conduction through liquids voltage(or potential difference),low pressure
- Electrolysis – Electrolysis is the chemical and with a small quantity of air. Pumping of air
decomposition of electrolytes and migration of out of the tube reduces the pressure.
its ions due to the flow of direct current (D.C) - Rate of emission or Conductivity of gases is
through it. It is demonstrated in a volatmeter, affected by applied voltage 𝑽,pressure of the
which contains an electrolyte, anode(positive gas 𝑷, and the quantity (mass) of gas present
𝑽
electrode) and cathode (negative electrode). 𝒎. Conductivity ∝ .
𝑷𝒎
- These ions moves randomly when no current is -At ordinary or high pressure and high voltage
passing. When current flows through the gases do not conduct but produce a spark or
voltameter, the positive ions are attracted to the discharge.
cathode while the negative ions are attracted to - At low pressure and high voltage gases produce
the anode. a glow due to steady conduction of current rather
- If the electrodes are similar metals and the than sparks or discharge.Further reduction of the
electrolyte is a salt of the metal,atoms of the pressure gives a dark space which splits into
metals from the anode are deposited on the bands extending through the length of the tube
cathode and the colour and density of the

235
Demystified Series Physics Demystified by Dr Timothy
and the glass glows green. on the anode C.copper sulphate is formed in the
- Conduction in gases at low pressure and high solution D.the filament of the bulb gradually
voltage is referred to as cold cathode or field burns out.
- The graph below is the current(𝐼)-voltage(𝑉) 7. The most suitable cell used for short interval
relationship in the conduction of electricity switches in electric bells is a A.lead acid
through gases. accumulator B.Daniel cell C.Leclanche cell
𝑰 D.Nickel-iron accumulator
Leclanche cells keeps unsteady current and for
supplying intermittent current.
8. Which of the following can be used to reduce
𝑽 local action in a Leclanche cell? A.a carbon rod as
- The colour of light emitted depends on the the positive pole B.pure zinc as the negative
nature of the gas used in th discharge tube e.g pole C.potassium permanganate solution in
Sodium vapour is yellow, Air is pink, Merucry contact with the positive pole D.common salt
vapour is light blue e.t.c. solution
Local action is caused by the presence of
Examples : impurities, thus using a pure zinc prevent its
1. A discharging electrochemical cell converts occurenece.
A.electrical to chemical energy B.electrical to - Potassium permanganate can act as a
light energy C.chemical to electrical energy depolarizer to prevent polarization.
D.chemical to light energy - Primary and secondary cells store chemical
During discharge,it supplies electricity from energy which is converted into electrical
chemical reactions in the cell and the process energy when supplying current.
reverses during re-charge as a direct current 9. In a Leclanche cell, the role of granulated
source supplied electricity to the cell. carbon mixed with manganese(IV) oxide is to
2. When a storage cell is re-charged A.chemical I.increase the emf of the cell II.prevent local
energy is converted into heat energy B.heat action in the cell III.prevent polarization in the
energy is converted into chemical energy cell IV.reduce the internal resistance of the cell.
C.chemical energy is converted into electrical Which of the above is/are correct? A.II B.III C.I
energy D.electrical is converted into chemical and II D.III and IV.
energy. 10. Which of the following is not a component of
3.Which of the following chemicals is used in a dry Leclanche cell? A.carbon rod B.paste of
simple voltaic cells? A.ammonium chloride dilute sulphuric acid C.zinc container
B.manganese dioxide C.powdered carbon D.manganese dioxide mixed with powdered
D.copper sulphate E.sulphuric acid carbon E.carboard disc at the bottom, separating
Sulphuric acid is the electrolyte in a simple the positive pole from the negative pole.
voltaic cell. The electrodes of Leclanche cell(wet or dry type)
4. In a simple cell, the zinc electrode is are carbon rod(positive electrode) and zinc
amalgamated with mercury inorder to prevent case(negative electrode).The electrolyte of a dry
A.local action B.polarization C.overheating Leclanche cell is paste of ammonium
D.corrosion chloride.Cardboard disc at the bottom of the cell
Amalgamation prevents local action. prevents the positive carbon pole from touching
5. Which of the following is not consequence of the negative zin container to prevent short
hydrogen bubbles covering the copper plate of a circuiting of the cell.
primary cell? A.polarization B.local action 11. A dry form of Leclanche cell slowly
C.formation of hydrogen electrode D.increase in deteriorates even when not in use due to the
the resistance of the cell E.generation of less A.presence of zinc case which is lined with
current by the cell absorbent paper B.internal chemical action
Polarization results when hydrogen bubbles caused by the presence of small impurities in
accumulate on the copper electrode of a simple the zinc plate C.presence of hydrogen bubbles
voltaic cell.This forms an hydrogen electrode i.e formed on the cathode when current flows
an opposing or back E.M.F, thereby increasing the through the cell D.presence of depolarizing agent
internal resistance of the cell and reducing the which oxidizes hydrogen to water.
amount of current produced. Local action is caused by the presence of iron and
6. A light bulb is connected to a simple voltaic carbon impurities in the zinc electrode, and they
cell, when the circuit is closed the bulb glows set up tiny cells which leads to the wearing away
brightly for a short while and then becomes dim. of the zinc case.It occurs either when the cell is
This is because A.hydrogen gas is produced at supplying current or not in use. Polarization
the cathode B.hydrogen bubbles accumulate only occurs when the cell is supplying

236
Demystified Series Physics Demystified by Dr Timothy
current. irreversible B.a secondary cell can be recharged
12. Which of the underlisted factors determine C.as a lead acid accumulator is being charged the
the emf of a voltaic cell? I.Quantity of the density of the acid in it increases D.a lead acid
electrolyte II.Type of electrodes III.Dimensions of accumulator can deliver a large current
the cell A.I B.II C.II and III D.I,II and III. because of its very large internal resistance
The emf of a cell is independent of the quatity of E.the conventional current in a circuit flows from
the electrolyte and the dimension or size of the the positive to the negative terminal of the
cell. E.M.F of a primary cell depends on the battery.
nature of the electrolyte and electrode (plates) Lead-acid accumulator can deliver a large
and the more the anode is prone to polarization current due to its low internal resistance.
the lesser the emf produced. 20. A safety precautiondesigned toprolong the
13. The internal resistance of a cell depends on life of a lead-acid accumulator is A.by topping up
I.cell’s arrangement II.the arramgement of the the cells with concentrated acid B.the use of
electrodes III.the quantity of the electrolyte naked flame when charging the battery
IV.the conductivity of the electrolyte. Which of C.keeping the specific gravity of the
the above are true? A.I and III B.II and IV C.I,II electrolyte within specific range D.by
and IV D.I,II,III and IV. connecting wires to the terminals directly when
14. Which of the following has the lowest testing the level of charging.
internal resistance when new? A.Leclanche cell 21. In order to check whether an accumulator
B.Daniel cell C.Accumulator D.Torce cell needs recharging, one may first A.determine
- Torch cell is a dry Leclanche cell. the relative density of the liquid B.check the
15. An accumulator is rated in the unit of level of the liquid C.short circuiting the terminals
ampere-hour. This is equivalent to the unit of wit a wire and observe the sparks D.measure the
A.power B.energy C.charge D.potential temperature of the liquid E.none of the above
difference. 22. Eight dry cells when connected in series
A battery with capacity of one ampere-hour will produce the same emf as a 12volt car battery, but
deliver a current of one ampere in one hour. This they will not be suitable for the starting of a are
is equal to a charge flow of 3600 coulombs i.e connected in series C.high internal resistance
1A×3600s. of the cells D.of the high internal resistance of
16. A car battery rated 45𝐴𝐻 is charged with a the starter motor E.of the high emf of the starter
charger whose rating is 2.5𝐴. How long will it motor
rake to charge the battery fully? A.20hrs B.10hrs The dry cells cannot supply the high current
C.25hrs D.18hrs needed to start the car due to its high internal
Capacity of the car battery=45𝐴𝐻, current resistance. A car battery(accumulator) can
𝐼=2.5𝐴, reachrge time 𝑡=?, supply a high current needed to start the car due
Capacity of a cell = 𝐼(𝐴) × 𝑡(𝐻) , 45 = 2.5 × 𝑡 , to its low internal resistance.
recharge time 𝑡 =
45
= 18hrs. 23. Which of the following leads to a deacreas in
2.5
the internal resistance of a lead-acid
17. A 60𝐴𝐻 uncharged car battery is to be re-
accumulator? I.increasing the areas of the
charged using a current of 12A. The time
electrodes II.decreasing the separation of the
required for the battery to acquire a full charge is
electrodes III.increasing the separation of the
A.5h B.12h C.60h D.720h
electrodes IV.decreasing the area of the
Capacity of the accumulator=60Ah, 𝐼=12𝐴 ,
electrodes A.I and II B.II and III C.II and IV D.III
reacharge time 𝑡=?,
and IV
Capacity of a cell = 𝐼(𝐴) × 𝑡(𝐻) , 60 = 12 × 𝑡 ,
60 Internal resistance of a cell is affected by the
recharge time 𝑡 = = 5hrs. separation(distance) of the electrodes and area
12
18. Which of the following statements is not of the electrodes. Internal resisatnce decreases
correct about a lead-acid accumulator? A.It can when the area of the electrodes increases (𝑟 ∝
be recharged B.it has an emf of 2.0V, which 1/𝐴) and reducing the distance of separation
remains steady on discharge C.it can deliver between the plates (𝑟 ∝ 𝑑).
more current than a battery of single cells with 24. Which of the following properties is an
an equal emf D.it can be discharged by leaving the advantage of lead-acid accumulator over an
two terminals connected for a long time alkaline accumulator? A.Possesses low internal
E.formation of excessive lead sulphate may occur resistance B.Can be recharged C.Has shorter life
under certain conditions. span D.Possess higher emf.
The emf and relative density of a lead-acid The emf of a lead-acid cell is about 2V while that
accumulator falls slowly when discharging. of an alkaline cell is about 1.25V. Alkaline cells
19. Which of the following statements is not have longer life span than lead-acid cells. Both
true? A.a chemical action in a primary cell is cells possess low internal resistance and can be

237
Demystified Series Physics Demystified by Dr Timothy
recharged. copper is 104g/cm3) 𝐼=0.5A, 𝐴=100cm2,
25. In the conductance of aqueous potassium 𝑡=100min=100×60=6000s, 𝑍=0.00033g/C,
tetraoxosulphate(VI) solution, current carriers 𝜌=104g/cm3, thickness 𝑡ℎ=?, 𝑚 = 𝑍𝐼𝑡,
are the A.ions B.electrodes C.hydrated ions 𝑚 = 𝜌𝑉 = 𝜌(𝐴 × 𝑡ℎ), 𝜌 × 𝐴 × 𝑡ℎ = 𝑍𝐼𝑡,
D.hydrated electrons. 104×100× 𝑡ℎ = 0.00033×0.5×6000,
Aqueous potassium tetraoxosulphate(VI) 𝑡ℎ =
0.00033×0.5×6000
= 0.99×10-6cm
104 ×100
solution is an electrolyte. Charge or current
th ≅ 1.0×10-6cm=10mm×10-6=10-5mm.
carriers in liquids are ions.
32. Copper of density 104g/cm3 and
26. Which of the following is an electrolyte?
electrochemical equivalent 3.3×10-4g/C is to be
A.sugar solution B.alcohol C.paraffin D.grape
plated on the cathode of a voltameter. If the total
juice.
surface area of the cathode is 66cm2 and a
27. Which of the following statements about
current of 50A is maintained for 1 hour, the
electrolytes is not correct? A.the substances in
thickness of copper plated will be approximately
solution in the electrolyte become ionized B.ions
A.1.80mm B.0.90mm C.0.18mm D.0.09mm
are electrically charged and are attracted
𝜌=104g/cm3=104×1000kg/m3=107kg/m3,
towards electrodes when a potential difference is
𝑍=3.3×10-4g/C, 𝐴=66cm2=66×10-4m2, 𝐼=50𝐴,
applied C.ions may be discharged at the
𝑡=1hr=3600s, 𝑡ℎ=?, 𝑚 = 𝑍𝐼𝑡,
electrodes, forming bubbles or deposits D.the
𝑚 = 𝜌𝑉 = 𝜌(𝐴 × 𝑡ℎ), 𝜌 × 𝐴 × 𝑡ℎ = 𝑍𝐼𝑡,
mass deposited depends upon the length of time
107×66×10-4× 𝑡ℎ = 3.3×10-4×50×3600,
for which current flows E.the mass of gas set 3.3×10−4 ×50×3600
free or metal deposited is proportional to the 𝑡ℎ = = 0.0009m = 0.90mm.
107 ×66×10−4
square of the current. 33. A charge of one coulomb liberates 0.0033g of
The mass of substance deposited is directly copper in an electrolytic process. How long will it
proportional to the current(𝐼) and the time(𝑡). take a current of 2A to liberate 1.98g of copper in
𝑚 ∝ 𝑄, 𝑄 = 𝐼𝑡 , 𝑚 ∝ 𝐼𝑡 or 𝑚 ∝ 𝐼 and 𝑚 ∝ 𝑡. such a process? A.5 minutes B.30 minutes C.50
28. The unit of quantity of electricity is called minutes D.60 minutes E.120 minutes
A.the ampere B.the volt C.the coulomb D.the 𝑍=electrochemical equivalent=mass liberated by
ammeter E.electromotive force. 1 coulomb of electricity=0.0033g, 𝐼=2A, 𝑚=1.98g,
29. In Faraday’s law of electrolysis, a graph of 𝑡=? 𝑚 = 𝑍𝐼𝑡, 1.98 = 0.0033×2× 𝑡,
mass deposited against the quantity of electricity 𝑡=
1.98
= 300s =
300
= 5 minutes.
is plotted. The slope of the graph gives A.the 0.0033×2 60
34. 1g of copper is liberated by a charge flow of
current flowing B.the energy released C.the
103C in a voltameter. In order to liberate 7.2g of
charge released E.the electrochemical
copper in the same voltameter, a current of 2A
equivalent.
should be maintained for A.6min B.30min
𝑚 𝑚
C.60min D.120min
𝑚 𝑚1 =1g, 𝑄1 =𝐼1 𝑡1 =103C, 𝑚2 =7.2g, 𝐼2 =2A, 𝑡2 =?
𝑍= 𝑍1 =𝑍2 =𝑍 i.e constant, as both are copper.
𝑄
𝑚 𝑚1 𝑚
𝑄 𝐼 or 𝑡 𝑚 = 𝑍𝐼𝑡, 𝑍 = , = 2 ,
𝐼𝑡 𝐼1 𝑡1 𝐼2 𝑡2
The graph of the mass 𝑚 deposited during 1 7.2 103 ×7.2
electrolysis and the quantity of electricity 𝑄 is a = ,𝑡2 = = 3600s =60min.
103 2×𝑡2 2×1
straight line raph with a slope equal to the 35. During the electrolysis of copper(II)
electrochemical equivalent. tetraoxosulphate(VI) solution, an ammeter
30. The electrochemical equivalent of a metal is shows a steady current reading of 1.0𝐴 for 30
0.126×10-6Kg/C. The mass of the metal that a minutes while 6.6×10-4kg of copper is liberated.
current of 5A will deposit from a suitable bath in Calculate the error in the ammeter reading [The
1 hour is A.0.0378×10-3Kg B.0.227×10-3Kg electrochemical equivalent of copper is 3.30×10-
C.0.378×10-3Kg D.0.595×10-3Kg E.2.268×10- 7kgC-1] A.0.001𝐴 B.0.011𝐴C.0.111𝑨 D.1.000𝐴
3Kg.
E.1.100𝐴
𝑍=0.126×10-6Kg/C, 𝐼=5A, 𝑡=1 hour=3600s, 𝑚=? Faulty ammeter reading 𝐼𝐴 =1.0𝐴,𝑚=6.6×10-4kg,
𝑚 = 𝑍𝐼𝑡, 𝑚 =0.126×10-6×5×3600, 𝑡=30min=30×60=1800s, 𝑍= 3.30× 10-7kgC-1,
𝑚 = 2268×10-6=2.268×10-3kg . Actual current=𝐼=? 𝑚 = 𝑍𝐼𝑡, 𝐼 = ,
𝑚
31. In copper plating, a current of 0.5A is allowed 6.6×10−4
𝑍𝑡

for a cathode area 100cm2. If the current is 𝐼= = 1.111𝐴 ,


3.3×10−7 ×1800
maintained constant for 100minutes the Error in ammeter reading= 𝐼 − 𝐼𝐴 = 1.111−1 ,
thickness of the copper deposited will be Error in ammeter reading = 0.111𝐴.
approximately A.10-2mm B.10-3mm C.10-4mm 36. In the process of silver-plating a metal M, the
D.10-5mm E.10-6mm (the electrochemical metal is at the A.anode and a direct current is
equivalent of copper is 0.00033g/C, density of used B.cathode and an alternate current is used

238
Demystified Series Physics Demystified by Dr Timothy
C.cathode and an alternate current is used
D.cathode and a direct current is used
The metal to be plated is placed at the anode, the
plating metal i.e silver in this case, is the anode
while the electrolyte is a soluble salt of the
plating metal(silver). A direct current and not an
alternate current, is used in electrolytic process.
37. Gases conducts electricity under A.high
pressure and high voltage B.low pressure and
low voltage C.normal pressure and low voltage
D.low pressure and high voltage.
Gases conduct electric current under low
pressure and high volatage. Charge carriers in
gases are ions and free electrons.
38. Which of the following is not true of a
discharge tube? A.the pressure of the gas must
be very low B.the glass tube must have two
electrodes at the ends C.the gas in the tube
must be carbon dioxide D.air is gradually
pumped out of the tube E.the coating on the walls
of the tube at a certain stage glows.
Any gas is used and the colour of light emitted is
dependent on the nature of the gas used.
39. Which of the following affects the
conductivity of gases in a tube I.the pressure of
the gas II.mass of the air III.applied voltage A.I
B.III C.I and III D.I,II and III.
Gases conducts electricity under high voltage,
low pressure and small quantity(mass) of gas.

Jamb past questions on electric cells,


conduction of electricity thrpugh liquids and
gases :
[1979/30,1984/11,1986/42,1989/44,1991/36
44,1992/35,1993/36,46,1998/47,2000/48,200
1/29,37,41,2002/40,2003/39,2005/3,5,31,200
6/22,2007/21,2010/45,2011/40,46,2012/38,
2013/45,2014/46,2015/39]

239
Demystified Series Physics Demystified by Dr Timothy

Chapter 23 – Current Electricity, Electric Power and Energy


● Current electricity – Current electricity conductor :
involves the flow of electrons or charges round a 𝟏
1.Area 𝑹 ∝ : Resistance decreases with area of
𝑨
conductor.
the wire i.e thick(large area) or thin wire(small
- Electric current 𝑰 – It is the rate of flow of
area).
electric charge round a circuit or the quantity of
𝑸 2.Length, 𝑹 ∝ 𝑳 : Resistance increases as length
electricity 𝑸, flowing per unit time 𝒕. 𝑰 = , of the wire increases.
𝒕
𝑰 = [Cs-1 or 𝑨]. The unit for current is ampere(A). 3.Temperature, 𝑹 ∝ 𝑻 : Resistance of a metallic
- Terminal potential difference or voltage 𝑽 – wire increases with temperature.
It is the workdone or energy in volt in moving a 4.Nature of the material i.e resistivity , 𝝆.
unit charge from one point in a circuit to the 𝑳 𝝆𝑳
𝑹 ∝ , 𝑹 = . Thus, a short thick wire has a
𝑨 𝑨
other. It is the P.d or voltage in a ciruit delivering
lesser resistance than a long thick wire of the
current i.e closed circuit.
same material.
- Electromotive force(emf) 𝑬 – It is the total
- For non-metallic conductors like graphite, their
workdone or energy in volt in moving a unit
resistance decreases with temeperature.
charge round an electrical circuit. It is also the
Manganin and Constantine are used for making
voltage or p.d in a circuit not delivering current
𝑾 standard resistors as they little affected by
(open circuit). 𝑬 = , 𝑬 = [𝑱C-1 or 𝑽]. The unit temperature changes.
𝑸
of potential difference is volt (V). - Conductivity 𝝈 of a material is the reciprocal of
𝟏
- Factors affecting the emf of a cell – its resistivity, 𝝈 = .
𝝆
1.Nature(quality) of chemicals or materials used
- The resistance and resistivity of a metallic
in a cell. It does not depend on the quantity
conductor increases with temperature. If the
(amount) of chemicals used or the size of the cell.
resistance 𝑹𝒐 and resistivity 𝝆𝒐 at 0℃ and 𝑹 and
i.e the stronger the electrolyte,the lower the
𝝆 at 𝑻 then, 𝑹 = 𝑹𝒐 (1+𝜶𝑻), 𝝆 = 𝝆𝒐 (1+𝜶𝑻) .
internal resistance and the greater the emf.
𝜶 is the temperature coefficient of resistance.
2.It depends on the number of cells, time and
- Positive temperature coefficient of
temperature. E.M.F decreases with time and
resistance – The resistance increases with
temperature e.g primary cells runs down with
temperature. e.g metals.
time.
- Negative temperature coefficient of
- Internal resistance of a cell 𝒓 – It is the
resistance – The resistance decreases with
opposition to the flow of current by the cell
temperature e.g semi-conductors (silicon) and
producing the current.
insulators.
- Factors affecting the internal resistance of a
- Zero temperature coefficient of resistance –
cell – It depends on E.M.F of the cell and :
The resistance if not affected by temperature e.g
1.Nature of chemicals or electrodes.
manganin and constatine.
2.Amount of chemicals or electrodes and the size
- A graph of the resistance 𝑹 of a metallic wire
of the cell.
𝟏 with temperature 𝑻 is a straight line graph with a
3.Area of the electrodes(plates), 𝒓 ∝ . positive slope (𝑹𝒐 𝜶) and intercept 𝑹𝒐 at the 𝑅-
𝑨
4.Separation/distance between the electrodes axis. 𝑹 = 𝑹𝒐 (1+𝜶𝑻), 𝑹 = 𝑹𝒐 + 𝑹𝒐 𝜶(𝑻) .
𝒓 ∝ 𝒅. 𝑹(Ω)
- Internal resistance determines the
maximum current that can be supplied by a 𝑹𝒐
cell i.e the more the internal resistance, the 0 𝑻(℃)
lesser current the cell can supply. - Ohm’s law – It states that the potential
𝑬
𝑰𝒎𝒂𝒙 = , 𝐼𝑚𝑎𝑥 =maximum current, 𝐸=E.M.F, difference 𝑽 across a metallic conductor is
𝒓
directly proportional to the current 𝑰 flowing
𝑟=internal resistance of the cell.
through it, provided temperature and other
- Lost volt v – It is the potential difference across
physical conditions like length and area of the
the internal resistance of a cell.
conductor, remains constant. 𝑽 ∝ 𝑰, 𝑽 = 𝑰𝑹,
- External Resistance or Electrical resistance
Resistance is the constant of proportionality,
𝑹 – It is the opposition to the flow of current in a 𝑽
material. 𝑹= .
𝑰
- Good conductors offers low resistance to - Ohmic conductors are the ones that obeys
current, while poor conductors (insulators) ohm’s law i.e they have a linear graph between
offers high resistance to current. voltage and current e.g all metals and resistors.
- Factors affecting the resistance of a - Non-ohmic conductors do not obey ohm’s law

240
Demystified Series Physics Demystified by Dr Timothy
i.e they have a non-linear graph between voltage Cell 𝐸2 will have a negative emf as it is wrongly
and current e.g electrolytes, diodes, transistors. connected with its negative pole connected to the
-Ohm’s law graph is a straight line graph showing negative pole of cell 𝐸1 and its positive pole
the relationship of current 𝐼 and voltage 𝑉. connected to the positive pole of cell 𝐸3 .
V(𝑽) 𝑰(𝑨) 3. Resistors in parallel –
I.The p.d across each resistor is the same and is
equal to the total voltage/p.d, 𝑽𝟏 = 𝑽𝟐 = 𝑽𝟑 = 𝑽.
𝑰(𝑨) V(𝑽) II.The current flowing through each resistor is
- Both graphs above represents ohm’s law but different and the total current is equal to the sum
their slope is different. The slope of the first of individual currents, 𝑰 = 𝑰𝟏 + 𝑰𝟐 + 𝑰𝟑 .
graph(𝑽 − 𝑰 graph) is equal to resistance 𝑹. III.Their effective resistance is given by the
The slope of the second graph(𝑰 − 𝑽 graph) is reciprocal sum of their individual resistance,
equal to conductance 𝑮 in (Ω)-1 or siemens.. 𝟏 𝟏 𝟏 𝟏
= + + . For two resistors, 𝑹 = 𝟏 𝟐 .
𝑹 𝑹
𝑹 𝑹𝟏 𝑹𝟐 𝑹𝟑 𝑹𝟏 +𝑹𝟐
- A resistors is a wire that offers resistance to 𝑽 𝑽 𝑽
current. When current flows through a - For cells in parallel, 𝑰𝟏 = , 𝑰𝟐 = , 𝑰𝟑 = .
𝑹𝟏 𝑹𝟐 𝑹𝟑
resistor, it converts electrical energy to heat 𝟏
Alternatively 𝑰 ∝ or 𝑰 ∝ 𝑮, as 𝑉=constant. Thus,
energy. 𝑹

- A rheostat is a variable resistor.Its resistance the higher resistance (lower conductance)


can be varied. Rheostat can also be used as a will have lesser current flowing through it.
1
potential divider to produce varying p.d. Conductance 𝐺 = .
𝑅
- Current is measured by an ammeter of low 𝑰(in 𝑹) =
𝑹 𝒊𝒏 𝒐𝒕𝒉𝒆𝒓 𝒓𝒆𝒔𝒊𝒔𝒕𝒐𝒓𝒔
× 𝑰𝑻
resistance, which is connected in series with the 𝒔𝒖𝒎 𝒐𝒇 𝒓𝒆𝒔𝒊𝒔𝒕𝒂𝒏𝒄𝒆

circuit. - For resistors of resistance 𝑅1 and 𝑅2 with


- Voltage (terminal P.d and emf), is measure by a conductance 𝐺1 and 𝐺2 , the currects are :
𝑹 𝑮
voltmeter of high resistance, which is 𝑰𝟏 (in 𝑹𝟏 ) = 𝟐 × 𝑰𝑻 OR 𝟏 × 𝑰𝑻
𝑹𝟏 +𝑹𝟐 𝑮𝟏 +𝑮𝟐
connected in parallel with the circuit. 𝑹𝟏 𝑮𝟐
𝑰𝟐 (in 𝑹𝟐 ) = × 𝑰𝑻 OR × 𝑰𝑻
- Resistance is measured by an ohmmeter. 𝑹𝟏 +𝑹𝟐 𝑮𝟏 +𝑮𝟐
- Multimeter is an instrument that measures 4. Resistors in series –
current,voltage and resistance e.g Avometer. I.The p.d across each resistor is different and the
- Series and Parallel arrangements of cells net p.d is equal to the sum of their individual p.d,
and resistors in a circuit – 𝑽 = 𝑽𝟏 + 𝑽𝟐 + 𝑽𝟑 .
1. Cells in parallel – The emf the cells are equal II.The current flowing through each resistor is
and is equal to the net emf. 𝑬𝟏 = 𝑬𝟐 = 𝑬𝟑 = the same and equal to the total current.
𝑬𝑻 ……..𝑬𝒏 2.The net internal resistance 𝑟𝑇 is 𝑰𝟏 = 𝑰𝟐 = 𝑰𝟑 = 𝑰.
𝒓
given by – 𝒓𝑻 = , 𝑛=number of cells. III. Their effective resistance is equal to the
𝒏 algebraic sum of their individual resistance.
- The advantage of connecting cells in parallel
𝑹 = 𝑹𝟏 + 𝑹𝟐 + 𝑹𝟑 .
is that the effective internal resistance is
- For cells in series 𝑽𝟏 = 𝑰𝑹𝟏 , 𝑽𝟐 = 𝑰𝑹𝟐 ,𝑽𝟑 = 𝑰𝑹𝟑 .
decreased, the current supplied increases
Alternatively 𝑽 ∝ 𝑹, as 𝐼=constant. Thus, the
(produces large amount of current), supplies
higher resistance will have a higher voltage.
steady current for a long period and the 𝑹 𝒊𝒏 𝒕𝒉𝒆 𝒓𝒆𝒔𝒊𝒔𝒕𝒐𝒓
efficiency is high. 𝑽(across 𝑹) = × 𝑽𝑻 .
𝒔𝒖𝒎 𝒐𝒇 𝒓𝒆𝒔𝒊𝒔𝒕𝒂𝒏𝒄𝒆
2. Cells in series – Cells arranged in series can be - For resistor with resistance 𝑅1 , 𝑅2 and 𝑅3 ,
connected with similar poles or opposite poles. 𝑽𝟏 (across 𝑹𝟏 ) =
𝑹𝟏
× 𝑽𝑻 .
- The net internal resistance for both 𝑹𝟏 +𝑹𝟐 +𝑹𝟑
𝑹𝟐
arrangement is the algebraic sum of their 𝑽𝟐 (across 𝑹𝟐 ) = × 𝑽𝑻 .
𝑹𝟏 +𝑹𝟐 +𝑹𝟑
individual resistance. 𝒓𝑻 = 𝒓𝟏 + 𝒓𝟐 + 𝒓𝟑 . 𝑹𝟑
𝑽𝟑 (across 𝑹𝟑 ) = × 𝑽𝑻 .
Alternatively, 𝒓𝑻 = n𝒓(if the cells are the same). 𝑹𝟏 +𝑹𝟐 +𝑹𝟑
I. Cells with opposite poles connected: The net - The resistance of an open circuit (a cell no
emf is the algebraic sum of their individual e.mf. delivering current) is infinite. 𝑉 = 𝐼𝑅 , 𝐼=0,
𝑉 𝑉
𝑬𝑻 = 𝑬𝟏 + 𝑬𝟐 + 𝑬𝟑 OR 𝑬𝑻 = 𝒏𝑬(if the cells are 𝑅 = = = ∞(infinite).
𝐼 0
the same). - The emf(also called norminal voltage) of a cell
𝑬𝟏 𝑬𝟐 𝑬𝟑 Opposite poles. is the algebraic sum the p.d acting in a circuit. Emf
is equal to the sum of all the terminal potential
II. Cells with similar poles connected: Cells difference and the lost volt.
which are wrongly connected (like poles) are 𝑬 = 𝑽 + v , 𝑽 = 𝑰𝑹, v = 𝑰𝒓, 𝑬 = 𝑰𝑹 + 𝑰𝒓
given a negative emf. 𝑬𝑻 = 𝑬𝟏 − 𝑬𝟐 + 𝑬𝟑 . 𝑽 𝑰𝑹 𝑽 𝑹
𝑬 = 𝑰(𝑹 + 𝒓) , = , = .
𝑬𝟏 𝑬𝟐 𝑬𝟑 𝑬 𝑰(𝑹+𝒓) 𝑬 𝑹+𝒓
- The current-voltage (𝐼 − 𝑉) graph of a cell with

241
Demystified Series Physics Demystified by Dr Timothy
internal resistance is descending line graph with 𝑹𝟏 𝑹𝟐
a negative slope. 𝐸 = 𝑉 + v, v = 𝐸 − 𝑉,
v = −𝑉 + 𝐸 , v = 𝐼𝑟 , 𝐼𝑟 = −𝑉 + 𝐸, G
−𝟏 𝑬 𝑳𝟏 𝑳𝟐
𝑰 = ( ) 𝑽 + , Comparing with equation of a
𝒓 𝒓
straight line ; 𝑦 = 𝑚𝑥 + 𝑐. 𝑚=slope, 𝑐=intercept
𝑬
The graph of 𝑰 against 𝑽 has a negative slope 𝑹𝟏 𝑹𝟐 𝑹𝟏 𝑳𝟏
𝟏 = OR = , 𝑳𝟐 = 100 −𝑳𝟏 .
of − (negative reciprocal of internal 𝑳𝟏 𝑳𝟐 𝑹𝟐 𝑳𝟐
𝒓
resistance), and the intercept on the 𝑰-axis is - Potentiometer – It used to determine actual or
𝑬 accurate value of the emf of a cell by comparing
(ratio of emf to the internal resistance or the with the emf of a standard cell or battery. It acts
𝒓
maximum current 𝑰𝒎𝒂𝒙 ) . as a voltmeter of infinite resistance or ideal
𝑰 voltmeter. It can also measure terminal p.d and
𝑬 𝟏
Slope = − very small internal resistance.
𝒓 𝒓

𝐿2
𝑽 𝐿1
- The voltage-current (𝑉 − 𝐼) for a cell with X Y
internal resistance is also a descending line A
graph. 𝑉 = 𝐸 − 𝐼𝑟, 𝑽 = −𝑰𝒓 + 𝑬, 𝐸1
The graph of 𝑽 against 𝑰 has a negative slope B
of −𝒓 and the intercept on the 𝑽-axis is 𝑬. 𝐸2
𝑬𝟏 𝑳𝟏
𝑽 = . 𝐸1 =emf of cell 𝐴, 𝐸2 =emf of cell 𝐵,
𝑬𝟐 𝑳𝟐
𝑬 Slope = −𝒓 𝐿1 =balance point of cell 𝐴, 𝐿2 =balance point of
cell 𝐵.
- Advantages of a potentiometer over a
𝑰 voltmeter for measuring emf – 1.Potentiomter
- The efficiency of cell 𝜖 is the ratio of electric does not use a pointer unlike a voltmeter
power output to power input. 2.Potentiometer measures emf to a high degree
𝑷𝒐𝒘𝒆𝒓 𝒐𝒖𝒕𝒑𝒖𝒕 𝑷
Efficiency = × 100 = 𝒐 × 100 , of precision than the voltmeter which may have
𝑷𝒐𝒘𝒆𝒓 𝒊𝒏𝒑𝒖𝒕 𝑷𝒊
𝑰𝟐 𝑹 𝑹 a zero error. 3.Potentiometer measures emf
𝝐= ×100 = × 100. when the cell is not delivering current i.e in open
𝑰𝟐 (𝑹+𝒓) 𝑹+𝒓
Efficiency 𝝐 =
𝑻𝒆𝒓𝒎𝒊𝒏𝒂𝒍 𝒑.𝒅
× 100 = × 100.
𝑽 circuit.It gives the actual measurement of emf
𝑬.𝑴.𝑭 𝑬 since, no correction is made for lost volt (v= 𝐼𝑟).
- Wheatstone bridge and Meter bridge – Both
- Kirchhoff’s Laws – It is used to find the current
instruments are used for measuring the
or voltage of complex circuits.There are two
resistance of a material. A galvanometer is
laws; I.Kirchhoff’s current law II.Kirchhoff’s
attached to the instrument for a null deflection or
voltage law
balance point(where the potential in the circuit is
1.Kirchhoff’s first(current) law 𝑲𝑪𝑳 – It is also
the same or p.d between two points is zero and
called the junction law or nodal analysis. It
the galvanometer shows no deflection i.e no
states that the algebraic sum of currents meeting
current is flowing).
at a junction or a point is zero i.e ∑ 𝑰 = 0 or the
Wheatstone bridge :
total current entering a junction equals the total
current leaving the region , ∑ 𝑰𝒊𝒏 =∑ 𝑰𝒐𝒖𝒕 .
𝑹𝟏 𝑹𝟐
-Kirchhiff’s current law is based on the law of
G
conservation of charge.
Current 𝐼1 , 𝐼2 , 𝐼3 , 𝐼4 and 𝐼5
𝑰𝟏 meet at a junction O.
𝑹𝟑 𝑹𝟒
O 𝑰𝟓
𝑰𝟐 𝑰𝟒
𝑰𝟑
𝑬
Current 𝐼1 , 𝐼2 and 𝐼3 flow towards the junction O
𝑰𝟏 = 𝑰𝟐 (as 𝑅1 and 𝑅2 are in series),𝑰𝟑 = 𝑰𝟒 (as 𝑅3
while current 𝐼4 and 𝐼5 flow away from the
and 𝑅3 are in series). 𝑽𝟏 = 𝑽𝟑 (as 𝑅1 and 𝑅3 are
junction O. 𝑰𝟏 + 𝑰𝟐 + 𝑰𝟑 − 𝑰𝟒 − 𝑰𝟓 = 𝟎or𝑰𝟏 +
parallel), 𝑽𝟐 = 𝑽𝟒 (as 𝑅2 and 𝑅4 are parallel).
𝑹𝟏 𝑹 𝑰𝟐 + 𝑰𝟑 = 𝑰𝟒 + 𝑰𝟓 .
= 𝟑 . The effective or total resistance 𝑅𝑇 in a - Currents flowing towards the junction are
𝑹𝟐 𝑹𝟒
wheatsone bridge is : 𝑹𝑻 =
(𝑹𝟐 +𝑹𝟐 )(𝑹𝟑 +𝑹𝟒 )
. positive while currents flowing away from the
𝑹𝟏 +𝑹𝟐 +𝑹𝟑 +𝑹𝟒 junction are negtative.
Meter bridge : 2.Kirchhoff’s second(voltage) law 𝑲𝑽𝑳 – It is

242
Demystified Series Physics Demystified by Dr Timothy
also called loop analysis.It states that the body either to change it temperature (𝒎𝒄∆θ)or
algebraic sum of all voltages or potential to change its state (𝒎𝑳).
difference in a closed circuit or loopis zero. i.e 𝑯 = 𝑰𝟐 𝑹𝒕 = 𝒎𝒄∆𝜽 or 𝒎𝑳.
∑ 𝑽 = 𝟎or the algebraic sum of voltage across - In an experiment to show the heating effect of
drops in any closed loop is equal to the sum of electric current, the relationship between the
emf’s in that loop , ∑ 𝑰𝑹 = ∑ 𝑬. temperature rise ∆𝜽and the current drawn is a
- This law is based on the law of conservation straight line graph with a slope equal to
𝑹𝒕
.
of energy or conservation of electrostatic 𝑹𝒕
𝒎𝒄

force. 𝑰𝟐 𝑹𝒕 = 𝒎𝒄∆𝜽, ∆𝜽 = 𝑰𝟐 , Thus, ∆𝜽 ∝ 𝑰𝟐


𝒎𝒄
- In 𝐾𝑉𝐿,the sign assigned to the voltage (𝐸 and ∆𝜽
𝑹𝒕
𝐼𝑅), is dependent on the direction (clockwise or Slope = .
𝒎𝒄
anticlockwise) as indicated in the circuit.
- Generally, when travelling a loop, if higher
𝑰𝟐
potential point(positive pole) is entered the
- The relationship between the current flowing
voltage takes +ve sign and if a lower potential
through a heating element and the time of flow is
point(negative pole) is entered the voltage takes 𝑯 𝑯 𝟏
a −ve sign. – 𝑯 = 𝑰𝟐 𝑹𝒕 , = 𝑰2𝒕, = constant, ∴ 𝑰𝟐 ∝ ,
𝑹 𝑹 𝒕
- Emf is positive when the direction moves from 𝑰𝟐 𝟏 𝒕𝟏 = 𝑰𝟐 𝟐 𝒕𝟐 .
−ve terminal to the +ve terminal of the cell and - House wiring – The house and appliances are
negative when the direction moves from +ve to protected from sparks,shocks and possible fires
−ve. that might arise from a faulty equipment by
- Voltage across drop(𝐼𝑅) is −ve if the direction colour coding, use of wires of different
travels through the resistor in the same direction thickness,contact breakers, fuses and
as current and +ve if the direction travels in arrangement of lamps.
opposite direction with the current. - Colour coding – The live wire is either red or
𝑰 𝑹𝟏 𝑬𝟏 𝑰 brown, neutral wire is either blue or black
while the earth wire is either green or yellow.
𝑹𝟒 𝑹𝟐 - The earth wire is connected to the casing of the
appliance for safety precautions. When short
𝑰 𝑰 circuiting occurs i.e the casing of the appliance is
𝑹𝟑 𝑬𝟐 in contact with the life wire, the earth wire
In the diagram above, the arrow indicates conducts the sufficient current to the earth,hence
anticlockwise loop and the current also blowing the fuse and breaking the circuit.
moving in anticlockwise direction : - Electrical current is delivered to a house using –
∴ 𝑬1−𝑬2−𝑰𝑹1−𝑰𝑹2−𝑰𝑹3−𝑰𝑹4= 𝟎, Live wire through current flows in and neutral
𝑬1−𝑬2= 𝑰𝑹1+𝑰𝑹2+𝑰𝑹3+𝑰𝑹4. wire through current returns to the power
station.
● Electrical energy, Power and Fuse - Circuit breakers – They are made with
bimetallic strip thermostat. They are used in
wire
high-current circuits and they switch off with
- Electrical energy 𝑬 – It is the workdone in
over current and manually switched on when
moving a given quantity of electricity 𝑄 through
fault is corrected.
a conductors across a p.d 𝑉.
𝑽𝟐 𝒕
- Fuse – A fuse is a thin strip of short lengh wire
𝑬 = 𝑸𝑽 = 𝑰𝑽𝒕 = 𝑰𝟐 𝑹𝒕 = = 𝑷𝒕 . made of low melting point(fusion point) material
𝑹
- The trade unit of electrical energy is kilowatt- such as an alloy of tin and lead. It is connected to
hour (𝐾𝑊h). 1𝐾𝑊h=3.6×106 joules. the live wire of an appliance.
- Electrical power 𝑷 – It the rate at which - Fuse rating is the maximum safe current
electrical energy is dissipated. permitted to flow in a fuse before it breaks. e.g if
𝑽𝟐 the fuse rating of a fuse is 13A and the appliance
𝑷 = 𝑰𝑽 = 𝑰𝟐 𝑹 = .
𝑹 draws a current higher than 13A,the fuse will
- Joule’s law of electrical heating – It states that melts as a result of overheating(i.e blowing of the
the quantity of heat energy produced in a fuse) and breaks the circuit.
metallic conductor as a result of current flowing - Fuse rating should be a bit higher or equal to
through it, is proportional to the resistance of the the normal current.
conductor 𝑹, the square of the current 𝑰𝟐 and the - Electrical lamps or bulbs in use –
time for which it flows 𝒕.𝑯 ∝ 𝑰𝟐 𝑹𝒕, 𝑯 = 𝑰𝟐 𝑹𝒕. 1.When lamps or bulbs are connected in series
1 calorie=4.3joules. the same flows through each lamp but the p.d
- Electrical calorimetry – Electrical energy is across each lamp is different and depends on
utilized to heat energy, which is transferred to a resistance. A single switch controls all the lamps

243
Demystified Series Physics Demystified by Dr Timothy
and if one of the lamps dies, it breaks the circuit Which of the above are correct? A.I and II B.I and
and the other lamps will not lit e.g at parties and III C.III and IV D.I,II,III and IV.
tiny bulbs in christmass tree. All metals e.g steel, silver, and resistors obey
2.When lamps or bulbs are connected in parallel ohm’s law. 5.
different current which depends on their Which of the following graphs correctly
resistance flows through them but the p.d across illustrates the relationship between the current
each of them is the same.They are controlled by (𝐼) and the voltage (𝑉) for cell with appreciable
different switch and malfunctio- ning of one of internal resistance?
the bulbs does not affect the others. e.g in homes A.𝐼 B.𝑰 C.𝐼 D.𝐼
or house wiring.
- Series combination of bulbs – The total power
is given by the sum of the reciprocal of their 𝑉 𝑽 𝑉 𝑉
𝟏
individual powers. = + + .
𝟏 𝟏 𝟏 The (𝐼 − 𝑉) of a cell with internal resistance is a
𝑷𝑻 𝑷𝟏 𝑷𝟐 𝑷𝟑 1
-The bulb of lesser power glows brighter than graph with a negative slope= − , the intercept
𝑟
that of higher power i.e 𝑃 = 𝐼 2 𝑅, 𝑃 ∝ 𝑅 (as on the 𝐼-axis is equal to the maximum current
𝐸
same current flows for series), hence the lamp of 𝐼𝑚𝑎𝑥 the cell can supply= and the intercept on
𝑟
higher power will offer a high resistance to 1 𝐸
the 𝑉-axis is equal to the emf 𝐸. 𝐼 = − (𝑉) + .
current and its brightness reduces. 𝑟 𝑟
- Also their power is related to their and Option A is Ohm’s law graph of metals and
𝑷 𝑷 𝑷 resistors. Option D is (𝐼 − 𝑉)graph of conduction
resistance by– 𝟏 = 𝟐 = 𝟑 .
𝑹𝟏 𝑹𝟐 𝑹𝟑 in liquids (electrolytes).
- Parallel combination of bulbs – The total 6. 𝑉 The figure avove represents the
power is given by the algebraic sum of their voltage-current(𝑉 − 𝐼) relationship
individual powers.𝑷𝑻 = 𝑷𝟏 + 𝑷𝟐 + 𝑷𝟑 . for a cell with appreciable internal
- The lamp of higher power glows brighter resistance.
𝑉2 1 0 𝐼
than that of lower power i.e 𝑃 = , 𝑃 ∝ (as
𝑅 𝑅
The intercept on the 𝑉-axis represents the
the voltage across each is the same), hence the
A.e.m.f of the cell B.external resostance of the
lamp of higher power will offer a low resistance
circuit C.lost volts of the cell D.maximum current
to current and its brightness increases.
E.potntial difference of the external resistance.
- Also their power is related to their resistance by
From the relation : 𝑉 = 𝐸 − 𝐼𝑟 = −𝐼𝑟 + 𝐸, a
– 𝑷𝟏 𝑹𝟐 = 𝑷𝟐 𝑹𝟐 = 𝑷𝟑 𝑹𝟑 .
graph of 𝑉 against 𝐼 is a straight line graph with a
- Two advantages of parallel connection over
negative slope which represents the internal
series are – 1.Less voltage is used i.e the voltage
resistance(−𝑟) and the intervept on the 𝑉-axis
across each lamps are equal. 2.Damage to any
represents the emf 𝐸 of the cell, when the current
circuit component does not affect other
is zero I,e no current is flowing in the circuit.
components.
7. The total energy required to send a unit
positive charge round a complete electrical
Examples :
circuit is the A.Kinetic energy B.Potential
1. A current of 180m𝐴 passes through a
difference C.Electromotive force D.Electrical
conductor for 5 minutes. The quantity of
energy.
electricity transported is A.54C B.0.9C
8. A radio is operated by eight dry cells each of
C.3.6×10-2 C D.6.0×10-4C .
emf 2.0𝑉 connected in series. If two of the cells
𝐼=180m𝐴=180×10-3𝐴, 𝑡=5min=5×60=300s,
are wrongly connected, the net emf of the radio is
𝑄 = 𝐼𝑡 =180×10-3×300=54C.
A.8𝑉 B.10𝑉 C.12𝑉 D.16𝑉
2. 60mC of electricity passes through a conductor
The cells which are connected wrongly are given
for 2 minutes. The current through the conductor
a negative emf. Six cells are connected properly
is A.7.2𝐴 B.120.0m𝐴 C.30.0m𝐴 D.0.5m𝑨
while two cells are connected wrongly. Emf of
𝑄=60mC=60×10-3C, 𝑡=2min=2×60=120s, 𝐼=?,
𝑄 60×10−3
cells properly connected, 𝐸𝑃 = 6×2 = 12𝑉. Emf
𝐼= = = 0.5×10-3𝐴 = 0.5m𝐴. of cells wrongly connected 𝐸𝑤 =2× −2 = −4𝑉.
𝑡 120
3. The internal resistance of an electric cell of Net emf = 𝐸𝑃 + 𝐸𝑤 = 12−4 = 8𝑉.
2.6𝑉 is able to deliver a maximum current of 4𝐴 9. The diagram below shows the supply source of
is A.2.00Ω B.0.41Ω C.0.65Ω D.0.26Ω an electrical circuit.
Maximum current 𝐼𝑚𝑎𝑥 =4𝐴, emf 𝐸=2.6𝑉, internal 𝐸1 𝐸2 𝐸3 𝐸3 𝐸3
resistance 𝑟=?,
𝐸 𝐸 2.6
𝐼𝑚𝑎𝑥 = , 𝑟 = = = 0.65Ω.
𝑟 𝐼𝑚𝑎𝑥 4
𝐸3
4. Ohm’s law states that the ratio 𝑉/𝐼 is constant Calculate the total effective emf of the source
for I.steel II.an electrolyte III.silver IV.a diode. A.𝐸1 + 𝐸2 +3𝐸3 B.𝐸1 + 𝐸2 +4𝐸3 C.𝑬𝟏 − 𝑬𝟐 +3𝑬𝟑

244
Demystified Series Physics Demystified by Dr Timothy
D.𝐸1 − 𝐸2 +4𝐸3 E.𝐸1 + 𝐸2 − 𝐸3 . points 𝑋 and 𝑌 in the circuit diagram shown
The cell 𝐸2 is given a negative emf as it is wrongly A.29.0Ω B.19.0Ω C.11.0Ω D.2.0Ω E.0.5Ω
connected i.e its negative pole is connected to the The current entering through junction 𝑋
negative pole of cell 𝐸1 and its positive pole is branches into three, one through 5Ω, one through
connected to the positive pole of the first cell 𝐸3 . 4Ω and one through 20Ω i.e they are all parallel.
The net emf of cells in parallel is equal the emf of 1 1 1
= + + =
1 4+5+1
=
10 1 1
= , 𝑅 =2Ω.
𝑅 5 4 20 20 20 𝑅 2
one of the cell i.e 𝐸3 . Total effective emf
15. Three 6Ω resistors are connected in parallel.
𝐸 = 𝐸1 − 𝐸2 + 𝐸3 + 𝐸3 + 𝐸3 = 𝐸1 − 𝐸2 + 3𝐸3 .
If a potential difference of 24𝑉 is appliued across
10. Two resistors 𝑅𝑎 and 𝑅𝑏 are connected in
the combination,the current in each resistor is
parallel. If 𝑅𝑎 < 𝑅𝑏 , the combined resitance is
A.12.0𝐴 B.4.0𝑨 C.1.5𝐴 D.0.8𝐴
A.greater than 𝑅𝑎 B.less than 𝑹𝒃 C.𝑅𝑎 + 𝑅𝑏
p.d across the combination 𝑉 =p.d across each
D.greater than 𝑅𝑎 but less than 𝑅𝑏
resistor=24𝑉(as they’re parallel), 𝑅 1=𝑅 2=𝑅 3=6Ω,
𝑅𝑎 and 𝑅𝑏 are connected in parallel, their
𝑉 = 𝐼𝑅, 𝐼 = 𝑉/𝑅 = 24/6 = 4𝐴.
combined or net resistance 𝑅 is given by :
𝑅 𝑅 𝑅
16.
𝑅 = 𝑎 𝑏 = 𝑅𝑎 ( 𝑏 ). The quantity in the
𝑅𝑎 +𝑅𝑏 𝑅𝑎 +𝑅𝑏
bracket is less than 1. Thus, 𝑅 is less than 𝑅𝑎 and
𝑅𝑏 . Generally, the effective resistance of a
number of parallel resistors is always less than 𝑉
the least individual resistance. Four cells each of emf 1.5𝑉 are connected as
11.. The total resistance measured at 𝑃𝑄in the shows in the digram. The reading on the
digram below is? voltmeter, 𝑉 is A.1.5𝑉 B.3.0𝑉 C.4.5𝑽 D.6.0𝑉
P The voltmeter reading=Net emf of the combined
1Ω 2Ω cells. The two cells in parallel have an equivalent
3Ω 2Ω emf of 1.5𝑉. The net emf of is given by the
Q algebraic sum of all the emf as the cells.
1Ω 2Ω 𝐸 =1.5+1.5+1.5 =4.5𝑉.
A.18.0Ω B.11.0Ω C.4.0Ω D.2.0Ω 17.
The three 2Ω resistors are in series, 2𝑉 2𝑉 2𝑉 2Ω
𝑅 =2+2+2 =6Ω. This 6Ω is now in parallel with
the 3Ω resistor, 𝑅 =
6×3 18
= = 2Ω. This 2Ω is An electric circuit is connected as illustrated
6+3 9 above. Determine the equivalent emf and current
now in series with the two 1Ω resistor flowing through the circuit
𝑅 = 2+1+1=4Ω. respectively,neglecting the internal resistance of
12. What is the total resistance in the below the cells A.2𝑽, 1.0𝑨 B.2𝑉, 4.0𝐴 C.6𝑉, 0.3𝐴 D.6𝑉,
diagram? A.5Ω B.15Ω C.25.0Ω D.35Ω E.45Ω 3.0𝐴 .
10 ohms The equivalent emf is equal to the emf of one cell,
10 ohms 𝑅2 10 ohms as the cells are parallel. 𝐸 = 2𝑉, Neglecting the
𝑅1 𝑅3 𝑅4 𝐸
internal resistance, 𝐸 = 𝐼𝑅, 𝐼 = , 𝑅=2Ω,
10 ohms 𝑅
2
𝑅 𝑅
The total resistance 𝑅 = 𝑅1 + 2 3 + 𝑅4 , 𝐼 = = 1.0𝐴.
𝑅2 +𝑅3 2
10×10 100 18. 27Ω 9Ω
𝑅 = 10 + + 10 = 10 + + 10 ,
10+10 20
𝑅 = 10 + 5+ 10 = 25Ω. 𝐼1 12Ω
13. 2Ω What is the resistance
of the circuit shown in 𝐼2
2Ω the figure 18𝑉
3Ω In the circuit shown,the current delivered, by the
4Ω battery isA.0.4𝐴 B.0.5𝐴 C.1.5𝐴 D.2.0𝑨
19 17
A.4Ω B.11Ω C. Ω D. Ω E.8Ω Total current delivered 𝐼 = 𝐼1 + 𝐼2 ,
5 4
Effective resistance 𝑅 =
2×2
+ 3 + 4, Effective resistance resistors in series,
4
2+2 𝑅 = 27+9 = 36Ω. 𝑉(across 36Ω) = 𝑉(across
𝑅 = + 7 = 1+7 = 8Ω. 12Ω), as they now in parallel. Current in the
4
𝑉 18
14. 5Ω 4Ω series resistors 𝐼1 = = = 0.5𝐴 .
𝑅 36
X Y 𝑉 18
Current in the parallel resistors 𝐼2 = = =
20Ω 𝑅 12
1.5𝐴. Alternatively, total resistance of the circuit,
36×12 432 𝑉 18
𝑅𝑇 = = = 9Ω. 𝐼 = = = 2𝐴.
36+12 48 𝑅𝑇 9
Calculate the effective resistance between the

245
Demystified Series Physics Demystified by Dr Timothy
19. 24𝑉 C.1.71𝑉 D.3.543𝑉.
The three resistiors, 250Ω,500Ω and 1kΩ
2Ω 1Ω (1000Ω) are in series.𝑉 ∝ 𝑅 , 𝐼 = constant or the
same for series resistors. 𝑉𝑇 =6𝑉,
2Ω 2Ω 𝑅
𝑉250Ω = 250Ω × 𝑉𝑇 =
250
×6,
𝑅𝑇 250+500+1000
250
2Ω 𝑉250Ω = × 6 = 0.857𝑉 = 0.86𝑉.
1750
In the circuit shown, the current through the 1Ω 23.The resistance of an open circuit is A.zero
resistor is A.0.4𝐴 B.0.5𝐴 C.8.0𝑨 D.9.6𝐴 B.half the normal resistanceof the circuit
The two 2Ω in both outside the cell are in series, C.double the normal resistance of the circuit
𝑅=2+2=4Ω. Also, the two 2Ω resistos inside the D.one E.Infinite.
cell are in series, 𝑅 = 2+2=4Ω. An open circuit is a circuit not delivery current i.e
These two 4Ω resistors are now in parallel to 𝑉 𝑉
𝐼=0. 𝑅 = = = ∞(infinite).
𝑅 4 4×4 𝐼 0
each other, 𝑅 = = = 2Ω or 𝑅 = = 2Ω This 24. 3.0𝐴
𝑛 2 4+4
2Ω resistor is now in series with the 1Ω resistor, 𝐴2
and the current flowing through each conductor 𝐴1
is the same (𝐼2Ω = 𝐼1Ω ) and is equal
to the total current flowing through the circuit. 6Ω 4Ω
𝑉 24
𝑉 = 𝐼𝑅, 𝐼 = = = 8.0𝐴.
𝑅 3
20. 25Ω 50Ω 100Ω
The ammeter 𝐴2 shown in the circuit diagram
read 3.0𝐴. Determine the reading of the ammeter
𝐴1 A.12𝐴 B.9𝐴 C.7𝐴 D.2𝐴
14𝑉 The ammeter 𝐴1 records the current through 6Ω
In the circuit diagram shown above, the p.d while the ammteter 𝐴2 records the current
across the 25Ω resistor is A.1𝑉 B.2𝑽 C.4𝑉 D.14𝑉 through 4Ω. 𝐼1 =?, 𝑅1 =6Ω, 𝐼2 =3.0𝐴, 𝑅2 =4Ω. The
For cells in series, the potential difference across p.d across the 6Ω and the 4Ω resistors is the same
each resistor is different but the same current as they are parallel. 𝑉1 = 𝑉2 , 𝑉 = 𝐼𝑅,
3×4
flows through each resistor. ∴ 𝐼1 𝑅1 = 𝐼2 𝑅2 , 𝐼1 × 6 = 3×4 , 𝐼1 = = 2𝐴.
6
𝑅𝑇 =25+50+100 = 175Ω.𝑉 = 𝐼𝑅, 25. 12Ω
𝑉 14
𝐼𝑇 = 𝑇 = = 0.08𝐴.𝑉25Ω = 𝐼𝑇 𝑅 ,
𝑅𝑇 175
10A 6Ω
𝑉25Ω = 0.08×25= 2𝐴. Alternatively,For cells in
𝑅
series, 𝑉 ∝ 𝑅 , 𝐼 = 𝐾, 𝑉25Ω = 25Ω × 𝑉𝑇 , 4Ω
𝑅𝑇
25 25
𝑉25Ω = × 14 = × 14 = 2𝑉.
25+50+100 175 The current in the 6Ω resistor shown in the
21. In the figure below, a voltage 𝑉 is applied
diagram above is A.1.7𝐴 B.2.7𝐴 C.3.3𝑨 D.60.0𝐴.
across the terminals 𝑃 and 𝑄. 1 1 1 1 1+2+3 6 1
4Ω 𝐼𝑇 =10𝐴, = + + = = = ,
𝑅𝑇 12 6 4 12 12 2
1Ω 𝑅𝑇 = 2Ω. 𝑉 = 𝐼𝑇 𝑅𝑇 = 10×2 =20𝑉 .
P Q P.d across 6Ω resistor=Total voltage (parallel
𝑉 20
resistors)=20𝑉, 𝐼6Ω = = = 3.3𝐴.
𝑅6Ω 6
4Ω 1
The voltage across the 1Ω is A.𝑉/8 B.𝑉/4 C.𝑽/3 Alternatively, for cells in parallel, 𝐼 ∝ or 𝐼 ∝ 𝐺 ,
𝑅
D.𝑉/2. 𝑉 =constant. Coductance 𝐺 = .
1
4×4 𝑅
Net resistance of the parallel resistors, 𝑅 = , 𝐺6Ω
1
4+4
16 𝐼6Ω = × 𝐼𝑇 = 6
1 1 1 × 10 ,
𝑅 = = 2Ω. The 2Ω resistor is now in series 𝐺12Ω +𝐺6Ω +𝐺4Ω + +
12 6 4
8 1 1
with the 1Ω resistor. Total resistance of the 𝐼6Ω = 6
× 10 = 6
= ×
1 12
× 10 = 3.3𝐴.
1+2+3 6
circuit 𝑅𝑇 = 2+1 = 3Ω. Total voltage=𝑉, total 12 12
6 6
𝑉
current flowing through the circuit 𝐼𝑇 = = .
𝑉
26. Using the diagram below, calculate the
𝑅𝑇 3
potential difference across the 20Ω resistor.
Current in the 1Ω = current in 2Ω = total current.
𝑉 {Neglect the internal resistance of the cell]
𝐼1Ω = 𝐼2Ω = 𝐼𝑇 = .
3
22. Three resistors with resistance 250Ω,500Ω
20Ω
and 1kΩ are connected in series. A 6𝑉 battery is
connected to either end of the combination
Calculate the potential difference between the
ends of the 250Ω resistor A.0.20𝑉B.0.86𝑽

246
Demystified Series Physics Demystified by Dr Timothy
A.5𝑉 B.10𝑉 C.20𝑉 D.60𝑽. When the key is closed, the two 4Ω resistance are
p.d across 5Ω 𝑉5Ω = p.d across 10Ω 𝑉10Ω , as now in parallel and gives an effective resistance
they are parallel. Current through 5Ω 𝐼1 =2𝐴, 4
of 𝑅 = =2Ω. This 2Ω resistor is now in series
2
𝑉5Ω = 𝑉10Ω , 𝐼1 𝑅1 = 𝐼2 𝑅2 , 2×5 = 𝐼2 ×10,
2×5 with the 1Ω resistor to give a total resistance of
𝐼2 = = 1𝐴. Current flowing through the 20Ω 𝑅 = 2+1 = 3Ω. The current which flows when the
10
resistor is the total current 𝐼𝑇 . 𝐼𝑇 = 𝐼1 + 𝐼2 = 15
key is closed, 𝐼2 = =5𝐴.
3
2+1 = 3𝐴. P.d across 20Ω resistor 𝑉20Ω =
Closing the key will increase the delivered by =
𝐼𝑇 𝑅20Ω = 3×20 = 60𝑉. 𝐼2 − 𝐼1 =5−3 =2𝐴.
27. 8Ω
30.

𝑉1 𝑉2 𝑉3

Four equal resistances are connected in series as
shown in the circuits diagram above 𝑉1 , 𝑉2 , 𝑉3 are
12𝑉 voltmeters. Which of the following equations is
In the circuit diagram shown above, the p.d
correct? A.𝑉1 =2𝑉2 = 𝑉3 B.𝑽𝟏 = 𝑽𝟑 = 𝑽𝟐 /2
across the 2Ω resistance is A.12𝑉 B.8𝑉 C.4𝑽 D.2𝑉
C.𝑉1 = 𝑉2 = 𝑉3 /2 D.𝑉1 = 𝑉2 = 𝑉3 .
The two 8Ω parallel resistance gives a combined The same current 𝐼 flows through each resistor,
𝑅 8
resistance of, 𝑅 = = =4Ω. as they are in series. The resistance of the
𝑛 2
The 4Ω resistor is now in series with the 2Ω resistors are equal 𝑅. 𝑉1 = 𝐼𝑅, 𝑉2 = 𝐼(𝑅 +
resistor, net resistance, 𝑅𝑇 = 4+2 = 6Ω. 𝑅) =2𝐼𝑅, 𝑉3 = 𝐼𝑅
𝑉 12
𝑉𝑇 = 12𝑉. 𝑉𝑇 = 𝐼𝑅𝑇 , 𝐼 = 𝑇 = = 2𝐴. ∴ 𝑉1 = 𝑉3 = 𝑉2 /2 or 2𝑉1 =2𝑉3 = 𝑉2 .
𝑅𝑇 6
Alternatively, for cells in series, voltage drop
𝐼2Ω = 𝐼4Ω = 𝐼 (series resistors) = 2𝐴.
across any resistor is directly proportional to the
𝑉2Ω = 𝐼𝑅2Ω =2×2 = 4𝐴. Alternatively, for cells
resistance 𝑉 ∝ 𝑅, 𝑅1 = 𝑅2 = 𝑅 hence, 𝑉1 = 𝑉3 .
in series, 𝑉 ∝ 𝑅 , 𝐼 =constant.
𝑅 2 𝑅2 =2𝑅 hence, 𝑉2 =2𝑉1 =2𝑉3 or 𝑉2 /2 = 𝑉1 = 𝑉3 .
𝑉2Ω = 2Ω × 𝑉𝑇 = × 12 = 4𝑉. 31. 𝑅1
𝑅𝑇 2+4
28. 𝑉
A 6V D 6Ω C 5𝐴
3𝐴 𝑅3
3Ω 12Ω A 2A 𝑅2
Which of the following statements are true of the
B diagram shown above? I.The current in 𝑅 1 is 8𝐴
In the circuit diagram shown above, the battery II.The current in 𝑅 2 is 2𝐴 III.Resistance 𝑅 1 is
𝐵 have negligible internal resistance. The p.d greater than 𝑅 2 IV.P.d across 𝑅 2=P.d across 𝑅 3
between 𝐷and 𝐶is A.2𝑉 B.4𝑉 C.6𝑉D.8𝑽 A.I and III B.II and III C.II and IV D.III,III and IV
Net resistance of 6Ω and 12Ω parallel resistor The current through the 𝑅3 =Total current
𝑅=
6×12 72
= =4Ω. 𝐼3Ω = 𝐼4Ω = 𝐼𝑇 =ammeter flowing through the circuit=Sum of current
6+12 18 flowing through 𝑅1 and 𝑅2 . Hence, current in
reading =2𝐴, p.d between 𝐷 and 𝐶 = 𝑉(across
𝑅1 =5−3 =2𝐴. Recall 𝐼 ∝ 𝑅, since the current in
4Ω) resistor, 𝑉4Ω = 2×4 = 8𝑉.
𝑅1 is less than that in 𝑅2 , resistance 𝑅1 is greater
- The emf 𝐸 of the battery 𝐵 equals the total
than 𝑅2 . P.d across 𝑅1 =P.d across 𝑅2 , as both
voltage in the cell, 𝐸 = 6+8 =14𝑉.
resistors are parallel. P.d across 𝑅3 ,𝑉3 =5𝑅3 .
29. 4Ω
32. 𝐴1
𝐴2
K 4Ω

𝐴3
15𝑉
In the circuit diagram shown above 𝐴1 , 𝐴2 and 𝐴3
If the key in the circuit shown above is closed, the
are ammeters. Which of the following statements
current delivered by the battery will A.increase
about the reading is/are correct? I.The reading
by 2𝑨 B.decrease by 2𝐴 C.increase by 3𝐴
of 𝐴1 is less than that of 𝐴3 II.The reading of 𝐴2 is
D.decrease by 3𝐴 .
twice of 𝐴1 III.The readings of 𝐴2 and 𝐴3 are equal
When the key is open, current flows only through
IV.The reading of 𝐴3 is the sum of 𝐴1 and 𝐴2 A.I
the 4Ω(the one below) and 1Ω resistors which
B.I and III C.I,II and III D.I,II,III and IV
are in series, 𝑅 = 4+1=5Ω
15 In a circuit the ammeter is connected in series in
The current when the key is open, 𝐼1 = =3𝐴. the circuit while the voltmeter is connected in
5

247
Demystified Series Physics Demystified by Dr Timothy
parallel. The same current flows through length of wire 𝑌,made of the same material. The
ammeter 𝐴2 and 𝐴3 hence,will have the same ratio of the resistance of 𝑋 to 𝑌 is A.1:4 B.4:1
reading.The current branches into two at the C.1:2 D.1:1
junction of the parallel resistors hence, the 𝐷𝑋 =2𝐷𝑌 , 𝐿𝑋 =2𝐿𝑌 , resistivity 𝜌=constant, as both
ammeter 𝐴1 reading decreases, due to decrease wires are made of same material. Area varies
in the current. The reading of 𝐴1 is less than that with the square of diameter, 𝐴 ∝ 𝐷2
of 𝐴1 and 𝐴3 . 𝜌=
𝑅𝐴
, 𝜌𝑋 =𝜌𝑌 ,
𝑅𝑋 𝐴𝑋
=
𝑅𝑌 𝐴𝑌 𝑅𝑋 𝐷𝑋 2
, =
𝑅𝑌 𝐷𝑌 2
33. In the circuit diagram below, the ammeter 𝐿 𝐿𝑋 𝐿𝑌 𝐿𝑋 𝐿𝑌
𝑅𝑋 𝐿𝑋 𝐷𝑌 2 2𝐿𝑌 𝐷𝑌 2 2 1 1
reads a current of 3 Amps when 𝑅 is 5 Ohms and = × = × = × = ,
𝑅𝑌 𝐿𝑌 𝐷𝑋 2 𝐿𝑌 (2𝐷𝑌 )2 1 4 2
reads 6 Amps when 𝑅 is 2 Ohms. 𝐿 𝐿
R X 𝑅𝑋 :𝑅𝑌 =1:2. Alternatively,𝑅 ∝ , 𝑅 ∝ ,
𝐴 𝐷2
2
Length and diameter are doubled, 𝑅 = ,
22
A 1
𝑅 = i.e resistance is reduced bby half or
𝐸 2
𝑅𝑋 :𝑅𝑌 =1:2.
The value of the unknown resistance 𝑋 is A.1Ω 39. A metal wire of length 1cm and of uniform
B.2Ω C.3Ω D.4Ω diameter has a resistance of 0.8Ω. What is the
𝐼=3𝐴, when 𝑅=5Ω, 𝐼=6𝐴, when 𝑅=2Ω resistance of a 50cm wire coil made of the same
Net resistance 𝑅𝑇 = 𝑅 + 𝑋, 𝑉= 𝐸= constant as material but having twice its diameter? A.250Ω
the same cell(battery) is used. B.125Ω C.40Ω D.10Ω
𝑉 = 𝐸 = 𝐼𝑅, 𝐸 = 3(5+𝑋), 𝐸 =6(2+𝑋), 𝐿1 =1cm, 𝐷1 =𝐷, 𝑅1 =0.8Ω, 𝐿2 =50cm, 𝐷2 =2𝐷, 𝑅2 =?,
3(5+𝑋) =6(2+𝑋), 5+𝑋 = 2(2+𝑋), resistivity is constant, as they are of
5+𝑋 = 4+2𝑋, 2𝑋 − 𝑋 =5−4, 𝑋 =1Ω. same material. 𝜌1 =𝜌2 =constant
𝑅𝐴 𝑅1 𝐴1 𝑅2 𝐴2 𝑅1 𝐷1 2 𝑅2 𝐷2 2
34. The resistance of a wire depends on A.the 𝜌= , 𝜌1 =𝜌2 , = , =
𝐿 𝐿1 𝐿2 𝐿1 𝐿2
length of the wire B.the diameter of the wire C.the 𝑅1 𝐿1 𝐷2 2 0.8 1 (2𝐷)2 4 2
temperature of the wire D.the resistivity of the = × , = × = = ,
𝑅2 𝐿2 𝐷1 2 𝑅2 50 𝐷2 50 25
wire E.all of the above 25×0.8
𝑅2 = =10Ω.
Resistance of a conductor depends on it length, 2
area(a measure of the diameter), temperature 40. A 4m long wire of cross-sectional area 2×10-
𝜌𝐿 8m2 has a resistance of 5Ω. The conductivity of
and nature of the material (resistivity), 𝑅 = .
𝐴 the wire is A.2.5×10-8Ω-1m-1 B.4.0×10-8Ω-1m-1
35. If the temperature of a resistance wire is C.2.5×107Ω-1m-1 D.4.0×107Ω-1m-1
gradually increased, its resistance will 𝑅=5Ω, 𝐴=2×10-8m2, 𝐿=4m, conductivity 𝜎=?
A.increase gradually B.decrease gradually 𝑅𝐴 1 𝐿 4
𝜌= , 𝜎= = , 𝜎= −8 ,
C.first increase and then decrease D.first 𝐿 𝜌 𝑅𝐴 5×2×10
decrease and then increase E.remain the same. 𝜎 = 4×107Ω-1m-1.
The resistance of a metallic conductor increases 41. 0.5m of wire 𝐴 is connected in series in a
with temperature i.e 𝑅 ∝ 𝑇. circuit to 1.0m of wire 𝐵. Wire 𝐴 and wire 𝐵 are
36. A nichrome wire is to be used in making a of the same thicknesss and have resistivities of
resistor. Which of these will give the highest 7.0×10-5Ωm and 3.5×10-5Ωm respectively.
resistance A.long and thick B.short and thick Calculate the ratio of p.d across wire 𝐴 to that
C.long and thin D.short and thin across wire 𝐵 when current flows A.1:1 B.1:2
𝑅=
𝜌𝐿 𝐿
, 𝑅 ∝ . The resistance of a material C.2:1 D.1:4 E.4:1
𝐴 𝐴 𝐿𝐴 =0.5m, 𝐿𝐵 =1.0m, 𝜌𝐴 =7.0×10-5Ωm, 𝜌𝐵 =3.5×10-
increases (i.e the current flowing decreases) 5Ωm, thickness of wire 𝐴 and 𝐵 is equal i.e
when the length of the wire increases and the constant area, 𝐴𝐴 =𝐴𝐵 . For cells in series, the
area of the wire decreases. Hence, a long same current flows, 𝑉 ∝ 𝑅,
(increased length) and thin(decreased area) has 𝑉𝐴 𝑉 𝑉 𝑅 𝜌𝐿
= 𝐵, 𝐴 = 𝐴 , 𝑅 = , 𝑅 = 𝜌𝐿 i.e area𝐴 is
a high resistance and current flowing through it 𝑅𝐴 𝑅𝐵 𝑉𝐵 𝑅𝐵 𝐴
is low/drops. constant,as both wires are of the same thickness,
37. Calculate the resistivity of a wire 4.00m long, 𝑉𝐴 𝜌𝐴 𝐿𝐴 7.0×10−5 ×0.5 𝑉𝐴 1
= = , = , 𝑉𝐴 :𝑉𝐵 = 1:2.
which has a cross-section of 0.500mm2 and a 𝑉𝐵 𝜌𝐵 𝑉𝐵 3.5×10−5 ×1 𝑉𝐵 2

resistance of 3.50Ω A.1.75×10-7Ωm B.2.80×10- 42. An electric cell of internal resistance 0.5Ω
7Ωm C.4.38×10-7Ωm D.4.38×10-6Ωm D.4.38×10- delivers a current of 2.0𝐴 when a resistance of 3Ω
5Ωm is connected across it. The electromotive force of
𝑅=3.5Ω, 𝐿=4m, 𝐴=0.5mm2=0.5×(10-3)2 = 0.5×10- the cell is A.1.0𝑉 B.1.5𝑉 C.2.5𝑉D.7.0𝑽
6m2, 𝜌=? 𝑅 =
𝜌𝐿 𝑅𝐴 𝐼=2𝐴, 𝑅=3Ω, 𝑟=0.5Ω, 𝐸=?, 𝐸 = 𝐼(𝑅 + 𝑟) ,
, 𝜌= ,
𝐴 𝐿 𝐸 = 2(3+0.5) = 2(3.5) = 7.0𝑉.
3.5×0.5×10−6
𝜌= = 4.38×10-7Ωm. 43. The potential difference between the
4
38. Wire 𝑋 has twice the diameter and twice the terminals of a battery is 2.8𝑉. With a 5Ω resistor
connected across the terminals of the cell, the p.d
248
Demystified Series Physics Demystified by Dr Timothy
is 2.5𝑉.The internal resistance of the cell A.0.30Ω 𝐼=
𝐸
=
1.1
=
1.1
= 0.3𝐴.
𝑅+𝑟 3+0.67 3.67
B.0.50Ω C.0.60Ω D.5.00Ω
49. A cell of emf 1.2𝑉 and internal resistance 1.5Ω
P.d when the cell is open or not delivery current
is connected in series with an ammeter of
or not connected to a resistor = emf 𝐸 = 2.8𝑉, P.d
resistance 0.5Ω and a load of resistance 4Ω. The
when the cell is closed or delivering current or
current in the circuit is A.0.20𝑨 B.0.24𝐴 C.0.30𝐴
connected to a resistor = terminal p.d 𝑉 = 2.5𝑉,
D.0.40𝐴
external resistance 𝑅=5Ω, internal resistance
𝑉 2.5 𝐸=1.2𝑉, 𝑟=1.5Ω, total resistance is the sume of
𝑟=?, current delivered by the cell, 𝐼 = = , the resistamce of the load and that of the
𝑅 5
𝐼 = 0.5A. 𝐸 = 𝐼(𝑅 + 𝑟), 2.8=0.5(5+𝑟), ammteter. 𝑅𝑇 = 𝑅𝑙𝑜𝑎𝑑 + 𝑅𝑎𝑚𝑚𝑒𝑡𝑒𝑟 ,
2.8 𝐸
= 5+𝑟 , 5.6 =5+𝑟, 𝑟 =5.6−5 =0.6Ω. 𝑅𝑇 = 4+0.5 = 4.5Ω , 𝐼=?, 𝐼 = ,
0.5 𝑅+𝑟
44. A cell of internal resistance 2Ω and emf 12𝑉 𝐼=
1.2
=
1.2
= 0.2𝐴.
4.5+1.5 6
is connected to a resistor of 4Ω. The terminal p.d
50. The voltage across the terminals of a cell
of the cells is A.12𝑉 B.10𝑉C.8𝑽D.6𝑉 drops to three-quarters of its norminal value
𝑉 𝑅
𝐸=12𝑉, 𝑟=2Ω, 𝑅=4Ω, 𝑉=?, = , when a resistance 𝑅 is connected across the cell.
𝐸 𝑅+𝑟
𝑉
=
4
,
𝑉 4
= , 𝑉=
4×12
= 8𝑉. The cells internal resistance is A.𝑅/4 B.𝑅/3
12 4+2 12 6 6
C.𝑹/2 D.𝑅
45. A cell delivers a current of 0.2𝐴 through a
Norminal voltage of the cell is same as emf of
resistance of 10Ω. When the resistance is
the cell.
changed to 6Ω the current delivered is 0.3𝐴. The 3 4 4
internal resistance of the cell is A.2.0Ω B.1.5Ω 𝑉 = 𝐸, 𝐸 = 𝐸, 𝐸 = 𝐼𝑅 [𝑉 = 𝐼𝑅] ,
4 3 3
C.1.0Ω D.0.7Ω 𝐸 = 𝑉 +v = 𝐼𝑅 + 𝐼𝑟, 𝐼𝑟 = 𝐸 − 𝐼𝑅,
4 4𝐼𝑅−3𝐼𝑅 𝐼𝑅
𝐸 = 𝐼(𝑅 + 𝑟), 𝐸−𝐼𝑅 𝐼𝑅−𝐼𝑅 𝐼𝑅 1
When 𝑅=10Ω, 𝐼=0.2𝐴, 𝐸 = 0.2(10+𝑟) 𝑟= =3 = 3
= 3
= × ,
𝐼 𝐼 𝐼 𝐼 3 𝐼
𝑅
When 𝑅=6Ω, 𝐼=0.3𝐴, 𝐸 = 0.3(6+𝑟) 𝑟= .
3
The cells are the same hence, 𝐸 = constant. 51. A resistance 𝑅 is connected across the
0.2(10+𝑟) =0.3(6+𝑟), 2+0.2𝑟 = 1.8+0.3𝑟, terminal of an electric cell of internal resistance
0.3𝑟 − 0.2𝑟 = 2−1.8, 0.1𝑟 = 0.2, 𝑟 = 2Ω. 2Ω and the voltage was reduced to 3/5 of its
46. A battery delivering a current of 1.0𝐴 has a nominal value. The value of 𝑅 is A.6Ω B.3Ω C.2Ω
terminal potential difference of 2𝑉. When D.1Ω
delivering a current of 1.5𝐴, the terminal p.d is 3 5 5
𝑟=2Ω, 𝑉 = 𝐸, 𝐸 = 𝑉 = 𝐼𝑅 [𝑉 = 𝐼𝑅] ,
1𝑉. The internal resistance of the battery is 5 3 3
A.0.5Ω B.1.0Ω C.2.0Ω D.4.0Ω 𝐸 = 𝑉 + v = 𝐼𝑅 + 𝐼𝑟 , 𝐼𝑟 = 𝐸 − 𝐼𝑅,
5 5𝐼𝑅−3𝐼𝑅 2
𝐸 = 𝑉 + v, lost volt v= 𝐼𝑟, 𝐸 = 𝑉 + 𝐼𝑟, 𝐼𝑟 = 𝐼𝑅 − 𝐼𝑅 = = 𝐼𝑅 ,
3 3 3
When 𝐼=1.0𝐴, 𝑉=2𝑉, 𝐸 = 2+𝑟, 𝑅=
3𝐼𝑟
= 3𝑟 = 3×2 = 6Ω.
When 𝐼=1.5𝐴, 𝑉=1𝑉, 𝐸 = 1+1.5𝑟, 2𝐼
2+𝑟 = 1+1.5𝑟, 1.5𝑟 − 𝑟 = 2−1 , 0.5𝑟 = 1 , 52. An electric bell with nominal voltage 𝐸 has a
1 resistance 3Ω connected across it. If the voltage
𝑟= = 2Ω.
0.5 falls to 0.6𝐸, the internal resistance of the cell is
47. P R A.4Ω B.3Ω C.2Ω D.1Ω
Nominal voltage or emf 𝐸, 𝑅=3Ω, 𝑉=0.6𝐸, 𝐸 =
A 𝑉
= =
5𝑉 5𝐼𝑅
, 𝑟=?, 𝐸 = 𝑉 + v ,
E 0.6 3 3
5𝐼𝑅
𝐸 = 𝐼𝑅 + 𝐼𝑟 , 𝐼𝑟 = 𝐸 − 𝐼𝑅 = − 𝐼𝑅 ,
3
In the circuit diagram shown, the ammeter 𝐼𝑟 =
5𝐼𝑅−3𝐼𝑅 2
= 𝐼𝑅 , 𝑟 =
2𝐼𝑅
=
2𝑅
=
2×3
,
𝐴registers a current of 2𝐴, when𝑃is 6Ω and 4𝐴 3 3 3𝐼 3 3
𝑟 = 2Ω.
when 𝑃 is 2Ω. The value of resistance 𝑅 is A.2Ω
53. 8Ω
B.3Ω C.4Ω D.6Ω
When 𝑃=6Ω, 𝐼=2𝐴, 𝐸 = 2(6+𝑅),
When 𝑃=2Ω, 𝐼=4𝐴, 𝐸 = 4(2+𝑅),
A 100mA
2(6+𝑅) = 4(2+𝑅) , 6+𝑅 = 2(2+𝑅),
𝑃 𝑄
6+𝑅 = 4+2𝑅, 2𝑅 − 𝑅 = 6−4, 𝑅 = 2Ω.
3𝑉,𝑟 4𝑉,0.5Ω
48. Three cells each of emf 1.1𝑉 and internal
In the circuit shown above, cell 𝑃 has an emf of
resistance 2Ω are connected in parallel across a
3𝑉 and an unknown internal resistance 𝑟 while
3Ω resistor. The current in the resistor is
cell 𝑄 has an emf of 4𝑉 and an internal resistance
A.0.12𝐴B.0.30𝑨 C.0.39𝐴 D.0.90𝐴
of 0.5Ω. If the ammeter registers a current
The three cells in parallel have equal emf,
100m𝐴, then 𝑟 is A.0.5Ω B.1.0Ω C.1.5Ω D.2.0Ω
𝐸𝑇 = 𝐸1 = 𝐸2 = 𝐸3 = 1.1𝑉, net internal
𝑟 2 𝐸𝑃 =3𝑉, 𝑟𝑃 =𝑟, 𝐸𝑄 =4𝑉,
resistance 𝑟𝑇 = , 𝑟𝑡 = = 0.67Ω, 𝑅=3Ω, 𝐼=?
𝑛 3 𝑟𝑄 =0.5Ω,𝐼=100m𝐴=0.1𝐴𝑅=8Ω. The cells are

249
Demystified Series Physics Demystified by Dr Timothy
connected wrongly i.e the positive pole of cell 𝑃 potential difference between 𝑃 and 𝑄 = 0.0𝑉 .
is connected to the positive pole of cell 𝑄. 59. 𝑃
Net emf , 𝐸𝑇 = 𝐸𝑄 − 𝐸𝑃 =4−3 =1𝑉, 45Ω 30Ω
𝑟𝑇 = 𝑟𝑃 + 𝑟𝑄 = 𝑟 +0.5, 𝐸 = 𝐼(𝑅 + 𝑟), G
1
1 = 0.1[8+(0.5+𝑟)] , =8+0.5+𝑟,
0.1 15Ω R
10 =8.5+𝑟, 𝑟 = 10−8.5 =1.5Ω. 𝑄
54. 𝐸𝑆 , 𝑟𝑆 12V
+ − Calculate the resistance of 𝑅 required to balance
the bridge in the circuit above A.5Ω B.10Ω C.15Ω
𝐸, 𝑟 D.22.5Ω
The circuit shows an accumulator of emf 𝐸 and 𝑅1 =45Ω, 𝑅2 =30Ω, 𝑅3 =15Ω, 𝑅4 =𝑅,
internal resistance 𝑟 being charged by a d.c 𝑅1 𝑅 45 30
supply of emf 𝐸𝑆 and internal resistance 𝑟𝑆 . The Wheatstone bridge : = 3 , = ,
𝑅2 𝑅4 15 𝑅
30 30
charging current measured by the ammeter 𝐴 is 3= , 𝑅 = = 10Ω.
𝐸 +𝐸 𝐸 −𝐸 𝑬 −𝑬 𝐸−𝐸 𝑅 3
given by A. 𝑆 B. 𝑆 C. 𝑺 D. 𝑆 60. In a meter bridge experiment, two resistors
𝑟𝑆 +𝑟 𝑟𝑆 −𝑟 𝒓𝑺 +𝒓 𝑟𝑆 +𝑟
In charging an accumulator, the d.c supply source 2Ω and 3Ω occupy the left and right gaps
and the accumulator are connected in series,with respectively. Find the balance point from the left
similar poles connected as shown above and the side of the bridge A.20cm B.40cm C.60cm
emf of the d.c supply must be greater that of the D.80cm
accumulator inorder to charge it. The net emf 2Ω 3Ω
𝐸𝑇 = 𝐸𝑆 − 𝐸, net internal resistance 𝑟𝑇 = 𝑟𝑆 + 𝑟,
𝐸 𝐸 −𝐸
𝐸𝑇 = 𝐼𝑟𝑇 , 𝐼 = 𝑇 = 𝑆 . G
𝑟𝑇 𝑟𝑆 +𝑟 X 100−X
55. The internal resistance of a cell supplying
current to a 4Ω resistor is 1.5Ω. Calculate the 0 100
efficiency of the cell A.70.5% B.80.2% C.75.5% 𝑋 is the balance point from the left side or zero
D.72.7% . end of the bridge, 𝑅1 =2Ω, 𝐿1 =𝑋, 𝑅2 =3Ω
𝑅=4Ω, 𝑟=1.5Ω, Efficiency 𝜖=?,, 𝐿2 =100−𝐿1 =100−𝑋 ,
𝑃𝑜𝑤𝑒𝑟 𝑜𝑢𝑡𝑝𝑢𝑡 𝐼𝑉 𝐼2𝑅 𝑅 𝑅 𝑅 𝑅 𝐿
𝜖= = = = , Meter bridge : 𝑅 ∝ 𝐿, 1 = 2 , 1 = 1 ,
𝑃𝑜𝑤𝑒𝑟 𝑖𝑛𝑝𝑢𝑡 𝐼𝐸 𝐼 2 (𝑅+𝑟) 𝑅+𝑟 𝐿1 𝐿2 𝑅2 𝐿2
4 4 2 𝑋
𝜖= × 100 = × 100 = 72.7%. = , 2(100−𝑋) = 3𝑋 , 200−2𝑋 = 3𝑋,
4+1.5 5.5 3 100−𝑋
56. A cell of internal resistance 𝑟 supplies current 200 = 3𝑋 + 2𝑋 = 5𝑋, 𝑋 =
200
= 40cm.
to a 6.00Ω resistor and its efficiency is 75%. Find 5
61. 4Ω X
the value of 𝑟 A.8.0Ω B.4.5Ω C.2.0Ω D.1.0Ω
𝑅
𝑅=6.00Ω, 𝜖=75%, 𝑟=?, 𝜖 = × 100, G
6
𝑅+𝑟
6×100 X
75 = × 100, 6+𝑟 = = 8, 40cm
6+𝑟 75
𝑟 = 8−6 = 2.0Ω. A B
In the diagram above, the galvanometer indicates
a null-deflection. Use the information to answer In the diagram above, a balance point is obtained
question 57 and 58. at 40cm from left end. The value of 𝑋 is A.2Ω B.3Ω
𝑃 C.6Ω D.12Ω
4Ω 2Ω Balance point from the left end 𝐿1 =40cm, Balance
G point from the left hand 𝐿2 =100−𝐿1 ,
𝑋 𝑅
𝐿2 =100−40=60cm, 𝑅1 =4Ω, 𝑅2 = (𝑅𝑇 = , i.e
2 𝑛
3Ω R parallel resistors).
𝑅1
=
𝑅2
,
4
=
𝑋/2
,
𝑄 𝐿1 𝐿2 40 60
𝑋 4×60
= = 6, 𝑋 = 6×2 = 12Ω.
2 40
𝐸 =2𝑉 62. A calibrated potentiometer can be used to
57. The value of 𝑅 is A.1.50Ω B.2.66Ω C.3.99Ω measure an unknown emf 𝐸 of a cell only when
D.4.50Ω E.6.00Ω I.the source of the emf is greater than 𝐸 II.the
𝑅1 =4Ω, 𝑅2 =2Ω, 𝑅3 =3Ω, 𝑅4 =𝑅, Wheatstone bridge cell’s internal resistance is negligible III.no
𝑅 𝑅 4 2 3×2
: 1= 3 , = , 𝑅= = 1.50Ω. current flows through the cell IV.the
𝑅2 𝑅4 3 𝑅 4
potentiometer wire has a uniform diameter.
58. The potential difference btween 𝑃 and 𝑄 is
Which of the above statements are correct? A.I
A.0.0𝑽 B.1.5𝑉 C.2.0𝑉 D.3.0𝑉 E.4.0𝑉
and II B.III and IV C.II,III and IV D.I,III and IV
The galvanometer indicates a null deflection, as
Potentiometer is used in measuring the emf of a
no current is flowing along 𝑃𝑄, because point 𝑃
cell at null point i.e when no current is flowing
and 𝑄 are at the same potential. Thus, the

250
Demystified Series Physics Demystified by Dr Timothy
through the cell. The internal resistance is not 𝐼=5𝐴, 𝑉=6𝑉, 𝑡=200s, 𝐻=electrical heat energy
considered, as no current is flowing through the 𝐻 = 𝐼𝑉𝑡 = 𝑃𝑡 = 𝐼 2𝑅𝑡 = 𝑉2𝑡/𝑅
cell. The potentiometer must have a uniform 𝐻 = 𝐼𝑉𝑡 = 5×6×200=6000𝐽 or
6000
=1428.6
4.2
diameter, for p.d to be proportional to the length
calorie or 1428.6cal or 1.428kcal.
of te wire (𝑉 ∝ 𝐿).
67. A standy generator is connected to fifteen
63. A standard cell of emf 1.6𝑉 gives a balance
40𝑊 lamps and a muscician’s 600𝑊 amplifying
point of 40cm on a potentiometer wire. If the
system. How much energy is used if the generator
standard cell is now replaced by a cell of emf
runs for 6hours? A.3.84k𝑊h B.7.20k𝑾h
2.4𝑉, the new balance point is at A.26.7cm
C.8.40k𝑊h D.12.00k𝑊h E.14.56k𝑊h
B.50.0cm C.60.0cm D.64.0cm
Power of fifteen 40𝑊 lamps=15×40=600𝑊,
𝐸1 =1.6𝑉, 𝐿1 =40cm, 𝐸2 =2.4𝑉, 𝐿2 =?,
𝐸1 𝐸 1.6 2.4 40×2.4 Power of musician’s amplifiying system=600𝑊,
= 2, = , 𝐿2 = = 60cm. Total power=600+600=1200𝑊=1.2k𝑊. 𝑡=5hrs
𝐿1 𝐿2 40 𝐿2 1.6
64. 4V 4Ω Energy, 𝐸= 𝑃 × 𝑡 = 1.2×5=7.2k𝑊h.
68. The heat produced in a conductor carrying an
electric current is A.iversely proportional to the
60cm D current and the resistance B.directly
A B proportional to the current,resistance and the
V time C.inversely proportional to the square of the
In the circuit diagram shown above, 𝐴𝐵 is a resistance,current and time D.directly
uniform wire of length 1m and resistance 6Ω. proportional to the square of the current, the
Neglecting the internal resistance of the cell, the resistance and the time
reading on the voltmeter 𝑉 is A.1.44𝑽 B.1.89𝑉 Joule’s law of electrical heating, 𝐻 ∝ 𝐼 2 𝑅𝑡 or 𝐻 ∝
C.4.00𝑉 D.6.00𝑉 𝐼 2 , 𝐻 ∝ 𝑅, 𝐻 ∝ 𝑡.
The net resistance in the circuit= Resistance of 69. It takes 4 minutes to boil a quantity of water
the resistor +Resistance of the wire 𝐴𝐵,𝑅𝑇 = electrically. How long will it take to boil same
4+6=10Ω. Emf of the cell, 𝐸= 𝑉𝑇 = 4𝑉, Emf of the quantity of water using the same heating coil but
wire 𝐴B or voltage across wire 𝐴𝐵=?, 𝑅=10Ω, with the current doubled? (Neglect any external
𝑅 AB=6Ω, 𝑉 ∝ 𝑅, heat lossed) A.64minutes B.32minutes
𝑅
𝑉𝐴𝐵 = 𝐸𝐴𝐵 = 𝐴𝐵 × 𝑉𝑇 =
6 ×4
=2.4𝑉. C.8minutes D.2 minutes E.1minute
𝑅𝑇 10 𝐼1 =𝐼, 𝑡1 =4 minutes, 𝐼2 =2𝐼, 𝑡2 =?, The relationship
Emf of the wire 𝐴𝐵 can also be gotten by: between the current flowing through a heating
𝑉 4
Total current in the circuit 𝐼 = 𝑇 = =0.4𝐴Emf element and the time of flow is – 𝐻 = 𝐼 2𝑅𝑡 ,
𝑅𝑇 10
𝐻 𝐻 1
or voltage along the wire 𝐴𝐵 𝐸𝐴𝐵 = 𝐼𝑅𝐴𝐵 , = 𝐼 2𝑡 , = constant, ∴ 𝐼 2 ∝ , 𝐼1 2 𝑡1 = 𝐼2 2 𝑡2 .
𝑅 𝑅 𝑡
𝐸𝐴𝐵 =0.4×6 =2.4𝑉. Voltmeter reading=Emf or 𝐼 2 ×4 = (2𝐼)2 × 𝑡2 4𝐼 2 = 4𝐼 2 × 𝑡2 ,
voltage across wire 𝐴𝐷𝐸𝐴𝐷 , 𝐿𝐴𝐷 =60cm, 𝐸𝐴𝐵 =2.4𝑉, 4𝐼 2
𝐸𝐴𝐵 𝐸 2.4 𝐸 𝑡2 = 2 = 1minute.
𝐿𝐴𝐵 =1m=100cm, 𝐸 ∝ 𝐿, = 𝐴𝐷 = 𝐴𝐷, 4𝐼
𝐿𝐴𝐵 𝐿𝐴𝐷 100 60 70. An electric iron is rated at 1000watts,
2.4×60
𝐸𝐴𝐷 =
100
=1.44𝑉 . 250𝑉.The corresponding maximum resistance
Alternatively, Resistance of a wire is directly and accompanying current is A.62.5𝐴, 4.0Ω
proportional to its length, 𝑅 ∝ 𝐿. 𝐿𝐴𝐷 =60cm, B.16.0𝐴, 62.5Ω C.62.5𝐴, 16.0Ω D.4.0𝑨, 62.5Ω
𝑅𝐴𝐷 =?, 𝐿𝐴𝐵 =1m=100cm, 𝑅𝐴𝐵 =6Ω,
𝑅𝐴𝐵 𝑅
= 𝐴𝐷 , E.4.0𝐴, 250Ω
𝐿𝐴𝐵 𝐿𝐴𝐷 𝑃=1000𝑊, 𝑉=250𝑉, 𝑅=?, 𝐼=?,
6 𝑅 6×60
= 𝐴𝐷, 𝑅𝐴𝐷 = =3.6Ω. 𝑃 = 𝐼𝑉 = 𝐼 2 𝑅 =
𝑉2
, 𝑃 = 𝐼𝑉, 1000= 𝐼 ×250, 𝐼 =
100 60 100
𝑅
𝑅𝐴𝐵 =6Ω, 𝑅(of the resistor)=4Ω, 𝑅𝑇 =6+4=10Ω (all 1000 𝑉2 2502
are in series). The cells are in series hence, =4𝐴. 𝑃 = , 1000 = ,
250 𝑅 𝑅
voltage varies directly with the resistance. 𝑉 ∝ 𝑅. 𝑅=
62500
= 62.5Ω.
𝑅 3.6 1000
𝑉𝐴𝐷 = 𝐴𝐷 × 𝑉𝑇 = × 4 1.44𝑉. Alternatively, 𝑃 = 𝐼 2 𝑅, 1000 =42 × 𝑅,
𝑅𝑇 10
1000
65. A current of 0.5𝐴 flows through a resistor 𝑅= = 62.5Ω.
16
when connected to a 40𝑉 battery. How much 71. A 6𝑉 battery of internal resistance 0.75Ω
energy is dissipated in 2minutes? A.1200𝐽 delivers current through an external load of
B.1500𝐽C.2400𝑱 D.96000𝐽 . 1.25Ω. What is the power supplied? A.18.0𝑾
𝐼=0.5𝐴, 𝑉=40𝑉, 𝑡=2 minutes=2×60=120s B.72.0𝑊 C.12.0𝑊 D.28.8𝑊
Electrical energy 𝐸=𝑄𝑉 = 𝐼𝑉𝑡 = 𝑉 2𝑡/𝑅 = 𝐼 2𝑅𝑡. 𝐸=6𝑉, 𝑅=1.25Ω, 𝑟=0.75Ω, Power supplied by
𝐸 = 𝐼𝑉𝑡 = 0.5×40×120 =2400𝐽. battery=Power input 𝑃𝑖 .
66. A potential difference of 6𝑉 is used to 𝐸2
produce a current of 5𝐴 for 200s through a 𝑃𝑖 = 𝐼𝐸 = 𝐼 2 (𝑅 + 𝑟) = ,
𝑅+𝑟
heating coil. The heat produced is A.4800cal 𝐸2 62 36
𝑃𝑖 = = = = 18𝑊
B.6000cal C.2400J D.240kcal E.6000J 𝑅+𝑟 1.25+0.75 2

251
Demystified Series Physics Demystified by Dr Timothy
72. P=4Ω

R=6Ω
A

250𝑉 A.2.4W B.4.0W C.6.0W D.6.7W E.15.0W


In the diagram above, three identical lamps each Resistors in parallel are of equal voltage,thus
of 100𝑊 are connected in parallel across a p.d of voltage is constant. 𝑃 = ,𝑃 ∝ , 𝑉 2 =constant,
𝑉2 1
250𝑉. Calculate the reading of the ammeter 𝐴 𝑅 𝑅

A.7.5𝐴 B.2.5𝐴 C.1.2𝑨 D.0.8𝐴 𝑃1 𝑅1 = 𝑃2 𝑅2 , 𝑅1 =4Ω, 𝑅2 =6Ω, 𝑃2 =10𝑊, 𝑃2 =?,


10×4
Total power of lamps is parallel is the algebraic 10×4= 𝑃2 ×6, 𝑃2 = ,
6
sum of their individual powers. 𝑃2 =6.7𝑊.
The lamps are identical thus, 𝑃1 =𝑃2 =𝑃3 =𝑃. 77. 𝑅=10Ω 𝑇=15Ω
Total power 𝑃𝑇 =3𝑃 =3×100 =300𝑊,
p.d across each lamp is the same as they are
parallel, 𝑉=250𝑉, Ammeter reading=Total
current flowing in the circuit 𝐼, 𝑃 = 𝐼𝑉 , In the diagram above, 𝑅 and 𝑇 are resistance 10Ω
𝐼= =
𝑃 300
=1.2𝐴. and 15Ω respectively. If the power dissipation in
𝑉 250
73. In homes, electrical appliances and lamps are 𝑅 is 40𝑊, then what is the power dissipation in
connected in parallel because A.less current 𝑇? A.60𝑾B.40𝑊 C.30𝑊 D.50𝑊
will be used B.less voltage will be used C.parallel Resistors in series are of equal current thus,
connection does not heat up the wires D.series current is constant. 𝑃 = 𝐼 2 𝑅 , 𝑃∝𝑅 ,
𝑃 𝑃
connection uses high voltage. 𝐼 2 =constant, 1 = 2 , 𝑃1 =40𝑊, 𝑅1 =10Ω, 𝑃2 =?,
𝑅1 𝑅2
The current flowing through lamps connected in 𝑅2 =15Ω, =
4
, 𝑃2 =
𝑃2
= 60𝑊.
4×15
series are the same and is equal to the total 10 15 10

current thus, a high current will be used. The 78. In the diagram below, the ratio of the electric
current flowing through lamps connected in power dissipated in the 6𝑊 and 3𝑊 resistors
parallel branches and flows thrpugh each lamp, respectively is
thus the current decreases and less current will 6Ω
be used.
74. A lamp 𝑌 is connected across a battery. If a 3Ω
second(identical) lamp 𝑍 is now connected in
series with 𝑌, A.𝑌 will become brighter B.𝒀 will
become dimmer C.the brightness of 𝑌 will 12𝑉 A.2:3 B.1:2 C.1:3 D.2:1.
remain unchanged D.the lamps will not glow due 𝑅1 =6Ω, 𝑅2 =3Ω,𝑃1 :𝑃2 =?, Resistors in parallel are
to short circuit of equal voltage thus, voltage is constant.
𝑉2 1
The brightness of a lamp is determined by the 𝑃= , 𝑃 ∝ , 𝑉 2 =constant, 𝑃1 𝑅1 = 𝑃2 𝑅2 ,
𝑅 𝑅
current flowing through it. Connecting lamp 𝑍 𝑃1
=
𝑅2 𝑃1
,
3 1
= = , 𝑃1 :𝑃2 = 1:2.
in series with lamp 𝑌 increases the total 𝑃2 𝑅1 𝑃2 6 2

resistance and the current flowing through lamp 79. 2Ω


𝑌 decreases,hence the brightness decreases and 2Ω
the lamp become dimmer. If lamp 𝑍 was 2Ω
connected parallel to lamp 𝑌, the bright of 𝑌 will
remain unchanged as the current i.e In the diagram above, if each of the resistors can
voltage/resistance of lamp 𝑌 is constant. dissipate maximum of 18𝑊 without becoming
75. A lamp is connected across a battery. A piece excessively heated, what is the maximum power
of low conductivity wire is connected in parallel the circuit can dissipate? A.27𝑊 B.18𝑊 C.9𝑊
with it. The lamp A.glow brighter B.glow dimmer D.5𝑊
C.remain unchanged D.quicly burn out. Power in each resistor 𝑃=18𝑊. The total
A material of low conductivity will have high maximum power dissipated in the circuit 𝑃𝑇 =
18×18 324
resistivity, hence high resistance.The brightness + 18 = + 18 = 9+18 = 27𝑊.
18+18 36
of lamp 𝑌 remains unchanged as the current 80. Two lamps rated 40𝑊 and 220𝑉 are
flowing through it is constant. Option B would connected in series. The totak power dissipated
have been correct, if the wire was connected in in both lamps is A.10𝑊 B.20𝑾 C.40𝑊 D.80𝑊
series with the lamp, due to reduction in the E.none of the above
current. 𝑃1 =𝑃2 =40𝑊. For two lamps connected in series,
76. In the diagram below, 𝑃 and 𝑅 are 4Ω and 6Ω 𝑃 𝑃
The total power 𝑃𝑇 = 1 2 =
40×40
,
respectively. If the power dissipated in 𝑃 is 10𝑊, 𝑃1 +𝑃2 40+40

then what is the power dissipated in 𝑅?

252
Demystified Series Physics Demystified by Dr Timothy
𝑃𝑇 =
1600
= 20𝑊. Alternatively, 𝑃𝑇 = , as their
𝑃
of operating for 24hours a lamp requiring 1𝐴 on
80 𝑛
40 a 200𝑉 line? A.24k B.55k C.40k D.26k E.32k
power rating is equal. 𝑃𝑇 = = 20𝑊. Cost per unit=5k per k𝑊h, 𝑡=24hrs, 𝐼=1𝐴,
2
81. Two resistors 5Ω and 10Ω are arranged first 𝑉=200𝑉, Power 𝑃 = 𝐼𝑉 = 1×200 = 200𝑊,
in series and later in parallel to a 24𝑉 volt source. 𝑃 = 0.2k𝑊. Cost of electricity = Power(k𝑊)
The ratio of total power dissipated in the series ×time(hrs)×cost per unit = 0.2×24×5 = 24k.
and parallel arrangements respectively A.2:9 88. A land lord has eight 40𝑊 electric light bulbs,
B.3:5 C.5:3 D.1:50 four 60𝑊 bulb and two 100𝑊 bulb in his house.
𝑅1 =5Ω, 𝑅2 =10Ω, 𝑉=24𝑉. If he has all the points on five hours daily and if
When the resistors are connected in series : NEPA charges 5k per unit, his bill for 30days is
𝑅𝑆 = 𝑅1 + 𝑅2 = 5+10 = 15Ω, A.₦5.70 B.₦7.25 C.₦3.65 D.₦8.05 E.₦4.50
𝑉2 242 Total power consumed,
Power dissipated in series 𝑃𝑆 = = ,
𝑅𝑆 15
576 𝑃 =(8×40+4×60+2×100)=320+240+200
𝑃𝑆 = = 38.4𝑊. =760𝑊 = 0.76k𝑊, 𝑡 = 5hr/day×30days=
15
When the resistors are connected in parallel : 150hrs, cost per unit=5k per unit,
𝑅 𝑅
𝑅𝑃 = 1 2 =
5×10 50
= = Ω.
10 Cost of electricity=Power(k𝑊)×time(hrs) ×cost
𝑅1 +𝑅2 5+10 15 3
per unit ,
𝑉2 242
Power dissipated in parallel 𝑃𝑃 = = , Cost of electricity=0.76 ×150×5=570k=₦5.70
𝑅𝑃 10/3
3 89. The maximum power transfer occurs in a cell
𝑃𝑃 = 576× = 172.8𝑊.
10 when the external resistance is A.the same as
𝑃𝑆 38.4 1 10 2
𝑃𝑆 :𝑃𝑃 = = = = = = 2:9 . the internal resistance of the cell B.greater
𝑃𝑃 172.8 4.5 45 9
82. A generator with power output of 12k𝑊 than the internal resistance of the cell C.less than
operating on 2k𝑉 distributes power to a the internal resistance of the cell D.twice the
workshop through cables of resistance 10Ω. internal resistance of the cell.
Calculate the power lost in the cables. A.6W Power transferred or power dissipated or
B.18𝑊 C.60𝑊 D.360𝑾 E.600𝑊 . power output 𝑷𝒐 is maximum when the
Power loss = 𝐼 2 𝑅, 𝑅=10Ω, Power output 𝑃𝑜 external resistance equals the internal
=12k𝑊=12000𝑊, 𝑉=2k𝑉= 2000𝑉, 𝑃𝑜 = 𝐼𝑉, resistance of the cell, 𝑹 = 𝒓. Power output is
𝑃 12000 maximum when power input is minimum.
𝐼= 𝑜= =6𝐴 . Power loss = 𝐼 2 𝑅, Power
𝑉 2000 90. A cell of internal resistance 𝑟 is connected to
loss =62×10 =36×10 =360𝑊. an external resistor 𝑅. The condition for
83. A 40k𝑊 electric cable is used to transmit maximum power transfer is A.𝑅 < 𝑟 B.𝑅=2𝑟
electricity through a resistor of resistance 2Ω at C.𝑹 = 𝒓D.𝑅 > 𝑟.
800𝑉. The power loss as internal energy is 91. The function of a 6𝐴 fuse included in a circuit
A.5.0×102𝑊 B.4.0×103𝑊 C.5.0×103𝑊 supplying a house hold refrigerator with power
D.4.0×10 𝑊 2
is to keep A.temperature of the refrigerator low
Power output 𝑃𝑜 =40k𝑊=40000𝑊, 𝑉=800𝑉,𝑃𝑜 = and constant B.current supplied to the
𝑃 40000
𝐼𝑉, 𝐼 = 𝑜 = = 50𝐴 , refrigerator below 6𝑨C.voltage supply is
𝑉 800
𝑅=2Ω,Power loss = 𝐼 2 𝑅 = 502×2 , constant D.current supplied to the refrigerator
Power loss = 2500×2 = 5000𝑊 = 5.0×103𝑊. constant and above 6𝐴.
84. An electrical generator has an e.m.f of 240𝑉 A fuse wire is a material of low melting point,
and an internal resistance 1Ω. If the current which melts and breaks off a circuit when flowing
supplied by a generator us 20𝐴 when the through an appliance is above its fuse rating. A
terminal voltage is 220𝑉, find the ratio of the fuse limits the total current drawn from the
power supplied to the power dissipated A.11:1 mains supply and keeps the circuit below fuse
B.1:11 C.12:11 D.11:12 rating.
𝐸=240𝑉, 𝑟=1Ω, 𝐼=20𝐴, 𝑉=220𝑉, 92. Every house supplied with electricity is
Power supplied=power input 𝑃𝑖 = 𝐼𝐸, provided with a box of fuses so that A.the
𝑃𝑖 = 20×240 = 4800𝑊. Power dissipated = consumption of electricity can be recorded
power output 𝑃𝑜 = 𝐼𝑉 = 20×220 = 4400𝑊. B.people residing in the house will not have an
𝑃
Ratio of power supplied to power dissipated, 𝑖 = electric shock if they touch a live wire C.the total
𝑃𝑜 current drawn from the mains can be limited
4800 12
= , 𝑃𝑖 :𝑃𝑜 = 12:11. Eit serves as a standby supply when there is a
4400 11
85. The electricity meters in houses measure power failure.
energy consumed in A.volt B.kilowatt-hour 93. Which of the following is an essential physical
C.ampere D.coulomb. property of the wires used for making fuses?
86. PHCN measures its electric energy in A.K𝑾h A.Low density B.High thermal conductivity
B.𝑊h C.J D.𝑊 . C.Low electrical resistivity D.Low melting point
87. If NEPA charges 5k per k𝑊h, what is the cost

253
Demystified Series Physics Demystified by Dr Timothy
Fuse wires are made of materials with low Power of two 2k𝑊 electric cookers=2×2 =
melting point or fusion point. 4k𝑊 = 4000𝑊, 𝐼 =
4000
=18.18𝐴,
220
94. The fuse in an electric device is always
Power of three 1.2k𝑊 air condition= 3×1.2 =
connected to the A.neutral side of an electric 3600
supply B.earth side of an electric supply C.live 3.6k𝑊 = 3600𝑊, 𝐼 = =16.36𝐴
220
side of an electric supply D.terminal side of an Power two 1.6k𝑊 electric irons=2×1.6 =
3200
electric supply 3.2k𝑊 = 3200𝑊, 𝐼 = =14.55𝐴
220
A fuse is connected to the live lineofan electric The currents in each of the appliances are greater
supply. than the fuse rating of the fuse i.e 13𝐴, hence they
In the wiring of houses, the fuse is connected to will all blow the fuse.
the wire coloured A.blue B.brown C.yellow 100. 12𝑉
D.yellow and green
A fuse coloured red or brown. The earth wire is Ↄ 𝐿1 3Ω
coloured yellow or green while the neutral wire 𝐿2 Ↄ
is coloured blue or black. 3𝐴 fuse
95. Electrical aplliances in homes are normally In the circuit diagram shown above, 𝐿1 and 𝐿2 are
earthed so that A.a person touching the lamps each, of resistance 1.5Ω. The 4𝐴 fuse will
appliances is safe from electric shock B.both cut when A.𝑲 is closed B.𝐾 is opened C.the 3Ω
a.c and d.c sources can be used C.the appliances resistor is doybled with 𝐾 closed D.the 3Ω is
are maintained at a higher p.d than the earth doubled with 𝐾 opened.
D.the appliances are maintained at a lower p.d When the key 𝐾 is opened, no current flows
than the earth. through 𝐿2 , Total circuit resistance= 𝐿1 + 𝑅 =
96. Which of the following is most suitable for 𝑉 12
1.5+3 =4.5Ω, 𝐼 = = =2.6𝐴. The fuse will
protecting the circuit of a 2000𝑊 electric iron 𝑅 4.5
connected to a 250𝑉 mains? A.13𝐴B.8𝑨 C.5𝐴 not melt and cut as the current (2.6𝐴) is less than
D.3𝐴 its fuse rating (4𝐴).
𝑃 When the key is closed,current flows round the
𝑃=2000𝑊, 𝑉=250𝑉, 𝐼=?, 𝑃 = 𝐼𝑉, 𝐼 = ,
2000
𝑉 circuit, 𝐿1 and 𝐿2 are parallel and of equal
𝐼= = 8𝐴. A fuse with a fuse rating equal or a resistance, their effective resistance=
250
1.5
bit above the current drawn from the main =0.75Ω, Total circuit resistance=3+0.75
2
circuit i.e 13𝐴 fuse is most suitable for protecting 𝑉 12
=3.75Ω , 𝐼 = = =3.2𝐴. The fuse will melt
the electric iron. 𝑅 3.75
97. A 3000𝑊 electric cooker is to be used on a and cut the circuit, as the current(3.2𝐴) is greater
200𝑉 mains circuit. Which of the fuses below can than its fuse rating(3𝐴).
be used safely with the cooker? A.2𝐴 B.5𝐴 101. Five accumulators of negligible resistance
C.10𝐴D.20𝑨 . each of emf 2.40𝑉, are to be charged in series
𝑃 from 60𝑉 d.c supply. If the charging current is
𝑃=3000𝑊, 𝑉=200𝑉, 𝐼=?, 𝑃 = 𝐼𝑉, 𝐼 = ,
3000
𝑉 3.0𝐴, find the value of the series resistance
𝐼= = 15𝐴 . A fuse with a fuse rating equal required A.16Ω B.24Ω C.14Ω D.12Ω .
200
or a bit above the current drawn from the main d.c supply
circuit should be used i.e 20𝐴 fuse. 𝐸𝑑.𝑐 =60𝑉𝐼 =3.0𝐴
98. Which of the following apparatus will require 𝑅
the smallest fuse rating for its protection? 𝐼 accumulators
A.60𝑊,240𝑉 B.60𝑊,40𝑉 C.40𝑊, 12𝑉 D.40𝑊, 5𝑉 𝐼 + −
For 60𝑊, 240𝑉 – 𝐼 = =
𝑃 60
= 0.25𝐴 𝐸𝑎 =5×2.40=12𝑉
𝑃
𝑉 240
60 NB: In charging an accumulator by a d.c source
For 60𝑊, 40𝑉 – 𝐼 = = = 1.5𝐴 similar poles are connected. i.e −ve to −ve or +ve
𝑉 40
For 40𝑊, 12𝑉 – 𝐼 =
𝑃
=
40
= 3.33𝐴 to +ve.
𝑉 12
𝑃 40 Emf of accumulators 𝐸𝑎 =5×2.4=12𝑉
For 40𝑊, 5𝑉 – 𝐼 = = = 8𝐴. Emf of d.c supply 𝐸𝑑.𝑐 =60𝑉, Current in the
𝑉 5
The apparatus with the smallest current will circuit𝐼 =3𝐴,resistor of resistance 𝑅 in series.
have the smallest fuse rating. , The current is flowing anticlokwisely and
99. Which of the following sets of appliance will assuming an anticlockwise direction, aplly
blow a 13𝐴 fuse in the mains when plugged into Kirchhoff/s voltage law : 𝐸𝑑.𝑐 − 𝐸𝑎 − 𝐼𝑅 =0. 𝐸𝑑.𝑐
220𝑉 sockets and switched on I.two 2k𝑊 electric is +ve, as the direction moves from −ve to +ve
cookers II.three 1.2k𝑊 air conditioners III.two terminal, 𝐸𝑎 is −ve,as the direction moves from
1.2k𝑊 electric irons A.III B.I and II C.I and III D.II +ve to −ve terminal, voltage drop (𝐼𝑅) is
and III E.I,II and III negative as current flows in in the anticlockwise
𝑃
𝑃 = 𝐼𝑉, 𝑉=220𝑉, 𝐼 = , direction assumed.
𝑉

254
Demystified Series Physics Demystified by Dr Timothy
𝐸𝑑.𝑐 − 𝐸𝑎 = 𝐼𝑅, 60−12 =3𝑅, 48 = 3𝑅,
48
𝑅 = =16Ω.
3

Jamb past questions on current electricity


and electrical power and energy :
[1978/25,27,28,1979/12,15,27,29,1980/21,46,
47,1981/13,14,18,21,45,1982/38,1983/19,23,
24,39,43,1984/10,46,47,1985/38,40,41,4244,
1986/39,40,1987/37,43,1988/39,40,43,44,
1989/35,38,40,41,42,1990/37,38,40,1991/35,
38,39,40,41,1992/36,37,38,1993/37,38,39,42,
1994/42,44,45,1995/41,44,45,1997/40,45,
1998/44,1999/29,30,42,2000/27,33,40,49,
2001/22,24,28,31,32,36,2002/22,31,32,34,36,
2003/28,30,32,2004/20,45,2005/18,21,2006/
25,2007/17,26,2009/37,40,41,2010/35,36,37,
38,2011/34,35,37,39,2012/39,2013/38,39,40,
41,2014/37,39,2015/10,29, 43]

255
Demystified Series Physics Demystified by Dr Timothy

Chapter 24 – Magnetic field,Magnetism and Electromagnetism


● Magnet – They attract and repel magnetic 2.Heating the magnet and allowing it to cool,
materials. Natural magnet is Loadstone or lying in the 𝐸-𝑊 direction.
magnetite. Artificial magnets are made from 3.Hammering or rough handling in the E−W
magnetic materials e.g iron,steel,nickel and direction.
cobalt. - Demagnitized materials are always kept in
- When a magnet is suspended freely, it always the E−W and not N−S direction, inorder to
comes to rest in the north-south direction. prevent induced magnetism from the earth’s
- Magnetic poles are regions where magnetic magnetic field which is oriented N−S
force is strongest in a magnet. North-seeking end direction.
is the north pole while south-seeking pole is the - Breaking a magnet into portions does not
south pole. destroy the magnet, but unlike poles appear on
- Magnetic length is the distance between the opposite sides on each portion.
poles of a bar magnet.
- Types of magnets –
1.Permanent or hard magnets – Magnets that
are not easily magnetized or demagnetized i.e
retain its magnetism for a long period.They are - Induced magnetism or Magnetization
made from steel and its allow which contains induction – An unmagnetized iron piece brought
nickel,cobalt and aluminium. close to the pole of a permanent magnet is
2.Temporary or soft magnets – Magnets that magnetized and is attracted to the magnet. The
are easily magnetized or demagnetized i.e does pole of the magnet induces an opposite pole on
not retain its magnetism for a long period. They the iron end near the magnet. When the magnet
are made from soft iron(pure iron). It is used in is removed, the iron piece loses its magnetism.
electromagnets. - Domain Theory of magnetism – It describes a
- Induced magnetism in iron is greater than that magnetic material as having several parallel
in steel i.e irons fillings clings more to iron end group of identical small magnets with like poles
that steel. adjacent to each other. This forms open chains
- The law of magnetism – Like poles repel while between the magnets and has a strong resultant
unlike poles attract. i.e South pole repels south magnetism. It describes a non-magnetic material
pole but attracts north pole. as having several parallel group of identical small
- Magnetization – Magnetic materials can be magnets arranged end to end or parallel with
manetized by: unlike poles adjacent to each other. This forms
1.Solenoid or electrical method – a magnetic closed chains between the magnets and has no
material is placed inside a solenoid conncted to a resultant magnetism outside it.
battery or d.c source. Clockwise direction of - Magnetic field – The region round a magnet
current gives S-pole while anticlockwise where a magnetic force is felt.
direction of current gives N-pole. - Magnetic flux or magnetic line of force –
2.Stroking method – i.Single touch ii.Divided Imaginary curved lines along which magnetic
touch. The end of the material last touched force acts. Direction of a magnetic flux at any
acquiresa pole opposite to that of the stroking point is the path an imaginary North pole will
pole. take if placed at that point and free to move. It
spreads radially outwards for north pole and
radialy inwards/towards a south pole i.e
starts from 𝑵-pole and ends at 𝑺-pole and are
closest where magnetic field is strongest.
Single touch Divided touch - Magnetic flux density or magnetic induction
3.Hammering the magnetic material in the north- 𝑩 – The magnetic flux that passes through a
south(𝑁-𝑆) direction to the earth. ∅
perpendicular area in the field. 𝑩 = . It is
- Demagnetization – Magnetic materials and 𝑨

magnets can be demagnetized by: measured in tesla(𝑻) or webers per meter


square(𝑾𝒃/𝒎2).
1.Solenoid or electrical method – a magnet or
magnetic material is placed inside a solenoid - Neutral point – It is the point where the
connected to an alternate current(a.c) source and resultant magnetic flux density is zero or point
gradually withdrawn in the east-west(E-W) where the horizontal component of the earth’s
flux density is equal and opposite to the magnetic
direction through a long distance or reducing the
current gradually to zero and placing in the E-W flux density of a magnet.It is also the point where
direction.
256
Demystified Series Physics Demystified by Dr Timothy
the earth’s magnetic field neutralizes the field the keepers oppose the demagnetization effect.
due to the magnet.
- Maximum density point – It is the point
wherethere is a greater concentration of fileds in
the same direction or the pont where the field
due to the magnet is equal and in the same
direction to the earth’s magnetic field. It is always - Chatacteristic of magnetic field lines – 1.They
opposite the neutral point. are imaginary lines that do not cross one another.
2.They begin from the north pole and end at the
south pole of a bar magnet. 3.They are lines
which an imaginary N-pole will move if placed at
that point.
Bar magnet Repulsion between like poles - Classification of magnetic materials –
1.Paramagnetic materials – These are
materials that sets its shortest axis perpendicular
or its longest axis in line or parallel to the
direction of a stroong uniform magnetic field. It
tends to move from weaker to stronger parts of a
Attraction between unlike poles non-uniform magnetic field.
2.Diamagnetic materials –These are materials
that sets its shortest axis parallel or in line or its
longer axis at right angles or perendicular to the
direction of a non-uniform magnetioc field. It
tends to move from stronger to weaker parts of a
non-uniform field.
Parallel magnets with Parallel magnets with 3.Ferromagnmetic materials – This the
similar poles,with two opposite poles,with one property of a material to aling with an external
neutral points X at the neutral point X at the
magnetic field or capable of being magnetized by
mid points(open chains, mid points (closed
strong fields). chains, weak fields).
weak magnetic fields.
- Direction of magnetic fields – Magnetic - Current carrying conductors – A current
compass needle and iron fillings can to carrying wire has a magnetic field around it.
determine the direction of magnetic field flux. Compass needle placed close to a current-
1.Iron fillings is used for less sensitive and carrying wire is deflected and deflects in
strong fields e.g poles of a magnet and current opposite direction if the current is reversed.
carrying wires. - The direction of magnetic field in a straight
2.Magnetic compass needle is used for current carrying conductor is given by:
sensitive and weak fields e.g earth’s field, 1.Right hand grip rule or Clenched fist rule –
fields between two magnets and points round It states that if a wire is griped with the right
a magnet but not the poles. hand with the tumb pointing in the current
●Application of magnetic field – direction, the curled fingers points in the
1.Magnetic shielding/screening – It involves direction of magnetic field.
the use of soft iron rings(or any other magnetic 2.Maxwell’scockscrew rule – It states that if a
material) to divert and protect an object placed right hand cockscrew turns such that its tip
within it from an unwanted external magnetic points in the current direction, the direction of
field. It is used to protect a moving coil rotation of the screw is the direction of the
galvanometer from earth’s magnetism. magnetic field.
-The direction of the field in a current carrying
wire can also be represented by clockwise
direction for into,cross,downwards or
southwards current and anticlockwise direction
for onto,dot,upwards or northwards current.
2.Magnetic keepers – The poles of a magnet NB – Clin C down south : Cl=clockwise, in=into,
tend to induce internal demagnetization field C=cross, down=downwards, south=southward.
when kept alone or due to self demagnetization The opposite hold for Northwards or upwards
Demagnetization effects is greater,when the current. Anon D up north: An=anticlockwise,
magnetic length(distance between poles) is on=onto, D=dot, up=upwards, north=northward.
shorter. Magnetic keepers are soft iron bars - Earth’s magnetic field – The earth behaves like
placed across the ends of bar magnets placed in it has a giant bar mangnet with its south pole at
pairs with adjacent unlike poles.Induced poles in the geographic north and south pole at

257
Demystified Series Physics Demystified by Dr Timothy
geophraphic north.Its magnetic field is a uniform 𝑷𝟏
field of parallel lines pointing northwards i.e
direction,field strength or density is constant at a S N
location but varies from place to place. 𝑷𝟏 𝑷𝟐
N S
The Earth’s magnetic field
𝑷𝟐
- Magnetic element of the earth – - Magnetic field of a straight current-carrying
1. Magnetic meridian – It is a vertical plane conductor and horizontal component of
about which a freely suspended magnet comes to earth’s magnetic field – Both fields interact to
rest or a vertical plane passing through the produce one neutral points and one maximum
magnetic axis of a freely suspended magnet density points depending on the current
under the influence of the earth’s magnetic field. direction, which can be determined by clenched
2. Geographic meridian – It is a vertical plane fist rule :
containing the geographic north and south poles 1.Southwards or downwards current gives
of the earth. clockwise,into,cross field with maximum density
3. Angle of declination or variation – It is the point due west and neutral point due east i.e Clin
angle between the geographic and magnetic C down south-west.
meridian or axis or angle between the direction 2.Northwards or upwards current gives
of the magnetic north and the direction of anticlockwise,onto, dot field with maximum
geographic north at a place. Compass needle density point due east and neutral point due
points to the direction of magnetic north. west. i.e Anon D up north-east.
- If a compass gives a bearing of Nθ°Eand the
angle of declination at that place is Nα°E, the true ● Magnetic effect of electric current
bearing of the compass needle=N(θ+α)°E, same - Magnetic effect of electric current – This is
direction. If a compass gives a bearing of 𝑁θ°𝐸 the production of temporary magnets or
and the angle of declination at that place is electromagnets from current. It explains how
𝑁α°𝑊, the true bearing of the compass current behaves like a magnet by having poles
needle=N(α−θ)°E, opposite direction. and obeying the fundamental law of attraction or
4. Angle of dip or inclination – It is the angle repulsion. The fields of a short coil or long
between the direction of the earth’s resultant coil(solenoid) is similar to that of a bar magnet
magnetic field and the horizontal. It is measured and its polarity is determined by point through
by a dip circle. Angle of dip is 0° at the which current enters or leaves the coil i.e The
magnetic equator and 90° at the magnetic end through which current enters acquires
pole. south pole while the end through which
- Angle of dip is used to distinguish between current leaves acquires north pole.
magnetic and non-magnetic materials by - Electromagnets – It is made when current is
suspending each at its mid-point. A freely flowing through a coil wire wound round a 𝑈-
suspended magnetic material comes to rest in the shaped soft iron bar in opposite direction. Soft
𝑁 − 𝑆 direction with its 𝑁-pole pointing iron is used in making electromagnets as the
downwards at angle of dip to the horizontal a strong magnetic field is produced and it is
while a non-magnetic material will balance easily magnetized and demagnetized. The
horizontally. magnetism remains as long as current is flowing.
Non-magnetic material Magnetic material -The strength of an electromagnet 𝐵 increases
with the number of turns in the iron bar 𝑁 and
current flowing in the coil 𝐼 and increases as the
S poles are made closer.
N - Electromagnets are used in electric bells,
5. Component of earth magnetic field – vertical telephone receiver i.e earpiece,magnetic lifting
component, 𝑽 = 𝑩 𝒔𝒊𝒏ɸ, horizontal component, devices and magnetic relays.
𝑯 = 𝑩𝒄𝒐𝒔ɸ. ɸ=angle of dip, 𝐵=magnetic flux - Telephone earpiece has a permanet magnet
𝑽
density or magnetic induction. 𝒕𝒂𝒏ɸ= . in addition to the electromagnet.
𝑯
- Bar magnet in earth’s magnetic field –Two - Force on a current-carrying conductor – The
neutral points are on the magnetic equator with force experience by a current carrying conductor
N-pole of the magnet pointing northwards while is directly proportional to the strength of the
two neutral points are on the magnetic axis with magnetic field 𝐵, current in the conductor 𝐼,
S-pole of the magnet pointing northwards. length of the conductor 𝐿 and position of the
𝑷𝟏 and 𝑷𝟐 are the neutral points conductor in respect to the field 𝜃.
- The force is maximum when the current

258
Demystified Series Physics Demystified by Dr Timothy
direction or conductor is perpendicular to the 𝝁 𝑰
𝑩𝟏 = 𝒐 𝟐.
𝟐𝝅𝒓
field and minimum or zero when the current
- Force due to current 𝐼2 , 𝑭𝟐 = 𝑩𝟐 𝑰𝟐 𝑳𝟐 .
direction or conductor is parallel or in the
- Flux density or magnetic field due to current 𝐼2 ,
direction of the field. 𝑭 = 𝑩𝑰𝑳𝒔𝒊𝒏𝜽, 𝝁 𝑰
𝑭 = 𝑩𝑰𝑳(maximum at 90°). 𝑩𝟐 = 𝒐 𝟏.
𝟐𝝅𝒓
- The direction of the force is determined by - Force on a current-carrying loop or a
Fleming’s left hand rule or motor rule – It rectangular flat coil – Two equal and opposite
states that if the thumb,forefinger and the middle forces acting on the arms of the coil constitute as
finger of the left hand are mutually perpendicular couple, hence a torque and a rotational
forefinger point in the direction of the field and movement is produced. The resultant force is
the second or middle finger pointing in the zero. The movement of the coil is towards or
currect direction,then the thumb points in the away from the 𝑁 or 𝑆-poles of a magnet and in
direction of the force or motion of the conductor. accordance with Fleming’s left hand rule.
- Based on Fleming’s left hand rule, the direction I. Maximum torque is produced with
of the force or motion of the conductor is into the perpendicular arms and parallel planes.
paper for upward current and is out of the II. No torque is produced with parallel arms
paper for downward current. and perpendicular planes.
Field Field - At the vertical position or when the arms of the
coil is parallel to the magnetic field, the net force
N S N S or toque is zero.
Upward current Downward current - The torque, 𝜏 on acting on the coil is given by –
- Force between two parallel current carrying 𝝉 = 𝑩𝑨𝑵𝑰𝒄𝒐𝒔𝜽,𝐵=magnetic flux density 𝐴=area
wires – Each wire behaves like a magnet and of the coil=𝐿𝑏, 𝐿= length of the coil, 𝑏=width of
experiences a force i.e concentric lines of force the coil/separation between forces, 𝑁=number
arounf each. They tend to move if not prevented of turns in the coil, 𝐼=current flowing through the
from moving. It is used in Ampere balance. coil, 𝜃=angle between the magnetic field and
1. Current flowing in the same direction, the plane of the coil.
force is directed towards each other i.e - If θ= 𝟎°, when plane of the coil is parallel
attraction and the field between the wires is with field and 𝝉 is maximum, 𝝉 = 𝑩𝑨𝑵𝑰 .
weaker than the field outside. A neutral point - If θ=90° when plane of coil is perpendicular
exist between the wires. to field and 𝝉 is zero.
2. Current flowing in opposite direction, the - Force on a moving charge within a magnetic
force is directed away from each other i.e field – Charges in motion experience a force
repulsion and the field between the wires is when they enert a magnetic field.This force
stronger than the field outside. No neutral point causes the charges to move in a spiral or circular
exist between the wires. path. 𝑭 = 𝒒𝒗𝑩𝒔𝒊𝒏𝜽. 𝑞=charge, 𝑣=velocity,
𝜃=angle between 𝑣 and 𝐵.
1.𝐹 is maximum when charge enters at 90° to the
field i.e 𝜃=90°. The charge moves in a circle in this
Current in same Current in opposite case.
direction direction 2.𝐹 is zero when charge enters in the direction of
- The force acting between two parallel current- the field or parallel to the field i.e 𝜃=0° or 180°.
carrying wires is proportional to the product of The charge moves in a straight line.
their current and inversely proportional to the 3.𝐹 is zero when the charge is stationary i.e 𝑣=0.
square of their distance apart.The force in one 4.𝐹 deflects charges with no change in speed or
wire is caused by the current in the other wire. kinetic energy.
𝑰 𝑰 𝑰𝟐 - The radius of the circular path through the
𝑭 ∝ 𝟏𝟐𝟐 or𝑭 ∝ 𝟐 (when equal current flows). 𝒎𝒗
𝒓 𝒓 charge moves is given by – 𝒓 =
- Force per unit length in both wires is equal 𝒒𝑩
𝑭 𝑭 𝑭 𝑟=radius of cicular path,𝑚=mass of charge.
and opposite, = constant. 𝟏 = 𝟐 .
𝑳 𝑳𝟏 𝑳𝟐 - The period or time taken for the charge 𝑞 of
𝑭 𝝁𝒐 𝑰𝟏 𝑰𝟐
Force per unit length, = . mass 𝑚 to complete one orbit 𝑻 in the circular
𝑳 𝟐𝝅𝒓 𝟐𝝅𝒓 𝟐𝝅𝒎
𝜇𝑜 =magnetic permeability or permittivity in a path is given by – 𝑻 = = .
𝒗 𝑩𝒒
vacuum=4𝜋 ×10-7henry per meter, 𝐻m-1. - Electric motor – It converts electrical energy
- Current 𝐼1 is due to magnetic field or flux density (electric current) to mechanical energy
due to 𝐼2 while current 𝐼2 is due to magnetic field (rotational energy). It consists of a permanent
or flux density due to 𝐼1 . magnet,carbon brushes,armature and a split-
- Force due to current 𝐼1 , 𝑭𝟏 = 𝑩𝟏 𝑰𝟏 𝑳𝟏 . ring commutator.
- Flux density or magnetic field due to current 𝐼1 , - As the plane of the coil passes through the

259
Demystified Series Physics Demystified by Dr Timothy
vertical position, the split-ring commutators connected in parallel to the galvanometer.
changes contact from the carbon brushes, 𝑰=
𝒊(𝑺+𝑮)
. 𝐼=larger current to be measured,
thereby reversing the direction of the current 𝑺
𝐺=resistance of galvanometer, 𝑖=current in the
and force inorder to maintain a continual
galvanometer 𝑆=resistance of the shunt.
clowkise rotation of the coil.
- Conversion of a galvanometer/
- The split-ring commutator – i.reverses the
milliammeter to a voltmeter – A high
direction of current ii.reverses the direction of
resistance multiplier is connected in series to
the forces acting on the side of the coil iii.ensures
the galvanometer. 𝑽 = 𝒊(𝑴 + 𝑮). 𝑉=voltage to
that the coil keeps rotating in the same direction
be measure, 𝑀=resistance of the multiplier.
or maintains the direction of rotation of the coil.
- Moving iron ammeter – It is used to measure
- Energy loses in electric motors – 1.Heat
both d.c and a.c, it gives the root mean square
losses by current and resistance(𝐻 = 𝐼 2 𝑅) in the
values of a.c. Its deflection is proportional to the
windings of the coil. 2. Eddy current in the soft
square of the current 𝜽 ∝ 𝑰𝟐 , hence it has a non-
iron armature.It is reduced by laminating the
linear(curved) or non-uniform scale. It is affected
iron cylinder. 3.Workdone against friction in the
by stray magnetic fields.
commutator or bearings.
- Moving coil galvanometer – It is used to detect
and measure small direct current(d.c) only and ● Electromagnetic induction –
cannot measure alternate current (a.c). It has a - Electromagnetic induction – It is the
regular or uniform scale provided by a production of electric current or emf(voltage)
radial/uniform magnetic field, not affected by from a magnet(without a cell or battery) due to
stray magnetic fields due to its stron field and can relative motion between a conductor and a
be adapted to read ranges of current and p.d, by magnet or when a conductor is placed in
fitting a suitable resistance. magnetic field of varying flux density.
- It consists of – I.Control hair spring; provides - Induced current or emf is produced is a coil
control couple for the coil and compensates for whenever there is a change in the magnetic lines
thermal expansion. II.Soft iron amature and of force i.e flux linkage linking the coil.
magnet; provides a radial magnetic field. -No currect is induced when the conductor
III.Pointer; attached to the armature to measure and the magnet are both stationary or moving
the angle of rotation of the armature which is in the same direction with the same speed.
proportional to the current, 𝜽 ∝ 𝑰. - The magnitude induced emf or current depends
- Factors increasing the sensitivity of a on number of turns of the coil, strength of the
moving coil galvanometer – magnet,relative speed between the magnet and
1.The permanent magnet is made of a special the coil,area of cross-section of the coil and
alloy to give a strong magnetic field or high presence of a soft iron core inside the coil.
magnetic flux. 𝑬 ∝ 𝝎𝑩𝑨𝑵.
2.Large number of turns and area in the coil - Total flux linkage, ɸ = 𝑩𝑨𝑵𝒄𝒐𝒔𝜽. Magnetic flux
3.Control hair spring is made weak and thinner linking a coil depends on magnetic field strength
to give a small control couple and light pointer. 𝐵, area of each turn 𝐴 and the number of turns in
-In summary sensitivity , 𝑺 ∝ 𝑩𝑨𝑵. the coil 𝑁.
- Factors increasing the sensitivity of a - Laws of electromagnetic induction –
suspended or reflecting/optical lever 1.Faraday’s law of eletcromagnetc induction
galvanometer – or Neuman’s law – It states that a changing or
1.Thinner suspension with small torsional varying magnetic flux linking a coil or circuit,
control, such that a small deflection of the coil induces an emf the magnitude of which is
provides a large movement of a pointer(light proportional to the rate of change of magnetic
beam). flux linking. i.e
𝒅∅ 𝒅∅ 𝑵(∅𝟐 −∅𝟏 ) 𝑵𝑨(𝑩𝟐 −𝑩𝟏 )
2.Large number of turns and area in the coil. 𝑬∝ , 𝑬=𝑵 = = .
𝒅𝒕 𝒅𝒕 𝒕 𝒕
3.Stronger magnetic field or magnetic flux. 𝑑∅
𝐸=induced emf, =rate of change of magnetic
- Current sensitivity of a galvanometer 𝑺𝑰 : 𝑑𝑡
𝜽 𝑩𝑨𝑵 flux(weber per second, 𝑤bs-1), 𝑁=number of
𝑺𝑰 = = , 𝜃=angle of rotation or deflection
𝑰 𝑲 turns, 𝐴=area of the coil, 𝐵=magnetic field
of pointer, 𝐼=current, 𝐵=magnetic flux density, density.
𝐴=area of the coil, 𝑁=number of turns in the coil, 2.Lenz’s law of electromagnetic induction – It
𝐾=force or stiffness constant of the spring. states that the induced emf or current opposes
- Voltage sensitivity of a galvanometer 𝑺𝑽 : the motion/change producing it. Lenz’s law is
𝜽 𝜽 𝑩𝑨𝑵
𝑺𝑽 = = = , 𝑉=voltage, 𝑅=resistance. based on conservation of energy (mechanical
𝑽 𝑰𝑹 𝑲𝑹
-Conversion of a galvanometer/milliammeter to electrical energy).
to an ammeter – A low resistance shunt is - The direction of the induced emf in a straight

260
Demystified Series Physics Demystified by Dr Timothy
conductor is determnined by Fleming’s right - Transformers – It is used to increase(step-up)
hand rule or dynamo rule – It states that if the or decrease(step-down) an alternating emf or
thumb,forefinger and middle finger of the right voltage from an alternating current mains. It
hand are mutually perpendicular, with the does not work with a dry cell i.e steady or
forefinger pointing in the direction of the field, direct voltage as direct current will not
and thumb pointing in the direction of motion, produces changes in magnetic flux.
then the middle finger will point in the direction - Transformers operates on the principle of
of induced emf or current. mutual inductance and obeys the principle of
- The magnitude of the induced emf is given by, conservation of energy. When a.c current flows
𝑬 = 𝑩𝑳𝒗𝒔𝒊𝒏𝜽 or 𝑬 = 𝑩𝑳𝒗 (maxiumum at 90°). through the primary coil 𝑃, a varying magnetic
𝐸=induced emf, 𝐿=length of the conductor, flux is set up which induces an a.c emf in the
𝑣=velocity of conductor. secondary coil. The emf on each winding is
I. 𝑬 is maximum when 𝑳 and 𝒗 are both proportional to the number of turn of the coil and
perpendicular to the field 𝑩 and 𝑬 is zero inversely proportional to the current, 𝑬 ∝ ,
𝑵
𝑰
when 𝑳 or 𝒗 is parallel to the field 𝑩. 𝑬𝑺 𝑵𝑺 𝑰𝑷
- Electromagnetic generator – They operate on = =
𝑬𝑷 𝑵𝑷 𝑰𝑺
electromagnetic induction. A generator - In a step-up transformer, number of turns in
converts mechanical energy to electrical the secondary coil is greater than that in primary
energy. It requires an external supply of coil and steps-up voltage or emf but steps-
energy to rotate the coil. D.C generators are down current, 𝑵𝑺 > 𝑵𝑷 , 𝑬𝑺 > 𝑬𝑷 , 𝑰𝑺 < 𝑰𝑷 .
called dynamos while A.C generators are called - In a step-down transformer,number of turns
alternators. in the secondary coil is less than that in primary
- An a.c generator can be converted to a d.c coil and steps-down voltage or emf but steps-
generator or electric motor by replacing the up current, 𝑵𝑺 < 𝑵𝑷 , 𝑬𝑺 < 𝑬𝑷 , 𝑰𝑺 > 𝑰𝑷 .
slip rings in a.c with split ring or commutator. - Turn ratio 𝑵𝑹 is the ratio of the number of
-An a.c generator and d.c generator both have turns in secondary coils to number of turns in
two carbon brushes,armature,field magnet in primary coil. 𝑵𝑹 = 𝑺
𝑵
. In a step-up
common, but an a.c generator hasslip rings 𝑵𝑷

while d.c generator has split rings or transformer, 𝑵𝑹 >1, while in a step-down
commutators. transformer, 𝑵𝑹 <1.
- Comparism between electric motors and - Efficiency 𝜖of a transformer is the ratio of the
generators – 1.Current is supplied by a cell to power in the secondary coil (power output) to
rotate the coil in electric motor while current is the power in the primary coil (power input).
𝑷𝒐𝒘𝒆𝒓 𝒐𝒖𝒕𝒑𝒖𝒕
produced or induced in a generator due to Efficiency = ×100,
𝑷𝒐𝒘𝒆𝒓 𝒊𝒏𝒑𝒖𝒕
rotation of the coil 2.Electric motor convert 𝝐=
𝑷𝒐
× 100 =
𝑰𝑺 𝑬𝑺
× 100.
electrical to mechanical energy while generator 𝑷𝒊 𝑰𝑷 𝑬𝑺
convert mechanical to electrical energy 3.Electric - The efficicency of a transformer is not 100%
motors operate on the force on a current carrying due to energy loses by :
conductor while generator operates on 1.Eddy current loses – Enegry loses in form of
electromagnetic induction 4.Electric motors is heat due to eddy current set up in the core by
based on Fleming’s left hand rule(motor rule) varying magnetic flux. It is reduced or
while generators is based on Fleming’s right minimized by laminating the iron core i.e iron
hand rule(dynamo rule) 5.Electric motors and core made of insulated sheets rather than a
generators are of the same structure except that solid block.
no cell in the external circuit of a generator as 2.Joule effects or Copper loses(𝑰𝟐 𝑹loses) –
current is induced or produced not supplied. Energy loses due to heat produced in the primary
- When the coil of an electric motor rotates, it and secondary coils as a result of their resistance.
functions as a d.c generator(dynamo) and an emf It is minimized by using a low resistance or
known as back emf 𝑬𝑩 is induced in the coil thick copper coils in the primary and
which opposes the emf supplied 𝑬 by the cell. secondary coils.
𝟐 3.Magnetic flux leakage or Flux design loses –
𝑬𝑩 = 𝝎𝑩𝑨𝑵.
𝝅 Energy loses due to leakage of the magnetic flux
- The energy stored in the loop of the coil, 𝑾 is between the primary and the secondary coils or
given by – 𝑾 = 𝑰(𝑬 − 𝑬𝑩 )𝒕. 𝐼=current flowing due to least flux linkage between the primary and
through the coil, 𝑡=duration of current flow. secondary coil. It is minimized by using an
- The instantaneous or effective value of induced effective core design or special form of coil
emf in a coil of 𝑁 number of turns is given by – winding.
𝑬 = 𝑬𝒐 𝒔𝒊𝒏𝝎𝒕, 𝐸𝑜 =amplitude or peak or 4.Hysteresis or lag response loses – Energy
maximum induced emf, 𝑬O= 𝝎𝑩𝑨𝑵, loses due to magnetic reversals i.e reversing
𝐸=instantaneous emf.
261
Demystified Series Physics Demystified by Dr Timothy
magnetization of core. It is minimized by the - The strength of secondary current is directly
use of soft iron cores or the use of special proportional to the rate of change of the
alloys in the core of the primary coils. 𝒅𝑰
primary current (𝑰𝑺 ∝ 𝑷 ), for circuits with
- Power transmission – Power transmission 𝒅𝒕
primary and secondary coils e.g transformers
through cables can either be high tension or low
and induction coils.
tension transmission.
- Induction coil is used in motor car ignition
1.High tension transmission(HTT) – It is the
system and operation of X-ray tubes.
transmission of alternate current over long
- Inductance – Inductance is the property by
distance at high voltage and low current, with
which a varying current in a coil induces an emf
a minimum energy loss. At the point the point
in the coil. It is the property of material to oppose
of use or consumption, the voltage is stepped
changes in current through it.
down by a transformer.
It is of two types: Self inductance and Mutual
2.Low tension transmission(LTT) – It is the
inductance.
transmission of direct current over short
1.Self inductance 𝑳– It is the production of
distance at low voltage and high current, with
induced emf in a coil due to change in the
a maximum or high energy loss.
magnetic flux in the coil by a changing or
- Power transmission is done with a.c rather than
alternate current in the coil itself.
d.c because, a.c voltage can be stepped up or 𝒅𝑰 𝑑𝐼
down by transformer, heat(𝐼 2 𝑅) loses is 𝑬 = 𝑳 , 𝐿=self inductance, =rate of change of
𝒅𝒕 𝑑𝑡
reduced,a.c is cheaper and easily produced. current, 𝐸=induced emf.
- Eddy current or Foucault current – Induced - Flux linkage per current 𝑳 = .
𝑵∅
𝑰
current in a conductor resulting from a varying 2.Mutual inductance 𝑴 – It is the production of
magnetic flux linked to it. It is also produced from
induced emf in a coil due to change in the
back emf induced in a coil mounted a metal
magnetic flux by a an alternating or changing
frame. e.g electric motor acting as a dynamo current in a neighbouring or another coil.
when rotating and current-carrying coil mounted 𝒅𝑰
on metal frame 𝑬 = 𝑴 𝑨 , 𝐸=induced emf in secondaery coils,
𝒅𝒕
𝑑𝐼𝐴
- Eddy currents cause energy losses in form of =rate of change of flux in the primary varying
𝑑𝑡
heat and it can be minimized by laminating the coil.
soft iron core. - Inductors : Inductors are the coils in which emf
- Eddy current is applied or used in – i.Pointer of is induced. They are used in electric circuits to
moving coil galvanometer ii.Brakes of large prevent changes or variation in current through
electric motors iii.Sensitive mass balances the circuit. The unit of inductance is Henry (𝑯).
iv.Speedometers of automobiles v.metal ● Arrangement of inductors –
detectors vi.Induction furnance . I.Series combination – 𝑳 = 𝑳1+ 𝑳2 + 𝑳3
- Carbon microphone – It is a device which 𝟏 𝟏 𝟏 𝟏
converts sound energy to electrical energy. It II.Parallel combination – = + + .
𝑳 𝑳𝟏 𝑳𝟐 𝑳𝟑
𝑳𝟏 𝑳𝟐
is used in a telephone mouthpiece system. It 𝑳= (for two inductors only).
𝑳𝟏 +𝑳𝟐
consists of: i.carbon blocks whose resistance
- Energy is stored in an inductor as work is done
varies with change in pressure,hence a change in
against back emf and this energy is drawn from
current. ii.a diaphragm which vibrates incoming 𝟏
sound waves. the battery. 𝑬 = 𝑳𝑰𝟐 .
𝟐
- Induction coil or Ruhmkorff coil – It is a
transformer for stepping up direct current (d.c) Examples :
i.e producing a very high intermittent emf from a 1.Which of the following about magnets is
low emf d.c source by electromagnetic induction. correct? A freely suspended magnet points in an
- It consists of – 1.Primary coil is of few turns and east west direction B.Brass is a magnetic material
thick wire while secondary coil is of many turns C.Like poles attract each other D.Permanent
and thin wire,both are wound on the same core. magnet are made of steel D.Unlike poles repel
This ensures that a large output emf is formed, each other.;
𝑬
𝑵 ∝ . 𝑁=number of turns, 𝐸=emf, th=thickness. A freely suspended magnets points in a 𝑁-𝑆
𝒕𝒉
direction. Brass is not a magnetic material. Like
2.Make-and-break device in the primary coil
poles repel while unlike poles attracts.
which gives a rapid decaying or varying magnetic
2. Which of the following is true of magnetism?
flux which induces a large voltage pulse in the
A.Iron filings cling mainly round the ends of a
secondary 3.Capacitor in the primary which
bar magnet B.The freely suspended bar magnet
prevents sparks in the make-and-break device so
comes to rest in the geographic north-south
as reduced back emf effect generated in the
direction C.like poles attract D.Loadstone is a
primary.It also promotes a more rapid decaying
non-magnetic oxide.
magnetic flux to induce a larger secondary emf.
262
Demystified Series Physics Demystified by Dr Timothy
The poles of a magnet are the points where the C.depends only on the number of magnets
magnetic effect is strongest. Freely suspended provided they are parallel D.is greater when the
magnets balances in the magnetic 𝑁-𝑆 direction magnets are arranged on a wooden surface than
but at an angle to the geographic 𝑁-𝑆. Loadstone on a metal surface.
is a natural magnet and a magnetic oxide of iron. This statement is in accordance with domain
3. Which of the following statements is not theory of magnetism. Magnets placed in parallel
correct? A.soft iron rather than steel is always with like poles adjacent to each other results in a
used in making electromagnets B.steel is more strong resultant magnetism.
difficult to magnetize than soft iron C.steel loses 9.The diagram shows a magnet 𝑋 with its south
its induced magnetism more quickly and pole moved along a soft iron bar 𝑃𝑄 in the
more easily than soft iron D.the distance direction as shown. After some time, the poles at
between the poles of a bar magnet is called the 𝑃 and 𝑄 respectively are
magnetic length E.nickel, iron and steel are X
magnetic materials.
Soft iron is easily magnetized and loses its S
induced magnetism more quickly than steel. P Q
4. A magnet is similar to a bi-polar charged A.north-north B.north-south C.south-north
system in the following respects – I.It has two D.south-south.
opposite poles- the 𝑁 and 𝑆 poles II.The earth’s The end of the iron bar 𝑃𝑄 last touched acquires
magnetic 𝑁-pole is close to the geographic 𝑆-pole opposite pole while the end first touched
and the earth’s magnetic 𝑆-pole is close to the acquires a similar pole as the stroking pole.
geopraphic 𝑁-pole III.The force of attraction or Hence, 𝑃 has N-pole as it is the end last touched
repulsion between opposite or like poles obeys a while 𝑄 has S-pole as it is the end first touched.
law identical to Coulomb’s law for electric 10. The direction of the magnetic field at a point
charges. Which of the above statements is/are in the vicinity of a bar magnet is A.always
true? A.I B.II C.I and III D.I,II and III. towards the north pole of the magnet B.always
Magnet has two opposite poles – 𝑁-pole and 𝑆- away from the south pole of the magnet C.along
pole i.e similar to a bipolar charged system of the line joining the point to the neutral point D.in
positive and negative charges. Like poles repel the direction the north pole of a compass
and unlike poles attract i.e similar to Coulombs needle would point.
law for electrostatic charges. The direction of magnetic field is towards the S-
5. The place in a magnet where the resultant pole and away form the N-pole.
attractive force appears to be concentrated are 11. Which of the following statements about
called the A.curvatures B.poles C.bars magnetic lines of force is not correct? A.a
D.lodestone. magnetic line of force is an imaginary line which
6. Which of the following methods cannot be the north pole of a magnet would describe if it is
used to convert a bar into magnet? I.Passing of an free to move B.magnetic lines of force do not
electric current through a solenoid II.Repeated cross one another C.magnetic lines of force are
stroking of the steel bar with a magnet III.Heating closely-packed together at neutral points
of the steel bar IV.Hammering the steel bar in the D.the presence of magnetic lines of force in a
earth’s magnetic field A.I and II B.III and IV C.I,II region indicates the presence of magnetic field.
and III D.I,II and IV. Magnetic lines of force are closest together
Heating of magnet or magnetic material where the magnetic field is strongest and farther
demagnetizes it. Hammering in a magnetic apart at neutral point i.e where the magnetic field
material in the earth’s magnetic field i.e in the 𝑁- is completely neutralized.
𝑆 direction, magnetizes it. 12. A magnetic field is said to exist at a point if a
7. Which of the following methods will effectively force is A.deflected at the point B.straightened at
demagnetize a bar magnet? I.Passing an electric the point C.exerted on a moving charge at a
current through the magnet II.Bringing its 𝑆-pole point D.exerted on a stationary charge at a point
in contact with the 𝑆-pole of a strong bar magnet Charges in motion has a magnetic field around it.
III.Heating the magnet A.I B.III C.I and III D.I,II 13. At different locations on earth’s surface, the
and III. earth’s magnetic field is A.different in
Statement II will increase the resultant magnitude but not in direction B.the same in
magnetism and not demagnetize it. magnitude and direction C.different in both
8. When a number of identical small magnets are magnitude and direction D.the same in
arranged in a line, the strength of the resultant magnitude but different in direction
magnet A.is largest when they are arranged end The earth’s magnetic field is a uniform field
to end B.is greatest when they are arranged which always comprises of parallel lines pointing
parallel with like poles adjacent to each other northwards at different locations. The strength

263
Demystified Series Physics Demystified by Dr Timothy
and magnitude of the field is constant in a field at the 𝑁-pole is very weak B.the magnetic
particular location but varies from one place to field at the 𝑵-pole is very strong C.the angle of
the other. declination at the north pole is zero D.the
14. The name given to the characteristic extremely low temperature affects the
behavior of a substance which set wit their length magnetism of the compass needle E.the magnetic
perpendicular to a strong magnetic field instead field at the north pole is vertical.
of in line with it is A.paramagnetism Compass needle always points in the direction of
B.diamagnetism C.ferromagnetism magnetic north. Magnetic compass is used for
D.demagnetization E.magnetization plotting the direction of weak and sensitive fields
Diamagnetic materials set its length or longer rather than strong fields at the poles of a magnet.
axis perpendicular or its shorter axis parallel to a 20. A sailor observes that his mariners’ compass
strong uniform magnetic field. Paramagnetic reads N10°W at a place where the angle of
materials sets its length or longer axis in line or variation is N15°E. Calculate the true bearing of
parallel or its shorter axis perpendicular to a the place. A.N25°W B.N5°W C.N25°E D.N5°E.
strong uniform magnetic field. 𝜃=angle of declination or variation=N15°E,
15. α=compass bearing=N10°W,
X True bearing = N(α−𝜃)°E = N(15−10)°E
Y True bearing = N5°E.
The bar magnet illustrated in the diagram above 21. At a location on earth where the declination
picks up an unmagnetized ssteel nail X. In turn, X is 15°E, a compass needle indicates the direction
picks up a similar nail Y and so on. The nails are N50°E. The true geographic bearing is A.N65°E
said to be magnetized by A.conduction B.single B.N50°E C.N35°E D.N15°E.
touch C.induction D.divided touch. 𝜃=angle of declination or variation=15°𝐸,
16. .𝑅 α=compass bearing=𝑁60°𝐸,
S True bearing =N(θ+α)°E=N(60+15)°E=N65°E.
.𝑄 22. The angle between the direction of the earth’s
N magnetic field and the horizontal is called the
.𝑃 A.angle of deviation B.angle of dip C.magnetic
Earth’s magnetic field declination D.magnetic meridian.
23. The angle of dip varies from 0° at the
A short bar magnet is placed in the earth’s equator to 90° at the poles. This can be
magnetic field as shown in the above diagram. explained by the fact that A.lines of force are
Where is/are the possible neutral positions of horizontal at the equator and vertical at the poles
the neutral point? A.𝑃 B.𝑅 C.𝑄 D.𝑷 and 𝑹. B.lines of force are vertical at the equator and
Neutral points exists at a point where the field of horizontal at the poles C.the north pole of a
the bar magnet is equal and opposite to the magnet dips downwards at the equator D.a
earth’s field(acting upwards). The field lines of magnet freely balanced at its centre of gravity
the bar magnet are directly downwards i.e will set horizontally at the poles E.the south pole
opposite to the earth’s field lines, at point 𝑃 close of a magnet dips downwards at the equator.
to N-pole and at point 𝑅 close to the S-pole. 24. Which of the following statements is not
17. The magnetic meridian at a place is the correct? A.A magnetic field is a region in which a
A.plane at right angles to the direction of the magnetic force can be detected B.A line of force is
earth’s magnetic flux B.plane containing the a line along which a magnetic north pole would
longitude of the place and the earth’s axis tend to move if it wre free C.Magnetic fields are
C.horizontal plane containing the latitude of the scalar quantities D.Neutral points are obtained
place D.direction in which a dip needle set when where the earth’s magnetic field is exactly equal
placed at that place E.vertical plane containing and opposite to that due to a magnet E.The dip
the magnetic axis of a freely suspended angle is the angle between the horizontal and the
magnet under the action of the earth’s field earth’s total magnetic field.
Magnetic meridian of a place is also a plane Magnetic field and all other fields are vector
containing the magnetic 𝑁-𝑆 line. quantities.
18. The angle between the geographic and the 25. The anle of dip at Ibadan, Nigeria is 6°𝑆. This
mahentic meridians at a point on the magnetic means that if a bar magnet is freely suspended at
equator is the A.angle of inclination B.angle of dip its centre of gravity, its A.south pole will point
C.angle of declination D.longitude of the point. towards the earth at an angle of 6° to the
Angle of declination or variation or magnetic horizontal B.magnetic axis would make an angle
delcination is zero at the geographic north-pole. of 6° with the earth’s magnetic axis C.south pole
19. The reading of a magnetic compass at the 𝑁- would point towards the earth at an angle of 6° to
pole would not be useful because A.the magnetic the vertical D.north pole would point towards the

264
Demystified Series Physics Demystified by Dr Timothy
earth at an angle of 6° to the vertical. cockscrew rule, downward or southward current
26. If the total earth’s field is 𝑄 at a place where gives a clockwise field i.e option A.ALternatievly,
the angle of dip is θ, then the horizontal applying the mnemonic: Clin C down south
component of the field is A.𝑄𝑠𝑖𝑛𝜃 B.𝑸𝒄𝒐𝒔𝜽 :Cl=clockwise, in=into, C=cross,
C.𝑄𝑡𝑎𝑛𝜃 D.𝑄 E.𝑄 ÷ 𝑐𝑜𝑠𝜃. down=downwards, south=southward i.e
Option A represents the vertical component of clockwise field is produced from southward or
the earth’s field. downward or into the plane, cross current.
27. The horizontal component of the earth’s 32. Which of the following statements is not true
magnetic fields is I.zero at the magnetic equator of the magnetic field around an infinite straight
II.zero at the magnetic poles III.maxiumum at the current carrying conductor A.the lines of
magnetic equator IV.maximum at the magnetic magnetic flux are in concentric circles B.the lines
poles. Which of the following is/are correct? A.I lie in a plane perpendicular to the conductor
B.II C.II and III D.I and IV. C.the strength of the magnetic field depends on
Angle of dip, φ is 0° at the equator and 90° at the the current flowing in the conductor D.the lines
poles. Horizontal component, 𝐻=𝐵𝑐𝑜𝑠φ. of magnetic flux are parallel to each other in
At the equator,𝐻=𝐵𝑐𝑜𝑠0°=𝐵 × 1=𝐵. i.e maximum the plane of the conductor E.the magnetikc flux
value. At the poles, 𝐻=𝐵𝑐𝑜𝑠90° = 𝐵 × 0=0 i.e decreases as the perpendicular distance from the
minimum value. The earth’s magnetic field is conductor increases
entirely vertical at the magnetic pole and entirely The magnetic lines of force or lines of magnetic
horizontal at the magnetic equator. flux in a straight current-carrying conductor are
28. In storing magnets, keepers are used to perpendicular(not parallel) to the plane of the
A.reduce self demagnetization B.cancel the conductor.
effect of the earth’s magnetic field C.protect the 33. A current-carrying conductor experiences a
magnet from stray electric fields D.increase the force when placed in a magnetic field because the
strength of the magnets. A.conductor is magnetized B.magnetic field of
29. I II III current interacts with external magnetic field
C.force due to motor principle D.electric field of
N N N current interacts with external magnetic field.
34. Which of the following diagrams correctly
Iron Plastic Wood illustrates the magnetic field pattern around a
A magnet is placed successively near the rings straight conductor which carries current out of
made of the materials indicated in the diagram the planeof the paper,if the earth’s magnetic
above. The polarity of the magnet is also field is neglected?
indicated. Inside which of therings will a A. B. C.
magnetic field be observed? A.I B.III C.I and III ● ● ●
D.II and III D.I,II and III.
Magnetic fields are screened or shielded by D.
diverting the field lines away from the rings by a ●
magnetic material e.g soft iron rings.Magnetic
fields lines will pass through non-magnetic field
materials e.g wood and plastic. Applying clenched fist rule or Maxwell’s
30. A plotting compass is placed near a cockscrew rule, current out of the plane of the
I.stationary positive charge II.bar magnet III.wire paper is same as upward current and will give an
carrying an electric current, In which of the anticlockwise field. i.e option B, the magnetic
above would a deflection of the compass occur? field of the current carrying wire alone.
A.III B.I and II C.I and III D.II and III E.I,II and III. Alternatively, Applying the mnemonic: Anon D
Magnetic compass is plotted to detect magnetic up north:An=anticlockwise, on=onto, D=dot,
field in a region and will deflect in presence of up=upwards, north=northward i.e anticlockwise
magnetic field. Bar magnets, current-carrying field is produced from northward or upward or
wires and charges in motion( not stationary) onto(out of) plane, dot current.
experience magnetic field. - The earth’s magnetic field is neglected, hence no
31. Which of the following diagrams shows the neutral or maximum density point is formed i.e
correct direction of the magnetic lines of force of option C and D is not correct.
a vertical wire 𝑋𝑌 carrying a current 𝐼? 35. The magnetic field around a straight wire 𝑋
X X X X carrying current into the plane of the paper
A. B. C. D. interacts with the horizontal component of
the earth’s magnetic field. Which of the
𝒀 𝑌 𝑌 𝑌 following diagrams correctly illustrates the
Applying the clenched fist rule or Maxwell’s resultant magnetic field pattern in the horizontal

265
Demystified Series Physics Demystified by Dr Timothy
plane? At point 𝐸, field lines points due
A. B. C. D. 𝑊 𝐸 North and points due South at
point 𝑊.
37. A B

Which of the following statements about the


A straight wire carrying current into the plane of electromagnet shown above is correct? A.when
the paper, cross, is same as downward or the current flows from 𝐴 to 𝐵, the end 𝐴 will be in
southward current, and will give a clockwise north pole B.when the current flows from 𝑩 to
field(According to clenched fist rule or Maxwell’s 𝑨, the end 𝑩 will be south pole C.when the
cockscrew rule). On super-imposing or current flows from 𝐵 to 𝐴, the end 𝐵 will be north
interacting with the earth’s magnetic field, at pole D.when the alternating current is passed
some point both fields points in the same through the coil, the end 𝐴 will remain a north
direction(maximum density point due west) and pole E.the polarity of the electromagnet cannot
points in opposite direction at some other be determined by the direction of current.
points(neutral points due east) i.e Option C. The end through which current enters (starting
i. ii. end) acquires south pole while the end through
𝑃 𝑄 which current leaves(terminating end) acquires
north pole. Hence, if current flows from 𝐴 to 𝐵,
The diagram in “i” represents the magnetic field end 𝐴 acquires S-pole while end 𝐵 acquires N-
due to the straight current-carrying wire only, pole. But if current flows from 𝐵 to 𝐴, end 𝐴
carrying current into the plane of the paper. On acquires N-pole while end 𝐵 acquires S-pole.
interacting with the earth’s magnetic field “ii”, at 38. An equipment that functions as a result of the
point 𝑃, the magnetic field direction due to the magnetic effect of electric current is A.rheostat
wire(moving upward) is in the same direction as B.thermostat C.carbon microphone D.electric
the earth’s magnetic field(moving upward also) bell.
hence, point 𝑃 which is due west is the maximum 39. The magnetic relay is a device for A.timing in
density point.At point 𝑄, the magnetic field sports B.storing magnetic field C.reading the
direction due to the wire(moving downward) is magnetic flux D.controlling another circuit
in opposite direction to the earth’s field(moving carrying larger current.
upward) hence, point 𝑄 which is due eastis the Magnetic relay devices are used as control
neutral point. Alternatively, applying the switches.
mnemonic: Clin C down south-westi.e 40. A motor effect occurs when A.an electric
Southwards or downwards current gives current flowing in a conductor stes up a magnetic
clockwise,into,cross field with maximum density flux round the conductor B.the flux from a
point due west and neutral point due east. Anon permanent magnet produces a magnetic force on
D up north-east i.e Northwards or upwards a conductor C.a current carrying conductpor
current gives anticlockwise,onto, dot field with in a magnetic field experiences a force D.a
maximum density point due east and neutral current carrying conductor is moved parallel to
point due west. the magnetic flux from a permanent magnet E.the
- A current carrying wire behaves like a magnet flux from a conductor carrying a current nullifies
whether it is a magnetic material or not. the flux from a permanent magnet.
36. The figure shows a straight Motor effect is same as the force on a current
vertical conductor 𝑃𝑄 which carrying conductor. It is experienced when a
carries a current 𝐼 in the current carrying wire is placed in a strong
direction indicated 𝑋 is a magnetic field.
compass mounted on a 41. A free current carrying conductor will move
horizontal cardboard east of the when it is placed in a magnetic field. The force
conductor. Neglecting the earth’s magnetic field, causing motion depends on I.the mass of the
the compass needle will point A.northward conductor II.the strength of the magnetic field
B.southward C.eastward D.westward. III.the length of the conductor IV.the strength of
The diagram shows a notthward or upward the current A.I,II and III B.I,II and IV C.I,III and
current, hence it will give an anticlockwise field. IV D.II,III and IV E.I,II,III and IV.
At point 𝑋 which is due east E ,of the conductor, The force on a current carrying conductor
the magnetic field direction is pointing due North depends on the strength of the magnetic field 𝐵,
or upwards while at point due west W, of the the strength of the current 𝐼 and the length of the
conductor, the magnetic field direction is due conductor 𝐿. i.e 𝐹 = 𝐵𝐼𝐿𝑠𝑖𝑛θ. NB: The force is
South or upward. greatest or maximum when 𝜃=90° and minimum
or zero when 𝜃=0° or 180°.

266
Demystified Series Physics Demystified by Dr Timothy
42. The direction of motion of a current-carrying currents 3𝐴 and 5𝐴 respectively. If the force
conductor placed within the opposite poles of a experienced per unit length by wire 𝑋 is 5×10-5𝑁,
magnet is determined using A.Fleming’s left- the force per unit length experienced by wire 𝑌 is
hand rule B.Fleming’s right-hand rule A.5×10-5N B.5×10-4N C.3×10-6N D.3×10-5N .
C.Maxwell’s cockscrew rule D.Ampere’s rule. The force per unit length,
𝐹
of two parallel
𝐿
43. A wire carrying current of 10𝐴 and 2.5m in
current-carrying wires are equal and opposite.
length is placed in a field of flux density 0.14𝑇,
50. The flux density, 𝐵 at a distance r from a long
what is the force in the wire if it is placed 60° to
straight wire carrying a current, 𝐼 is given by the
the field? A.3.03N B.20.5N C.15.3N D.10.5N .
A.μO𝐼/2𝑟 B.μO𝐼/𝜋𝑟 C.μO𝐼/𝜋𝑟 2 D.μO𝑰/2𝝅 .
𝐵=0.14𝑇, 𝐼=10𝐴, 𝐿=2.5m, 𝐹=? θ=60°,
51. The diagram shows a current
𝐹 = 𝐵𝐼𝐿𝑠𝑖𝑛𝜃 = 0.14×10×2.5 sin60° = 3.03N.
-carrying copper placed bet-
44. A conductor of length 2m carries a current of
N S ween the horse-shoe magnet
0.8𝐴 while kept in a magnetic field of magnetic
The wire will tend to move
flux density 0.5𝑇. The maximum force acting on it
A.out of the paper B.into the paper C.towards
is A.0.2N B.0.8N C.3.2N D.8.0N.
the N-pole D.towards the S-pole.
𝐵=0.5𝑇, 𝐿=2m, 𝐼=0.8𝐴, 𝐹=?, Maximum force, 𝐹 =
Applyging Fleming’s left hand rule – The fore-
𝐵𝐼𝐿 = 0.5×0.8×2 = 0.8N.
finger pointing in the direction of the field i.e due
45. A current of 5𝐴 passes through a straight
east from N-S , the second or middle finger points
wire in a uniform magnetic field density2.0×10-
3𝑇. Calculate theforce per unit exerted on the
in the direction of the current i.e upward, then
the thumb points in the direction of the force or
wire when it is inclined at 30° to the field.
motion of the conductor i.e into the paper.
A.0.0025Nm-1 B.0.0005Nm-1 C.0.005Nm-1
Current(second finger)
D.0.00025Nm-1.
(Thumb)Force
𝐼=5𝐴, 𝐵=2.0×10-3𝑇, θ=30°, 𝐹 = 𝐵𝐼𝐿𝑠𝑖𝑛θ,
𝐹
Force per unit length, = 𝐵𝐼𝑠𝑖𝑛θ Field(Fore finger)
𝐿
𝐹
= 2.0×10-3×5 sin30° = 0.005Nm-1. - Upward current gives an into the paper force or
𝐿
motion while downward current produce out of
46. The force experienced by a current-carrying
the paper force or motion.
conductor moving in an electric field is employed
52.
in the working of the I.moving coil ammeter
N S
II.electric bell III.electric motor A.I and II B.II and
III C.I and III D.I,II and III.
In the above diagram, a straight vertical wire is
47. Which of the following statements is not
placed between two opposite poles which are
correct about two parallel conductors carrying
separated horizontally by a short distance. If a
equal currents in the same direction A.each of
current is passed down the wire, in which
the conductors will move if not prevented from
direction will move? A.toward the observer
doing so B.each of the conductors will experience
B.away from the observer C.towards the N-pole
a force C.the forces on the conductor will be equal
D.towards the south pole E.upwards.
D.there are concentric lines of force around each
Applying Fleming’s left hand rule – The fore
conductor E.the two conductors will repel
finger points in the direction of the field i.e due
each other .
east from N−S, the second or middle finger
For two parallel current-carrying conductors
points in the direction of the current i.e
placed in a magnetic field, repulsion occurs when
southward, then the thumb points in the
current flows in opposite direction while
direction of the force or motion of the conductor
attraction occurs when current flows in the same
i.e towards the observer(you).
direction.
Field(force finger)
48. 𝑃
Force(Thumb)
𝑄
Current(second finger)
53.
In the figure above, 𝑃 and 𝑄 are two parallel
N S
circular coils, each carrying current as shown.
Then A.𝑃 and 𝑄 repel each other B.𝑷 and 𝑸
The diagram above shows a current carrying
attracts each other C.𝑃 moves, 𝑄 stays still D.𝑄
wire between the poles of a magnet. In which
moves, 𝑃 stays still E.𝑃 and 𝑄 both turn around .
direction would the wire tend to move? A.into
The diagrams shows that current flows in the
the paper B.out of the paper C.towards the
same direction in coil 𝑃 and 𝑄 which are parallel
south pole of the magnet D.towards the top of the
to each other hence, they will attract each other.
page .
49. Two long parallel wires 𝑋 and 𝑌 carry

267
Demystified Series Physics Demystified by Dr Timothy
Applying Flemings left hand rule – The fore finger towards you(observer). In side 𝑇𝑅, the current is
points in the direction of the field i.e due east flowing upward from 𝑅 to 𝑇, hence the force or
from 𝑁 − 𝑆, the second or middle finger points in motion of side 𝑇𝑅 is out of the paper i.e away
the direction of the current i.e southward, then from you (observer).
the thumb points in the direction of the force or 57. Q
motion i.e out of the paper(your finger points S P N
away from you).
Field(fore finger) S
R
Force(fore finger) PQRS is a rectangular coil with PQ perpendicular
Current(second current) to the magnetic field as shown in the diagram
54. Force above. If a current flows in the direction PQRS, in
what direction will the coil move? A.upward
Field B.downwards C.towards the north D.towards
the south pole.
In the diagram above, if the magnetic field points According to Fleming’s left hand rule –
into the paper and the force on a current carrying Considering the side 𝑄𝑅, the fore finger points in
conductor points upwards, what is the direction the direction of the field which is due west from
of the current? A.right B.left C.downwards N-S, the second finger points in the direction of
D.upwards . the currect i.e downwards, hence the force is
Applying Fleming’s left hand rule – The current directed into the paper or towards the observer
points to the right as the force or motion points i.e downwards.
upwards and the field is into the paper(your (fore finger)Field
finger points towards you).
Force(Thumb) (Thumb)Force
(fore finger)Field Current(second finger)
(second finger) 58. A vertical reactangular coil of sides 3cm and
Current 2cm has 20 turns and carries a current of 2𝐴.
55. Which of the following is/are correct about a Calculate the torque on the coil when it is placed
plane rectangular current-carrying coil placed in in a uniform horizontal magnetic field of 0.1𝑇
a uniform magnetic field? I.The forces in the two with its plane parallel to the field.
parallel sides produces a resultant translational A.1.0×10-3Nm B.2.4×10-3Nm C.4.8×10-2Nm
motion II.The torque is maximum when the plane D.6.0×10-2Nm.
of the coil is parallel to the direction of the field Length 𝐿=3cm=0.03m, breadth 𝑏=2cm=0.02m
III.The forces on the two parallel sides are equal 𝐴 = 𝐿𝑏, 𝐼=2𝐴, 𝐵=0.1𝑇, N=20turns, 𝜃=0° (planes
and opposite A.I B.II C.III D.I and III E.II and III . parallel to the field), Torque 𝜏=?
The forces in the two parallel sides of a plane 𝜏 = 𝐵𝐴𝑁𝐼𝑐𝑜𝑠𝜃, Maximum torque, 𝜏 = 𝐵𝐴𝑁𝐼
rectangular coil are equal and in opposite 𝜏 = 𝐵𝐿𝑏𝑁𝐼 = 0.1×0.03×0.02×20×2
direction but the currents are in opposite 𝜏 = 0.00024Nm = 2.4×10-4Nm.
direction. The forces constitute a couple, which - The magnitude of the troque increases with the
produces a rotational motion of the coil. The strength of the magnetic field 𝐵, area of the coil 𝐴
torque decreases as the angle between the plane i.e length and breadth, number of turns in the coil
of the coil and the field increases i.e torque is 𝑁 and current in the coil, 𝜏 ∝ 𝐵𝐴𝑁𝐼.
maximum when the plane of the coil is parallel or 59. An electron is projected at an angle of 50° to
the arms is perpendicular to the field and a magnetic field of flux density 15𝑇 with a
minimum or zero when the plane of the coil is velocity of 7.0×107ms-1, calculate the magnitude
perpendicular or the arms is parallel to the field. of the force exreted on it [e=1.6×10-19C]
56. 𝑃 𝑇 A.1.1×109N B.4.9×107N C.1.3×10-10N
S N D.2.6×10-11N.
𝑞=e=1.6×10-19C, 𝑣=7.0×107ms-1, θ=50°, 𝐵=15𝑇,
𝑄 𝑅 𝐹=? 𝐹 = 𝑞𝑣𝐵𝑠𝑖𝑛𝜃,
In the figure above 𝑃𝑄𝑅𝑇 is a rectangular coil 𝐹 = 1.6×10-19×7×107×15 sin50° = 1.3×10-10N.
situated in a magnetic field. If the current flows in 60. An electric charge of 0.05C moves with a
the direction indicated, the sides 𝑃𝑄 will tend to speed of 2ms-1 at right angle to a uniform
move A.towards the 𝑆-pole B.toward the 𝑁-pole magnetic field of flux density 0.04𝑇. Calculate the
C.out of the paper D.into the paper . magnitude of the magnetic force acting on the
The field is due west or leftwards from 𝑁-𝑆, and charge A.0.000N B.0.004N C.0.001N D.1.600N.
the current is downwards from 𝑃 to 𝑄, hence the 𝑞=0.05C, 𝑣=2ms-1, 𝜃=90°(charge at right angles
force or motion of the side 𝑃𝑄 is into the paper i.e to the field), 𝐵=0.04𝑇, 𝐹=?,

268
Demystified Series Physics Demystified by Dr Timothy
𝐹 = 𝑞𝑣𝐵𝑠𝑖𝑛𝜃, Maximum force, 𝐹 = 𝑞𝑣𝐵 ., magnetic current. It has a non-linear scale and is
𝐹 = 0.05×2×0.04 = 0.004N. used for measuring high frequency a.c and d.c.
61. In order to make a moving electron follow a 67. Which of the following actios will not
circular path A.a magnetic field is applied increase the sensitivity of a moving coil
perpendicular to its path B.a magnetic field is galvanometer? A.Using strong temporary
applied parallel to its path C.an electric field is magnet B.Increasing the area and number of
applied parallel to its path D.an electric field is turns of the coil C.Using weak spiral spring
applied perpendicular to its path. D.Using light pointer .
62. An electron moves with a speed of The sensitivity of a moving coil galvanometer is
2.00×107ms-1 in an orbit in a uniform magnetic increased with a special alloy permanent magnet,
field of 1.20×10-3𝑇. Calculate the radius of the coil of large area and large number of turns,use
orbit [mass of an electron = 9.11×10-31kg, charge of weak hair springs(strength of the spring) and
of an electron= 1.61×10-19C] A.4.72×10-2m light pointer.
B.9.43×10-2m C.1.89×10-2m D.3.78×10-2m. I.The soft iron cylinder determine the magnitude
𝑣=2.00×107ms-1, 𝐵=1.20×10-3𝑇, m=9.11×10- of the radial magnetic field. II.Rotation of the coil
𝑚𝑣
31kg, 𝑞=𝑒=1.61×10-19C, 𝑟=?, 𝑟 = , which is indicated by the pointer depends on the
𝑞𝐵
9.11×10−31 ×2.00×107
strength of current i.e θ ∝ pointer ∝ 𝐼.
𝑟= = 9.43×10-2m. 68. Which of the following statements is
1.61×10−31 ×1.2×10−3
63. If a charge ion goes through combined incorrect? The sensitivity of a galvanometer
electric and magnetic fields, the resultant which incorporates an optical lever arrangement
𝑬
emergent velocity of the ion is A. B.𝐸𝐵 C.
𝐵 may be increased by providing A.a larger
𝑩 𝐸 number of turns in the coil CB.a thinner
D.𝐸 − 𝐵.
suspension with a small torsional control C.a
Electric force on charge, 𝐹 E= 𝑞𝐸, 𝑞=charge,
special ally permanent magnet which gives a high
𝐸=electric field intensity. Magnetic force on
magnetic flux D.all of the above E.a light source
charge, 𝐹 M= 𝑞𝑣𝐵. When both fields interact,
𝑞𝐸 𝐸 with greater intensity.
𝐹 E = 𝐹 M, 𝑞𝐸 = 𝑞𝑣𝐵, 𝑣 = = . Sensitivity of any galvanometer is increased by
𝑞𝐵 𝐵
64. A straight horizontal length of copper wire increasing the strength of magnetic field (using a
has a current of 28𝐴 through it. Find the special alloy permanent magnet) and number of
magnitude of the minimum field needed to turns in the coil.
suspend the wire. The linear density of the wire Sensitivity of a reflecting or suspended coil
is 46.6g/m A.1.6×10-2𝑻 B.1.5×10-2𝑇 C.1.4×10- galvanometer i.e galvanometer with optical lever
2𝑇 D.1.3×10-2𝑇. arrangementor light beam as a pointer, is
𝐼=28𝐴, 𝑔=10ms-2, Linear density=mass per unit increased by using a thin suspension with small
𝑚
length= =46.6g/m=0.0466kg/m torsional control. Sensitivity of a moving coil
𝐿
On suspending the wire in the magnetic field, the galvanometer is increased by using a weak hair
force on the current carrying wire is completely spring with small torsional couple.
balanced by the weight of the wire 𝐹 = 𝑊, 𝐹 = 69. The instrument used for measuring both a.c
𝑔(𝑚/𝐿) and d.c is A.an inverter B.a moving iron
𝐵𝐼𝐿, 𝑊 = 𝑚𝑔, 𝐵𝐼𝐿 = 𝑚𝑔, 𝐵 = , ammeter C.a current balance D.moving coil
𝐼
𝐵=
10(0.0466)
= 1.6×10-2𝑇. ammeter .
28
65. On which of the following is the operation of Moving iron ammeter measures low frequency
a moving coil galvanometer based? A.magnetic a.c and d.c by the magnetic effect of electric
effect of electric current B.electromagnetic current.
induction C.chemical reaction porudced by Moving coil galvanometer measures d.c only by
electricity D.force on a current carrying force on a current carrying conductor.
conductor in a magnetic field E.electrical Current balance or ampere balance measures d.c
energy and power . by force between parallel current carrying wires.
Moving coil and moving irons instruments works Inverter changes the magnitude of voltage or
on the motor effect i.e force on a current carrying current betweeb input and output.
conductor but also depends on magnetic effect of 70. A rectangular coil of wire can rotate in a
electric current. magnetic field. The ends of the coil are soldered
66. Wich of the following devices does bot make to the two halves of a split-ring. Two carbon
use of magnetic effect of an electric current? brushes are made to press lightly against the split
A.moving coil ammeter B.hot wire ammeter ring and when these are connected in circuit with
C.moving iron ammeter D.moving coil a battery and rheostat, the coil rotates. This is a
loudspeaker E.telephone receiver . description of A.a suspended-coil galvanometer
Hot wire ammeter works on heating effect of B.a moving coil ammeter C.d.c generator D.an
electric motor E.an induction coil.

269
Demystified Series Physics Demystified by Dr Timothy
In an electric motor, an external supply of current shunt resistance required is A.1.9Ω B.2.0Ω
from battery is needed to rotate the coil while in C.36.2Ω D.38.0Ω .
a d.c generator is rotates when supplied 1 𝐼
𝑖= 𝐼, =20, 𝐺=20Ω, 𝑆=?, 𝐼 =
𝑖(𝑆+𝐺) 𝐼
, =
𝑆+𝐺
,
energy(not current) and current is induced in the 20 𝑖
𝑆+ 38
𝑆 𝑖 𝑆

generator. 20 = , 20𝑆 = 𝑆 + 38 , 20𝑆 − 𝑆 = 38,


𝑆
38
71. Which of the following is not a part of a d.c 19𝑆 = 38 , 𝑆 = = 2Ω.
19
electric motor? A.field magnet B.ammature 77. A milliammeter of resistance 100Ω and full
C.commutator D.transformer. scale deflection of 50m𝐴 is to be converted into
72. What is the effect of using split rings in a voltmeter with a full scale of 100𝑉. The
simple d.c motor? A.The direction of rotation of resistance required is A.0.05Ω in parallel with
the coil is reversed B.The current in the coil flows the milliammeter B.0.05Ω in series with the
in the same direction C.The current in the coil milliammeter C.1,900Ω in parallel with the
becomes alternating D.The direction of the milliammeter D.1,900Ω in series with the
force is reversed E.The contact with the voltage milliammeter
source is cut off . 𝑉=100𝑉, 𝑖=50m𝐴=0.05𝐴, 𝐺=100Ω, 𝑀=?,
A split ring commutator half changes contact 𝑉 = 𝑖(𝑀 + 𝐺), 100 = 0.05(𝑀 + 100),
between carbon brushes, thereby reversing the 100
current and force direction and maintains the = 𝑀 + 100 , 2000 = 𝑀 +100 ,
0.05
direction of rotation of the coil. 𝑀 = 2000−100 = 1900Ω.
73. Which of the following statements are not 78. A moving coil meter of internal resistance
true of a moving coil milliammeter? I.It can be 20Ω has a full scale deflection of 10m𝐴. A series
used to measure alternating current II.It has a resistance of 980Ω is connected to the
linear scale III.It can be adapted to read higher galvanometer in order to convert it into a
values of current IV.A resistor connected in voltmeter. The full scale deflection of the
parallel with the milliammeter would convert it voltmeter is A.1𝑉 B.10𝑽 C.20𝑉 D.100𝑉.
to a voltmeter A.I and IV B.II and III C.III and IV 𝑖=10m𝐴=0.01𝐴, 𝐺=20Ω, 𝑀=980Ω, 𝑉=?,
D.I,II and III E.I,III and IV . 𝑉 = 𝑖(𝑀 + 𝐺), 𝑉 = 0.01(980+20) ,
Moving coil ammeter is used to measure direct 𝑉 = 0.01(1000) = 10𝑉.
current only and it has a linear scale. It can be 79. A moving coil galvanometer has a full scale
adapted to measure higher values of current by deflection of 3𝐴 equivalent to 30° deflection. The
connecting a low resistance shunt in parallel to it. sensitivity of the instrument is A.0.1° B.10.0°
It is converted to voltmeter, by connecting a high C.33.0° D.90.0°.
resistance resistor (multiplier) in series(not Current, 𝐼=3𝐴, angle of rotation or deflection,
𝜃
parallel) to it. 𝜃=30°, current sensitivity 𝑆𝐼 =? , 𝑆𝐼 = ,
𝐼
74. A galvanometer can be converted into an 30
𝑆𝐼 = = 10°𝐴-1 .
ammeter by connecting A.a low resistance 3
shunt in parallel with the galvanometer B.a 80. A bar magnet is placed close to and lying
low resistance shunt in series with the along the axis of a closed coil. There will be no
galvanometer C.a high resistance multiplier in current in the coil when A.both the coil and the
parallel with the galvanometer D.a high magnet are held stationary B.the coil is moved
resistance multiplier in series with the towards the stationary magnet C.the coil is
galvanometer moved away from the stationary magnet D.the
Option D is used to convert a galvanometer into a magnet is moved away from the stationary coil .
voltmeter. Emf(and current) is induced in a coil whenever
75. A moving coil galvanometer of 300Ω there is relative motion between the magnet and
resistance gives a full scale deflection fro 1.0m𝐴. the coil. No emf or current is induced when both
The resistance 𝑅 of the shunt that is required to the magent and the coil are stationary or moving
convert the galvanometer into a 3.0𝐴 ammeter is in the same direction with the same speed.
A.899.7Ω B.10.00Ω C.0.10Ω D.0.01Ω . 81.
𝑖=1.0m𝐴=0.001𝐴, 𝐺=300Ω, 𝐼=3𝐴, 𝑆=𝑅=?,
𝑖(𝑆+𝐺) 0.001(𝑅+ 300) S N
𝐼= , 3= ,
𝑅 𝑅
3𝑅
3𝑅 = 0.001(𝑅 +300) , = 𝑅 + 300 ,
0.001
3000𝑅 = 𝑅 + 300 , 3000𝑅 − 𝑅 = 300, G
2999𝑅 = 300 , 𝑅 =
300
= 0.10Ω. The figure above represents an insulated
2999 cylindrical coil copper wire wound round a paper
76. A galvanometer of resistance 38Ω is to be tube. The ends of the coil are connected to a
connected with a shunt that 1/20 of the current sensitive galvanometer 𝐺. When a magnet was
in a circuit passes through the galvanometer. The plunged into te coil it was observed that the

270
Demystified Series Physics Demystified by Dr Timothy
galvanometer needle gave a deflection. Which of 87. A band of 500 rectangular loops of wire of
the following statements correctly describes this area 20cm by 20cm, encloses a region of
experiment? A.the strength of the current magnetic field which changes from 0.4𝑇 to 1.0𝑇
produced us inversely proportional to the speed within 5 seconds, calculate the induced emf
with which the magnet is plunged into the coil A.5.60𝑉 B.24.00𝑉 C.0.24𝑉 D.2.40𝑽.
B.the effect illustrated is induced 𝑁=500 loops, 𝐴=𝐿𝑏, 𝐿=20cm=0.2m,
electromotive force C.the direction of the 𝑏=20cm=0.2m, 𝐵 2=1𝑇, 𝐵 1=0.4𝑇, 𝑡=5s, 𝐸=?
current is always such that it enhances the 𝐸=𝑁 =𝑁
𝑑𝜙 𝐴∆𝐵
=
𝑁𝐴(𝐵2 −𝐵1 )
,
change producing it D.the galvamometer gave a 𝑑𝑡 𝑑𝑡
500×(0.2×0.2)×(1−0.4)
𝑡

permanent deflection E.on removing the magnet 𝐸= = 100×0.04×0.6 =2.4𝑉.


5
from the coil the galvanometer deflects in the 88. The coil of an electric generatir has 500 turns
same direction . and 8.0cm diameter. If it rotates in a magnetic
The strength of the induced emf (hence current) field of density 0.25𝑇, calculate the angular speed
is directly proportional to relative speed of the when its peak voltage is 480𝑉. (𝜋=3.142)
magnet,number of turns in the coil, area of the A. 191rads-1 B. 382rads-1 C. 764rads-1
coil and the strength of the magnetic field i.e 𝐸 ∝ D. 1528rads-1.
𝜔𝐵𝐴𝑁. The direction of the induced current(or 𝑁=500turns, 𝑑=8cm, 𝑟=4cm=4×10-2m, 𝐵=0.25𝑇,
emf) is walways such that it opposes the change 𝐸𝑜 =480𝑉, 𝜔=?, 𝐸𝑜 = 𝜔𝐵𝐴𝑁 , 𝐴=𝜋𝑟 2 ,
producing it i.e Lenz’s law. The galvanometer 𝐸𝑜 = 𝜔𝐵(𝜋𝑟 2)𝑁 ,
deflects with the induced current and returns to 480 = 𝜔 ×0.25× 𝜋 ×16×10-4×500 ,
its original position i.e opposite direction, on 480 = 0.2𝜋𝜔 , 𝜔 =
480
= 763.94 = 764rads-1.
0.2𝜋
removal of the current due to removal of the
89. A conductor of length 1m moves with a
magnet.
velocity of 50ms-1 at an angle of 30° to the
82. A circuit has an area of 0.4m2 and contains 50 direction of a uniform magnetic field of flux
loops of wire. If the loops are twisted and allowed
density 1.5𝑊𝑏m-2. What is the emf induced in the
to rotate at a constant angular velocity of 10rads-
1 in a uniform magnetic field of 0.4𝑇, the
conductor? A.50.5𝑉 B.37.5𝑽 C.75.0𝑉 D.80.5𝑉.
𝐿 =1m, 𝑣=50ms-1, θ=30°, 𝐵=1.5𝑊𝑏m-2 or 1.5𝑇,
amplitude of the induced voltage is A.80𝑽 B.20𝑉 𝐸=?, 𝐸 = 𝐵𝐿𝑣 𝑠𝑖𝑛𝜃 = 1.5×1×50 𝑠𝑖𝑛30° = 37.5𝑉.
C.8𝑉 D.16𝑉.
90. A wire of length 30cm is moved with a speed
𝜔=10rads-1, 𝐵=0.4𝑇, 𝐴=0.4m2, 𝑁=50 loops, 𝐸𝑜 =?. of 2ms-1 at right angles to a magnetic field of flux
Effecctive value of induced voltage, 𝐸 = 𝐸𝑜 sin𝜔𝑡, density 0.4𝑇. Calculate the emf in the wire
amplitude of induced voltage 𝐸𝑜 = 𝜔𝐵𝐴𝑁
A.0.15𝑉 B.0.24𝑽 C.2.70𝑉 D.24.00𝑉
𝐸𝑜 =10×0.4×0.4×50 =80𝑉. 𝐿=30cm=0.3m, 𝑣=2ms-1, 𝐵=0.4𝑇, 𝐸=?,
83. Induced emfs are best explained using 𝐸 = 𝐵𝐿𝑣 = 0.4×0.3×2 = 0.24𝑉.
A.Faraday’s law B.Coulumb’s law C.Lenz’s law
91. A horizontal bar magnet is being moved
D.Ohm’s law.
relative to a vertical coil olaced close to it. Which
Induced emfs are explained by Faraday’s law or of the folllwing diagrams correctly illustrates the
Neuman’s law of electromagnetic induction
direction of the induced current in the coil?.
while induced currents is explained by Lenz’s
A. 𝑁 𝑆 B. 𝑵 𝑺 C. 𝑁 𝑆
law.
84. Which of the following will not lead to an
increase in the induced emf in a coil of wire
D. 𝑁 𝑆
rotating between the poles of a magnet?
Increasing the A.strength of magnet B.number of
turns of coil C.speed of rotation of coil D.gap
According to Len’z law – the direction of induced
between poles of the magnet.
current is in such a way that it opposes the
85. Lenz’s law is a law of the conservation of changes producing it. If the 𝑁-pole of a magnet is
A.electric current B.momentum C.energy
moving closer to the coil, the magnetic flux
C.electric charge.
linking the coil induces current in the coil such
86. The magnetic flux in a coil having 200 turns that it opposes the motion of the magnet towards
changes at the time rate of 0.08𝑊𝑏s-1. The the coil, hence anticlockwise current
induced emf in the coil is A.250.0𝑉 B.25.0𝑉
direction(Option B),which in turns forms a 𝑁-
C.16.0𝑉 D.1.6𝑉 pole in the coil and the magnet is repelled from
𝑑𝜙
𝑁=200 turns, =0.08𝑊𝑏s-1, 𝐸=?, the coil.
𝑑𝑡
Farday’s law of electromagnetic induction, 92. A dynamo primarily converts A.mechanical
induced emf is proportional to the rate of change energy to electrical energy B.electrical energy
𝑑𝜙
of magnetic flux, 𝐸 ∝ , 𝐸 = 𝑁 ,
𝑑𝜙 into kinetic energy C.potential energy into kinetic
𝑑𝑡 𝑑𝑡 energy D.kinetic energy into potential energy
𝐸 = 200×0.08 = 16𝑉.

271
Demystified Series Physics Demystified by Dr Timothy
An electric motor converts electrical energy into B.they are good conductors C.their resistance
mechanical energy. changes wth temperature D.their resistance
93. Which of the following methods can be used canges with pressure .
to increase the emf produced by a simple 100. Which of the following devices converts
dynamo? I.Increase the speed of rotation of the sound into electrical energy? A.a microphone
coil II.Using a longer field magnet III.Increasing B.telephone earpiece C.an electric bell D.a horn .
the cross sectional area of the coil IV.Increasing A telephone mouthpiece and a microphone
the number of coil windings A.I and III B.I and IV converts sound energy into electrical energy
C.I,III and IV C.I,II,III and IV . while telephone earpiece converts electrical to
The induced emf produced by a dynamo(d.c sound energy.
generator) increases with the strength of the 101. The back emf induced in a current-carrying
magnetic field(not the magnetic length of the coil causes A.resistance B.inductance C.eddy
magnet), the number of coil windings,area of the currents D.resonance .
coil(not the cross sectional area of the coil wire) Eddy current is produced form back emf set up.
and the speed of rotation of the coil , 𝐸 ∝ 𝜔𝐵𝐴𝑁. 102. Which of the following devices does not
Winding of the coil on a soft iron armature/core make use of eddy current for its action? A/an
increases the magnetic flux and strength through A.galvanometer B.speedometer C.induction
the coil.The resistance of the coil e.g copper coil, furnance D.induction coil.
does not affect the value of the induced emf. 103. Which of the following statements about a
94. On an a.c generator which of the following transformer is correct? A.The input is a.c and the
does not apply? A.field magnet B.slip rings output is d.c B.The input is a.c and the output
C.commutators D.armature . is a.c C.The input is d.c and the output is a.c
Commutators or split rings are found in d.c D.Energy loss is increased by laminating the core
generators only. E.The core is usually made of steel .
95. A generator manufacturing company was A transformer works with a.c and not d.c, hence
contracted to produce an a.c dynamo but the input at the primary coil is a.c and the output
inadvertently produced a d.c dynamo. To correct at the secondary coil is also a.c.
this error, A.armature coil should be made of 104. When a transformer has more secondary
silver B.commutator should be replaced with windings than primary windings, it A.has
split rings C.armature coil should be made of smaller secondary current B.has a greater
aluminium D.commutator should be replaced output power C.is a step down transformer
with slip rings . D.increases the total enrgy output.
Slip rings is in a.c generator while split rings is in Step-up transformer has more secondary
d.c generator. A.c generator can be converted to windings than the primary winding (𝑁𝑆 > 𝑁𝑃 ) or
a d.c generator when the slip ring of the a.c is 𝑁
turn ratio, 𝑆 > 1. A step-up transformer steps up
𝑁𝑃
replaced woth a split ring.
emf and steps down current , 𝐸𝑆 > 𝐸𝑃 and 𝐼𝑆 < 𝐼𝑃 .
96.In a simple generator, the main function of a
The opposite holds for step-down. A transformer
commutator is to A.change the direction of
steps-up or step-down a.c voltage(emf) without
rtation of the armature B.provide a uniform
altering the frequency ,power or energy.
magnetic field around the armature C.change
105. An ideal step down transformer steps up
the direction of current in the coil D.increase
A.power B.energy C.current D.voltage.
the magnetic flux linking the coil.
106. The main purpose of the transformer in an
97. Which of the following statements about a
a.c radio set is to A.increase power to the radio
generator is not correct? A.It can produce direct
B.convert energy from a.c to d.c C.step down
current B.It can produced alternate current C.It
voltage D.step up voltage .
requires an external supply of energy to rotate
Step-down transformer is used in various
the coil D.It requires an external supply of
electronic devices to step-down voltage for
current to the coil E.It may require the use of a
utilization in homes.
commutator.
107. The efficiency of a transformer can be
98. A steady current of 2𝐴 flows in a coil of emf
increased if A.high resistance copper coils are
12𝑉 for 0.4s. A back emf of 3𝑉 was induced
used B.the core is designed to increase eddy
during this period. The energy stored in the loop
currents C.the core is designed to make sure that
that can be utilized is A.7.2J B.12.0J C.4.7J D.9.6J.
little primary flux is linked with the secondary
Induced emf in coil 𝐸=12𝑉, back emf 𝐸𝐵 =3𝑉,
D.the core is made of soft magnetic materials
𝐼=2𝐴, 𝑡=0.4s, The energy stored in the loop of the
thereby reducing the energy for magnetic
coil , 𝑊 = 𝐼(𝐸 − 𝐸𝐵 )𝑡.
reversals .
𝑊 = 2×(12−3)×0.4 = 0.8×9 = 7.2J.
The heat energy loss in a transformer is reduced
99. Carbon particles are used in carbon
inorder to increase its efficiency.
microphine because A.they mught amplify sound

272
Demystified Series Physics Demystified by Dr Timothy
Use of low resistance or thick copper coils reduce 𝐼𝑃 =
9×45×100
= 3.42𝐴 .
92.5×128
copper losses or joules effect.
113. A transformer has 400 turns and 200 truns
Use of laminated iron core(iron sheets) reduced
in the primary and secondary windings
eddy current loses.
respectively. If the current in the primary and
Use of efficienct core design to reduce flux
secondary windings are 3𝐴 and 5𝐴 respectively,
leakage and to ensure maximum flux linkage or
calculate the efficiency of the transformer
reduce design losses.
A.85.0% B.83.3% C.37.5% D.30.0% .
Use of soft iron or special allow magnetic
𝑁𝑃 =400 turns, 𝑁𝑆 =200 turns, 𝐼𝑃 =3𝐴, 𝐼𝑆 =5𝐴,
materials as core to reduce hysteresis loses due 𝑁𝑆 200 1 𝑁 𝐸 1
to magnetic reversals. = = , 𝑆 = 𝑆= ,
𝑁𝑃 400 2 𝑁𝑃 𝐸𝑃 2
108. What precaution should a manufacturer 𝜖=
𝑃𝑜
× 100 =
𝐼𝑆 𝐸𝑆
× 100 =
𝐼𝑆
×
𝐸𝑆
× 100,
𝑃𝑖 𝐼𝑃 𝐸𝑃 𝐼𝑃 𝐸𝑃
take to ensure that energy loss in a transformer 5 1
is minimized? A.the winding of the transformer 𝜖 = × × 100 = 83.3% .
3 2
should be made of high resistance wires B.the 114. Which of the following can be described as
core should be made of thin sheets of metal high tension transmission? A.high resistance and
C.no magnetic material should be used to make low voltage B.low current and high voltage
the core D.the flux linking the primary with the C.high current and low voltage D.high voltage and
secondary coil should be minimum. zero current E.high current and low resistance .
109. A transformer has primary coil with 500 High tension transmission is transmission of
turns and a secondary coil with 2500 turns. power at high voltage and low current while low
When the voltage input to the primary coil is tension transmission is transmission of power at
120𝑉, the output is A.6000𝑉B.600𝑽C.240𝑉 low voltage and high current.
D.60𝑉 E.24𝑉 . 115. It is usual to transmit electric power at high
𝑁𝑃 =5000 turns, 𝑁𝑆 =2500 turns, Input voltage= voltage and low current. Which of rhe following
primary voltage 𝐸𝑃 =120𝑉, Output voltage= are possible advantageous of the method? I.heat
𝑁 𝐸
secondary voltage 𝐸𝑆 =?, 𝑆 = 𝑆 , losses are reduced because the currents are small
𝑁𝑃 𝐸𝑃
2500 𝐸 II.thin wires can be used because small currents
= 𝑆 , 𝐸𝑆 = 5×120 = 600𝑉. are flowing III.the power can flow faster because
500 120
110. A dry battery of 6𝑉 is joined to the primary the voltage is high A.I B.II and III C.I and II D.I
coil of a commercial transformer. The secondary and III E.I,II and III.
has twice the number of turns of the primary. The High tension transmission involes high voltage
primary resistance is 2Ω. The voltage obtained in and low current, as heat losses are reduced due
the secondary is A.1.5𝑉 B.3𝑉 C.12𝑉D.0𝑽 E.6𝑉 to low current i.e 𝐻 ∝ 𝐼 2 . A thin wire will offer
𝑁𝑆 =3𝑁𝑃 , A transformer is operates with a.c 1
high resistance to current [𝑅 ∝ ] hence, only
hence, connecting a dry battery(d.c source) to the 𝐴
small current flows.
primary will not induce any voltage or current in
116. Power supply is transmitted in high voltage
the secondary.
and low current in order to A.increase the power
111. A transformer steps down from 240𝑉 to
supply B.prevent overheating of coil C.make it
12.0𝑉 and supplies a 12.0𝑉 24.0𝑊 lamp. What is
travel fast D.make it pass through transformers .
the efficiency of the transformer if the current
117. Which of the following modes is the most
drawn from te 240𝑉 supply is 0.120𝐴. A.57.2%
economical mode of transmitting electrical
B.66.6% C.83.3% D.100%.
power over long distances? A.Alternating
𝐸𝑃 =240𝑉, 𝐼𝑃 =0.12𝐴, 𝐸𝑆 =12𝑉, Power input=
current at high voltage and low current
power in primary=𝐼𝑃 𝐸𝑃 , power output=Power in
B.Alternating current at low voltage and high
secondar=𝐼𝑆 𝐸𝑆 =24𝑊, Efficiency 𝜖=?,
𝑃𝑜𝑤𝑒𝑟 𝑜𝑢𝑡𝑝𝑢𝑡 𝑃 current C.Alternating current at high voltage and
Efficiency = × 100 = 𝑜 × 100 , high current D.Direct current at low voltage and
𝑃𝑜𝑤𝑒𝑟 𝑖𝑛𝑝𝑢𝑡 𝑃𝑖
𝜖=
𝐼𝑆 𝐸𝑆
× 100 =
24
× 100 = 83.3%. high current.
𝐼𝑃 𝐸𝑃 0.12×240 118. The induction coil works on the same
112. A transformer has an efficiency of 92.5%. principle as A.transformer B.a d.c generator
The ratio of the number of turns in the primary C.an a.c generator D.an inductor.
coil to that in the secondary coil is 128:45. If the Induction coil and transformer both works on
current passing through the secondary coil is mutual inductance of emf in a secondary coil due
9.0𝐴, calculate the current passing through the to varying magnetic flux or alteration in current
primary coil. A.2.93𝐴 B.3.16𝐴 C.3.42𝑨 D.27.68𝐴. in a primary coil. A.c and d.c generators works on
𝑁 128 𝑁 𝐸 128 𝐸 45
𝜖=92.5%, 𝑃 = , 𝑃 = 𝑃 = , 𝑆 = , self induction. But they all are electromagnetic
𝑁𝑆 45 𝑁𝑆 𝐸𝑆 45 𝐸𝑃 128
𝑃𝑜 𝐼𝑆 𝐸𝑆 induction.
𝐼𝑆 =9𝐴, 𝐼𝑃 =?, 𝜖 = × 100 = × 100 ,
𝑃𝑖 𝐼𝑃 𝐸𝑃 119. An induction coil is generally used to
𝐼𝑆 𝐸𝑆 9 45
𝜖= × × 100, 92.5 = × × 100 , A.rectify an alternating current B.produce a large
𝐼𝑃 𝐸𝑃 𝐼𝑃 128
intput voltage C.smoothen a pushing direct
273
Demystified Series Physics Demystified by Dr Timothy
D.modulate an incoming radio signal E.produce 1
𝐿=5m𝐻=5×10-3𝐻, 𝐼=6𝐴, 𝐸=?, 𝐸 = 𝐿𝐼 2 ,
2
a large output voltage . 1
Induction coil produces a large out voltage in the 𝐸 = ×5×10-3×62 =9×10-2𝐽.
2
secondary when a low d.c voltage source is
connected to the primary i.e steps up d.c voltage. Jamb past questions on magnetic field,
120. The best material for the core of the primary magnetism and electromagnetism :
and secondaery coils of an induction coil is [1978/30,36,1979/26,33,1980/6,1981/9,45,47,
A.copper, because it is a good conductor of 48,1982/39,1983/33,40,1984/33,34,1985/37,
electricity B.steel, because it becomes magnetic 43,45,1986/45,47,48,1987/38,45,1988/37,45,
and retains its magnetism for a long time 1989/34,45,1990/34,1991/33,42,43,1992/31,
C.rubber, because it is a good insulator and the 32,1993/33,34,43,44,1998/46,1999/36,2000/
user is prevented from shock D.soft iron, 47,2001/43,46,48,50,2002/42,44,47,2003/43,
because it becomes magnetic but easily loses 2004/46,2005/1,6,15,20,2006/30,2008/42,43,
its magnetism E.glass, because it is transparent . 2010/42,2011/47,2012/41,2014/40,41,42,43,
The iron core of an induction coil is made from 2015/18,44]
bundle of wire or laminated into sheets to
minimize eddy currents.
121. The principle of operation of an induction
coil is based on A.Ohm’s law B.Faraday’s law
C.Ampere’s law D.Coulomb’s law .
Operation of an induction coil is based on
Faraday’s law of electromagnetic induction.
122. Which of the following reduces the effect of
the back emf generated in the primary coil of an
induction coil? I.The capacitor in the circuit
II.The make-and-break contact in the circuit
III.The ratio of turns in the secondary A.I B.II C.I
and II D.II and III E.I,II and III .
The capacitor in the primary circuit prevents
sparks in the make-and-break device by reducing
the back emf generated in the primary. The
make-and-break device gives a rapid decaying or
varying magnetic flux which induces a large
ouput d.c voltage in the secondary. The ratio of
turns in the secondary determined the strength
𝐸
of the the induced emf , 𝑁 ∝ .
𝑡ℎ
123.The device used to prevent wearing away of
the make-and-break contacts of an induction coil
is called a/an A.fuse B.electrscope C.resistor
D.capacitor .
124. Given three inductors of inductance 5mH,
10mH and 20mH connected in series, the
effective inductance is A.0.35mH B.35.00mH
C.3.50mH D.2.90mH.
For inductors in series, effective inductance is the
arithmetic sum of their individual inductance,
𝐿 = 𝐿1 + 𝐿2 + 𝐿3 = 5+10+20 = 35mH.
125. Two inductors of inductance 4𝐻 and 8𝐻 are
arranged in series and a current of 10𝐴 is passed
through them. What is the energy stored in them?
A.133J B.250J C.600J D.50J.
Effective inductance, 𝐿 = 4+8=12H, 𝐼=10𝐴,
1
Energy stored in an inductor, 𝐸 = 𝐿𝐼 2
2
1
𝐸 = ×12×102 = 6×100 = 600J.
2
126. The energy stored in aninductor of
inductance 5m𝐻 when a current of 6𝐴 flows
through it is A.1.4×10-2J B.1.8×10-2J C.1.4×10-3J
D.9.0×10-2J.

274
Demystified Series Physics Demystified by Dr Timothy

Chapter 25 – Simple A.C circuits


- Direct current(d.c) produced by a battery resistance offered by the capacitor is capacitive
(cell) or d.c generator(dynamo), has a constant inductance, 𝑿𝑪 =
𝟏
=
𝟏
.
𝝎𝑪 𝟐𝝅𝒇𝑪
value and flows in one direction unless the
battery terminals are reversed. - The energy stored is due to the capacitance
𝟏
- Alternate current(a.c) produced by an a.c (stored in the capacitor). 𝑬 = 𝑪𝑽𝟐 .
𝟐
generator(alternator), has a varying value and 3.Inductance and capacitace – The circuit is
direction with time. predominantly inductive if 𝑋 L> 𝑋C and current
- Instantaneous value of emf or current is its lags voltage(or voltage leads current). The cicuit
value at a particular instant of time. is predominantly capacitive if 𝑋L< 𝑋C and
𝑰 = 𝑰𝒐 𝐬𝐢𝐧(𝜽 + ∅) = 𝑰𝒐 𝐬𝐢𝐧(𝝎𝒕 + ∅). current leads voltage(or voltage lags current).
𝑽 = 𝑽𝒐 𝒔𝒊𝒏(𝜽 + ∅) = 𝑽𝒐 𝒔𝒊𝒏(𝝎𝒕 + ∅). - The energy stored is due to both the
𝐼 or 𝑉=instantaneous value of current and 𝟏
inductance and capacitance. 𝑬= 𝑳𝑰𝟐 + 𝑪𝑽𝟐 .
𝟏

voltage, 𝐼𝑜 or 𝑉𝑜 =maximum or amplitude or peak 𝟐 𝟐


- In a series 𝑅-𝐿-𝐶, circuit current and voltage
value of current and voltage, 𝜔=angular
2𝜋 are in phase.
frequency or angular velocity=2𝜋𝑓 = , - Impendance 𝒁 – It is the total opposition to the
𝑇
𝑓=frequency, 𝑇=period, ∅=phase angle of phase flow of current in an a.c. It is the ratio of the rms
difference or phase shift between 𝐼 and 𝑉. voltage to rms current in an a.c circuit.
𝑻 𝟏 𝑽 𝑽
- At 𝒕 = 𝑻 or 1cycle, 𝜽 =360°. At 𝒕 = or cycle, 𝒁 = 𝒓𝒎𝒔 = 𝒐. 𝒁𝟐 = 𝑹𝟐 + 𝑿𝟐 , 𝑿 = 𝑿𝑳 or 𝑿𝑪 or
𝟐 𝟐 𝑰𝒓𝒎𝒔 𝑰𝒐
𝑻 𝟏 𝑻
𝜽 =180°. At 𝒕 = or cycle, 𝜽 =90° . At 𝒕 = or 𝑿𝑳 − 𝑿𝑪 or 𝑿𝑪 − 𝑿𝑳 .
𝟒 𝟒 𝟖
𝟏 - 𝒁 = 𝑿 = 𝑿𝑳 − 𝑿𝑪 or 𝑿𝑪 − 𝑿𝑳 (for an a.c circuit
cycle, 𝜽 =45°.
𝟖 containing inductor and capacitor only).
- Effective or root mean square(rms) value of - The total voltage 𝑉𝑇 – 𝑽𝑻 𝟐 = 𝑽𝑹 𝟐 + 𝑽𝑳 𝟐 or
voltage and current is the value in which a.c is
𝑽𝑻 𝟐 = 𝑽𝑹 𝟐 + 𝑽𝑪 𝟐 or 𝑽𝑻 𝟐 = 𝑽𝑹 𝟐 + (𝑽𝑳 − 𝑽𝑪 )𝟐 .
quoted. The effective or root mean square
- 𝑽𝑻 = 𝑽𝑳 − 𝑽𝑪 or 𝑽𝑪 − 𝑽𝑳 (for an a.c circuit
(rms) value of an a.c is its d.c equivalent i.e
containing inductor and capacitor only).
has the same heating effect with a direct
𝑽 𝑰 - Voltage across resistor 𝑽𝑹 = 𝑰𝑹 .
current. 𝑽rms = 𝒐 , 𝑰rms = 𝒐 , -Voltage across inductor 𝑽𝑳 = 𝑰𝑿𝑳 .
√𝟐 √𝟐
- Phasor diagrams – Vector diagrams that -Voltage across capacitor 𝑽𝑪 = 𝑰𝑿𝑪 .
shows the relationship between 𝐼 and 𝑉. - Phase angle ∅ between current and voltatge is
1.Resistance in a.c circuit i.e purely resistive given by the quations below –
circuit – Current and voltage are in phase i.e 𝑿 −𝑿
1. 𝒔𝒊𝒏∅ = 𝑳 𝑪 = 𝑳 𝑪 .
𝑽 −𝑽
𝒁 𝑽𝑻
phase angle=0°. 𝑹 𝑽𝑹
2.Inductor in a.c circuit i.e purely inductive 2. 𝒄𝒐𝒔∅ = =
𝒁 𝑽𝑻
circuit – Current lags voltage or voltage leads 3. 𝒕𝒂𝒏∅ =
𝑿𝑳 −𝑿𝑪
=
𝑽𝑳 −𝑽𝑪
𝜋 1 𝑹 𝑽𝑹
current by 90° or radians or cycle.
2 4 - Power in an a.c circuit – Power is dissipated
3.Capacitor in a.c circuit i.e purely capacitive only in a resistor in an a.c circuit. Thus, power
circuit – Current leads voltage or voltage lags by is not dissipated in an inductor or capacitor.
𝜋 1
90° or radians or cycle. - Average power 𝑃𝑎𝑣 of a 𝑅-𝐿-𝐶 a.c circuit, 𝑷𝒂𝒗 =
2 4
- Series 𝑹-𝑳-𝑪 circuit – In a 𝑅-𝐿-𝐶 circuit, current 𝑰𝑽𝒄𝒐𝒔∅.𝐼=rms current,𝑉=rms voltage.
𝑹 𝑽
and voltage are out-of-phase by a phase angle - Power factor = 𝒄𝒐𝒔∅ = = 𝑹 .
𝒁 𝑽𝑻
between 0 to 90°. - Power factor factor of a purely reactive or
- Reactance 𝑿 – This is the opposition to the flow inductive or capacitive circuit is zero or phase
of a.c and is numerically equal to the ratio of the angle ∅=90° and average power is zero 𝑷𝒂𝒗 =
amplitude of voltage to the amplitude of current 0.
𝑽
in inductive or capacitive circuit. 𝑿 = 𝒐 . - The power factor of a DC circuit is always 1.
𝑰𝒐
1.Resistance and Inductance – Current lags - Instantaneous power 𝑷 = 𝑰𝑽, 𝐼=instantaneous
voltage(or voltage leads current) at ∅. The current, 𝑉=instantaneous voltage.
resistance offered by the inductor is inductive - In a purely reistive circuit or circuit with
inductance , 𝑿𝑳 = 𝝎𝑳 =2𝝅𝒇𝑳. current and voltage in phase, 𝑷𝒂𝒗 = 𝑰𝟐 𝑹 = 𝑰𝑽 =
- The energy stored is due to the inductance 𝑽𝟐
. 𝐼 and 𝑉 are rms value.
𝟏 𝑹
(stored in the inductor only). 𝑬 = 𝑳𝑰𝟐 . - Power factor of a purely resistive circuit is
𝟐
2.Resistance and capacitance – Current lead unity or phase angle ∅=0°.
voltage( or voltage lags current) at ∅. The - Graphical representation of the relationship

275
Demystified Series Physics Demystified by Dr Timothy
betweeen reactance,frequency, capacitance than the maximum current. Current is
and inductance – maximum at 𝒇 = 𝒇𝒐 .
1. 𝑋𝐿 = 2𝜋𝑓𝐿 then, 𝑿𝑳 ∝ 𝑳 and 𝑿𝑳 ∝ 𝒇. Inductive 𝑰
reactance is directly proportional to inductance 𝑰𝒎𝒂𝒙
and frequency i.e Inductive reactance increases
with inductance and frequency.
𝑿𝑳

𝒇𝒐 𝒇
- In summary – 1. At frequency lower than
𝑳 or 𝒇 resonance frequency, 𝑓 < 𝑓𝑜 : 𝑋𝐿 < 𝑋𝐶 or 𝑋𝐶 > 𝑋𝐿
2. 𝑋𝐶 =
1 𝟏 𝟏
then, 𝑿𝑪 ∝ and 𝑿𝑪 ∝ . Capacitive or 𝑉𝐿 < 𝑉𝐶 and 𝐼 leads 𝑉 or 𝑉 lags 𝐼.
2𝜋𝑓𝐶 𝑪 𝒇
2. At frequency higher than resonance frequency
reactance is inversely proportional to , 𝑓 > 𝑓𝑜 : 𝑋𝐿 > 𝑋𝐶 or 𝑋𝐶 < 𝑋𝐿 or 𝑉𝐿 > 𝑉𝐶 and 𝐼 lags
capacitance and frequency i.e capacitive 𝑉 or 𝑉 leads 𝐼.
reactance decreases with capacitance and 3. At frequency equal to resonance frequency ,
frequency. 𝑓 = 𝑓𝑜 : 𝑋𝐿 = 𝑋𝐶 or 𝑉𝐿 = 𝑉𝐶 and 𝐼 and 𝑉 are in
𝑿C phase.

Examples :
Use the diagram below to answer question 1 and
𝑪 or 𝒇 2;
- Resonance in 𝑹-𝑳-𝑪 a.c circuit – Resonance is Current(𝐴)
said to occur in an a.c circuit when ; 𝑄
1.Inductive reactance equals capacitive
reactance (𝑋𝐿 = 𝑋𝐶 ) or Net reactance is zero 𝑂 𝑆 𝑇 time(𝑠)
(𝑋 = 𝑋𝐿 − 𝑋𝐶 = 0). 𝑅 𝑈
2.Current and power are maximum.
3.Impendance is minimum and is equal to 1. An alternating current 𝐼 = 𝐼 O𝑠𝑖𝑛𝜔𝑡has the
resistance(𝑍 = 𝑅). waveform shown in the figure above, the
4.Phase angle is zero, ∅ =0° or current and quantity 𝐼 O is equal to A.𝑂𝑇 B.𝑂𝑆 C.𝑸𝑹 D.𝑆𝑈.
voltage are in phase. 𝐼 O is the amplitude or peak current and its
𝑅
5.Power factor is unity, 𝑐𝑜𝑠∅ = = 1. represented by the maxium displacement of the
𝑍
6.Volage across the inductor equals the voltage waveform from the equilibrium position i.e𝑄𝑅.
across the capacitor or inductive voltage equal 2. The length 𝑂𝑈 in the diagram is equal to A.𝜔
capcitive voltage (𝑉𝐿 = 𝑉𝐶 ) and the net voltage is 𝜔 𝜔
B. C. D. .
𝟐𝝅
2𝜋 𝜋 𝝎
equal to the voltage across a resistor (𝑉𝑇 = 𝑉𝑅 ).
The length 𝑂𝑈 is the period 𝑇 i.e time taken to
- At resosnance, the frequency called resonance 2𝜋
frequency 𝒇𝒐 is given by – complete one cycle. 𝑇 = .
𝜔
𝒇𝒐 =
𝟏
, 𝝎𝒐 =
𝟏
. 3. An alternating voltage with a frequency of
𝟐𝝅√𝑳𝑪 √𝑳𝑪 50Hz has a period of A.0.02s B.0.05s C.0.20s
- Graphical representation of the relationship D.0.50s .
between impendance 𝑍, inductive reactance 𝑋𝐿 , 1 1
𝑓=50Hz, period, 𝑇 = = =0.02s.
capacitive reactance 𝑋𝐶 and resistance 𝑅 with 𝑓 50
frequency at resonance : 𝒁, 𝑿𝑳 , 𝑿𝑪 and 𝑹 all 4.
coincides at resonance frequency 𝒇𝒐 .
3𝑇
Resistance 𝑹 is independent of frequency i.e 𝑂
2
not affected by frequency. 𝑇 𝑇 𝑇
𝑇 𝑡(𝑠)
𝒁, 𝑿𝑳 𝑿𝑳 8 4 2
𝑿𝑪 , 𝑹 𝒁 The diagram above illustrates the variation of the
voltage 𝑉 in an a.c circuit with time 𝑡. If the peak
𝑹 value of voltage is 180𝑉 and 𝑇 is the period, what
𝑇
is the instantaneous voltage at time, 𝑡 = ?
8
A.45𝑉 B.90𝑉C.90√𝟐𝑽 D.180𝑉 E.180√2𝑉
𝑿𝑪 𝑇 2𝜋
𝑉𝑜 =180𝑉, 𝑡 = , 𝜔 = , 𝑉=instantaneous
𝒇𝒐 𝒇 8 𝑇
2𝜋
- Graphical representation of the relationship voltage=? 𝑉 = 𝑉𝑜 𝑠𝑖𝑛𝜔𝑡 = 𝑉 O𝑠𝑖𝑛 𝑡,
𝑇
between current and frequency at resonance – At 𝑉 = 180𝑠𝑖𝑛
2𝜋 𝑇
× = 180 sin
𝜋
(𝜋 rad=180°)
frequencies above or below resonant 𝑇 8 4
frequency 𝒇𝒐 , the current is minimum or less
276
Demystified Series Physics Demystified by Dr Timothy
𝑉 = 180𝑠𝑖𝑛
180°
= 180𝑠𝑖𝑛45° (𝑠𝑖𝑛45°= )
1 D.20.10𝑉
2
1 180 180 √2 180√2
√2
The a.c supply with rms value will have the same
𝑉 = 180× = = × = =90√2𝑉. brightness(or heating effect) as the 12𝑉
√2 √2 √2 √2 2
5. The instantaneous value of induced emf as a battery(d.c source).𝑉𝑟𝑚𝑠 =12𝑉, 𝑉𝑜 =?
𝑉
function of time 𝜀 = 𝜀O𝑠𝑖𝑛𝜔𝑡 where 𝜀 O is the peak 𝑉𝑟𝑚𝑠 = 𝑜 , 𝑉𝑜 = 𝑉𝑟𝑚𝑠 √2 =12√2 = 16.97𝑉.
√2
value of emf. The instantaneous value of the emf, 14. In an a.c circuit, a voltage supply of 20𝑉 rms
𝜀 𝜀
one quarter of the period is A. 𝑜 B.𝜺O C.0 D. 𝑜 . is connected to a 5Ω resistor. Find the peak value
4 2
𝑇
𝑡 = , 𝜀 = 𝜀𝑜 𝑠𝑖𝑛𝜔𝑡 = 𝜀𝑜 𝑠𝑖𝑛
2𝜋
𝑡, of the current A.2.50𝐴 B.4.00𝐴 C.6.00𝐴D.5.66𝑨
4
2𝜋 𝑇 𝜋
𝑇
𝑉𝑟𝑚𝑠 =20𝑉, 𝑅=5Ω, 𝐼𝑜 =?, 𝑉𝑟𝑚𝑠 = 𝐼𝑟𝑚𝑠 𝑅,
𝜀 = 𝜀𝑜 𝑠𝑖𝑛 × = 𝜀𝑜 sin (𝜋 rad=180°) 𝑉 20 𝐼
𝑇 4 2 𝐼𝑟𝑚𝑠 = 𝑟𝑚𝑠 = =4𝐴 , 𝐼𝑟𝑚𝑠 = 𝑜 , 𝐼𝑜 =
180° 𝑅 5 √2
𝜀 = 𝜀𝑜 𝑠𝑖𝑛 = 𝜀𝑜 𝑠𝑖𝑛90° = 𝜀O.
2 𝐼𝑟𝑚𝑠 √2 =4√2=5.66𝐴.
6. The peak value of the p.d in an AC circuit is 15. Calculate the amount of heat generated in an
240𝑉. The instantaneous p.d at a phase angle of external load of resistance 8Ω if an a.c of peak
90° is A.240𝑽 B.120𝑉 C.60𝑉 D.0𝑉 value 5𝐴 is passed through it for 100s.
𝜋 1
Phase angle 𝜔𝑡=90° or or cycle, 𝑉 O=240𝑉, A.1.0×104J B.2.0×104J C.5.0×104J D.4×104J.
2 4
𝑉 = 𝑉𝑜 𝑠𝑖𝑛𝜔𝑡 = 240𝑠𝑖𝑛90° = 240𝑉. 𝐼𝑜 =5𝐴, 𝑅=8Ω, 𝑡=100s, 𝐼𝑟𝑚𝑠 =?
𝐼 5
7. In an a.c circuit, the ratio of r.m.s value to peak 𝐼𝑟𝑚𝑠 = 𝑜 = = 3.54𝐴.
𝟏 1 √2 √2
value of current is A.2 B.√2 C. D. . The heat energy, 𝐻 = 𝐼 2 𝑅𝑡 =3.542×8×100 ,
√𝟐 2
𝐼𝑟𝑚𝑠 =
𝐼𝑜
,
𝐼𝑟𝑚𝑠
=
1
. 𝑉𝑟𝑚𝑠 =
𝑉𝑜
,
𝑉𝑟𝑚𝑠
=
1
. 𝐻 = 10025.28 = 1.0×104J.
√2 𝐼𝑜 √2 √2 𝑉𝑂 √2 16. When an a.c voltage is applied to a capacitor,
8. The peak voltage of a 110𝑉 rms mains supply no power is consumed because the current A.and
is A.220𝑉 B.156𝑽 C.120𝑉 D.110𝑉 voltage are 180° out of phase B.and voltage are in
𝑉
𝑉𝑟𝑚𝑠 =110𝑉, 𝑉𝑜 =? , 𝑉𝑟𝑚𝑠 = 𝑜 , 𝑉𝑜 = 𝑉𝑟𝑚𝑠 √2 , phase C.lags behind the voltage by 90° D.leads
√2
𝑉𝑜 =110√2 = 155.56𝑉 =156𝑉. the voltage by 90° D.leads the voltage by 90°.
9. The rms value of an alternating current is 2𝐴. In a purely capacitive circuit, current leads
What is the peak value of the current? A.4.0𝐴 voltage or voltage lags behind current by 90°.
B.3.4𝐴 C.2.8𝑨 D.1.4𝐴. Thus, power dissipated is zero.
𝐼 17. In a purely inductive circuit, the current
𝐼𝑟𝑚𝑠 = 𝑜 , 𝐼𝑜 = 𝐼𝑟𝑚𝑠 √2 =2√2=2.8𝐴.
√2 A.lags behind the voltage in phase by 90°
10. In a radio circuit, the a.c voltage across a B.leads the voltage in phase by 90° C.is in the
resistor is given by 𝑉 =4𝑠𝑖𝑛60𝜋𝑓𝑡. Calculate the same phase with tge voltage D.leads the voltage
root mean square(rms) value of the a.c. A.2√𝟐𝑽 by 180°.
B.3√2𝑉 C.4𝑉 D.6𝑉 . In a purely inductive circuit, current lags behind
Comparing 𝑉 =4sin60𝜋𝑓𝑡 with 𝑉 = 𝑉𝑜 𝑠𝑖𝑛𝜔𝑡, 𝑉 = voltage or voltage leads current in phase by 90°.
𝑉
𝑉𝑜 𝑠𝑖𝑛2𝜋𝑓𝑡, 𝑉𝑜 =4𝑉, 𝑉𝑟𝑚𝑠 = 𝑜 =
4
, Thus, power is dissipated in is zero.
√2 √2 18. In a d.c circuit, a 10microfarad(μF) capacitor
4 √2 4√2
𝑉= × = = 2√2𝑉. is placed in series with a 10Ω resistor. The total
√2 √2 2
11. The voltage of the domestic electric supply is resistance of the combination is A.10Ω B.1Ω C.0Ω
represented by the equation, 𝑉 =311𝑠𝑖𝑛314.2𝑡. D.20Ω E.infinite .
Determine the frequency of the a.c supply In a d.c circuit, current flows in a constant
A.50.00Hz B.100.0Hz C.311.0Hz D.314.2Hz direction,hence it does not oscillate and has no
(𝜋=3.142) . frequency.𝑓=0Hz, 𝑅=10Ω, 𝑋𝐶 =? ,
1 1 1
Comparing 𝑉 =311𝑠𝑖𝑛314.2𝑡 with 𝑉 = 𝑉𝑜 𝑠𝑖𝑛𝜔𝑡 𝑋𝐶 = = −6 = = ∞ or infinite.
2𝜋𝑓𝐶 2𝜋×0×10×10 0
or 𝑉 = 𝑉𝑜 𝑠𝑖𝑛2𝜋𝑓𝑡, angular frequency or angular In a d.c circuit, the total resistance in series,
velocity 𝜔=314.2rads-1. 𝜔 = 2𝜋𝑓 , 314.2 = 2𝜋𝑓, 𝑅𝑇 = 𝑅 + 𝑋𝐶 = 10+∞ = ∞ or infinite.
314.2 314.2
𝑓= = = 50Hz. 19. An a.c 2.5𝐴 passes through a capacitor when
2𝜋 2×3.142
12. The current through a resistor in an a.c circuit the potential difference across its terminals is
is given as 2sin𝜔𝑡. Determine the d.c equivalent 450𝑉. If the angular frequency is 200rads-1,
1
of the current A. 𝐴 B.2√2𝐴 C.2𝐴D.√𝟐𝑨 . determine its capacitance A.2.78μF B.4.42μF
√2 C.14.30μF D.2.80μF.
D.c equivalent of the a.c current is same as 𝑉𝐶 =450𝑉, 𝐼=2.5𝐴, 𝜔=200rads-1, 𝐶=?
effective or rms square value of current. 𝑉 450
𝑉𝐶 = 𝐼𝑋𝐶 , 𝑋𝐶 = 𝐶 = = 180𝑉.
𝐼 = 𝐼𝑜 𝑠𝑖𝑛𝜔𝑡, 𝐼 = 2sin 𝜔𝑡 , 𝐼𝑜 =2𝐴, 𝐼 2.5
1 1 1 1
𝐼𝑟𝑚𝑠 =
𝐼𝑜
=
2
=
2
×
√2
=
2√2
= √2𝐴. 𝑋𝐶 = = , 180 = , 𝐶=
𝜔𝐶 2𝜋𝑓𝐶 200×𝐶 180×200
√2 √2 √2 √2 2
13. An a.c supply lights a lamp with the same 𝐶 = 27.8×10-6F = 27.8μF.
brightness as a 12𝑉. What is the peak voltage of 20. Determine the inductive reactance when
the a.c supply? A.0.12𝑉 B.8.49𝑉 C.16.97𝑽 30.0m𝐻 inductor when a negligible resistance is

277
Demystified Series Physics Demystified by Dr Timothy
connectd to a 1.30×103Hz oscillator A.39.0Ω 26. 𝑋𝐿 =5Ω 𝑋𝐶 =8Ω 𝑅=4Ω
B.122.5Ω C.245.0Ω D.3900.0Ω
𝐿=30m𝐻=3×10-3𝐻, 𝑓=1.30×103Hz, 𝑋L=? 𝐴
𝑋𝐿 = 𝜔𝐿 = 2𝜋𝑓𝐿 = 2×3.142×1.30×103×3×10-3
𝑋𝐿 = 245Ω.
21. 𝐿=0.2𝐻 The reactance of the inductor 110𝑉, 50Hz
shown in the figure is 100Ω. In the 𝑅-𝐿-𝐶 circuit above, the current indicated
The frequency of the A.C by the ammeter 𝐴 is A.22.0𝑨 B.15.7𝐴 C.10.7𝐴
source is D.6.5𝐴 .
500 𝟐𝟓𝟎
A. Hz B. Hz C.50Hz 𝑍2= 𝑅2+(𝑋L−𝑋C)2 or 𝑅 2+(𝑋C−𝑋L)2,
𝜋 𝝅
D.20Hz . 𝑍2= 42+(8−5)2 = 16+9 = 25, 𝑍 = √25 =5Ω,
𝑉 110
𝐿=0.2𝐻, 𝑋𝐿 =100Ω. 𝑓=?, 𝑋𝐿 = 2𝜋𝑓𝐿, 𝑉 = 𝐼𝑍, 𝐼 = = = 22𝐴.
𝑍 5
𝑋 100 250
𝑓= 𝐿 = = Hz. 27. 𝐿 𝐶 𝑅
2𝜋𝐿 2𝜋×0.2 𝜋
60
22. A 120𝑉, Hz a.c source is connected across
𝜋 18𝑉 13𝑉 12𝑉
a 2μF capacitor. Calculate the current passing
through the circuit A.4.9×10-2𝐴 B.2.9×10-2𝑨
C.2.2×10-2𝐴 D.2.0×10-2𝐴. 𝑉, 60Hz
60
𝑉𝑟𝑚𝑠 =120𝑉, 𝑓= Hz, 𝐶=2μ𝐹=2×10-6𝐹, 𝐼𝑟𝑚𝑠 =?, The applied voltage 𝑉 in the 𝑅-𝐿-𝐶 circuit digram
𝜋
1 1 106 shown above is A.43𝑉 B.19𝑉 C.17𝑉 D.13𝑽
𝑋𝐶 = , 𝑋𝐶 = 60 = = 4166.7Ω. 𝑉𝐿 =18𝑉, 𝑉𝐶 =13𝑉, 𝑉𝑅 =12𝑉, 𝑉𝑇 =?
2𝜋𝑓𝐶 2𝜋× ×2×10−6 240
𝜋
𝑉𝑟𝑚𝑠 = 𝐼𝑋𝐶 , 𝐼 =
𝑉𝑟𝑚𝑠
=
240
= 2.9×10-2𝐴. 𝑉𝑇 2 = 𝑉𝑅 2 + (𝑉𝐿 − 𝑉𝐶 )2 = 122 +(18−13)2
𝑋𝐶 4166.67
𝑉𝑇 2 = 144+25 = 169, 𝑉𝑇 = √169 = 13𝐴.
23. 𝐿 The inductance 𝐿 of the
28. A series 𝑅-𝐿-𝐶 circuit has inductance 12m𝐻,
coil in the A.C circuit
capacitance 1.6μF and resistance 1.5Ω. Find the
𝐴 5𝐴 shown above is
time when the amplitude of charge oscillating
A.30.00H B.17.60H
will be 50% of its initial value A.11ms C.10ms
C.4.40H D.0.04H.
500 C.8ms D.9ms.
220𝑉, Hz The charge on the capacitor in a series 𝑅-𝐿-𝐶
𝜋
500
𝑉𝑟𝑚𝑠 =220𝑉, 𝑓= Hz, 𝐼𝑟𝑚𝑠 =5𝐴, 𝐿=?, circuit decreases exponentially with time and is
𝜋 𝑅𝑡
𝑋𝐿 =2𝜋𝑓𝐿, 𝑉𝑟𝑚𝑠 = 𝐼𝑟𝑚𝑠 𝑋𝐿 = 𝐼𝑟𝑚𝑠 (2𝜋𝑓𝐿), given by the equation; 𝑄 = 𝑄𝑜 𝑒 −2𝐿
𝑉
𝐿 = 𝑟𝑚𝑠 =
220
50 = 0.04𝐻. 𝑄=charge after time 𝑡, 𝑄𝑜 =maximum or initial
2𝜋𝑓𝐼𝑟𝑚𝑠 2𝜋×
𝜋
×5 charge, 𝑅=resistance, 𝐿=inductance.
24. 2.5H 25μF 𝑄 =50%𝑄𝑜 =
50 𝑄
× 𝑄𝑜 = 𝑜 , 𝐿=12mH= 0.012H,
100 2
𝑅=1.5Ω, 𝐶=1.6μF=1.6×10-6F,
1.5×𝑡 1.5𝑡
𝑄𝑜 1
= 𝑄𝑜 𝑒 −2×0.012, = 𝑒 −0.024 (take natural
2 2
logarithm i.e In, of both sides).
The AC circuit illustrated in the diagram above 1
In = −
1.5𝑡
, −0.693 = −
1.5𝑡
,
100 2 0.024 0.024
has a frequency of Hz. The effective reactance −0.693×0.024
𝜋 𝑡= = 0.011s = 11ms.
of the circuit is A.700Ω B.500Ω C.300Ω D.200Ω −1.5
100 28. An alternating current with a peak value of
𝐿=2.5𝐻, 𝐶=25μ𝐹=25×10-6𝐹, 𝑓= Hz ,
𝜋 5𝐴 passes through a resistor of resistance 10Ω.
100
𝑋𝐿 =2𝜋𝑓𝐿 = 2𝜋 × × 2.5= 500Ω. Calculate the rate at which energy is dissipated in
𝜋
1 1 106 the resistor A.50W B.125W C.200W D.250W
𝑋𝐶 = = = =200Ω.
2𝜋𝑓𝐶 2𝜋×
100
𝜋
×25×10−6 5000 E.500W.
Effective or net reactance 𝑋 = 𝑋𝐿 − 𝑋𝐶 , Rate at which energy is dissipated = Power.
𝑋 = 500−200 = 300Ω. For a purely resistive circuit, the average power
1
25. In an electrical circuit, an inductor of is given by – 𝑃 = 𝐼𝑟𝑚𝑠 2 𝑅 = 𝐼𝑜 2 𝑅 ,
2
inductance 0.5H and resistance 50Ω is connected 1
𝐼𝑜 =5𝐴, 𝑅=10Ω, 𝑃=?, 𝑃 = × 52×10 ,
to an alternating current source of frequency 2
60Hz. Calculate the impendance of the circuit 𝑃 =25×5=125𝑊.
A.50.0Ω B.150.5Ω C.195.0Ω D.1950.1Ω . 29. In a pure resistive a.c circuit, the current 𝐼 =
𝐿=0.5𝐻, 𝑅=50Ω, 𝑓=50Hz, 𝑋𝐿 =2𝜋𝑓𝐿 , 𝐼𝑜 𝑖𝑛𝜔𝑡 and the voltage 𝑉 = 𝑉𝑜 𝑠𝑖𝑛𝜔𝑡. Caclulate the
𝑋𝐿 = 2×3.142×60×0.5 = 188.52Ω instantaneous power dissipated in the circuit in
𝐼 𝑉
Impendance, 𝑍 2 = 𝑋𝐿 2 + 𝑅2 = 188.522+502 , time 𝑡. A.𝐼𝑜 𝑉𝑜 B.2𝐼𝑜 𝑉𝑜 𝑠𝑖𝑛𝜔𝑡 C. 𝑜 𝑜 D.𝑰𝒐 𝑽𝒐 𝒔𝒊𝒏𝟐 𝝎𝒕
2
𝑍 2 =35539.8+2500 =38039.8, E.𝐼𝑜 2 𝑉𝑜 2
𝑍 = √38039.8, 𝑍 = 195.0Ω.
278
Demystified Series Physics Demystified by Dr Timothy
Instantaneous power, 𝑃 = 𝐼𝑉, where 𝐼 and 𝑉 are 36. A circuit contains 100μ𝐹 capacitor, 10Ω
instantaneous value, resistor and an inductor 𝐿, all connected in series
𝑃 = 𝐼𝑜 𝑠𝑖𝑛𝜔𝑡 × 𝑉𝑜 𝑠𝑖𝑛𝜔𝑡 = 𝐼𝑜 𝑉𝑜 (𝑠𝑖𝑛𝜔𝑡)2 , if the resonant frequency of the circuit is 100Hz,
𝑃 = 𝐼𝑜 𝑉𝑜 𝑠𝑖𝑛2 𝜔𝑡. 1
find the value of 𝐿 A.4𝜋2H B.2𝜋2H C. H D. 𝟐H.
𝟏
2𝜋 𝟒𝝅
30. The power dissipated in an a.c circuit with an
𝐶=100μF=100×10-6F, 𝑅=10Ω, 𝑓𝑜 =100Hz, 𝐿=?
𝑟𝑚𝑠 current of 5𝐴, 𝑟𝑚𝑠 voltage of 10𝑉 and a 1
phase angle of 60° is A.25W B.70W C.120W At resonance, 𝑋𝐿 = 𝑋𝐶 , 2𝜋𝑓𝐿 = ,
2𝜋𝑓𝐶
D.125W. 2𝜋 × 100 × 𝐿 =
1
2𝜋×100×100×10 −6 ,
Average power dissipated in an a.c circuit, is 1 1
given by – 𝑃 = 𝐼𝑉𝑐𝑜𝑠∅, 𝐼and 𝑉 are the 𝑟𝑚𝑠 𝐿= = 𝐻.
2𝜋×100×2𝜋×100×100×10−6 4𝜋2
1
values. 𝐼=5𝐴, 𝑉=10𝑉, ∅=60°, Alternatively, 𝑓𝑜 = ,
2𝜋√𝐿𝐶
𝑃 = 5×10𝑐𝑜𝑠60° = 25W. 1 1
100 = , 200𝜋 = ,
31. When an alternating current given by 2𝜋√𝐿×100×10−6 √𝐿×100×10−6
𝐼 =10𝑠𝑖𝑛(120𝜋)𝑡 passes through a 12Ω resistor, 1 10−4
(200𝜋) = 2 , 4×10 𝜋 =
4 2 ,
the power dissipated in the resistor is A.1200W 𝐿×100×10−6 𝐿
10−4 1
B.600W C.120W D.30W. 𝐿= = 2 H.
4×104 𝜋2 4𝜋
Compare 𝐼 =10𝑠𝑖𝑛(120𝜋)𝑡 with the equation 37. The resonant frequency of an a.c circuit is
𝐼 = 𝐼𝑜 𝑠𝑖𝑛2𝜋𝑓𝑡, 𝐼𝑜 =10𝐴, 𝑅=12Ω, 1000kHz. If each of the capacitance and
𝑉2
1
𝑃 = 𝑃 = 𝐼𝑟𝑚𝑠 2 𝑅 = 𝐼𝑜 2 𝑅 = 𝑅, inductance is reduced by 50%, and no other
2 𝑍
1 changes are made, the resonant frequency will
𝑃= ×102×12 = 100×6 = 600W. become A.250kHz B.750kHz C.1000kHz
2
32. A coil of resistance 3Ω has an inductive D.2000kHz.
reactance of 4Ω. The power dissipated in the coil Initial resonant frequency 𝑓𝑜 =1000kHz,
when connected to an A.C supply of 50𝑉𝑟𝑚𝑠 is 𝐿 is reduced by 50% = 50% =
50
= 0.5.
A.300W B.400W C.500W D.600W. 100
50
𝑅=3Ω, 𝑋𝐿 =4Ω, 𝑉=50𝑉, The circuit is non-purely 𝐶 is reduced by 50% = 50%= = 0.5.
100
resistive i.e 𝐼 and 𝑉 are not phase, the average 𝑓𝑜 =
1
, 𝑓𝑜 ∝
1
=
1
=
1
=
1
=2,
power is given by – 𝑃 = 𝐼𝑉𝑐𝑜𝑠∅ , 2𝜋√𝐿𝐶 √𝐿𝐶 √0.5×0.5 √0.25 0.5
𝑅 𝑅 Thus, the new resonance frequency will be twice
𝑃 = 𝐼𝑉 (𝑐𝑜𝑠∅ = ), 𝑅=3Ω, 𝑋L=4Ω, 𝑍=?, the initial = 2×1000 = 2000kHz. Alterntaively :
𝑍 𝑍
𝑍2 = 𝑋𝐿 2
+ 𝑅2 = 32+42=9+16 =25 , 𝑓𝑜 =
1
, 𝑓𝑜 √𝐿𝐶 =
1
= constant,
2𝜋√𝐿𝐶 2𝜋
𝑍 = √25 = 5Ω , 𝑉𝑟𝑚𝑠 = 𝐼𝑟𝑚𝑠 𝑍, 𝑉𝑟𝑚𝑠 =50𝑉, Initial values : 𝑓𝑜 =1000kHz, 𝐿=𝐿, 𝐶=𝐶 ,
𝑉 50 𝑅
𝐼𝑟𝑚𝑠 = 𝑟𝑚𝑠 = =10𝐴. 𝑃 = 𝐼𝑉 , Final values : 𝑓𝑜 =?, 𝐿=50%𝐿=0.5𝐿, 𝐶=50%𝐶=0.5𝐶,
𝑍 10 𝑍
3
𝑃 = 10×50× = 300𝑊. 1000× √𝐿𝐶 = 𝑓𝑜 × √0.5𝐿 × 0.5𝐶 ,
5
33. The current is a series 𝑅-𝐿-𝐶 circuit attains its 𝑓𝑜 =
1000×√𝐿𝐶
= 1000× √
𝐿𝐶
= 1000× √
1
,
maximum value when the A.impendance is √0.25𝐿𝐶
1
0.25𝐿𝐶 0.25

greater than the capacitive reactance 𝑓𝑜 = 1000× = 2000kHz.


0.5
B.inductive reactance is equal to capacitive 37. In the circuit shown below, calculate the
reactance C.inductive reactance is greater than energy stored in the inductor at resonance.
resistance D.capacitive reactance is less than 0.1H 0.1μF 100Ω
resistance E.inductive reactance is greater than
the capacitive reactance .
At resonance, current is maximum, impendance
is minimum and equal to the resistance,
inductive reactance equals capacitive reactance 100𝑉, 50Hz
and net reactance is zero. A.0.100J B.0.050J C.0.010J D.0.005J.
34. In alternating current circuit at resonance, 𝐿=0.1𝐻, 𝐶=0.1μ𝐹, 𝑅=100Ω, 𝑉=110𝑉, 𝑓=50Hz,
𝜋 𝜋
the angle of lead or lag is A. B.0° C. D.𝜋 . 𝑉
At resonance, 𝑍 = 𝑅, 𝐼 = = =
𝑉 100
=1𝐴
2 3 𝑍 𝑅 100
At resonance, phase angle or angle of lead or lag 1
Energy stored in an inductor, 𝐸 = 𝐿𝐼 2
is 0° i.e current and voltage are in phase. Power 1
2
factor 𝑐𝑜𝑠∅ is unity at resonance. 𝐸 = × 0.1 × 12= 0.05𝐽.
2
35. In a series 𝐿-𝐶 circuit, the inductance and the
capacitance are 0.5H and 20μF respectively. Jamb past questions on simple a.c circuits :
Calculate the resonant frequency of the circuit [1992/45,46,1994/49,1995/48,1998/45,1999/
A.24.2Hz B.36.7Hz C.50.3Hz D.60.5Hz E.80.0Hz . 35,39,482000/36,2001/38,40,2002/46,2004/
𝐿=0.5H, 𝐶=20μF=20×10-6F, 𝑓𝑜 =? 22,29,2006/27,2007/39,46,2009/44,2012/43].
1 1 1
𝑓𝑜 = = −6
= −5
,
2𝜋√𝐿𝐶 2𝜋√0.5×20×10 2𝜋√10
𝑓𝑜 = 50.3Hz.
279
Demystified Series Physics Demystified by Dr Timothy

Chapter 26 – Elementary Atomic Physics


● Atomic theories and discovery of orbital acceleration. 4.The angular momentum
atoms and energy of an electron in any given orbit are
- Dalton’s atomic theory – 1.Matter is quantized. 5.It explains line spectra of hydrogen
composed of discrete units called atoms, which and hydrogen-like atoms with one electron. 6.It
are the smallest, indivisible part of an element. can explain all atomic phenomena, stability of
2.Atoms can neither be created nor destroyed. Its atoms
limitation is seen in radioactivity. 3.Atoms of the - Limitations of Bohr’s model – 1.It cannot
same element are identical in every aspect but explain the spectra lines of atoms with more than
differ from atoms of all other elements. one electron. 2.It provides no insight into what
- Limitations of Dalton’s theory – 1.It does not happens during transition form one orbit to
recognize the constituents of atoms i.e proton, another. 3.An electron moving in one of Bohr’s
electron and neutron. 2.It is limited to circular orbits forms a current loop and should
matter,elements,molecules and atoms. 3.The produce a magnetic moment, but hydrogen atom
discovery of isotopes shows that atoms of the in the ground state has no magnetic moment due
same element are not exactly the same. to orbital motion.
- Thompson’s atomic model or plum-pudding - Electron cloud model – It describes an atom as
or diffuse model – 1.Atoms consists of a sphere consisting of a small massively positive-charged
of positive charge surrounded by electrons. nucleus,with electron moving rapidly round the
2.Electrons were all alike i.e their specific charge nucleus and spending most of its time in high
or charge per mass ratio is constant. 3.Masss of probability region. The density of electron cloud
electron is approximately two thousandth the is greater in the region nearer the nucleus than
mass of hydrogen. the region farther from the nucleus.
- Limitations of Thompson’s model – 1.It - Atomic structure and properties – An atoms
cannot explain the nucleus and its constituents i.e is made up of a tiny missive nucleus and electrons
atomic phenomena. 2.It is limited to atoms and revolving round it.
electrons only. - The nucleus consists of protons and neutrons.
- Rutherford’s atomic model or planetary Particles in the nucleus are called nucleons.
model – 1.Atoms consists of a central positively- - In a neutral atom, the number of protons is
charged core called nucleus with electrons same as the number of electrons.
revolving round it. 2.Electrons are negatively -The mass of protons is same as the mass of
charged and are located around the periphery of neutrons.
the nucleus. 3.Mass of an atoms is concentrated -Neutrons forms a shield around the
in the nucleus. 4.A wide empty gap exists positively charged protons. Hence, the net
between the nucleus of an atom and the orbiting charge on an atom is electrically zero or
electron. neutral.
- Limitations to Rutherford’s model – 1.It -An ion is formed when an atom loses or gains
cannot explain atomic spectra e.g line spectra 2.It electrons.
suggest that electrons in atoms revolves round - An atom of element 𝑋 is represented by 𝐴𝑍𝑋
the nucleus and radiates energy continuously where 𝐴 is the mass or nucleon number and 𝑍 is
and spiral into the nucleus,which is not so. 3.It is the atomic or proton number. 𝐴𝑍𝑋 is called the
limited to unstable atoms i.e where energy is nuclide of 𝑋. The number of neutrons in the atom
radiated continuously. is given by, 𝑛 = 𝐴 − 𝑍.
- Bohr’s atomic model – 1.Electrons occupy
restricted orbit or revolve round the nucleus in ● Energy Quqnatization and line
certain obits or energy level of definite energy. spectra
Electrons are stable and do not emit radiations at - Energy quantization – Electrons occupy
these orbits. 2.Discrete amount of energy is discrete(quantized) energy levels and an energy
absorbed or radiated when electrons move from level can be occupied by only two electrons of
one energy level or orbit to the other i.e electron opposite spins.
gain discrete amount of energy to move to a -The gaps between energy levels are called
higher orbit or energy level and lose same forbidden energy gap i.e they cannot be occupied
amount of energy in form of radiation to return by electrons.
to the lower orbut or energy level. 3.Electron in - Electron in the lowest energy level is said to be
an atom with a single electron e.g hydrogen atom, in the ground state 𝑬O. Excitation of the electron
can occupy any energy level surrounding i.e when it absorbs a certain amount of energy, it
it,without radiating energy as it moves despite moves to a higher energy level called the excited

280
Demystified Series Physics Demystified by Dr Timothy
state. - Ionization energyis the energy change
- Line spectra : Absorption and emission required to remove an electron completely from
- Atomic spectra – When light of certain an atom. Ionization is the transition from the
wavelength(colours) is passed through the ground state 𝐸 O to the highest energy level 𝐸∞ ,
vapour of an atom in the ground state, light in hence Ionization energy is given by,
form of radiation is absorbed and they are 𝑬i= ∆𝑬 = 𝑬∞ − 𝑬𝒐 . Ionization energy is the
excited to a higher energy level.This appears as positive value of the ground state energy.
dark lines in the radiation and the absorbed - The energy of an atom is constant unless the
radiation is called absorption spectrum. An atom is excited and no intermediate energy
atom in the excited state is untable and would change occurs.
readily emit radiation form of light of certain
wavelength(colour) called emission spectrum, Examples :
as they return to their stable ground state. 1. Which of the porperties of atomic models does
Emission spectrum is perculiar to a particular not recognize the consituuents of atom A.Dalton
atoms and is used for the identification of the B.J.J Thompson C.Rutherford D.Bohr .
vapour of an atom. 2. Which of the following agree with Thompson’s
- Line spectra (absorption and emission spectra) model of atom? I.an atom consisted of a sphere of
are of one wavelength(colour) and are exhibited positive charge of electricity with the electrons
by atoms of gase(vapours). Continuous spectrum surrounding the sphere II.electrons were like
are of range of wavelengths(colours) and are cathode rays III.electrons were all alike and have
exhibited by atoms of liquids and solids. the same properties IV.number of electrons per
- The energy of the radiation is given by planck’s atom is constant and is of the order of the atomic
equation weight V.mass of electron was approximately
𝒉𝒄 1
– 𝑬 = 𝒉𝒇 = . ℎ=planck’s constant=6.63×10- of mass of hydrogen i.e nearly two
𝝀 1830
34𝐽𝑠, 𝑓=frequency of radiation, 𝑐=speed of thousandths of hydrogen A.I,II and III B.I,II and V
light=3×108ms-1, =wavelength of radiation. C.I,III and V D.I,II,III and IV E.I,II,III,IV and V.
- Energy level – Energy levels occupied by 3. Which of the following models produce valid
electrons are arranged in an increasing order statements throughout except that it cannot
from bottom to top. The ground state is the first explain all atomic phenomena A.Rutherford B.J.J
energy level - n=1, 𝐸 = 𝐸𝑜 i.e the first line, second Thompson C.Bohr D.Dalton.
energy level - n=2, 𝐸 1 i.e the second line. Thompson’s atomic model explains the
- The energy at any energy level is given by –𝑬𝒏 = properties of electrons but it cannot explain the
𝑬
− 𝒐𝟐 , 𝒏=principal quantum number or number of nucleus and neutrons i.e atomic phenomena.
𝒏 4. In the Rutherford scattering experiment, a
energy level. - If an electron beam of alpha particles was fired at a thin gold
is excited from a lower energy level 𝐸 1 to a higher film with some of the particles being considered
energy level 𝐸 2, it falls back to 𝐸 1 emitting energy deflected. This shows that A.a gold nucleus
which is equal to the energy change or difference contains protons,neutrons and electrons
given by – uniformly distributed in a tiny volume B.the gold
𝒉𝒄
∆𝑬 = 𝑬𝟐 − 𝑬𝟏 = 𝒉𝒇 = . nucleus is positively charged and is
𝝀
- The energy difference or gap between any concentrated in a tiny volume C.the gold
two energy levels decreases, as the energy nucleus emitted alpha particles D.the gold
level increases from ground state to excited nucleus is concentrated in a tiny volume and
state i.e bottom to top. contains alpha particles
𝑬∞ 𝒏=∞ (ionization) In Rutherford’s scattering experiment, alpha
𝑬𝟑 𝒏=4 particles which are positively charged are fired at
𝑬𝟐 𝒏=3 the nucleus of a thin gold film, some of the alpha
𝑬𝟏 𝒏=2 particles which comes in contact with the nucleus
are deflected as the nucleus is positively charged
𝑬𝒐 𝒏=1 (ground state) and concentrated in a tiny volume.
- The ground state energy of hydrogen 𝑬𝒐 = 5. Which of the following is not correct about
−13.6𝒆𝑽. Rutherford’s model of the atom? A.the mass of an
- Transition from higher to lower energy level atom is concentrated B.the nucleus of an atom is
produce emission spectra while transition from positively charged C.the nucleus is surrounded
lower to higher energy level produce absorption by electron cloud D.the model is applicable to
spectra. atoms with only one electron in the other
- Emission of greatest energy is with shortest shell E.the diameter of the nucleus is of order of
wavelength and highest frequency. 𝑬 ∝ .
𝒇 10-15m.
𝝀 It is Bohr’s model that is applicable to atoms with

281
Demystified Series Physics Demystified by Dr Timothy
only one electron in the outer shell e.g hydrogen 14. The nucleons number and the proton number
atom. of a neutral atom of an element 238 and 92
6. Bohr’s model of the atom could explain the respectively. What is the number if neutrons of
A.composition of the nucleusB.distribution of the atom? A.330 B.238 C.146 C.93 D.193
charges in an atom C.penetration of an alpha Nucleon number=mass number =𝐴=238, Proton
particle thorugh a metal foil D.repulsion of an number=atomic number=𝑍=92, Number of
alpha particle through a metal foil E.stability of neutrons=𝑛 = 𝐴 − 𝑍 =238−92, 𝑛 =193.
an atom. 15. Which of the following representations is
Bohr’s model explains that atoms are stable and correct fro an atom 𝑋 with 36 electrons and 40
radiate energy in form of line spectra which are neutrons? A. 4036𝑋 B. 40𝑋 C. 40𝑋 D. 𝟒𝟎𝑿 E. 76𝑋 .
36 76 𝟕𝟔 40

of dicrete values rather than continuous spectra, In a neutral atom, proton number 𝑍 =electron
which denotes instability of an atom. number=36. Neutron number=40, Nucleon
7. Bohr’s atomic model proves most successful number=mass number 𝐴 =40+36=76. The
for the explanation of the I.structur of the neuclide of an atom is represented as 𝐴𝑧𝑋 = 76 40𝑋 .
hydrogen atom II.line spectra of the hydrogen 16. A nuclide represented as 70 32 𝑋 has a neutron-
atom III.multi-electron atoms. Which of the proton ratio of A.0.5 B.0.8 C.1.2 D.1.5
following statements above is/are correct A.I B.II Mass number 𝑍=70, Atomic number 𝐴=32,
C.I and II D.I and III. Number of neutrons 𝑛=𝐴 − 𝑍=70−32=38,
8. Which of the following statements are true in 𝑛 38
Neutron-proton ratio = = =1.875 ≈1.2.
the structure of atoms? A.the atoms of the 𝑍 32
17. Which of the following constituents of an
different elements differ only in the number of
atom moves round the nucleus? A.Electrons
protons and electrons B.the number of protons
B.Neutrons C.Nucleons D.Photons E.Protons.
equals the number of electrons C.atoms are
18. Which of the following groups of particles
constructed according to the plan of our solar
determine the mass of an atom? A.Protons and
system D.all of the above E.A and C only.
electrons B.Neutrons and protons C.Electrons
Atoms of different elements also differ in their
and neutrons D.Neutrons and photons.
number of neutrons. Number of protons equals
19. Which of the following statements about
number of electrons and mass of protons equals
energy levels in atoms is not correct? A.the level
mass of neutrons.Atoms are constructed based
can be represented by horizontal lines where
on the nuclear force binding its constituents
each line indicates a particular energy level
together. Option C is the planetary model by
B.every atom has a characteristic set of energy
Rutherford.
level C.an atom can accept ‘packets’ of energy
9. It is known that a neutron exists in a light
of all sizes D.all energy levels have negative
atomic nucleus. Which of the following also exists
values because the energy of an electron at rest
in the nucleus? A.an electron B.a β-particle D.a
outside an atom is taken as zero
proton E.an atom.
An atom accepts discrete packets of energy called
10. An atom is usually electrically neutral
quanta i.e they are of definite amount of energy.
because it contains equal number of A.protons
20. Calculate the energy of the third level of an
and neutrons B.protons and electrons
atom if the ground state energy is −24.8𝑒𝑉
C.neutrons and electrons D.photons and protons
A.−8.20𝑒𝑉 B.−2.75𝒆𝑽 C.−1.75𝑒𝑉 D.−9.20𝑒
E.photons and electrons.
𝐸 O=−24.8𝑒𝑉, 𝑛=3 i.e third level, 𝐸 3=?
11. The net charge on an atom is zero because 𝐸 24.8 24.8
A.the electrons and the protons that are charged 𝐸 3= − 𝑜2 = − 2 = − = −2.75𝑒𝑉.
𝑛 3 9
are in the nucleus B.the total charge on the 21. In which of the following transitions is the
electrons is equal to the total charge on the largest quantum of energy liberated by an
neutron C.the neutrons form a shield around hydrogen atom when the electron changes
the charged protons D.the number of the levels? [n is the quantum number] A.n=2 to n=1
electrons in an atom is half the number of B.n=1 to n=2 C.n=2 to n=3 D.n=3 to n=2
protons E.A and B. - Energy is absorbed when electron transitions is
The net charge on an atom is zero also because from lower energy level to higher energy level e.g
the total charge on the electrons is equal to the from n=1 to n=2 or n=1 to n=3 or n=2 to n=3.
total charge on the protons. - Energy is liberated when electron transitions is
12. The mass of a nucleus is the A.total number from higher energy level to lower energy level e.g
of its protons and neutrons B.number of its from n=3 to n=1 or n=3 to n=2 or n=2 to n=1.
protons C.mass of its electrons D.total number of - The energy difference or gap between any two
its protons and electrons. energy levels decreases, as the energy level
13. When an atom loses or gains a charge, it increases from ground state to excited state i.e
becomes A.an electron B.a neutron C.a proton bottom to top i.e the energy change (energy
D.an ion. liberated) from, 𝐸 1−𝐸 O> 𝐸 2−𝐸 1> 𝐸 3−𝐸 2. Hence,

282
Demystified Series Physics Demystified by Dr Timothy
the largest quantum energy is liberated when is II.𝐸 2=0.0𝑒𝑉, 𝐸 1=−5.7𝑒𝑉, ∆𝐸 =0−(−5.7) = 5.7𝑒𝑉.
liberated from n=2 to n=1. III.𝐸 2=−2.6𝑒𝑉,𝐸 1=−12.0𝑒𝑉, ∆𝐸 = −2.6−(−12.0),
22. 𝐸∞ 0.0𝑒𝑉 ∆𝐸 = −2.6+12.0 = 9.4𝑒𝑉.
𝐸3 −1.6𝑒𝑉 IV.𝐸 2=−5.7𝑒𝑉,𝐸 1=−12.0𝑒𝑉,∆𝐸 = −5.7−(−12.0),
𝐸2 −3.7𝑒𝑉 ∆𝐸 = −5.7+12.0 = 6.3𝑒𝑉.
𝐸1 −5.5𝑒𝑉 V.𝐸 2=0.0𝑒𝑉,𝐸 1=−12.0𝑒𝑉,∆𝐸=0−(−12.0)=12.0𝑒𝑉
I. 2.6𝑒𝑉 II. 5.7𝑒𝑉 III. 9.4𝑒𝑉 IV. 6.3𝑒𝑉 V. 12.0𝑒𝑉
𝐸O −10.4𝑒𝑉 The emission of smallest energy will produce the
The energy levels in the mercury atom is longest wavelength and smallest frequency and
illustrated in the diagram above. Calculate the 𝑓
vice-versa i.e 𝐸 ∝ . Transition in I, with energy
𝜆
ionization energy of the atom. A.+2.2𝑒𝑉
change 2.6𝑒𝑉 will have the longest wavelength
B.+10.4𝒆𝑽 C.−10.4𝑒𝑉 D.−21.2𝑒𝑉
and shortest frequency, as it has the smallest
Ionization energy, 𝐸 i= 𝐸∞ − 𝐸 O, 𝐸 O=−10.4𝑒𝑉,
energy change.
𝐸∞ =0.0𝑒𝑉, 𝐸 i =0−(−10.4) = +10.4𝑒𝑉. OR
26. Which will produce the emission if highest
Ionization energy is the positive value of the
frequency? A.I B.II C.III D.IV E.V.
ground state energy −10.4𝑒𝑉, 𝐸 i=+10.4𝑒𝑉.
Transition in V, with highest energy change
The table below shows the energy distribution
12.0𝑒𝑉, will produce the highest frequency.
for various levels of an atom. Use it to answer 21
27. The ionization potential of hydrogen is
and 22.
13.6𝑉. Calculate the minimum wavelength of the
Energy 1 2 3 4
radiation the hydrogen atom can emit.
level(n) [electronic charge, 𝑒=1.6×10-19C, Plancks
Energy(𝑒𝑉) −13.6 −3.39 −1.51 −0.85 constant, ℎ=6.63×10-34𝐽𝑠, 𝑐=3.0×108ms-1]
[ℎ=6.6×10-34𝐽𝑠, 𝑒=1.6×10-19C, 𝑐=3.0×108ms-1] A.1.46×10-26m B.2.71×10-24m C.9.14×10-8m
23. Calculate the first excitation energy of the D. 1.09×10-7m.
atom A.1.60×10-19J B.1.02×10-18J C.1.60×10-18J ℎ𝑐
∆𝐸 = 𝑒𝑉 = , ∆𝐸=𝐸 i=ionization energy,
D.1.02×10J. 𝜆
Excitation energy is the energy absorbed when 𝑉=𝑉 i=ionization potential, 𝜆=𝜆i=minimum
electrons move from lower to higher energy wavelength of radiation. The minimum
level. First excitation energy of the atom is the wavelength of the radiation is that which aligns
ℎ𝑐
energy it absorbs when it moves from energy with the ionization potential. 𝑒𝑉 i= ,
𝜆𝑖
level n=1 to energy level n=2. 6.63×10−34 ×3.0×108
1st excitation energy=∆𝐸 = 𝐸 2−𝐸 1, 1.6×10-19×13.6 =
𝜆𝑖
𝐸 2=−3.39𝑒𝑉, 𝐸 1=−13.6𝑒𝑉, 6.63×10−34 ×3.0×108
𝜆 I= = 9.14×10-8m.
∆𝐸 = −3.39−(−13.6) = −3.39+13.6 =10.21𝑒𝑉. 1.6×10−19×13.6
[1𝑒𝑉 = 1.6×10-19J] , ∆𝐸 = 10.21×1.6×10-19 , ● Question Drill – Calculate the wavelength of
∆𝐸 = 1.63×10-18J ≈ 1.60×10-18J. the radiation emitted when an electron falls from
24. If the atom de-excites from n=2 the an energy level of 0.43𝑒𝑉 to −13.6𝑒𝑉.
wavelength of the emitted radiation is A.2.4×10- [1𝑒𝑉=1.6×10-19𝐽, ℎ=6.63×10-34𝐽𝑠, 𝑐=3.0×108ms-
1] Ans=8.86×10-8m.
7m B.1.2×10-7m C.2.4×10-9m D.1.2×10-9m.

𝐸 2−𝐸 1=1.63×10-18J, ℎ=6.6×10-34Js, 𝑐=3.0×108 28. In a model of the hydrogen atom, the energy
𝑅
ms-1, 𝜆=wavelength. If the atom de-excites from levels Wn are given by Wn= − 2, 𝑛 is an integer
𝑛
n=2 it comes to n=1, the photon energy ∆𝐸 : and 𝑅 is a constant. Determine the energy
ℎ𝑐 6.6×10−34 ×3×108 𝑅 𝑅
∆𝐸 = 𝐸 2−𝐸 1= . 1.63×10-18 = , released in transition from 𝑛=3 to 𝑛=2. A. B.−
𝜆 𝜆 5 4
6.6×10−34 ×3×108 𝟓𝑹 5𝑅
𝜆= = 1.2×10-7m. C. D.− .
1.63×10−18 𝟑𝟔 36
𝑅 𝑅 𝑅 𝑅
- Photons are packets of light energy. W 3= − 2 = − , W 2= − 2 = − ,
3 9 2 4
The diagram below illustrates the energy Energy released = ∆𝐸 = 𝐸 3−𝐸 2 = W3−W2
transition of five electrons of an atom. 𝑅 𝑅 𝑅 𝑅 −4𝑅+9𝑅 5𝑅
∆𝐸 = − −(− ) = − + = = .
𝑒 𝑒 𝑒 0.0𝑒𝑉 9 4 9 4 36 36
I 𝑒 −2.6𝑒𝑉 29. A photon of wavelength 𝜆O is emitted when
II III 𝑒 −5.7𝑒𝑉 an electron in an atom makes a transition form
IV V −12.0𝑒𝑉 level of energy 2𝐸 K to that of 𝐸 K. If the electron
4
transits from 𝐸 K to 𝐸 K level, determine the
3
Use the diagram to answer questions 23 – 24 wavelength of the photon that would be emitted
25. Which of the transitions will produce the 3 1
A.3𝝀o B.2𝜆o C. 𝜆o D. 𝜆o .
emission of longest wavelength? A.I B.II C.III D.IV ℎ𝑐
4 3
E.V ∆𝐸 = ℎ𝑓 = , 𝜆∆𝐸 = ℎ𝑐 = constant,
𝜆
Energy change, ∆𝐸 = 𝐸 2−𝐸 1 4
𝜆𝑜 (2𝐸𝐾 − 𝐸𝐾 ) = 𝜆( 𝐸𝐾 − 𝐸𝐾 ), 𝜆𝑜 𝐸𝐾 = 𝜆
4𝐸𝐾 −3𝐸𝐾
,
I. 𝐸 2=0.0𝑒𝑉, 𝐸 1=−2.6𝑒𝑉, ∆𝐸 =0−(−2.6) =2.6𝑒𝑉. 3 3

283
Demystified Series Physics Demystified by Dr Timothy
𝜆𝑜 𝐸 K =
𝜆𝐸𝐾
, 𝜆=
3×𝜆𝑜 𝐸𝐾
= 3𝜆𝑜 . melting point, low work function and high
3 𝐸𝐾
mechanical strength.
31. When light from a source is sent through a - Applications of thermionic emission –
gas I.certain wavelengths are absorbed II.an 1.Thermionic diodes are ‘electrical valves’ used
emission line spectrum results III.a dark line is for current rectification i.e conversion of a.c
left. Which of the following is/are correct? A.I to d.c or allowing current to flow in one
B.III C.I and II D.I and III E.II and III. direction only.
A light sent through a gas produces an absorption 2.Discharge tube – It produces cathode rays
spectrum i.e certain wavelengths (colours) are which are stream of fast moving electrons when
absorbed and a dark line results. It’s a result of the cathode is excited.
excitation of an electron in an atom causing - Characteristics of cathode rays :
transion of the electron from lower to higher i.They are streams of fast moving electrons
energy level. which are negatively charged, travelling in a
32. The atoms of a gas are excited to produce a straight line. They are attracted towards the
discharge which is examined with a anode,positive plate of an electric field or 𝑁-pole
spectrometer. The examination reveals A.a of a magnetic field.
totally dark background B.the colours of the ii.They cause greenish glow or flouorescence in
rainbow C.an emission spectrum D.an glass or zinc sulphide.
absorption spectrum E.a continuous spectrum. iii.They have mass, energy and momentum,
Emission spectrum is produced when a gas atom hence will turn a light paddle wheel in the tube.
is excited by a high voltage to produce a iv.They are highly energetic particles and can
discharge, hence returning from higher energy produce intense heat on objects which stop them.
level to lower energy level. Excitation without They can affect photographic plates.
any discharge is an absorption spectrum and the v.They can produce x-rays from high density
background will be totally dark. metals e.g tungsten, when stopped by it.
33. The minimum energy required to remove an 3. Cathode ray oscilloscope or cathode ray
electron from a given atom is known as A.photon tube – It is used for studying alternate current
energy B.ionization energy C.excitation energy waveforms, direct measurement of voltage and
D.binding energy E.kinetic energy. time and to measure the frequencies and
34. The following statements relate to atomic amplitudes of voltages of electronic devices e.g
spectrum I.all elements emit and absorb television picture tubes and computer
characteristic spectra II.spectra analysis is displays.
important method of identifying environmental - It contains of – 1.Electron gun – this consists of;
pollutants III.the chemical composition of stars heated filament, cathode and anode. The anode is
could be determined using spectra analysis. made more positive with respect to the cathode
Which of the statements above are correct? A.I inorder to attract electrons emitted from the
and II B.I and III C.II and III D.I,II and III. cathode and it also acts as a focusing lens to
accelerate and focus electron onto a spot.
● Thermionic emission, X-ray 2.Deflector plates 𝑋𝑋’ and 𝑌𝑌’ – they control the
production and Photoelectric path of electron beam, when an alternating p.d is
emission or phtotoelectricity applied across the plates. The 𝑋𝑋’ pair or 𝑋-
- Thermionic emission or Hot cathode plates deflects the beam horizontally while the
emission – It is the emission of electrons from a 𝑌𝑌’ pair oor 𝑌-plates deflects it horizontally.
metallic surface when heated in a vacuum. 3.Fluorescent screen.
- Emission of electron from the anode of a Similarities between liquid vaporization and
thermionic valve is called secondary emission. thermionic emission :
- Heating a valve cathode causes thermal 1.Both processes involves heat.
emission from the cathode. 2.Both occur at the surface of a material.
-It is the conversion of heat to electrical 3.The rate of both phenomenons increases with
energy. temperature.
- Anode is made more positive than the cathode 4.Saturation point of particles i.e the particles
i.e anode voltage increases as cathode current leaving the surface equals those returning to it, is
decreases. reached by both processes.
- The number of electrons emitted per second or Difference between liquid vaporization and
rate of emission of electrons 𝑅𝑇ℎ depends on the thermionic emission :
temperature 𝑇, mass 𝑀, area 𝐴 and work function 1.Vaporization occurs at liquid surface while
𝑊 of the cathode, 𝑹𝑻𝒉 ∝
𝑨𝑻𝑴
. themionic emission occurs at metallic surface.
𝑾 2.Vaporization occurs at definte temperation i.e
- A good hot cathode should have a high boiling point while thermionic emissionmay not

284
Demystified Series Physics Demystified by Dr Timothy
necessarily occur at a fixed temperature. metal surface or material of the photocathode
3.Water molecules are released in vaporization e.g alkali metals (group I) such as sodium,
while electrons are emitted in thermionic caesium e.t.c, emits electrons by visible light
emission. while other metals requires ultraviolet lights.
- X-rays or Röentgen rays – They are - Threshold frequency depends on work
electromagnetic waves and have no charges. function.
They pass thorugh magnetic or electric fields - Photoelectrons are emitted from a metallic
undeflected. The production of X-rays is the surface only when :
reverse of photoelectric effect. i.the frequency of the incident light/photon is
- Characteristics of X-rays ; greater that the threshold frequency of the metal
1.They are produced when fast moving electrons ii.energy of the incident light is greater than the
strike a metal surface or when electrons are workfunction of the metal
accelerated to a heavy target e.g tungsten in a iii.the wavelength of the incident light is smaller
vacuum. than the threshold wavelength of the metal.
2.They originate from electron change in an atom - Einstein’s photoelectric equation –
and X-ray spectra have similar patterns in all 𝑬𝒊 = 𝑾𝒐 + 𝑲. 𝑬𝒎𝒂𝒙 , 𝒉𝒇 = 𝒉𝒇𝒐 + 𝑲. 𝑬𝒎𝒂𝒙
metals, as it involves the innermost electrons 𝐸𝑖 =energy of incident radiation, 𝑊 o=work
close to the nucleus. Optical or atomic spectra are function or surface energy of the metal
different for different metals as it involves the surface,𝐾. 𝐸𝑚𝑎𝑥 =maximum kinetic energy of
outermost electrons. emitted photoelectrons.
3.They are difrracted by crystals as their - The number of emitted photoelectrons per
wavelength is of same order of magnitude as second or rate of emission of photoelectrons
atomic spacings. depends on the intensity of incident light.
𝑛
4.X-rays are best absorbed by thick lead blocks. - The number of photoelectrons per seconds , is
𝑡
5.They produce ionization of gases and are equal to the ratio of the photocurrent and
detected by their effect on a photographic plate 𝐼 𝒏 𝑰
electronic charge . = .
or fluorescent screen. 𝑒 𝒕 𝒆
6.Number of electrons emitted per second or rate - The maximum kinetic energy of emitted
of emission depends on the intensity while the electrons depends on :
intensity depends on the hot filaments current. I.frequency or wavelength of incident light or
𝒇
7.The velocity of the electrons increases with photon i.e 𝑲. 𝑬𝒎𝒂𝒙 ∝ .
𝝀
increase in the applied p.d or voltage, 𝑣 ∝ 𝑉. II.Colour of incident photon or light i.e colour of
8.Hard X-rays have high penetration power, high light is dependent on wavelength. e.g the
frequency and short wavelength while Soft X- wavelength of the colours of light decreases in
rays have low penetration power, low frequency the order ; 𝑅𝑂𝑌𝐺𝐵𝐼𝑉, hence the kinetic energy of
and long wavelength. emitted photoelectrons increases in that same
- X-rays are used – I.for identifying the internal order. Red light will produce photelectrons of
structure of solids. II.in medicine for examining lesser kinetic energy than orange light or green
bones,teeths and various internal organs. III.in light will photoelectron produce a greater kinetic
industry for revealing flaws in metals and welded energy than red light.
joints. III.Work function or nature of the metal surface.
- Photoelectric emission or photoelectric IV.Applied p.d by the cell, 𝑲. 𝑬𝒎𝒂𝒙 ∝ 𝑽.
effect – It is the emission of electrons from the - The workdone to accelerate the emitted
surface of a metal by light of appropriate photoelectron from the cathode towards the
frequency and wavelength or emission of anode by an applied p.d 𝑉 is ; 𝑾 = 𝒆𝑽. The rate
electrons due to elastic collision between at which work is done to accelerate the
incident photon and electrons in an atom of a phtotelectron or electrical power of emitted
material. It is the conversion of light energy to photoelectron is ; 𝑷 = 𝑰𝑽. 𝐼=photocurrent,
electrical energy. 𝑒=electronic charge=1.6×10-19C, 𝑉=applied p.d.
- Work function or surface energy of a metal - The minimum reverse potential required to
𝑾𝒐 or ɸ is the minimum amount of energy stop the emission of photoelectrons is stopping
required to liberate photoelectrons from a potential or cut off voltage 𝑽𝒐 . It is equivalent
metallic surface. to the maximum kinetic energy of
- Threshold frequency 𝒇𝒐 and wavelength 𝒐 is 𝟏
photoelectrons. 𝑲. 𝑬max= 𝒎𝒗𝒎𝒂𝒙 = 𝒆𝑽𝒐 .
𝟐
the minimum amount of frequency wavelength 𝟐
required to liberate photoelectrons. - Stopping potential depends on the
𝒉𝒄 frequency of the incident radiation , 𝑽𝒐 ∝ 𝒇
𝑾𝒐 = 𝒉𝒇𝒐 = .
𝒐 but independent on the intensity of the
- Work function depends on the nature of the incident radiation.

285
Demystified Series Physics Demystified by Dr Timothy
- Graphical representation of Einstein’s protons, current C.electrons, anode, current
photoelectric equation – D.protons, cathode, electrons E.protons,
1. 𝑲. 𝑬𝒎𝒂𝒙 electrode, current.
5. Cathode rays I.consists of tiny particles
bearing negative electric charges II.consist of
𝑾𝒐 𝒇o 𝒇 fast-moving protons and are deflected in a
𝑲. 𝑬𝒎𝒂𝒙 = 𝒉𝒇 − 𝑾𝒐 : 𝑦 = 𝑚𝑥 + 𝑐 magnetic field III.are deflected in an electric field
Slope of the graph m = planck’s constant 𝒉. but not in a magnetic field IV.can cause the
Intercept on the 𝐾. 𝐸𝑚𝑎𝑥 -axis = Work fuction emission of x-rays on hitting a metal target.
𝑾𝒐 . Intercept on the 𝑓-axis=Threshold Which of the above statements are correct? A.I
frequency 𝒇𝒐 . and II B.III and IV C.I and IV D.II and III .
𝑽o Cathode rays are streams of fast-moving
electrons and are deflected in both a magnetic
and electric field.
6. Which of the following statements about
𝒇o 𝒇 cathode rays is not correct? A.they are bent by a
𝑾𝒐 strong field B.they are stream of electrons C.they
𝒆
are bent away by a magnetic field D.they casue
𝑲. 𝑬𝒎𝒂𝒙 = 𝒉𝒇 − 𝑾𝒐 , 𝒆𝑽𝒐 = 𝒉𝒇 − 𝑾𝒐 ,
𝒉 𝑾 𝒉 fliorescence E.they are more penetrating than
𝑽𝒐 = (𝒇) − 𝒐 . Slope of the graph 𝑚 = , gamma rays
𝒆 𝒆 𝒆
Intercept on the 𝑉𝑜 -axis=
𝑾𝒐
. Intercept on the 𝑓- Gamma rays have the highest penetration power.
𝒆
7. A stream of cathode rays is projected into an
axis= 𝒇𝒐 .
electric field between two plates 𝑃 and 𝑄 as
𝑰
shown in the diagram
𝑃 +

𝑄 −
𝑽𝒐 𝑽
Which of the following is correct? The cathode
The graph shows the relationship between
rays A.are accelerated and continued in a
photocurrent 𝐼 and applied p.d 𝑉 at constant
straight line B.change direction at the limits of
intensity of incident photon or light. Intercept on
the plate C.are deflected towards the positive
the negative 𝑉-axis = stopping potential 𝑉𝑜
plate D.are directed towards the negative plate
- Application of photoelectric emission or
E.are deflected normal to the paper.
photoelectric effect – e.g photocell
Cathode rays are deflected towards the positive
I.Setting off of burglar alarms. II.Production of
plate of an electric field, they are stream of fast-
sound from film tracks and in photography to
moving electrons which are negatively charged.
control light intensity. III.Control switches in
8. The cathode ray oscilloscope can be used for
automatic doors or signal devices. IV.In
the direct measurement of I.resistance II.voltage
photocells. V.Photographic light merters.
III.time IV.capacitance A.I and II B.I and III C.II
and III D.II and IV E.III and IV.
Examples :
9. In a cathode ray tube, the function of the X-
1. The process trough which free electrons leave
plates is to A.deflect the electron beam
the surface of a hot metal is known as
horizontally B.deflect the electron beam
A.thermionic emission B.photo emission
vertically C.reflect the electron beam D.generate
C.photon emission D.electron emission.
suinusoidal waves.
2. Which of the following is most essential for the
10. Which of the following is not true about X-
production of electrons by thermionic emission?
rays? A.they can destroy living cells B.they are
A.electron lens B.deflector plate C.fluorescent
electromagnetic waves C.they are produced
screen D.hot cathode E.tungsten target .
when fast moving electrons bombard metal
3. Thermionic emission can occur when A.metal
surfaces D.they are deflected by the magnetic
is bombarded by alpha particles B.certain
field E.they travel at the speed of light.
metals are heated to a very high temperature
11. Which of the following is not applicable to X-
C.X-rays fall on a hot surface D.ultra-violet light
rays? They A.travel in straight lines B.travel with
falls on ametal plate.
the speed of light C.are deflected by magnetic
4. Which of the following sets will correctly
field D.affect photographic plates.
complete the statement below in the order given?
X-rays are electromagnetic waves produced
In a diode, a heated filament emits …… which may
when charged particles are accelerated.They
be attracted by the …… causing ….. to flow.
travel in a straight line and move with the speed
A.electrons, cathode, thermions B.electrons,

286
Demystified Series Physics Demystified by Dr Timothy
of light, they are not deflected by a magnetic or 20. If light with photon energy 3𝑒𝑉 is incident
electric field. suitably on the surface of a metal with work
12. The process of detecting a pin mistakenly function 4𝑒𝑉, then A.no electron is emitted
swallowed by a child is x-ray A.diagnosis B.the few electrons emitted will have a maximum
B.therapy C.crystallography D.mammography kinetic energy of 1𝑒𝑉 C.the few electrons emitted
Objects can be detected by diffraction of x-rays will have maximum kinetic energy 3𝑒𝑉 D.many
through crystals. eelctrons will be emitted.
13. Which type of radiation does not originate Photon energy=energy of incident photon
within the nucleus A.Alpha B.Beta C.Gamma 𝐸 i=3𝑒𝑉, Workfunction=surface energy
D.Neutron E.X-rays . 𝑊 o=4𝑒𝑉. No photoelectron is emitted as the
X-rays orginate electron change in the innermost energy of incident photon(3𝑒𝑉) is less than the
shell of metals. work function of the metal(4𝑒𝑉).
14. X-rays cannot be used A.to take photographs 21. Which of the following statements is/are not
of bone structure in the body B.to detect finger true about the emission of photoelectrons from
prints C.to detect flaws in metal castings D.to metallic surfaces? I.All metallic surfaces emit the
detect alterations to work of art . same number of photoelectron in a given light
Finger prints are transparent to X-rays i.e X-rays II.Most metallic surfaces emit the same number
can easily pass through finge prints. of photoelectrons in visible and infra red light
15. Which of the following elements is the best III.A metallic surface emits photoelectrons only if
absorber of X-rays? A.hydrogen B.oxygen the frequency of the incident light is higher than
C.calcium D.copper E.lead . its own characteristics threshold frequency. A.I
X-rays are absorbed by lead, due to its high B.II C.III D.I and II E.II and III .
density. Statement I and II are not true because the
16. In an operating X-ray tube, a high p.d is number of photoelectrons emitted from a
applied between the cathode and the anode for metallic surface depends on the intentsity of the
the purpose A.heating the cathode B.producing a radiation or incident photon and not on the metal
stream of electrons C.concentrating electrons on surface i.e work function and threshold
the target D.accelerating the electrons to a frequency of the metal. Only statement III is true.
very high energy E.heating the anode. 22. The kinetic energy of a photoelectron ejected
17. Which of the following phenomena is called from a metal surface illuminated with radiation
phtotelectric effect? A.high energy electons depends on the I.wavelength of the radiation
impinge on a metallic anode which then emits II.intensity of the radiation III.source of the
photons B.a high energy photon emits proton as radiation IV.nature of surface. Which of the aove
it is slowed down C.a metal absorbs quanta of statements is/are correct? A.I B.I and IV C.II and
light and then emits electrons D.two electrons III D.I,II and IV .
are created from quantum of light . The maximum kinetic energy of photoelectron
Photoelectric emissiom is the emission of ejected from a metal surface depends on the
electrons from a metal surface, when quantum or wavelength of radiation/colour of incident
packets of light called photons are incident on it. light,frequency of radiation and nature of the
18. For photoelectric emission, the threshold surface i.e work function and threshold
frequency is the A.the maximum frequency of frequency. It does not depend on the intensity of
incident radiation that will eject electron radiation or source of radiation.
B.minimum frequency of incident radiation 23. The threshold frequency for photoelectric
required to eject electrons C.frequency which effect depends on the A.frequency of incident
results in the emission of the greater number of light B.intensity of incident light C.p.d between
electrons D.frequency which imparts greatest the cathode and the anode D.material of the
kinetic energy. photocathode
19. In photoelectricity electrons will leave the Threshold frequency depends on work function
metal surface when illuminated by light of which is a characteristic of the metallic surface i.e
appropriate frequency, if photon energy is photocathode.
A.greater than work function B.less than the 24. In photoelectric effect, the number of
work function C.equal to the work function electrons emitted per second from a metallic
D.equal to the maximum kinetic energy of the surface is proportional to the A.intensity of the
electrons . incident radiation B.frequency of incident
Inorder for photoelectrons to be emitted from a radiation C.energy of the incident radiation
metal surface ; the energy of incident photon D.work function of the metal .
must be greater than the work function or the The rate of emission of photoelectrons or
frequency of incident photon is greater than the number of emitted photoelectrons per second
threshold frequency. depends on the intensity of incident photon.

287
Demystified Series Physics Demystified by Dr Timothy
25. A metallic surface is exposed in turn to green, 6.7×1014Hz is A.1.42×10-19𝑱 B.3.00×10-19𝐽
red and violet lights. The maximum kinetic C.4.42×10 𝐽D.7.42×10-19𝐽[ℎ=6.60×10-34𝐽𝑠]
-19

energy of photoelectrons ejected from the 𝑊 o=3×10-19𝐽, 𝑓=6.7×1014Hz, ℎ=6.60×10-34𝐽𝑠,


surface increases in the order A.green, red, violet 𝐸𝑖 = 𝑊𝑜 + 𝐾. 𝐸𝑚𝑎𝑥 , 𝐾. 𝐸𝑚𝑎𝑥 = 𝐸𝑖 − 𝑊𝑜 ,
B.violet, green, red C.red, green, violet D.violet, 𝐾. 𝐸𝑚𝑎𝑥 = ℎ𝑓 − 𝑊𝑜 [𝐸𝑖 = ℎ𝑓]
red, green. 𝐾. 𝐸𝑚𝑎𝑥 =6.60×10-34×6.7×1014−3×10-19 ,
The wavelength of the colours of light in visible 𝐾. 𝐸𝑚𝑎𝑥 =4.422×10-19−3×10-19=1.42×10-19J.
spectrum decreases in the order – 32. Light of energy 5.0𝑒𝑉 falls on a metal of work
red,orange,yellow,green,blue,indigo and violet function 3𝑒𝑉and electrons are liberated. The
[𝑅𝑂𝑌𝐺𝐵𝐼𝑉], red as the highest wavelength and stopping potential is A.15.0V B.8.0𝑉 C.2.0𝑽
violet has the lowest wavelength, hence the D.1.7𝑉.
maximum kinetic energy of electrons increases 𝐸𝑖 =5𝑒𝑉, 𝑊𝑜 =3𝑒𝑉, 𝐾. 𝐸𝑚𝑎𝑥 =?, 𝐾. 𝐸𝑚𝑎𝑥 = 𝐸𝑖 − 𝑊𝑜 ,
1
in that same order as 𝐸 ∝ . The maximum 𝐾. 𝐸𝑚𝑎𝑥 = 5−3 = 2𝑒𝑉, 𝐾. 𝐸𝑚𝑎𝑥 = 𝑒𝑉𝑜 , stopping
𝜆 𝐾.𝐸 2𝑒𝑉
kinetic energy of photeelctrons increases in the potential 𝑉𝑜 =?, 𝑉𝑜 = 𝑚𝑎𝑥 = = 2𝑉.
𝑒 𝑒
order red, green and violet. 33. Light of wavelength 5.00×10-7m is incident
26. Which of the following statements about on a material of work function 1.90𝑒𝑉. Calculate
photoelectrons is correct? A.A faint green light the stopping potential A.0.477𝑉 B.0.457𝑉
produces photoelectrons with less kinetic energy C.0.575𝑽 D.0.375𝑉 [ℎ=6.60×10-34J𝑠,
than a bright red light B.A red light releases 𝑐=3×108ms-1, 1𝑒𝑉=1.6×10-19J, 𝑒=1.6×10-19C]
smaller number of electrons than a green light 𝜆=5.00×10-7m, 𝑊 o=1.90𝑒𝑉=1.90×1.6×10-19,
C.A faint green light produces photoelectrons 𝑊 o=3.04×10-19𝐽,𝑐=3×108ms-1, ℎ=6.60×10-34𝐽𝑠,
with greater kinetic energy than a bright red electronic charge, 𝑒=1.6×10-19C, 𝐸𝑖 = ,
ℎ𝑐
𝜆
light D.A red light produces more photoelectrons ℎ𝑐
than a green light. 𝐾. 𝐸𝑚𝑎𝑥 = 𝐸𝑖 − 𝑊𝑜 , 𝑒𝑉𝑜 = 𝐸𝑖 − 𝑊𝑜 = − 𝑊𝑜 ,
𝜆
27. The threshold frequency for a metal surface 6.6×10−34 ×3×108
1.6×10-19× 𝑉𝑜 = −3.04×10-19,
5×10−19
is 3.0×1014Hz, the work function for the surface
1.6×10 𝑉𝑜 =3.96×10 −3.04×10-19,
-19 -19
is? A.4.54×1048J B.3.00×10-16J C.2.00×10-18J
1.6×10-19𝑉𝑜 = 0.92×10-19,
D.1.98×10-19J. [Take Planck’s constant=6.60× 0.92×10−19
10-34Js] . 𝑉𝑜 = = 0.575𝑉.
1.6×10−19
𝑓𝑜 =3.0×1014Hz, ℎ=6.60×10-34𝐽s, 𝑊𝑜 =?, 33. The energy 𝐸 of a photon and its wavelength
Work function or surface energy 𝑊𝑜 = ℎ𝑓𝑜 , are related by 𝐸𝜆 = 𝑋. The numerical value of 𝑋
𝑊𝑜 = 6.60×10-34×3×1015 = 1.98×10-19J. is A.6.60×10-26 B.1.99×10-25 C.1.99×10-27
28. The work function of a metal is 2.7𝑒𝑉. Which D.6.60×10-28 [ℎ=6.63×10-34𝐽𝑠, 𝑐=3×108ms-1]
of the following pairs correspond to the 𝐸=
ℎ𝑐
, 𝐸𝜆 = ℎ𝑐, 𝑋 = 𝐸𝜆 = ℎ𝑐,
threshold frequency and wavelength of the metal 𝜆
𝑋 = 6.63×10-34×3×108 = 1.99×10-25 .
respectively? [ℎ=6.60×10-34𝐽𝑠, 𝑐=3×108ms-1,
34. An electron is accelerated from rest through
1𝑒𝑉=1.6×10-19𝐽] A.4.6×10-26Hz, 6.5×1034m
a potential difference of 18.2k𝑉 in vacuum. If
B.4.6×10-7Hz, 6.5×1014m C.6.5×1014Hz,
electronic charge=1.6×10-19C and mass of
4.6×10-7m D.6.5×1014Hz, 4.6×107m
electron=9.1×10-31kg, the maximum speed
E.6.5×1034Hz, 4.6×10-26m
acquired by the electron is A.4.00×107ms-1
ℎ=6.60×10-34𝐽𝑠, 𝑐=3×108ms-1, 𝑊𝑜 =2.7𝑒𝑉,
B.5.66×107ms-1 C.8.00×107ms-1
1𝑒𝑉=1.6×10-19J, 𝑊𝑜 =2.7×1.6×10-19=4.32×10-19J,
ℎ𝑐 𝑊𝑜 4.32×10−19
D.1.13×108ms-1.
𝑊𝑜 = ℎ𝑓𝑜 = , 𝑊 o= ℎ𝑓𝑜 , 𝑓o= = , Kinetic energy of electron=Work done on
𝜆𝑜 ℎ 6.6×10−34
1
𝑓o = 6.5×1014Hz. 𝑊𝑜 =
ℎ𝑐
, 𝜆𝑜 =
ℎ𝑐
, electron, 𝐾. 𝐸𝑚𝑎𝑥 = 𝑊, 𝑚𝑣 2 = 𝑒𝑉,
𝜆𝑜 𝑊𝑜 2
6.6×10−34 ×3×108 𝑚=mass of electron=9.1×10-31kg, 𝑣=maximum
𝜆𝑜 = = 4.6×10-7m. speed of electron=? 𝑒=electronic charge or
4.32×10−19
29. A metal having a work function of 4.65𝑒𝑉 is charge on electron=1.6×10-19C, 𝑉=accelerating
illuminated with a radiation of 6.86𝑒𝑉. The p.d=18.2k𝑉=18.2×103𝑉,
kinetic energy of the electrons emitted from the 1
×9.1×10-31× 𝑣 2 =1.6×10-19×18.2×103,
metal surface is A.2.21𝒆𝑽 B.4.24𝑒𝑉 C.6.82𝑒𝑉 2
4.55×10-31𝑣 2 =29.12×10-16,
D.11.51𝑒𝑉 29.12×10−16
Energy of incident phton, 𝐸 i=6.86𝑒𝑉, Work 𝑣2 = = 6.4×1015,
4.55×10−31
function, 𝑊 o=4.65𝑒𝑉, maximum kinetic energy, 𝑣= √6.4 × 1015 =8.00×107ms-1.
𝐾. 𝐸 max=? 𝐸𝑖 = 𝑊𝑜 + 𝐾. 𝐸𝑚𝑎𝑥 , 𝐾. 𝐸𝑚𝑎𝑥 = 𝐸𝑖 − 𝑊𝑜 35. If the frequency of an emitted X-ray is
𝐾. 𝐸𝑚𝑎𝑥 = 6.86−4.65 = 2.21𝑒𝑉. 1.6×1016Hz, the accelerating potential is A.6.6𝑉
31. Caesium has a work function of 3×10-19𝐽. The B.66.3𝑽 C.663.0𝑉 D.6630.0𝑉
maximum kinetic energy of liberated electrons [𝑒=1.6×10-19C, ℎ=6.63×10-34Js]
when it is illuminated by light of frequency of

288
Demystified Series Physics Demystified by Dr Timothy
Workdone in accelerating electrons, 𝑊 = 𝑒𝑉, particles are deflected or deviated in a
Energy of 𝑋-rays 𝐸 = ℎ𝑓, 𝑊 = 𝐸, ℎ𝑓 = 𝑒𝑉, magnetic or electric field. Gamma rays,
𝑉= =
ℎ𝑓 6.63×10−34 ×1.6×1016
= 6.63𝑉. neutrons and X-rays don’t have a charge
𝑒 1.6×10−19 hence are not deflected or deviated in any
36. A functioning cathode rat tube generates field.
16.00m𝐴. Determine the number of rays flowing - Radioactive decay – Spontaneous
per second [𝑒=1.6×10-19C] A.10-15 B.10-14 C.1015 disintegration of an unsable nuclide to form a
D.1014. stable nuclide with an accompany emission of
𝐼=16.00m𝐴=0.016𝐴, 𝑒=1.6×10-19C, radiation e.g α,β or 𝛾.
𝑛 𝐼
Number of rays flowing per seconds, = , 1.Alpha decay – It is the emission of alpha
𝑡 𝑒
𝑛
=
0.016
= 1015𝑠-1. particle represented by helium nucleus ( 42𝐻𝑒 ). It
𝑡 1.6×10−19
is accompanied with a decrease in the nucleon or
37. In which of the following is the principle of
mass number by 4 and decrease in atomic
photoelectric effect put in use? I.Photographic
number or nuclear charge by 2. e.g
light meters II.Activation of automatic doors
92𝑈 → 90𝑇ℎ + 2𝐻𝑒 (α-decay)
238 234 4
III.Setting off of burglary alarms IV.Production of
2.Beta decay – It is the emission of beta particle
sound from film tracks A.I and II B.III and IV C.I,II
and III D.I,II,III and IV. represented by electron ( −10𝑒 ). It is accompanied
by an increase in the atomic number or nuclear
charge by 1 and no change in the mass number.
● Radioactivity and Nuclear reactions
90𝑇ℎ → 91𝑃𝑎 + −1𝑒 . (β-decay)
234 234 0
- Radioactivity – It is the emission of high energy 3.Gamma decay – It is the emission of gamma
radiations due to spontaneous decay of the rays represented as 00𝛾. It is not accompanied by
nucleus on an atom. Natural radioactivity occurs a change in mass or atomic number. It can
in heavy elements i.e elements with unstable follow any other decay.
nuclei. Artificial radioactivity occurs by 4.Positron decay (positive electron) – It is
bombarding stable nuclide with high energy
represented by 01𝑒. It is accompanied by no
protons, neutrons e.t.c.
change is mass number and a decrease in atomic
- Radiations emiited during radioactivity –
number by 1. 11𝐻 → 10𝑛 + 01𝑒 (positron decay).
1.Alpha(α) particle – I.It is a helium nucleus
- In balancing the equations of nuclear reactions,
( 42𝐻𝑒 ). II.It has a large mass, low velocity, high
the total mass numbers in the left hand side must
momentum. III.It is positively charged. IV.It is
be equal to the total mass numbers on the right
deflected towards the negative plate in an
hand side and the total atomic numbers on the
electric field or S-pole in a magnetic field. It is
left hand side must be equal to the total atomic
slightly defelcted due to its mass. V.Highest
numbers on the right hand side.
ionizing effect. VI.It has the weakest penetrating
- Activity of radioactive elements or count
power hence, stopped by a sheet of paper.
rate or decay rate – It is the numer of emission
VIII.Travels with a constant speed different from
of radiation or disintegration per unit time.
the speed of light.
- Activity is not affected by chemical or
2.Beta(β) particle – I.It is a fast moving
physical factors like temperature, pressure,
electrons ( −10𝑒 ). II.It has a small mass, high humidity or any external condition.
velocity, low momentum. III.It is negatively
- Actvity depends on the ionizing ability of the
charged. IV.It is deflected towards the positive characteristic radiation and the number of
plate in an electric field or 𝑁-pole in a magnetic
atoms originally present. Activity of a
field. It is more strongly deflected due to its small
radioactive sample also decreases with time.
mass. V.Intermediate ionizing effect. - Decay constant– It is the rate of decay per unit
VI.Intermediate penetrating power hence, atom of a substance or the ratio of decaying
stopped by an aluminium sheet/foil. VII.Travels
atoms per seconds to the total number of atoms.
with a variable speed close to the speed of light.
- Half life 𝒕𝟏/𝟐 – It is the time taken for a given
3.Gamma(𝜸) rays – I.Electromagnetic radiation
quantity of a radioactice material or the activity
of shortest wavelength and highest frequency
of a nuclide to decay/reduce to half its original
II.It is massless/negligible mass, high velocity
value.
and negligible momentum. III.It has no charge
- Longer half life denotes low activity and low
IV.It pass through a magnetic or electric field
decay constant i.e more stable nuclide while
undeflected in a straight line. v.Least ionizing
shorter half life denotes high activity and high
effect. VI.Highest penetrating power hence,
decay constant i.e less stable nuclide.
stopped by a thick lead block. VII.Travel with the
- The number 𝑁 of radioactive nuclei present at
speed of light.
any time 𝑡 during radioactive decay is given by –
- Only charged particles e.g cathode rays, 𝟏 𝒏 𝑵 𝟏 𝒏 𝑵𝒐
electrons, protons, alpha particles and beta 𝑵 = 𝑵𝒐 𝒆−𝝀𝒕 , 𝑵 = 𝑵𝒐 ( ) , =( ) = 𝟐𝒏 .
𝟐 𝑵𝒐 𝟐 𝑵

289
Demystified Series Physics Demystified by Dr Timothy
𝒏=
𝒕
. 𝑁=Quantity remaining after decay, 2. Nuclear fusion – It is the combination of two
𝒕𝟏/𝟐
or more light nuclei to form a heavier nucleus
𝑁o=Initial quantity, 𝑛=number of half lives, (which are not radioactive) with the release of a
𝑡=time taken for decay, 𝑡1 =half life. very huge amount of energy. The fusion
2
- Fraction remaining after decay 𝑭𝑹 – element is hydrogen. e.g fusion of deuterium 21𝐻
𝑵 𝟏
𝑭𝑹 = = ( )𝒏 . and tritium 31𝐻 isotopes of hydrogen to form
𝑵𝒐 𝟐 helium 41𝐻𝑒 .
- Fraction lost or decayed after decay 𝑭𝑫 – - Nuclear fusion occurs in the sun, stars and
𝑵 𝟏 𝒏
𝑭𝑫 = 𝟏 − =𝟏−( ) . source of energy in hydrogen bombs or fusion
𝑵𝒐 𝟐
- Decay constant is related to half life by – bombs.
𝑰𝒏𝟐 𝟎.𝟔𝟗𝟑 - Nuclear fusion occurs at very high temperature
= = . hence it is called a thermonuclear reaction.
𝒕𝟏/𝟐 𝒕𝟏/𝟐
- Graphical representation of activity or amount - Nuclear fusion releases much amount of
of a radioactive sample against time – energy than nuclear fission.
𝑵 - Artificial transmutation – This is the change of
𝑵o the nucleus of an atom to another nucleus by
artificial radioactivity when bombarded with
𝑵𝒐 neutrons or protons or α-particles , e.g
27 4 30 1
13𝐴𝑙 + 2𝐻𝑒 → 15𝑃 + 0𝑛
𝟐

- Isotopes – They are nuclei of the same element


𝒕𝟏/𝟐 𝒕
with the same atomic number but difeerent mass
- The graph is an exponential curve i.e the number due to difference in the number of
amount of radioactive substance decreases with neutrons.
time. - They have the same chemical properties but
- The time taken for the amount of the sample to different physical properties.
𝑁
be half its initial amount, 𝑜 is its half life - They can be separated by physical process and
2
𝑡1/2 . not chemical processes e.g deflection in a
- As the curve does not touch the time axis, the magnetic field or diffusion.
sample can never vanishes completely i.e all the - Isotopes of an element can be formed by
radioactive sample cannot be decayed irradiation or bombardment artificially by
completely. neutron 10𝑛. For example ; if nitrogen (𝑁) has 25
- Nuclear reactions – Reactions which involves isotopes and the lightest isotope is represented
the neclei of atoms e.g nuclear fission and fusion. as 147𝑁. We can deduce the other isotopes as
1. Nuclear fission – It is the splitting a heavy follows –
nucleus into smaller nuclei(which are also Second isotope → 147𝑁 + 10𝑛→ 157𝑁 i.e (1st isotope
radioactive) by bombarding the fission element + 1neutron, 1n).
i.e uranium, with neutrons and is accompanied Third isotope → 157𝑁 + 10𝑛→ 167𝑁 or 147𝑁 + 2 10𝑛
by the release of huge amount of energy in form → 167𝑁 i.e (2nd isotope + 1 neutron or 1st isotope
of heat.More neutrons are also produced which + 2 neutrons).
leads to a chain reaction e.g bombarding Fourth isotope → 167𝑁 + 10𝑛→ 177𝑁 or 147𝑁 + 3 10𝑛
uranium-235 ( 23592𝑈 ) by slow moving electrons. → 177𝑁 i.e (3rd isotope + 1 neutron or 1st isotope
- Nuclear fission is applied in nuclear reactors + 3 neutrons)
and atomic bombs. Hence the twenty-fifth (25th isotope) → 147𝑁 +
- Nuclear reactors – Energy realesed nuclear 24 10𝑛 → 387𝑁 (1st isotope + 24 neutrons).
fission due to chain reaction is used generate - Binding energy – It is the amount of energy
electricity. It consists of a nuclear fuel e.g that holds or binds nucleons(protons and
uranium (fissionable materiel), moderators e.g neutrons) together in the nucleus or the amount
graphite rods and heavy water, which prevents energy that will be required to disintegrate the
the chain reaction from getting out of control by nucleus when formed.
slowing down fast moving neutrons i.e reduce - In nuclear reactions, the mass of the reactants is
speed of neutrons, control rods e.g boron and always greater than the mass of the products. The
cadmium rods, which controls the level of fission mass difference is called mass defect ∆𝒎.
by absorbing neutrons intermittently or controls Einstein’s mass-energy equation shows that the
the rate of neutron production, cooling systems loss in mass is , accounted for the nuclear energy
e.g water or liquid sodium, to reduce excess heat 𝑬 released in form of heat in nuclear reactions.
produced in the reaction. 𝑬 = ∆𝒎𝒄2. 𝐸=energy released, ∆𝑚=mass defect
- The major component in a nuclear reactor or difference between the masses before and
used to control chain reaction are after reaction, 𝑐=speed of light.
moderators and control rods. Reactant=mass of protons + mass of neutrons

290
Demystified Series Physics Demystified by Dr Timothy
Product=mass of nucleus formed. The mass of the depends in A.temperature and purity B.purity
products and reactants are in unified atomic and age C.temperature and age D.age, purity and
mass unit 𝒖. 1𝑢=931𝑀𝑒𝑉, 1𝑀𝑒𝑉=106𝑒𝑉. temperature E.none of the above.
𝑀𝑒𝑉=mega electron-volt. 1𝑒𝑉=1.6×10-19𝐽. 4. Number
- Binding energy = (𝑴𝑹 − 𝑴𝑷 )𝒖, 𝑀𝑅 =mass of of atoms
reactants, 𝑀𝑃 =mass of products. A
- The graphical relationship between energy
released and the mass defetct when mass is B Time
converted to energy is given as – The points marked A and B respectively
𝑬(energy) represents A.Half number of atoms initially
Slope=
𝑬
= 𝒄 2. presentand time of decay B.Decay constant and
∆𝒎
half life C.Half number of decayed atoms and half-
life D.Half number of atoms initially present
∆𝒎(mass defect)
and half-life .
𝐴 represents half number of atoms originally
- Application of radioisotopes – 𝑁
1.In medicine – It is used to destroy cancerous present 𝑜 , 𝐵 represents the half- life 𝑡1/2 .
2
cell or treat cancer, iodine-131 is used to threat 5. Which of the following rays originated from
thyroid disorders, they can also be used as the nucleus of an atom A.X-rays B.cathode rays
medical tracers for detecting defcts in the body C.Ultra violet rays D.Gamma rays E.Infra red
and to sterilize surgical instuments. rays.
2.In agriculture – Mutations induced by 6. Which of the following is not correct about
radiation is used to yield more desirable and radioactivity? A.gamma radiation has an
adpted species. electrical charge B.gamma rays have negligible
3.In industries – Defects in metals and charge C.a beta particle has a negative charge
controlling filling of tins are checked by cobalt- D.an alpha particle has a positive charge E.the
60. velocity if gamma rays is the same as that of light.
4.In dating – Carbon-14 can be used to predict 7. Which of the following is true of the particles
the age of natural materials e.g rocks,earth, trees emitted in radioactive disintegration? A.the
e.t.c. alpha particles is helium nucleus 32𝐻𝑒 B.the alpha
particle is lighter than the beta particle C.the
Examples : beta particle is deflected more than alpha
1. Which of the following features is not a particle in a magnetic field D.the particle is
characteristic of natural radioactivity? deflected more than beta particle in a magnetic
A.radioactivity is a nuclear phenomenon field
B.The radioactivity of an element is affected Alpha particles is helium nucleus 42𝐻𝑒 . Alpha
by electric and magnetic fields in the particles are heavier than beta particles. Beta
surroundings C.radioactivity is exhibited only particles deflect more than alpha particles in a
by elements of mass number greater than 206 magnetic and electric field, as they are lighter
D.radioactive substances emit three radiations; than alpha particles.
α-rays, β-rays and 𝛾-rays E.the radioactive 8. Which of the following may be found in light
element is transformed into a new element . nuclei? I.β-particles II.protons III.neutrons IV.α-
The activity of radioactive elements is not particles A.I and II B.I and III C.II and IV D.II and
affected by physical or chemical factors, electric III E.I,II and III .
and magnetic fields. It depends on the ionization Protons and neutrons make up nucleons which
ability of the characteristic radiation (with α- are particles in the nucleus. α and β particles are
particles ionizing most) and the number of atoms radiations.
originally present. 9. Which of the following rays are deviated by
2. The rate of decay of atoms in a radioactive magnetic field? I.cathode II.alpha III.beta
sample is proportional to A.the half-life period IV.gamma. A.I,II and III B.I,II and IV C.I,III and IV
B.number of decayed atoms C.number of D.II,III and IV E.I,II,III and IV.
undecayed atoms D.the mass(nucleon) number 10. Of the following types of radiation, one that
of the atom E.the atomic(proton) number of the does not originate from the nucleus is A.alpha
atom B.beta C.x-rays D.gamma.
Rate of decay of atoms in a radioactive sample is 11.Which of the following cannot be deflected by
directly proportional to the number of atoms of an electric field or a magnetic field? I.α-rays II.β-
𝑑𝑁
the sample present i.e undecayed i.e − ∝ 𝑁. rays III.𝛾-rays IV.x-rays A.I and II B.III and IV C.I
𝑑𝑡
𝑑𝑁 and III D.II and IV.
= −𝜆𝑁.
𝑑𝑡
3. The activity of a radioactive substance

291
Demystified Series Physics Demystified by Dr Timothy
12. 𝑌 is/are correct? A.I B.II C.III D.I and II E.I and III .
Alpha-particle=42𝐻𝑒 , Beta-particle= −10𝑒 , proton=
1𝐻 , electron= −1𝑒 . Momentum increases with
𝑄 1 0

𝑃 mass. Alpha particle is most massive hence, has


𝑋 the highest momentum. Momentum is least in
A beam of radiation passes through an electric beta particles, the momentum of proton is
field located between plates 𝑋 and 𝑌 in the figure intermediate,due to its low mass. Beta particles
above. If 𝑃 is deflected downwards while 𝑄 and electrons have least momentum due to their
passes undeflected, then 𝑃 and 𝑄 could be, extremely small mass.
respectively I.β-rays, 𝛾-rays II.α-rays, 𝛾-rays 18. The main difference between X-rays and
III.β-rays, x-rays IV.α-rays, x-rays. Which of the gamma rays lies in their A.ionizing ability
above is/are correct? A.I B.II C.I and III D.II and B.absorption plate C.mode of production
IV . D.mode of propagation.
The electric field lines are directed way from 19. Which of the following arrangements of
plate 𝑋 and towards plate 𝑌, hence plate 𝑋 is radiations is in order of decreasing penetrating
positively charged and plate 𝑌 is negatively power? A.α, 𝛾, β B.α, β, 𝛾 C. β, 𝛾, α D.𝜸, β, α.
charged.𝑃 deflects towards positively charge 19. Which of the following are emited from a
plate 𝑋, hence 𝑃 is negatively charge i.e β-rays. 𝑄 radioactive substance without altering the mass
is undeflected i.e x-rays or 𝛾-rays. number or atomic number of the substance
13. Which of the following radiations will pass A.proton B.alpha-particles C.neutron D.gamma
through a sheet of paper? I.Alpha rays II.Beta rays E.electrons.
rays III.Gamma rays. A.I B.II C.III D.II and III 20. Which of the following are true for α-decay
Alpha particles have low penerating power, I.mass number decreases by four II.atomic
hence is stopped by a sheet of paper and will not number decreases by two III.mass number does
pass through it. Beta particles have a fairly high not change A.I and II B.II and III C.I and III D.I,II
penentration power,hence will pass through a and III.
sheet of paper. Gammay rays have the highest 21. In a radioactive series,
penetrating power, hence will pass through a 92𝐿 → 93𝑀 → 94𝑁 → 90𝑂 . The particles emitted
235 235 235 231

sheet of paper. are respectively A.β, β, α particles B.α, 𝛾, β


14. Which of the following is correct? A.alpha particles C.𝛾, α, β particles D.α, α, β particles D. β,
particles have large mass and high penetrating α, β particles .
power B.alpha particles have a small mass and a 92𝐿 → 93𝑀 + −1𝑒 . In this nuclear reaction, the
235 235 0

low penetrating power C.beta particles have no mass number is unchanged but the atomic
mass and a high penetrating power D.beta number increases by 1 i.e β-particle( −10𝑒 ) is
particles have a large mass and a low penetrating emitted.
power E.gamma rays have no mass and a high 93𝑀 → 94𝑁 + −1𝑒 ,
235 235 0
the mass number is
penetrating power . unchanged but the atomic number increases by 1
Alpha particles have a large mass and a low i.e β-particle( −10𝑒 ) is emitted.
penetrating power. Beta particles have low mass 94𝑁 → 90𝑂 + 2𝐻𝑒 , the mass number decreases
235 231 4
and fairly high penetrating power. by 4 but the atomic number decreases by 2 i.e α-
15. Which of the following travels in a vacuum particle( 42𝐻𝑒 ) is emitted. Therefore, the emission
with a velocity deifferent from that of light? is β, β, α particles.
A.radio waves B.ultraviolet waves C.alpha rays 22. In the radioactive series :
D.gamma reays E.x-rays. 92𝑊 → 93𝑋 → 91𝑌 → 92𝑍 . The particles emitted
235 235 231 231
16. One of the emissions of radioactive material are respectively A. α, α, β particles B.α, β, α
has the following properties I.momentum particles C.β, β, α particles D.β, α, β particles.
ii.deflection by a magnetic field III.fairly high
92𝑊 → 93𝑋 + −1𝑒 , the mass number is
235 235 0
penetrating power. The emission is A.alpha unchanged but the atomic number increases by 1
particles B.beta particles C.gamma radiation i.e β-particle( −10𝑒 ) is emitted.
D.X-rays E.neutrons.
93𝑋 → 91𝑌 + 2𝐻𝑒 , the mass number decreases
235 231 4
Beta particles have low momentum, deflected by 4 but the atomic number decreases by 2 i.e α-
towards the 𝑁-pole in a magnetic field and has a particle( 42𝐻𝑒 ) is emitted.
fairly high penetrating power.
91𝑌 → 92𝑍 + −1𝑒 , the mass number is
231 231 0
17. When an alpha particle, beta particle, a unchanged but the atomic number increases by 1
proton and an electron are given the same linear i.e β-particle( −10𝑒 ) is emitted. Therefore, the
speed in specific direction, the linear momentum emission is β, α, β particles.
of the I.alpha particle is the greatest II.proton is 23. The effect of emission of gamma rays from
the least III.electron is equal to that of beta the atom is to reduce the A.size of the atom
particle. Which of the following statements above

292
Demystified Series Physics Demystified by Dr Timothy
B.energy of the atom C.atomic number of the have disintegrated after A.30 days B.20 days
atom D.mass number of the atom. C.5days D.2½days .
24. Which of the folllwoig nstatements is not true Initial quantity, 𝑁𝑜 =8g, quanity decayed 𝑁𝐷 =7g,
of 𝛾-rays? A.𝛾-rays belong to the electromagnetic quantity remaining after decay, 𝑁=8−7=1g, 𝑛 =
spectrum B.𝛾-rays do not carry an electron 𝑡
, 𝑛=number of half-lives, time taken for decay
𝑡1/2
charge C.𝜸-rays obey the inverse cube law in
air D.the thickness of aluminium which stopes 𝑡, half-life 𝑡1/2 =10days.
𝑁𝑜 8
particles has negligible effect on 𝛾-radiation E.the = 2𝑛 , = 2𝑛 , 8 = 2𝑛 , 23 = 2𝑛 , 𝑛=3,
𝑁 1
difference between 𝛾-rays and X-rays is that 𝛾- 𝑛=
𝑡
, 3=
𝑡
, 𝑡 =3×10=30days.
rays originate from energy changes in the nuclei 𝑡1 10
2
of atoms while X-rays come from energy changes 31. Radium has a half life of 1600years. A lead
associated with the electron surface structure of box contained 24×106atoms of randium in 1976.
atoms. How many atoms of randium will remain in the
Gamma rays(𝛾-rays) is an electromagnetic box in the year 5176. A.24×106atoms
spectrum. All electromagnetic waves obey B.18×10 atoms C.12×10 atoms D.6×106atoms
6 6
inverse square law. Gamma rays pass through E.no atoms 𝑡1/2 =1600yrs,
aluminium sheet due to its high penetrating 𝑡=5176−1976=3200yrs, 𝑁𝑜 =24×106atoms, 𝑁=?
power while alpha and beta particles are stopped 𝑡 3200 𝑁𝑜 24×1024
aluminium sheet and will not pass through it. 𝑛= = =2. = 2𝑛 , = 22 = 4,
𝑡1/2 1600 𝑁 𝑁
25. A certain radioisotope 235 92𝑈 , emits four alpha 24×1024
𝑁= = 6×106atoms.
particles and three beta particles. The mass 4
number and atomic number of the resultant 32. A radioactive sample initially contains 𝑚
element respectively are A.219 and 87 B.84 ans atoms. The number of atoms that would have
𝑚 𝑚
223 C.223 and 87 D.219 and 81 . disintegrated after four half-lives is A. B.
16 8
235 4 0 𝐴
92𝑈 →4 2𝐻𝑒 + 3 −1𝑒 + 𝑍𝑋
7𝑚 𝟏𝟓𝒎
C. D. .
235=4(4)+3(0)+𝐴, 92=4(2)+3(−1)+𝑍, 8 𝟏𝟔
𝑁𝑜 𝑚 𝑚
235=16+𝐴, 92=8−3+𝑍 =5+𝑍, 𝑁𝑜 =𝑚, 𝑁=?, 𝑛=4, = 2𝑛 , = 24 , = 16,
𝑁 𝑁 𝑁
𝑚
𝐴 =235−16=219, 𝑍 =92−5=87. 𝑁= . Number of atoms decayed or
16
26. The radioisotope 235 92𝑈 decays by emitting disintegrated = Initial quantity−Number of
two alpha particles, three beta particles and a 𝑚
atoms remaining , 𝑁𝐷 = 𝑁𝑜 − 𝑁 = 𝑚 − ,
gamma ray. What is the mass number and atomic 16𝑚−𝑚 15𝑚
16
number of the resulting daughter element? A.91 𝑁𝐷 = = .
16 16
and 227 B.92 and 238 C.227 and 91 D.215 and 33. A radioactive material has an initial activity
88 . of 7000 counts per second and an activity of 875
235 4 0 0 𝐴
92𝑈 →2 2𝐻𝑒 + 3 −1𝑒 + 0𝛾 + 𝑍𝑋 counts per second after 6 hours. The half-life od
235=2(4)+3(0)+ 𝐴, 92=2(2)+3(−1)+0+𝑍, the material is A.6hr B.3hr C.2hr D.1hr.
235=8+0+𝐴, 92=4−3+𝑍 =1+𝑍, Initial activity 𝐴𝑜 =7000 counts per second,
𝐴 =235−8=227, 𝑍 =92−1=91. Activty after decay or disintegration 𝐴=875
27. Determine the value of 𝑥 from nuclear counts per second, 𝑡=6hrs, 𝑡1/2 =?,
reaction below, 222 𝑥
86𝑅𝑛 → 88𝑅𝑎 +2β+energy,
𝐴𝑜
= 2𝑛 ,
7000
= 2𝑛 , 8 = 2𝑛 , 23 = 2𝑛 , 𝑛=3,
A.221 B.222 C.223 D.224 . 𝐴
𝑡
875
6 6
β= −10𝑒 , 222 𝑥 0
86𝑅𝑛 → 88𝑅𝑎 +2 −1𝑒 +energy,
𝑛= , 3= , 𝑡1/2 = = 2hrs.
𝑡1/2 𝑡1/2 3
222= 𝑥 + 2(0) , 𝑥 =222. 34. A radioactive element has a half-life of 4 days.
28. 226
86𝑅𝑎 →𝑌 + 86𝑅𝑛 . In the nuclear reaction
222 3
The fraction that has decayed in 16 days is A.
above, 𝑌 is A.a beta particle B.an alpha particle 4
1 𝟏𝟓 1
C.a gamma ray D.an electron. B. C. D. .
16 𝟏𝟔 4
88𝑅𝑎 → 𝑍𝑌 + 86𝑅𝑛 ,
226 𝐴 222 𝑡 16
𝑡=16 days, 𝑡1/2 =4 days, 𝑛= = = 4 half-lives,
226= 𝐴 + 222, 88 = 𝑍 + 86, 𝑡1/2 4

𝐴 =226−222=4, 𝑍 = 88−86 = 2. Fraction lost or decayed after decay


1 𝑛 1 𝑛
𝑍𝑌 = 2𝑌 = 2𝐻𝑒 i.e alpha particle.
𝐴 4 4 𝑁
𝐹𝐷 = 1 − = 1 − ( ) , 𝐹𝐷 = 1 − ( ) ,
𝑁𝑜 2 2
29. Which of the following is the correct
1 4 1 16−1 15
definition of half life? A.the average life time of a 𝐹𝐷 = 1 − ( ) = 1 − = = .
2 16 16 16
radioactive material B.time it takes a 35. A radioactive element of decay constant 𝜆 has
radioactive material to decay to half of its 𝐼𝑛2 𝐼𝑛2 2𝐼𝑛2 𝑰𝒏𝟐
a half-life of A. 2 B.− 2 C. 2 D. .
original quality C.half the period a radioactive 𝜆 𝜆 𝜆 𝝀
material decay completely D.half of the decay 36. A radioactive substance with a half life of 5s
constant E.the decay constant times half life time. has a decay constant of A.0.693s-1 B.0.347s-1
30. A radioactive substance of initial mass 8g has C.0.218s-1 D.0.139s-1.
a half-life of 10days. 7g of the substance would

293
Demystified Series Physics Demystified by Dr Timothy
𝑡1/2 =5s , 𝜆 =
0.693
=
0.693
=0.139s-1. 1 neutron.
𝑡1/2 5
Therefore,mass number of the 20th isotope
37. A radioisotope has a decay constant of 10-7s- (heaviest isotope) is increased by 19 i.e 19
1. The average life of the radioisotope is
neutrons. 108
50𝑆𝑛 + 19 0𝑛 → 50𝑆𝑛 .
1 127
A.6.93×106s B.1.00×10-4s C.1.00×10-4s 45. Which of the following statements is not true
D.1.00×10 s E.6.93×10 s
7 7
of nuclear fission? A.a mother nucleus is broken
Average life of radioisotope is the same as its half into two daughter nuclei of roughly equal parts
life.Decay constant, 𝜆=10-7s-1, 𝑡1/2 =? B.neutron is normally used nin bombarding
0.693 0.693 0.693
𝜆= , 𝑡1 = = =6.93×106s. heavy nucleus C.two ligt nuclei are made to
𝑡1/2 2 𝜆 10−7
combine to form a heavy nucleus D.a very
38. A piece of radioactive material contains
enormous energy is released E.there is an
1020atoms. If the half-life of the material is 20
apparent loss in mass
seconds, the number of disintegrations in the
In fission process, the mass of the products is less
first second is A.3.47×1018 B.6.93×1020
than the mass of the reactants, hence an apparent
C.3.47×1020 D.6.93×1018
loss in mass. The number electrons produced by
𝑡1/2 =20 seconds, number of atoms 𝑁=1020 atoms,
fission process exceeds the number that caused
number of disintegration or decaying atoms per
𝑑𝑁 𝑑𝑁 0.693 𝑑𝑁 0.693 𝑁 fission, hence a chain reaction is initiated.
seconds =? = −𝜆𝑁, 𝜆 = , = , 46. One of the featured of fission processs is that
𝑑𝑡 𝑑𝑡 𝑡1/2 𝑑𝑡 𝑡1/2
𝑑𝑁 0.693×1020 A.its products are not radioactive B.it leads to
= = 0.03465×1020≈3.47×1018 chain reaction C.neutrons are not released D.the
𝑑𝑡 20
39. Two isotopes of uranium are designated as sum of the masses of the reactants equals the
238𝑈and 235𝑈. The numbers 238 and 235
sum of the masses of the products .
represenrs their A.atomic numbers B.nucleon The products of nuclear fission are also
numbers C.proton numbers D.neutron numbers. radioactive. It leads to chain reactions due to
41. Which of the following statements are true of exceess or additional neutrons produced. The
isotopes? I.isotopes of an element have the same mass of the products is not equal to the mass of
chemical properties because they have the same the reactants, due to the conversion of mass into
number of electrons II.isotopes of elements are energy.
normally separated using physical properties 47. When the nucleus of uranium atoms is split
III.isotopes of elements have the same number of into two fragments of nearly equal masses, the
protons in their nuclei A.I and II B.I and III C.II sum of te masses of the fragments is less than the
and III D.I,II and III. mass of the original nucleus. The difference is a
Isotopes of an element have the same atomic neausre of the A.atomic mass B.change of
number, i.e same number of protons and momentum of each fragment C.potential energy
electrons but different mass number, due to D.kinetic energy E.nuclear energy released
difference it the number of neutrons. They have The difference between the masses of the parent
the same chemical properties but different nucleus and the products represents the nuclear
physical properties, hence they can be separated energy released in the form of heat.
physically. 48. Which of the following are produced after a
42. The electrically neutral isotopes of oxygen nuclear fusion process? I.one heavy nucleus
are different in their I.proton number II.nucleon II.neutrons III.protons IV.energy A.I and II B.I
number III.number of orbital electrons and IV C.II and III D.II and IV .
IV.number of neutrons. Which of the statements Nuclear fusion involves the combination of
above are correct? A.I and II B.I and III C.I and IV lighter nuclei to for a heavier nucleus with the
D.II and III E.II and IV. liberation of tremendous amount of energy.
43. Neutrons are used to induce artificial Neutrons are not always produced in nuclear
radioactivity because they A.are energetic fusion.
B.have no charge C.have no mass D.are ionizing. 50. Which of the following explains the process
44. Tin(𝑆𝑛) has twenty isotopes of which the by which protons are converted into helium
lightest is represented by the symbol 108 50𝑆𝑛 , atoms with tremendous release of energy?
knowing that all possible isotopes exist. The A.photoelectric emission B.thermionic emission
symbol of the heaviest isotope is C.thermonuclear fusion D.field emission
A. 109
50 𝑆𝑛 B. 𝟏𝟐𝟕
𝟓𝟎 𝑺𝒏 C. 110
50𝑆𝑛 D. 125
50 𝑆𝑛 . E.nuclear fission.
Isotopes of an element are formed when one of Thermonuclear fusion or nuclear fusion is the
its isotope is bombarded or irradiated artificially combination of lighter nuclei(i.e protons) to form
neutrons. a hevier nucleus(i.e helium nucleus). Protons are
First isotope; 108 50𝑆𝑛 represented as hydrogen atoms 21𝐻 while helium
Second isotope; 108 50𝑆𝑛 + 0𝑛 → 50𝑆𝑛, the mass
1 109
atoms are 42𝐻𝑒 .
number of the second isotope is increased by 1 i.e 1𝐻 + 1𝐻 → 2𝐻𝑒 + energy.
2 2 4

294
Demystified Series Physics Demystified by Dr Timothy
51. In a certain fusion reaction, a deuteron 21𝐻 Energy released, 𝐸 = ∆𝑚𝑐2
interacts with a tritium 31𝐻 and produces an α- 𝐸 =0.01×10-27×(3.0×108)2 ,
particle 42𝐻𝑒 and a second product. The second 𝐸 =1.0×10-29×9×1016 = 9×10-13J.
product is A.a proton B.an electron C.a neutron 58. In the fusion of hydrogen isotopes into
D.a gamma ray . helium, the decrease in mass is about 0.65%.
1𝐻 + 1𝐻 → 2𝐻𝑒 + 𝑍𝑌 ,
2 3 4 𝐴 Calculate the energy obtainable when 1.0g of
2+3=4+𝐴, 1+1=2+𝑍, hydrogen is used. [𝑐=3.0×108ms-1] A.5.85×109J
5=4+𝐴, 2=2+𝑍, B.5.85×1010J C.5.85×1011J D.5.85×1012J.
𝐴 =5−4=1, 𝑍 =2−2=0. Initial mass=1g=1×10-3kg, mass defect or
𝐴
𝑍 𝑌 = 1
0 𝑌 = 1
0 𝑛 . Hence, the second product is a decrease in mass=∆𝑚=0.65% of 1×10-3kg,
0.65
neutron. ∆𝑚= × 1 × 10−3 =6.5×10-6kg,
100
52. In a nuclear fusion process four protons each 𝐸 = ∆𝑚𝑐 2=6.5×10-6×(3.0×108)2 ,
of mass 𝑀P, wre fused to produce a nucleus 𝑋 of 𝐸 =6.5×10-6×9×1016=5.85×1011𝐽.
mass 𝑀X. Which of the following equation is 57. When a nucleus is formed by bringing
correct? A.4𝑴p > 𝑀 X B.4𝑀P= 𝑀X C.4𝑀P< 𝑀X protons and neutrons together, the actual mass
D.𝑀P = 𝑀X. of the firmed nucleus is less than the sum of
4𝑀p →𝑀X+ energy. In nuclear reactions, the masses of the constituent protons and neutrons.
mass of the reactant is always greater than the The energy equivalent to the mass difference is
mass of the product or the mass of the reactant is A.lost energy B.work function C.stability energy
always less than that of the products. i.e D.binding energy.
4𝑀p > 𝑀X or 𝑀X< 4𝑀P. 58. In a fission of uranium isotope, a mass defct
53. Nuclear fission is preferable to nuclear fusion of 0.25 a.m.u is observed. Calculate the ninding
in the generation of energy because A.the raw energy of the isotope. A.1.25×10-19𝐽B.3.72×10-
materials for fusion are not easily obtained 11𝑱C.1.49×10-19𝐽 D.1.86×10-9𝐽
B.very high temperatures are required for [1 a.m.u=931𝑀𝑒𝑉, 1𝑒𝑉=1.6×10-19𝐽]
fusion C.the by-products of fusionare very Mass defect, ∆𝑚= 0.25 a.m.u, 1𝑀𝑒𝑉=106𝑒𝑉,
dangerous D.energy obtained from fusion is 1a.m.u=931𝑀𝑒𝑉=931×106𝑒𝑉,1𝑒𝑉=1.6×10-19𝐽
relatively smaller. ∴ 0.25 a.m.u=0.25×931×106×1.6×10-19
54. Which of the following materials is used to Binding enegy=3.72×10-11𝐽.
control the rate of neutron production in a 59. In the nuclear reaction, 11𝐻 + 73𝐿𝑖 →2 42𝐻𝑒, the
nuclear reaction? A.concrete shield B.graphite atomic masses of hydrogen and lithium are
rods C.boron rods D.uranium rods. Concrete 1.008𝑈 and 7.106𝑈 respectively and the atomic
shield are biological shield which protects mass of the helium atom is 4.004𝑈. I.energy is
nuclear reactor opeartors from intense exposure released by nuclear fission II.the energy released
to radiation. Graphite rods acts as moderator to can be calculated from 𝐸 = ∆𝑚𝑐2 , where ∆𝑚
slow down the neutrons, without absorbing it, to is the mass change produced and 𝑐 is the speed of
prevent dying out of the chain reaction. Boron light respectively III.the relation 𝐸 = ∆𝑚𝑐2, can
rods are control rods which absorbs excess be verified by measuring the initial energy of the
neutrons produced by the chain reaction, which α-particles A.I B.III C.I and II D.II and III E.I,II and
might speed up the reaction. Uranium rods are III
fissionable materials used as nuclear fuel. - The nuclear reaction above is an example of
55. In a modern nuclear reactor, the escaping nuclear fussion.
radiation is prevented from reaching the - ∆𝑚=mass defect=mass of reactants−mass of
opeartors by providing them with A.white products. mass of reactants=mass of hydrogen
laboratory jackets B.gas masks C.lead-line nucleus+mass of lithium nucleus=
jackets D.water proof jackets. 1.008+7.016=8.024𝑈, mass of product= 2(mass
56. Which of the following are the essential parts of helium nucleus)=2(4.004)=8.008𝑈, mass
of an atomic bomb? A.uranium and neutrons defect, ∆𝑚 =8.024−8.008=0.016𝑈.
B.radium and polonium C.nitrogen and neutrons - Measuring the initial energy of the alpha
D.uranium and α-particles. particle ( 42𝐻𝑒 ) can be used to verify 𝐸=∆𝑚𝑐2.
57. In a thermonuclear reaction, the total initial 60. A possible fusion reaction is ;
mass is 5.02×10-27kg and the total final mass is
1𝐻 + 1𝐻 → 1𝐻 + 1𝐻 + 𝑄, where 𝑄 is the energy
2 2 3 1
5.01×10-27kg. The energy released in the process released as a result of the reaction. If 𝑄=4.03𝑀𝑒𝑉.
is A.9.0×10-10J B.9.0×10-11J C.9.0×10-
12J D.9.0×10-13J [𝑐=3.0×108ms-1].
Calculate the atomic mass of 31𝐻 in atomic mass
unit. [ 21𝐻 =2.01410𝑢, 11𝐻 =1.00783𝑢, 1𝑢=931𝑀𝑒𝑉]
Initial mass=5.02×10-27kg, final mass= 5.01×10-
27kg, 𝑐=3.0×108ms-1,Mass defect or loss in
A.3.01604𝒖B.2.01504𝑢 C.3.01705𝑢 D.4.03702𝑢
Mass of reactants= 21𝐻 + 21𝐻 =2( 21𝐻 )
mass=∆𝑚, ∆𝑚=Initial mass−Final mass
Mass of reactants=2(2.01410)=4.02820𝑢,
∆𝑚=5.02×10-27−5.01×10-27=0.01×10-27J,

295
Demystified Series Physics Demystified by Dr Timothy
Mass of products= 31𝐻 + 11𝐻 , 31𝐻 =𝑥, - Wave particle duality or wave particle
Mass of products=(𝑥 + 1.00783)𝑢, paradox of light and matter – It is also called
Mass defect, ∆𝑚=mass of reactant−mass of the principle of complementarity. It is the fact
products=4.02820−( 𝑥 + 1.00783) or idea that light and matter sometimes exists as
Mass defect, ∆𝑚 =4.02820−1.00783+𝑥, wave and as particle other times but not
Mass defect, ∆𝑚 =(3.02037+𝑥)𝑢, simultaneously.
Energy released=Binding energy=4.03𝑀𝑒𝑉, - Wave nature or theory of light – It is explained
4.03
1𝑢=931𝑀𝑒𝑉, ∴4.03𝑀𝑒𝑉= =0.004329𝑢. by the properties of wave i.e reflection,
931
refraction, diffraction, interference and
Energy released=difference between mass of
polarisation.
reactants and products=(𝑀R−𝑀P)𝑢
- The invention of electron microscope and
0.004329=3.02037+𝑥,
scanning tunneling microscope(STM) is based on
𝑥 = 31𝐻 =3.02037−0.004329=3.01604𝑢.
the wave nature of matter.
Alternatively, 21𝐻 + 21𝐻 → 31𝐻 + 11𝐻 + 𝑄,
- Particles used in explaining the wave nature of
Substitute their given value into the nuclear
matter are ; neutrons,electrons/beta particles,
equation , 2(2.01410)= 𝑥 + 1.00783+0.004329
alpha particles/helium nucleus and
,
proton/hydrogen atom.
4.02820= 𝑥 +1.01216, 𝑥 =4.02820−1.01216
- Louis De Broglie’s principle proposed the
𝑥 =3.01604𝑢.
wave nature of matter just like light, as moving
61. The binding energy of helium 42𝐻𝑒 is
particles with wave properties. Louis De
A.2.017𝑢 B.0.333𝒖 C.4.033𝑢 D.0.330𝑢.
Broglie’s wavelength , is given by;
[atomic mass of proton=1.00783𝑢, atomic mass 𝒉 𝒉 𝒉
of neutron=1.00867𝑢]  = 𝑷 = 𝒎𝒗 = 𝒎𝒄 ,
Atomic mass of proton=1.00783𝑢, Atomic mass 𝑃=momentum of the particle=𝑚𝑣 or 𝑚𝑐,
of neutron=1.00867𝑢, Number of protons=2, 𝑚=mass, 𝑣=velocity, 𝑐=velocity of light,
Number of neutrons=4−2=2, ℎ=planck’s constant=6.6×10-34𝐽𝑠.
Total mass of reactants 𝑀 R=Mass of nucleons - Electron diffraction – It is used in testing or
=Mass of protons+mass of neutrons, experimenting the wave nature of matter, by
𝑀R=2(1.00783)+2(1.00867) difrracting electrons or waves matter using
𝑀R=2.01566+2.01734=4.033𝑢, crystals. Electrons produced by a hot filament are
Mass of product 𝑀P =Mass of helium 42𝐻𝑒 nucleus accelerated from rest through a voltage or p.d 𝑉.
=4𝑢. Binding energy=difference between mass Increasing the voltage 𝑉 on the anode causes the
of reactants(nucleons) and product (nucleus)= velocity 𝑣, of the electrons to increase. The
𝑀R−𝑀P=4.033𝑢 −4𝑢, wavelength  of the material object is give by ;
Binding energy=0.033𝑢. = =
𝒉 𝒉
=
𝒉
=
𝒉𝒗
.
𝒑 𝒎𝒆 𝒗 √𝟐𝒎𝒆 𝒆𝑽 𝟐𝒆𝑽
62. Calculate in joules, the binding energy of 73𝐿𝑖
[Atomic mass of 73𝐿𝑖 =7.01600𝑢, Masss of 𝒗=√
𝟐𝒆𝑽
. 𝑚𝑒 =mass of electron, 𝑒=electronic
𝒎𝒆
neutron=1.00867𝑢, Mass of proton=1.00783𝑢
Unified atomic mass unit, 𝑢=931𝑀𝑒𝑉, charge or charge on electron, 𝑉=accelerating p.d
1𝑒𝑉=1.6×10-19J] A.5.17×10-11J B.6.28×10-12J or voltage, 𝑣=speed of electron.
C.7.04×10-12J D.8.35×10-11J. - Diffraction of particles heavier than electron e.g
Mass of proton=1.00783𝑢, number of protons =3, neutrons, is done by reducing the velocity. In
Total mass of protons = 3(1.00783)= 3.02349𝑢, neutron diffraction is done with slow moving
Mass of neutron=1.00867𝑢, number of electrons where the velocity is reduced inoder to
neutrons=7−3=4, Total mass of neutrons = increase the wavelength and decrease the
𝑷
4(1.00867)= 4.03468𝑢. momentum i.e 𝒗 ∝ .
𝝀
Mass of reactants 𝑀R=Mass of nucleons=Mass of - In the wave nature of electron, increase in speed
protons+Mass of neutrons, of a photoelectron will decrease its wavelength
𝑀R=3.02349+4.03468=7.05817𝑢, 𝟏
and increase it energy. i.e 𝒗 ∝ and energy ∝ 𝒗2.
Mass of product 𝑀P=Atomic mass of 73𝐿𝑖 𝝀
nucleus=7.01600𝑢. - Particle nature of light and matter – It is
Binding energy=(𝑀R−𝑀P)𝑢 explained by Brownian motion, diffusion,
Binding energy=7.05817−7.01600=0.04217𝑢 osmosis, crystal structure, thermionic
1𝑢=931𝑀𝑒𝑉, 1𝑀𝑒𝑉=106𝑒𝑉,1𝑒𝑉=1.6×10-19J, emission, photoelectric emission, Compton
0.04217𝑢 =0.04217×931×106×1.6×10-19 , effect, radiation from heated bodies, electron
Binding energy in joules = 6.28×10-12J. momentum and line spectra (absorption and
emission).
- Heisenberg uncertainty principle or
● Wave particle duality or wave indeterminacy principle – It states that the
particle paradox position 𝒙 and momentum 𝑷 of a particle cannot

296
Demystified Series Physics Demystified by Dr Timothy
be accurately determined at the same instant of 8. An electron of mass 9.1×10-31kg moves with a
time OR It is impossible to determine acurrately, speed of 107ms-1. Calculate the wavelength of the
both the wave properties of matter and its associated wave. [ℎ=6.6×10-34𝐽𝑠] A.9.10×10-24𝐽𝑠
particulate properties at the same time. B.4.55×10-17m C.7.25×10-11m D.6.20×10-8m
∆𝒙 . ∆𝑷 ≥ 𝒉 = .
𝒉 𝑚e=9.1×10-31kg, 𝑣=107ms-1, ℎ=6.6×10-34𝐽𝑠,
𝟐𝝅 ℎ ℎ 6.6×10−34
Heinsenberg uncertainty principles can also be =𝑝=𝑚 =
9.1×10−31 ×107
=7.25×10-11m
𝑒𝑣
expressed in terms of energy 𝑬 and time 𝒕 or 9. If electrons are accelerated from rest through
position 𝒙 and velocity 𝒗. a potential difference of 10k𝑉, what is the
𝒉 𝒉
∆𝑬 . ∆𝒕 ≥ 𝒉 = . ∆𝒙 . ∆𝒗 ≥ 𝒉 = . wavelength of the associated electrons?
𝟐𝝅 𝟐𝝅
- Position and momentum, energy and time, A.1.22×10-11m B.3.87×10-10m C.2.27×1011m
position and velocity are known as D.2.27×1014m [𝑚𝑒 =9.1×10-31kg, 𝑒=1.6×10-19C
complementary variables. ℎ=6.6×10-34Js]
- Heisenberg uncertainty principle does not 𝑉=10k𝑉=10×103𝑉=1.0×104𝑉,𝑚𝑒 =9.1×10-31kg
apply to objects above atomic sizes i.e 𝑒=1.6×10-19C, ℎ=6.6×10-34𝐽𝑠, 𝜆=?
ℎ 6.6×10−34
macroscopic matter, but applies to 𝜆= = ,
√2𝑚𝑒 𝑒𝑉 √2×9.1×10−31 ×1.6×10−19 ×1×104
particulate matter e.g atoms, because 𝒉 is
𝜆 = 1.22×10-11m.
extremely small (6.63×10-34Js).
10. Calculate the minimum wavelength of X-rays
when a voltage of 60k𝑉 is applied to an X-ray
Examples :
tube. A.2.06×10-11m B.1.03×10-11m C.3.09×10-
1. Which of the following phenomena best 11m D.4.12×10-11m [𝑒=1.6×10-19C, ℎ=6.6×10-
supports the wave nature of matter? A.Electron 34Js, 𝑐=3.0×108ms-1]
diffraction B.Photoelectric effect C.Electron
𝑉=60k𝑉=60×103𝑉=6.0×104𝑉, 𝑒=1.6×10-19C,
momentum D.Black body radiation
ℎ=6.6×10 Js, 𝑣=𝑐=3.0×10 ms ,
-34 8 -1
Electron diffraction is explained in terms of ℎ𝑣 6.6×10−34 ×3×108
waves. Photoelectric effect, electron momentum 𝜆= = =1.03×10-11m.
2𝑒𝑉 2×1.6×10−19 ×6×104
and black body radiation are explained in terms 11. The mass and wavelength of a moving
of particle nature of matter. electron are 9.0×10-31kg and 1.0×10-10m
2. Electron passing through crystals are respectively. Calculate the kinetic energy of the
diffracted because they A.are reppeled by the electron [ℎ=6.6×10-34𝐽𝑠] A.1.21×10-17J
atoms in the crystal B.are attracted by atoms in B.2.31×10 J C.2.42×10 J D.3.24×10-17J.
-17 -17
crystal C.are particles D.possess wave 𝑚𝑒 =9.0×10-31kg, 𝜆=1.0×10-10m, ℎ=6.6×10-34Js,
properties. 𝜆=

, 𝑣=
ℎ 1
, 𝐾. 𝐸 = 𝑚𝑒 𝑣 2
3. Which of the following is not an evidence of the 𝑚𝑒 𝑣 𝑚𝑒 𝜆 2
ℎ 2 ℎ2 ℎ2
particulate nature of matter? A.Diffusion 1
𝐾. 𝐸 = 𝑚𝑒 ( ) = ×
1 𝑚𝑒
× = ,
2 𝑚𝑒 𝜆 2 𝑚𝑒 2 𝜆2 2𝑚𝑒 𝜆
B.Brownian motion C.Diffraction D.Crystal 2
(6.6×10−34 ) 4.356×10−67
structure E.Photoelectricity. 𝐾. 𝐸 = = ,
2×9×10−34 ×1×10−10 1.8×10−50
4. A moving electron possesses the 𝐾. 𝐸 =2.42×10 J. -17
characteristics of A.particles only B.waves only 12. It is always not possible to determine exactly
C.both particles and waves D.momentum only and simulataneously the position and
A moving electron has momentum and momentum of a particle. This statement is known
energy(particle nature of matter).Moving as the A.De Broglie’s law B.Heinsenberg’s
electrons also have velocity and principle C.Compton’s effect D.Frank Hertz
undergointerference(wave nature of matter). experimental law E.wave particle paradox.
5. The particle nature of light is demonstrated by 13. Which of the following statements is a correct
the A.speed of light B.colours of light consequence of the uncertainty principle? A.The
C.diffraction of light D.photoelectric effect uncertainty in our knowledge of energy and
Speed of light, colours(wavelength) of light due duration taken to measure it are each less than
to dispersion and diffraction of light are wave plank’s constant B.The complete knowledge of
properties). Photoelectric effect and position of a particle implies incomplete
spectra(emission and absorption) formation are ignorance of its energy C.It is possible to measure
particle nature of light. exactly both the position and momentum of a
6. The wave theory of light cannot be used to particle at the same time D.Both momentum and
explain A.phtoelectricity B.polarization energy of a particle can be known with absolute
C.diffraction D.interference. certainty E.A particle’s kinetic energy cannot
7. Which of the scientists postulated that moving be measured accurately at any time
particles exhibit wave properties? A.Niel Bohr Complementary variable in Heinsenberg’s
B.Ernest Rutherford C.Wemer Heisenber D.Louis uncertainty principles are; position and
De Broglie E.Erwin Schroedinger. momentum, energy and time, position and

297
Demystified Series Physics Demystified by Dr Timothy
velocity. The product of the uncertainty of each
complementary variable is greater than or equal Jamb past questions on elementary atomic
to planck’s constant. physics :
14. The uncertainty principle can be most [1979/31,1980/4,5,20,41,1981/29,1982/40,
usefully apllied to measurement on A.the 1983/34,1984/25,42,44,1985/49,50,1986/50,
momentum of a train at a particular position 1987/47,48,49,50,1988/49,50,1989/46,47,49,
B.the energy of a satellite at a time in space C.the 50,1990/46,48,50,1991/47,1992/47,49,50,199
energy of an electron in a cathode ray tube at 3/47,49,50,1994/50,1995/49,50,1997/50,1998
a particular time D.the momentum of a football /48,49,50,1999/43,44,47,49,2000/28,37,2001/
as it just of an electrons at a particular velocity 38,39,42,44,2002/41,43,45,49,2003/33,40,44,
Heinsenberg’s uncertainty principle only apply 46,48,2004/23,28,44,2005/5,7,9,12,2006/2,4,8,
to non-massive bodies i.e atomic sizes e.g 2007/10,12,2008/46,2009/45,46,48,2010/46,
electrons. Train,satellite and football are massive 47,2011/41,43,2012/47,2013/47,2014/44,45,
bodies. 48,2015/9,21]
15. If ∆𝑥 is the uncertainty in the measurement
of the position along the x-axis and ∆𝑃𝑥 is the
uncertainty in the measurement of linear
momentum along the x-axis, then uncertainty
principle relation is given as A.∆𝑷𝒙 ∆𝒙 ≥
𝒉B.∆𝑃𝑥 ∆𝑥 = ℎ C.∆𝑃𝑥 ∆𝑥 =2𝜋 D.∆𝑃𝑥 ∆𝑥 = 2𝜋/ℎ
E.∆𝑃𝑥 ∆𝑥 =1.
16. If the uncertainty in the measurement of the
position of a particle is 5×10-10m, the uncertainty
in the momentum of the particle is A.1.32×10-
44Ns B.3.30×10-44Ns C.1.32×10-24Ns D.3.30×10-
24Ns [ℎ=6.6×10-34Js]

Uncertainty in position, ∆𝑥=5×10-10m,


ℎ=6.6×10 𝐽𝑠,Uncertainty in momentum, ∆𝑃,
-34
ℎ 6.6×10−34
∆𝑥 . ∆𝑃 ≥ ℎ , ∆𝑃 ≥ ≥ ,
∆𝑥 5×10−10
∆𝑃 ≥ 1.32×10 Ns.
-24

17. The uncertainty in determining the duration


during which an electron remains in a particular
energy level before returning to the ground state
is 2.0×10-9s. Calculate the uncertainty in

detrmining its energy at that level. [Take =
2𝜋
ℎ = 1.054×10 Js] A.5.27×10 J B.4.16×10 J
-34 -26 -26

C.3.13×10-26J D.2.14×10-26J.
Uncertainty in time,∆𝑡 = 2.0×10-9𝑠,

= 1.054×10-34Js, Uncertainty in energy, ∆𝐸=?,
2𝜋
ℎ ℎ 1
∆𝐸 . ∆𝑡 ≥ , ∆𝐸 ≥ × ,
2𝜋 2𝜋 ∆𝑡
1
∆𝐸 ≥ 1.054×10-34× ≥ 5.27×10-26𝐽.
2.0×10−9
18. The uncertainty in the velocity 𝑣 of a moving
electron of mass 10-30kg is 3×106ms-1. Calculate
the uncertainty of the simultaneously
measurement of its position 𝑥. [ℎ=6.62×10-34Js]
A.1.11×10-10m B.2.21×10-10m C.3.22×10-10m
D.4.12×10-10m
Mass of electron,𝑚=10-30kg, velocity of
electron=3×106ms-1, Uncertainty in momentum,
∆𝑃 = ∆(𝑚 × 𝑣), ∆𝑥 . ∆𝑃 ≥ ℎ, ∆𝑥 . ∆(𝑚 × 𝑣) ≥ ℎ,
ℎ 6.62×10−34
∆𝑥 ≥ ≥ ≥ 2.21×10-10m.
∆(𝑚×𝑣) 10−30 ×3×106

298
Demystified Series Physics Demystified by Dr Timothy

Chapter 27 – Basic Electronics : Semiconductos,


diodes and transistors
● Semi-conductors – Materials whose resistivty - Impure or Extrinsic semi-conductors are
and conductivity is between that of conductors formed by doping. Doping is the addition of
and insulators e.g silicon and germanium. impurities to a pure or intrinsic semi-conductor
- Pure or intrinsic semi-conductors are group inoder to improve its conducting ability.
IV elements i.e tetravalent or quadrivalent. - Impure or Extrinsic semi-conductors are of two
- Characteristics of semi-conductors – types namely ; Positive type (P-type) and
1.They exhibit covalent bonds between their Negative type(N-type).
atoms. 1.P-type is formed when a pure or intrinsic semi-
2. Pure or intrinsic semi-conductors are poor conductor is doped with a group III elements
conductors, it will only conduct electricity when (acceptor or trivalent impurities) e.g boron,
a battery is connected across its end. aluminium,thalium,indium,gallium [BATIG].
3. The current carriers are free electrons and 2.N-type is formed when a pure or intrinsic semi-
holes. Pure or intrinsic semi-conductors conductor is dopped with a group V element
contain equal numbers of holes and free (donor or pentavalent impurities) e.g bismuth,
electrons. phosphorus,arsenic,antimony [BPA2].
4.Semi-conductors have negative temperature - Mnemonic: PTNP – PT : P-type, trivalent, NP
coefficient of resistance or resistivity i.e their : N-type, pentavalent.
resistance decreases with increase in - In impure of extrinsic semi-conductors the
temperature, hence their conductivity increases number of holes is not equal to the number of
with temperature increase. electrons. The one that is more in number is
5. They conduct electricity better with called the majority carrier, while the one that is
impurities(dopants) and heat. lesser in number is called minority carrier.
- At absolute zero temperature(0K), no free - In P-type : majority carrier are holes, while
electrons or holes are found in a pure or minority carrier are free electrons.
intrinsic semi-conductor. - In N-type : majority carriers are free electrons
- Energy band theory or Band theory is the while minority carriers are holes.
theory of electronic conduction in solids, which - P-n junction diode or depletion layer or
satisfactorily explains the porperties of potential barrier – It is the contact surface of a
conductors,insulators and semi-conductors and P-type and N-type semi-conductor joined
also gives distinguishes them. together.
- Valence band is the low energy region or range - When voltage is applied i.e connecting a battery,
of energies of valence electrons. across the junction, biasing (forward or
- Conduction band is the high energy region reverse) occurs.
with free electron accelerated from the valence 1.Forward biased – It occurs when the positive
band or range of energies of free electrons. terminal of a cell is connected to the P-type and
- Forbidden energy band or fabrication band the negative terminal is connected to the N-type
is the energy difference or the gap between the of a junction diode. Forward biasing drives
valence band and the conduction band. majority carriers (holes for P-type and electrons
1. In semi-conductors – I.The valence band for N-type), towards the junction and drives
contains holes while the conduction band minority carrirers (free electrons for P-type and
contains free electrons. II.The number of holes holes for N-type) away from the junction.
equals the number of free electrons. III.The - Features of forward biasing – I.Large current
forbidden energy gap or fabrication band is is produced as current flow is due to majority
narrow or small. carriers. II.Depletion layer reduces in width or
2. In conductors – I.The valence band and narrowed. III.Juction offers low resistance to
conduction band contains electrons. II.The current flow. IV.Internal barrier voltage is
number of electrons in the conduction band is overcomed. V.Potential barrier is reduced.
greater than that in the valence band. 2.Reverse biased – It occurs when the positive
III.Forbidden energy gap or fabrication band is terminal of a cell is connected to the N-type and
little or no gap. the negative terminal is connected to the P-type.
3. In insulators – I.The valence band contains Reverse biasing drives majority carriers (holes
electrons while no electrons is present in the for P-type and electrons for N-type) away from
conduction band i.e empty. II.Forbidden energy the junction and drives minority carriers (free
gap or fabrication band is large or wide. electrons in P-type and holes in N-type) towards
the junction.

299
Demystified Series Physics Demystified by Dr Timothy
- Features of reverse biasing – I.Extremely 𝐷=diode.
small current called reverse saturation current 𝐷 d.c
or leakage curret, is produced and current flow is Input a.c 𝑅 𝑉 Output
due to minority carriers. II.Depletion layer voltage voltage
increases in width or widened. III.Junction offers 𝑉𝑖
a high resistance to current. IV.Breakdown Input voltage (a.c)
voltage is overcomed. V.Potential barrier is
increased. 𝑡
- Current - voltage (𝑰 − 𝑽) characteristric
curve/graph of a p-n junction diode shows that Output voltage (d.c)
the junction has a low resistance in forward-bias 𝑉𝑜 𝑡
direction i.e larger current flow and a high
resistance in reverse-bias direction i.e small II.Full-wave rectfification – It is done with 2
current flow. diodes in series or 4 diodes in bridge
𝑰 arrangement. The negative half of a.c flips to the
Breakdown
voltage
positive half cycle such that both halves are in
−𝑽 𝑽 same direction. Full wave rectification is of a
higher d.c output and efficiency than half wave
Barrier voltage rectification. The frequency of the output d.c
𝒇𝒅.𝒄 is twice the frequency of the input a.c 𝒇𝒂.𝒄
- The conductivity of semi-conductors is not 𝒇𝒅.𝒄 =2𝒇𝒂.𝒄 .
affected by its small size.
- Advantages of semi-conductors diodes over 𝐷1 d.c
thermionic diodes (vacuum diodes) – Input a.c 𝑅 𝑉 Output
1.Small size. 2.Low power requirements, require voltage 𝐷2 voltage
no filament power and produce less heat.
3.Highly durable, less liable to break. 4.It needs 𝐷 1 and 𝐷 2=diodes.
only small p.d or voltage to operate in a radio 𝐷1 𝐷2
receiver. 5.It requires no time to heat up inoder
to produce current carriers, as they are already 𝐷3 𝐷4
available in semi-conductor. Input 𝑅 𝑉 Output
- Disadvantages of semi-conductors diodes – voltage voltage
1.Easily damaged by high temperature 𝑉𝑖
(extremely heat/temperature sensitive). Input voltage (a.c)
2.Destroyed by slight overload for a short time.
- Zener diodes are diodes which conducts well 𝑡
in both forward and reverse bias direction,but
they are operated in reverse bias direction. Output voltage (d.c)
Zener diodes are used as voltage regulators 𝑉𝑜 𝑡
and stabilizer for power supply .
- Uses of p-n junction or semi conductor 2.The p-n junction diode can acts as a switch in a
diodes – circuits. It closes the circuit i.e switch on in the
1. Current rectification – It is used as a rectifier forward biased and opens the circuit i.e switch
in alternate circuit to convert a.c to d.c, because it off in the reverse biased.
allows current in one direction only i.e forward 3.The p-n junction is used as a regulator or
biased direction. In a rectifier circuit, the input control in circuits, detecting faults and protecting
voltage is a.c while the output voltage is d.c. circuits.
- Rectification can be half-wave rectification or ● Transistors – These are semi-conductor
full-wave rectification depending on the devices with three layers or terminals namely
arrangement of the diodes. emitter(E), collector(C) which forms the outer
I.Half-wave rectification – It is done with 1 layersand base(B) which forms the middle layer.
diode connected in series with to am a.c voltage It is formed when two p-n junctions; emmiter-
source.It conducts on the positive half of the cycle base junction and collector-base junction are
i.e forward biased direction and cuts off current sandwiched together.Hence, it is also called a
i.e no current is conducted, on the negative half bipolar junction device.
cycle i.e reverse bias direction.The frequency of I. Emitter – It is the heavily doped layer or
the output d.c 𝒇𝒅.𝒄 equals the frequency of the terminal rich in holes.It supplies the majority
input a.c 𝒇𝒂.𝒄 . carriers and is always forward biased in respect
to the base with an applied voltage.

300
Demystified Series Physics Demystified by Dr Timothy
II. Collector – It is the lightly dopped layer or C
terminal rich in electrons.It collects majority
carriers and is always reverse biased in respect B Output
to the base with an applied voltage. Input
III. Base – It is the thinnest slightly doped layer ●
or terminal poor in electrons. E (Common emitter)
- Transistors are operated by current i.e they 2.Common collector(C.C) or emitter follower
are controlled by small current to regulate connection – Collector is common to both input
large current. (base and collector) and output (emitter and
- Transistors are of two types; p-n-p transistor collector). The properties of C.C configuration are
and n-p-n transistor. high current, medium power amplification
I.p-n-p transistor – It consists of two outer P- and voltage buffering i.e does not amplify
type and a N-type placed between them. Acceptor voltage.
impurities from the P-type diffuses on both sides E
of the N-type. Current flows into the circuit.
II.n-p-n transistor – It consists of two outer N- B Output
type and a P-type placed between them. Donor Input
impurities from the N-type diffuses on both sides ●
of the P-type. Current flows out of the circuit. C (Common collector)
- Circuits symbols of transistors : 𝑰𝑬 = 𝑰𝑩 + 𝑰𝑪 . 3.Common base (C.B) connection – Base is
Emitter E Collector C Collector C common to both input (emitter and base) and
output (base and collector). The properties of C.B
Base B configuration are high voltage and low power
Emitter E amplification.It does not amplify current.
Base B 𝐸 𝐶
n-p-n transistor
Emitter E Collector C Collector C

Base B 𝐵 (Common base)
Emitter E - Tansistor as a switch –
Base B I.Cut off current region – When the switch is
p-n-p transistor opened such that the no current is flowing
- In-P ; Inward arrorw p-n-p, Out-N ; Outward through the transistor i.e input current is
arrow n-p-n. zero,then the output current is zero and the out
- The emitter and collector are slant with the put voltage is maximum.
emitter bearing an arrow end indicating II.Saturation or maximum current region –
direction of current flow. Current flows out of When the switch is closed such that current is
the circuit in n-p-n transistors while current flowing through the transistor i.e input current is
flows into the circuit in p-n-p transistor. maximum, then the output current is maximum
- Uses of transistors – and the output voltage is zero.
1.Transistors are used as amplifiers for the - Current-voltage (𝑰 − 𝑽) characteristic curve
amplication of a.c current,voltage and power. of a transistor –
2.Switches when amplifying a.c voltage. 𝑰
3.Regulates current, voltage or power.
- Transistor amplification configuration – A
transistor when used as an amplifier, has 1 of its
terminals common to the input and output 𝑽
terminals, hence it has 2 inputs and 2 outputs - Transistors and p-n junction diodes are non-
terminals. There are 3 types of transistors ohmic conductors.
amplification configuration connection:
1.Common emitter(C.E) connection – Emitter Examples :
is common to both the input (emitter and base) 1. Which of the following is a semi-conductor
and output (emitter and collector). The material? A.Copper B.silicon C.Steel D.Iron.
properties of C.E configuration are high current, 2. Which of the following is a pure semi-
high voltage and high power amplification. conductor? A.Silicon B.Phosphorus C.Transistor
Common-emitter provides satisfactory D.Boron E.Carbon
current amplification and is most widely used Pure or intrinsic semi-condutors are group IV
in audio(low) frequency amplifiers. elements e.g silicon and germanium. Phosphorus
is a group V element. Transistor is an impure or

301
Demystified Series Physics Demystified by Dr Timothy
extrinsic semi-conductor formed fron the joining valence band D.twice the number of bands in the
two p-n junction together. A pure semi- valence band.
conductor is an insulator or poor conductor, but 11. When a pure semi-conductor is heated, its
graphite(a form of carbon) is a good conductor. resistance A.increases B.decreases C.remains
3.Intrinsic semi-conductor atoms are bonded the same D.increases then decreases.
together by A.covalent bonding C.ionic bonding 12. When impurities are added to semi-
C.thermal bonding E.metallic bonding conductors, the conductivity of the semi-
Intrinsic or pure semi conductor atoms are conductor A.increeases B.decreases C.remain
bonded by covalent bonding. constant D.increases then decreases.
4. For semi conductors to have negative 13. Which of the following statements are true
temperature coefficient of resistance implies that about semi-conductors I.doped semi-conductors
A.their resistance increases with temperature are extrinsic semi-conductor II.Germanium can
B.their resistance is constantly changing with be used as an intrinsic semi-conductor III.Silicon
temperature C.their resistance decreases with can be used as an extrinsic semi-conductor
temperature D.they have electrons and holes at IV.materials used for doping semi-conductors
high temperature. have four valence electrons A.I and II B.II and IV
5. Intrinsic semi-conductors A.do not conduct C.I,II and III D.I,II and IV E.II,III and IV
electricity at all B.conduct better at higher Doped semi-conductors are impure or extrinsic
temperature C.are used as conductors in semi-conductors. Pure semi-conductors are
electronics D.have low resistivity at lower intrinsic semi-conductors. Silicon and
temperatures E.have high resistivity at higher Germanium are pure or intrinsic semi-
temperatures conductors. Doping materials have three and five
The resistance of semi-conductors decreases valence electrons i.e group III and V.
with increase in temperarure,hence their 14. If a donor impurity is introduced into a silicon
conductivity increases with temperature. lattice, the resulting material is A.n-type B.p-
6. I.small size II.Low power requirement III.Not type C.richer in electron-hole pairs D.richer in
easily damaged by high temperature IV.Highly holes only .
durable. Which of the above are the advantages Donor impurity is a pentavalent or group V
of semi-conductors? A.I,II and III B.II,III and IV element.Doping silicon with a donor impurity
C.I,II and IV D.I,II,III and IV. forms a n-type semi-conductor.
7. In semi-conductors, the carriers of current at 15. Pure silicon can be converted to p-type
room temperature are A.electrons only material by adding controlled amount of
B.electrons and holes C.holes D.electrons and A.pentavalent atoms B.hexavalent atoms
ions. C.tetravalent atoms D.trivalent atoms .
8. The major difference between a pure semi- Doping a pure silicon with a trivalent atom i.e
conductor and a pure metal is that A.metals are group III element, acceptor impurity, forms a p-
harder than semi-conductors B.while the type semi-conductor.
resistance of metals decreases with temperature, 16. A certain manufacture wishes to make an n-
the reverse is the case of semi-conductors type semi-conductor. Which materials would he
C.metals have forbidden gaps while semi- select for doping? A.Arsenic and antimony
conductors have not D.the resistance of metals B.Aluminium and indium C.Boron and antinomy
increases with temperature, while for semi- D.Helium and boron
conductors, it is the reverse. Arsenic and antimony are pentavalent or group V
The resistance of metals increases with elements i.e donor impurities, hence when used
temperature as they have positive temperature to dope a semi-conductor forms a n-type semi-
coefficient of resistance. Metals i.e good conductor.
conductors don’t have energy gap while semi- 17. Silicon doped with aluminium and
conductors have narrow or small forbidden germanium doped with arsenic become A.n-type
energy gap. 9. semi-conductors B.p-type semi-conductorsC.n-
The theory of electronic conduction in solids, and p-type respectively D.p- and n-type semi-
which satisfactorily explains the properties of conductor respectively
conductors, insulators and semi-conductors is Silicon doped with aluminium(trivalent or group
A.particle theory B.band theory C.group theory III element or acceptor impurity) forms a p-type
D.string theory. semi-conductor while germanium doped with
10. In a pure semi-conductor, the number of arsenic(pentavalent or group V element or donor
electrons in the conduction band is A.equal to impurity) forms a n-type semi-conductor.
the number of holes in the valence band 18. As the voltage across a junction diode
B.greater than the number of holes in the valence increases in the forward biased direction
band C.less than the number of holes in the A.forward resistance increases B.current

302
Demystified Series Physics Demystified by Dr Timothy
decreases C.current increases D.contains 27. A diode is not used for A.regulation
remains constant E.potential barrier increases B.amplification C.detection D.instrument
In the forward biased direction, current protection E.switching
increases as voltage across the p-n junction diode Transistors are used for amplification not diodes.
increases due to a decrease in the resistance and 28. A typical graph of a p-n junction diode is
potential barrier. shown by
19. The current in a reverse-biased junction is A. 𝐼 B. 𝐼
due to A.electrons B.majority carriers C.holes
D.minority carriers 𝑉 𝑉
Current in reverse-biased direction is due to
minority carriers while current in forward-
biased direction is due to majority carriers. C. 𝐼 D. 𝑰
20. The battery connection that permits current
to flow across a p-n junction is called 𝑉 𝑽
A.amplification B.rectification C.forward biased
D.reverse biased.
21. In a semi-conductor junction diode, as the The graph shows that it is forward biased in one
depletion or barrier layer is forward biased, the direction and reverse biased in the other
layer A.narrows B.remains constant C.widens direction as shown in option D.
then narrows D.widens 29. Zener diode is used for A.current
The depletion layer narrows or reduces its width amplification B.power amplification C.voltage
in forward biased, in order to increase the flow of regulation D.energy conversion
majority carriers, hence increase in current flow. 30. In bipolar transistors, conduction is by
22. If a reversed-biased voltage is applied across A.charges and current B.electrons and position
a p-n junction, the depletion layer width is C.electrons only D.electrons and holes.
A.halved B.constant C.decreased D.increased 31. A transistor is operated by A.current
The depletion layer widens or increases its width B.voltage C.voltage and current D.resistance
in reverse biased, inroder to decrease the flow of Transistors are controlled by small current to
majority carriers and permit the flow of minority regulate large current.
carriers, hence decrease in current flow. 32.Transistors are used for the A.conversion of
23. The p-n junction diode is primarily used as a a.c to d.c B.amplification of signals
A.transistor B.rectifier C.an amplifier D.a shunt C.rectification of signals D.conversion of d.c to a.c.
p-n junction diode is majorly used as a rectifier Transistors are used as amplifiers.
and minorly as a switch. 33. The minority carriers in a n-p-n transistor are
24. A p-n junction diode is used as A.a rectifier in the A.holes B.electrons C.ioms D.electrons and
a d.c circuit B.an amplifier in an a.c circuit C.a holes
rectifier in an a.c circuit D.an amplifier in an d.c In a n-p-n transistor, the majority carriers are
circuit free electrons while the minority carriers are
A p-n junction is used as a rectifier in a.c circuit holes. In a p-n=p transistor, the majority carriers
for the rectification of a.c to d.c. are holes while the minority carriers are free
25. A semi-conductor diode is used in rectifying electrons.
alternating current into direct current mainly 34. I.For current amplification II.For power
because it A.allows current to flow in either amplification III.For voltage stabilization IV.As a
direction B.is non-linear C.offers a high input switch. Which of the above are the uses of
resistance D.allows current to flow only in one transistor? A.I,II,III and IV B.I,III and IV C.I,II and
direction III D.I,II and IV.
Semi-conductor diode i.e p-n junction diode is Zener diodes are used as voltage regulators or
used in rectifying alternate current because it stabilizers.
allows current to flow in one direction as it offers 35. A transistor is used in the amplification of
low resistance and cuts off current in the other signals because it A.allows doping B.contains
direction as it offers high resistance. electrons and hole carriers C.consumes a lot of
26. The p-n junction can act as rectfiers because power D.controls flow of current
they A.conduct current when forward biased Transistors are controlled by small current to
B.conduct current when reverse biased C.block regulate large current i.e a sma;; input current
current when forward biased D.conduct current produces a large output current.
in both directions 36. 𝑅 In the circuit diagram of a
The p-n junction diode can act as rectifiers 𝑄 transistor above, the terminals
because they conduct current when forward 𝑃 𝑃, 𝑄 and 𝑅 represents
biased and block current when reverse biased. A.COLLECTOR,EMITTER and BASE B.EMITTER,

303
Demystified Series Physics Demystified by Dr Timothy
BASE and COLLECTOR C.BASE, EMITTER and 42. When a base current of the common emitter
COLLECTOR D.BASE, COLLECTOR and EMITTER. transistor is kept at zero, it operates in the
37. Which of the following diagrams correctly A.active-on region B.breakdown region C.cut-off
represents a p-n-p transistor? region D.saturation region
A. 𝒃 B. 𝑏 In a common emitter transistor,with switch
opened such that the no current is flowing
𝒆 𝒄 𝑒 𝑐 through the transistor i.e. input current(base
current) is zero, then the output current (colletor
C. 𝑏 D. 𝑏 current) is zero and the out put voltage is
maximum i.e. cut off region. While with switch
𝑒 𝑐 𝑒 𝑐 closed such that current is flowing through the
transistor i.e. input current(base current) is
In p-n-p transistor, current flows into the circuit maximum, then the output current (collector
i.e In-P; Inward current p-n-p as shown in option current) is maximum and the output voltage is
A. Option B represents n-p-n transistor, current zero i.e. saturation or maximum region.
flows out of the circuit i.e Out-N; Outward 43. A typical transistor characteristic is
current n-p-n. Current flows in and out of a represented as
transistor through the emitter only. A. 𝐼 B. 𝑰
38. In common-emitter transistor amplifier, the
input signal is applied between the A.emitter and
ground B.base and emitter C.base and collector
D.emitter and collector E.collector and ground. 𝑉 𝑽
In common-emitter connection, the input signal C. 𝐼 D. 𝐼
or voltage is through the emitter-base junction or
through the base while the output signal or
voltage is through the emitter-collector junction
or through the collector. 𝑉 𝑉.
𝐶
Jamb past questions on Basic electronics :
𝐵 Output [1990/49,2001/47,2005/8,2006/6,2007/14,
Input 2009/49,50,2010/49,2011/48,49,50,2015/ 19]

𝐸 (Common emitter)
39. In common emitter configuration, the output
voltage is through the A.base B.emitter
C.collector D.resistor.
40. In the common collector amplifier circuit, the
input signal is applied between the A.base and
the collector B.emitter and the junction
C.collector and the common base D.collector and
the emitter junction E.the base and the emitter
In a common collector connection, the input
signal or voltage is through the collector-base
junction or through the base while the output
signal or voltage is through the emitter-collector
junction or through the emitter.
𝐸

𝐵 Output
Input

𝐶 (Common collector)
41. In which of the following configurations can
a transistor be used as a voltage amplifier? I.
common-collector configuration II. common-
base configuration III. common-emitter
configuration A.I B.II C.III D.I and II E.II and III
Common-collector does not amplify voltage.

304

You might also like